You are on page 1of 444

QUANTITATIVE

-Training & Placement Cell


APTITUDE VOLUME-I

for Campus
Interview

VOLUME-I

Dinesh Khattar
Cover image: Shutterstock.com www.pearson.co.in
The Pearson Guide to

Quantitative Aptitude
for
Competitive Examinations

Volume 1

A01_VOL 1_9997_SE_FM.indd 1 2/4/2016 11:39:01 AM


Copyright © 2016 Pearson India Education Services Pvt. Ltd

Published by Pearson India Education Services Pvt. Ltd, CIN: U72200TN2005PTC057128, formerly
known as TutorVista Global Pvt. Ltd, licensee of Pearson Education in South Asia.

No part of this eBook may be used or reproduced in any manner whatsoever without the publisher’s prior
written consent.

This eBook may or may not include all assets that were part of the print version. The publisher reserves the
right to remove any material in this eBook at any time.

ISBN: 9789332569997
eISBN 978-93-325-7865-4

Head Office: A-8 (A), 7th Floor, Knowledge Boulevard, Sector 62, Noida 201 309, Uttar Pradesh, India.
Registered Office: 4th Floor, Software Block, Elnet Software City, TS-140, Block 2 & 9,
Rajiv Gandhi Salai, Taramani, Chennai 600 113, Tamil Nadu, India.
Fax: 080-30461003, Phone: 080-30461060
www.pearson.co.in, Email: companysecretary.india@pearson.com

A01_VOL 1_9997_SE_FM.indd 2 2/5/2016 4:59:47 PM


Table of Contents

Chapter 1 Numbers 1
Chapter 2 H.C.F. and L.C.M. of Number 31
Chapter 3 Square Root and Cube Root 43
Chapter 4 Simplification 60
Chapter 5 Percentage 92
Chapter 6 Average 124
Chapter 7 Ratio and Proportion 148
Chapter 8 Partnership 181
Chapter 9 Profit and Loss 192
Chapter 10 Time, Work and Wages 224
Chapter 11 Pipes and Cisterns 257
Chapter 12 Time and Distance 273
Chapter 13 Boats and Streams 322
Chapter 14 Races and Games of Skill 334
Chapter 15 Alligation or Mixture 342
Chapter 16 Problems on Ages 364
Chapter 17 Simple Interest 383
Chapter 18 Compound Interest 404
Chapter 19 Logarithms 424

A01_VOL 1_9997_SE_FM.indd 3 2/4/2016 11:39:01 AM


This page is intentionally left blank

JEE_MAIN_EXAM_Chemistry-FM.indd 4 5/26/2014 2:16:04 PM


1 Numbers

Introduction The set of all natural numbers can be represented by


In Hindu Arabic System, we use ten symbols 0, 1, 2, 3, 4, 5, N = {1, 2, 3, 4, 5, ...}.
6, 7, 8, 9 called digits to represent any number. This is the Whole Numbers
decimal system where we use the numbers 0 to 9. 0 is called If we include 0 among the natural numbers, then the
insignificant digit whereas 1, 2, 3, 4, 5, 6, 7, 8, 9 are called numbers 0, 1, 2, 3, 4, 5, ... are called whole numbers.
significant digits. The set of whole numbers can be represented by
A group of figures denoting a number is called a W = {0, 1, 2, 3, 4, 5, ...}
numeral. For a given numeral, we start from extreme right
Clearly, every natural number is a whole number but 0
as unit’s place, ten’s place, hundred’s place and so on.
is a whole number which is not a natural number.
Illustration 1  We represent the number 309872546 as
shown below: Integers
All counting numbers and their negatives including zero are
known as integers.
The set of integers can be represented by
Z or I = {... – 4, –3, –2, –1, 0, 1, 2, 3, 4, ...}
Positive Integers
The set I + = {1, 2, 3, 4, ....} is the set of all positive
integers. Clearly, positive integers and natural numbers
We read it as are synonyms.
“Thirty crores, ninety-eight lakhs, seventy-two Negative Integers
thousands five hundred and forty-six.” The set I – = {–1, –2, –3, ...} is the set of all negative
In this numeral: integers. 0 is neither positive nor negative.
The place value of 6 is 6 × 1 = 6.
The place value of 4 is 4 × 10 = 40. Non-negative Integers
The place value of 5 is 5 × 100 = 500. The set {0, 1, 2, 3, ... } is the set of all non-negative integers.
The place value of 2 is 2 × 1000 = 2000 and so on. Rational Numbers
p
The face value of a digit in a number is the value itself The numbers of the form , where p and q are integers
wherever it may be. q
Thus, the face value of 7 in the above numeral is 7. The 4 3 5 0
and q ≠ 0, are known as rational numbers, e.g. , ,  , ,
face value of 6 in the above numeral is 6 and so on. 7 2 8 1
2
Number System − , etc.
3
Natural Numbers The set of all rational numbers is denoted by Q.
Counting numbers 1, 2, 3, 4, 5, ... are known as natural
numbers.
{ p
i.e. Q = x:x = ; p, q ≠ I, q ≠ 0 .
q }

Chapter_01.indd 1 1/30/2016 1:18:37 PM


2  Chapter 1

a The set of all real numbers is denoted by R.


Since every natural number ‘a’ can be written as s, Note that the sum, difference or product of a rational
1
0 and irrational number is irrational, e.g. 3 + 2, 4 – 3,
so it is a rational number. Since 0 can be written as and
1 2
a – 5 , 4 3 , – 7 5 are all irrational.
every non-zero integer ‘a’ can be written as , so it is also 3
1
a rational number. Even Numbers
All those numbers which are exactly divisible by 2 are
Every rational number has a peculiar characteristic
called even numbers, e.g. 2, 6, 8, 10, etc., are even numbers.
that when expressed in decimal form is expressible either
in terminating decimals or non-terminating repeating Odd Numbers
decimals. All those numbers which are not exactly divisible by 2 are
called odd numbers, e.g. 1, 3, 5, 7, etc., are odd numbers.
1 1 22
For example, = 0.2, = 0.333 ..., = 3.1428714287, Prime Numbers
5 3 7
8 A natural number other than 1 is a prime number if it is
= 0.181818 ..., etc. divisible by 1 and itself only.
44
For example, each of the numbers 2, 3, 5, 7, etc., are
The recurring decimals have been given a short
prime numbers.
notation as
Composite Numbers
0.333... = 0.3
Natural numbers greater than 1 which are not prime are
4.1555...= 4.05 known as composite numbers.
0.323232... = 0.32. For example, each of the numbers 4, 6, 8, 9, 12, etc., are
composite numbers.
Irrational Numbers
Those numbers which when expressed in decimal form are Notes:
neither terminating nor repeating decimals are known as 1. The number 1 is neither a prime number nor a composite
irrational numbers, e.g. 2, 3, 5, p, etc. number.
2. 2 is the only even number which is prime.
22 22 3. Prime numbers up to 100 are:
Note that the exact value of p is not . is rational
7 7 2, 3, 5, 7, 11, 13, 17, 19, 23, 29, 31, 37, 41, 43, 47, 53,
22
while p is irrational number. is approximate value of p. 59, 61, 67, 71, 73, 79, 83, 89, 97, i.e. 25 prime numbers
7 between 1 and 100.
Similarly, 3.14 is not an exact value of it.
4. Two numbers which have only 1 as the common factor
Real Numbers are called co-primes or relatively prime to each other,
e.g. 3 and 5 are co-primes.
The rational and irrational numbers combined together are
Note that the numbers which are relatively prime
13 2 3
called real numbers, e.g. , ,  , 3, 4  2, etc. are need not necessarily be prime numbers, e.g. 16 and 17 are
21 5 7 relatively prime although 16 is not a prime number.
real numbers.

addition and subtraction (short-cut methods)

The method is best illustrated with the help of following To obtain 1 at unit’s place add 9 to
examples: make 31. In the answer, write 9 at
unit’s place and carry over 3.
Illustration 2  54321 – (9876 + 8967 + 7689) = ?
Step 1 Add Ist column: Step 2 Add 2nd column:
6 + 7 + 9 = 22 3 + 7 + 6 + 8 = 24

Chapter_01.indd 2 1/30/2016 1:18:38 PM


Numbers 3

To obtain 2 at ten’s place, add 8 to make 32. In the To obtain 4 at thousand’s place add 7 to make 34.
answer, write 8 at ten’s place and carry over 3. In the answer, write 7 at thousand’s place and carry
Step 3 Add 3rd column: over 3.
3 + 8 + 9 + 6 = 26 Step 5 5th column:
To obtain 3 at hundred’s place, add 7 to make 33. To obtain 5 at ten-thousand’s place add 2
In the answer, write 7 at hundred’s place and carry to it to make 5. In the answer, write 2 at the
over 3. ten-thousand’s place.
Step 4 Add 4th column:
∴ 54321 – (9876 + 8967 + 7689) = 27789.
3 + 9 + 8 + 7 = 27

MULTIPLICATION (SHORT-CUT METHODS)

Illustration 5 Multiply
1. Multiplication of a given number by 9, 99, 999,
(a) 7054 × 15 (b) 3897 × 25
etc., that is by 10n – 1
(c) 4563 × 35
Method: Put as many zeros to the right of the
Solution:
multiplicant as there are nines in the multiplier and from 1
the result subtract the multiplicant and get the answer. (a) 7054 × 15 = (7054 × 30)
2
Illustration 3 Multiply 1
= (211620) = 105810.
(a) 3893 by 99 (b) 4327 by 999 2
(c) 5863 by 9999 1 1
(b) 3897 × 25 = (3897 × 50) = (194850)
Solution: 2 2
(a) 3893 × 99 = 389300 – 3893 = 385407. = 97425.
1 1
(b) 4327 × 999 = 4327000 – 4327 = 4322673 (c) 4536 × 35 = (4563 × 70) = (319410)
2 2
(c) 5863 × 9999 = 58630000 – 5863 = 58624137.
= 159705.
2. Multiplication of a given number by 11, 101,
1001, etc., that is, by 10n + 1. 4. Multiplication of a given number by 5, 25, 125,
625, etc., that is, by a number which is some
Method: Place n zeros to the right of the multiplicant
power of 5.
and then add the multiplicant to the number so
obtained. Method: Place as many zeros to the right of the
multiplicant as is the power of 5 in the multiplier,
then divide the number so obtained by 2 raised to the
Illustration 4 Multiply same power as is the power of 5.
(a) 4782 × 11 (b) 9836 × 101
(c) 6538 × 1001
Illustration 6 Multiply
Solution: (a) 3982 × 5 (b) 4739 × 25
(a) 4782 × 11 = 47820 + 4782 = 52602
(c) 7894 × 125 (d) 4863 × 625
(b) 9836 × 101 = 983600 + 9836 = 993436 Solution:
((c)) 6538 × 1001 = 6538000 + 6538 = 6544538 39820
(a) 3982 × 2 = = 19910
2
3. Multiplication of a given number by 15, 25, 35,
473900 473900
etc. (b) 4739 × 25 = 2
= = 118475
2 4
Method: Double the multiplier and then multiply the
7894000 7894000
multiplicant by this new number and finally divide (c) 7894 × 125 = 3
=
the product by 2. 2 8
= 986750

Chapter_01.indd 3 1/30/2016 2:41:06 PM


4 Chapter 1

48630000 48630000 Illustration 7 438 × 637 + 438 × 367 = ?


(d) 4863 × 625 = =
2 4
16 Solution: 438 × 637 + 438 × 367 = 438 × (637 + 367)
= 3039375. = 430 × 1000 = 438000.
distributive Laws Illustration 8 674 × 832 – 674 × 632 = ?
For any three numbers a, b, c, we have
Solution: 674 × 832 – 674 × 632 = 674 × (832 – 632)
(a) a × b + a × c = a × (b + c)
(b) a × b – a × c = a × (b – c) = 674 × 200 = 134800.

sQuares (short-cut methods)

1. To square any number ending with 5.


Method: (A5)2 = A(A + 1)/25
Illustration 9
(a) (25)2 = 2 (2 + 1)/25 = 6/25 = 625
(b) (45)2 = 4 (4 + 1)/25 = 20/25 = 2025
((c)) (85)2 = 8 (8 + 1)/25 = 72/25 = 7225

2. To square a number in which every digit is one.


Method: Count the number of digits in the given
number and start writing numbers in ascending order
from one to this number and then in descending Dividend = (Divisor × Quotient ) + Remainder
order up to one. Dividend  Remainder
or, Divisor =
Quotient
Illustration 10
(a) 112 = 121 (b) 1112 = 12321
2
(c) 1111 = 1234321 Illustration 12 On dividing 7865321 by a certain number,
(d) 2222 = 22 (111)2 = 4 (12321) = 49284 the quotient is 33612 and the remainder is 113. Find the
(e) 33332 = 32 (1111)2 = 9 (1234321) = 11108889 divisor.
3. To square a number which is nearer to 10x. Dividend  Remainder
Solution: Divisor =
Method: Use the formula: Quotient
x2 = (x2 – y2) + y2 = (x + y)(x – y) + y2 7865321  113 7865208
= = = 234.
Illustration 11 33612 33612
(a) (97)2 = (97 + 3) (97 – 3) + 32
Illustration 13 A number when divided by 315 leaves
= 9400 + 9 = 9409
remainder 46 and the value of quotient is 7. Find the
(b) (102)2 = (102 – 2) (102 + 2) + 22
number.
= 10400 + 4 = 10404
(c) (994)2 = (994 + 6) (994 – 6) + 62 Solution: Number = (Divisor × Quotient) + Remainder
= 988000 + 36 = 988036 = (315 × 7) + 46 = 2205 + 46
(d) (1005)2 = (1005 – 5) (1005 + 5) + 52 = 2251.
= 1010000 + 25 = 1010025.
dIVISIon Illustration 14 Find the least number of 5 digits which is
Division is repeated subtraction. exactly divisible by 632.
For example, when we divide 63289 by 43, it means 43 Solution: The least number of 5 digits is 10000. Dividing
can be repeatedly subtracted 1471 times from 63289 and the this number by 632, the remainder is 520. So, the required
remainder 36 is left. number = 10000 + (632 + 520) = 10112.

Chapter_01.indd 4 1/30/2016 1:18:39 PM


Numbers 5

4. Divisibility by 5: A number is divisible by 5 if


the unit’s digit in the number is 0 or 5.
For example, 13520, 7805, 640, 745, etc., are all
divisible by 5.
5. Divisibility by 6: A number is divisible by 6 if the
number is even and sum of its digits is divisible by 3.
Illustration 15 Find the greatest number of 5 digits which For example, the number 4518 is divisible by 6
is exactly divisible by 463. since it is even and sum of its digits 4 + 5 + 1 + 8 =
Solution: The greatest number of 5 digits is 99999. Dividing 18 is divisible by 3.
this number by 463, the remainder is 454. So, the required 6. Divisibility by 7: The unit digit of the given
number = 99999 – 454 = 99545. number is doubled and then it is subtracted from
the number obtained after omitting the unit digit.
If the remainder is divisible by 7, then the given
number is also divisible by 7.
For example, consider the number 448. On
doubling the unit digit 8 of 448 we get 16.
Then, 44 – 16 = 28.
Since 28 is divisible by 7, 448 is divisible by 7.
7. Divisibility by 8: A number is divisible by 8, if
Illustration 16 Find the number nearest to 13700 which is the number formed by the last 3 digits is divisible
exactly divisible by 235. by 8.
Solution: On dividing the number 13700 by 235, the For example, the number 41784 is divisible by 8 as
remainder is 70. Therefore, the nearest number to 13700, the number formed by last three digits, i.e. 784 is
which is exactly divisible by 235 = 13700 – 70 = 13630. divisible by 8.
8. Divisibility by 9: A number is divisible by 9 if the
sum of its digits is divisible by 9.
For example, the number 19044 is divisible by 9
as the sum of its digits 1 + 9 + 0 + 4 + 4 = 18 is
divisible by 9.
9. Divisibility by 10: A number is divisible by 10, if
it ends in zero.
teStS oF dIVISIbILIty For example, the last digit of 580 is zero, therefore,
580 is divisible by 10.
1. Divisibility by 2: A number is divisible by 2 if the
unit’s digit is zero or divisible by 2. 10. Divisibility by 11: A number is divisible by 11, if
the difference of the sum of the digits at odd places
For example, 4, 12, 30, 18, 102, etc., are all
and sum of the digits at even places is either zero or
divisible by 2.
divisible by 11.
2. Divisibility by 3: A number is divisible by 3 if the For example, in the number 38797, the sum of the
sum of digits in the number is divisible by 3. digits at odd places is 3 + 7 + 7 = 17 and the sum of
For example, the number 3792 is divisible by 3 the digits at even places is 8 + 9 = 17. The difference
since 3 + 7 + 9 + 2 = 21, which is divisible by 3. is 17 – 17 = 0, so the number is divisible by 11.
11. Divisibility by 12: A number is divisible by 12 if
3. Divisibility by 4: A number is divisible by 4 if the
number formed by the last two digits (ten’s digit it is divisible by 3 and 4.
and unit’s digit) is divisible by 4 or are both zero. 12. Divisibility by 25: A number is divisible by 25 if
For example, the number 2616 is divisible by 4 the number formed by the last two digits is divisible
since 16 is divisible by 4. by 25 or the last two digits are zero.

Chapter_01.indd 5 1/30/2016 1:18:40 PM


6 Chapter 1

For example, the number 13675 is divisible by For example, the number 5250 is divisible by
25 as the number formed by the last two digits is 125 as 250 is divisible by 125.
75 which is divisible by 25. 14. Divisibility by 18: An even number satisfying
13. Divisibility by 125: A number is divisible by 125 the divisibility test of 9 is divisible by 18.
if the number formed by the last three digits is 15. Divisibility by 88: A number is divisible by 88 if
divisible by 125 or the last three digits are zero. it is divisible by 11 and 8.

some useFul short-cut methods

Illustration 20 A number on being divided by 10 and 11


1. Test to find whether a given number is a prime successively leaves the remainders 5 and 7, respectively.
Step 1 Select a least positive integer n such that n2 Find the remainder when the same number is divided by
> given number. 110.
Step 2 Test the divisibility of given number by Solution: The required remainder
every prime number less than n. = d1 × r2 + r1 = 10 × 7 + 5 = 75.
Step 3 The given number is prime only if it is not
divisible by any of these primes. 4. To find the number of numbers divisible by a
certain integer.
Illustration 17 Investigate whether 571 is a prime number. The method is best illustrated with the help of
Solution: Since (23)2 = 529 < 571 and (24)2 = 576 > 571 following example.
\ n = 24.
Prime numbers less than 24 are 2, 3, 5, 7, 11, 13, 17, 19, Illustration 21 How many numbers up to 532 are divisible
23. Since 24 is divisible by 2, 571 is not a prime number. by 15?
Solution: We divide 532 by 15.
Illustration 18 Investigate whether 923 is a prime number.
532 = 35 × 15 + 7
Solution: Since (30)2 = 900 < 923 and (31)2 = 961 > 923
\ n = 31. The quotient obtained is the required number of
Prime numbers less than 31 are 2, 3, 5, 7, 11, 13, 17, numbers. Thus, there are 35 such numbers.
19, 23, 29. Since 923 is not divisible by any of these primes,
Illustration 22 How many numbers up to 300 are divisible
therefore 923 is a prime number.
by 5 and 7 together?
2. The least number which when divided by d1, Solution: L.C.M. of 5 and 7 = 35
d2 and d3 leaves the remainders r1, r2 and r3, We divide 300 by 35
respectively, such that (d1 – r1) = (d2 – r2) = (d3
300 = 8 × 35 + 20
– r3) is (L.C.M. of d1, d2 and d3) – (d1 – r1) or (d2
– r2) or (d3 – r3). Thus, there are 8 such numbers.

Illustration 19 Find the least number which when 5. Two numbers when divided by a certain divisor
divided by 9, 10 and 15 leaves the remainders 4, 5 and 10, give remainders r1 and r2. When their sum is
respectively. divided by the same divisor, the remainder is r3.
Solution: Here 9 – 4 = 10 – 5 = 15 – 10 = 5 The divisor is given by r1 + r2 – r3.
Also, L.C.M. (9, 10, 15) = 90
\ the required least number = 90 – 5 = 85. Illustration 23 Two numbers when divided by a certain
divisor give remainders 473 and 298, respectively. When
3. A number on being divided by d1 and d2 their sum is divided by the same divisor, the remainder is
successively leaves the remainders r1 and r2, 236. Find the divisor.
respectively. If the number is divided by d1 × d2,
then the remainder is (d1 × r2 + r1). Solution: The required divisor
= 437 + 298 – 236 = 499.

Chapter_01.indd 6 1/30/2016 1:18:41 PM


Numbers  7

Multiple Choice QuestionS

1.
Let a, b, c, d be the four integers such that a + b + c + d = 8. Which of the following integers is the square of an integer
4m + 1, where m is a positive integer. Given m, which one for every integer n?
of the following is necessarily true? (a) n2 + l (b) n2 + n
(a) The minimum possible value of (c) n2 + 2n (d) n2 + 2n + 1
a2 + b2 + c2 + d 2 is 4m2 – 2m + 1 [Based on MAT, 2004]
(b) The minimum possible value of 9. Given that N = (521)125 × (125)521, find the last two digits
a2 + b2 + c2 + d 2 is 4m2 + 2m + 1 of N.
(c) The maximum possible value of (a) 75 (b) 25
a2 + b2 + c2 + d 2 is 4m2 – 2m + 1 (c) 45 (d) None of these
(d) The maximum possible value of 10. The sum of the digits of a 3-digit number is subtracted
a2 + b2 + c2 + d 2 is 4m2+ 2m + 1. from the number. The resulting number is always
[Based on CAT, 2003] (a) Divisible by 6 (b) Not divisible by 6
2.
How many three-digit positive integers with digits x, y (c) Divisible by 9 (d) Not divisible by 9
and z in the hundred’s, ten’s and unit’s place, respectively,
[Based on MAT, 2004]
exist such that x < y, z < y and x ≠ 0?
11. The least number that must be subtracted from each of the
(a) 245 (b) 285
numbers 14, 17, 34 and 42 so that the remainders may be
(c) 240 (d) 320 proportional is
[Based on CAT, 2003]
(a) 0 (b) 1
3.
The number of positive integers n in the range 12 ≤ n
(c) 2 (d) 7
≤ 40 such that the product (n – 1) (n – 2) ... 3 × 2 × 1 is
not divisible by n is [Based on MAT, 2003]

(a) 5 (b) 7 12. The highest power of 5 that is contained in 125125 – 2525
(c) 13 (d) 14 is
4.
Let x and y be positive integers such that x is prime and y (a) 25 (b) 50
is composite. Then,
(c) 75 (d) 125
(a) y – x cannot be an even integer.
(b) xy cannot be an even integer. 13. The remainder when 5163 is divided by 1000 is
( x  y) (a) 125 (b) 625
(c) cannot be an even integer.
x (c) 25 (d) None of these
(d) None of these. [Based on CAT, 2004]
14. Of the 120 people in the room, three-fifths are women. If
5.
If a, a + 2 and a + 4 are prime numbers, then the number two-thirds of the people are married, then what is the
of possible solutions for a is maximum number of women in the room who could be
(a) one (b) two unmarried?
(c) three (d) None of these (a) 40 (b) 20
[Based on CAT, 2004] (c) 30 (d) 60
What is the remainder when 496 is divided by 6?
6. [Based on MAT, 2003]
(a) 0 (b) 2 2/3 1/3
15. If x = 2 + 2 +2 , then the value of x3 – 6x2 + 6x is
(c) 3 (d) 4
(a) 3 (b) 2
[Based on CAT, 2004]
(c) 1 (d) None of these
7. A student was asked to divide a number by 6 and add 12
to the quotient. He, however, first added 12 to the number [Based on MAT, 2002]
and then divided it by 6, getting 112 as the answer. The 16.
A number of three digits in scale 7 when expressed in
correct answer should have been scale 9 has its digits reversed in order. The number is
(a) 122 (b) 118 (a) 248 (b) 348
(c) 114 (d) 124 (c) 148 (d) 448
[Based on MAT, 2004] [Based on MAT, 2002]

Chapter_01.indd 7 1/30/2016 1:18:41 PM


8  Chapter 1

17.
For every positive real number 25. In three coloured boxes: red, green and blue, 108 balls
are placed. There are twice as many in the green and red
 x   x + 1
 2  +  2  = ... boxes combined as they are in the blue box and twice
as many in the blue box as they are in the red box. How
where ([]) is the greatest integer function. many balls are there in the green box?
(a) 0 (b) 1 (a) 18 (b) 36
(c) [x + 1] (d) [x] (c) 45 (d) None of these
[Based on MAT, 2002] [Based on FMS Delhi, 2004]

18. How many five-digit multiples of 11 are there, if the five 26. If a = 12, b = 23, c = 34 ... z = 2627. In the product of all
digits are 3, 4, 5, 6 and 7 in the same order? the alphabets, how many zeros exist in the end?
(a) 12 (b) 13 (a) 100 (b) 104
(c) 10 (d) None of these (c) 80 (d) 106
[Based on MAT, 2002] [Based on FMS Delhi, 2004]

19. The smallest number by which 3600 can be divided to 27. The unit’s digit of a two-digit number is one more than the
make it a perfect cube is digit at ten’s place. If the number is more than five times
of the sum of the digits of the number, then find the sum
(a) 9 (b) 50 of all such possible numbers.
(c) 300 (d) 450 (a) 246 (b) 275
[Based on MAT, 2002] (c) 290 (d) 301
20. The least number having four digits which is a perfect [Based on FMS Delhi, 2004]
square is
28. Let 20 × 21 × 22 × ... × 30 = A. If A is divisible by 10x,
(a) 1004 (b) 1016 then find the maximum value of x.
(c) 1036 (d) None of these (a) 3 (b) 4
[Based on MAT, 2002] (c) 5 (d) 6
The remainder when 784 is divided by 342 is
21. [Based on FMS Delhi, 2004]
(a) 0 (b) 1 29. A student was asked to find the sum of all the prime
(c) 49 (d) 341 numbers between 10 to 40. He found the sum as 180.
Which of the following statements is true?
[Based on MAT, 2001]
(a) He missed one prime number between 10 and 20.
22.
If the sum of n consecutive integers is 0, which of the
following must be true? (b) He missed one prime number between 20 and 30.
I. n is an even number. (c) He added one extra prime number between 10 and 20.
II. n is an odd number. (d) None of these.
III. The average of the n integers is 0.  [Based on FMS Delhi, 2004]

(a) I only (b) II only 30. 1 is not defined but it is denoted by i. Clearly, i is not
(c) III only (d) II and III a real number, so it is called imaginary number. Now
100
23.
A two-digit number is such that the product of the digits find ∑ (i) n

is 14. When 45 is added to the number, then the digits n =1

interchange their places. Find the number. (a) i (b) 1


(a) 72 (b) 27 (c) –1 (d) 0
(c) 37 (d) 14 [Based on FMS Delhi, 2004]
[Based on MAT, 2001] 31. (a + b + c + d + e)/(v + w + x + y + z) = N, where a, b, c,
24. A player holds 13 cards of four suits of which seven are d, e are five consecutive even integers and v, w, x, y, z are
black and six are red. There are twice as many diamonds five consecutive odd integers. If v = a + 1 and n represent
as spades and twice as many hearts as diamonds. How natural numbers, then which of the following is the most
many clubs does he hold ? suitable value of N?
(a) 4 (b) 5 (a) (n + 4)/(n + 5) (b) (n + 3)/(n + 4)
(c) 6 (d) 7 (c) (n + 2)/(n + 3) (d) (n + 2)/(n + 2.5)
[Based on FMS Delhi, 2004] [Based on FMS Delhi, 2004]

Chapter_01.indd 8 1/30/2016 1:18:41 PM


Numbers  9

32. Manu and Tanu are playing mathematical puzzles. Manu 40. Consider a 99-digit number created by writing side by
asks Tanu: “which whole numbers, greater than one, side the first fifty four natural numbers as follows:
divide evenly all the nine numbers, i.e., 111, 222, 333, 1 2 3 4 5 6 7 8 9 10 11 12 13 _ _ _ _ 53 54
444, 555, 666, 777, 888, 999?” Tanu immediately gave
the above number when divided by 8 will leave a
the desired answer. It was
remainder
(a) 7, 37, 111 (b) 3, 37, 111
(a) 6 (b) 4
(c) 9, 37, 111 (d) 9, 13, 111
(c) 2 (d) 0
33. The smallest prime number that is the fifth term of
an increasing arithmetic sequence for which all four 41. The denominator of a rational number is 3 more than
preceding terms are also prime its numerator. If the numerator is increased by 7 and the
denominator is decreased by 2, we obtain 2. The rational
(a) 17 (b) 37
number is
(c) 29 (d) 53
1 5
34. When 1012 – 1 is divided by 111, the quotient is
(a) (b)
4 8
(a) 9009009 (b) 9000009
7 8
(c) 9009009009 (d) 9000000009 (c) (d)
10 11
35. A number N is defined as the addition of 4 different
[Based on FMS Delhi, 2003]
integers. Each of the four numbers gives a remainder
zero when divided by four. The first of the four numbers 42. A teacher gave the simple multiplication exercise to the
defined as A is known to be as 461. The other three numbers kids. But one kid reversed the digits of both the numbers
arranged in the increasing order and defined as B, C and D and carried out the multiplication and found that the
are each 4 times more than the previous number. Thus, the product was exactly the same as the one expected by the
number B = 4 × A, similarly C = 4 × B and also D = 4 × C. teacher. Only one of the following pairs of numbers will
Thus the number N so formed is perfectly divisible by fit in the description of the exercise. Which one is that?
(a) 11 (b) 10 (a) 14, 42 (b) 42, 28
(c) 3 (d) 13 (c) 19, 63 (d) 13, 62
36. Which of the following is a prime number? 15 15
43. Find the remainder when (1117  1311 ) is divided by 7.
(a) 889 (b) 997
(c) 899 (d) 1147 (a) 0 (b) 1
[Based on FMS Delhi, 2004] (c) 2 (d) 3
37. A cube is cut into n identical pieces. If it can be done so 44. In a division sum, the divisor is 12 times the quotient and
in only one way, then which of the following could be the 5 times the remainder. If the remainder is 48, then what is
value of n? the dividend?
(a) 179 (b) 203 (a) 240 (b) 576
(c) 143 (d) 267 (c) 4800 (d) 4848
[Based on IIT Joint Man. Ent. Test, 2004] [Based on IIFT, 2003]
38. A gardener has to plant trees in rows containing equal 45. Which of the following integers has the most divisors?
number of trees. If he plants in rows of 6, 8, 10 or 12, then
(a) 88 (b) 91
five trees are left unplanted. But if he plants in rows of 13
trees each, then no tree is left. What is the number of trees (c) 99 (d) 101
that the gardener plants? [Based on SCMHRD Ent. Exam., 2003]
(a) 485 (b) 725 46. What is the smallest value of n for which (n13 – n) (52n – 1)
(c) 845 (d) None of these is divisible by 169?
[Based on IIT Joint Man. Ent. Test, 2004]
(a) 5 (b) 4
39. I think of a number. I double the number, add 6 and
multiply the result by 10. I now divide by 20 and subtract (c) 1 (d) 2
the number I first thought of. The result is: 47. If a number is divided by 2 the remainder is 1. If it is
(a) Depends upon the number thought divided by 3 the remainder is 2. What is the remainder
(b) 1 when the number is divided by 6?
(c) 2 (a) 0 (b) 1
(d) 3 (c) 4 (d) 5

Chapter_01.indd 9 1/30/2016 1:18:41 PM


10  Chapter 1

48. In which of the following pairs of numbers, it is true that missed one number in the sequence during addition. The
their sum is 11 times their product? number he missed was
(a) 1, 1/11 (b) 1, 1/10 (a) less than 10 (b) 10
(c) 1, 1/12 (d) 1, 10 (c) 15 (d) more than 15
 [Based on SCMHRD, 2002] [Based on CAT, 2002]

49. If m, n, o, p and q are integers, then m (n + o) (p – q) must 55. When 2256 is divided by 17, the remainder would be
be even when which of the following is even? (a) 1 (b) 16
(a) m + n (b) n + p (c) 14 (d) None of these
(c) m (d) p [Based on CAT, 2002]
[Based on REC Tiruchirapalli, 2002] 56. After the division of a number successively by 3,4 and 7,
the remainders obtained are 2, 1 and 4, respectively. What
50. If there are 10 positive real numbers n1 < n2 < n3
will be the remainder if 84 divides the same number?
... < n10 ... . How many triplets of these numbers
(n1, n2, n3), (n2, n3, n4) ... can be generated such that in (a) 80 (b) 75
each triplet the first number is always less than the second (c) 41 (d) 53
number, and the second number is always less than the [Based on CAT, 2002]
third number? 6n 6n
57. 7 – 6 , where n is an integer > 0, is divisible by
(a) 45 (b) 90 (a) 13 (b) 127
(c) 120 (d) 180 (c) 559 (d) None of these.
[Based on CAT, 2002] [Based on CAT, 2002]
51. Number S is obtained by squaring the sum of digits of a 58. If x2 < 51 and y2 < 21 and x and y are integers, then which
two-digit number D. If difference between S and D is 27, of the following is the least number which when divided
then the two digit number D is by the least value of x and least value of y gives a negative
(a) 24 (b) 54 quotient?
(c) 34 (d) 45 (a) 28 (b) 56
[Based on CAT, 2002] (c) –28 (d) –56
52. The owner of a local jewellery store hired 3 watchmen to 59. What is the product of remainders when 64 is divided by
guard his diamonds, but a thief still got in and stole some 24 and 75 is divided by 142?
diamonds. On the way out, the thief met each watchman, (a) 7 (b) 5
one at a time. To each he gave half of the diamonds he had
(c) 0 (d) 4
then, and 2 more besides. He escaped with one diamond.
How many did he steal originally? 60. Of 128 boxes of oranges, each box contains at least 120
and at most 144 oranges. The number of boxes containing
(a) 40 (b) 36
the same number of oranges is at least:
(c) 25 (d) None of these
(a) 5 (b) 103
[Based on CAT, 2002]
(c) 6 (d) None of these
53.
A rich merchant had collected many gold coins. He did
[Based on CAT, 2001]
not want anybody to know about him. One day, his wife
asked, “how many gold coins do we have?” After pausing 61.
In a 4-digit number, the sum of the first two digits is equal
a moment, he replied, “well! if I divide the coins into two to that of the last two digits. The sum of the first and last
unequal numbers, then 48 times the difference between the digits is equal to the third digit. Finally, the sum of the
two numbers equals the difference between the squares of second and fourth digits is twice the sum of the other two
the two numbers. “The wife looked puzzled. Can you help digits. What is the third digit of the number?
the merchant’s wife by finding out how many gold coins (a) 5 (b) 8
the merchant has? (c) 1 (d) 4
(a) 96 (b) 53 [Based on CAT, 2001]
(c) 43 (d) None of these 62. Anita had to do a multiplication. Instead of taking 35 as
[Based on CAT, 2002] one of the multipliers, she took 53. As a result, the product
54. A child was asked to add first few natural numbers (that went up by 540. What is the new product?
is 1 + 2 + 3 + ...) so long his patience permitted. As (a) 1050 (b) 540
he stopped, he gave the sum as 575. When the teacher (c) 1440 (d) 1590
declared the result wrong, the child discovered he had [Based on CAT, 2001]

Chapter_01.indd 10 1/30/2016 1:18:42 PM


Numbers  11

63. m is the smallest positive integer such that for any integer (a) n
n ≤ m, the quantity n3 – 7n2 + 11n – 5 is positive. What is (b) n + 1
the value of m? (c) k × n, where k is a function of n
(a) 4 (b) 5
(d) n +   
2
(c) 8 (d) None of these [Based on CAT, 2000]
7
[Based on CAT, 2001]
64. Three friends, returning from a movie, stopped to eat at a 70.
Let N = 1421 × 1423 × 1425. What is the remainder when
restaurant. After dinner, they paid their bill and noticed a N is divided by 12?
bowl of mints at the front counter. Sita took one-third of (a) 0 (b) 9
the mints, but returned four because she had a monetary (c) 3 (d) 6
pang of guilt. Fatima then took one-fourth of what was
[Based on CAT, 2000]
left but returned three for similar reasons. Eswari then
took half of the remainder but threw two back into the 71.
The integers 34041 and 32506, when divided by a three-
bowl. The bowl had only 17 mints left when the raid was digit integer n, leave the same remainder. What is the
over. How many mints were originally in the bowl? value of n?
(a) 38 (b) 31 (a) 289 (b) 367
(c) 41 (d) None of these (c) 453 (d) 307
[Based on CAT, 2001] [Based on CAT, 2000]
65. In a number system, the product of 44 and 11 is 1034. 72. f (a, b, c) = a + b + c and g (a, b, c) = a × b × c.
The number 3111 of this system, when converted to the Then how many such integer triplets a, b, c are there for
decimal number system, becomes which f (a, b, c) = g (a, b, c)? (a, b, c are all distinct).
(a) 406 (b) 1086 (a) 0 (b) Only 1
(c) 213 (d) 691 (c) 2 (d) More than 2
 [Based on CAT, 2001] Let N = 553 + 173 – 723. N is divisible by
73.
66. A set of consecutive positive integers beginning with 1 (a) both 7 and 13 (b) both 3 and 13
is written on the blackboard. A student came along and
(c) both 17 and 7 (d) both 3 and 17
erased one number. The average of the remaining numbers
[Based on CAT, 2000]
7 74.
Which of the following numbers is exactly divisible by
is 35 . What was the number erased?
77 99?
(a) 7 (b) 8 (a) 114345 (b) 135792
(c) 9 (d) None of these (c) 3572404 (d) 913464
[Based on CAT, 2001] [Based on MAT, 2005]
67. Let D be a recurring decimal of the form D = 0. ala2ala2ala2 75.
Of the three numbers, the sum of the first two is 45; the
..., where digits a1 and a2 lie between 0 and 9. Further, at sum of the second and the third is 55 and the sum of the
most one of them is zero. Which of the following numbers third and thrice the first is 90. The third number is
necessarily produces an integer, when multiplied by D?
(a) 20 (b) 25
(a) 18 (b) 108
(c) 30 (d) 35
(c) 198 (d) 288
[Based on MAT, 2005]
 [Based on CAT, 2000]
1 2 4
68. What is the value of the following expression? 76. If of of of a number is 12, then 30 per cent of the
8 3 5
 1   1   1   1  number will be
 2 + + + 
 (2 − 1)   (42 − 1)   (62 − 1)   (202 − 1)  (a) 48 (b) 64

9 10 (c) 54 (d) 42
(a) (b)
19 19 [Based on BSRB Chennai Bank PO, 2000]
77. When any number is divided by 12 then, dividend
10 11 becomes one-fourth of the other number. By how much
(c) (d)
21 21 per cent is first number greater than the second number?
[Based on CAT, 2000]
(a) 200 (b) 150
69. Consider a sequence of seven consecutive integers. The
average of the first five integers is n. The average of all the (c) 300 (d) Data inadequate
[Based on BSRB Chennai Bank PO, 2000]
seven integers is

Chapter_01.indd 11 1/30/2016 1:18:42 PM


12  Chapter 1

78. Which of the following numbers has maximum factors? 87. The number of people in a row is equal to the number
(a) 36 (b) 76 of rows in a playground. If total number of people in the
(c) 82 (d) 191 playground is 19044, find the number of rows.
79. Which of the following numbers has minimum factors? (a) 128 (b) 138
(a) 58 (b) 88 (c) 148 (d) 158
(c) 137 (d) 184 88. Let R be the remainder when 35n + 1 is divided by 7.
Which of the following statements are true?
80. From 1–90 how many numbers end in 4?
(a) 25 per cent (b) 30 per cent I. R = 4, when n is even.
II. R = 5, when n is even.
(c) 20 per cent (d) 10 per cent
III. R = 6, when n is odd.
81. The sum of the digits of a two digit number is of the IV. R = 3, when n is odd.
difference between the number and the number obtained
by interchanging the positions of the digits. What definitely (a) I and III (b) II and III
is the difference between the digits of that number? (c) II and IV (d) I and IV
(a) 5 (b) 9 89. If 2x – 1 is an odd number and 3y – 1 is an even number,
(c) 7 (d) Data inadequate which of the following is/are necessarily even?
[Based on BSRB Chennai Bank PO, 2000]
I. x2 – 2y + 2
82. A number gets reduced to its one-third when 48 is II. y2 – 2x + 3
subtracted from it. What is two-thirds of that number?
III. 4x2 – y – 1
(a) 24 (b) 72
(c) 36 (d) 48 (a) I only (b) II only
[Based on BSRB Bhopal Bank PO, 2000] (c) I and II (d) II and III
83.
The sum of three consecutive numbers is given, what is 90.
A number is decreased by 4 and divided by 6, the result
the difference between first and third number? is 9. What would be the result if 3 is subtracted from the
(a) 1 (b) 3 number and then it is divided by 5?
(c) either 1 or 2 (d) 2
2 1
[Based on BSRB Bhopal Bank PO, 2000] (a) 9 (b) 10
5 5
84. From 10–99 both inclusive how many numbers have their
unit digit smaller than the other digit? 2
(c) 11 (d) 11
(a) 90 (b) 45 5
  [Based on BSRB Delhi Bank PO, 2000]
(c) 32 (d) 26
91. A two digits number is seven times the sum of its digits. If
each digit is increased by 2, the number thus obtained is 4
5 13 5 13 more than six times the sum of its digits. Find the number.
85. If x =  6   6 ... to infinite terms,
2 4 2 4 (a) 42 (b) 24
then x = (c) 48 (d) data inadequate
3 2 3 5 [Based on BSRB Patna Bank PO, 2001]
(a) (b)
2 2 92. If A and B are positive integers such that 9A2 = 12A + 96
and B2 = 2B + 3, then which of the following is the value
2 5 3 5 1
(c) (d) of 5A + 7B?
3 2
(a) 31 (b) 41
86.
If the two digits of the age of Mr. Manoj are reversed (c) 36 (d) 43
1 
then, the new age so obtained is the age of his wife. [Based on BSRB Patna Bank PO, 2001]
11 93. The digit in the units place of a number is equal to the
of the sum of their ages is equal to the difference between
digit in the tens place of half of that number and the digit
their ages. If Mr Manoj is elder than his wife then find the
in the tens place of that number is less than the digit in
difference between their ages?
units place of half of the number by 1. If the sum of the
(a) Cannot be determined digits of the number is seven, then what is the number?
(b) 10 years (c) 8 years (a) 52 (b) 16
(d) 7 years (e) 9 years (c) 34 (d) Data inadequate
[Based on BSRB Bangalore Bank PO, 2000]  [Based on SBI Bank PO, 2001]

Chapter_01.indd 12 1/30/2016 1:18:42 PM


Numbers  13

94. Which of the following statements is/are true? 101.


The sum of two numbers, one of which is one-third of the
I. np – n is divisible by p where n and p are integers. other is 36. The smaller number is
II. np – n is divisible by p where n is a whole number and (a) 6 (b) 7
p is a natural number. (c) 8 (d) 9
III. np – n is divisible by p where n is an integer and p is a [Based on MAT (Sept), 2009]
prime number. 102.
If such numbers which are divisible by 5 and also those
(a) only I (b) only II which have 5 as one of the digits are eliminated from the
(c) only III (d) I and III numbers 1 to 60, how many numbers would remain?
(a) 40 (b) 47
95. Two fifths of one-third of three-sevenths of a number is
15. What is 40 per cent of that number? (c) 53 (d) 45
(a) 136 (b) 140 [Based on MAT (May), 2009]
(c) 72 (d) None of these 103.
How many numbers are there between 500 and 600 in
[Based on IBPS Jr. Executive Exam, 2002]
which 9 occurs only once?
96. If 3167 is added to 4093 and the sum is divided by 145, (a) 19 (b) 18
approximately what will be the outcome? (c) 20 (d) 21
(a) 50 (b) 75 [Based on MAT (Feb), 2009]
(c) 60 (d) 90 104.
One of a group of swans, 7/2 times the square root of the
(e) 80 number are playing on the shore of the pond. The two
 [Based on IBPS Jr. Executive Exam, 2002] remaining are inside the pond. What is the total number of
swans?
97. When a number is divided by 31 the remainder is 29.
When the same number is divided by 16, what will be the (a) 10 (b) 14
remainder? (c) 12 (d) 16
(a) Data inadequate (b) 13 [Based on MAT (Dec), 2008]

(c) 15 (d) 11 105.


A girl counted in the following way on the fingers of her
left hand; she started by calling the thumb 1, the index
[Based on IBP Jr. Executive Exam, 2002] finger 2, the middle finger 3, the ring finger 4, the little
98. If the digit in the units place of a two-digit number finger 5 and then reversed direction calling the ring figure
is halved and the digit in the tens place is doubled, the 6, the middle finger 7 and so on. She counted upto 1994.
number thus obtained is equal to the number obtained She ended counting on which finger?
by interchanging the digits. Which of the following is (a) The middle finger (b) The index finger
definitely true? (c) The thumb (d) The ring finger
(a) Digits in the units place and the tens place are equal. [Based on MAT (Sept), 2008]
(b) Sum of the digits is a two-digit number. 106.
An Army Commander wishing to draw up his 5180 men
(c) Digit is the units place is half of the digit in the tens in the form of a solid square found that he had 4 men less.
place. If he could get four more men and form the solid square,
the number of men in the front row is
(d) Digit in the unit’s place is twice the digit in the tens
place. (a) 72 (b) 68
[Based on PNB Management Trainee Exam, 2003] (c) 78 (d) 82
[Based on MAT (Feb), 2008]
99. Twenty times a positive integer is less than its square by
107.
To win an election, a candidate needs three-fourths of
96. What is the integer?
the votes cast. If after two-thirds of the votes have been
(a) 24 (b) 20 counted, a candidate has 5/6 of what he needs, then what
(c) 30 (d) Cannot be determined part of the remaining votes does he still need?
[Based on Bank of Maharashtra PO Exam, 2003] (a) 1/8 (b) 7/12
100.
Two times a two-digit number is 9 times the number (c) 1/4 (d) 3/8
obtained by reversing the digits and sum of the digits is 9. [Based on MAT (Feb), 2008]
The number is 108.
The sum of the place values of 3 in the number 503535 is
(a) 72 (b) 54 (a) 3300 (b) 0.6
(c) 63 (d) 81 (c) 60 (d) 3030
[Based on MAT (Feb), 2010] [Based on MAT (Feb), 2008]

Chapter_01.indd 13 1/30/2016 1:18:42 PM


14  Chapter 1

109.
Find the whole number which when increased by 20 is 117.
A number consists of two digits. If the digits in the unit’s
equal to one-sixth times the new number. place and the ten’s place are 7 and x respectively, the
(a) 7 (b) 5 number is
(c) 3 (d) 4 (a) x + 7 (b) 10(x + 7)
[Based on MAT (Sept), 2007] (c) 70 + x (d) 10x + 7
110.
A number when divided by 765 leaves a remainder 42. [Based on MAT, 1998]
What will be the remainder if the number is divided by 118. The sum of the digits of a three-digit number is 16. If the
17? ten’s digit of the number is three times the unit’s digit and
(a) 8 (b) 5 the unit’s digit is one-fourth of the hundredth digit, then
(c) 7 (d) 6 what is the number?
[Based on MAT (Sept), 2007] (a) 446 (b) 561
111.
After being set up, a company manufactured 6000 (c) 682 (d) 862
scooters in the third year and 7000 scooters in the [Based on MAT, 1998]
seventh year. Assuming that the production increases
119.
If one-third of a number is 3 more than one-fourth of the
uniformly by a fixed number every year, what is the
number, then what is the number?
production in the tenth year?
(a) 7850 (b) 7650 (a) 18 (b) 24
(c) 7750 (d) 7950 (c) 30 (d) 36
[Based on MAT, 1998]
[Based on MAT (May), 2006]
120.
A 2-digit number is four times the sum of the two digits.
112.
In a class, the number of girls is one less than the number
If the digits are reversed, the number so obtained is 18
of the boys. If the product of the number of boys and that
more than the original number. What is the original
of girls is 272, then the number of girls in the class is
number?
(a) 15 (b) 14
(a) 36 (b) 24
(c) 16 (d) 17
[Based on MAT (Feb), 2011]
(c) 48 (d) None of these
113.
A number of friends decided to go on a picnic and planned 121.
What is the least fraction which when added to or
to spend `96 on eatables. Four of them, did not turn up. As 29 15
subtracted from + will make the result a whole
a consequence, the remaining ones had to contribute `4 12 16
each extra. The number of those who attended the picnic number?
was 21 31
(a) 8 (b) 16 (a) (b)
38 38
(c) 12 (d) 24
[Based on MAT (Feb), 2006] 31 17
(c) (d)
114.
A box of light bulbs contains 24 bulbs. A worker replaces 48 48
17 bulbs in the shipping department and 13 bulbs in the [Based on MAT, 1999]
accounting department. How many boxes of bulbs did the 1  ....( n terms)

worker use?  1  1 4
 1 1 
1    1  3  
(a) 1 (b) 1 122.  2 2 
   is equal to
4    
3   
(c) 1 (d) 2  
4
[Based on MAT (Sept), 2003] 1

115.
Of the numbers 7, 9, 11, 13, 29, 33 how many are prime (a) (2) n1 (b) 2n
numbers? n
(a) 3 (b) 4 (c) 2 n1 (d) 2log n
(c) 5 (d) 6 123.
The number 311311311311311311311 is
[Based on MAT, 1998]
(a) divisible by 3 but not by 11
116. A three-digit number is selected such that it contains no
zeros. Now this three-digit number is written beside itself (b) divisible by 11 but not by 3
to form the six-digit number. Its factor is (c) divisible by both 3 and 11
(a) 5 (b) 11 (d) neither divisible by 3 nor by 11
(c) 4 (d) None of these [Based on SNAP, 2007]

Chapter_01.indd 14 1/30/2016 1:18:42 PM


Numbers  15

If p = 23n + 1, then which of the following is correct


124. 132.
A number n is said to be perfect, if the sum of all its
about p? divisors (excluding n itself) is equal to n. An example of
(a) p is always divisible by 24. perfect number is
(b) p is never divisible by 24. (a) 9 (b) 15
(c) p is always divisible by 22. (c) 21 (d) 6
(d) p is never divisible by 22. [Based on XAT, 2006
125.
A 3 digit number 4a3 is added to another 3-digit number n
984 to give the 4-digit number 13b7, which is divisible by 133.
For how many integers n, is the square of an
20  n
11. Then, (a + b) is integer?
(a) 10 (b) 11 (a) 0 (b) 1
(c) 12 (d) 15 (c) 2 (d) 3
[Based on FMS (MS), 2006]
 [Based on XAT, 2007]
126.
A three-digit number has, from left to right, the digits 134.
Let p be any positive integer and 2x + p = 2y, p + y = x
h, t and u with h > u. When the number with the digits and x + y = z. For what value of p would x + y + z attain its
reversed is subtracted from the original number, the units’ maximum value?
digit in the difference is 4. The next two digits, from right
to left, are (a) 0 (b) 1
(a) 5 and 9 (b) 9 and 5 (c) 2 (d) 3
(c) 5 and 4 (d) 4 and 5  [Based on XAT, 2007]
[Based on FMS, 2011] 135.
Let S be the set of rational numbers with the following
127.
In our number system the base is ten. If the base were properties
changed to four, you would count as follows: 1
(i) S;
1, 2, 3, 10, 11, 12, 13, 20, 21, 22, 23, 30, 2
The twentieth number would be 1 x
(ii) If x ≠ S, then both  S and S
(a) 110 (b) 104 x 1 x 1
(c) 44 (d) 38
Which of the following is true?
 [Based on FMS, 2011]
128.
If the square of a number of two digits is decreased by the (a) S contains all rational numbers in the interval 0 < x < 1.
square of the number formed by reversing the digits, then (b) S contains all rational numbers in the interval –1 < x < 1.
the result is not always divisible by
(c) S contains all rational numbers in the interval –1 < x < 0.
(a) 9
(d) S contains all rational numbers in the interval –1 < x < ∞.
(b) the product of the digits.
(e) S contains all rational numbers in the interval – 1 ≤ x≤ 1.
(c) the sum of the digits.
[Based on XAT, 2007]
(d) the difference of the digits.
 [Based on FMS, 2011] 136.
We define a function f on the integers f (x) = x/10, if x is
divisible by 10, and f (x) = x + 1 if x is not divisible by 10.
129.
If the digit 1 is placed after a two-digit number whose
If A0 = 1994 and An+1 = f (An), what is the smallest n such
ten’s digit is t, and unit’s digit is u, the new number is
that An = 2?
(a) 10t + u + 1 (b) 100t + 10u + 1
(a) 9 (b) 18
(c) 1000t + 10u + 1 (d) t + u + 1
(c) 128 (d) 1993
[Based on FMS, 2011]
130.
The smallest perfect square that is divisible by 7! is (e) An never equals 2
(a) 44100 (b) 176400  [Based on XAT, 2007]

(c) 705600 (d) 19600 137.


Four digits of the number 29138576 are omitted so that
[Based on IIFT, 2010] the result is as large as possible. The largest omitted digit
131.
How many two-digit numbers have their square as 1 more is
than a multiple of 24? (a) 9 (b) 8
(a) 30 (b) 31 (c) 7 (d) 5
(c) 32 (d) 29  [Based on XAT, 2008]

Chapter_01.indd 15 1/30/2016 1:18:43 PM


16  Chapter 1

irections (Q. 138): The question given below is followed by


D 143. The sum of the squares of two consecutive even numbers
two statements labelled as I and II. You have to decide if these is 6500. Which is the smaller number?
statements are sufficient to conclusively answer the question. (a) 54 (b) 52
Choose the appropriate answer from options given below (c) 48 (d) 56
(a) If Statement I alone is sufficient to answer the question. [Based on Punjab National Bank PO, 2010]
(b) If Statement II alone is sufficient to answer the question. 144. The sum of five consecutive even numbers of set-A is
(c) If Statement I and Statement II together are sufficient 220. What is the sum of a different set of five consecutive
but neither of the two alone is sufficient to answer the numbers whose second lowest number in 37 less than
question. double of the lowest number of set-A?
(d) If either Statement I or Statement II alone is sufficient to (a) 223 (b) 225
answer the question. (c) 235 (d) None of these
(e) Both Statement I and Statement II are insufficient to  [Based on CBI PO, 2010]
answer the question.
145. If x, y and z are consecutive negative integers, and if x > y
138.
A, B, C, D, E and F are six integers such that E < F, B > A, > z, which of the following must be a positive integer?
A < D < B. C is the greatest integer? (a) x – yz (b) xyz
I. E + B < A + D
(c) x + y + z (d) (x – y)(y – z)
II. D < F [Based on XAT, 2008]
(e) x(y – z) [Based on MHT-CET, MBA, 2010]
If x and y are real numbers, then the minimum value of x2
139.
146.  The product of two consecutive even numbers is 9408.
+ 4xy + 6y2 – 4y + 4 is
Which is the greater of the two numbers?
(a) –4 (b) 0
(a) 96 (b) 98
(c) 2 (d) 4
(c) 94 (d) 92
 [Based on XAT, 2010]
 [Based on Andhra Bank PO, 2008]
140.
Let X be a four-digit positive integer such that the unit
digit of X is prime and the product of all digits of X is also 147. The product of two successive even numbers is 6888.
prime. How many such integers are possible? Which is the greater of the two numbers?
(a) 4 (b) 8 (a) 78 (b) 82
(c) 12 (d) 24 (c) 86 (d) None of these
[Based on XAT, 2010] [Based on Uttrakhand GBO PO, 2007]

141.
The micro manometer in a certain factory can measure 148. Sum of three numbers is 132. First number is twice the
the pressure inside the gas chamber from 1 unit to 999999 second and third number is one-third of the first. Find the
units. Lately this instrument has not been working second number.
properly. The problem with the instrument is that it always (a) 18 (b) 36
skips the digit 5 and moves directly from 4 to 6. What is (c) 20 (d) 16
the actual pressure inside the gas chamber if the micro
manometer displays 003016? 149. The number obtained by interchanging the digits of a two-
digit number is less than the original number by 63. If the
(a) 2201 (b) 2202
sum of the digits of the number is 11, what is the original
(c) 2600 (d) 2960 number?
 [Based on XAT, 2011] (a) 29 (b) 92
142.
Let an = 1111111 ... 1, where 1 occurs n number of times. (c) 74 (d) Cannot be determined
Then, [Based on SBI PO, 2008]
I. a741 is not a prime. II. a534 is not a prime. 150. What is the least value of K so that the number 6735K1 is
III. a123 is not a prime. IV. a77 is not a prime. divisible by 9?
(a) (I) is correct (a) 5 (b) 7
(b) (I) and (II) are correct (c) 4 (d) 3
(c) (II) and (III) are correct 151. The number obtained by interchanging the two digits of a
(d) All of these are correct two digit number is less than the original number by 27. If
the difference between the two digits of the number is 3,
 [Based on XAT, 2011]
what is the original number?

Chapter_01.indd 16 1/30/2016 1:18:43 PM


Numbers  17

(a) 74 (b) 63 159.


If x and y are any natural numbers, then which of the
(c) 85 (d) Cannot be determined following is an odd number?
 [Based on IRMA, 2009] (a) xy + yx (x – y) (xy + x) (b) xy (x + y) (xy + x)
152. In certain games, each player scores either 2 points or 5 (c) yx (x2 – y) (xy – x) (d) None of these
points. If n players score 2 points and m players score 5 [Based on ATMA, 2008]
points and the total number of points scored is 50, what is
the least possible positive difference between n and m? 160.
a, b and c are positive integers divisible by 5, 3 and 12
respectively and p is a two-digit prime number, then
(a) 5 (b) 3
which of the following statement (s) is/are TRUE?
(c) 1 (d) 7
I. Product of abcp is zero.
[Based on NMAT, 2005]
a + b + c + p is odd.
II.
153. What least number must be added to 7231 so that the
resulting number is exactly divisible by 5 and 9 together? III. (b2 + c2) – (p2 – a2) is odd.
(a) 20 (b) 18 a (p – c) + a (c + b) is divisible by 5.
IV.
(c) 14 (d) 16 (a) I and IV only (b) II and III only
154. If the digit in the unit’s place of a two-digit number is (c) II and IV only (d) I, II, III and IV
halved and the digit in the ten’s place is doubled, the [Based on ATMA, 2008]
number thus obtained is equal to the number obtained
by interchanging the digits. Which of the following is 161.
If x, y and z and positive integers such that x is a factor of
definitely true? y and x is a multiple of z, which of the following is NOT
necessarily an integer?
(a) Digit in the unit’s place and the ten’s place are equal.
(b) Digit in the unit’s place is twice the digit in the ten’s xy y+z
(a) (b)
place z x
(c) Sum of the digits is a two-digit number yz x+ y
(c) (d)
(d) Digit in the unit’s place is half of the digit in the ten’s x z
place [Based on NMAT, 2005] [Based on ATMA, 2008]
155. If m and n are two integers such that m × n = 64, which of
If a is a positive integer and if the unit’s digit of a2 is 9 and
162.
the following cannot be the value of m + n?
(a + 1)2 is 4, what is the unit’s digit of (a + 2)2?
(a) 20 (b) 65
(a) 1 (b) 3
(c) 16 (d) 35 [Based on ATMA, 2005]
(c) 5 (d) 14
156. If the quotient is positive, which of the following must be
[Based on ATMA, 2008]
true?
(a) a > 0 (b) b > 0 163.
When 10 is divided by the positive integer n, the remainder
is n – 4. Which of the following could be the value of n?
(c) a – b > 0 (d) ab > 0
[Based on ATMA, 2006] (a) 3 (b) 4
157.
If a positive integers n is divisible by both 5 and 7, n must (c) 7 (d) 12
also be divisible by which of the following? [Based on ATMA, 2008]
I. 12 II. 35 164. 4109 + 6109 is divided by 25, the remainder is
III. 70 (a) 20 (b) 10
(a) None (b) II only (c) 5 (d) 0
(c) I and II (d) II and III [Based on JMET, 2006]
[Based on ATMA, 2006] 165. What is the digit in the units place of 10251?
158.
A number consists of there digits whose sum is 10. The (a) 2 (b) 4
middle digit is equal to sum of the other two and the
number will be increased by 99, if the final digit and the (c) 6 (d) 8
third digit are interchanged. The digit in the hundreds [Based on JMET, 2006]
place is 166. Find the least number which must be subtracted from
(a) 3 (b) 5 9269 so that resulting number is exactly divisible by 73.
(c) 4 (d) 2 (a) 17 (b) 57
[Based on ATMA, 2006] (c) 71 (d) 63

Chapter_01.indd 17 1/30/2016 1:18:43 PM


18  Chapter 1

167. Find the least number which must be added to 15463 so 176.
If a and b are odd numbers, then which of the following is
that the resulting number is exactly divisible by 107? even?
(a) 52 (b) 71 (a) a + b + ab (b) a + b – 1
(c) 55 (d) 19 (c) a + b + 1 (d) a + b + 2ab
168. If a, b, c, d and e are real numbers such that a + b < c + d, [Based on SSC (GL), 2011]
16
b + c < d + e, c + d < e + a and d + e < a + b, then 177.
2 – 1 is divisible by
(a) the largest number is a and the smallest is b. (a) 11 (b) 13
(b) the largest number is a and the smallest is c. (c) 17 (d) 19
(c) the largest number is e and the smallest is c. [Based on SSC (GL), 2011]
(d) the largest number is c and the smallest is b. 178.
The sum of two numbers is 24 and their product is 143.
[Based on GBO, Delhi University, 2011] The sum of their squares is
Let 2x + y = 10, 2y + z = 20 and 2z + x = 30 where x, y and z
169. (a) 296 (b) 295
are any three real numbers. The value of 2x is (c) 290 (d) 228
3 [Based on SSC (GL), 2011]
(a) (b) 15 372
2 179.
The unit digit in the sum (124) + (124)373 is
6 (a) 5 (b) 4
(c) (d) 15
2 (c) 2 (d) 0
[Based on GBO, Delhi University, 2011] [Based on SSC (GL), 2011]
170. What is the number just more than 5000 which is exactly 180.
If the sum of two numbers be multiplied by each number
divisible by 73? separately, the products so obtained are 247 and 114. The
(a) 5001 (b) 5009 sum of the numbers is

(c) 5037 (d) 5027 (a) 19 (b) 20


(c) 21 (d) 23
171. The sum of two numbers is 100 and their difference is 37.
[Based on SSC (GL), 2011]
The difference of their squares is
181.
Find a number, one-seventh of which exceeds its eleventh
(a) 37 (b) 100
part by 100.
(c) 63 (d) 3700
(a) 1925 (b) 1825
172.
A number is of two digits. The position of digits (c) 1540 (d) 1340
is interchanged and the new number is added to [Based on SSC (GL), 2011]
the original number. The resul­tant number will always be
divisible by 182. 6  6  6  ... 
?
(a) 8 (b) 9
(a) 2.3 (b) 3
(c) 10 (d) 11
(c) 6 (d) 6.3
[Based on U.P.P.C.S., 2012]
[Based on SSC (GL), 2011]
173.
The number of times 79 be subtracted from 50000, so that
183. The nearest figure to 58701 which is divisible by 567 is
the remainder be 43759 is
(a) 58968 (b) 58434
(a) 69 (b) 79
(c) 58401 (d) None of these
(c) 59 (d) None of these
184. The number of five figures to be added to a number of
When 223 is divided by 10, the remainder will be
174. four fives to obtain the least number of six figures exactly
(a) 2 (b) 3 divisible by 357 is
(c) 4 (d) 8 (a) 94762 (b) 94802
[Based on SSC (GL), 2011] (c) 94485 (d) None of these
245 72
175.
Find the unit digit in the product (4387) × (621) . 185. The least value to be given to * so that the number
(a) 1 (b) 2 5 * 3457 is divisible by 11 is
(c) 5 (d) 7 (a) 2 (b) 3
[Based on SSC (GL), 2011] (c) 0 (d) 4

Chapter_01.indd 18 1/30/2016 1:18:43 PM


Numbers  19

EXPLANATORY ANSWERS

1.
(b) a, b, c and d are four integers such that a + b + c + d (d) If 42 is divided by 6, remainder is 4
6.
= 4m + 1. If 43 is divided by 6, remainder is 4
Minimum possible value of a2 + b2 + c2 + d2 is when If 44 is divided by 6, remainder is 4
a, b, c and d are as close to each other as possible.
... ... ...
Since RHS is not the multiple of 4, as, b, c and d can’t
be equal to m. If 496 is divided by 6, remainder is 4.
Hence the numbers may be of the form, m, m, m and 7.
(a) Let x be the number,
m + 1. x  12
\ a2 + b2 + c2 + d2 = 4m2 + 2m + 1. \ (x + 12) ÷ 6 = 112 ⇒ = 112
6
2.
(c) We have to find the number of three-digit numbers in ⇒ x = 112 × 6 – 12
which the digit at ten’s place is greater than the digit
⇒ x = 672 – 12 = 660
at unit’s and hundred’s places. That is,
x  12
Hundred Ten Unit \ Correct answer =
6
x y z
660
x<y>z =  12 = 110 + 12 = 122
6
The following chart shows the number of ways in
which it can be formed. (d) (n + 1)2 = n2 + 2n + 1
8.
Number of ways in Digit at ten’s Number of ways (b) Last 2 digits of (125)521 will be 25.
9.
which unit’s place, place, i.e., y in which unit’s To find the last two digits of (521)125, we need to
i.e., x can be filled place i.e., y can consider (21)125 only.
be filled
The last 2 digits for different powers of 21 are:
1(i.e., 1) 2 2(i.e., 0, 1)
2(i.e., 1, 2) 3 3(i.e., 0, 1, 2) (21)1 → 21

... ... ... (21) 2 → 41
8(i.e., 1, 2, 3, ... 8) 9 9(i.e., 0, 1, 2, ...9) 
(21)3 → 61
\ Total no. of possible 3-digit numbers  It is a cycle of 5 for the last two digits.
(21) 4 → 81
= (1 × 2) + (2 × 3) + (3 × 4) + (4 × 5) 
+ ... + (7 × 8) + (8 × 9) = 240. (21)5 → 01
3. (b) Product (n – 1) (n – 2) ... 3 × 2 × 1 is not divisible by (21)6 → 21
n if n is 4 or a prime number. So, 125 being divisible by 5, the last 2 digits of
We have to find the number of primes in (521)125 will be 01.
12 ≤ n ≤ 40. Thus, 25 × 01 = 25
i.e., {13, 17, 19, 23, 29, 31, 37} Therefore, last two digits of N are 25.
\ No. of positive integers in the range 10.
(c) Let the three-digit number be 100x + 10y + z
12 ≤ n ≤ 40 is 7. \ (100x + 10y + z) – (x + y + z) = 99x + 9y
4. (d) Take any arbitrary value of x and y = 9 (11x + y)
Let x = 2 (prime number) which is always divisible by 9.
y = 50 (composite number)
11.
(c) Let x must be subtracted from 14, 17, 34 and 42 such
Going through the options, that
x y
(a), (b) and (c) are wrong because y – x, xy and (14 – x) (42 – x) = (17 – x) (34 – x) ⇒ x = 2
x
are even integers for x = 2 and y = 50 12. (b) 125125 – 2525 = 5375 – 550 = 550(5325 – 1).
\ None of the statements are true. Now 5325 – 1 is not divisible by 5. Hence, the highest
5. (a) The set of prime numbers 3, 5, 7 is the only set which power of 5 that is contained in the given expression is
satisfies the given condition. 50.

Chapter_01.indd 19 1/30/2016 1:18:43 PM


20  Chapter 1

13. (a). After 54, the remainder left when 5n is divided by 22.
(d) For every integer a, a + (– a) = 0. Therefore, by
1000 is 125 when n is odd and 625 when n is even. pairing 1 with –1, 2 with –2, and so on, one can
Hence, the remainder is 125. see that in order for the sum to be zero, a list of
consecutive integers must contain the same number
2 of positive integers as negative integers, in addition
14.
(a) No. of women in the room =  120 = 72
5 to the integer ‘0’. Therefore, the list has an odd
number of consecutive integers and their average
2
No. of married people =  120 = 80 will also be 0.
5
No. of unmarried people = 40 23.
(b) Let the digits be a and b such that the number is
10a + b.
No. of men in the room = 48
\ ab = 14 and 10a + b + 45
If all the men are supposed to be married, then number
of married women could be 80 – 48 = 32 = 10b + a
\ Maximum number of unmarried women could be i.e., 9a – 9b = – 45
72 – 32 = 40 i.e., a – b = – 5
\ (a + b)2 = (a – b)2 + 4ab = 81
15.
(d) x = 2 + 22/3 + 21/3 ⇒ x > 2
⇒ a + b = 9
For x = 2, x3 – 6x2 + 6x = –4
⇒ a = 2, b = 7
For x = 3, x3 – 6x2 + 6x = –9
\ x3 – 6x2 + 6x < 0 \ The number is 27.
24.
(c) No. of Spades = 1
16.
(a) 248 in the scale of 7 is written as 503. In scale 9, it is
written as 305. No. of Diamonds = 2
No. of Hearts = 4,
x x 1
17.
(d) Given expression =  No. of Clubs = 6.
2 2
2x  1 1 25.
(d) No. of balls in Red Box = 18
= = x  = [x]
2 2 No. of balls in Blue Box = 36
18.
(a) 5 3 6 4 7 is a multiple of 11 because the difference of No. of balls in Green Box = 54.
the sum of the digits in the odd places and the sum of
(d) The given product contains 5106 and 2x
26.
the digits in the even places is divisible by 11, because
where x > 106
(5 + 6 + 7) – (3 + 4) = 11
\ Total number of five-digit multiples of 11 \ There will be 106 zeroes in the product, because zero
will come only by multiplying 2 and 5.
= 3! (Permutation of 5, 6 and 7 in the odd places)
  × 2! (Permutation of 3 and 4 in the even places) 27.
(c) Such numbers are 56, 67, 78 and 89
= 6 × 2 = 12. Sum of these numbers = 290.

3600 28.
(b) 20 × 21 × 22 × 23 × 24 = 5100480
19.
(d) = 8 = 23.
450 25 × 26 × 27 × 28 = 491400
20.
(d) 1024. 29 × 30 = 870.
3 29.
(d) Sum of the prime numbers between 10 and 40 = 11 +
21.
(b) 7 = 343, when divided by 342, leaves a remainder
of 1 13 + 17 + 19 + 23 + 29 + 31 + 37 = 180.
74 = 2401, when divided by 342, leaves a remainder 100
of 7.
5
30.
(d)  in = (i + i2 + i3 + i4) + (i5 + i6 + i7 + i8) +
7 = 16807, when divided by 342, leaves a remainder n 1
of 49. ... + (i97 + i98 + i99 + i100)
76 =
117649, when divided by 342, leaves a = (i – 1 – i + 1) + (i – 1 – i + 1) + ...
remainder of 1. = 0 + 0 + ... + 0 = 0.
And so on. 31.
(d) Let the five consecutive even numbers 2n, 2n + 2,
\ 784, when divided by 342, will leave a remainder 2n + 4, 2n + 6, 2n + 8 be respectively equal to a, b, c,
of 1. d and e, where n is a natural number.

Chapter_01.indd 20 1/30/2016 1:18:44 PM


Numbers  21

Then v, w, x, y and z are equal to 2n + 1, 2n + 3, In the given number last three digits are 354. So, the
2n + 5, 2n + 7, 2n + 9. remainder is 2.
2n  2n  2  2n  4  2n  6  2n  8 p
⇒ N = 41.
(b) Let the rational number be
2n  1  2n  3  2n  5  2n  7  2n  9 q
\ q = p + 3
10n  20 n2
= = . p7
10n  25 n  2.5 \ = 2 ⇒ p + 7 = 2p + 2
p 32
32.
(b) For the number to be divisible by 3, the sum of the
⇒ p = 5
digits of a number should be divisible by 3. Also, for
the number to be divisible by 9, the sum of the digits 5
⇒ Given rational number = .
of a number should be divisible by 9. Hence options 8
(c) and (d) are ruled out as all the given numbers are 42.
(d) Let the two numbers be ab and xy.
not divisible by 9 (because the sum of their digits is
not divisible by 9). Option (b) is the answer as 3 and \ (100a + b) × (100x + y) = (100b + a) + (100y + x)
37 are factors of 111 and 111 is the divisor of all the ⇒ 10000ax + 100ay + 100bx + by
given numbers. = 10000by + 100bx + 100ay + ax
33.
(c) The prime numbers are 2, 3, 5, 7, 11, 13, 17, 19, 23, 29 ⇒ 9999ax = 9999by
etc.
⇒ ax = by
The numbers 5, 11, 17, 23, 29 form an increasing
Now, check from the options.
sequence for which 29 is the fifth term.
For option (d): a = 1, b = 3, x = 6, y = 2
34.
(c) 1012 – 1 = (106 – 1) (106 + 1)
\ ax = 1 × 6 = 6 and by = 3 × 2 = 6.
= (103 – 1) (103 + 1) (106 + 1)
Hence ax = by.
= 999 × 1001 × 1000001
999  1001  1000001 (b) When 1715 is divided by 6
43.
So, (1012 – 1 ¸ 111) =
111 (18  1)15 , remainder = 5
= 9 × 1001 × 1000001 6
= 9009009009 \ 1715 can be written as 6K + 5
(b) N = 461 + 462 + 463 + 464
35. 116K+5 (7+4)6K+5 46K+5
= 461(1 + 4 + 16 + 64) = 461 × 85 \ = =
7 7 7
= 461 × 5 × 17 = 460 × 4 × 5 × 17
16 × (43 ) 2K+1 16 × (63 + 1) 2K+1
= 460 × 2 × 17 × 10 = =
7 7
Hence, it is divisible by 10
Remainder = 2 × 1 = 2
36.
(b) 889 = 7 × 127 15

899 = 29 × 31 1311 (14 – 1)odd


    = ⇒ Remainder = 6
7 7
1147 = 31 × 437.
\ Remainder when 111715 + 131715 is divided by
37.
(a) 179 is a prime number. 7 is 1.
203 = 7 × 29
44.
(d) Divisor = 12 × Quotient
143 = 11 × 13
Divisor = 5 × Remainder
267 = 3 × 89.
Remainder = 48
38.
(c) Multiple of 120 + 5, which is divisible by 13.
⇒ Divisor = 240,
39.
(d) Let n be the number. \ Quotient = 20
Then, the result is Hence,
 {(2n  6)10}  2n  6  2n Dividend = 240 × 20 + 48 = 4848.
=   –x= =3
 20  2 45.
(a) Divisors of 88 are 2, 4, 8, 11, 22, 44.
40.
(c) By the rules of divisibility, we know that any number Divisors of 91 are 7 and 13.
is divisible 8, if the last three digits of the number is
Divisors of 99 are 3, 9, 11, 33.
also divisible by 8.

Chapter_01.indd 21 1/30/2016 1:18:44 PM


22  Chapter 1

(d) n13 – n is divisible by 13 for all n ∈ whole numbers


46. 56.
(d) 3 {4 (7x + 4) + 1} + 2 = 84x + 53
52n – 1 is divisible by 13 for even n Therefore, remainder is 53.
The smallest even number is 2. (b) 76n – 66n
57.
\ When n = 2, the expression is divisible by 169. Put n = 1
47.
(d) When a number is divided by 6 possible remainders 76 – 66 = (73 – 63) (73 + 63)
are 1, 2, 3, 4, 5 (x = 6y + remainder). But only odd This is a multiple of 73 – 63 = 127.
numbers are possible as with even numbers the
remainder when divided by 2 would be 0. 58. (a) Here the least value of x = 49 = –7
Of 1, 3, 5 only for 5, division by 3 has remainder 2. and the least value of y = 16 = –4
\ Remainder when divided by 6 = 5 So, the least number here which when divided by –7
and –4 gives a negative quotient in each case is 28
1 11 28 28
48.
(b) 1 = since = –4 and = –7
10 10 –7 –4
1 1 59. (c) Since 64 ÷ 24 = 1296 ÷ 16 = 81
1 =
10 10 and remainder 0.
11 1 So, we need not calculate the remainder in second
\ Sum = = 11  = 11 × Product. case as the product will be 0
10 10
49.
(c) m(n + 0)( p – q) is even ⇒ m must be even. 60.
(a) Since he has to put minimum 120 oranges and
maximum 144 oranges, i.e., 25 oranges need to be
50.
(c) Total possible arrangements = 10 × 9 × 8 filled in 128 boxes with same number of oranges in
Now three numbers can be arranged among themselves the boxes.
in = 3! ways = 6 ways Therefore, total 125 = 25 × 5 oranges could be filled
Given condition is satisfied by only 1 out of 6 ways. in the boxes, i.e., 25 in each of the 5 boxes which
Hence, required number of arrangements would be the minimum and have the same number of
oranges.
10  9  8
= = 120. Hence, the answer is 5.
6
51.
(b) Check choices 61.
(a) Let the 4-digit number be abcd. Then,
a + b = c + d ...(1)
54 ⇒ S = (5 + 4)2 = 81
b + d = 2 (a + c) ...(2)
⇒ S – D = 81 – 54 = 27.
and a + d = c ...(3)
52.
(b) Escaped with 1 From Eqs. (1) and (3), b = 2d.
Before 3rd watchman, he had (1 + 2) × 2 = 6 From Eqs. (1) and (2), 3b = 4c + d
Before 2nd watchman, he had (6 + 2) × 2 = 16 ⇒ 3 (2d) = 4c + d
Before 1st watchman, he had (16 + 2) × 2 = 36. ⇒ 5d = 4c
53.
(d) Let the no. of gold coins = x + y 5
⇒ c =  d
48 (x – y) = x2 – y2 4
⇒ 48 (x – y) = (x –­ y) (x + y) Now d can be 4 or 8. But if d = 8, then c = 10 is not
⇒ x + y = 48. possible. So d = 4 which gives c = 5.
62.
(d) Let the number be x
n2  n Increase in product = 53x – 35x = 18x
54.
(d) 575 = x
2
⇒ 18x = 540 ⇒ x = 30
⇒ 1150 = n2 + n – 2x Raised product = 53 × 30 = 1590.
For n
= 34, 63.
(d) Let y = n3 – 7n2 + 11n – 5
40 = 2x At n = 1, y = 0
\ x = 20. \ (n – 1) (n2 – 6n + 5) = (n – 1)2 (n – 5)
55.
(a) (2 ) = (17 – 1)64 = 17n + (–­1)64
4 64 Now, (n – 1)2 is always positive.
= 17n + 1 Also, for n < 5, the expression gives a negative
quantity. Therefore, the least value of n will be 6.
Hence, remainder = 1.
Hence, m = 6.

Chapter_01.indd 22 1/30/2016 1:18:44 PM


Numbers  23

64.
(d) Let there be x mints originally in the bowl. 71.
(d) Let r be the remainder. Then 34041 – r and 32506 –
1 r are perfectly divisible by n. Hence, their difference
Sita took , but returned 4. should also be divisible by the same.
3
2 (34041 – r) – (32506 – r) = 1535
So now the bowl has x  4 mints. which is divisible by only 307.
3
1 72.
(d) Any triplet of the form (– n, 0, n) satisfies the given
Fatima took of remainder, but returned 3. condition e.g., (–2, 0, 2).
4
32  (d) N can be written either (54 + 1)3 + (18 – 1)3 – 723 or
73.
So, the bowl has  x  4  3 mints. (51 + 4)3 + 173 – (68 + 4)3.
43
Eswari took half of remainder that is The first form is divisible by 3, and the second by 17.
1 3 2   74.
(a) A number divisible by 99 must be divisible by 9 as
 x  4  3 . She returns 2, so the bowl now
2  4  3  well as 11.
\ 114345 is divisible by both.
1 3 2  
has  x  4  3 + 2 = 17 ⇒ x = 48.
2  4  3 
75.
(c) Let the numbers be x, y and z.
x + y = 45, y + z = 55 and 3x + z = 90
65.
(a) The product of 44 and 11 is 484
y = 45 – x,
Here 3x3 + 4x2 + 1x1 + 4 × x0 = 484
z = 55 – y = 55 – (45 – x) = 10 + x
⇒ 3x3 + 4x2 + x = 480
\ 3x + 10 + x = 90
This equation is satisfied only when x = 5. or,   x = 20
In decimal system, the number 3111 can be written y = 45 – 20 = 25
as 406.
z = 10 + 20 = 30
66.
(a) Let the highest number be n.
\ Third number is 30.
n (n  1)
x
2 7 602 76.
(c) Let the number be x
Then, = 35 = ,
(n  1) 77 17 1 2 4
and, of of × x = 12
where x is the number erased. 8 3 5
Hence, n = 69 and x = 7 satisfy the above conditions. 3x 3
\ = × 12 × 15 = 54.
aa 10 10
(c) 99 × D = a1a2. Hence D = 1 2 . So D must be
67.
99 77.
(d) Here neither the remainder nor the dividend nor
multiplied by 198 as 198 is a multiple of 99. the second number is given, so number cannot be
1 1 1 1 determined.
68.
(c)    ... 
1.3 3.5 5.7 19.21 78.
(a) 36 = 2 × 2 × 3 × 3.
1  1 1  1 1 1  1 1  1 1 1 Hence, divisors of
= 1            ...   
2  3 2  3 5  2  5 7  2  19 21 36 = 1, 2, 3, 4, 6, 9, 12, 18, 36, i.e., 9 in all.
1 1 (21  1) 20 10 10 76 = 2 × 2 × 19
=      . Hence, divisors of
2 42 42 42 21 21
76 = 1, 2, 4, 19, 38, 76 i.e., 6 in all.
69.
(b) Use any 7 consecutive numbers to check the answers.
82 = 2 × 41.
(1  2  3  4  5)
n = = 3, average of 7 integers is Hence, divisors of
5
82 = 1, 2, 41, 82 i.e., 4 in all.
(1  2  3  4  5)
k = = 4. So k = n + 1. 191 = 1 × 191.
7
Hence, divisors of
70.
(c) N = 1421 × 1423 × 1425. When divided by 12, it shall 191 = 1, 191 i.e., 2 in all.
[(1416  5)  (1416  7)  (1416  9)] 79.
(c) 58 = 2 × 29.
look like .
12 Hence, divisors of
Now the remainder will be governed by the term 5 × 7
58 = 1, 2, 29, 58 i.e., 4 in all.
× 9, which when divided by 12 leaves the remainder 3.
88 = 2 × 2 × 2 × 11

Chapter_01.indd 23 1/30/2016 1:18:45 PM


24  Chapter 1

Hence, divisors of 2
 2 5  = 13
88 = 1, 2, 4, 8, 11, 22, 44, 88, i.e., 8 in all. Þ  x    6x
2 4
137 = 1 × 137
(Squaring both sides again)
Hence, divisors of
137 = 1, 137 i.e., 2 in all. Going by the choices, only x = 3  5 satisfies the
2
Hence, divisors of equation above.
184 = 1, 2, 4, 8, 23, 46, 92, 184 i.e., 8 in all. 86.
(e) Let the age of Mr. Manoj be (10x + y) years.
80.
(d) Total number of numbers, which end with 4 = 9. \ His wife’s age = (10y + x) years
Total numbers from 1 to 90 = 90 1
Then, (10x + y + 10y + x)
9 11
Therefore, required percentage = = 10%
90 = 10x + y – 10y – x
81.
(a) Let the two digit number be 10x × y or, x + y = 9 x – 9y  or, 8x = 10y
1 x 5
Then, x + y = (10x + y – 10y – x) or, =
5 y 4
\ x = 5 and y = 4
9
or, x + y = (x – y) [Q any other multiple of 5 will make x of two digits]
5
\   Difference = 10x + y – 10y – x = 9x – 9y
x 7
or, 4x – 14y = 0 ⇒ = = 9 (x – y) = 9 (5 – 4) = 9 years.
y 2
Using componendo and dividendo, we have 87. (b) Assume the number of rows be n.
Then n × n = 19044
x y 72 9
= = or n = 138
x y 72 5
i.e., x – y = 5k
0 1
88. (a) n = 0 ⇒ 35 + 1 = 4, n = 1 ⇒ 35 + 1 = 244
Here k has the only possible value, k = 1. The remainders can be seen to be R = 4, when n = 0,
Because the difference of two single-digit numbers i.e., even and R = 6 when n = 1, i.e., odd. Therefore
will always be of a single digit. I and III are true.
82.
(d) Let the number be x. 89. (d) 2x – 1 is an odd number.
x ⇒ x can be either odd or even.
Then, x – = 48
3
3y – 1 is an even number.
2 ⇒ y is an odd number.
\ x = 48.
3
I. In x2 – 2y + 2, 2y is even, but x2 can be either odd or
83.
(d) Let the three consecutive numbers be x, x + 1 and
even, so we can’t say whether
x + 2 respectively.
x2 – 2y + 2 is odd or even.

\ Difference between first and third number
= x + 2 – x = 2. II. In y2 – 2x + 3, y2 is odd, 2x is even and 3 is odd
84.
(b) There are 99 – 10 + 1 = 90 two digit numbers in all. ⇒ y2 – 2x + 3 is even.
We can have 0–9 digits at unit’s place. For 0 in unit’s III. In 4x2 – y – 1, 4x2 is even, y is odd and 1 is odd
place we can have 1–9 digits at tens place i.e., we have ⇒ 4x2 – y – 1 is even.
9 choices. For 1 in unit’s place we have 8 choices and
so on. Hence, total numbers satisfying given condition 90.
(d) Let the number be x
= 9 + 8 + . . . + 1 = 45. x4
\ = 9 ⇒ x = 58
85.
(b) Squaring both sides of the given equation 6
5 13 x3 58  3 55
x2 =   6x Again, = = = 11.
2 4 5 5 5
5 13 91.
(a) Let the two digit number be 10x + y
Þ x2  =  6x
2 4 10x + y = 7 (x + y) ⇒ x = 2y ...(1)

Chapter_01.indd 24 1/30/2016 1:18:45 PM


Numbers  25

10 (x + 2) + y + 2 = 6 (x + y + 4) + 4 99.
(a) Let the positive integer be x.
or, 10x + y + 22 = 6x + 6y + 28 Now, x2 – 20x = 96
2
⇒ 4x – 5y = 6 ...(2) or, x – 20x – 96 = 0
Solving equations (1) and (2), we get x = 4, y = 2. or, x2 – 24x + 4x – 96 = 0
92.
(b) 9A2 = 12A + 96 ⇒ 3A2 – 4A – 32 = 0 or, x (x – 24) + 4 (x – 4) = 0
or, (x – 24) (x + 4) = 0
4  16  384 8
\ A = = 4, or, x = 24, – 4.
6 3
100.
(d) Let the two-digit number = xy
B2 = 2B + 3 ⇒ B2 – 2B – 3 = 0
\ 2(10x + y) = 9(10y + x)
2  4  12
\ B = = 3, – 1 ⇒ 88y – 11x = 0 ...(1)
2
Also, x + y = 9 ...(2)
\ 5A + 7B = 5 × 4 + 7 × 3 = 20 + 21 = 41.

Solving Eqs. (1) and (2), we get
1 x = 8 and y = 1
93.
(a) Let of the number = 10x + y and the number =
2 So, the number is 81.
10V + W
101.
(d) Let the numbers be 3x and x.
From the given conditions,
3x + x = 36
W = x and V = y – 1
⇒ 4x = 36
Thus, the number = 10 ( y –1) + x ...(A)
⇒ x = 9
\ 2(10x + y) = 10( y – 1) + x
102.
(a) Eliminated numbers are 5, 10, 15, 20, 25, 30, 35, 40,
⇒ 8y – 19x = 10 ...(1) 45, 50, 51, ..., 60
Again, from the question, So, total eliminated numbers are 20.
V + W = 7 ⇒ y – 1 + x = 7 \ 40 numbers would remain.
\ x + y = 8 ...(2) 103.
(b) Required numbers are 509, 519, 529, 539, 549, 559,
Solving equations (1) and (2), we get 569, 579, 589, 590, 591, 592, ..., 1598
x = 2 and y = 6 104.
(d) Let the total number of swans be x.
\ From equation (A) 7
The number of swans playing on shore = x
Number = 10(y – 1) + x = 52. 2
Number of remaining swans = 2
(c) For any integer n, n3 – n is divisible by 3, n5 – n is
94. \ x =
7
x+2
divisible by 5, n11 – n is divisible by 11 but n4 – n is 2
not necessarily divisible by 4. Thus, statement III is 7
true. ⇒ (x – 2) = x
2
95.
(d) Let the number be ‘x’. Then,
105. (d) Thumb Index Middle Ring Little
2 1 3
× × × x = 15 1 2 3 4 6
5 3 7
2x 15  35 9 ← 8 7 6
or, = 15  or,  x = 10 11 12 13
35 2
17 ← 16 15 14
2 15  35
\   40% of x = × = 105. 18 19 20 21
5 2
25 ← 24 23 22
3167  4093 7260 26 27 28 29
96.
(a) Required number = = ≈ 50.
145 145 33 ← 32 31 30
97.
(a) Quotient is not given. Hence, remainder cannot be From the above counting pattern, we find that every
determined. multiple of 8 comes on index finger and moves
towards thumb therefore, the last multiple of 8 which
98.
(d) Suppose the two digit number is 10x + y
1994
Then, 10y + x = 20x + y/2 appears on index finger will be ⇒ 1992.
8
or, 20y + 2x = 40x + y  or,  y = 2x.
Hence, 1994 will be on ring finger.

Chapter_01.indd 25 1/30/2016 1:18:46 PM


26  Chapter 1

106.
(a) Total number of men = 5180 + 4 = 5184 Hence, number of friends who attended the picnic

\ Number of men in first row = 5184 = 72 = 12 – 4 = 8
114.
(d) Number of boxes used
107.
(b) Let total number of votes cast be x.
2 17 + 13 30 5 1
Total number of counted votes = x = = = = 1
3 24 24 4 4
Since, the number of boxes used should be a whole
5 2 5
Votes that candidate got = × x= x number, hence the number of boxes used is 2.
6 3 9
115.
(b) There are four prime numbers, viz., 7, 1 1, 13, 29.
3 5 7
Votes still need to win = x − x =x
4 9 36 116.
(b) Let the number be abc; so the 6-digit number is abcabc
Now, the sum of alternate digits is
1
Remaining uncounted votes = x (i) a + c + b
3
7 3 7 (ii) b + a + c

\ Required part = × =
36 1 12 Both being equal, the 6-digit number is definitely
divisible by 11.
108.
(d) Required sum = 3000 + 30 = 3030
117.
(d) 10 × x + 7 = 10x + 7
109.
(d) Let the whole number be x.
118.
(d) Let x, y and z be the digits at the hundredth place, ten’s
1
\ x = ( x + 20) place and unit’s place respectively.
6
\ x + y + z = 16 ...(1)
⇒ 6x = x + 20
⇒ 5x = 20 y = 3z ...(2)
⇒ x = 4 1
z = x …(3)
4
110.
(a) Let the number be (765x + 42).
When this number is divided by 17, then quotient will 3
\ (2) ⇒ y = x …(4)
be (45x + 2) and remainder will be 8. 4
111.
(c) Production in third year = 6000 Using (3) and (4) in (2), we get
Production in seventh year = 7000 3 1
x+ x + x = 16
\ Production in fourth year =1000 4 4
⇒ x = 8
i.e., Production increases @ 250 scooters every year.
\ Production in tenth year \ y = 6, z = 2
= (7000 + 250 × 3) = 7750 Hence the number is 862.

112.
(c) Let the number of girls and boys be x and y. 1 1
119.
(d) = K K +3
Then x – 1 = y 3 4
and xy = 272 ⇒ K = 36
⇒ x(x – 1) = 272 120.
(b) All the options satisfy the first condition. So testing
⇒ x2 – x – 272 = 0 the options for second condition, only option (b) i.e.,
⇒ (x + 17)(x – 16) = 0 24 satisfies the second condition i.e., 24 + 18 = 42.
⇒ x = 16 29 15 116 + 45 161 17
121.
(d) + = = = 3
113.
(a) Let there were x friends, then contribution of one 12 16 48 48 48
96
friend = 1 2 n 1
x  ...
122.
(a) The given expression = 2 2 3 n 1 = (2) n  1
If four friends have left, then contribution of each
96 123.
(d) The number is neither divisible by 3 nor by 11.
friend =
x−4 124.
(d) p may or may not be divisible by 24. But, p is never
96 96 divisible by 22 because
\ − = 4 ⇒ x = 12
x−4 x 23n + 1 = (22 + 1)n + 1 = 22k + 2

Chapter_01.indd 26 1/30/2016 1:18:46 PM


Numbers  27

125.
(a) 4 a 3 So, p = x – y = 2(y – x)
984 The condition is satisfied only when x = y

13 b 7 Then, p = 0
As 13 b 7 is divisible by 11 135.
(a) We know that
\ a = 1 and b = 9 for any rational number,
\ a + b = 1 + 9 = 10 1
< 1
h t u x 1
126.
(b) Given − u t h x
and < 1
− − 4 x 1
The difference between a three-digit number and its Hence, (a) is the correct answer.
reverse is always a multiple of 99. The only multiple 136.
(a) A0 = 1994, which is not divisible by 10.
of 99 and less than 1000 that ends in 4 is 594. Thus, Hence, f (A0) = A0 + 1 = 1995. Since,
the remaining two digits in that order are 9 and 5.
Am+1 = f (Am) ⇒ A1 = f (A0) = 1995, similarly A2 =
127.
(a) In base 4, the 20th number will be
1996, A3 = 1997, A4 = 1998, A5 = 1999, A6 = 2000,
= 42 (1) + 41 (1) + 40 (0) = 110
2000
128.
(b) Let the two digit numbers = xy which is divisible by 10. Hence, f (A6) = = 200
10
The square of xy = (10x + y)2 = A7 similarly A8 = 20 and A9 = 2.
The square of the number formed by reversing the
digits of xy = (10y + x)2 137.
(d) Four digits of the number 29138576 are omitted so
that result is large.
(10x + y) – (10y + x)2 = 99 (x2 – y2)
2
\ Omitted digits are 1, 2, 3, 5.
= 99 (x – y) (x + y)
Hence, the largest omitted digit is 5.
Thus, it will always be divisible by 9, the sum of the
digits as well as the difference of the digits. But, it is 138.
(a) A < B
not divisible by their product xy. A < D < B, C is the greatest integer.
129.
(b) The value of the three digit number tu 1 \ With the help of 1st statement E + B < A + D, the

= 100t + 10u + 1 result can be obtained.
130.
(b) 7! = 7 × 6 × 5 × 4 × 3 × 2 × 1 (c) x2 + 4xy + 6y2 – 4y + 4
139.
= 24 × 32 × 51 × 71
= ( x) 2  2 2 x 2 y  (2 y ) 2  ( 2 y ) 2
Thus, the least perfect square which is divisible by 7!
should be (24 × 32 × 51 × 71)(51 × 71) i.e.,  2 2 y 2  ( 2) 2  2

5040 × 35 = 176400
= ( x  2 y ) 2  ( 2 y  2) 2  2
131.
(a) If the square of any natural number n leaves a
remainder of 1 when divided by 24, that natural Now, on putting the value of x = – 2 and y = 1, we get
number must be of the form 6p ± 1 (since n must the minimum value of expression.
be divisible by neither 2 nor 3) where p is a natural
140.
(a) Numbers can be 1112, 1113, 1115, 1117
number.
\ the two digit numbers must be of the form 6p ± 1, 141.
(a) The metre skips all the numbers in which there is a 5.
From 0000 to 0099, 5 occurs 10 times in the tens place
There are 15 two-digit numbers in the form 6p + 1
and 10 times in the units place, (which includes the
and the same number of two digit numbers in the form
number 55).
6p – 1.

\ It occurs in a total of 10 + 10 – 1 numbers i.e., 19
\ a total of 30 two-digit numbers satisfy the given
numbers. Similarly, from 0100 to 0199, from 0200
condition.
to 0299, 0300 to 0399 from 0400 to 0499, 0600 to
132.
(d) Factors of 6 are 1, 2, 3. Now 1 + 2 + 3 = 6. 0699, ... 0900 to 0999. It occurs in 8 (19) numbers.
133.
(c) By hit and trial, we see that n = 10 and n = 16 satisfy From 0500 to 0599, there are 100 numbers. The
the conditions. micromanometer reading could change from 0499 to
134.
(a) Given 2x + p = 2y, p + y = x 0600.
and x + y = z Total number of numbers skipped from 0000 to 0999
⇒ x + y + z = 2z = 2(x + y) = 19 (9) + 100 = 271

Chapter_01.indd 27 1/30/2016 1:18:46 PM


28  Chapter 1

Similarly, from 1000 to 1999 and from 2000 to 2999, ⇒ x = 96


271 + 271 numbers are skipped. Finally, 3005 and \ Largest number = x + 2 = 98 (Largest number)
3015 are also skipped.
147.
(d) Suppose the greater number is x. Then,
\ Total number of skips = 271 (3) + 2 = 815
x(x – 2) = 6888
\ Actual pressure = 3016 – 815 = 2201
⇒ x2 – 2x – 6888 = 0
142.
(d) I has 741 ones, II has 534 ones and III has 123 ones.
Sum of the digits of each of I, II, and III is divisible ⇒ x2 – 84x + 82x – 6888 = 0
by 3. ⇒ x(x – 84) + 82(x – 84) = 0

\ I, II, III are all divisible by 3 and hence not prime. ⇒ (x – 84) (x + 82) = 0
Choice (d) follows, since only this supports that all \ x = 84 or – 82
these three are not prime.
So, the greater number is 84.
143. (d) Let the two consecutive numbers = x and x + 2 then
sum of their square = x2 + (x + 2)2 = 6500 148. (b) Let the second number be 3x, so that the first number
2 2 is 6x and the third one is 2x.
⇒ x + x + 4x + 4 = 6500
\ 6x + 3x + 2x = 132
⇒ 2x2 + 4x – 6496 = 0
⇒ 11x = 132 or x = 12
⇒ x2 + 2x – 3248 = 0
Second number = 3x = 3 × 12 = 36
⇒ x2 + 58x – 56x – 3248 = 0
⇒ x(x + 58) – 56(x + 58) = 0 149. (b) Suppose the number is 10x + y
(x + 58)(x – 56) = 0 Q (10x + y) – (10y + x) = 63
\ x = 56 or –58 ⇒ 9x – 9y = 63

144. (d) Let the first number is x than five consecutive even ⇒ x – y = 7
numbers are x, x + 2, x + 4, x + 6, x + 8 and x + y = 11
According to question, Q x = 9
x + x + 2 + x + 4 + x + 6 + x + 8 = 220 y = 2
⇒ 5x + 20 = 220 So, required number = 92
⇒ 5x = 200 – 20 150. (a) 6 + 7 + 3 + 5 + K + 1 = 22 + K
200 The least number greater than 22 and divisible by 9
⇒ x = = 40
5 is 27
Again, suppose different set of five consecutive even \ 27 = 22 + K
number’s second lowest number = y + 2 which is 37
less than double of the lowest number of set A ⇒ K = 5
= 40 × 2 – 37 = 43 151. (d) Let the unit digit be y and tens digit be x.
\ First lowest number = 43 – 1 = 42 \ The number = 10x + y
and Sum = 42 + 43 + 44 + 45 + 46 On interchanging the digits the number = 10y + x
= 220 \ 10x + y – 10y – x = 27
145. (d) If we put consecutive negative integers as x = –1, y = ⇒ x – y = 3
–2 and z = –3, (already given in the question)
then from option (d), Now, y ≠ 0 and the set of digits satisfying the condition
(–1 + 2)(–2 + 3) = 1 × 1 are (9, 6), (8, 5), (7, 4), (6, 3), (5, 2), (4, 1).
= 1 (Positive integer) \ We can not reach on a distinct answer.
146.
(b) Suppose the numbers are x and x + 2 152. (b) 2n + 5m = 50
\ x(x + 2) = 9408 \ Possible value of n and m are
⇒ x2 = 2x – 9408 = 0 (25, 0), (10, 6), (20, 5), (15, 4), (5, 8)
⇒ x2 + 98x – 96x – 9408 = 0 Hence, least difference between 5 and 8 is 3.
⇒ x(x + 98) – 96(x + 98) = 0
153. (d) Divide 7231 by 45, the remainder is 31.
⇒ (x – 96) (x + 98) = 0
\ Required number = 45 – 31 = 14

Chapter_01.indd 28 1/30/2016 1:18:46 PM


Numbers  29


154. (b) Let a two-digit number = 48 \ Unit digit of (a + 2)2
When unit digit is halved = 4 ⇒ (3 + 2)2 = 52
Ten’s digit is doubled = 8 ⇒ = 25
\ Number = 84
i.e., 5
Hence, digit in the unit’s place is twice the digit in the 163.
(c) After dividing 10 by 7,
ten’s place.
we get remainder n – 4
155. (d) According to question
i.e., 7 – 4 = 3
=
16 × 4 64 =
64 × 1 64 = 8 × 8 64 = 7 × 5 35
164. (c) We see that 42 + 62 = 52 when divided by 25, remainder
(16 +=4) 20 (64 += 1) 65 (8 +=8) 16 (7 +=5) 12
is 2
156.
(d) ab > 0 because a and b both are positive. 43 + 63 = 280, divide by 25, remainder is 5
157.
(b) n must be divisible by 35. 44 + 64 = 1552, divide by 25, remainder is 2
158.
(d) Let the number be 253. When taking m odd, the remainder is 5.
Which unit place is 2 When taking m even, the remainder is 2.
\ Digit at 100 place of original number is 2. Hence, remainder = 5
159.
(a) x and y are natural numbers. 165. (d) Unit’s digit in 102 is 2.
We know that for any natural number p, The digit in the unit’s place of 10251 will be same as in
pn + p is even 251 or 23 = 8 [Q 51 = 4.12 + 3]
n
and p – p is even 166. (c) Divide 9269 by 73, the remainder is 71.
When, we multiply an even number to any natural \ 71 is the required least number.
number the resultent number is even.
167.
(a) Divide 15463 by 107, the remainder is 55, therefore,
160.
(d) (I) Product of 4 positive numbers cannot be zero. the number to be added = 107 – 55 = 52
(II) a can be odd or even b can be odd or even c is
even p is odd. We cannot definitely say that a + b 168.
(a) a + b < c + d ... (1)
+ c + p is odd. b + c < d + e ... (2)
2 2 2 2 2 2
(III) (b + c ) – (p – a ) here b + c can be odd or c + d < e + a ... (3)
even (p2 – a2) can be odd or even. d + e < a + b ... (4)
(IV) a (p – c) + a (c + b) = a [p – c + c + b] From (1) and (4),
Where a is divisible by 5 a + b + d + e < c + d + a + b
So, a (p – c) + a (c + b) will be divisible by 5 ⇒ e < c
So, only (IV) is correct. From (2) and (4),
161.
(b) x is a factor of y b + c + d + e < d + e + a + b
\ y = ax (Suppose) ⇒ c < a
x is a multiple of z From (1) and (3),
\ x = bz (Suppose) a + b + c + d < c + d + e + a
xy ⇒ b < e
(a) = by, it is an integer
z 169.
(b) zx + y = 10, zy + z = 20
y + z ax + z 2x + z = 30
(b) = , it is not an integer
x x ⇒ 2z + y × 2y + z × 2z + x = 10 × 20 × 30 = 6000
yz
(c) = az, it is an integer ⇒ 22 (x + y + z) = 6000
x
⇒ 22(y + z) = 400
x + y bz + abz
(d) = = (b + ab), it is an integer 6000
z z ⇒ 22 (y + y + z – y – z) = = 15
400
2
162.
(c) Given that unit digit of a = 9 ⇒ 22x = 5
and (a + 1)2 = 4
⇒ 2x = 15

i.e., unit digit of a must be 3

Chapter_01.indd 29 1/30/2016 1:18:47 PM


30  Chapter 1

170. (c) Dividing 5000 by 73, the remainder is 36. The number (d) 41 = 4; 42 = 16; 43 = 64; 44 = 256; 45 = 1024
179.
greater than 5000 is obtained by adding to 5000 the On dividing 372 by 4, the remainder = 0.
difference of divisor and the remainder.
On dividing 373 by 4, the remainder = 1.
\ The required number So, required unit digit
= 5000 + (73 – 36) = unit’s digit of the sum of 6 + 4 = 0
= 5037 180.
(a) Let the numbers be x and y.
171. (d) Let the numbers be a and b. \ (x)(x + y) = 247 ...(1)
Then, a + b = 100 and a – b = 37 and (y)(x + y) = 114 ...(2)
\ a2 – b2 = (a + b) (a – b) On adding (1) and (2), we get
= 100 × 37 = 3700 x2 + xy + xy + y2 = 361
172.
(d) Let the number be (10x + y), then ⇒ (x + y)2 = 361
10x + y + 10y + x = 11x + 11y ⇒ x + y = 19
= 11 (x + y) Hence, the sum of numbers is 19.
Hence, resultant number will be divisible by 11. 181.
(a) Let the number be x.
1 1
173.
(b) 50000 = 79 × quotient + 43759 ⇒ x − x = 100
7 11
\ 50000 – 43759 = 79 × quotient
11x − 7 x
or, 6241 = 79 × quotient ⇒ = 100
77
6241 ⇒ 4x = 7700
\ Required number of times = = 79
79 ⇒ x = 1925.
(a) 21 = 2; 22 = 4; 23 = 8; 24 = 16; 25 = 32 i.e., The digits
174.
at unit’s place repeats itself after power 4. 182.
(b) 6 + 6 + 6 + = ?
On dividing 33 by 4, we get 1 as remainder.
Let x = 6+ 6+ 6+
Therefore, digit at unit place in the product of 2 = 2
Hence, remainder on division by 10 = 2 ⇒ x = 6 + x
1 2 3 4 5 On squaring both the sides,
175.
(d) 7 = 7; 7 = 49; 7 = 343; 7 = 2401; 7 = 16809 i.e.,
The digit at unit place repeats itself after power 4. x2 = 6 + x
On dividing 343 by 4, we get 1 as remainder. 2
⇒ x – x – 6 = 0
Therefore, unit’s digit in the product of (4387)245 ⇒ x = 3, –2
= (621)72 = unit’s digits in the product of (4387)245 Since sum of positive integers cannot be negative,
× (621)+72 = units’s digits in the product of (4387)1 so ignore –2.
× (621)+2 = 7 × 1 = 7
183. (a) On dividing 58701 by 567
176.
(d) The sum of two odd numbers is even. The product of 1
two odd numbers is also even. Therefore, a + b + 2ab Remainder = 300 > (567)
2
= Even number.
\ Integer nearest to 58701 and divisible by 567
177.
(c) 216 – 1 = (28)2 – 1 = 58701 + (567 – 300)
= (28 + 1)(28 – 1)
= 58701+ 267 = 58968
= (256 + 1)(256 – 1)
184. (a) The least no. of six figures is 100000
= 257 × 255 = 65535
On dividing 100000 by 357, remainder = 40
which is exactly divisible by 17.
\ Least number of six figures which is divisible by
178.
(c) Let the numbers be x and y
357 = 100000 + (357 – 40)
Given, x + y = 24 = 100317
and xy = 143 \ Required number = 100317 – 5555 = 94762
So, x2 + y2 = (x + y)2 – 2xy
185. (a) Let the least value to be given to * be x
= (24)2 – 2 × 143 Then, x + 4 + 7 = 5 + 3 + 5
= 576 – 286 = 290 x = 2

Chapter_01.indd 30 1/30/2016 1:18:47 PM


2 H.C.F. and L.C.M.
of Number

COMMON FACTOR Illustration 3 Find the H.C.F. of 360 and 132.


A common factor of two or more numbers is a number Solution: 360 = 23 × 32 × 5
which divides each of them exactly. 132 = 22 × 31 × 11
For example, 4 is a common factor of 8 and 12. \ H.C.F. = 22 × 31 = 12.
HIGHEST COMMON FACTOR Illustration 4 If x = 23 × 35 × 59 and y = 25 × 37 × 511, fi nd
Highest common factor (H.F.C.) of two or more numbers is H.C.F. of x and y.
the greatest number that divides each one of them exactly. Solution: The factors common to both x and y are 23, 35
For example, 6 is the highest common factor of 12, 18 and and 59.
24. Highest common factor is also called Greatest Common
Divisor or Greatest Common Measure. \ H.C.F. = 23 × 35 × 59.
Symbolically, these can be written as H.C.F. or G.C.D. or II. Method of Division
G.C.M., respectively.
A. For two numbers:
METHODS OF FINDING H.C.F.
Step 1 Greater number is divided by the smaller one.
I. Method of Prime Factors Step 2 Divisor of (1) is divided by its remainder.
Step 1 Express each one of the given numbers as the
product of prime factors. Step 3 Divisor of (2) is divided by its remainder.
[A number is said to be a prime number if it is exactly This is continued until no remainder is left.
divisible by 1 and itself but not by any other number, H.C.F. is the divisor of last step.
e.g. 2, 3, 5, 7, etc. are prime numbers]
Step 2 Choose common factors.
Step 3 Find the product of these common factors. Illustration 5 Find the H.C.F. of 3556 and 3444.
This is the required H.C.F. of given numbers.
3444 ) 3556 ( 1
Illustration 1 Find the H.C.F. of 70 and 90. 3444
Solution: 70 = 2 × 5 × 7 112 ) 3444 ( 30
90 = 2 × 5 × 9 3360
Common factors are 2 and 5.
\ H.C.F. = 2 × 5 = 10. 84 ) 112 ( 1
84
Illustration 2 Find the H.C.F. of 3332, 3724 and 4508
Solution: 3332 = 2 × 2 × 7 × 7 × 17 28 ) 84 ( 3
3724 = 2 × 2 × 7 × 7 × 19 84
4508 = 2 × 2 × 7 × 7 × 23 ×
\ H.C.F. = 2 × 2 × 7 × 7 = 196. \ H.C.F. = 28.

Chapter_02.indd 31 1/30/2016 11:20:08 AM


32 Chapter 2

COMMON MULTIPLE
B. For more than two numbers:
A common multiple of two or more numbers is a number
Step 1 Any two numbers are chosen and their H.C.F.
which is exactly divisible by each one of them.
is obtained.
For example, 32 is a common multiple of 8 and 16.
Step 2 H.C.F. of H.C.F. (of (1)) and any other 18 × 4 = 32
number is obtained. 16 × 2 = 32.
Step 3 H.C.F. of H.C.F. (of (2)) and any other LEAST COMMON MULTIPLE
number (not chosen earlier) is obtained. The least common multiple of two or more given numbers
This process is continued until all numbers have been is the least or lowest number which is exactly divisible by
chosen. H.C.F. of last step is the required H.C.F. each of them.
For example, consider the two numbers 12 and 18.
Illustration 6 Find the H.C.F. of 13915, 9499 and 2553 by Multiples of 12 are 12, 24, 36, 48, 72, ...
division method. Multiples of 18 are 18, 36, 54, 72, ...
Solution: Common multiples are 36, 72, ...
\ Least common multiple, i.e. L.C.M. of
12 and 18 is 36.

METHODS OF FINDING L.C.F.

A. Method of Prime Factors


Step 1 Resolve each given number into prime
factors.
Step 2 Take out all factors with highest powers that
occur in given numbers.
Step 3 Find the product of these factors. This
product will be the L.C.M.

Illustration 8 Find the L.C.M. of 32, 48, 60 and 320.


Solution: 32 = 25 × 1
48 = 24 × 3
Now, in the next step, we will fi nd the H.C.F. of 23 and 60 = 22 × 3 × 5
2553. 320 = 26 × 6
\ L.C.M. = 26 × 3 × 5 = 960.

B. Method of Division
Step 1 The given numbers are written in a line
separated by common.
Step 2 Divide by any one of the prime numbers 2,
3, 5, 7, 11, ... which will divide at least any two of
Thus, H.C.F. of 13915, 9499 and 2553 = 23. the given numbers exactly. The quotients and the
Illustration 7 Find the greatest possible length which can undivided numbers are written in a line below the fi rst.
be used to measure exactly the lengths 7 m, 3 m 85 cm, Step 3 Step 2 is repeated until a line of numbers
12 m 95 cm. (prime to each other) appears.
Solution: Required length Step 4 Find the product of all divisors and numbers
in the last line which is the required L.C.M.
= (H.C.F. of 700, 385, 1295) cm = 35 cm.

Chapter_02.indd 32 1/30/2016 11:20:09 AM


H.C.F. and L.C.M. of Number 33

Illustration 9 Find the L.C.M. of 12, 15, 20 and 54. L.C.M. = 2 × 2 × 3 × 5 × 1 × 1 × 1 × 9 = 540.
Solution: 2 12, 15, 20, 54 Note: Before finding the L.C.M. or H.C.F., we must ensure
2 6, 15, 10, 27 that all quantities are expressed in the same unit.
3 3, 15, 5, 27
5 1, 5, 5, 9
1, 1, 1, 9

SOME USEFUL SHORT-CUT METHODS

∴ H.C.F. of 616 and 1300 is 4.


1. H.C.F. and L.C.M. of Decimals
Thus, the required H.C.F. = 0.04.
Step 1 Make the same number of decimal places in all
the given numbers by suffixing zero(s) if necessary. 2. L.C.M. and H.C.F. of Fractions
Step 2 Find the H.C.F./L.C.M. of these numbers L.C.M. of the numbers in numerators
without decimal. L.C.M =
H.C.F. of the numbers in denominators
Step 3 Put the decimal point (in the H.C.F./L.C.M.
of step 2) leaving as many digits on its right as there H.C.F. of the numbers in numerators
H.C.F. =
are in each of the numbers. L.C.M. of the numbers in denominators

Illustration 10 Find the L.C.M. of 1.2, 0.24 and 6. 2 3 6


Illustration 12 Find the L.C.M. of , and .
Solution: The given numbers can be written as 5 10 25
Solution: L.C.M. of numerators 2, 3 and 6 is 6.
1.20, 0.24 and 6.00.
H.C.F. of denominators 5, 10 and 25 is 5.
Now, ignoring the decimal we find the L.C.M. of
L.C.M. of numerators
120, 24 and 600. ∴ Required L.C.M. =
H.C.F. of denominators
2 120, 24, 600
6
2 60, 12, 300 = .
5
2 30, 6, 150
4 10 20
3 15, 3, 75 Illustration 13 Find the H.C.F. of , and .
9 21 63
5 5, 1, 25
Solution: H.C.F. of numerators 4, 10 and 20 is 2.
1, 1, 5 L.C.M. of denominators 9, 21 and 63 is 63.
∴ L.C.M. = 2 × 2 × 2 × 3 × 5 × 1 × 5 = 600 H.C.F. of numerators
∴ Required H.C.F. =
Thus, the required L.C.M. = 6.00, i.e., 6. L.C.M. of denominators
Illustration 11 Find the H.C.F. of 6×16 and 13. 2
= .
Solution: The given numbers can be written as 63
Notes:
6.16 and 13.00.
Now, ignoring the decimals we find the H.C.F. of 616 1. If the given set of numbers includes fractions as well
and 1300. as whole numbers, treat whole number too as fraction
with 1 in its denominator.
2. The H.C.F. of a number of fractions is always a fraction,
but the L.C.M. may be a fraction or an integer.

3. Product of two numbers


= L.C.M. of the numbers × H.C.F. of the numbers

Chapter_02.indd 33 1/30/2016 2:46:03 PM


34 Chapter 2

Illustration 14 The H.C.F. and the L.C.M. of any two


8. To fi nd the least number which when divided by
numbers are 63 and 1260, respectively. If one of the two
x, y and z leaves the same remainder r in each case.
numbers is 315, fi nd the other number.
Required number = (L.C.M. of x, y and z) + r.
Solution: The required number
L.C.M.  H.C.F. 1260  63 Illustration 19 Find the least number which when divided
= = = 252. by 12, 16 and 18, will leave in each case a remainder 5.
First number 315
Solution: The required smallest number
4. To fi nd the greatest number that will exactly
= (L.C.M. of 12, 16 and 18) + 5
divide x, y and z.
Required number = H.C.F. of x, y and z. = 144 + 5 = 149.

Illustration 15 Find the greatest number that will exactly 9. To fi nd the greatest number that will divide x, y
divide 200 and 320. and z leaving the same remainder in each case.
Solution: The required greatest number (a) When the value of remainder r is given:
= H.C.F. of 200 and 320 = 40. Required number = H.C.F. of (x – r), (y – r)
and (z – r).
5. To fi nd the greatest number that will divide x, y
and z leaving remainders a, b and c, respectively. (b) When the value of remainder is not given:
Required number = H.C.F. of | (x – y) |, | (y –
Required number = H.C.F. of (x – a), (y – b) and (z – c).
z) | and | (z – x) |
Illustration 16 Find the greatest number that will
divide 148, 246 and 623 leaving remainders 4, 6 and 11, Illustration 20 Find the greatest number which will divide
respectively. 772 and 2778 so as to leave the remainder 5 in each case.
Solution: The required greatest number
Solution: The required greatest number
= H.C.F. of (772 – 5) and (2778 – 5)
= H.C.F. of (148 – 4), (246 – 6) and (623 – 11),
= H.C.F. of 767 and 2773
i.e., H.C.F. of 144, 240 and 612 = 12.
= 59.
6. To fi nd the least number which is exactly Illustration 21 Find the greatest number which on dividing
divisible by x, y and z. 152, 277 and 427 leaves equal remainder.
Required number = L.C.M. of x, y and z. Solution: The required greatest number
Illustration 17 What is the smallest number which is = H.C.F. of |(x – y)|, |(y – z)| and |(z – x)|
exactly divisible by 36, 45, 63 and 80? = H.C.F. of |(152 – 277)|, |(277 – 427)|
Solution: The required smallest number and |(427 – 152)|
= L.C.M. of 36, 45, 63 and 80 = H.C.F. of 125, 275 and 150
= 5040. = 25.
7. To fi nd the least number which when divided 10. To fi nd the n-digit greatest number which, when
by x, y and z leaves the remainders a, b and c, divided by x, y and z,
respectively. It is always observed that (x – a) =
(a) leaves no remainder (i.e., exactly divisible)
(y – b) = (z – c) = k (say)
Step 1 L.C.M. of x, y and z = L
\ Required number = (L.C.M. of x, y and z) – k.
Step 2 L ) n-digit greatest number (
Illustration 18 Find the least number which when divided Remainder = R
by 36, 48 and 64 leaves the remainders 25, 37 and 53,
Step 3 Required number
respectively.
= n-digit greatest number – R
Solution: Since (36 – 25) = (48 – 37) = (64 – 53) = 11, (b) leaves remainder K in each case
therefore, the required smallest number Required number
= (L.C.M. of 36, 48 and 64) – 11
= (n-digit greatest number – R) + K.
= 576 – 11 = 565.

Chapter_02.indd 34 1/30/2016 11:20:11 AM


H.C.F. and L.C.M. of Number 35

Illustration 22 Find the greatest number of 4 digits which,


when divided by 12, 18, 21 and 28, leaves 3 as a remainder (b) leaves remainder K in each case.
in each case. Required number
= n-digit smallest number + (L – R) + k.
Solution: L.C.M. of 12, 18, 21 and 28 = 252.
252 ) 9999 ( 39 Illustration 24 Find the least number of four digits which
9828 is divisible by 4, 6, 8 and 10.
171 Solution: L.C.M. of 4, 6, 8 and 10 = 120.
\ The required number = (9999 – 171) + 3 = 9931.
120 ) 1000 ( 8
Illustration 23 Find the greatest number of 4 digits which, 960
when divided by 12, 15, 20 and 35 leaves no remainder.
Solution: L.C.M. of 12, 15, 20 and 35 = 420. 40

420 ) 9999 ( 23 \ The required number = 1000 + (120 – 40) = 1080.


9660 Illustration 25 Find the smallest 4-digit number, such that
339 when divided by 12, 18, 21 and 28, it leaves remainder 3 in
\ The required number = 9999 – 339 = 9663. each case.
Solution: L.C.M. of 12, 18, 21 and 28 = 252.
11. To fi nd the n-digit smallest number which when
divided by x, y and z 252 ) 1000 ( 3
(a) leaves no remainder (i.e. exactly divisible) 756
Step 1 L.C.M. of x, y and z = L
244
Step 2 L ) n-digit smallest number (
Remainder = R \ The required number
Step 3 Required number = 1000 + (252 – 244) + 3
= n-digit smallest number + (L – R).  

MULTIPLE CHOICE QUESTIONS

1. The L.C.M. of two numbers is 4800 and their HCF is 160. 4. A simple mechanical device has four gears A, B, C and D
If one of the numbers is 480, then the second number is such that they mesh in the order A—B—C—D. The number
(a) 16 (b) 16000 of teeth in these gears are 10, 21, 12, 17, respectively.
If we start the mechanism from rest, how may revolutions
(c) 160 (d) 1600
will the largest gear have to turn before one full cycle is
[Based on MAT, 2004] completed and the gears are in the position from where
they started?
2. What is the least number which when divided by 12, 18,
36, and 45 leaves remainders 8, 14, 32 and 41, respectively? (a) 270 (b) 340
(a) 176 (b) 88 (c) 7140 (d) 285
(c) 98 (d) 42 5. A stamp collector likes to arrange and rearrange his
collection in a symmetrical order. Sometimes he lines
3. An electronic device makes a beep after every 60 s.
his stamps up in pairs, sometimes in threes, sometimes in
Another device makes a beep after every 62 s. They
fours, occasionally in fi ves, and once in a while in groups
beeped together at 10 a.m. The time when they will next
of six. After arranging them in any of these groups, he
make a beep together at the earliest is
invariably has one stamp left over. When he arranges
(a) 10.30 a.m. (b) 10.31 a.m. his stamps in groups of seven, he fi nds that there is not a
(c) 10.59 a.m. (d) 11 a.m. single stamp to spare. How many stamps will be left over
[Based on MAT, 2005] if he arranges them in groups of 8?

Chapter_02.indd 35 1/30/2016 11:20:11 AM


36  Chapter 2

(a) 5 (b) 7 (a) 180 (b) 360


(c) 1 (d) 0 (c) 540 (d) 1260
6.
The H.C.F. of two numbers is 8. Which one of the [Based on SI of Police Rec. Exam, 1997]
following can never be their L.C.M.? 16. The sum of two numbers is 528 and their H.C.F. is 3. The
(a) 24 (b) 48 number of such pairs is
(c) 56 (d) 60 (a) 2 (b) 3
[Based on SSC (GL) Prel. Exam, 2000] (c) 4 (d) 5
[Based on SI of Police Rec. Exam, 1997]
7.
Find the number of four-digit numbers that are divisible
by 30 and 35 but not by 140. 17. A rectangular floor in my office has its area equal to 56
m2. The minimum number of tiles required, if all the tiles
(a) 21 (b) 22 are in square shape is
(c) 43 (d) 44 (a) 15 (b) 9
7 33 28 15 (c) 14 (d) cannot be determined
8.
Find the L.C.M. of , , and .
14 21 3 63 18.
H.C.F. and L.C.M. of two numbers are 11 and 385
(a) 220 (b) 4260 respectively. If one number lies between 75 and 125, then
(c) 356 (d) none of these that number is
(a) 77 (b) 88
9.
The L.C.M. of two numbers is 1820 and their H.C.F. is 26.
If one number is 130 then the other number is (c) 99 (d) 110
(a) 70 (b) 1690 [Based on SI Rec. COP Exam, 1998]

(c) 364 (d) 1264 19.


The sum of two numbers is 2000 and their L.C.M. is
21879. The two numbers are
[Based on SSC (GL) Prel. Exam, 2002]
(a) 1993, 7 (b) 1991, 9
10.
H.C.F. and L.C.M. of two numbers are 7 and 140,
(c) 1989, 11 (d) 1987, 13
respectively. If the numbers are between 20 and 45, the
[Based on Assistant’s Grade Exam, 1998]
sum of the numbers is
(a) 70 (b) 77 20.
L.C.M. of two numbers is 495 and their H.C.F. is 5. If the
sum of two numbers is 100 then their difference is
(c) 63 (d) 56
(a) 10 (b) 46
[Based on SSC (GL) Prel. Exam, 2003]
(c) 70 (d) 90
11. Find the least common multiple of 6804 and 9828 given
their highest common factor is 756. [Based on SSC (GL) Prel. Exam, 1999]

(a) 84852 (b) 88452 21. H.C.F. and L.C.M. of 24, 82, 162, 203 are
(c) 85482 (d) 88542 (a) 23; 32000 (b) 24; 32000
4
12. Three rings complete 60, 36 and 24 revolutions in a minute. (c) 2 ; 25600 (d) 22; 3200
They start from a certain point in their circumference 22.
L.C.M. of 35,85 and a number k is 7,735. The H.C.F. is 5.
downwards. By what time they come together again in the What is the least possible value of k?
same position? (a) 65 (b) 25
(a) 5 s (b) 6 s (c) 325 (d) 13
(c) 8 s (d) 1 s
23.
HCF and LCM of two numbers are 21 and 4641,
13. H.C.F. of two numbers is 43 and their sum is 430. Total respectively. If one of the numbers lies between 200 and
number of distinct pairs of two such numbers is 300, then the two numbers are
(a) 5 (b) 2 (a) 273, 357 (b) 273, 361
(c) 6 (d) data insufficient (c) 273, 359 (d) 273, 363
14. Find the greatest possible length of the planks, if three [Based on MAT (May), 2006]
pieces of timber 42 m, 49 m and 63 m long have to be 24.
LCM and HCF of two numbers are 84 and 21, respectively.
divided into planks of the same length? If the ratio of the two numbers is 1:4, then the larger of the
(a) 8 m (b) 49 m two numbers is
(c) 7 m (d) 63 m (a) 12 (b) 48
15. H.C.F. of 4 × 27 × 3125, 8 × 9 × 25 × 7 and 16 × 81 × 5 × (c) 84 (d) 108
11 × 49 is [Based on MAT, 1997]

Chapter_02.indd 36 1/30/2016 11:20:11 AM


H.C.F. and L.C.M. of Number  37

25.
About the number of pairs which have 16 as their HCF 33.
The smallest positive number x, which leaves a remainder
and 136 as their LCM, we can definitely say that 1 when divided by 2, 3, 4 and 5, is
(a) only one such pair exists (a) greater than 75 (b) divisible by 7
(b) only two such pairs exist (c) a prime number (d) none of these
(c) many such pairs exist [Based on IIFT, 2005]
(d) no such pair exists
[Based on MAT, 2000] 34. Five bells first begin to toll together and then at intervals
of 3 s, 5 s, 7 s, 8 s and 10 s. Find after what interval they
26.
L.C.M. of first 100 natural numbers is N. What is the
will again toll together. How many times do they toll
L.C.M. of first 105 natural numbers ?
together in one hour?
(a) 5! × N (b) 10403N
(a) 14 min, 3 times (b) 12 min, 4 times
(c) 105N/103 (d) 4N
(c) 14 min, 4 times (d) 12 min, 3 times
27.
The traffic lights at three different road crossings change
after every 48 s, 72 s and 108 s, respectively. If they all 35.
A chocolate dealer has to send chocolates of three brands
change simultaneously at 8:20:00 h, then they will again to a shopkeeper. All the brands are packed in boxes of
change simultaneously at same size. The number of boxes to be sent is 96 of brand
(a) 8:27:12 hr (b) 8:27:24 hr A, 240 of brand B and 336 of brand C. These boxes are
to be packed in cartons of same size containing equal
(c) 8:27:36 hr (d) 8:27:48 hr
number of boxes. Each-carton should contain boxes of
[Based on MAT, 2000] same brand of chocolates. What could be the minimum
28.
The smallest perfect square number which is divisible by number of cartons that the dealer has to send?
8, 12 and 15 is
(a) 20 (b) 48
(a) 2,500 (b) 3,600
(c) 42 (d) 38
(c) 3,721 (d) 6,400
[Based on MAT, 2000]  [Based on XAT, 2010]

29.
Six bells commence tolling together and toll at intervals 36.
The greatest common divisor of 123456789 and
of 2 s, 4 s, 6 s, 8 s, 10 s and 12 s, respectively. In 30 min, 987654321 is
how many times do they toll together?
(a) 1 (b) 3
(a) 4 (b) 10
(c) 9 (d) greater than 9
(c) 15 (d) 16
[Based on JMET, 2011]
[Based on FMS (MS), 2006]
30.
21 mango trees, 42 apple trees and 56 orange trees have to 37. What is the least number of cut pieces of equal length that
be planted in rows such that each row contains the same can be cut out of two lengths 10 m 857 mm and 15 m
number of trees of one variety only. Minimum number of 87 mm?
rows in which the trees may be planted is (a) 174 (b) 172
(a) 20 (b) 17
(c) 164 (d) 184
(c) 15 (d) 3
[Based on FMS, 2005] 38. Philip, Tom and Brad start jogging around a circular
31.
Five bells begin to toll together and toll respectively at field and complete a single round in 18 s, 22 s and 30 s,
intervals of 6 s, 7 s, 8 s, 9 s and 12 s. How many times respectively. In how much time, will they meet again at
they will toll together in one hour, excluding the one at the the starting point?
start? (a) 3 min 15 s (b) 21 min
(a) 3 (b) 5
(c) 16 min 30 s (d) 12 min
(c) 7 (d) 9
[Based on FMS, 2005] [Based on Indian Bank PO, 2011]

32.
A number which when divided by 10 leaves a remainder of 39. L.C.M. of two numbers is 12 times their H.C.F. The sum
9, when divided by 9 leaves a remainder of 8, by 8 leaves of H.C.F. and L.C.M. is 403. If one number is 93, find the
a remainder of 7, etc., down to where, when divided by 2, other.
it leaves a remainder of 1, is
(a) 134 (b) 124
(a) 59 (b) 419
(c) 128 (d) None of the above
(c) 1259 (d) 2519
[Based on FMS, 2011] [Based on NMAT, 2006]

Chapter_02.indd 37 1/30/2016 11:20:11 AM


38  Chapter 2

40. Let x denote the greatest 4-digit number which when (a) 48 (b) 36
divided by 6, 7, 8, 9 and 10 leaves a remainder of 4, 5, (c) 24 (d) 16
6, 7 and 8, respectively. Then, the sum of the four-digits
[Based on SSC (GL), 2010]
of x is
(a) 25 (b) 18 47.
Two numbers are in the ratio 3:4. Their L.C.M. is 84. The
greater number is
(c) 20 (d) 22
(a) 21 (b) 24
[Based on JMET, 2009]
(c) 28 (d) 84
41. Amit, Sucheta and Neeti start running around a circular [Based on SSC (GL), 2010]
track and complete one round in 18 s, 24 s and 32 s,
respectively. In how many seconds will the three meet 48. Find the greatest number of five digits which when
again at the starting point if they all have started running divided by 8, 9 and 10 leaves 3 as remainder in each case.
at the same time? (a) 99996 (b) 99723
(a) 196 (b) 288 (c) 99983 (d) None of these
(c) 324 (d) Cannot be determined 49.
H.C.F. and L.C.M. of two numbers are 12 and 924,
 [Based on Bank of India PO, 2010] respectively. Then the number of such pairs is
42. Three friends A, B and C start running around a circular (a) 0 (b) 1
stadium and complete a single round in 24 s, 36 s and (c) 2 (d) 3
30 s, respectively. After how many minutes will they meet [Based on SSC (GL), 2011]
again at the starting point?
50.
What is the least number which, when divided by 5, 6, 7,
(a) 12 (b) 6 8 gives the remainder 3 but is divisible by 9?
(c) 8 (d) 15 (a) 1463 (b) 1573
 [Based on IDBI PO, 2009] (c) 1683 (d) 1793
43. Seema, Meena and Reena start jogging around a circular [Based on SSC (GL), 2011]
stadium and complete one round in 54 s, 42 s and 63 s, 51.
L.C.M. of two numbers is 120 and their H.C.F. is 10.
respectively. Approximately after how many minutes they Which of the following can be the sum of those two
will meet again at the starting point? numbers?
(a) 8 min (b) 10 min (a) 140 (b) 80
(c) 3 min (d) 6 min (c) 60 (d) 70
[Based on Syndicate Bank PO, 2010] [Based on SSC (GL), 2011]

44. L.C.M. and H.C.F. of two numbers x and y are 3 and 52. A heap of stones can be made up into groups of 21. When
1 1 made up into groups of 16, 20, 25 and 45, there are 3
105, respectively. If x + y = 36, the value of + is stones left in each case. How many stones at least can
x y
there be in the heap?
1
(a) 1 (b) (a) 7203 (b) 2403
6
(c) 3603 (d) 4803
12 4
(c) (d) 53.
The traffic lights at three different road crossings change
315 35
after 24 s, 36 s and 54 s, respectively. If they, all change
[Based on U.P. P.C.S., 2012] simultaneously at 10:15:00 a.m., then at what time will
45.
The greatest number, which when subtracted from 5834, they again change simultaneously?
gives a number exactly divisible by each of 20, 28, 32 and (a) 10:16:54 a.m. (b) 10:18:36 a.m.
35, is (c) 10:17:02 a.m. (d) 10:22:12 a.m.
(a) 1120 (b) 4714 [Based on SSC (GL), 2011]
(c) 5200 (d) 5600 54.
Find the greatest number of four digits which must be
[Based on SSC (GL), 2010] added to 5231 so that the final number becomes exactly
46.
H.C.F. and L.C.M. of two numbers are 8 and 48, divisible by 12, 15, 27, 32 and 40.
respectively. If one of the numbers is 24, then the other (a) 7929 (b) 7829
number is (c) 9729 (d) 7729

Chapter_02.indd 38 1/30/2016 11:20:12 AM


H.C.F. and L.C.M. of Number  39

55. Find the greatest number of four digits which is exactly 56. Find the least number which when decreased by 11 is
divisible by 24, 28, 30 and 35. divisible by 14, 15, 21, 32 and 60.
(a) 9225 (b) 9240 (a) 4371 (b) 3271
(c) 9250 (d) 9260 (c) 3371 (d) 3360

EXPLANATORY ANSWERS

HCF  LCM are 9660  1260  1 


1.
(d) Second number = 21
First number 420

4800  160 \ The number of multiples of 210 which are not


= = 1600. divisible by 420 are 43 – 21 i.e., 22.
480
 7 23 28 15 
2. (a) L.C.M. of 12, 18, 36 and 45 8. (d) L.C.M. of  , , , 
 14 21 3 63 
⇒ L.C.M. = 5 × 3 × 3 × 2 × 2 = 180
L.C.M. of (7, 23, 28, 15)
⇒ Required answer = 180 – 4 = 176 =
H.C.F. of (14, 21, 3, 63)
Note: Here 12 – 8 = 18 – 14
15  28  11
= 36 – 32 = 45 – 41 = 4 = = 4620
1
3.
(b) L.C.M. of 60 s and 62 s is 1860 s = 31 min. 1820  26
9.
(c) Required number = = 364.
\ They will beep together at 10.31 a.m. 130
4. (b) Let n be the minimum number of teeth advanced 10.
(c) Since the H.C.F. is 7 therefore, possible numbers
before all the wheels are in starting position again. could be 21, 28, 35 and 42. L.C.M. of the numbers
Hence, n must be every divisible by 10, 21 12 and 17. does not have the factor of 3. But 21 and 42 are the
numbers which have 3 as a factor. So, 21 and 42 will
Hence n = L.C.M. = 7140. not be the numbers. If 21 and 42 are not the numbers,
So, the required number of revolutions in largest gear then the numbers are 28 and 35.
= 7140/21 = 340 \ Sum of the numbers = 28 + 35 = 63.
5. (a) Number of tickets with the collector 11. (b) We use the fact that, for any two numbers, a and b,
= L.C.M. of (3, 4, 5, 6) + 1, i.e., 60 + 1 = 61 a × b = H.C.F. × L.C.M.
or one more than the multiple of 60 i.e., 61, 121, 181, 6804  9828
⇒ L.C.M. =
241, 301 etc. 756
Since it must be exactly divisible by 7, so the collector = 9 × 9828 = 88452
has 301 stamps and when stamps are arranged in
groups of 8, he will have 5 stamps left over. 60
12. (a) Time taken by each ring in one revolution are s,
60
6.
(d) Non-multiples of 8 are not the L.C.M. 60 60
s and s respectively.
7. (b) If a number is divisible by 30 and 35, it is divisible by 36 24
L.C.M. (30, 35) i.e., 210. 5 5
i.e., 1, and s
If a number is divisible by 210 but not by 140, then it 3 2
is not divisible by L.C.M. (210, 140) i.e., 420. So, the 5 5
Requiring time = L.C.M. of 1, , = 5 s
number is divisible by 210 but not by 420. The least 3 2
and the greatest four digit multiples of 210 are 1050
13. (b) 43k + 43l = 430
and 9870, respectively.
⇒ 43(k + l) = 430
\ The number of multiples of four-digit multiples of
9870 – 1050 8820 ⇒ k + l = 10
210 are + 1= + 1 = 43
210 210 But k, l must be co-primes.
Similarly, the number of four-digit multiples of 420 So (k, l) = (1, 9) and (3, 7)

Chapter_02.indd 39 1/30/2016 11:20:12 AM


40  Chapter 2

14. (c) Required length = H.C.F. of (42, 49, 63) = 7 m 23.


(a) Product of the numbers = H.C.F. × L.C.M. = 21 × 4641
15.
(c) 4 × 27 × 3125 = 21 × 3 × 7 × 13 × 17 = 3 × 7 × 3 × 7 × 13 × 17
= 2 × 2 × 3 × 3 × 3 × 5 × 5 × 5 × 5 × 5 × 5 \ The numbers can be 3 × 7 × 13 and 3 × 7 × 17, i.e.,
8 × 9 × 25 × 7 273 and 357.
= 2 × 2 × 2 × 3 × 3 × 5 × 5 × 7 24.
(c) Let the numbers be x and y
16 × 81 × 5 × 11 × 49 x 1
\ xy = 84 × 21 and = ⇒ 4x = y
= 2 × 2 × 2 × 2 × 3 × 3 × 3 × 3 × 5 × 7 × 7 × 11 y 4
\ H.C.F. = 2 × 2 × 3 × 3 × 3 × 5 = 540. Now, putting the value of y in the above equa­tion, we
have
16.
(c) (33, 495); (99, 429); (165, 363); (231, 297)
x × 4x = 84 × 21
17. (b) 56 = 1 × 56
\ x = 21 and y = 21 × 4 = 84
⇒ H.C.F. is 1, hence number of tiles = 56
Hence, the larger number = 84
2 × 28 ⇒ H.C.F. is 2, hence number of tiles
25. (d) Using the relation,
= 1 + 14 = 15
4 × 14 ⇒ H.C.F. is 2, therefore number of tiles First No. × Second No. = H.C.F. × L.C.M.
= 2 + 7 = 9 = 16 × 136 = 16 × (8 × 17)
7 × 8 ⇒ H.C.F. is 1, therefore number of tiles It is clear from here that no such pair is pos­sible.
= 7 + 8 = 15 26.
(b) If we look at the numbers 100 < N ≤ 105, we see only
Hence, the minimum number of tiles can be 9 and 101 and 103 do not have their factors in N (because
the dimension of a tile is 2 m each side. Thus, (b) is these are primes). So, obviously the new L.C.M. will
be 101 × 103 × N.
correct option.
27.
(a) Let us first calculate L.C.M. of 48, 72, 108
18.
(a) Let the numbers be x and y
\  xy = 11 × 385 = 11 × 5 × 7 × 11 = 77 × 55. 2 48, 72, 108
19.
(c) From the given choices we can see that L.C.M. of 2 24, 36, 54
1989 and 11 is 21879. 2 12, 18, 27

Therefore, required numbers are 1989, 11. 3 6, 9, 27
20.
(a) Let the two numbers be x and (100 – x) 3 2, 3, 9
L.C.M. × H.C.F. = Product of the numbers 2, 1, 3
495 × 5 = x(100 – x) \ L.C.M. of 48 s, 72 s, 108 s
or, 2
x – 100x + 2475 = 0 = 2 × 2 × 2 × 3 ×3 × 2 × 3 = 432 s
or, x2 – 55x – 45x + 2475 = 0 Thus, the second time the three lights will change
or, (x – 55)(x – 45) = 0 after 432 s = 7 min 12 s
or x = 45 or x = 55 Hence, next time the three lights will change
Thus, the numbers are 45 and 55. simultaneously at
When x = 55, we get 100 – x = 45 and vice versa. 8:20:00 + 0:7:12 = 8:27:12 hrs.
Hence, their difference = 55 – 45 = 10. 28.
(b) Let us first calculate L.C.M. of 8, 12, 15
21. (b) H.C.F. of (24, 82, 162, 203) = 24 \ L.C.M. of 8, 12, 15 = 2 × 2 × 3 × 2 × 5 = 120
L.C.M. of (24, 82, 162, 203) = 28 × 125 = 32000 Hence, the smallest perfect square number di­visible
22. (a) Prime factors of 35 = 5 × 7 by 8, 12, 15 i.e. by 120 is 3600.
85 = 5 × 17 29.
(c) L.C.M. of 2, 4, 6, 8, 10, 12 = 120
L.C.M. = 7735 = 5 × 7 × 17 × 13 30  60
Total no. of times = = 15
Since H.C.F. = 5, Let k = 5a 120
\ 35 × 85 × 5a = 5 × 7 × 17 × 13
30.
(b) H.C.F. of 21, 42, 56 = 7
⇒ a = 13
119
\ least possible value of k = 5 × 13 = 65 \ Number of rows = = 17
7

Chapter_02.indd 40 1/30/2016 11:20:12 AM


H.C.F. and L.C.M. of Number  41

31.
(c) L.C.M. of 6, 7, 8, 9,12 = 504 s 40. (a) The number will be a multiple of 6, 7, 8, 9, 10.
1 h = 1 × 60 × 60 s = 3600. L.C.M. of 6, 7, 8, 9, 10 = 2520
3600 \ Largest 4-digit number divided by this = 7560
\ Number of times = ≈7
504 \ Required number = 7558

32.
(d) Every time the difference between divisor and Sum of the digits of this number = 25
remainder is 1. So, the number is of the form L.C.M. 41. (b) Time taken,
(10, 9, 8, 7, 6, 5, 4, 3, 2) K – 1
2 18, 24, 32
= 2520 K – 1 2 9, 12, 16
when K = 1, 2 9, 6, 8
2520 – 1 = 2519 3 9, 3, 4
33.
(c) L.C.M. of 2, 3, 4, 5 = 60 3, 1, 4
\ Smallest number will be 60 + 1 = 61 2 × 2 × 2 × 3 × 3 × 4 = 288 s
and 61 is a prime number. 42. (b) 24 = 2 × 2 × 2× 3
34.
(c) Required time interval 36 = 2 × 2 × 3 × 3
= L.C.M. of (3, 5, 7, 8 and 10) 30 = 2 × 3 × 5
= 840 seconds = 14 minutes L.C.M. = 2 × 2 × 2 × 3 × 3 × 5 = 360
Number of times they will toll together in one hour Hence, all three friends will meet again after 360 s.
60
= = 4 times (ignoring the fraction part) 360
14 i.e., = 6 min
60
35.
(b) For minimum number of cartoons, there should be 43. (d) L.C.M. of 54, 42, 63 = 378 s
maximum number of chocolates in a cartoon that is 378
H.C.F. of 96, 240 and 336, which is 48. ≈ 6.3 min ≈ 6 min
60
36.
(c) We notice that 9 divides both numbers. We can use
44.
(d) Multiple of two numbers
Euler’s algorithm to obtain the GCD.
= Multiple of L.C.M. and H.C.F. of that numbers
123456789 ) 987654321 ( 8
     \ xy = 3 × 105
987456321 1 1 x+ y 36
+ = =
9 x y xy 3 × 105

12 4
 A = =
If x divides A and x divides B, x divides the Rem   105 35
B
where (A > B) 45.
(b) Number divisible by 20, 28, 32 and 35 is L.C.M. of
these numbers
\ The GCD is 9.
20 = 4 × 5
37. (d) H.C.F. of 10857 and 15087 is 141.
28 = 4 × 7
The least number of cut pieces
32 = 5 × 4 × 2
= (10857 + 15087) ÷ 141 = 184
35 = 4 × 7
38. (c) The L.C.M. of 18, 22, 30 is 990. L.C.M. = 4 × 5 × 7 × 8
So, they will meet each other after 990, i.e., 16 min = 1120
and 30 s.
Required number = 5834 – 1120 = 4714
39. (b) Let H.C.F. be x, then, L.C.M. = 12x 46.
(d) Product of the numbers
\ 12x + x = 403 ⇒ x = 31 = H.C.F. × L.C.M.
\ L.C.M. = 12 × 31 = 372 8 × 48
⇒ Second number = = 16
Other number = 31 × 272 ÷ 93 = 124 24

Chapter_02.indd 41 1/30/2016 11:20:13 AM


42  Chapter 2

47.
(c) The numbers are 21 and 28. Hence, sum of the numbers
48.
(b) L.C.M. of 8, 9, 10 = 360 = 30 + 40 = 70
52.
(a) L.C.M. of 16, 20, 25, 45 = 3600
1st number = 3600 × 1 + 3 = 3603 which is not
divisible by 21.
2nd number = 3600 × 2 + 3 = 7203 which is divisible
by 21.
53.
(b) L.C.M. of 24 s, 36 s and 54 s = 216 s = 3 min 36 s
Greatest number of 5 digits which is divisible by 360 Required time = 10:18:36 a.m.
= 99999 – 279 = 99720
54.
(d) L.C.M. of 12, 15, 27, 32, 40 = 4320. Let us add 5231
\ Required number = 99729 + 3 = 99723 to the greatest number of four digits and then divide
49.
(c) Let the numbers be 12x and 12y respectively, where x by 4320 to find the remainder.
and y are prime to each other
Therefore, L.C.M = 12xy

so, 12xy = 924
924 Required greatest number of four digits
⇒ xy = = 77
12 = 9999 – 2270 = 7729
Hence, possible pairs are (1, 77) and (7, 11) 55.
(b) L.C.M. of 24, 28, 30 and 35
50.
(c) L.C.M. of 5, 6, 7, 8 2 24, 28, 30, 35
= 35 × 24 = 840 2 12, 14, 15, 35
Therefore, the required number 3 6, 7, 15, 35
= 840x + 3, 5 2, 7, 5, 35
7 2, 7, 1, 7
which is exactly divisible by 9.
2, 1, 1, 1
For x = 2, it is divisible by 9.
Hence, = 2 × 2 × 2 × 3 × 5 × 7 = 840
Required number = 840x + 3 Greatest number of four digits = 9999
= 840 × 2 + 3 Quotient when 9999 is divided by 840 is 11 and
remainder is 759.
= 1683
\ Greatest number of four digits in this case
51.
(d) Let the number be 10x, and 10y, respectively and x
= 9999 – 759 = 9240
and y are prime to each other.
56. (c) Required number
Therefore, L.C.M. = 10xy
= (L.C.M. of 14, 15, 21, 32, 60) + 11
⇒ 10xy = 120
= 3360 + 11 = 3371
120
⇒ xy = = 12
10
Possible pairs = (3, 4) or (1, 12)

Chapter_02.indd 42 1/30/2016 2:14:56 PM


3 Square Root
and Cube Root

SQUARE II. Method of Division


A number multiplied by itself is known as the square of the This method is used when the number is large and the
given number. For example, square of 6 is 6 × 6 = 36. factors cannot be easily determined.
Square Root The working rule is explained with the help of following
Square root of a given number is that number which when example:
multiplied by itself is equal to the given number.
For example, square root of 81 is 9 because 92 Step 1: The digits of a number, whose square
= 9 × 9 = 81. root is required, are separated into periods
The square root of a number is denoted by the symbol of two beginning from the right. The last
 or , called the radical sign. period may be either single digit or a pair.

Thus, 81 = 9, 64 = 8 and so on. 476

Note that 1 = 1. 8 22 65 76
87 16
Methods of Finding the Square Root
665
I. Prime Factorization Method 609

1. Find the prime factors of the given number. 946 5676


2. Group the factors in pairs. 5676
3. Take one number from each pair of factors and then ×
multiply them together.
This product is the square root of the given number. Step 2: Find a number (here, 4) whose square may
be equal to or less than the first period
Illustration 1 Find the square root of (here, 22).
(i) 4761 (ii) 207025 Step 3: Find out the remainder (here, 6) and bring
down the next period (here, 65).
Solution: (i) 4761 = 23  23  3
 3 Step 4: Double the quotient (here, 4) and write to
the left (here, 8).
∴ 4761 = 23 × 3 = 69. Step 5: The divisor of this stage will be equal to
 5 × 7
(ii) 207025 = 5  7 × 13  13 the above sum (here, 8) with the quotient of
 this stage (here, 7) suffixed to it (here, 87).
∴ 207025 = 5 × 7 × 13 = 455. Step 6: Repeat this process (step 4 and step 5) till
all the periods get exhausted.
Note: The above method is used when the given number is
The quotient (here, 476) is equal to the square
a perfect square or when every prime factor of that number root of the given number (here, 226576).
is repeated twice.

Chapter_03.indd 43 1/30/2016 2:29:03 PM


44  Chapter 3

Illustration 2  Find the square root of (ii) 11.27


(i) 180625 (ii) 1498176 1 1 27. 01 29
Solution: (i) 425 1 .
8 18 06 25 21 27
21
82 16
222 601
206
444
164
2447 15729
845 4225 15729
4225                ×
        × \ 127.0129 = 11.27
Thus, 180625 = 425. (iii) Since the number of decimal places is odd, we
make it even by affixing one 0 to the right.
(ii)   1224
0.423
1 1 49 81 76
4 0. 17 90 13 60
1
16
22 49
44 82 190
164
242 581
843 2613
484
2529
2444 9776
846 8460
9776
              × In the above solution, after bringing down the last
period, we note that the remainder is not zero. So, a pair
Thus, 1478176 = 1224. of zeros can be annexed and process can be continued to
find the square root up to 4 places of decimals. The above
Square Root of a Decimal process can be continued still further and square root up to
If the given number is having decimal, we separate the required number of decimal places can be obtained.
digits of that number into periods of two to the right and left Note: If a decimal has an odd number of decimal places, its
starting from the decimal point and then proceed as in the square root cannot be found exactly.
following illustration: (iv) .025
Illustration 3  Find the square root of 0. 00 06 25
(i) 12.1801 (ii) 127.0129 00
(iii) 0.1790136 (iv) 0.000625    02   06
Solution:  (i) 3.49   04
3 12. 18 01    45    225
16    225
64 665       ×
609
\   0.000625 = 0.025.
689 6201
6201 Square Root of a Fraction
(a) If the denominator is a perfect square:  The square
        ×
root is found by taking the square root of the numerator
\   12.1801 = 3.49. and denominator separately.

Chapter_03.indd 44 1/30/2016 11:18:47 AM


Square Root and Cube Root 45

(b) If the denominator is not a perfect square: The Illustration 5 Find the cube root of 42875.
fraction is converted into decimal and then square root Solution: Resolving 42875 into prime factors, we get
is obtained or the denominator is made perfect square 42875 = 5  5 × 5  7 × 7
7
by multiplying and dividing by a suitable number and
then its square root is obtained. 3
\ 42875 = 5 × 7 = 35.
Illustration 4 Find the square root of
II. Short-cut Method to Find Cube Roots of Exact
2704 44
(i) (ii) Cubes Consisting of up to 6 Digits:
49 25
Before we discuss the method to find the cube
354 461 roots of exact cubes, the following two remarks
(iii) (iv)
43 32 are very useful and must be remembered by heart.
2704 2704 52  52 52 1. 13 = 1; 23 = 9; 33 = 27; 43 = 64; 53 = 125; 63
Solution: (i) = = = = 216; 73 = 343; 83 = 512;
49 49 77 7
93 = 729; 103 = 1000.
3
= 7 . 2. If the cube ends in 1, then its cube root ends in 1
7
If the cube ends in 2, then its cube root ends in 8
44 44 44 44 6.6332 If the cube ends in 3, then its cube root ends in 7
(ii) = = = =
25 25 55 5 5 If the cube ends in 4, then its cube root ends in 4
= 1.3266 (nearly). If the cube ends in 5, then its cube root ends in 5
If the cube ends in 6, then its cube root ends in 6
354
(iii) = 8.2325 = 2.8692 (nearly) If the cube ends in 7, then its cube root ends in 3
43 If the cube ends in 8, then its cube root ends in 2
461 461  2 922 30.3644 If the cube ends in 9, then its cube root ends in 9
(iv) = = =
32 32  2 64 8 If the cube ends in 0, then its cube root ends in 0
= 3.7955 (nearly). Clearly from above
Cube 1 ↔ 1, 4 ↔ 4, 5 ↔ 5, 6 ↔ 6, 9 ↔ 9, 0 ↔ 0
Cube of a number is obtained by multiplying the number 2 ↔ 8, 3 ↔ 7.
itself thrice.
For example, 27 is the cube of 3 as 27 = 3 × 3 × 3. The method of finding the cube root of a number up
to 6 digits which is actually a cube of some number
Cube Root consisting of 2 digits is best illustrated with the help of
following examples:
Cube root of a given number is that number which when
raised to the third power produces the given number, that is Illustration 6 Find the cube roots of the following:
the cube root of a number x is the number whose cube is x. (i) 2744 (ii) 9261
3 (iii) 19684 (iv) 54872
The cube root of x is written as x.
(v) 614125
For example, cube root of 64 is 4 as 4 × 4 × 4 = 64.
Solution: (i) Make groups of 3 digits from the right side.
Methods to Find Cube Root 2 744
2 lies between 13 and 23, so left digit is 1.
I. Method of Factorization 744 ends in 4, so right digit is 4.
1. Write the given number as product of prime factors. Thus, cube root of 2744 is 14.
(ii) 9 261
2. Take the product of prime numbers, choosing one
out of three of each type. 9 lies between 23 and 33, so left digit is 2.
This product gives the cube root of the given number. 261 ends in 1, so right digit is 1.
Thus, cube root of 9261 is 21.

Chapter_03.indd 45 1/30/2016 11:18:48 AM


46  Chapter 3

(iii)
19 683 872 ends in 2, so right digit is 8.
19 lies between 23 and 33, so left digit is 2. Thus, cube root of 19683 is 38.
683 ends in 3, so right digit is 7. (iv) 614 125
Thus, cube root of 19683 is 27. 614 lies between 83 and 93, so left digit is 8.
(iv) 54 872 125 ends in 5, so right digit is 5.
54 lies between 33 and 43, so left digit is 3. Thus, cube root of 614125 is 85.

Multiple Choice QuestionS

6. The smallest number by which 3600 must be multiplied to


1. Taking 2 = 1.4l4, 3 = 1.732, 5 = 2.236 and 6 make it a perfect cube is
9 2 6 2 (a) 40 (b) 60
= 2.449, then the value of  to three
5 3 5 3 (c) 20 (d) 15
places of decimals is 7. By what least number, 2450 be multiplied, so that the
(a) 9.231 (b) 13.716 resulting number is perfect square?
(c) 11.723 (d) 15.892 (a) 8 (b) 10
[Based on MAT, 2002] (c) 5 (d) 2
2
2. If n = 12345678987654321, what is n? 8. The largest number of five digits which is a perfect square
(a) 12344321 (b) 1235789 is
(c) 111111111 (d) 11111111 (a) 97344 (b) 98596
3. Let n (> 1) be a composite integer such that n is not an (c) 99856 (d) None of these
integer. Consider the following statements: 9. The smallest number which when subtracted from the
A: n has a perfect integer–valued divisor which is greater number 62512 makes it a perfect square is
than 1 and less than n (a) 22 (b) 32
B: n has a perfect integer–valued divisor which is greater (c) 12 (d) 2
than n but less than n 4/3
 8 
(a) Both A and B are false 10.  simplifies to
 125 
(b) A is true but B is false
(c) A is false but B is true 625 625
(a) (b)
16 8
(d) Both A and B are true
625 16
4. One-fourth of a herd of camels was seen in the forest. (c) (d)
32 625
Twice the square root of the herd had gone to mountains
[Based on SSC (GL) Prel. Exam, 2000]
and the remaining 15 camels were seen on the bank of a
river. Find the total number of camels. 11. What least number should be subtracted from the square
15
(a) 32 (b) 34 root of 21 so that the result is a whole number?
289
(c) 35 (d) 36
[Based on MAT, 2005]
(a) 15/289 (b) 7/17
5. An operation ‘$’ is defined as follows (c) 10/17 (d) 5/17
For any two positive integers x and y,
 1 2 1 2 
 x 12.    simplifies to
y  5 3 5  3
x$y =
   then which of the following is an
 y x 
(a) 5  6 (b) 2 5  6
integer?
(a) 4$9 (b) 4$16 (b) 5  6 (d) 2 5  3 6
(c) 4$1 (d) None of the above [Based on SSC (GL) Prel. Exam, 2000]

Chapter_03.indd 46 1/30/2016 11:18:49 AM


Square Root and Cube Root  47

1  1  1  1  1 
13. If x = 7 – 4 3, then the value of x + is 22. The value of 1   1  1 1 is
x  x   x  1  x  2   x  3 
(a) 3 3 (b) 8 3 1
(a) 1  (b) x + 4
(c) 14 + 8 3 (d) 14 x4
[Based on SSC (GL) Prel. Exam, 2000] 1 x4
(c) (d)
14. A general wishing to draw up his 16160 men in the form x x
of a solid square found that he had 31 men over. The [Based on SSC (GL) Prel. Exam, 2000]
number of men in the front row is
 1 1 1 1 2 5 3 7
(a) 127 (b) 123 23. When      is divided by    , the
 2 4 5 6 5 9 5 18
(c) 137 (d) 129
result is
5 1 5 1 1 1
15. If a = and b = , then the value of (a) 5 (b) 2
5 1 5 1 10 18
a 2  ab  b 2 1 3
is (c) 3 (d) 3
a 2  ab  b 2 6 10
[Based on SSC (GL) Prel. Exam, 2000]
3 4
(a) (b)
4 3 2 2
 5  3  5  3
24.     is equal to
3 5  5  3  5  3
(c) (d)
5 3 (a) 64 (b) 62
(c) 66 (d) 68
3 3 3 3 3
16. 3 =? [Based on SSC (GL) Prel. Exam, 2000]
25
(a) 331/64 (b) 331/32 25. If 25 is divisible by 26, the remainder is
(c) 31/64 (d) None of these (a) 1 (b) 2
17. A general wishing to draw up his 16160 men in the form (c) 24 (d) 25
of a solid square, found that he had 31 men left over. Find [Based on SSC (GL) Prel. Exam, 2000]
the number of men in the front.
(a) 126 (b) 125 26. 6  6  6  ...
(c) 128 (d) 127
(a) 62/3 (b) 6
18. 71/4 × (343) 0.25 is equal to (c) 31/3 (d) 3
(a) 7 (b) 49 [Based on SSC (GL) Prel. Exam, 2000]

(c) 7 (d) 7 7 27. A certain number of people agree to subscribe as


[Based on SSC (GL) Prel. Exam, 2000] many rupees each as there are subscribers. The whole
subscription is `2582449. Find the number of subscribers.
19. One less then (49)15 is exactly divisible by
(a) 1607 (b) 1291225
(a) 50 (b) 51
(c) 14 (d) 8 (c) 1503 (d) 1603
[Based on SSC (GL) Prel. Exam, 2000] 28. (16)0.16 × (16)0.04 × (2)0.2 is equal to
20. A man plants his orchard with 5625 trees and arranges (a) 1 (b) 2
them so that there are as many rows as there are trees in a (c) 4 (d) 16
row. How many rows are there?
[Based on SSC (GL) Prel. Exam, 2000]
(a) 125 (b) 25
(c) 75 (d) None of these 29. A number divided by 68 gives the quotient 269 and
remainder zero. If the same number is divided by 67, the
21. Find the number whose square is equal to the difference remainder is
between the squares of 75.15 and 60.12. (a) 0 (b) 1
(a) 225.9 (b) 67.635 (c) 2 (d) 3
(c) 45.09 (d) 15.03 [Based on SSC (GL) Prel. Exam, 2000]

Chapter_03.indd 47 1/30/2016 11:18:49 AM


48  Chapter 3

38. A man spends one-third of his income on food, two-


2176 x
30. If 1 1 = 1  , value of x is fifths of his income on house rent and one-fifth of his
2401 7 income on clothes. If he still has `400 left with him, his
(a) 3 (b) 1 income is
(c) 5 (d) 7 (a) `4000 (b) `5000
(c) `6000 (d) `7000
31. Which of the following numbers, wherein some of the
[Based on SSC (GL) Prel. Exam, 2003]
digits have been suppressed by symbols, can possibly be
the perfect square of a 3-digit odd number? 39. If a * b = 2a + 3b , then the value of 2 * 3 + 3 * 4 is
(a) 65 ××× 1 (b) 9 ×× 1 (a) 24 (b) 31
(c) 10 ××× 4 (d) 9 ×××××× 5 (c) 32 (d) 34
[Based on SSC (GL) Prel. Exam, 2002]
32. Find the value of * in the following:
4 4
2 2  1 2 1 3 6  6 3 
1    1   40. The simplified value of  29    29  is
3 7 7 4 3 6    
1 (a) 216 (b) 212
(a) (b) 0.6
6 (c) 28 (d) 24
(c) 0.006 (d) 6 [Based on SBI PO Exam, 2000]
[Based on SSC (GL) Prel. Exam, 2002] 41. The value of the following is
33. A certain amount of money is distributed among A, B and
10  25 108 154 
3 1 225
C. A gets and B gets of the whole amount. If C gets
16 4 (a) 10 (b) 8
`. 81, then B gets (c) 6 (d) 4
(a) `30 (b) `36 [Based on Bank of Baroda PO, 1999]
(c) `32 (d) `40
42. What will come in place of the question mark (?) in the
[Based on SSC (GL) Prel. Exam, 2002] following equation
34. The areas of two square fields are 420.25 m2 and 441 m2 257.5 × 52.5 ÷ 1251.5 = 5 ?
respectively. The ratio of their sides is
(a) 16 (b) 17.5
(a) 20:21 (b) 40:41 (c) 8.5 (d) 13
(c) 41:42 (d) 40:42 [Based on PNB Management Trainee Exam, 2003]
35. If * means adding 6 times the second number to the first 43. Two-fifths of one-third for three-sevenths of a number is
number then (1 * 2) * 3 equals 15. What is 40 per cent of that number?
(a) 121 (b) 31 (a) 136 (b) 140
(c) 93 (d) 91 (c) 72 (d) None of these
[Based on SSC (GL) Prel. Exam, 2003] [Based on IBPS Jr. Executive Exam, 2002]

2 3 44. By how much is two-fifths of 200 greater than three-fifths


36. Find the value of  of 125?
1 5 3 1
1 of  1
1 6 2 4 (a) 15 (b) 3
1
2 (c) 5 (d) 30
(a) 6 (b) 8 [Based on Canara Bank PO, 2003]
(c) 4 (d) 2 45. Which of the following has fractions in ascending order?
[Based on SSC (GL) Prel. Exam, 2003] 2 3 7 9 8 3 2 7 9 8
(a) , , , , (b) , , , ,
3 5 9 11 9 5 3 9 11 9
3 4
37. If 12 = 3.464, value of + 2 is
4 3 8 9 7 3 2 3 2 9 7 8
(c) , , , , (d) , , , ,
(a) 3.17 (b) 3.464 9 11 9 5 3 5 3 11 9 9
(c) 3.1753 (d) None of these [Based on NABARD Asst. Manager Exam, 2002]

Chapter_03.indd 48 1/30/2016 11:18:50 AM


Square Root and Cube Root  49

46. What should come in place of the question mark (?) in the
5 10
following equation 54.
If 5 = 2.236, then the value of – + 125 is
2 5
 47 
?
477.5 ÷ 473/2 × 47–3 = equal to
(a) 7.826 (b) 8.944
1
(a) 3 (b) 2 (c) 5.59 (d) 10.062
2
[Based on MAT, 1998]
(c) 6 (d) 3.5
[Based on BSRB Patana PO, 2001] 2
1 1 1
55.
If a =   , b = and c = , then which of the
47. Given that 4096 = 64, the value of 4096 + 40.96  10  5 100
+ .004096 is following statements is correct?
(a) 70.4 (b) 70.464 (a) a < b < c (b) a < c < b
(c) 71.104 (d) 71.4 (c) b < c < a (d) c < a < b
[Based on MAT, 1998]

48.
10008.992  3589 × 0.4987 = ? 56.
Which is the smallest of the following numbers?
10009.001
(a) 7 (b) 1/ 7
(a) 3000 (b) 300000
(c) 3000000 (d) 5000 (c) 7 /7 (d) 1/7
[Based on BSRB Bhopal PO, 2000] [Based on MAT, 1999]

49. The least number by which 14175 be divided to make it a 57.


The value of 3 0.000064 is
perfect square is (a) 0.02 (b) 0.2
(a) 3 (b) 5 (c) 2.0 (d) None
(c) 7 (d) 15 [Based on MAT, 1999]
50. Multiply the difference between the two lowest numbers 58.
Square root of 117649 is
with the difference between the two highest numbers in
(a) 347 (b) 343
the following sequences:
(c) 353 (d) 357
89, 7, 91, 72, 31, 25, 18, 89, 16, 58, 38, 42, 86
[Based on MAT, 2000]
(a) 18 (b) 77
(c) 81 (d) 16 59.
Cube root of 658503 is
 [Based on NABARD, 1999] (a) 83 (b) 77
51. One-fifth of a number is equal to five-eighths of the second (c) 87 (d) 97
number. If 35 is added to the first number it becomes four [Based on MAT, 2000]
times of second number. What is the value of the second
number? 60. 110.25  0.01  0.0025  420.25 equals to
(a) 125 (b) 70 (a) 0.75 (b) 0.50
(c) 40 (d) 25 (c) 0.64 (d) 0.73
[Based on SNAP, 2010]
 [Based on NABARD, 1999]
61.
Three cubes of iron whose edges are 6 cm, 8 cm and
52.
If x + 49 = 8.2, then the value of x is equal to 10 cm, respectively, are melted and formed into a single
(a) 1.20 (b) 1.40 cube. The edge of the new cube formed is
(c) 1.44 (d) 1.89 (a) 12 cm (b) 14 cm
[Based on MAT, 1998] (c) 16 cm (d) 18 cm
2 [Based on FMS (MS), 2006]

 −1  3
53.
The value of   is 62.
If 841 = 29, then the value of
 216 

(a)
1
(b) –
1  841  8.41  0.0841  0.000841 is 
36 36 (a) 322.19 (b) 32.219
(c) – 36 (d) 36 (c) 34.179 (d) 31.129
[Based on MAT, 1998] [Based on FMS (MS), 2006]

Chapter_03.indd 49 1/30/2016 11:18:51 AM


50  Chapter 3

a 71. (35) 2 ÷ 3 125 + (25) 2 ÷ 125 =


?
63.
If (0.05  0.5  a )  0.5  0.05  b , then is equal to
b (a) 200 (b) 250
(a) 0.0025 (b) 0.025
(c) 150 (d) 100
(c) 0.25 (d) None of the above [Based on Punjab National Bank PO, 2010]
[Based on FMS (MS), 2006]
72. (?)2 × (12)2 ÷ (48)2 = 81
a (a) 26 (b) 32
64.
If 0.04  0.4  a  0.4  0.04  b , than is
b (c) 9 (d) None of these
(a) 0.016 (b) 0.16 [Based on Punjab National Bank PO, 2010]
(c) 1 (d) 16 73. 1225 = ?
[Based on FMS, 2005] (a) 25 (b) 45
65.
A gardener plants 17956 trees in such a way that there are (c) 55 (d) None of these
as many rows as there are trees in a row. The number of [Based on Haryana Grameen Bank PO, 2009]
trees in a row are
74. Which number should replace both the question marks in
(a) 136 (b) 134 the following equation
(c) 144 (d) 154 ? 16
[Based on FMS, 2006]
=
49 ?
66.
[1 / ( 9  8)  1 / ( 8  7)  1 / ( 7  6) (a) 48 (b) 18
(c) 38 (d) 28
1 / ( 6  5)  1 / ( 5  4)] 
? [Based on Haryana Grameen Bank PO, 2009]
(a) 0 (b) 1
75. 898 × (12.005)2 + ? = 5000
(c) 5 (d) 1/3
[Based on FMS, 2006]
(a) 680 (b) 720
(c) 750 (d) 620
67.
The number 8  3 7 is equal to
[Based on RBI (Grade ‘B’) PO, 2009]
8  7
(a)   (b) 8  3 7 76.   44944 + 52441 = ?
 2 
(a) 312 (b) 441
8  7
(c) 2 2  3 7 (d)  (c) 485 (d) None of these
 
2  [Based on Andhra Bank PO, 2008]
[Based on FMS, 2009] 77.  Which number should replace both the question marks in
4 3 the following equation?
68.
The expression  is equal to ? 76
3 4 =
3 3 171 ?
(a) (b) – (a) 114 (b) 116
6 6
(c) 57 (d) 176
3 5 3
(c) (d) [Based on Andhra Bank PO, 2008]
6 6
78. (72)2 + (61)2 = (199)2 – (?) – 420
[Based on FMS, 2010]
(a) 165 (b) 198
6407522209
69.
2 = is equal to (c) 182 (d) 174
3600840049 [Based on Uttrakhand GBO PO, 2007]
(a) 0.666039 (b) 0.666029
79. Which number should replace both the question marks in
(c) 0.666009 (d) None of these the following equation?
[Based on IIFT, 2008]
? 59
3 2 3 ? =
70. 9 × 81 ÷ 27 = (3) 944 ?
(a) 3 (b) 4 (a) 218 (b) 236
(c) 5 (d) 6 (c) 244 (d) 264
[Based on Punjab and Sindh Bank PO, 2010] Based on Uttrakhand GBO PO, 2007]

Chapter_03.indd 50 1/30/2016 11:18:51 AM


Square Root and Cube Root  51

2
80. 3
4663 + 349 = ? ÷ 21.003 90. ( 2 392 − 21) + ( 8 − 7 ) = (?)2
(a) 7600 (b) 7650 (a) 4 (b) – 4
(c) 7860 (d) 7680 (c) 12 (d) 6
[Based on IBPS Bank PO, 2011]  [Based on Corporation Bank PO, 2011]
81. 6354 × 34.993 = ?
91. In a class of 40 students and 5 teachers, each student got
(a) 3000 (b) 2800 sweets that are 25 per cent of the total number of students
(c) 2500 (d) 3300 and each teacher got sweets that are 15 per cent of the
[Based on IBPS Bank PO, 2011] total number of students. How many sweets were there?
82. 3 1331 = ? (a) 480 (b) 440
(a) 27 (b) 21 (c) 430 (d) 450
(c) 17 (d) None of these  [Based on IRMA, 2008]
[Based on OBC PO, 2009]
92. 915849 + 795664 = (?)2
4 2
83. 24 + 224 = ? × 20 (a) 1849 (b) 79
(a) 20 (b) 4
(c) 33 (d) None of these
(c) 2 (d) 16
 [Based on IRMA, 2009]
[Based on United Bank of India PO, 2009]
84. Which number should replace both the question marks (?) 93. 956240 = ?
in the following equation? (a) 979 (b) 864
? 256 (c) 1009 (d) 647

576 ?  [Based on IRMA, 2009]
(a) 384 (b) 398
(c) 404 (d) 416 5− 3
94. Simplify
[Based on Indian Overseas Bank PO, 2009] 5+ 3
85. Which number should replace both the question marks in (a) 1 (b) 2
the following equation? (c) 3 (d) None of these
? 243 [Based on NMAT, 2006]
=
432 ?
95. If x = 0.888, y = 0.888 and z = (0.888)2, then which of
(a) 308 (b) 312 the following is true?
(c) 324 (d) 316 (a) y < x < z (b) y < z < x
 [Based on SBI PO, 2008]
(c) x < z < y (d) z < x < y
86. What is the least number that can be added to the number [Based on ATMA, 2005]
1020 to make it a perfect square?
96.
The positive integer n is divisible by 25. If n is greater
(a) 65 (b) 12
than 25, which of the following could be the value of
(c) 59 (d) 4 n
[Based on Indian Bank PO, 2011] ?
25
87. (?)3 = 4913 (a) 26 (b) 23
(a) 27 (b) 19 (c) 25 (d) 22
(c) 17 (d) 29 [Based on ATMA, 2008]
[Based on Indian Bank PO, 2011]
If x = 3 + 2 2 , then the value of (x1/2 – x– 1/2) is
97.
88. 348 ÷ 29 × 15 + 156 = (?)3 + 120
(a) 2 (b) 2
(a) 12 (b) 6
(c) 36 (d) 9 (c) 2 − 1 (d) 2 + 1
[Based on Corporation Bank PO, 2011] [Based on ATMA, 2008]
3 4 4 ?+4 1/2 1/2 3
89. (4 × 4) ÷ (512 ÷ 8) × (32 × 8) = (2 × 2) 98. ( 8 × 8) + (9) = (?) + 8 − 340
(a) 8 (b) 12 (a) 7 (b) 19
(c) 6 (d) 14 (c) 18 (d) 9
[Based on Corporation Bank PO, 2011] [Based on Bank of Baroda PO Exam, 2011]

Chapter_03.indd 51 1/30/2016 11:18:52 AM


52  Chapter 3

99. Sum of square of first number and cube of second number (a) V ∝ t2 (b) V ∝ t
is 568 together. Also square of the second number is 15 1
less than square of 8. What is the value of three-fifths of (c) V ∝ (d) V ∝ t
t
the first number? (assuming both the numbers are positive) [Based on GBO Delhi University, 2011]
(a) 18 (b) 8
1 1
(c) 9 (d) 16 107.
The expression (2 + 2) + + equals
(2 + 2) (2 − 2)
[Based on Bank of Baroda PO Exam, 2011]
100. (32)2 + ? – (23)2 = 536 (a) 4 + 2 (b) 2 2
(a) 1764 (b) 1849 (c) 4 − 2 (d) 2 + 2
(c) 1521 (d) 1681 [Based on U.P. P.C.S., 2012]
 [Based on NMAT, 2008]
108. The square root of 0.09 is
101. 1562000 = ? (a) 0.30 (b) 0.03
(a) 1175 (b) 1200 (c) 0.81 (d) 0.081
(c) 1250 (d) 1325 [Based on SSC (GL), 2010]
 [Based on NMAT, 2008] x 32
109. If 1 + = then the value of x is
102. (?)3 = 729 961 31
(a) 14 (b) 7 (a) 63 (b) 61
(c) 19 (d) None of these (c) 65 (d) 64
[Based on Bank of India PO, 2010] [Based on SSC (GL), 2011]

103. 2809 = ? x 13
110.
If 1 + = , then the value of x is
(a) 43 (b) 47 9 3
(c) 57 (d) 53 1439
(a) (b) 160
 [Based on IDBI PO, 2009] 9
1443
104.
The value of 3
1.001001001 − 3 1.001001 is closest to (c) (d) 169
9
(a) 0 (b) 10 –6 [Based on SSC (GL), 2011]

(c) 3.10– 9 (d) (3.3).10– 10 4 3+5 2


111.
If = a + b 6, then the values of a and b are,
[Based on GBO Delhi University, 2011] 48 + 18
105.
If the square root of a number is between 6 and 7, then the respectively,
cube root of the number will be between 9 4 3 4
(a) 1 and 2 (b) 2 and 3 (a) , (b) ,
15 15 11 33
(c) 3 and 4 (d) 4 and 5 9 2 3 4
[Based on GBO Delhi University, 2011] (c) , (d) ,
10 5 5 15
106.
V is inversely proportional to the square root of m and m is [Based on SSC (GL), 2011]
inversely proportional to the square of t. The relationship
between V and t is

Chapter_03.indd 52 1/30/2016 2:29:34 PM


Square Root and Cube Root  53

EXPLANATORY ANSWERS

9.
(a)
9 2 6 2
1.
(c)  250
5 3 5 3
2 62 5 12
9 5  9 3  10  6  6 5  6 3  10  6
= 4
2
45 225
15 5  3 3  2 6
= 225
2
50 12
15  2.236  3  1.732  2  2.449
=
2 So, 12 is the smallest number which when subtracted
33.54  5.196  4.898 from 62512 makes it a perfect square
= = 11.723.
2 62500 = 250
2. (d) Square root of 12345678987654321 is 1111111 4/3 4 4
 8   2  5 625
10.
(a)  =   =   = .
3. (d) Consider a number n = 6  125   5  2 16
n = 245 2
15 6084  78 
A: We have a divisor 2 which is greater than 1 and 11.
(c) 21 = =  
289 289  17 
less than 6 .
78 10
B: We have a divisor 3 which is greater than but less \ Square root = =4
than 6. 17 17
Thus, both statements are true. 10
\ Least fraction to be subtracted =
17
36
4.
(d) No. of camels seen in the forest = =9
4 (1  2)( 5  3)  (1  2)( 5  3)
12.
(c)
No. of camels gone to the mountains = 2 36 = 12 53
No. of camels seen on the bank of river = 15 5  3  10  6  5  3  10  6
=
\ Total number of camels = 9 + 12 + 15 = 36. 2
5.
(d) By direct substitution
=
2  5 6 = 5  6.
6.
(b) 3600 = 2 × 2 × 2 × 2 × 3 × 3 × 5 × 5 2
= 22 × 22 × 32 × 52 13.
(d) x = 7 – 4 3
\ 3600 should be multiplied by 2 × 2 × 3 × 5. That is
60 to make it a perfect cube. 1 1 74 3
= 
7. (d) 2450 = 5 × 5 × 7 × 7 × 2 x 74 3 74 3
\ 2450 must be multiplied by 2 = 7 + 4 3
8.
(c) Largest number of 5 digits = 99999 1
\ x+ = 14.
316 x
3 9 99 99 14. (a) The number of men in the front row is the square root
of 16160 – 31, that is 16129 which is 127.
9
61 99 5 1 5 1 ( 5  1) 2
15. (b) a =  =
61 5 1 5 1 ( 5) 2  (1) 2
625 3899 5 1 2 5
=
3756 5 1
143
62 5 3 5
2
= =
Required number = (316) = 99856 4 2

Chapter_03.indd 53 1/30/2016 11:18:55 AM


54  Chapter 3

5 1 5 1 ( 5  1) 2  ( 5  3)( 3  3) 
2
 ( 5  3)( 5  3) 
2
b =  = =      
5 1 5 1 ( 5) 2  (1) 2  ( 5  3)( 5  3)   ( 5  3)( 5  3) 
5 1 2 5
= = 62 5 = 3 5  5  3  2 15 
2
 5  3  2 15 
2
5 1 4 2 =      
 5  3   53 
2 2
a2 + b2 = (3  5)  (3  5)
2 2
4 =    4  15    4  15 
956 5 956 5
= = 28 = 7
5 4 =  16 + 15 + 8 15 + 16 + 15 – 8 15
ab = 1 =  16 + 15 + 16 +15  or,  62.
2 2 7 1
a  ab  b 8 4 (d) 2525 = (26 – 1)25
25.
\ = = =
a 2  ab  b 2 7 1 6 3
= 2625 + × 2624 × (–1)1 + × 2623

3 3 3
  × (– 1)2 + ... + (–1)25
3 3 3
16. (b) 3.31/2
= 3.3 3/4
= 3.37/8
[using Binomial theorem]
= 3.3 15/16
= 331/32 Now, all the terms are divisible by 26 except the last
term (–1)25. So, the remainder is 26 – 1 = 25.
17. (d) 16160 – 31 = 16129; 16129 = 127
6 6
(c) 71/4 × (73)1/4 = 7.
18. 26.
(d) Let x = 6 
n On squaring, we get
19.
(d) a – 1 is always divisible by a – 1
\ 4915 – 1 is divisible by 48, i.e. 8. x2 = 6 + x  or  x2 – x – 6 = 0
20.
(c) Let the number of trees be n or,   (x – 3) (x + 2) = 0  or,  x = 3, – 2
\ n2 = 5625 But –ve value cannot be accepted.
⇒ n = 5625 = 75 \ x = 3.
2 2 2
21.
(c) x = (75.15) – (60.12) 27.
(a) Let the number of subscribers be n
= 5647.52 – 3614.41 ⇒ n2 = 258249
= 2033.11
\ n = 2582449 = 1607
\ x = 45.09
28.
(b) Given expression = (24)0.16 × (24)0.04 × (2)0.2
 1  1  1  1 
22.
(d) 1   1  1 1
 x   x  1  x  2   x  3  = 20.64 × 20.16 × 20.2
= 21 = 2.
 x  1  x  2   x  3   x  4 
= 
 x   x  1   x  2   x  3  29.
(b) Number = 269 × 68 = 18292
= 67 × 273 + 1.
x4
= .
x 2176 x
30. (b) 1  1  = 1 +
1 1 1 1 30  15  12  10 17 2401 7
23.
(a)      = =
2 4 5 6 60 60
x 225 15
⇒ 1+ = 1 = 1
2 5 3 7 36  50  54  35 5 7 2401 49
   = =
5 9 5 18 90 90
64 8 1
17 / 60 51 1
= = =1+
According to question = = 5 . 49 7 7
5 / 90 10 10 \ x = 1
2 2 31. (a) The square of an odd number cannot have 4 as the unit
 5  3  5  3
24.
(b)     digit. The square of a 3-digit number will have at least
 5  3   5  3 5 digits and at the most 6 digits.

Chapter_03.indd 54 1/30/2016 11:18:56 AM


Square Root and Cube Root  55

5 2  5 2 1 42.
(d) 257.5 × 52.5 ÷ 1251.5 = 5?
32.
(d)   =  
3 7 7 4 3 6 or, 52×7.5 × 52.5 ÷ 53×1.5 = 5?
5 7  5 2 6 or, 515 × 52.5 ÷ 54.5 = 5?
or,   =  
3 2 7 4 3 1 1
or, 517.5 × = 5?  or,  513 = 5?
5 2 6 3 2 54.5
or, * =    
4 3 1 5 7 or, ? = 13.
Hence, * = 6. 43.
(d) Let the number be x
33.
(b) Suppose that certain amount is `x
2 1 3
 3 x Then,    x = 15
Then,  x  x   = 81 5 3 7
 16 4
2x 15  35
or, 16x – 3x – 4x = 81 × 16 or, = 15 or x =
81  16 35 2
or, x = = 144
9 40 15  35
\ 40 % of x =  = 105.
1 100 2
Hence, B gets = 144 × = `36.
4
2 3
34.
(c) Ratio of their sides is the ratio of their square roots 44.
(c) Required number = × 200 – × 125
5 5
= 420.25 : 441 = 80 – 75 = 5.
= 20.5:21
2 2  165 330
= 41:42 45.
(b) = =
3 3  165 495
35.
(b) 1 * 2 = 1 + 2 × 6 = 1 + 12 = 13
3 3  99 297
(1 * 2) * 3 = 13 * 3 = 13 + 3 × 6 = 31. = =
5 5  99 495
36.
(d) The given expression 7 7  55 385
= =
2 2 3 9 9  55 495
=  =  3 = 2.
2 5 5 3
1  9 9  45 405
2 1 4 4 = =
11 11  45 495
3 4  1 2
37.
(c) 2 = 12    8 8  55 440
4 3  4 3 = =
9 9  55 495
3.464  11
= = 3.1753 3 2 7 9 8
12 Ascending order , , , , .
5 3 9 11 9
 1 2 1 1
38.
(c) Man has 1      =
 3 3 5 15
(c) ((47)1/2)15 ÷ ((47)1/2)3 × ((47)1/2)–6 = ( 47)?
46.
\ Man’s income = 400 × 15 = `6000.
= ( 47)15 ÷ ( 47)3 × ( 47) 6
39.
(b) 2 * 3 + 3 * 4
= [2 (2) + 3 (3)] + [2 (3) + 3 (4)] = ( 47)?
= [4 + 9] + [6 + 12] = 31. ⇒ ( 47)1536 = ( 47)?
40.
(d) Given expression \ ? = 6.
4 4



=  (2 )


9 1/6 1/3 


   (2 ) 
9 1/3 1/6 
 47.
(b) 4096  40.96  .004096

= (21/2)4 × (21/2)4 = 22 × 22 = 24. 4096 4096


= 4096  
100 1000000
41.
(d) Given expression
64 64
10  25 108 10  25 = 64 + 
= 154  15 = 108  13 10 1000

= 10 
25  11 = 10  6 = 16 = 4. = 64 + 6.4 + 0.064 = 70.464.

Chapter_03.indd 55 1/30/2016 3:14:26 PM


56  Chapter 3

56.
(d)
(10008.99) 2
48.
(b) ? = × 3589 × 0.4987
10009.001 57.
(b) Given expression = 3 0.008 = 0.2

≈ (10009) × 3600 × 0.50 58.


(b) 117649 = 7 × 7 × 7 × 7 × 7 × 7
≈ (10009) × 60 × 0.50 ≈ 300000. \ 117649 = 7 × 7 × 7 = 343
49.
(c) 14175 = 5 × 5 × 3 × 3 × 3 × 3 × 7
59.
(c) 658503 = 3 × 3 × 3 × 24389
= 52 × 34 × 7
= 3 × 3 × 3 × 29 × 29 × 29
It must be multiplied by 7.
\ 3 658503 = 3 × 29 = 87
50. (a) Difference between two lowest numbers
= 16 – 7 = 9 60.
(b) 110.25 =10.5
Difference between two highest numbers 0.01 = 0.1
= 91 – 89 = 2
0.0025 = 0.05
\ Product of these two numbers = 9 × 2 = 18.
420.25 = 20.5
1 5
51.
(c) I = II
5 8 \ 110.25  0.01  0.0025  420.25
I 25
\ = 0.1
II 8 = 10.5 × – 20.5
0.05
I + 35 = 4 II
1.05
25 =  20.5
or,   II + 35 = 4 II 0.05
8
= 21 – 20.5 = 0.5
\ II = 40.
Hence, option (b).
52.
(c) x = 8.2 – 7 = 1.2 ⇒ x = 1.44 (a) 63 + 83 +103 = New cube
61.
2 2 216 + 512 + 1000 = New cube
− − −2
 −1  3  −1  3  1 ⇒ 3
1728 = 12 cm
53.
(d)   =  3 = − 
 216  6   6
29 29 29
1 1 62.
(b) 29   
= = = 36 10 100 1000
 1
2 1
− = 29 + 2.9 + 0.29 + 0.029
  36
 6 = 32.219
63.
(b) On squaring both the sides, we get
5 10
54.
(a) − + 125 0.05 × 0.5 × a = (0.5)2 × (0.05)2 × b
2 5
a
5  20  2 5 125 5  20  60 ⇒ = 0.025
= = b
2 5 2 5
35  5 64.
(a) 0.04  0.4  a = 0.4 × 0.04 × b
=
2 5 5 On squaring both the sides, we get
7 7 0.04 × 0.4 × a = (0. 4)2 × (0.04)2 × b
=  5 =  2.236 = 7 × 1.118 = 7.826.
2 2 a
\ = 0.016
2 b
1 1
55.
(b) a =  =  = 0.1
 
10 100
65.
(b) 17956 = 134
1
b = = .2 1 1 1
5 66.
(c)  
9 8 8 7 7 6
1
c = = .01 .1
= 1 1
100  
\ a < c < b 6 5 5 4

Chapter_03.indd 56 1/30/2016 2:31:00 PM


Square Root and Cube Root  57

= ( 9  8)  ( 8  7)  ( 7  6) ⇒ ? = 49 × 16
 ( 6  5)  ( 5  4) ⇒ ? = 7×7×4×4
= ( 9  4) = 3 + 2 = 5 ⇒ ? = 7 × 4
⇒ ? = 28
67.
(c) 8 3 7 = 2 2 3 7
75. (a) 898 × (12.005)2 + ? = 5000
4 3 4 4 3 3 900 × (12)2 + ? ≈ 5000

68.
(a)  =
3 4 3 4 [898 ≈ 900; 12.005 ≈ 12]
43 1 ⇒ 30 × 144 + ? ≈ 5000
= =
12 12 ⇒ ? + 4320 ≈ 5000
? = 5000 – 4320
12 2 3
=  ≈ 680
12 12
3 76.
(d) ?  = 44944 + 52441
=
6 ⇒ ? = 212 + 229
6407522209 80047 ⇒ ? = 441
69.
(a) 2  = 2 
3600840049 60007 ⇒ ? = 21
= 2 – 1.3339610 ? 76
= 0.666039 77.
(a)       =
171 ?
70. (c) 93 × 812 ÷ 273 = (3)? \ (?)2 = 171 × 76
2×3 4×2 3×3 ?
3 ×3 ÷3 = (3) ⇒ ? = 171 × 76
3 × 3 ÷ 3 = (3)?
6 8 9
⇒ ? = 12996
(3)6+8–9 = (3)?
⇒ ? = 114
(3)5 = (3)?
(d) (72)2 + (61)2 = (199)2 – (?)2 – 420
78.
? = 5
(?)2 = (199)2 – 420 – (72)2 – (61)2
71. (b) (35)2 ÷ 3
125 + (25)2 ÷ 125 = ? (?)2 = 39601 – 420 – 5184 – 3721
1225 625 (?)2 = 39601 – 9325
+ = ? (∵ 3 125 = 5)
5 125 (?) = 30276 = 174
245 + 5 = ?
? 59
250 = ? 79.
(b)       =
944 ?
72. (d) (?)2 × (12)2 ÷ (48)2 = 81
(?) = 944 × 59 = 55696
(?) 2 × 12 × 12
= 81 ? = 55696 = 236
48 × 48
(?)2 = 81 × 16 = (9 × 4)2 3 ?
80.
(d) 4913 + 349 =
\ ? = 36 21
⇒ (17 + 349) × 21 = ?
73.
(d) ? = 1225
⇒ ? = 366 × 21 = 7686 ≈ 7680
⇒ ? = 5×5×7×7
81.
(b) 6400 × 35 = 80 × 35 = 2800
⇒ ? = 5 × 7
⇒ ? = 35 82.
(d) ? = 3
1331
? 16
74. (d) = = 3 11 × 11 × 11
49 ?
⇒ ?2 = 49 × 16 = 11

Chapter_03.indd 57 1/30/2016 11:19:00 AM


58  Chapter 3

40 × 25 40 × 15
83. (c) 244 + 224 = ? × 202 91. (c) 40 × + 5× = 400 + 30 = 430
100 100
⇒ 242 + 224 = ? × 400
92. (d) It will be square of 43.
⇒ 576 + 224 = ? × 400
93. (a)
⇒ 800 = ? × 400
800 5− 3 5− 3 8 − 2 15
⇒ ? = =2 94. (d) × =
400 5+ 3 5− 3 2
   = 4 – 15
? 256
84. (a) =
576 ? 95. (d) x = 0.888,
⇒ ?2 = 256 × 576 y = 0.888 = 0.942
⇒ ? = 256 × 576 z = (0.888)2 = 0.78
= 16 × 24 = 384 \ z < x < y

? 243 96.
(a) Since, n > 25 and also n is divisible by 25 then the
85. (c) = number could be 650
432 ?
n 650
⇒ (?)2 = 432 × 243 \ = = 26
25 25
? = 104976 = 324.
97.
(a) x = 3 + 2 2 = (1 + 2 )2
86. (d)
Now, taking (x1/2 – x– 1/2)
87. (c) (?)3 = 4913
(?)3 = (17)3  1 
= (1 + 2) −  
\ ? = 17 1+ 2 
88. (b) 348 ÷ 29 × 15 ÷ 156 = (?)3 + 120  2 −1
= (1 + 2) −  
⇒ 12 × 15 + 156 = (?)3 + 120  2 −1 
⇒ 180 + 156 – 120 = (?)3
= 1 + 2 – 2 +1
⇒ 216 = (?)3
= 2
⇒ (6)3 = (?)3
1 1
\ ? = 6 98. (a) (?)3 + 8 – 340 = ( 8 × 8) 2 + (9) 2
89. (c) (4 × 4)3 ÷ (512 ÷ 8)4 × (32 × 8)4 = (2 × 2)?+4
⇒ (?)3 + 8 – 340 = 8 +3
⇒ (16)3 ÷ (64)4 × (256)4 = (4)?+4
⇒ (4)2×3 ÷ (4)3×4 × (4)4×4 = (4)?+4 ⇒ (?)3 = 8 + 3 – 8 + 340
⇒ 6 12
(4) ÷ (4) × (4) = (4)16 ?+4 ⇒ (?)3 = 343
⇒ (4)6–2+16 = (4)?+4 ⇒ ? = 3 343
⇒ (4)10 = (4)?+4 ⇒ ? = 7
⇒ 10 = ? + 4 99. (c) Suppose first number = x
\ ? = 6 and second number = y
2 Then,
90. (d) (2 392 − 21) + ( 8 − 7 ) = (?)2
82 – y2 = 15
2 ⇒ 64 – y2 = 15
⇒ ( 2 × 14 2 − 21) + ( 8 − 7 ) = (?)2
⇒ y2 = 49
2
⇒ 28 2 − 21 + ( 8 ) − 2 × 8 × 7 + (7) 2 = (?)2 ⇒ y = 49 = 7
Again,
⇒ 28 2 − 21 + 8 − 28 2 + 49 = (?)2 x2 + y3 = 568
⇒ –21 + 8 + 49 = (?)2 ⇒ x2 + 73 = 568 (on putting value y = 7)
⇒ 36 = (?)2 ⇒ 2
x + 343 = 568
\ ? = 6 ⇒ x2 = 225

Chapter_03.indd 58 1/30/2016 11:19:01 AM


Square Root and Cube Root  59

⇒ x = 225 = 15 x 1024
⇒ = − 961
961 961
Hence, three-fifths of the first number
3 x 63
= × 15 ⇒ =
5 961 961
= 9 ⇒ 961x = 63 × 961
100. (d) 63 × 961
⇒ x = = 63
101. (c) 1250 × 1250 = 1562500 (1562000 approx.) 961
102. (d) (?)3 = 729
x 13
(?)3 = (9)3 110.
(b) 1+ =
9 3
? = 9
On squaring both the sides, we get
103. (d) ? = 2809 = 53 × 53 = 53 x 169
1+ =
104. (a) 9 9
105. (c) x 169 − 1 160
⇒ = =
a b 9 9 9
106.
(b) V = ,m= 2,
m t 160
⇒ x = × 9 = 160
where a and b are constants 9
a a a
V = = = ×t 4 3+5 2
m b b 111.
(d) L.H.S. =
48 + 18
c2

⇒ V µ t 4 3+5 2
=
1 1 4 3+3 2
107.
(a) (2 + 2) + +
(2 + 2) (2 − 2) On rationalizing the denominators,
2 (2 + 2) + (2 − 2) + (2 + 2) 4 3 +5 2 4 3 −3 2
= = ×
(2 + 2) (2 − 2) 4 3 +3 2 4 3 −3 2

2 (2 + 2) + 4 16 × 3 − 12 6 + 20 6 − 15 × 2
= =
2 (4 3) 2 − (3 2) 2
= (2 + 2) + 2
48 + 8 6 − 30
=
= 4 + 2 48 − 18

108.
(a) 0.09 = 0.3 48 + 8 6 9 4 6
= = +
30 15 15
x 31
109.
(a) 1+ = 3 4 6
961 32 = +
5 15
On squaring both the sides, we get
Therefore,
2
x  32  3 4 6
⇒ 1+ =   = +
961  31  5 15
x 1024 = a + b 6
⇒ 1+ = ,
961 961
3 4
x 1024 ⇒ a = and b =
⇒ = −1 5 15
961 961

Chapter_03.indd 59 1/30/2016 2:31:49 PM


4 Simplification

Simple Arithmetic Operations 17 16 9 16 69 


=    
It is a common need to simplify the expressions formulated 2  5 2 3 8 
according to the statements of the problems relating to the 17 16 24 69 
practical life. To do this, it is essential to follow in sequence =    
2  5 1 8
the mathematical operations given by the term “BODMAS”.
17 16 1 69 
BODMAS =   
2  5 24 8 
Each letter of the word BODMAS stands as follows:
B for bracket : [{(–)}] 17  16 69 
=  
There are four brackets, namely, – bar, ( ), { } and [ ]. 2 120 8 
They are removed, strictly in the order –, ( ), { } and [ ]. 17 16  1035 
O for of : = 
of 2  120 
D for division : ÷
17 1051
M for multiplication : × = 
2 120
A for addition : +
S for subtraction : – 1020  1051 31
= =  .
120 120
The order of various operations in exercises involving
brackets and fractions must be performed strictly Illustration 2  Simplify
according to the order of the letters of the word
BODMAS. 1  1  1 1 1  
5  4   3  2   
3  3  3 3 3  
Note: Here, – 5  8 = – (–3) = 3.
Illustration 1  Simplify Solution: Given expression

1  1 1 1   1 5   16 13  10 7 1  
8  3  4 of 5  11   3  1    =     
2  5 2 3   4 8   3  3  3 3 3  
Solution: Given expression
16 13  10 6  
=      
17 16 9 16   5 5   3  3  3 3 
=    of  11   3    
2  5 2 3   4 8  
16 13 4 
=   
17 16 9 16   5   3  3 3
=    of  11   3   
2 5 2 3   8   16  9  16 9 7 1
=   =  = = 2 .
17 16 9 16  19  3 3 3 3 3 3
=   of  11  
2  5 2 3  8  Use of Algebraic Formulae
17 16 9 16 69  The following important formulae are sometimes found
=    of 
2 5 2 3 8  useful in dealing with the simplifications:

Chapter_04.indd 60 1/30/2016 12:02:50 PM


Simplification 61

1. (a + b)2 = a2 + 2ab + b2 4  0.345  0.255


= [Q (a + b)2 – (a – b)2 = 4ab]
2 2
2. (a – b) = a – 2ab + b 2 4  0.345  0.255
= 1.
3. (a + b)2 + (a – b)2 = 2(a2 + b2)
0.682  0.682  0.318  0.318
4. (a + b)2 – (a – b)2 = 4ab (v)
0.682  0.318
5. a2 – b2 = (a + b) (a – b)
Solution: Given expression
6. (a + b)3 = a3 + 3a2b + 3ab2 + b3
= a3 + b3 + 3ab(a + b) (0.682) 2  (0.318) 2
=
0.682  0.318
7. (a – b)3 = a3 – 3a2b + 3ab2 – b3
= a3 – b3 – 3ab(a – b)  a 2  b2 
= (0.682 + 0.318)   a  b
8. a3 + b3 = (a + b) (a2 – ab + b2)  a b 
9. a3 – b3 = (a – b) (a2 + ab + b2) = 1.
a3  b3  c3  3abc (3.29) 2  (0.81) 2
10. = (a + b + c). (vi)
a 2  b 2  c 2  ab  bc  ca 4
11. a4 – a4 = (a2 + b2) (a + b) (a – b). Solution: Given expression
(3.29) 2  (0.81) 2
Illustration 3 Simplify the following: =
3.29  0.81
(i) 0.32 × 0.32 + 0.64 × 0.68 + 0.68 × 0.68  a 2  b2 
Solution: Given expression = (3.29 – 0.81)   a  b
 a b 
= 0.32 × 0.32 + 2 × 0.32 × 0.68 + 0.68 × 0.68 = 2.48.
= (0.32)2 + 2 × 0.32 × 0.68 + (0.68)2 (vii) (2.35)3 + 1.95 × (2.35)2 + 7.05 × (0.65)2 + (0.65)3
= (0.32 + 0.68)2 [Q a2 + 2ab + b2 = (a + b)2] Solution: Given expression
2
= 1 = 1. = (2.35)3 + 3 × 0.65 × (2.35)2
(ii) 2.45 × 2.45 – 0.9 × 2.45 + 0.45 × 0.45 + 3 × 2.35 × (0.65)2 + (0.65)3
Solution: Given expression = (2.35 + 0.65)3
= 2.45 × 2.45 – 2 × 2.45 × 0.45 + 0.45 × 0.45 [Q a3 + 3a2b + 3ab2 + b3 = (a + b)3]
= (2.45)2 – 2 × 2.45 × 0.45 + (0.45)2 = (3)3 = 27.
= (2.45 – 0.45)2 [Q a2 – 2ab + b2 = (a – b)2] (4.32)3  0.96  (4.32) 2  12.96  (0.32) 2  (0.32)3
2 (viii)
= (2) = 4. 444
7  {(146  92) 2  (146  92) 2 } Solution: Given expression
(iii) (4.32)3  3  0.32  (4.32) 2  3  4.32  (0.32) 2  (0.32)3
(146) 2  (92) 2 =
444
Solution: Given expression
(4.32  0.32)3
7  2{(146) 2  (92) 2 } = [Q a3 – 3a2b + 3ab2 – b3 = (a – b)3]
= 2 2 43
(146)  (92)
3
[Q (a + b)2 + (a – b)2 = 2(a2 + b2)]  4
=   = 1.
= 14.  4

(0.345  0.255) 2  (0.345  0.255) 2 885  885  885  115  115  115
(iv) (ix)
0.345  1.02 885  885  115  115  885  115
Solution: Given expression Solution: Given expression
(885)3  (115)3
(0.345  0.255) 2  (0.345  0.255) 2 =
= (885) 2  (115) 2  885  115
4  0.345  0.255

Chapter_04.indd 61 1/30/2016 12:02:51 PM


62 Chapter 4

  (xi)
a 3  b3
= (885 + 115)  2 2
 a  b (2.3)3  (1.5)3  (1.2)3  3  2.3  1.5  1.2
 a  ab  b 
2.3  2.3  1.5  1.5  1.2  1.2  2.3  1.5  2.3  1.2  1.5  1.2
= 1000.
Solution: Given expression
0.62  0.62  0.62  0.41  0.41  0.41 (2.3)3  (1.5)3  (1.2)3  3  2.3  1.5  1.2
(x) =
0.62  0.62  0.62  0.41  0.41  0.41 (2.3) 2  (1.5) 2  (1.2) 2  2.3  1.5  2.3  1.2  1.5  1.2
Solution: Given expression = (2.3 + 1.5 + 1.2)
3
(0.62)  (0.41) 3  a3  b3  c3  3abc 
=  2 2 2
 a  b  c
(0.62)  0.62  0.41  (0.41) 2
2  a  b  c  ab  ac  bc 
= 5.
 a 3  b3 
= (0.62 – 0.41)  2  a  b Surds and Indices
2
 a  ab  b  an is called a surd if n is a fraction and an is called an index
= 0.21. if n is an integer. a is called the base.

SOME USEFUL FORMULAE

1. am × an = am+n
1 a b a b
2. am ÷ an = am–n 16. = =
a b ( a  b )( a  b ) a b
3. (am)n = (an)m = amn
m m 1 a b a b
 17. = =
 a n  b n a b ( a  b )( a  b ) a b
4.   =  
 b  a
18. If x = n(n + 1), then
5. am ÷ b–n = am × bn
(a) x  x  x  ...  = n
6. ( n a ) n = a, where ‘n’ is a +ve integer and ‘a’ a
+ve rational number
(b) x  x  x  ...  = (n + 1).
n
7. a n b = n ab , where ‘n’ is a +ve integer and
‘a’, ‘b’ are rational numbers Illustration 4 Find the value of (243)0.8 ÷ (243)0.4.
n
a a Solution: (243)0.8 ÷ (243)0.4 = (243)0.8–0.4
8. n
= n , where ‘n’ is a +ve integer and ‘a’,
b b [∵ am ÷ an = am–n]
‘b’ are rational numbers = (243)0.4
mn
9. a = mn a = n m a , where ‘m’, ‘n’ are +ve 5
2

integers and ‘a’ is a +ve rational number = (3 ) 5 = 32 = 9.

10. nm k m
(a ) = n a k = mn a km , where ‘m’, ‘n’, ‘k’ Illustration 5 Find the value of (27)2/3 ÷ (64)–4/3
are +ve integers and ‘a’ is a +ve rational number Solution: (27)2/3 ÷ (64)–4/3 = (33)2/3 × (64)4/3
11. a a =a [∵ am ÷ b–n = am × bn]
= 3 × (43)4/3
2
12. a b = ab
= 9 × (44) = 9 × 256 = 2304.
2
13. ( a  b ) = a + b + 2 ab
( 2)( 4)
( 2)
2 Illustration 6 Find the value of ( 3)
14. ( a  b ) = a + b – 2 ab
( 4)
(2)( 2) ( 2)( 4)
15. a + b =c+ d ⇒ a = c and b = d. ( 2)( 4)  1  1
( 2)
Solution: ( 3) =   =  
 3  9

Chapter_04.indd 62 1/30/2016 3:01:25 PM


Simplification  63

( 4)
(2) 2 3
= (9) =
4 (2) 2  ( 3) 2
1
= (81)–4 =   2 3
 81 = =2– 3.
43
4
1 1
=  4  = . 7 3 5 2 7 3 5 2
3  3 (ii) =
48  18 4  3  32  2
2
5
Illustration 7  Find the value of x if 2 x  7 – 3 = 0.
7 3 5 2 7 3 5 2 4 3 3 2
Solution: We have = = 
5
4 3 3 2 4 3 3 2 4 3 3 2
2 x  7 – 3 = 0
5 (7 3  5 2)(4 3  3 2)
⇒ 2 x  7 = 3 =
(4 3  3 2)(4 3  3 2)
⇒ ( 5 2 x  7)5 = 35
7 34 3 7 33 2 5 2 4 3 5 2 3 2
⇒ 2x – 7 = 243 [Q ( n a )n = a] =
(4 3) 2  (3 2) 2
⇒ 2x = 250 or x = 125.
28 3  3  21 3  2  20 2  3  15 2  2
Illustration 8  Find the value of 5
64  5 512 =
16  3  9  2
Solution: 5 64  5 512 28  3  21  6  20 6  15  2
=
n 48  18
= 5
64  512  [ anb 
n
ab ]
84  (21  20) 6  30 114  41 6
5 2
8  83 = 5 5 n
a n = a] = = .
= 8 = 8. [Q 30 30

Illustration 9  Find the value of 3 2


729 . Illustration 13  If a and b are rational numbers, find the
values of a and b in the following equation:
6
Solution:   3 2 729 = 729  [ mn
a  mn a ]
3 2
= a + b 6.
= 6 6
3 = 3. [ n n
a  a] 3 2

3 2 3 2 3 2
7 5
(217 )5 Solution: = 
Illustration 10  Find the value of . 3 2 3 2 3 2
Solution: Given expression
5 3
(75 )3 ( 3  2) 2
=
( 3) 2  ( 2) 2
7
(21)7  nm p m n  p
=   (a )  a  322 3  2 52 6
5 (7)5   = =
32 1
21 n
= = 3.[Q a n = a] = 5 + 2 6
7
3 2
\ = a + b 6
Illustration 11  Find the value of 5  125. 3 2
Solution: 5  125 = 625  [ a  b  ab ] ⇒ 5 + 2 6 = a + b 6.
= 25. On equating rational and irrational parts, we get
Illustration 12  Simplify each of the following by rational- a = 5 and b = 2.
ising the denominator. Illustration 14  Find the value of
1 7 3 5 2
(i) (ii) ( 72  72  72  ...  ) ÷ ( 12  12  12  ...  )
2 3 48  18
1 1 2 3 Solution: Since 72 = 9 × 8
Solution: (i) = 
2 3 2 3 2 3 therefore, 72 + 72  72  ...  = 9

Chapter_04.indd 63 1/30/2016 12:02:55 PM


64 Chapter 4

Also, since 12 = 4 × 3 2. Comparison of Fractions


The following points are found useful while comparing two
therefore, 12  12  12  ... = 3.
or more fractions:

Thus, the given expression = 9 = 3. (a) If the denominators of the fractions are same,
3 the largest is one whose numerator is the largest.
Fractions
Illustration 16 Which is the largest fraction among
1. Continued Fraction
3 7 5
, and ?
2 8 8 8
Fractions of the form 7 
3
5 7
2 Solution: .
4 8
1
3
4 (b) If the numerators of the fractions are same, the
are called continued fractions. largest is one whose denominator is the smallest.
To simplify a continued fraction, we start from the
bottom and work upwards. Illustration 17 Which is the largest fraction among
1 5 5 5
Illustration 15 Simplify 3  . , and ?
7 2 7 9
4
5 5
9 Solution: .
2 2
6
3
1 1
(c) If neither the numerators nor denominators
Solution: 3  = 3 of the fractions are same then they are
7 7
4 4 converted into equivalent fractions of the
5 15
9 9 same denominator by taking the L.C.M. of
2 20
6 the denominators of the given fractions. Then,
3
 Multiply the numerator and denominator of the lowest  the fractions are compared according to (1).
 5 15 
 term by 3 to get .  Illustration 18 Which is the largest fraction among
  2 20 
  6    1 2 4 5
 5  , , and ?
2 3 5 8
1 1
= 3 = 3 Solution: L.C.M. of 2, 3, 5 and 8 = 120.
7 28
4 4 1 1  60 60
3 33 Then, = =
9 2 120
4 2  60
 Multiply the numerator and denominator  2 2  40
 = = 80
 of the lowest term 7 28  3 3  40 120
by 4 to get
  3 33  4 = 4  24 = 96
  9  
  5  24 120
 4  5
5 5  15
= 3
33 and, = = 75
160 8 8  15 120
Now, the denominator of these fractions are same and
 Multiply the numerator and denominator of the 
  the largest numerator is 96. Hence, the largest fraction is
 term 1 33 
by 33 to get 96 4
  28  160  , that is, .
  4+   120 5
 33 
(d) Two fractions can also be compared by cross
480  33 447 127
= = = 2 . multiplication method.
160 160 160

Chapter_04.indd 64 1/30/2016 12:02:56 PM


Simplification 65

6 5 Illustration 22 Which of the following fractions is the least?


Illustration 19 Which is greater or ?
13 7 2 4 6 8
, , ,
Solution: Step 1. Cross multiply the two given fractions 5 11 17 23
6 5
Increase in numerator 1
 ,
13 7 Solution: Since = 2 = is less
Increase in denominator 6 3
we get 6 × 7 = 42 and 13 × 5 = 65.
2 8
Step 2. Since 65 is greater than 42 and in 65, the than the first fraction , therefore, the last fraction is
5 23
numerator of 5 is included, \ 5 is greater than the least.
6 7 7
.
13 3. Inserting a fraction between two given fractions
To insert a fraction between two given fractions
(e) If the difference of the numerator and
a1 a
denominator of each of the given fractions be and 2 , the following steps may be useful:
b1 b2
same then the fraction of the largest numerator
is the smallest.
Step 1 The numerators of the two given
Illustration 20 Which one of the following fractions is the fractions are added to get the numerator of the
largest? resulting fraction, that is, a1 + a2.
2 3 5 9 Step 2 The denominators of the two given
, , and
3 4 6 10 fractions are added to get the denominator of the
Solution: Since in each of the given fractions the difference resulting fraction, that is, b1 + b2.
between the numerator and denominator is same and the a1  a2
9
Step 3. Resulting fraction = .
largest numerator is 9, therefore, the largest fraction is . b1  b2
10

x  na
(f) In the given fractions, x , x  a , x  2a , ..., , 4
Illustration 23 Insert one fraction between 2 and .
y y  b y  2 b y  nb
where a < b 5 7
Increase in numerator
Solution: Using the above method,
(a) If > first fraction,
Increase in denominator 2 24 4 2 6 4 2 1 4
, , = , , or , , .
the last value is the greatest 5 57 7 5 12 7 5 2 7
Increase in numerator
(b) If < first fraction, 9
Increase in denominator Illustration 24 Insert three fractions between 5 and .
7 11
the last value is the least.
Solution: Using the above method,
Increase in numerator
(c) If = first fraction, 5 59 9 5 14 9 5 7 9
Increase in denominator , , = , , or , , .
7 7  11 11 7 18 11 7 9 11
all values are equal.
Further,
Illustration 21 Which one the following fractions is the 5 57 7 79 9 5 12 7 16 9
, , , , = , , , ,
greatest? 7 7  9 9 9  11 11 7 16 9 20 11
3 4 5 6 7 5 3 7 4 9
, , , , . or , , , , .
8 11 14 17 20 7 4 9 5 11
Increase in numerator 1
Solutoin: Since, = is less than 9
Increase in denominator 3 Thus, the three fractions inserted between 5 and are
7 11
3 3
the first fraction , therefore, the first fraction is the 3 7 4
8 8 , and .
greatest. 4 9 5

Chapter_04.indd 65 1/30/2016 12:02:58 PM


66  Chapter 4

Multiple Choice QuestionS

2 (a) 22 × 53 × 72 (b) 22 × 35 × 72
 1 3 1
1. If  a   = 3, then what is the value of a  ? (c) 23 × 35 × 72 (d) 25 × 52 × 72
 a a3
[Based on MAT, 2001]
10 3
(a) (b) 0
3 21/2 31/3 41/4 34/3 57/5
8.
The value of  is
1/5 3/5
(c) 3 3 (d) 6 3 10 5 4 3/5 6
 [Based on MAT, 2003] (a) 5 (b) 6
(c) 10 (d) 15
x y ab
2.
If a = and b = then is equal to
x y x y ab  2.75  2.75  2.75  2.25  2.25  2.25 
9.
The value of  is
 2.75  2.75  2.75  2.25  2.25  2.25 
xy x2  y 2
(a) (b) (a) 0.30 (b) 0.50
x2  y 2 xy
2
(c) 3 (d) 5
x  y 
(c) (d)   [Based on MAT, 2005]
x y  x  y 
[Based on MAT, 2003]  1   1  1   1 x
If 1   1   1   ... 1   =
10. , then what is
 2   3   4   70  70
4ab x  2a x  2b the value of x?
3. If x = , then the value of  is equal
ab x  2a x  2b
(a) 69 (b) 35
to
1 (c) 20 (d) 1
(a) 0 (b) 1
9
(0.6)0  (0.1) 1
1 11. 1 3 1
is equal to
(c) 2 (d) None of these  3  3  1
9  3      
[Based on MAT, 2003] 2 2 3
3 1
x  4  x  10 5 (a)  (b) 
4.
Given  . The value of x is 2 2
x  4  x  10 2
2 3
331 (c) (d)
(a) 1 (b) 3 2
5
[Based on MAT, 2005]
263 17
(c) (d) 12.
What is the missing figure in the expression given below?
20 21
 [Based on MAT, 2002]
16 16 * 9 9 9
     1
7 7 7 7 7 7
1.073  1.073  0.927 (34 ) 4  96
5.
The value of + is (a) 1 (b) 7
1.073  0.927 (27)7  (3)9
(c) 4.57 (d) 32
1 1
(a) 2 (b) 2 [Based on MAT, 2000]
3 5
1 13.
If the numerator and the denominator of a proper fraction
(c) 2 (d) 3
9 are increased by the same quantity, then the resulting
fraction is
6. 96 +7, when divided by 8, would have a remainder of
(a) Always greater than the original fraction.
(a) 0 (b) 6
(c) 5 (d) None of these (b) Always less than the original fraction.
[Based on MAT, 2001] (c) Always equal to the original fraction.
7. H.C.F. of 3240, 3600 and a third number is 36 and their (d) None of the above.
L.C.M. is 24 × 35 × 52× 72. The third number is  [Based on MAT, 2001]

Chapter_04.indd 66 1/30/2016 3:03:07 PM


Simplification  67

21. Find the remainder when 721 + 722 + 723 + 724 is divided
a 3  b3 13 ab
14.
If = , then find by 25.
a 3  b3 14 ab
(a) 0 (b) 2
3 (c) 4 (d) 6
(a) (b) 1
2 [Based on FMS (Delhi), 2004]
(c) 2 (d) None of these
22. A four-digit number is formed, using digits 1, 2, 3 and 4,
[Based on FMS (Delhi), 2004]
without repeating any one of them. What is the sum of all
15. Arrange the following in ascending order of values: such possible numbers?
2 2 (a) 66600 (b) 66660
87  87  87  87 
,  , , 
83  83  89  89  (c) 66666 (d) 60000
[Based on FMS (Delhi), 2004]
2 2
87 87  87   87  23. How many multiples of 9 can be found which are less than
(a) , ,  , 
89 89  89   83  9999 and are perfect cubes?
2 2 (a) 5 (b) 6
87 87  87   87 
(b) , ,  ,  (c) 7 (d) 8
83 89  83   89 
[Based on FMS (Delhi), 2004]
2 2
 87  87 87  87  b
(c)   , , ,   24. If a 5b  a 2 then (a, b) could be
 89  89 83  83 
2 2 (a) (3, 4) (b) (2, 12)
 87  87 87  87 
(d)   , , ,   (c) (4, 18) (d) (6, 4)
 83  83 89  89 
[Based on IIT Joint Man. Ent. Test, 2004]
[Based on FMS (Delhi), 2004]
25. Manmohan spends one-fifth part of his money as pocket
2 2 money and four-fifths of the remainder in other affairs. If
16.
A man completes th of his journey by aeroplane, th he is left with `48 per month, what is the monthly income?
15 5
by train and the rest by taxi. What part of his journey does (a) `360 (b) `400
he complete by taxi? (c) `320 (d) `300
8 7
(a) (b) 26. What is the smallest number with which 1800 must be
15 15 multiplied to make it a perfect cube?
9 (a) 12 (b) 5
(c) (d) None of these
15 (c) 18 (d) 15
17.
If two-thirds part of a number is 96, what is the value of [Based on IIT Joint Man. Ent. Test, 2004]
three-fourths part of the same number? 27. If a man spends five-sixths part of money and then again
(a) 48 (b) 192 earns part of the remaining money, what half part of his
money is with him now?
(c) 108 (d) 72
(a) 1/2 (b) 1/4
18.
If the difference between four-fifths part and three-fourths (c) 2/3 (d) 3/4
part of a number is 4, what is the number?
28. The difference between the squares of two consecutive
(a) 60 (b) 100
odd integers is always divisible by
(c) 80 (d) 40
(a) 8 (b) 7
19. The unit digit of 3 × 4 × 63 × 74 × 82 × 95 is
6 7
(c) 6 (d) 3
(a) 6 (b) 5 [Based on Narsee Manjee Inst. of Man. Studies, 2003]
(c) 4 (d) 2 29. The number 39 + 312 + 315 + 3n is a perfect cube of an
20. The unit digit of the product of all the prime numbers integer for natural number n equalling
between 1 and (11)11 is (a) 12 (b) 13
(a) 6 (b) 5 (c) 14 (d) 15
(c) 4 (d) 0 [Based on IITTM, Gwalior, 2003]

Chapter_04.indd 67 1/30/2016 12:03:00 PM


68  Chapter 4

30. The number of ways, in which 8064 can be resolved as the s


product of two factors, is 37. The term (r2 + s)1/2 is approximately equal to r  .
2r
(a) 10 (b) 16 Which of the following is the closest approximation to
(c) 24 (d) 48 (85)1/2?
[Based on IITTM, Gwalior, 2003] (a) 9.06 (b) 9.34
31. Find the positive integer, which when added to the (c) 9.22 (d) 9.28
2 [Based on REC Tiruchiraplli, 2003]
numerator and denominator of , will result in a fraction
3 38.
The highest score in an inning was two-ninths of the
13 total score and the next highest was two-ninths of the
nearest to . remainder. These scores differ by 8 runs. What was the
15
total score in the innings?
(a) 6 (b) 5
(a) 162 (b) 152
(c) 4 (d) 3
(c) 142 (d) 132
[Based on IITTM, Gwalior, 2003]
2
39.
(1.06 + 0.04) – ? = 4 × 1.06 × 0.04
 
32. If x = 5  2 6, then  x  1 is equal to (a) 1.04 (b) 1.4
 x 
(c) 1.5 (d) Cannot be determined
(a) 2 3 (b) 3 885  885  885  115  115  115
40. =?
(c) 2 2 (d) None of these 885  885  115  115  885  115
[Based on IITTM, Gwalior, 2003] (a) 1000 (b) 770
1 1 1 (c) 885 (d) 115
33. The largest number in the sequence 1, 2 2 , 33 , 44 is [Based on FMS (Delhi), 2003]
1  1  6 1
(a) 1 (b) 2 2 41. If  x   = 3, then the value of  x  6  is
 x x
1 1
(a) 927 (b) 414
(c) 33 (c) 4 4
[Based on IITTM, Gwalior, 2003]
(c) 364 (d) 322
[Based on FMS (Delhi), 2003]
34.
What is the difference between the largest and the smallest
5 21 9 6 (2.3)3  0.027
fractions , , and ? 42. The value of is
8 35 16 7 (2.3) 2  0.69  0.09
33 11 (a) 2 (c) 3
(a) (b)
112 37 (d) 2.327 (d) 2.273
13 9 [Based on FMS (Delhi), 2003]
(c) (d)
41 35
 1  1 
43.
The value of 1   1  
35.
When one-fourth of a number is subtracted from one-third  x  1  x  2
of the same number, the remainder obtained is 12. The
number is  1  1 
× 1   1   is
(a) 144 (b) 72  x  3  x  4
(c) 120 (d) 63 1 1
(a) 1 + (b)
0 x5 x5
 1
Simplify    (64) 1/2  ( 32) 4/5
36.
 64  1 x5
(c) x + (d)
x5 x 1
1 3
(a) 17 (b) 17 [Based on FMS (Delhi), 2003]
8 8
2 2 ab ab
a b
7 7 44.
If = , then find the value of in terms
(c) 11 (d) 17 c2  d 2 cd ab
8 8
of c and d only.

Chapter_04.indd 68 1/30/2016 12:03:01 PM


Simplification  69

cd cd 51. Let (a/b) – (b/a) = x:y and (x – y) = (a/b) + (b/a), then
(a) (b) x is equal to
cd cd
(a) (a + b)/a (b) (a + b)/b
cd cd
(c) (d) (c) (a – b)/a (d) None of the above
cd cd
[Based on IIFT, 2003]
(x+y) (x–y)
1 x 4
1 x 1 2 52. If (3) = 81 and 81 = 3, then the values of x and y are
45.
The value of   =? (a) 17/8, 9/8 (b) 17/8, 15/8
1 x x x (1  x)
(c) 17/8, 11/8 (d) 15/8, 11/8
1
(a) (b) 1 + x  [Based on IIFT, 2003]
x
Given that x and y are real numbers, let P (x, y) = x2 – y2.
53.
(c) 1 –x2 (d) 1
Then P (3, P (3,4)) = ?
[Based on FMS (Delhi), 2003]
(a) – 40 (b) –7
x–1 x–3
46.
If (a/b) = (b/a) , then x is equal to (c) 40 (d) 7
(a) 1 (b) 1/2 [Based on IIFT, 2003]
(c) 7/2 (d) 2 54. Of the two-digit numbers (those from 10 to 99, both
[Based on FMS (Delhi), 2003]
inclusive), how many have a second digit greater than the
first digit?
1 1 (a) 38 (b) 40
47.
The expression +
1  x (b  a )  x ( c  a ) 1  x( a b)  x(c b) (c) 36 (d) 41
1 [Based on SCMHRD Ent. Exam., 2003]
+
1 x (b  c )
x (a c) 55. How much more is 1/2 of 2/3 than 3/4 of 1/3?
a–b–c
(a) 1/4 (b) 1/3

(a) x (b) 1
(c) 1/12 (d) 7/12
(c) 0 (d) None of these [Based on SCMHRD Ent. Exam., 2003]
[Based on FMS (Delhi), 2003] 56. Assuming that in Harappan era, rocks, stones and pebbles
1 1 were used for money. The following used to be the
48. n  m  is equal to currency valuations:1 rock = 7 stones, 1 stone = 7 pebbles.
1 a 1  amn
If a person used 6 rocks to purchase a cave that costs 5
(a) 0 (b) 1 rocks, 2 stones and 3 pebbles, then how much would the
(c) 1/2 (d) am+n change be?
[Based on FMS (Delhi), 2003] (a) 4 stones, 4 pebbles (b) 5 stones, 4 pebbles
49. Consider the following statements: (c) 1 rock, 5 stones, 4 pebbles
A. If a x= b, b y = c, c z = a, then xyz = 0.
(d) 5 stones, 5 pebbles
[Based on SCMHRD Ent. Exam., 2003]
B. If p = a x, q = a y, (p yq x) z = a2, then xyz = 1

C. x a = y b = z c, ab + bc + ca = 0, then xyz = 1.
 1  3  5  999 
57.
The value of is  2 −  2 −  2 −  ...  2 − 
 3  5  7  1001 
Of these statements:
(a) A and B are correct (b) B and C are correct 1003 1003
(a) (b)
3 1001
(c) Only A is correct (d) A and C are correct
[Based on FMS (Delhi), 2003] 1
(c) (d) None of these
50.
Simplify 1001
58. The sum of the two digits of a number is 13 and the
a1/2  a 1/2 1  a 1/2
 difference between the number (x) and that formed by
1 a 1 a reversing the digits (y) is 27, i.e., (x – y) is 27. Find the
a a 1 number.
(a) (b) (a) 72 (b) 58
a 1 2
(c) 27 (d) 85
2 2 [Bases on IMT Ghaziabad, 2002]
(c) (d)
a 1 1 a

Chapter_04.indd 69 1/30/2016 12:03:01 PM


70  Chapter 4

59. Find a positive number which when increased by 17 is (a) 8 (b) 17


equal to 60 times the reciprocal of the number. (c) 64 (d) 136
(a) 20 (b) 10
If x * y = (x + 2)2(y – 2) then 7 * 5 = ?
66.
(c) 3 (d) 17
(a) 234 (b) 243
[Based on IMT Ghaziabad, 2002]
(c) 343 (d) 423
60.
In an examination, a student was asked to find
three-fourths of a certain number. By mistake he found
5 1 5 1
three-fourths of it. His answer was 150 more than the 67.
If a = and b = , then the value of
correct answer. The given number is 5 1 5 1
(a) 180 (b) 240  a 2  ab  b 2 
 2 2
would be
(c) 280 (d) 290
 a  ab  b 
a 17 ab (a) 3/4 (b) 4/3
61.
If = , what is equal to?
ab 23 ab (c) 3/5 (d) 5/3
11 17 [Based on FMS (Delhi), 2002]
(a) (b)
23 32
68. If (x + y):(x – y) = 4:1, then (x2 + y2):(x2 – y2) = ?
23 23 (a) 25:9 (b) 16:1
(c) (d)
11 17 (c) 8:17 (d) 17:8
62. Four of the following five parts numbered (1), (2), (3), (4) [Based on FMS (Delhi), 2002]
and (5) are exactly equal. The number of the part which is If m and n are whole numbers such that mn = 121, then
69.
not equal to the remaining four parts will be your answer. (m – 1)n+1 = ?
(a) 36 × 15 + 27 × 13 (a) 10 (b) 102
(b) 53 × 4 + 64 ÷ 16 × 7 (c) 103 (d) 104
(c) 328 ÷ 41 × 21 + 9 × 23
70.
If we multiply a fraction by itself and divide the product
(d) 1024  11  16  7 by the square of its reciprocal, the fraction so obtained is
[Based on IRMA, 2002] 13
3 . The original fraction is
81
63. What approximate value should come in place of question
mark (?) in the following equation? 16 8
(a) (b)
9 9
95.9753.5 ÷ 16.0013.5 × 6.0021.5 ÷ 35.992 = ?
4 1
(a) 36 (2) 16 (c) (d)
3 3
(c) 96 (d) 6
71. A number consists of two digits whose sum is 7. If the
 [Based on IRMA, 2002] digits are reversed, then the number is increased by 27.
64.
If we multiply a fraction by itself and divide the product The number is
26 (a) 25 (b) 34
by its reciprocal, the fraction thus obtained is 18 . The
27 (c) 16 (d) 52
original fraction is
[Based on FMS (Delhi), 2002]
8 2
(a) (b) 2 72. Suppose a = 2/3 b, b = 2/3 c, and c = 2/3 d. What would be
27 3
the value of b as a fraction of d?
1
(c) 1 (d) None of these (a) 2/3 (b) 4/3
3
(c) 4/9 (d) 8/27
8 [Based on I.P. Univ., 2002]
65.
A boy was asked to multiply a given number by .
17 73. Find x and y:
8 x2 – xy = 4 and y2 – xy = –3
Instead, he divided the given number by and got the
17 (a) (4, 3) or (–4, –3) (b) (4, 1) or (–4, –1)
result 225 more than what he should have got if he had
(c) (3, 4) or (–3, –4) (d) x = y = 0
8
multiplied the number by . The given number was [Based on SCMHRD, 2002]
17

Chapter_04.indd 70 1/30/2016 12:03:02 PM


Simplification  71

ab  1  4
74.
If a * b = , find 3 * (3 * –1) 82. 1   1 is equal to
ab 1  7
 1 
 1
(a) –3 (b) –1.5 1
 3 
(c) 2/3 (d) 3
[Based on SCMHRD, 2002] 1 1
(a) 1 (b) 1
x
 9  8 
x1
2 3 4
75.
Solve     =
 4   27  3 1
(c) 1 (d) None of these
(a) 1 (b) 2 7
(c) 3 (d) 4 [Based on FMS (Delhi), 2006]
[Based on SCMHRD, 2002] 1 1 1 1
83.
The value of    correct to three
76. The expression 2 2.3 2.3.4 2.3.4.5
(2a – 3b) (4a + 0.5b) – a (8a – 11b) places of decimal is
(a) Is always positive for all a and b (a) 0.713 (b) 0.715
(b) Is always negative for all a and b (c) 0.717 (d) 0.718
(c) Is zero
(d) Depends on the values of a and b 72.9
84. 3 is equal to
[Based on SCMHRD, 2002] 0.4096
 3   1  = 7 3 , find the values of x and y. (a) 0.5625 (b) 5.625
77.
 2    y 
x 2 4 (c) 182 (d) 13.6
(a) (3, 19) (b) (3, 14) [Based on SSC (GL) Prel. Exam, 2000]
(c) (14, 3) (d) (24, 6) 85. If the square root of 5 is 2.236, then the square root of 80
3 1 5 1 equals = 2.236 times of
78.
The difference between the sum of 1 , 2 , 3 , 5
4 3 12 5 (a) 2 (b) 2.5
1
and 2 and the nearest whole number is (c) 4 (d) 5
6
[Based on SSC (GL) Prel. Exam, 2000]
2 13
(a) (b)
15 15 6
86.
A person was to multiply a fraction by . Instead, he
11 7
(c) (d) None of these divided and got an answer which exceeds the correct
60
1
answer by . The correct answer was
79. At the first stop on his route, a driver unloaded two-fifths 7
of the packages in his van. After he unloaded another three
6 36
packages at his next stop, half of the original number of (a) (b)
packages in the van remained. How many packages were 13 91
in the van before the first delivery? 7
(c) (d) None of these
(a) 10 (b) 25 13
(c) 30 (d) 36 87. The digit in the unit’s place in the cube root of 21952 is
[Based on REC Tiruchirapalli, 2002]
(a) 8 (b) 6
80. Which of the given numbers is the greatest?
(c) 4 (d) 2
(a) 6 3 5 (b) 8 3 2 [Based on SSC (GL) Prel. Exam, 2000]
3 3
(c) 2 130 (d) 900 1
[Based on REC Tiruchirapalli, 2002]
88. Given 5 = 2.2361, 3 = 1.7321, then is
5 3
81. If x = 2 + 22/3 + 21/3, then the value of x3 – 6x2 + 6x is equal to
(a) 3 (b) 2 (a) 1.984 (b) 1.9841
(c) 1 (d) None of these (c) 1.98 (d) 2
[Based on REC Tiruchirapalli, 2002] [Based on SSC (GL) Prel. Exam, 2000]

Chapter_04.indd 71 1/30/2016 12:03:03 PM


72  Chapter 4

1
89.
What fraction must be subtracted from the sum of and 96. Given 2 = 1.414, then the value of 8 + 2
32 – 3 128
4 4
1 1 + 50 is
to have an average of of all the three fractions? (a) 8.484 (b) 8.526
6 12
(c) 8.426 (d) 8.876
1 1
(a) (b) [Based on SSC (GL) Prel. Exam, 2003]
2 3
1 1 97. 3 0.004096 is equal to
(c) (d)
4 6 (a) 4 (b) 0.4
(c) 0.04 (d) 0.004
90. The square root of (2722 – 1282) is [Based on SSC (GL) Prel. Exam, 2003]
(a) 256 (b) 200
5
(c) 240 (d) 144 98. If 15 = 3.88, then what is the value of ?
3
[Based on SSC (GL) Prel. Exam, 2000] (a) 1.293 (b) 1.2934
(c) 1.29 (d) 1.295
91. The square root of 0.9 is equal to
[Based on SSC (GL) Prel. Exam, 2003]
(a) 0.3 (b) 0.03
(c) 0.94 (d) 0.81 99.
If the square root of 5625 is 75, then 5625 + 56.25 +
[Based on SSC (GL) Prel. Exam, 2000] 0.5625 =
92.
If we multiply a fraction by itself and divide the product (a) 9 (b) 83.25
26 (c) 82.80 (d) 8.325
by its reciprocal, the fraction thus obtained is 18 . The [Based on SSC (GL) Prel. Exam, 2002]
27
fraction is 100. What approximate value should come in place of the
8 2 question mark (?) ?
(a) (b) 2
27 3 36.0001 ÷ 5.9998 × ? = 108.0005
1 (a) 18 (b) 16
(c) 1 (d) None of these
3 (c) 256 (d) 325
 [Based on Bank of Maharashtra PO Exam, 2003]
0.342  0.684
93. The square root of is 1 1
0.000342  0.000171 2 2
101.
Simplify: 7 2 1 .
(a) 250 (b) 2500
1 1 1
(c) 2000 (d) 4000 2 1 2
4 7 1
2
[Based on SSC (GL) Prel. Exam, 2002] 1
2
2
94.
In a certain college, the number of girls is twice the
number of boys. One-fifth of the girls and one-eighth of 1 1
(a) (b) –
the boys took part in a social camp. What part of the total 2 8
number of students took part in the camp?
1 1
7 7 (c) – (d) –
(a) (b) 6 4
40 80
1 1
2 1 102.
How many s are there in 37 ?
(c) (d) 8 2
12 24 (a) 300 (b) 400
3 (c) 500 (d) Cannot be determined
95. If cube root of 175616 is 56, then the value of 175.616
+ 3
0.175616 + 3
0.000175616 is equal to 3.001
103. 10000 + of 1891.992 = ?
(a) 0.168 (b) 62.16 4.987
(a) 2500 (b) 1230
(c) 6.216 (d) 6.116
(c) 1640 (d) 1525
[Based on SSC (GL) Prel. Exam, 2002]
[Based on Canara Bank PO, 2003]

Chapter_04.indd 72 1/30/2016 12:03:04 PM


Simplification  73

7 111.
If (x – 3)(2x + 1) = 0, then possible values of 2x + 1 are
104.
Which of the following fractions is less than and (a) 0 only (b) 0 and 3
8
1 1
greater than ? (c) − and 3 (d) 0 and 7
3 2
1 23 [Based on MAT, 1997]
(a) (b)
4 24 1  1  1  1 1 1  
112. 7   2  1  ? 1     = 3
11 17 2  4  4  2 3 6  
(c) (d)
12 24 1 3
(a) (b)
105.
In a college, one-fifths of the girls and one-eighths of the 4 4
boys took part in a social camp. What of the total number 4
of students in the college took part in the camp? (c) (d) None of these
3
13 13 113.
Simplify 2 ÷ [2 + 2 ÷ {2 + 2 ÷ (2 + 2 ÷ 3)}]
(a) (b)
40 80 (a) 13/15 (b) 17/15
2 (c) 11/15 (d) None of these
(c) (d) Data inadequate
13
3 3  3 3 1 2 2
114.
Simplify  of  2  2  + ÷ 1 
10 7  10 5 5 5 7
 1 1 1 1 2 1 7  2 1 
106. 7   of   2  1 of 1  1   = ?
 2 2 2 4 5 3 8  5 3  (a) 1 (b) 2
(c) 0 (d) 3
1 1
(a) 3 (b) 2
5 24   1 
115.
1  1  1  1  1    = ?
  3 
1
(c) 4 (d) None of these
30 7 2
(a) (b)
5 3
1 1 1
107.
The value of 1 +   up to four places 4
4  3 4  32 4  33 (c) (d) None of these
of decimals is 5

(a) 1.1202 (b) 1.1203 9 4 3 2


116. 48  12   of  of  = ?
(c) 1.1204 (d) None of these 8 3 4 3
108.
Which of the following is true? (a) 9 (b) 12
A. 99/101 < 97/99 < 95/97 (c) 15 (d) None of these
B. 95/97 < 97/99 < 99/101 If ax = b, by = c and cz = a, then the value of xyz = ?
117.
2 2 2
C. (95/97) > (97/99) > (99/101) (a) 0 (b) 1
D. (99/101)2 > (97/99)2 > (95/97)2 (c) –1 (d) 2
(a) Only A (b) Only B  [Based on MAT, 1997]
(c) B and C (d) B and D 118.
If ax = b, by = c and cz = a , then xyz is equal to
[Based on MAT (Dec), 2006] (a) 0 (b) –1
4 (c) 1 (d) a + b + c
109.
If G = H + , then L equals
L [Based on MAT, 1998]
2 2
(a) 4/(G – H) (b) 4(G – H) 2 2 2
119.
If x = y and y = z and z = w, what fraction of w
(c) 4/(G2 – H2) (d) 4(G2 – H2) 3 3 3
[Based on MAT, 1997] is y?
1 1 1 8 4
110.
If = + , then z equals (a) (b)
x y z 27 9
(a) xy/(x – y) (b) x – y 2 4
(c) (d)
(c) xy/(y – x) (d) (x – y)/xy 3 3
[Based on MAT, 1997] [Based on MAT, 1998]

Chapter_04.indd 73 1/30/2016 12:03:05 PM


74  Chapter 4

Find the remainder when 721 + 722 + 723 + 724 is divided


120. 1 1
by 25. 130.
It 2 p + = 4 the value of p3 + 3 is
p 8p
(a) 0 (b) 2
(a) 4 (b) 5
(c) 4 (d) 6 (c) 8 (d) 15
121.
A four-digit number is formed, using digits 1, 2, 3 and 4, [Based on SSC (GL), 2010]
without repeating any one of them. What is the sum of all 2 2
131.
The value of (1/x ) + (1/y ), where x = 2 + 3 and
such possible numbers?
(a) 66600 (b) 66660 y = 2 – 3 , is
(a) 14 (b) 12
(c) 66666 (d) 60000
(c) 10 (d) 16
122.
How many multiples of 9 can be found which are less than [Based on MAT, 1999]
9999 and are perfect cubes? 9
(a) 5 (b) 6 If 9 x =
132. then x is
3x
(c) 7 (d) 8 (a) 1/3 (b) 2/3
(c) 3 (d) 4/3
(3 2 − 3)
123.
If 6 = 2.45 , then the value of is [Based on MAT, 1999]
(3 2 + 2) 5a + 3b 23
133.
If = , then the value of a:b is
(a) 0.40 (b) 0.41 2a − 3b 5
(c) 0.42 (d) 0.44 (a) 2:1 (b) 1:4
[Based on MAT, 1998] (c) 1:2 (d) 4:1
124.
Three-fourths of 68 is less than two-thirds of 114 by [Based on MAT, 1999]
(a) 12 (b) 25 (0.1 × 0.01 × 0.001 × 107) is equal to
134.
(c) 35 (d) 48 1
(a) 100 (b)
[Based on MAT, 1998] 10
125.
What is the smallest number with which 1800 must be 1
(c) (d) 10
multiplied to make it a perfect cube? 100
[Based on SSC (GL), 2010]
(a) 12 (b) 5
5

( )
(c) 18 (d) 15  −5/3 
Simplified form of  5 x −3/5
135.  is
The remainder when 87185 is divided by 7 is
126.  
(a) 5 (b) 1 (a) x5 (b) x–5
(c) 6 (d) 4 1
(c) x (d)
127. 9 12 15
The number 3 + 3 + 3 + 3n is a perfect cube of an x
integer for natural number n equalling [Based on SSC (GL), 2010]
(a) 12 (b) 13
 1  1  1   1 
(c) 14 (d) 15 136. 1 − 1 − 1 −  ... 1 −  is equal to
 3  4  5   25 
128.
The rank of 2/9 in the following fraction when expressed
in ascending order is –2/3, 1/7, 0, 4/9, 2/9, 14/15, 9/11. 2 1
(a) (b)
25 25
(a) 4 (b) 5
(c) 6 (d) 9 19 1
(c) 1 (d)
25 325
p
129.
1.27 in the form is equal to [Based on SSC (GL), 2010]
q
3 2 4 3 6
127 73 137.
The value of − + is
(a) (b) 3+ 6 6+ 2 3+ 2
100 100
(a) 4 (b) 0
14 11
(c) (d)
11 14 (c) 12 (d) 3 6
[Based on SSC (GL), 2010] [Based on SSC (GL), 2011]

Chapter_04.indd 74 1/30/2016 12:03:06 PM


Simplification  75

138.
The sum of the squares of two numbers is 3341 and the 1 17
difference of their squares is 891. The numbers are 145.
If x is an integer such that x   , then the value of
x 4
(a) 25, 46 (b) 35, 46
1
(c) 25, 36 (d) None of these x– is
x
[Based on FMS (MS), 2006]
13
(a) 4 (b)
 1  1  1  1 4
139. 1   1   1   ... 1   is equal to
3 4 5 n 15 1
(c) (d)
1 2 4 4
(a) (b) [Based on FMS, 2006]
n n
2 2(n  1) x 1 x 1
(c) (d) 146.
If  2 , then the value of x is

n(n  1) n 1 x x 6
[Based on FMS (MS), 2006] 6 4 3 2
(a) or (b) or
1 2 13 13 2 3
2 −1 5 2 9 4
140.
The value of 3 11 is (c) or (d) or
1 2 3 13 13
3+
1 [Based on FMS, 2009]
3+
1 ab cd
3+
3 147.
If  , then
bc d a
38 109 (a) a must equal c
(a) (b)
109 38 (b) a + b + c + d must equal zero
116 (c) either a = c or a + b + c + d = 0, or both
(c) 1 (d)
109 (d) a(b + c + d) = c(a + b + d)
[Based on SSC (GL), 2011] [Based on FMS, 2009]
148.
Let r be the result of doubling both the base and the
1 1 1
141.
The value of 3 + + + is exponent of ab, b ¹ 0. If r equals the product of ab by xb,
3 3+ 3 3 −3 then x equals
(a) 3 + 3 (b) 3 (a) 2a (b) 4a
(c) 2 (d) 4
(c) 1 (d) 0
[Based on FMS, 2010]
[Based on SSC (GL), 2011]
m 4 r 9 3mr  nt
 1  1 149.
If  and  , the value of is
If  x 4  4  322, the value of
142.
 x   is n 3 t 14 4nt  7 mr
 x   x
1 11
(a) 4 (b) 6 (a) –5 (b) 
2 14
(c) 8 (d) 3 2 1 11
[Based on FMS, 2005] (c) 1 (d)
4 14
143.
The value of [Based on FMS, 2011]
4 4
( x a /xb )( a  b )  ( xb /x c )(b  c )  ( x c /x a )( c  a ) is Simplify  3 6 a9   6 3 a9  ; the result is
150.

(a) a16 (b) a12
(a) 0 (b) x abc
(c) a8 (d) a4
a+b+c
(c) x (d) 1 [Based on FMS, 2011]
[Based on FMS, 2006]
1 1
144.
The value of [1/(216) – 213 + 1/(256) – 314 + 1/(243) 151.
The expression 1   equals
1 3 1 3
– 1/5] is
(a) 107 (b) 105 (a) 1  3 (b) 1
(c) 103 (d) None of these (c)  3 (d) 3
[Based on FMS, 2006] [Based on FMS, 2011]

Chapter_04.indd 75 1/30/2016 12:03:07 PM


76  Chapter 4

1 1 5 7
152.
The expression 2  2   equals (c) (d)
2 2 22 7 5
[Based on SSC (GL), 2010]
(a) 2 (b) 2  2
p
(c) 2  2 (d) 2 2 159.
The number 0.121212 ... in the form is equal to
q
[Based on FMS, 2011]
4 2
2 n4
 2(2 ) n (a) (b)
153. when simplified is 11 11
2(2n  3 )
4 2
1 (c) (d)
n1
(a) 2  (b) –2n1 33 33
8 [Based on SSC (GL), 2010]
7
(c) 1 – 2n (d) 3 2 1
160. 3 + 4 − 3 = ?
8 4 5 8
[Based on FMS, 2011]
1 1
2 (a) 4 (b) 5
154.
If x + = 10 then the value of x is 40 40
4
3+
7 1 3
5+ (c) 6 (d) 5
6 40 40
1276 53 [Based on Bank of Baroda PO, 2010]
(a) (b)
135 6
161. 52 × 14 − 6 × 7 + (4)? = 18
(c) 4.35 (d) 9
[Based on SSC (GL), 2011]
(a) 1 (b) 3
(c) 4 (d) None of these
155.
If u1
 3, u2
 3 , u3
3 3 3 3 etc, u10 : u9 is [Based on Bank of Baroda PO, 2010]
162. 67.99% of 1401 – 13.99% of 1299 = ?
1
(a) 3 (b) 310 (a) 700 (b) 720
(c) 770 (d) 800
1
[Based on Bank of Baroda PO Exam, 2011]
(c) 3 20 (d) None of these
[Based on IIFT, 2007]
2
 24  399 41
156.
The value of 3 +
3 163.   × ÷ =?
1  9  39 899
3+
1 (a) 1600 (b) 1650
3+
3 (c) 1700 (d) 1550
40 43 [Based on Bank of Baroda PO Exam, 2011]
(a) (b)
11 11 164. (15 × 0.40)4 ÷ (1080 ÷ 30)4 × (27 × 8)4 = (3 × 2)?+5
46 41 (a) 8 (b) 3
(c) (d) (c) 12 (d) 16
11 11
[Based on Bank of Baroda PO Exam, 2011]
[Based on SSC (GL), 2011]

5 +1 1 1 5 (?) 2 5
157.
If x = , then, the value of 5x2 – 5x – 1 is 165. 3 + 2 − 1 = +1
4 2 6 10 12
5 −1
(a) 0 (b) 3 (a) 25 (b) 5
(c) 4 (d) 5 (c) 625 (d) 5
[Based on SSC (GL), 2011] [Based on Bank of Baroda PO Exam, 2011]
3+ x + 3− x 166. 92 × 576 ÷ 2 1296 = (?)3 + 49
158.
If = 2, then x is equal to
3+ x − 3− x (a) 3 (b) (9)2
5 12 (c) 9 (d) 27
(a) (b)
12 5 [Based on Bank of Baroda PO Exam, 2011]

Chapter_04.indd 76 1/30/2016 3:06:05 PM


Simplification  77

1 1 171. The value of expression


167. of (92)% of 1 of (650) = 85 + ?
6 23  xb 
b+c−a
 xc   xa 
c+a−b a+b−c

(a) 18 (b) 21  c  a  b
x  x  x 
(c) 19 (d) 28
[Based on Bank of Baroda PO Exam, 2011]
(a) xab + bc + ca (b) 1
(c) xabc (d) xa + b + c
168.
What is the value of x that would satisfy
[Based on NMAT, 2005]
x x
( 3 + 2) + ( 3 − 2) = 10
172. When simplified, the product

{ }{ }{ } { }
1 1 3 5 999
(a) ± 2 (b) − 2− 2− 2 − ... 2 −
3 3 5 7 1001
(c) 4 (d) – 4
is equal to
[Based on ATMA, 2008]
1003 1001
169. Which of the following is/are true? (a) (b)
13 13
I. (xb–c)a (xc–a)b (xa–b)c = 1
991
3 19 16 (c) (d) None of these
II. Fractions , of are in descending order. 1001
18 20 19 [Based on NMAT, 2005]
III. If log (x + 1) – log (x – 1) = log 2, then x = 3 173. Seema bought 20 pens, 8 packets of wax colours, 6
(a) I and II (b) Only I calculators and 7 pencil boxes. The price of one pen is
(c) I and III (d) II and III `7, one packet of wax colour is `22, one calculator is
[Based on ATMA, 2005] `175 and one pencil box is `14 more than the combined
price of one pen and one packet of wax colours. How
1 much amount did Seema pay to the shopkeeper?
170. Find the value of x4 + , if x = 3 + 2 2
x4 (a) `1491 (b) `1725
(a) 1154 (b) 1024 (c) `1667 (d) `1527
(c) 1734 (d) None of these [Based on IBPS Bank PO, 2011]
[Based on NMAT, 2006]

EXPLANATORY ANSWERS

2
 1 1 6ab  2a 2 6b 2  2ab
1.
(b)  a   = 3 ⇒ a  = 3
a a x  2a x  2b ab ab
3.
(a)  = 2

3
x  2a x  2b 2ab  2a 2ab  2b 2
 1 3 1  1
\  a   = a  3  3  a   ab ab
a a a
2a (3b  a ) 2b (3b  a )
1 = 
⇒       3 3  a3  3 3 2a (b  a ) 2b (a  b)
a3
3b  a 3b  a
1 =  = 0.
⇒ 3
a  = 0. ba ba
a3 4.
(c) Given expression
( x  4  x  10) 2 25
x y ⇒ =
2.
(a) a= , b = ( x  4)  ( x  10) 4
x y x y
x  4  x  10  2 x 2  6 x  40 5
xy x2  y 2 ⇒ =
\ ab = , a+b= 14 2
x2  y 2 x2  y 2
⇒ 2 x  2 x 2  6 x  40 = 41
ab xy
\ = .
a  b x2  y 2 ⇒ (2x – 41)2 = [ 2 x 2  6 x  40]2

Chapter_04.indd 77 1/30/2016 12:03:09 PM


78  Chapter 4

⇒ 4x2 + 1681 – 164x = 4 (x2 – 6x – 40)


a 3  b3
9.
(b) By using =a–b
= 4x2 – 24x – 160 a  ab  b 2
2

⇒ 140x = 1841 (2.75)3  (2.25)3


= 2.75 – 2.25 = 0.50.
⇒ x =
263
. (2.75)  2.75  2.25  (2.25) 2
2

20
1 1 1 1 x
4 4 6
(d) 1   1   1   ... 1   =
10.
1.073  1.073  0.927  0.927 (3 )  (9)  2   3  4   70  70
5.
(c) +
1.073  0.927 (27)7  (3)9 1 2 3 69 x
\    ... =
2 2 4 4 2 6 2 3 4 70 70
(1.073)  (0.927) (3 )  (3 )
=  3 7 1 x
1.073  0.927 (3 )  (3)9 \ =
70 70
(1.073  0.927)(1.073  0.927) 328 \ x = 1
=  30
1.073  0.927 3 1
1
1  
1 1 1  10  1  10
=2+ =2+ =2 11.
(a) = 3 3
32 9 9  23   3  3 2 3
 3   2   ( 3)  3
  3 23
(a) Consider 9, 92, 93, 94, 96. Each of the these numbers,
6.
when divided by 8, will leave a remainder of 1. 9 3
= =  .
\ 96 + 7, when divided by 8, will leave a remainder 93 2
of 0.
7.
(b) The product of the numbers = H.C.F × L.C.M. * 9 256 81
12.
(d)   =   1
7 7 49 49
Let the third number be x.
⇒ – * × 9 = – 256 – 81 + 49
\ 3240 × 3600 × x = 36 × 24 × 35 × 52 × 72
⇒ * = 32.
36  24  35  52  7 2
⇒ x =
(23  34  5)  (24  32  52 ) 1 11 1 2
13.
(a) < ⇒ <
2 2 1 2 3
 3240  23  34  5 
  2 2 1 2 3
 3600  24  32  52  < ⇒ <
3 3 1 3 4
(22  32 )  24  35  52  7 2 2 2 1 2 1
⇒ x = < ⇒ < .
23  34  5  24  32  52 5 5 1 5 2
26  37  52  7 2 a 3  b3 13
= 7 6 3 14.
(c) =
2 3 5 a 3  b3 14
= 22 × 35 × 72. ⇒ 14a3 – 14b3 = 13a3 + 13b3
21/2  31/3  41/4 34/3  57/5 a
8.
(c)  ⇒ a3 = 27b3 ⇒ =3
10 1/5
5 3/5
4 3/5
6 b
a
21/2  31/3  (22 )1/4  101/5 34/3  43/5 1
ab 4
=  \ = b = = 2.
53/5 57/5  6 ab a 2
1
b
21/2  31/3  21/2  21/5  51/5 57/5  2  3
= 3/5
 2
5 34/3  26/5  87 
15.
(c)   = 0.9555
1 1 1 6 1 4 1 3 7
89
   1  1  
= 2 2 2 5 5  33 3  55 5 5 87
= 0.9775
1 0 1
= 2 × 3 × 5 = 2 × 5 = 10 89

Chapter_04.indd 78 1/30/2016 12:03:11 PM


Simplification  79

87 22.
(b) Sum of the digits in the thousand’s place
= 1.0481
83 = 6000 + 12000 + 18000 + 24000 = 60000
Sum of the digits in the hundred’s place
2
 87  = 600 + 1200 + 1800 + 2400 = 6000
  = 1.0985
83
Sum of the digits in the ten’s place
If x < 1 ⇒ x2 < x = 60 + 120 + 180 + 240 = 600
If x > 1 ⇒ x2 > x. Sum of the digits in the unit’s place
16. (b) Journey completed by aeroplane and train = 6 + 12 + 18 + 24 = 60.
2 2 26 8 23.
(b) 27, 216, 729, 1728, 3375, 5832, 9261.
=  = =
15 5 15 15
24.
(b) Put a = 2, b = 12 in ab = 5b + a2
8 7
\ Remaining journey = 1 – =
15 15 \ 212 = 5 × 12 + 4 = 64
⇒ 26 = 64, which is true.
7
\ He completed th part of his journey by taxi. 25. (d) Let the monthly income of Manmohan be `1
15
1 1
2 \ Pocket money = of `1 = `
17. (c) Q part = 96 5 5
3 1 4
and remainder = 1 – =`
3 3 3 5 5
\ part = 96 × × = 108
4 2 4 4 4 16
\ Other expenses =of ` = `
18. (c) Let the number be 1 5 5 25
4 4 3 3 4 16 4
\ Saving = – =`
\ of 1 =  and,  of 1 = 5 25 25
5 5 4 4
4
4 3 1 \ Monthly income = 48 ¸ = `300
\ Difference = – = 25
5 4 20
26. (d) 1800 = 2 × 2 × 2 × 3 × 3 × 5 × 5
1 \ 1800 must be multiplied by 3 × 5 = 15 so that the
\ Number = 4 ÷ = 80
20 resulting number becomes a perfect cube.
(a) The unit digit of 36 is 9
19. 27.
(b) Let the money with the man at first be `1
7
The unit digit of 4 is 4 5 5
\ Money spent = of 1 = `
3
The unit digit of 6 is 6 6 6
The unit digit of 74 is 1 5 1
\ Remaining money = 1 – =`
The unit digit of 82 is 4 6 6
The unit digit of 15 is 9 1 1 1
and money earned = of ` = `
Therefore the unit digit of the given expression is 6 2 6 12
(Since 9 × 4 × 6 × 1 × 4 × 9 = 7776). \ Total money with him now
20.
(d) The set of prime numbers. 1 1 3 1
= + = ` =`
S = {2, 3, 5, 7, 11, 13, ...}. 6 12 12 4
Since there is one 5 and one 2 which gives 10 after 1
\ th part of his money is with him now.
multiplying mutually, it means the unit digit will be 4
zero.
(a) (2x + 3)2 – (2x + 1)2 = 4x2 + 12x + 9 – (4x2 + 4x + 1)
28.
21 2 3
21.
(a) Given expression = 7  (1 + 7 + 7 + 7 ) = 8x + 8 = 8 (x + 1).
= 721 × 400, 29.
(c) For n = 14, 39 (1 + 33 + 36 + 35)
which is completely divisible by 25. = 39 (1 + 27 + 729 + 243)
Hence, remainder is zero. = 39 × 103.

Chapter_04.indd 79 1/30/2016 12:03:12 PM


80  Chapter 4

30.
(c) 8064 is divisible by 1, 2, 3, 4, 6, 7, 8, 9, 12, 14, 16, 18, = 1 + (82)–1/2 + (–1 × 32)4/5
21, 24, 28, 32, 36, 42, 48, 56, 63, 64, 72, 84. = 1 + 8–1 + [(–1)4/5 × (32)4/5]
2K 13 = 1 + 8–1 + [((–1)2)2/5 × (25)4/5]
31.
(b) Let = ⇒ 2K = 9 ⇒ K = 4.5 ≈ 5.
3 K 15 1 1
=1+ + [1 × 16] = 17
8 8
32.
(d) x = 5  2 6
37.
(c) r = 9, s = 4
⇒ x = 52 6 = A  B , say s 4 2
\ 851/2 = r  = 9  = 9 = 9.22.
\ x = 5  2 6 = A + B + 2 AB 2r 18 9

⇒ A + B = 5, AB = 6 38.
(a) Let the total score be x runs, such that
⇒ A = 3, B = 2 2 2  2  2 2 7
x    x  x = 8 or, x   x = 8
9 9  9  9 9 9
⇒ x = 3 2
2 2
or x  = 8 or, x = 162
x 1 3  2 1 3 2 9 9
\ = 
x 3 2 3 2 39. (a) Putting x for ? and solving
(1.06 + 0.04)2 – x = 4 × 1.06 × 0.04
= 1  3  2 = 1  ( 3  2).
Here, 1.06 = a and 0.04 = b
2
33.
(c) 11/2 = 1, (21/2)12 = 26 = 64 \ (a + b) – x = 4ab
(31/3)12 = 34 = 81, (41/4)12 = 43 = 64. \ x = (a + b)2 – 4ab = (a – b)2
= (1.06 – 0.04)2
34.
(a) L.C.M. of 7, 8, 16 and 35 = 560
= (1.02)2 = 1.0404
5 5 × 70 350
\ = =
8 8 × 70 560
21 21 × 16 336 40.
(a) a3 + b3 = (a + b) (a2 + b2 – ab)
= = Put a = 885, b = 115, we have
35 35 × 16 560
a 3  b3
9 9 × 35 315 ? = 2 = a + b = 1000.
= = a  b 2  ab
16 16 × 35 560
3
6 6 × 80 480 1  1
and, = = 41.
(d) x = 3 ⇒  x   = 27
7 7 × 80 560 x  x
\ Difference between the largest and the smallest 1 1 1
⇒ x3  +3×x×  x   = 27
fractions x3 x x
6 9 480 315 1
= − = − ⇒ x3  = 18
7 16 560 560 x3
2
165  3 1
33 ⇒  x  3  = 324
= = x
560 112
6 1
35.
(a) Let the number be 1 ⇒ x  6 + 2 = 324
x
1 1 1 1 1
\ of 1 =  and,  of 1 = ⇒ x 6  6 = 322.
3 3 4 4 x
1 1 43 1
\  = = (2.3)3  0.027
3 4 12 12 42.
(a)
(2.3) 2  0.69  0.09
1
\ Number 12 ÷ = 144
12 (2.3) 2  (.3)3
=
0
(2.3) 2  2.3  .3  (.3) 2
(a)  1   (64)  1/2  (  32) 4/5
36. = 2.3 – .3 = 2
 
64 [Q a3 – b3 = (a – b) (a2 + ab + b2)]

Chapter_04.indd 80 1/30/2016 12:03:14 PM


Simplification  81

43.
(d) Given expression
a1/2  a 1/2 1  a 1/2
x2 x3 x4 x5 50.
(d) 
=    1 a 1 a
x 1 x  2 x  3 x  4
x5 a1/2  a 1/2 1  a 1/2
= . = 1/2 1/2
+
x 1 (1  a )(1  a ) 1  a1/2
a1/2  a 1/2  (1  a 1/2 )(1  a1/2 )
ab a 2  b2 2ab =
a 2  b2 (1  a1/2 )(1  a1/2 )
44.
(d) = or, 2 =
2
c d 2 cd c  d 2 2 cd
1/2
1/2
= a  a  1  a 1/2  a1/2  1 = 2
a 2  b 2  2ab c 2  d 2  2cd 1 a 1 a
or, =
a 2  b 2  2ab c 2  d 2  2cd
x a b a 2  b2
[by componendo and dividendo] 51.
(d) =  =
y b a ab
2 2
or,  a  b  =  c  d  a b a 2  b2
    x – y =  =
a  b cd b a ab
ab cd a 2  b2 a 2  b2 abx
\ = ⇒ x =  y =  2
ab cd ab ab a  b2

(1  x 2 )(1  x)(1  x) x 1  ab  a 2  b2
45.
(d) ? = ×  = 1. ⇒ x 1  2  =
(1  x) 1  x 2 x (1  x)
2
 a  b  ab
a 2  b2 (a 2  b 2 )
x 1 x 3  x 3 ⇒ x =  2 .
 a  b  a ab a  b 2  ab
46.
(d)   =   =  
b  a  b
\ x – 1 = –x + 3 ⇒ x = 2. 52.
(b) 3x+y = 81 = 34, 81x–y = 3
47.
(b) Given expression ⇒ x + y = 4, (34)(x–y) = 31
1 1 1 ⇒ x + y = 4, 4x – 4y = 1
= b c
+ a c
 b
= 1.
x x x x x xa
1 a  a 1 b  b 1 c  c 17 15
x x x x x x x = , y= .
8 8
1 1 am an
48.
(b)  =  = 1. 53.
(a) P (x, y) = x2 – y2
an am am  an am  an
1 m 1 n \ P (3, 4) = 32 – 42 = 9 – 16 = –7
a a
⇒ P (3, P (3, 4)) = P (3, –7)
49.
(b) (B) is correct.
= (3)2 – (–7)2 = 9 – 49
(p yq x)z = a2 ⇒ (axy × ayx)z = a2
= ­–40.
⇒ (a2xy)z = a2 54.
(c) 12 to 19:8, 23 to 29 : 7,
⇒ a2xyz = a2 34 to 39:6, 45 to 49 : 5,
⇒ 2xyz = 2 ⇒ xyz = 1 56 to 59:4, 67 to 69 : 3,
(C) is correct. 78 to 79:2, 89 : 1
xa = yb = zc Total : 36.
⇒ a log x = b log y = c log z 1 2 3 1 1 1 1
55.
(c) of  of =   .
\ ab + bc + ca = 0 2 3 4 3 3 4 12
log z log z log z log z 56.
(a) Cost of Cave
⇒ c c + c ccc = 0.
log x log y log y log x = 5 rocks + 2 stones + 3 pebbles
⇒ log z log z + log x log z + log y log z = 0 = 35 stones + 14 pebbles + 3 pebbles
⇒ log z + log x + log y = 0 = 245 pebbles + 14 pebbles + 3 pebbles
⇒ log (xyz) = 0 = log 1 ⇒ xyz = 1. = 262 pebbles

Chapter_04.indd 81 1/30/2016 3:08:03 PM


82  Chapter 4

    1 rock = 49 pebbles


963.5  61.5 1
\ To use 6 rocks, it requires 294 pebbles 63.
(d) Given expression = 
163.5 362
\ Change required = 294 – 262 = 32 pebbles
65
= 4 stones and 4 pebbles. = = 6.
64
57.
(a) Given product
1 26 512
5 7 9 1003 1003 64.
(b) x × x ÷ = 18 or, x3 =
=    ...  = x 27 27
3 5 7 1001 3 3
 8 8 2
58.
(d) Let the number be 10p + q \ x3 =   and so x = = 2
 3 3 3
\ p + q = 13 and (10p + q) – (10q + p) = 27
(x = 10p + q, y = 10q + p) 17 8 225
65.
(d) x × –x× = 225 or, x = 225
i.e., 9p – 9q = 27,  i.e.,   p – q = 3 8 17 136
\ x = 136
\ p = 8, q = 5
66.
(b) Substituting x = 7 and y = 5, we get
Hence the required number = 85.
7 * 5 = (7 + 2)2 (5 – 2) = (9)2 × 3 = 243
59.
(c) Let the number be x
60 5  1 ( 5  1) 2 62 5 3 5
\ x + 17 = ⇒ x2 + 17x – 60 = 0 67.
(b) a =  = 
x 5 1 4 4 2
⇒ (x + 20) (x – 3) = 0
5  1 ( 5  1) 2 62 5 3 5
⇒ x = 3 (x > 0). b =  = 
5 1 4 4 2
3 3 15
60.
(c) x  x = 150 or, x = 150 3 5
2
4 14 28 14  6 5
a2 =   
150  28  2  4
\ x = = 280
15 2
3 5 14  6 5
b2 =   
a 17  2  4
61.
(c) Given that =
ab 23
i.e., if a = 17, then a + b = 23 or, b = 6  5  1  5  1 4
ab =    = 1 = 1.
a – b = 17 – 6 = 11  5  1  5  1
ab 23 a 2  ab  b 2 32 4
\ = \  = .
ab 2 2 24 3
11 a  ab  b
62.
(a)
(1) = 36 × 15 ÷ 27 × 13 x y 4 5
15 68.
(d) =  ⇒ x = y
= 36 × × 13 = 260 x y 1 3
27
25 2 34 y 2
(2) = 53 × 5 + 64 ÷ 16 × 7 \ x2 + y2 = y  y2 
64 9 9
= 53  4  7
16 25 2 16 y 2
x2 –­ y2 = y  y2 
= 212 + 28 = 240 9 9
328 x2  y 2 34 17
 21  9  23
(3) = \  = .
41 x2  y 2 16 8
= 168 + 9 × 8 (c) Given that mn = 121 ⇒ mn = 112
69.
= 168 + 72 = 240 Hence, m = 11, n = 2. Substituting these values
(4) = 1024  11  16  7 (m – 1)n+1 = (11 – 1)2+1 = 103 = 1000
= 32 × 11 – 112 70.
(c) Let x be the fraction
= 352 – 112 = 240 2 4
 1 13 256 4
x × x ÷   = 3 ⇒ x4 = =  
(5) = 17  18  121  6  x 81 81  3
= 17 × 18 – 11 × 6 4
\ x =
= 306 – 66 = 240. 3

Chapter_04.indd 82 1/30/2016 12:03:18 PM


Simplification  83

71.
(a) Let the number be 10x + y. 7 7 41 26 13
78.
(a) Sum =    
\ x + y = 7 4 3 12 5 6
and 10y + x = 10x + y + 27 105  140  205  312  130
=
⇒ y = 5, x = 2 60
\ The number = 10x + y = 25. 892 13
= = 14
60 15
2 2
72.
(c) b = c, c = d which is nearer to 15 than 14
3 3
13 2
2 2 4 Difference = 15 – 14 =
⇒ b = × d = d. 15 15
3 3 9
79. (c) Suppose there were x packages in the van before
73.
(a) x2 – xy = 4 ⇒ x (x – y) = 4 ...(1) delivery.
y2 – xy = –3 ⇒ y (y – x) = –3 ...(2) \ After first delivery, the number of packages in the van
Equations (1) and (2) 2 3
= x  x x
4 3 x 4 5 5
⇒          ⇒  .
x y y 3 After second delivery, the number of packages in the
van
74.
(a) Given expression 3 3 x  15
= x  3 
 3  ( 1)   3  5 5
= 3 *   3* 
 3  ( 1)   2 3 x  15 x
\        (Given)
 3  5 2
3  9
 2 ⇒ x = 30.
= = 2 = –3.
 3  3
3  80.
(a) (6  3 5)3 = 216 × 5 = 1080
 2 2
(8  3 2)3 = 512 × 2 = 1024
x x1
 9  8 2
75.
(d)      =
4 27 3 (2  3 130)3 = 8 × 130 = 1040

 3
2x
 2
3( x  1)
2 ( 3 900)3 = 900.
⇒     =
2  3 3
81. (b) x = 2 + 22/3 + 21/3
 2
2 x
 2
3 x 3
 2
1 ⇒ (x – 2) = 22/3 + 21/3
⇒   =  
 3   3  3 ⇒ (x – 2)3 = (22/3 + 21/3)3

⇒ – 2x + (3x – 3) = 1 ⇒ x = 4. = 4 + 2 + 3 × 22/3 × 21/3
[22/3 + 21/3]
3
(b) Given expression =  b 2 . Since b2 is always positive
76. = 6 + 3 × 2 (x – 2)
2
⇒ (x – 2)3 = 6 + 6x – 12 = 6x – 6
3 2
therefore,  b is always negative. ⇒ x3 – 8 – 6x (x – 2) = 6x – 6
2
⇒ x3 – 6x2 + 6x = 2.
77.
(c) Taking the quotient 2, y and 7, we get 2y = 7, which
gives the quotient as 3  1  4
82.
(d) 1  1
1  7
\ y = 3. Substituting the value of y, we get  1 
 1
3 1 1 
3  3 
2  3 = 7
x 2 4
 1  11  4  11
3 = 1   = 1   
7 3 7  7 7
4 = 2 3 ⇒ 2 3 = 2 3  1 
Now,  4
1 x 14 x
3 11 11 22
2 =  =
\ x = 14, y = 3 7 7 7

Chapter_04.indd 83 1/30/2016 12:03:19 PM


84  Chapter 4

1  a
3
 8
3
83.
(c) = 0.50000 ...(1) or
2   =  3 
b
1
= 0.16667 ...(2)  (divide (1) by 3) a 8 2
2.3 \ = =2
b 3 3
1
= 0.04167 ...(3)  (divide (2) by 4) 0.342  0.684 342  684  106
2.3.4 93.
(c) =
0.000342  0.000171 342  171
1
= 0.00833 ...(4)  (divide (3) by 5)   = 4 × 106
2.3.4.5
Square root of 4 × 106 = 2 × 103 = 2000.
Adding, we have 0.71667 or 0.717 up to three places.
94. (a) Let total number of students = x
.9  .9  .9  100 .9 2x
84.
(b) 3 = = 5.625. Number of girl students =
.16  .16  .16  100 .16 3
x
85.
(c) 80 = 16  5 = 4 = 4 5 × 2.236. Number of boy students =
3
86.
(b) Let x be the fraction Number of girls who took part in camp
7 6 1 6 1  2x 
x  x = ⇒x= 2
6 7 7 13 =   = x
5 3  15
6 6 6 36 Number of boys who took part in camp
Correct answer = x =  =
7 7 13 91 1  x x
=   =
87.
(a) 21952 = 4 × 4 × 4 × 7 × 7 × 7 8  3 24
3
Total number of students who took part in camp
\ 21952 = 4 × 7 = 28.
2 x  16  5 
= x =  x
1 1 5 3 15 24  120 
88.
(b) = 
5 3 5 3 5 3 7
= x
5 3 2.2361  1.7321 40
= =
53 2
3 175616 56
3.9682 95.
(c) 175.616 = 3 = = 5.6
= = 1.9841. 10000 10
2
3 175616 56
1 1 1 0.175616 = 3 = = 0.56
89.
(d) Let   x = 3 × . 1000000 100
4 6 12
1 1 1 1 175616 56
Then,   x = or, x = 3 0.000175616 = 3=
4 6 4 6 1000000000 1000
= 0.056
90.
(c) 2722  1282 = (272  128)(272  128) \ Required answer = 5.6 + 0.56 + 0.056 = 6.216.

= 400  144 (a) 8  2 32  3 128  4 50


96.
= 20 × 12 = 240. = 2 2 +2× 4 2 –3× 8 2 +4× 5 2
9 3 10 = 2 2  8 2  24 2  20 2
91.
(c) 0.9 = = 
10 10 10
= 6 2 = 6 × 1.414 = 8.484.
3  3.16 9.48
= = = 0.94.
10 10 97.
(b) 3
0.004096 = ((0.16)3 )1/3
a = 0.16 = 0.4.
92.
(b) Let the fraction be , then
b
 a a b 26 512 5 53 15 3.88
98.
(a) = = = = 1.293.
    = 18 27 = 27 3 33 3 3
b b a

Chapter_04.indd 84 1/30/2016 3:09:15 PM


Simplification  85

106.
(c) Given expression
99.
(b) 5625 + 56.25 + 0.5625
15 1 1 2 7 15  7 4
=      of   
5625 5625 75 75 2 2 8 5 3 8  5 3
= 5625   = 75  
100 1000 10 100 15 2 7 15 1
= 4   of
= 75 + 7.5 + 0.75 = 83.25. 2 5 3 8 15
36 15 2 7 1
100.
(d)  ? = 108 = 4  
6 2 5 3 8
108 15 2 7 8
or   ? = = 4  
6 2 5 3 1
or ?   = 18 15 112 23 112
= 4 = 
or   ? = 324 ≈ 325. 2 15 2 15
101.
(d) Given expression 121 1
= = 4
15 5 30 30

1 107.
(b) Given expression
= 7 2 
9 8 1 108  9  3  1 121
 2 = = = 1.1203
4 7 2 108 108
2
3 2
99  99 
5 28 1 108.
(d) = 0.9802,   = 0.9607
=   101  101 
14 95 2  3 2
8 97  97 
= 0.9797,   = 0.9600
99  99 
2 8 2 19 1
=  =  = 2
19 19 19 8 4 95  95 
= 0.9793,   = 0.9592
97  97 
1 75 1 75 Hence, only B and D are correct.
102.
(a) Number of ’s =  =  8 = 300
8 2 8 2
4
109.
(a) G = H +
3.001 L
103.
(b) 10000 + of 1891.992 = ?
4.987 4
⇒ (G – H)2 =
3 L
or ? ≈ 100 + of 1900 = 100 + 1140 ≈ 1230.
5 4
⇒ L =
1 7 (G − H ) 2
104.
(d) = 0.33 and = 0.875
3 8 1 1 1 y−x
110.
(c) = − =
1 z x y xy
= 0.25 does not lie between 0.33 and 0.875
4 xy
⇒ z =
23 y−x
= 0.96 which exceeds 0.875
24
111.
(d) (x – 3) (2x + 1) = 0
11 ⇒ x – 3 = 0 or 2x + 1 = 0
= 0.92 which exceeds 0.875
12 If x = 3, 2x + 1 = 7
17 Hence, possible values of 2x + 1 are 0 and 7.
= 0.708 which lies between 0.33 and 0.875 112.
(b) Let missing figure = x
24
15  9  5  3 1 1  
105.
(c) Out of the 5 girls, 1 took part in the camp.      x      = 3
2 4 4  2 3 6  
Out of the 8 boys, 1 took part in the camp.
Out of the 13 students, 2 took part in the camp. 15  9  5 
     x  = 3
2 2 4 4 
\ th of total number of students took part in the
13 15 9/4
 3 =
camp. 2 5/4  x

Chapter_04.indd 85 1/30/2016 12:03:23 PM


86  Chapter 4

9 9 118. (c) Here a = cz = (by)z = byz = (ax)yz = axyz


=
2 5  4x \ a1 = axyz
5 – 4x = 2 \ xyz = 1
3 2 2 2
or x = 119.
(b) Given x = y, y = z, z = w
4 3 3 3
113.
(c) Given expression 3 2
\ y =z= w
   2   2 3
= 2 ÷  2  2  2  2   2   
   3   2 2 4
or y = × w= w
  3  3 3 9
= 2 ÷  2  2  2  2  
  8  (a) Given expression = 721 (1 + 7 + 72 + 73)
120.
 11   4 = 721 × 400
= 2 ÷ 2  2   = 2 ÷  2  2  11  which is completely divisible by 25.
 4  
Hence, remainder is zero
30 11 11
=2÷ =2× = 121.
(b) Sum of the digits in the thousand’s place
11 30 15
= 6000 + 12000 + 18000 + 24000 = 60000
114.
(c) Given expression
Sum of the digits in the hundred’s place
3 3  23 13  1 5 2
=  of       = 600 + 1200 + 1800 + 2400 = 6000
10 7  10 5  5 7 7
Sum of the digits in the ten’s place
3 3 49 1 2 3 21 1 = 60 + 120 + 180 + 240 = 600
=  of   =  
10 7 10 7 7 10 10 7 Sum of the digits in the unit’s place
3 10 1 1 1 = 6 + 12 + 18 + 24 = 60
=   =  =0
10 21 7 7 7 122.
(c) 27, 216, 729, 1728, 3375, 5832, 9261
115.
(a) Given expression
3 2− 3 3 2− 3
  2  123.
(d) Given expression = =
= 1 + 1 ÷ 1  1     3 2+ 2 4 2
  3 
3 1 3
 3 = −
= 1 + 1 ÷ 1  1   4 4 2
 2
3 1 6 3 1 2.45
 3 5 = − = − ×
= 1 + 1 ÷ 1   = 1 + 1 ÷ 4 4 2 4 4 2
 2 2
2 2 7 3
= – 0.31 = 0.75 – 0.31 = 0.44
=1+1× =1+ = 4
5 5 5
116.
(b) Given expression 2 3
124.
(b) of 114 – of 68 = 76 – 51 = 25
3 4
9 4 3 2
= 48 ÷ 12 ×  of  of 
8 3 4 3 125.
(d) 1800 = 2 × 2 × 2 × 3 × 3 × 5 × 5
\ 1800 must be multiplied by 3 × 5 = 15 so that the
48  9 4   3 2  
=          resulting number becomes a perfect cube.
12  8 3   4 3  
n
48  3  (b) Since (a + 1) leaves always remainder 1.
126.
=    2 = 4 × 3 = 12 a
12  2 
81785 (7 + 1)1785
117.
(b) ax = b, by = c, cz = a \ = gives the remainder 1.
7 7
On multiplying, we get
ax × by × cz = a × b × c 127.
(c) For n = 14, 39 (1 + 33 + 36 + 35)
⇒ (abc)xyz = (abc)1 = 39 (1 + 27 + 729 + 243)
⇒ xyz = 1 = 1 = 39 × 103

Chapter_04.indd 86 1/30/2016 12:03:25 PM


Simplification  87

2 1 2 4 9 14 3 2 4 3 6
128.
(a)  , 0, , , , , 137.
(b) − +
3 7 9 9 11 15 3+ 6 6+ 2 3+ 2
27 3 14
129.
(b) 1.27 = 1 =1 = 3 2( 3 − 6) 4 3( 6 − 2)
99 11 11 = –
( 3 + 6)( 3 − 6) ( 6 + 2)( 6 − 2)
1
130.
(b) 2p + = 4 6( 3 − 2)
p +
( 3 + 2)( 3 − 2)
1
⇒ p+ = 2
2p 3 6 − 6 3 12 2 − 4 6 3 2 − 2 3
= − +
3 (−3) 4 1
 1 
Therefore,  p + 
 2p   62 3 3 2 6
= 
1 1
1 1  1 
⇒ p3 + +3×P× 1 + 
8 p3 2p  2p  = − 6 + 2 3 − 3 2 + 6 + 3 2 − 2 3 = 0
1
3 138.
(b) x2 + y2 = 3341
3
⇒ 8 = p + 3
+ ×2
6p 2 x2  y 2  891
1 2 x 2  4232
⇒ p3 + = 8 – 3 = 5  
8 p3 ⇒ x = 46, y = 35
3 1 3 4 2 3 4 5 n 1 2
= × = × 139.
(b)     ...  =
5 2 5 8 3 4 5 6 n n
1 1 1 1
131.
(a) + 2 = + 1 2 7 13
x 2
y (2 + 3) 2
(2 − 3) 2 2 −1 −
140. (a) 3 11 = 3 11
1 1 1 1
= + = 14 3+ 3+
7+4 3 7−4 3 1 3
3+ 3+
9 +1 10
9 32 1
132.
(b) 9x = x
⇒ 32x =
3 3x 77 − 39 38
⇒ 32x = 32–x = 2x = 2 – x 33
= = 33
⇒ x = 2/3 1 10
3+ 3+
a 33 33
5 +3
5a + 3b 23 b 23 10
133.
(d) = ⇒ =
2a − 3b a

5 2 −3 5 38 33 38
b = × =
33 109 109
a a
⇒ 25 + 15 = 46 − 69
b b 1  1 1 
141.
(b) 3 + + − 
a a 4 3 3+ 3 3− 3
⇒ 21 = 84 ⇒ =
b b 1
1 3− 3 3− 3
(d) 0.1 × 0.01 × 0.001 × 107
134. = 3+ + − 
3  3 + 3 3 − 3 
= 10–6 × 107 = 101 = 10
1 3 1 1
[( x −3/5 ) −5/3 ]5 = (x–3/5)1/5 × –5/3 × 5 = 3 + − = 3+ −
5
135.
(c) 3 3 3 3
= x–3/5 × –5/3 = x = x =3
 1  1  1   1  1  1
(a) 1 − 1 − 1 −  ... 1 − 1 − 
136. 142.
(a) Given x4  = 322
 3  4  5   24  25  x4
2
2 3 4 23 24 2  2 1
= × × × ... × × = ⇒  x  2   2 = 322
3 4 5 24 25 25 x

Chapter_04.indd 87 1/30/2016 12:03:27 PM


88  Chapter 4

2 148.
(b) According to the question,
 2 1
⇒  x  2  = 324 r = (2a)2b = 22b × a2b = (4)b × (ab)2
x
Also, r = ab × xb
2 1
\ x  2 = 18 ⇒ a × xb = 4b × (ab)2
b
x
2
⇒ xb = 4b × ab
 1
⇒  x    2 = 18 ⇒ xb = (4a)b
x
\ x = 4a
1
⇒ x = 16 = 4
x m 4 r 9
149.
(b) Given,
  and
143.
(d) (x) (a – b) (a + b) + (b – c) (b + c) + (c – a) (c + a) n 3 t 14
= x0 = 1 Put the values and calculate
144.
(c) The given expression is 3mr  nt 3  4  9  3  14
=
62 + 43 + 3 = 36 + 64 + 3 = 103 4nt  7 mr 4  3  14  7  4  9

1 17 1 108  42
145.
(c) x = , x– =? =
x 4 x 168  252
2 2
 1  1 1 66 11
 x   =  x    4  x 
Now, =  
x x x 84 14
Putting the value, we get
1 15 4 4
x = 3 6  6 3 
(d)  a9   a9  = (((a9)1/6)1/3)4 (((a9)1/3)1/6)4
150.
x 4    
x 1 x 13 = (a2) (a2) = a4
146.
(d) Given  =
1 x x 6
1 1 1+ 3 −1 1
151.
(a) 1 − + = +
x 1 x x(1  x) 169 1+ 3 1− 3 1+ 3 1− 3
⇒  2 =
1 x x x(1  x) 36
3 1
= +
x 2  (1  x) 2 169 1+ 3 1− 3
⇒ 2 =
x(1  x) 36
3 (1 − 3) + 1 + 3
=
x2  1  x2  2 x  2 x  2 x2 169 1 − ( 3) 2
⇒ =
x(1  x) 36
3 − 3 +1+ 3 2 3  2
1 169 = =
⇒ = −2 2
x(1  x) 36

= 1 − 3
9 4
⇒ x = or
13 13
1 1
152.
(a) 2 + 2 + +
ab cd 2+ 2 2−2
147.
(c) =
bc ad
1 1
⇒ a2 + ad + ab + bd = bc + bd + c2 + cd = 2 + 2 + −
2+ 2 2− 2
⇒ a2 + ad + ab = c2 + bc + cd
⇒ ad + ab – bc – cd = c2 – a2 2− 2 −2− 2
= 2 + 2 +
⇒ a(b + d) – c(b + d) = (c – a) (a + c) (2 + 2) (2 − 2)
⇒ (a – c) (b + d) = – (a – c) (a + c)
2 2
⇒ (a – c) (b + d) + (a – c) (a + c) = 0 = 2 + 2 −
2
⇒ (b + d + a + c) (a – c) = 0
So, either a = c or a + b + c + d = 0 or both. 2 2  2 =2

Chapter_04.indd 88 1/30/2016 12:03:29 PM


Simplification  89

2n + 4 − 2 (2n ) 2n + 4 − 2n + 1 3 3
153.
(d) = = 3 + = 3+
2 (2 n+3
) 2 n+4 3 30 + 3
3+
10 10
2n + 4 2n + 1 1 7 30 10
− = 1 − = = 3 + = 3+
2n + 4 2n + 4 8 8 23 11
2 33 + 10 43
154.
(a) x + = 10 = =
4
3+ 11 11
30 + 7
6
5 +1 5 +1 ( 5 + 1) 2
2 157.
(c) x = × =
⇒ x+ = 10 5 −1 5 +1 5 −1
4×6
3+
37 ( 5 + 1) 2 5 +1
=
2 4 2
⇒ x+ = 10
3+
24 Therefore, 5x2 – 5x – 1
37 2
 5 + 1 5( 5 + 1)
2 = 5   − −1
⇒ x+ = 10  2  2
111 + 24
37  5 +1+ 2 5  5 5 + 5
= 5   − −1
2 × 37  4  2
⇒ x+ = 10
135
3+ 5  5 5 +5
74 = 5   − −1
⇒ x+ = 10  2  2
135
15 + 5 5 − 5 5 − 5 − 2 8
74 = = =4
⇒ 10 − = x 2 2
135
1350 − 74
⇒ x = 10 −
135 3+ x + 3− x
158.
(b) = 2
1276 3+ x − 3− x
⇒ x =
135
( 3  x  3  x )2
1/2 3/4 7/8 ⇒ =2
155.
(d) u1 = 3 
, u2  , u3
3 3 3 3
3 3 3 (3  x  3  x)
2n 1
3 + x + 3 − x + 2 9 − x2
⇒ un = 3 2n ⇒ = 2
2x
210 1 29 1
10 ⇒ 6 + 2 9 − x 2 = 4x
⇒ u10 = 3 2 , u9  3 29

210 1 29 1 ⇒ 2 9 − x 2 = 4x – 6
u10 
10 9
\ = 3 2 2 Squaring both sides
u9
4 (9 – x2) = 16x2 + 36 – 48x
210 − 1 − 2(29 − 1)
210
⇒ 36 – 4x2 = 16x2 + 36 – 48x
= 3
⇒ 20x2 = 48x
210 1  210  2 1
210 48 12
= 3 210 2 ⇒ x = =
20 5
3 159.
(c) 0.121212 ...
156.
(b) Expression = 3 +
1 12 4
3+ = =
1 99 33
3+
3

Chapter_04.indd 89 1/30/2016 12:03:30 PM


90  Chapter 4

3 2 1 (?) 2 17 13 5 11
160. (b) ? = 3 + 4 − 3 ⇒ + = + −
4 5 8 10 12 4 2 6
15 22 25
? = + − (?) 2 13 5 11 17
4 5 8 ⇒ = + − −
10 4 2 6 12
150  176  125
? =
40 (?) 2 39 + 30 − 22 − 17
⇒ =
10 12
201 1
? = = 5
40 40 (?) 2 69 − 39
⇒ =
10 12
161. (d) 52 × 14 − 6 × 7 + (4)? = 18

(?) 2 30
⇒ 52 × 14 – 6 × 7 + (4)? = (18)2 ⇒ =
10 12
⇒ 25 × 14 – 42 + (4)? = 324
⇒ 350 – 42 + (4)? = 324 30 × 20
⇒ (?)2 =
? 12
⇒ 308 + (4) = 324
⇒ (4)? = 324 – 308 ⇒ (?)2 = 25

⇒ (4)? = 16 ⇒ ? = 25 = 5
? 2
⇒ (4) = (4) 166. (c) (?)3 + 49 = 92 × 576 ÷ 2 1296
⇒ ? = 2
⇒ (?)3 + 7 = 92 × 576 ÷ 2 × 36
162. (c) ? = 67.99% of 1401 – 13.99% of 1299 ⇒ (?)3 + 7 = 92 × 576 ÷ 72
68 14 ⇒ (?)3 + 7 = 92 × 8
⇒ ? = 1401 × – 1300 ×
100 100 ⇒ (?)3 + 7 = 736
⇒ ? = 952.68 – 182 ⇒ (?)3 = 736 – 7 = 729
⇒ ? = 770.68 ⇒ ? = 3
729
⇒ ? = 770 (Approx.)
⇒ ? = 9
2
 24  399 41 1 1
163. (d) ? =   × ÷ 167. (c) 85 + ? = of (92)% of 1 of 650
 9  39 899 6 23
2
 24  399 899 1 92 24
⇒ ? =   × × ⇒ 85 + ? = × × × 650
 9  39 41 6 100 23
⇒ ? = 7.11 × 10.23 × 21.92 ⇒ 85 + ? = 104
⇒ ? = 159.435 = 1550 (Approx.) ⇒ ? = 104 – 85
⇒ ? = 19
164. (b) (3 × 2)?+5 = (15 × 0.40)4 ÷ (1080 ÷ 30)4
× (27 × 8)4 168.
(a) By putting x = 2, we have
?+5 4 4 4
⇒ (3 × 2) = (6) ÷ (36) × (216) ( 3 + 2) 2 + ( 3 − 2) 2
?+5 4 2 4 3 4
⇒ (6) = (6) ÷ (6 ) × (6 )
?+5 = 3 + 2 6 + 2 + 3 − 2 6 + 2 = 10
⇒ (6) = (6)4 + (6)8 × (6)12
⇒ (6)?+5 = (6)–4 × (6)12 Again putting x = – 2 we get
⇒ (6)?+5 = (6)8 ( 3 + 2) − 2 + ( 3 − 2) − 2
⇒ ? + 5 = 8
1 1
⇒ ? = 8 – 5 = 3 = +
( 3 + 2) 2 ( 3 − 2) 2
(?) 2 5 1 1 5
165. (d) + 1 = 3 + 2 − 1 ( 3 − 2) 2 + ( 3 + 2) 2
10 12 4 2 6 =
( 3 + 2) 2 ( 3 − 2) 2

Chapter_04.indd 90 1/30/2016 12:03:32 PM


Simplification  91

10 10 171. (b) x(b – c)(b + c – a) × x(c – a)(c + a – b) × x(a – b)(a + b – c)


= =
= x0 = 1
{( 3) 2 − ( 2) 2 }2 (3 − 2) 2
172. (d) When simplified
= 10/1 = 10.
So, that x = ± 2 will satisfy the equation. { }{ }{ } {
2−
1
3
2−
3
5
5
2 − ... 2 −
7
999
1001 }
ab–ac+bc–ab+ac–cb
169. (c) x = 1 (Statement I) 5 7 9 11 1003
According to statement III, = × × × × ... ×
3 5 7 9 1001
log (4) – log (2) = log 2
1003
4 =
log   = log 2 3
2
2 = 2 173.
(c) Q Price of one pencil box = 7 + 22 + 14 = `43
Hence, required amount paid by Seema to shopkeeper
170. (a) x = 3 + 2 2
= 20 × 7 + 8 × 22 + 6 × 175 + 7 × 43
= 3 + 2 × 1.414
= 140 + 176 + 1050 + 301 = `1667
= 5.828, i.e., less than 6
Number must be close and less than 64 = 1296
\ Required number = 1154

Chapter_04.indd 91 1/30/2016 12:03:32 PM


5 Percentage

INTRODUCTION
The term per cent means per hundred or for every hundred. Thus, 40 per cent will be written as 40%.
It is the abbreviation of the Latin phrase per centum. A fraction whose denominator is 100 is called a percentage
Scoring 60 per cent marks means out of every 100 marks and the numerator of the fraction is called rate per cent, e.g.
the candidate scored 60 marks. 5
and 5 % means the same thing, i.e., 5 parts out of every
The term per cent is sometimes abbreviated as p.c. The 100
symbol % is often used for the term per cent. hundred parts.

SOME BASIC FORMULAE

1. To convert a fraction into a per cent: 3. To find a percentage of a given number:


l x
To convert any fraction to rate per cent, x % of given number (N) = × N.
m l 100
multiply it by 100 and put % sign, i.e., ×
m
100%. Illustration 3 75% of 400 = ?
75
3 Solution: 75% of 400 = × 400 = 300.
Illustration 1 What percentage is equivalent to ? 100
5
3 Illustration 4 Find a number whose 4% is 72.
Solution: × 100 = 60%.
5 Solution: Let the required number be x.
Then, 4% of x = 72
2. To convert a per cent into a fraction:
4 100
To convert a per cent into a fraction, drop the ⇒ × x = 72 ⇒ x = × 72 = 1800.
per cent sign and divide the number by 100. 100 4
Illustration 5 What per cent of 25 kg is 3.5 kg?
2 Solution: Let x% of 25 kg be 3.5 kg.
Illustration 2 What fraction is 16 %?
3 Then, x% of 25 kg = 3.5 kg
 50  x 3.5  100
  ⇒ × 25 = 3.5 ⇒ x = = 14.
2 3  50 1  1
Solution: 16 % = =   = . 100 25
3 100  3 100  6
Hence, 3.5 kg is 14% of 25 kg.

Chapter_05.indd 92 1/30/2016 3:26:46 PM


Percentage 93

soMe useFul shoRt-Cut MethoDs

Solution: Here x = 40.


1. (a) If A is x% more than that of B, then B is less
 x 
than that of A by \ Required answer =   100 %
 100  x 
 x 
100  x  100  %.  40 
=   100 %
 100  40 
(b) If A is x% less than that of B, then B is more 2
than that of A by = 66 %
3
 x 
100  x  100  %. 2. If A is x% of C and B is y% of C, then
x
A= × 100% of B.
Explanation y
x 100  x
Given A=B+ B= B Explanation
100 100 x A
Given A= C ⇒ C = 100
100  x 100 x
\ A–B= B–B
100 y B
and B= C ⇒ C = 100
 100  x  x 100 y
=   1 B = B. A B x
 100  100 \ C = 100 = 100 ⇒ A = B
x x y y
B x
A B 100 x
So, = = or × 100% of B.
A 100  x 100 x y
B
100 Illustration 8 If A is 20% of C and B is 25% of C, then
 x  what percentage is A of B?
⇒ A–B=   100 % of A.
 100  x  Solution: Here x = 20 and y = 25.
x
Therefore, B is less than that of A by A= × 100% of B
y
 x 
  100 %. 20
100  x  = × 100% of B, i.e., 80% of B.
25
Similarly, (b) can be proved.
3. (a) If two numbers are, respectively, x% and y%
Illustration 6 If Mohan’s salary is 10% more than that of
more than a third number, then the first number
Sohan, then how much per cent is Sohan’s salary less than
that of Mohan?  100  x 
is   100 % of the second and the
Solution: Here x = 10.  100  y 
 x   100  y 
\ Required answer =   100 % second is   100 % of the first.
 100  x   100  x 
 10  (b) If two numbers are, respectively, x% and y%
=   100 %
 100  10  less than a third number, then the first number
1  100  x 
= 11 %. is   100 % of the second and the
9  100  y 
Illustration 7 If A’s income is 40% less than B’s
’s income,  100  y 
second is   100 % of the first.
then how much per cent is B’s income more than A’s ’s  100  x 
income?

Chapter_05.indd 93 1/30/2016 12:43:39 PM


94 Chapter 5

Explanation Explanation
Let A, B and C be the three numbers. Let the original price of the commodity be ` 100.
Given P
Then, the increased price = 100 +  100
x  100  x   100  100
A=C+ C=   C ⇒ C = A  
100 100  100  x  = ` (100 + P).
y  100  y   100  Therefore, to keep the price unchanged, there should
and, B=C+ C=   C ⇒ C =B  be a reduction in the consumption of the commodity by ` P.
100 100   100  y 
Q Decrease in ` (100 + P) = ` P
 100   100 
\ A  =B  P
 100  x   100  y  \ Decrease in ` 100 =  100
100  P
 100  x   100  x  \ Required reduction in consumption is
⇒ A=   B or  × 100% of B
 100  y   100  y   P 
  100 %.
 100  y   100  y  100  P 
and, B=  A or  × 100% of A.
 100  x   100  x  Similarly, (b) part can be proved.
Similarly, (b) can be proved. Illustration 11 If the price of sugar increases by 25%, find
Illustration 9 Two numbers are respectively 20% and 50% how much per cent its consumption be reduced so as not to
more than a third number. What per cent is the first of the increase the expenditure.
second? Solution: Reduction in consumption
Solution: Here x = 20 and y = 50.  P 
=   100 %
 100  x   100  P 
\ First number =  × 100% of the second
 100  y 
 25 
=   100 % or 20%.
 100  20   100  25 
=  × 100% of the second
 100  50 
i.e., 80% of the second. Illustration 12 If the price of a commodity decreases by
25%, find how much per cent its consumption be increased
Illustration 10 Two numbers are, respectively, 32% and so as not to decrease the expenditure.
20% less than a third number. What per cent is the first of Solution: Increase in consumption
the second?
Solution: Here x = 32 and y = 20.  P 
=   100 %
 100  P 
 100  x 
\ First number =   100 % of the second  25  1
 100  y  =   100 % or 33 %
 100  25  3
 100  32 
=   100 % of the second
 100  20  5. If a number is changed (increased/decreased)
i.e., 85% of the second. successively by x% and y%, then net % change
 xy 
4. (a) If the price of a commodity increases by is given by  x  y   % which represents
 100 
P%, then the reduction in consumption so as
increase or decrease in value according as the sign
not to increase the expenditure is
is +ve or –ve.
 P 
  100 %. If x or y indicates decrease in percentage, then put
100  P 
–ve sign before x or y, otherwise +ve sign.
(b) If the price of a commodity decreases by P%,
then the increase in consumption so as not to Explanation
decrease the expenditure is Let the given number be N.
 P  If it is increased by x%, then it becomes
  100 %.
100  P  Nx N ( x  100)
N + x% of N = N + = .
100 100

Chapter_05.indd 94 1/30/2016 12:43:41 PM


Percentage 95

If it is further increased by y%, then it becomes Illustration 15 If the side of a square is increased by 20%,
N ( x  100) y N ( x  100) its area is increased by k%. Find the value of k.
  Solution: Since side × side = area
100 100 100
\ Net % change in area
N ( x  100) ( y  100)
=  xy   20  20 
(100) 2 = x y  % =  20  20  %
 100 100 
N ( x  100) ( y  100)
\ Net change = –N [Here x = 20 and y = 20]
(100) 2 = 44%.
N (100 x  100 y  xy ) Therefore, the area is increased by 44%.
=
(100) 2 Here k = 44.
 xy  1 100 Illustration 16 The radius of a circle is increased by 2%.
\ % change = N  x  y    Find the percentage increase in its area.
 100  100 N
 xy  Solution: Since π × radius × radius = area
= x y  %. \ Net % change in area
 100 
Illustration 13 If salary of a person is first increased by  xy   2  2
= x y  % =  2  2  %
15% and thereafter decreased by 12%, what is the net  100  100 
change in his salary? [Here x = 2 and y = 2]
Solution: Here x = 15 and y = –12. 1
= 4 %
\ The net change in the salary 25
 xy   15  12  1
= x y  % = 15  12   % or 1.2%. Therefore, the percentage increase in area is 4 %.
 100  100  25
Since the sign is +ve, the salary of the person increases Illustration 17 The tax on a commodity is diminished by
by 1.2%. 15% and its consumption increases by 10%. Find the effect
Illustration 14 The population of a town is decreased on revenue.
by 25% and 40% in two successive years. What per cent Solution: Since tax × consumption = revenue
population is decreased after two years? \ Net % change in revenue
Solution: Here x = – 25 and y = – 40.  xy   15  10 
= x y  % =  15  10  %
\ The net % change in population  100 100 
xy  [Here x = – 15 and y = 10]

= x y % = – 6.5%.
 100 
 25  40  \ The revenue decreases by 6.5%.
=  25  40   % or – 55%.
 100  7. If the present population of a town (or value
Since the sign is –ve, there is decrease in population of an item) be P and the population (or value of
after two years by 55%. item) changes at r% per annum, then
6. If two parameters A and B are multiplied to get a (a) Population (or value of item) after n years
n
product and if A is changed (increased/decreased) by  r 
= P 1 
x% and another parameter B is changed (increased/  100 
decreased) by y%, then the net % change in the
(b) Population (or value of item) n years ago
 xy 
product (A × B) is given  x  y   % which P
 100  = n
.
represents increase or decrease in value according  r 
1  
as the sign in +ve or –ve. 100 
If x or y indicates decrease in percentage, then put where r is +ve or –ve according as the population
–ve sign before x or y, otherwise +ve sign. (or value of item) increases or decreases.

Chapter_05.indd 95 1/30/2016 12:43:42 PM


96 Chapter 5

Explanation Solution: Here P = 144000, x = 5, y = – 10 and z = 15.


Population at the end of first year \ Population after 3 years
r  r   x  y  z 
=P+ P = P 1  . = A 1  1 1
100  100   100   100   100 
   
Now, the population at the beginning of second year  5  10   15 
= 144000 1  1 1
 r   100   100   100 
   
= P 1  .
 100  144000  105  90  115
= = 156492.
\ Population at the end of second year 100  100  100
2
 r  r  r   r  9. In an examination, the minimum pass percentage
= P 1   P 1 = P 1 
 100  100  100   100  is x%. If a student secures y marks and fails
   by z marks, then the maximum marks in the
n 100( y  z )
 r  examination is .
Population at the end of n years = P 1  . x
 100 
Illustration 18 The population of a town increases 5% Explanation
annually. If its present population is 84000, what will it be Let the maximum marks be m.
in 2 years time? Given x% of m = y + z
x 100( y  z )
Solution: Here P = 84000, r = 5 and n = 2. ⇒ × m = y + z or m = .
\ Population after 2 years 100 x
n 2
Illustration 21 In an examination, a student must get 60%
 r   5  marks to pass. If a student who gets 120 marks, fails by 60
= P 1  = 84000 1 
 100   100  marks, find the maximum marks.
105 105 Solution: Here x = 60, y = 120 and z = 60.
= 84000 ×  = 92610. \ Maximum marks
100 100
Illustration 19 The population of a town increases at the 100( y  z ) 100(120  60) 100  180
= = = = 300.
rate of 5% annually. If the present population is 4410, what x 60 60
it was 2 years ago?
10. In an examination x% and y% students
Solution: Here P = 4410, r = 5 and n = 2. respectively fail in two different subjects while
\ Population of the town 2 years ago z% students fail in both the subjects, then the
percentage of students who pass in both the
P 4410 4410
= = = = 4000. subjects will be (100 – (x + y – z))%.
 r 
n
 5 
2 105 105

1   1   100 100 Explanation
100  100 
% of students who failed in one subject only = (x – z)%.
8. If a number A is increased successively by x% % of students who failed in other subject only = (y – z)%.
followed by y% and then by z%, then the final % of students who failed in both the subjects = z%.
value of A will be \ % of students who passed in both the subjects
= [100 – [(x – z) + (y – z) + z]]%
 x  y  z 
A 1  1 1 = (100 – (x + y – z))%.
 100   100   100 
Illustration 22 In an examination, 42% students failed in
In case a given value decreases by any percentage, Mathematics and 52% failed in Science. If 17% failed in
we will use a negative sign before that. both the subjects, find the percentage of those who passed
in both the subjects.
Illustration 20 The population of a town is 144000. Solution: Here x = 42, y = 52 and z = 17.
It increases by 5% during the first year. During the second \ Percentage of students passing both the subjects.
year, it decreases by 10% and increases by 15% during the = (100 – (x + y – z))%
third year. What is the population after 3 years? = (100 – (42 + 52 – 17))% or 23%.

Chapter_05.indd 96 1/30/2016 12:43:43 PM


97
Percentage 

Multiple Choice QuestionS

1.
A person has some amount with him. 25% of it is stolen 7.
Ram’s weight is 140% of Manu’s weight. Tanu’s weight is
in a bus, 10% is lost through a hole in the pocket, 50% 90% of Mahesh’s weight. Mahesh weighs twice as much
of remainder is spent on food. He then purchases a book as Manu. What percentage of Ram’s weight is Tanu’s
worth `26 from the remainder. He walks back home weight?
because all his money is over. What was the initial (a) 64.3% (b) 77.8%
amount?
(c) 90% (d) 128.6%
(a) `160 (b) `1,230
(c) `90 (d) `80 8.
If A’s salary is 25% higher than B’s salary, then how much
per cent is B’s salary lower than A’s?
2.
The population of a city increases at the rate of 4% per (a) 15% (b) 20%
annum. There is an additional annual increase of 1% in
the population due to the influx of job seekers. Therefore, 1
(c) 25% (d) 33 %
the per cent increase in the population after 2 years will be 3
(a) 10 (b) 10.25 [Based on MAT, 2001]
(c) 10.50 (d) 10.75 9.
A businessman’s earning increases by 25% in one year
[Based on MAT, 2004] but decreases by 4% in the next. After 5 years his total
earnings would be `72,000. What is his present earning?
3.
Three papers were set in an examination and the maximum
marks per paper were in the ratio of 1:2:2, respectively. (a) `10,000 (b) `80,000
If a student obtained 50% in the first paper, 60% in the (c) `40,000 (d) `54,000
second, and 65% in the third, what per cent did he obtain
overall? 10.
A reduction of 20% in the price of sugar enables a
(a) 58.3% (b) 66.66% 1
purchaser to obtain 2 kg more for `160. Find the
2
(c) 33.33% (d) 60%
original price per kg of sugar.
4.
A recipe gives directions to mix 4 parts of substance (a) `12 (b) `15
A with 7 parts of substance B. These substances are to
(c) `16 (d) `18
be taken by weight, but by mistake they were taken by
volume. Find the error in the percentage of the weight of [Based on MAT, 2001]
A in the mixture, if 117 cm3 of the substance A weighs as 11.
Successive discounts of 20% and 15% are equivalent to a
much as 151 cm3 of the substance B. single discount of
(a) 5.05% (b) 6.00% (a) 35% (b) 32%
(c) 7.05% (d) None of these (c) 17.5% (d) 17%
[Based on MAT, 2003] [Based on MAT, 2001]
5.
Of the total number of candidates in an examination 40% 12.
A man invests `1,200 at 10% p.a. At the end of the year
fail in first subject, of the rest 10% fail in the second and he withdraws 30% of total amount and pays `24 as
of those that pass in these two subjects, only 75% pass transaction fee. At the end of 2nd year he withdraws 30%
in the remaining subject. The number of unsuccessful of the amount and pays `93 as transaction fee. What is the
candidates exceeds that of the successful ones by 570. balance at the end of the third year?
What is the total number of candidates? (a) `660 (b) `825
(a) 2,800 (b) 8,400 (c) `500 (d) `770
(c) 3,000 (d) 1,200
13.
A family’s ratio of savings to expenditure for last month
6.
A manufacturer offers a 20% rebate on the marked price was 2:13. This month, due to unforeseen expenditure,
of a product. The retailer offers another 30% rebate on the savings fell to 50% of the amount saved last month.
reduced price. The two reductions are equal to a single Salary of the last month was `10,000. This month there
reduction of was an increase of 15% in the salary. How much did the
(a) 50% (b) 44% family spend this month?
(c) 46% (d) 40% (a) `667 (b) `11,167
[Based on MAT, 2002] (c) `10,833 (d) `9,833

Chapter_05.indd 97 1/30/2016 12:43:43 PM


98  Chapter 5

14.
500 kg of ore contained a certain amount of iron. After (a) 75% of the failed students are boys.
the first blast furnace process, 200 kg of slag containing (b) 55% of the first-divisioners are girls.
12.5% of iron was removed. The percentage of iron in (c) Number of passed girls is more than that of the boys.
the remaining ore was found to be 20% more than the
(d) If x students failed, 2x got first division.
percentage in the original ore. How many kg of iron were
[Based on FMS (Delhi), 2004]
there in the original 500 kg ore?
(a) 212 (b) 89.2 21.
A shopkeeper has certain number of eggs of which 5% are
found to be broken. He sells 93% of the remainder and
(c) 85 (d) 145
still has 266 eggs left. How many eggs did he originally
15.
If a > b, then by what percentage is less then its reciprocal? have?
 b 1  1 a2  (a) 3800 (b) 4000
(a) 100  2   (b) 100   
a b a b  (c) 4200 (d) None of these
[Based on IIT Joint Man. Ent. Test, 2004]
 b2   a2 
(c) 100 1   (d) 100   1 22.
If 3/5 of a number is 23 more than 50% of the same
2 2
 a  b  number, then what will be 80% of the number?
16.
X’s salary is increased by 20%. On the increase, the tax (a) 92 (b) 184
rate is 10% higher. Find the percentage of increase in the (c) 180 (d) 186
tax liability. [Based on IIT Joint Man. Ent. Test, 2004]
(a) 20 (b) 22 23.
One kg of tea and one kg of sugar together cost `95. If the
(c) 23 (d) Indeterminate price of tea falls by 10% and that of sugar rises by 20%,
[Based on MAT, 2001] then the price of one kg of each combined comes to `90.
17.
Vishal goes to a shop to buy a radio costing `2568. The The original price of tea in `per kg is
rate of sales tax is 7%. He tells the shopkeeper to reduce (a) `72 (b) `55
the price of the radio to such an extent that he has to pay (c) `60 (d) `80
`2568, inclusive of sales tax. Find the reduction needed in [Based on IIT Joint Man. Ent. Test, 2004]
the price of the radio.
24.
Two numbers are respectively 30% and 40% less than a
(a) `179.76 (b) `170 third number. What is the second number as a percentage
(c) `168 (d) `169 of the first?
[Based on MAT, 2001] (a) 70% (b) 75%
18.
Sunder purchased an office bag with a price tag of `600 5 2
in a sale where 25% discount was being offered on the (c) 85 % (d) 116 %
7 3
tag price. He was given a further discount of 10% on the
amount arrived at after giving usual 25% discount. What [Based on Narsee Monjee Inst. of Man. Studies, 2003]
was the final amount paid by Sunder? 25.
A trader sells soaps in economy packs of four soaps
(a) `210 (b) `540 per pack, each pack being charged at the listed price of
(c) `405 (d) `450 three soaps. For every set of five such packs bought by
a customer, the trader gives him one extra soap as a free
[Based on MAT, 2001]
gift. If a customer buys 12 economy packs, what is the
19.
In a society, there are 100 members. Each of them has effective percentage of discount that he gets?
been allotted membership number from 1 to 100. They 4
started a business in which the nth member contributed (a) 28% (b) 28 %
7
`(10 × 2n – 5). After one year, 4th member gets `62 as his
share. Find the total profit in the business after one year. 5
(c) 4% (d) 35 %
7
(a) `8 [2100 – 26] (b) `4 [299 – 26]
100
(c) `2 [2 – 26] (d) None of these 26.
If 1 micron = 10,000 angstroms, then 100 angstroms is
what per cent of 10 microns?
[Based on FMS (Delhi), 2004]
(a) 0.0001% (b) 0.001%
20.
In a school, 60% of the students of Class X were boys.
75% of the boys passed the Class X exams. 40% of the (c) 0.01% (d) 0.1%
passed boys got first division. 80% of the total students [Based on REC Tiruchirapalli, 2003]
passed the exam and 50% of the passed students got 27.
A man walked diagonally across a square lot. Approx.,
first division. Which of the following conclusion is not what was the per cent saved by not walking along the
correct? edges?

Chapter_05.indd 98 1/30/2016 12:43:43 PM


99
Percentage 

(a) 30 (b) 20 34.


At what percentage above the cost price must an article be
(c) 33 (d) 24 marked so as to gain 33% after allowing the customer a
[Based on REC Tiruchirapalli, 2003] discount of 5%?
(a) 48% (b) 43%
28.
The normal dosage of a particular medicine is t tablets per
day for each patient. A hospital’s current supply of these (c) 40% (d) 38%
tablets will last p patients for d days. If the recommended [Based on IIFT, 2003]
dosage increases by 20% and the number of patients 35.
In a group of 20 singers and 40 dancers, 20% of the
decreases by one-third, then for how many days will the singers are less than 25 years old and 40% of the entire
hospital’s supply last? group are under 25 years. What percentage of dancers are
5d 4d under 25 years?
(a) (b)
4 5 (a) 20% (b) 40%
4 pt (c) 50% (d) 60%
(c) (d) None of these.
5  [Based on IIFT, 2003]
[Based on REC Tiruchirapalli, 2003]
36.
Tom’s salary is 125% of Tina’s salary. Tito’s salary is
29.
In a town, 60% of the adult population is male. a% of 80% of Tina’s salary. The total of all the three salaries is
the adult males and b% adult females are educated. The `61,000. What is Tito’s salary?
total number of educated adult males and uneducated (a) `16,000 (b) `16,500
adult females is equal in number to the total number of
(c) `15,500 (d) `15,000
uneducated adult males and educated adult females. If
a and b are both integers, which of the following could be  [Based on IIFT, 2003]
the set of values (a, b)? 37.
Mayur weighs twice as much as Shweta. Shweta’s weight
(a) (20, 30) (b) (20, 10) is 60% of Deepika’s weight. Rakesh weighs 50% of
(c) (30, 15) (d) (30, 20) Vikas’s weight. Vikas weighs 190% of Mayur’s weight.
Which of these people weighs the least?
30.
A clothing supplier stores 800 coats in a warehouse, of (a) Mayur (b) Deepika
which 15 per cent are full-length-coats. If 500 of the
shorter length coats are removed from the warehouse, (c) Shweta (d) Rakesh
then what per cent of the remaining coats are full-length? [Based on SCMHRD Ent. Exam., 2003]
(a) 5.62% (b) 34% 38.
What percentage of numbers from 1 to 70 have squares
(c) 40% (d) 48% that end in the digit 1?
[Based on REC Tiruchirapalli, 2003] (a) 20 (b) 14
31.
5% income of A is equal to 15% income of B and 10% (c) 1 (d) 21
income of B is equal to 20% income of C. If the income of [Based on SCMHRD Ent. Exam., 2003]
C is `2000, then the total income of A, B and C (in rupees) 39.
Ajay and Vikas are sharing a flat in Delhi, with an
is arrangement of equally dividing the household expenses.
(a) 6000 (b) 9000 Ajay went to Pune, where a sale was going on and bought
(c) 12000 (d) 18000 batteries for the house, worth `150 on 20% discount. But
[Based on FMS (Delhi), 2003] he lost them on his way back and had to buy new ones,
after he reached Delhi. How much did he end up spending
32.
If 1/4th of x equals 30% of y which is intern equal to 45% on the batteries?
of z, then x as a percentage of z is
(a) `280 (b) `195
(a) 160% (b) 170%
(c) `270 (d) `75
(c) 180% (d) 190%
[Based on SCMHRD Ent. Exam., 2003]
33.
If the numerator of a fraction is increased by 20% and the
denominator is diminished by 10%, then the value of the Directions (Q 40 to 43): It is given that 5% increase in X always
fraction is 16/21. The original fraction is means 3% increase in Y and 5% increase in Y always implies 2.5%
increase in Z.
3 4
(a) (b)
5 7 40.
Relationship between X and Z could be
2 5 (a) 1.05X = 1.025Z (b) 0.5X = 0.25Z
(c) (d)
3 7 (c) 25X = 7.5Z (d) 250X = 213Z
[Based on FMS (Delhi), 2003] [Based on IMT Ghaziabad, 2002]

Chapter_05.indd 99 1/30/2016 12:43:44 PM


100  Chapter 5

If Y is increased by 30%, then Z2 should increase by


41. 49.
If the charges per hour of internet surfing are increased by
(a) 32.25% (b) 60% 25%, then find the percentage decrease in the time period
of surfing of a user (a net savvy) who can afford only a
(c) 69% (d) 90%
10% increase in expenditure
[Based on IMT Ghaziabad, 2002]
(a) 22% (b) 12%
42.
If X is increased to 23 from 20, then Y should increase (c) 15% (d) 9.09%
from 100 to
50.
Lagaan is levied on 60% of the cultivated land. The
(a) 103 (b) 106
revenue department collected total `3,84,000 through the
(c) 109 (d) 112 lagaan from the village of Sukhiya. Sukhiya, a very rich
[Based on IMT Ghaziabad, 2002] farmer, paid only `480 as lagaan. The percentage of total
43.
If X is increased from 10 to 15, then Z must increase from land of Sukhiya in the total cultivated land of the village is
30 to (a) 0.15% (b) 15%
(a) 35 (b) 45 (c) 0.125% (d) None of these
(c) 60 (d) 75 51.
The price of cooking oil has increased by 25%. The
[Based on IMT Ghaziabad, 2002] percentage of reduction that a family should effect in the
use of cooking oil so as not to increase the expenditure on
44.
Salary of an officer increases every year by 20%. His this account is
salary in the year 2001 was `26640. What was his salary
(a) 25% (b) 30%
in 1999?
(c) 20% (d) 15%
(a) `20000 (b) `19028
[Based on FMS (Delhi), 2002]
(c) `18500 (d) `18840
52.
A flat costs `10 lakh. Incidental expenses and taxes
 [Based on IRMA, 2002]
amount to `36000 a year. What rent per month must the
45.
In a market survey, 20% opted for product A, whereas owner receive to earn at least 6 per cent on his investment?
60% opted for product B. The remaining individuals were (a) `6000 (b) `6500
not certain. If the difference between those who opted for (c) `8000 (d) `8500
product B and those who were uncertain was 720, how
[Based on FMS (Delhi), 2002]
many individuals were covered in the survey?
(a) 3,600 (b) 1,440 53.
A businessman charges 20% over cost price. He allows
his customers 10% off on their bills for cash payment. His
(c) 1,800 (d) Data inadequate net gain per cent is
46.
In a certain shop, which stocks four types of caps, there (a) 10 (b) 8
are one-third as many red caps as blue and half as many
(c) 12 (d) 15
green caps as red caps. There are equal numbers of green
[Based on FMS (Delhi), 2002]
caps and yellow caps. If there are 42 blue caps, then what
per cent of the total caps in the shop are blue? 54.
X gets 25% more than Y and Y gets 20% more than Z. The
(a) 70% (b) 28% share of Z out of a sum of `740 is
(a) `200 (b) `300
(c) 60% (d) 14%
[Based on Narsee Monjee Inst. of Man. Studies, 2002]
(c) `250 (d) `350
[Based on FMS (Delhi), 2002]
47.
A bag contains 600 pens of brand A and 1200 pens of
brand B. If 12% of brand A pens and 25% of brand B pens 55.
In my office there are 30% female employees and 30% of
are removed, then what is the approximate percentage of these earn greater than `8,000 per month and 80% of male
total pens removed from the bag? employees earn less than `8,000 per month. What is the
percentage of employees who earn more than `8,000 per
(a) 37% (b) 36% month?
(c) 22% (d) 18% (a) 30% (b) 23%
[Based on Narsee Monjee Inst. of Man. Studies, 2002] (c) 60% (d) can’t be determined
48.
A invested 125% as much money as B. C invested 80% 56.
In a municipal election, there were two candidates. One
as much money as B. The total of all the three is 61,000. who got 30% of the votes polled was defeated by 16000
How much did C invest? votes. Calculate the total number of votes polled.
(a) 25000 (b) 16000 (a) 24000 (b) 28000
(c) 20000 (d) 45000 (c) 30000 (d) 40000
[Based on Narsee Monjee Inst. of Man. Studies, 2002] [Based on I.P. Univ., 2002]

Chapter_05.indd 100 1/30/2016 12:43:44 PM


101
Percentage 

57.
A person usually spent `48 to buy groundnuts for roasting 65.
In an examination a candidate got 30% marks and failed
and resale. On one occasion he could buy 1.5 kg of by 30 marks. If the passing marks are 60% of the total
groundnuts less for `48 as the price had gone up by 25%. marks, then the maximium marks will be
What was the earlier price of groundnut per kg? (a) 450 (b) 600
(a) `8 (b) `6.40 (c) 300 (d) 100
(c) `7.20 (d) None of these
66.
The price of an article was first increased by 10% and
[Based on I.P. Univ., 2002]
then again by 20%. If the last increased price be `33, the
58.
A house-owner was having his house painted. He was original price was
advised that he would require 25 kg of paint. Allowing for (a) `30 (b) `27.50
15% wastage and assuming that the paint is available in 2
kg cans, what would be the cost of paint purchased, if one (c) `26.50 (d) `25
can costs `16? [Based on SSC (GL), 2010]
(a) `240 (b) `180 67.
A rainy day occurs once in every 10 days. Half of the rainy
(c) `160 (d) `360 days produce rainbows. What per cent of all the days do
not produce rainbow?
59.
X, Y and Z shared `7400 so that X received 25% more than
Y, and Y received 20% more than Z. What amount did Z (a) 95% (b) 10%
receive? (c) 50% (d) 5%
(a) `3500 (b) `3000 [Based on SNAP, 2007]
(c) `2400 (d) `2000 68.
If an electricity bill is paid before due date, one gets a
[Based on I.P. Univ., 2002] reduction of 4% on the amount of the bill. By paying the
bill before due date a person got a reduction of `13. The
60.
What would be the value of 20% of m as a percentage of
amount of his electricity bill was
p, if 8% of m = 4% of p?
(a) `125 (b) `225
(a) 80% (b) 16%
(c) 10% (d) None of these (c) `325 (d) `425
[Based on I.P. Univ., 2002] [Based on SSC (GL), 2010]

61.
On April 1, 2005 my salary increased from `10,000 to 69.
In a recent survey 40% houses contained two or more
`16,000. Simultaneously the rate of income tax decreased people. Of those houses containing only one person 25%
by 37.5%. If so the amount of income tax paid by me were having only a male. What is the percentage of all
remains constant, what is the value of income tax I pay. houses which contain exactly one female and no males?
(a) `3,000 (b) `6,000 (a) 75 (b) 40
(c) `1,600 (d) can’t be determined (c) 15 (d) None of these
 [Based on SBI PO Exam, 2000]
62.
If S is 150% of T, then T is what per cent of S + T ?
70.
When the price of sugar decreases by 10%, a man could
1 buy 1 kg more for `270. Then the original price of sugar
(a) 33 % (b) 40%
3 per kg is
(c) 75% (d) 80% (a) `25 (b) `30
[Based on REC Tiruchirapalli, 2002]
(c) `27 (d) `32
63.
At a school, 20% of the students are seniors. If all of [Based on SSC (GL), 2011]
the seniors attended the school play, and 60% of all the
students attended the play, then what per cent of the non- 71.
First and second numbers are less than a third number by
seniors attended the play? 30% and 37%, respectively. The second number is less
than the first by
(a) 20% (b) 40%
(a) 7% (b) 4%
(c) 50% (d) 100%
(c) 3% (d) 10%
[Based on REC Tiruchirapalli, 2002]
[Based on SSC (GL), 2011]
64.
The price of LPG increases by 20%. By what per cent
must a family reduce the consumption of LPG, so that the 72.
If the height of a triangle is decreased by 40% and its base
expenditure on gas is the same as before: is increased by 40%, what will be the effect on its area?
(a) 83.33 (b) 80 (a) No change (b) 16% increase
(c) 20 (d) 16.66 (c) 8% decrease (d) 16% decrease
[Based on FMS (Delhi), 2006] [Based on SBI PO, 1999]

Chapter_05.indd 101 1/30/2016 12:43:44 PM


102  Chapter 5

73.
In 1 kg mixture of sand and iron, 20% is iron. How boys playing only badminton is 50% of the number of
much sand should be added so that the proportion of iron boys and the total number of boys playing badminton is
becomes 10%? 60% of the total number of boys. The number of children
(a) 1 kg (b) 200 gms playing only table tennis is 40% of the total number of
children and a total of 12 children play badminton and
(c) 800 mgs (d) 1.8 kg
table tennis both. What is the number of girls playing only
[Based on SBI PO Exam, 1999] badminton?
74.
The price of a commodity rises from `6 per kg to `7.50 (a) 16 (b) 14
per kg. If the expenditure cannot increase, the percentage (c) 17 (d) Data inadequate
of reduction in consumption
[Based on SBI Associates PO, 1999]
(a) 15 (b) 20
81.
If the numerator of a fraction is increased by 2 and the
(c) 25 (d) 30 5
[Based on SSC (GL), 2011] denominator is increased by 1, the fraction becomes
8
75.
There is a ratio of 5:4 between two numbers. If 40% of the and if the numerator of the same fraction is increased
first number is 12 then what would be the 50% of second by 3 and the denominator is increased by 1 the fraction
number? 3
becomes . What is the original fraction?
(a) 12 (b) 24 4
(c) 18 (d) Data inadequate 2
(a) Data inadequate (b)
[Based on Bank of Baroda PO, 1999] 7
4 3
76.
When 30% of a number is added to another number the (c) (d)
second number increase to its 140%. What is the ratio 7 7
between the first and the second number? [Based on Guwahati PO, 1999]
(a) 3:4 (b) 4:3 82.
When 50% of one number is added to a second number,
(c) 3:2 (d) Data inadequate the second number increases to its four-thirds. What is the
[Based on Bank of Baroda PO, 1999] ratio between the first number and the second number?
77.
Suresh’s monthly income is 30% more than that of Vinod. (a) 3:2 (b) 3:4
Vinod’s monthly income is 20% less than that of Vinay. (c) 2:3 (d) Data inadequate
If the difference between the monthly incomes of Suresh [Based on Guwahati PO, 1999]
and Vinay is `800, what is the monthly income of Vinod? 83.
Raman scored 456 marks in an exam and Seeta got 54%
(a) `16000 (b) `20000 marks in the same exam which is 24 marks less than
(c) `12000 (d) Data inadequate Raman. If the minimum passing marks in the exam is
[Based on Bank of Baroda PO, 1999] 34%, then how much more marks did Raman score than
the minimum passing makrs?
78.
If 25% of a number is subtracted from a second number. If (a) 184 (b) 196
second number reduces to its five-sixths. What is the ratio
(c) 190 (d) 180
between the first number and the second number?
[Based on Bank of Baroda PO Exam, 2011]
(a) 2:3 (b) 3:2
84.
The difference between a discount of 35% and two
(c) 1:3 (d) Data inadequate
successive discounts of 20% and 20% on a certain bill
[Based on SBI Associates PO, 1999]
was `22. Find the amount of the bill.
79.
A petrol pump owner mixed leaded and unleaded petrol in (a) `1100 (b) `200
such a way that the mixture contains 10% unleaded petrol.
(c) `2200 (d) Data inadequate
What quantity of leaded petrol should be added to 1 litre
[Based on BSRB Mumbai PO, 1999]
mixture so that the percentage of unleaded petrol becomes
5%? 85.
A rabbit on a controlled diet is fed daily 300 g of a mixture
of two foods, food X and food Y. Food X contains 10%
(a) 1000 ml (b) 900 ml
protein and food Y contains 15% protein. If the rabbits
(c) 1800 ml (d) None of these diet provides exactly 38 g of protein daily, how many
[Based on SBI Associates PO, 1999] grams of food X are in the mixture?
80.
Out of a total of 85 children playing badminton or table (a) 100 (b) 150
tennis or both, total number of girls in the group is 70% (c) 200 (d) 140
of the total number of boys in the group. The number of [Based on ATMA, 2008]

Chapter_05.indd 102 1/30/2016 3:28:20 PM


103
Percentage 

86.
In an examination paper of 5 questions, 5% of the 92.
An article when sold for `960 fetches 20% profit. What
candidates answered all of them and 5% none. Of the would be the per cent profit/loss if 5 such articles are sold
rest, 25% answered only 1 question and 20% answered for `825 each?
1 (a) 3.125% profit (b) 3.125% loss
only 4 questions. If 24 % of the entire candidates
2 (c) Neither profit nor loss (d) 16×5% profit
answered only 2 questions and 200 candidates answered [Based on BSRB Bhopal PO, 2000]
only 3 questions, how many candidates appeared at the
examination? 93.
Rakesh solved 80% of the questions in an examination
correctly. If out of 41 questions solved by Rakesh 37
(a) 1000 (b) 800
questions are correct and of the remaining questions out
(c) 600 (d) 400 of 8 questions 5 questions have been solved by Rakesh
[Based on ATMA, 2008] correctly then find the total number of questions asked in
87.
In a certain water body 50 fish were caught, tagged and the examination.
released to the same water body. A few days later, 50 fish (a) 75 (b) 65
were caught again, of which 2 were found to have been (c) 60 (d) Cannot be determined
tagged on previous occasion. If the per cent of tagged fish
[Based on Bsrb Bangalore PO, 2000]
in the second catch approximates the per cent of tagged
fish in the water body, what is the approximate number of 94.
In a class of 60 children, 30% children can speak only
fish in the water body? English, 20% Hindi and English both and rest of the
(a) 10000 (b) 625 children can speak only Hindi. How many children can
speak Hindi?
(c) 1250 (d) 2500
(a) 42 (b) 36
[Based on ATMA, 2008]
(c) 30 (d) 48
88.
The contents of a certain box consist of 14 apples and 23
[Based on BSRB Patna PO, 2001]
oranges. How many oranges must be removed from the
box so that 70% of the pieces of fruit in the box will be 95.
The ratio of males and females in a city is 7:8 and the
apples? percentage of children among males and females is 25%
(a) 12 (b) 6 and 20%, respectively. If the number of adult females in
the city is 156800, what is the total population?
(c) 17 (d) 36
[Based on ATMA, 2005] (a) 245000 (b) 367500
(c) 196000 (d) 171500
89.
Of the 50 researchers in a workgroup, 40% will be assigned
to team A and the remaining 60% to team B. However, [Based on BSRB Patna PO, 2001]
70% of the researchers prefer team A and 30% prefer team 96.
When income tax is 3 paise in a rupee, a person’s net
B. What is the least possible number of researchers who income is `23700. What will it be when the income tax is
will not be assigned to the team they prefer? raised to 7 paise?
(a) 15 (b) 20 (a) `38 (b) `2330
(c) 35 (d) 30 (c) `3460 (d) None of these
[Based on ATMA, 2005] [Based on NMAT, 2006]
90.
Income tax is raised from 4 paise in a rupee but the 97.
X, a businessman, had income in the year 1995 such that he
revenue is increased by 10% only. Find the decrease per earned a profit of 20% on his investment in the business.
cent in the amount taxed. In the year 1996 his investment was less by `5000 but
(a) 12 (b) 14 still had the same income (Income = Investment + Profit)
(c) 16 (d) None of the above as that in 1995. Thus the per cent profit earned in 1996
[Based on NMAT, 2006] increased by 26%. What was his investment in 1995?

91.
Ram gave 40% of the amount to Deepak. Deepak in turn (a) `100000 (b) `100500
gave one-fourth of what he received from Ram to Subhash. (c) `105000 (d) Data inadequate
After paying `200 to taxi driver out of the amount he got [Based on SBI PO, 2001]
from Deepak, Subhash now has `600 left with him. How
98.
The production of a company has ups and downs every
much amount did Ram have?
year. The production increase for two consecutive years
(a) `1200 (b) `4000 consistently by 15% and in the third year it decreases by
(c) `8000 (d) Data inadequate 10%. Again, in the next two years it increases by 15%
[Based on BSRB Chennai PO, 2000] each year and decreases by 10% in the third year. If we

Chapter_05.indd 103 1/30/2016 12:43:44 PM


104  Chapter 5

start counting from the year 1990 approximately what will 105. An HR Company employs 4800 people, out of which 45%
be the effect on the production of the company in 1994? are males and 60% of the males are either 25 year or older.
(a) 37% increase (b) 42% increase How many males are employed in HR Company who are
younger than 25 year?
(c) 52% increase (d) 32% increase
(a) 2640 (b) 2160
[Based on Corporation Bank PO, 2002]
(c) 1296 (d) 864
99.
In a city, 35% of the population is composed of migrants, [Based on IBPS Bank PO, 2011]
20% of whom are from rural area. Of the local population,
48% is female while this figure for rural and urban migrants 106. Six-elevenths of a number is equal to 22% of second
is 30% and 40% respectively. If the total population of the number. Second number is equal to the one-fourth of third
city is 728400 what is its female population? number. The value of the third number is 2400. What is
(a) 509940 (b) 349680 the 45% of first number?
(c) 324138 (d) None of these (a) 109.8 (b) 111.7
[Based on NMAT, 2005] (c) 117.6 (d) None of these
[Based on IBPS Bank PO, 2011]
100. A clothing supplier stores 800 coats in a warehouse of
which 15% are full length coats. If 500 of the shorter 107. Bhawna decided to donate 12% of her salary to an
length coats are removed from the warehouse, what per Orphanage. On the day of donation she changed her mind
cent of the remaining coats is full-length? and donated `2400 which was 125% of what she had
(a) 35% (b) 9.37% decided earlier. How much is Bhawna’s salary?
(c) 5.62% (d) 40% (a) `14750 (b) `16000
[Based on NMAT, 2005] (c) `18500 (d) Cannot be determined
[Based on Uttrakhand GBO PO, 2007]
101. A company received two shipments of ball bearings. In
the first shipment, 1% of the ball bearings were defective. 108.
If the numerator of a fraction is increased by 400% and
In the second shipment, which was twice as large as the the denominator is increased by 500%. The resultant
first, 4.5% of the ball bearings were defective. If the fraction is 20/27. What was the original fraction?
company received a total of 100 defective ball bearings, 9 11
how many ball bearings were in the first shipment? (a) (b)
8 12
(a) 2000 (b) 1000
3
(c) 990 (d) 3000 (c) (d) None of these
4
[Based on NMAT, 2005]
[Based on New Indian Insurance PO, 2009]
102. In an Entrance Examination Ritu scored 56% marks,
Smita scored 92% marks and Rina scored 634 marks. The 109.
Two numbers are less than a third number by 30% and
maximum marks of the examination are 875. What are the 37%, respectively. How much per cent is the second
average marks scored by all the three girls together? number less than the first?
(a) 929 (b) 815 (a) 7% (b) 10%
(c) 4% (d) 3%
(c) 690 (d) 643
[Based on SSC (GL) Prel. Exam, 2002]
[Based on IBPS Bank PO, 2011]
110. 8% of the voters in an election did not cast their votes. In
103. In a test, a candidate secured 468 marks out of maximum this election, there were only two candidates. The winner
marks ‘A’. If the maximum marks ‘A’ were converted to by obtaining 48% of the total votes defeated his rival by
700 marks, he would have secured 336 marks. What were 1100 votes. The total number of voters in the election was
the maximum marks of the test?
(a) 2100 (b) 23500
(a) 775 (b) 875 (c) 22000 (d) 27500
(c) 975 (d) 1075 [Based on SSC (GL) Prel. Exam, 2003]
[Based on IBPS Bank PO, 2011]
111. A candidate appearing for an examination has to secure
104.
The market price of a watch is `800. A shopkeeper gives 35% marks to pass. But he secured only 40 marks and
two successive discounts and sells the watch for `612. If failed by 30 marks. What would be the maximum marks
the first discount is 10%, then the second discount is of test?
(a) 12% (b) 20% (a) 280 (b) 180
(c) 15% (d) 10% (c) 200 (d) 150
[Based on SSC (GL) Prel. Exam, 2002] [Based on Corporation Bank PO, 2009]

Chapter_05.indd 104 1/30/2016 12:43:44 PM


105
Percentage 

112. In a test, minimum passing percentage for girls and boys the company declared a dividend of 12.5%. How much
is 35% and 40% respectively. A boy scored 483 marks dividend does Raveendra get in 2009 as the percentage
and failed by 117 marks. What are the minimum passing of his initial investment?
marks for girls? (a) 24.5% (b) 23.9%
(a) 425 (b) 520 (c) 24.1% (d) 23%
(c) 500 (d) None of these [Based on IIFT, 2009]
[Based on CBI (PO), 2010]
118.
A person gave 20% of his income to his elder son, 30% of
113.
There are two types of employees in Sun Metals, general the remaining to the younger son and 10% of the balance
graduates and engineers. 40% of the employees in Sun he denoted to a trust. He is left with `10080. His income
Metals are general graduates and 75% of the engineers was
earn more than `5 lakh/year. If 50% of the organization’s (a) `50000 (b) `40000
employees earn more than `5 lakh/year, what proportion
of the general graduates employed by the organization (c) `30000 (d) `20000
earn `5 lakh or less? [Based on SSC (GL) Prel. Exam, 2003]

(a) 3/5 (b) 3/4 119.


In view of the present global financial crisis, the
(c) 1/2 (d) None of these Finance Minister decided to slash the excise duties to
boost demand and propel economic growth. The excise
[Based on XAT, 2010]
duty on cement was reduced by 30% of its present
114.
A survey shows that 61%, 46% and 29% of the people amount to boost the spending in the infrastructure. What
watched “3 idiots”, “Rajneeti” and “Avtaar” respectively. should be the percentage increase in the consumption of
25% people watched exactly two of the three movies and cement so that the revenue of the government remains
3% watched none. What percentage of people watched all unchanged?
the three movies?
5 6
(a) 39% (b) 11% (a) 42 % (b) 42 %
7 7
(c) 14% (d) 7%
6 5
[Based on IIFT, 2010] (c) 34 % (d) 34 %
7 7
115.
A trader marked the selling price of an article at 10%
[Based on FMS, 2009]
above the cost price. At the time of selling he allows
certain discount and suffers a loss of 1%. He allowed a 120.
In a public school, one-fifth of girls and one-fourth of boys
discount of are under 12 year of age. If the total strength of the school
(a) 11% (b) 10% is 1000 and number of girls is two-fifths of the total, what
part of the total strength of the school is accounted for by
(c) 9% (d) 10.5% those which are 12 year or more of age?
[Based on SSC (GL) Prel. Exam, 2003]
(a) 23% (b) 45%
116.
Bennett distribution company, a subsidiary of a major (c) 55% (d) 77%
cosmetics manufacturer Bavlon, is forecasting the
[Based on FMS (MS), 2006]
zonal sales for the next year. Zone I with current yearly
sales to `193.8 lakh is expected to achieve a sales 121. A reduction of 20% in the price of rice enables a person to
growth of 7.25%; Zone II with current sales of `79.3 buy 3×5 kg more rice for `385. The original price of rice
lakh is expected to grow by 8.2% and Zone III with per kg is
sales of `57.5 lakh is expected to increase sales by (a) `20 (b) `22.50
7.15%. What is the Bennett’s expected sales growth for
the next year ? (c) `25 (d) `27.50
[Based on SSC (GL) Prel. Exam, 2003]
(a) 7.46% (b) 7.53%
(c) 7.88% (d) 7.41% 122.
Wheat is now being sold at `27 per kg. During last
month its cost was `24 per kg. Find by how much per
[Based on IIFT, 2009]
cent a family reduces its consumption so as to keep the
117.
In 2006, Raveendra was allotted 650 shares of Sun expenditure fixed?
Systems Ltd in the initiail public offer, at the face (a) 10.2% (b) 12.1%
value of `10 per share. In 2007, Sun Systems declared
a bonus at the rate of 3:13. In 2008, the company (c) 12.3% (d) 11.1%
again declared the bonus at the rate of 2:4. In 2009, [Based on SNAP, 2009]

Chapter_05.indd 105 1/30/2016 12:43:45 PM


106  Chapter 5

123.
The price of an article is raised by 30% and then two 129.
When the price of sugar was increased by 32%, a
successive discounts of 10% each are allowed. Ultimately family reduced its consumption in such a way that the
the price of the article is expenditure on sugar was only 10% more than before.
(a) increased by 10% (b) increased by 5.3% If 30 kg per month were consumed before, find the new
monthly consumption.
(c) decreased by 3% (d) decreased by 5.3%
(a) 42 kg (b) 35 kg
[Based on SSC (GL) Prel. Exam, 2003]
(c) 25 kg (d) 16 kg
124.
A tradesman gives 4% discount on the marked price and [Based on MAT (Dec), 2010]
gives 1 article free for buying every 15 articles and thus
gains 35%. The marked price is increased above the cost 130.
A man’s income is increased by `1200 and at the same
price by time, the rate of tax to be paid is reduced from 12% to
10%. He now pays the same amount of tax as before.
(a) 40% (b) 39%
What is his increased income, if 20% of his income is
(c) 50% (d) 20% exempted from tax in both cases?
[Based on SSC (GL) Prel. Exam, 2003] (a) `6300 (b) `7200
125.
A 14.4 kg gas cylinder runs for 104 h when the smaller (c) `4500 (d) `6500
burner on the gas stove is fully opened while it runs for 80 [Based on MAT (Dec), 2010]
h when the larger burner on the gas stove is fully opened.
131.
Vellu buys a generator for `100000 and rents it. He puts
Which of these values are the closest to the percentage
12.5% of each month’s rent aside for upkeep and repairs,
difference in the usage of gas per hour, of the smaller
pays `325 per year as taxes and realizes 5.5% annually on
burner over the larger burner?
his investment. Find the monthly rent.
(a) 26.23% (b) 30%
(a) `634.76 (b) `654.76
(c) 32.23% (d) 23.07%
(c) `554.76 (d) `456.32
[Based on SNAP, 2008] [Based on MAT (Dec 2010, May), 2009]
126. The total tractor production in a state is 294000, out of 132.
A person spends 40% of his salary on his educational
which 150000 are made by Mahindra and Mahindra. Out expenses. He spends 60% of it in purchasing books and
of every 1000 Mahindra tractors, 98 are red in colour, but one-half of the remaining in purchasing stationery items.
only 53% of the total tractor production is red. Find the If he saves `160 every month, which is one-fourth of the
percentage of non-Mahindra tractors that are red out of balance after spending over books and stationery items,
total non-Mahindra tractors. what is his monthly salary?
(a) 5.025% (b) 5.130% (a) `8000 (b) `4800
(c) 0.6125% (d) 6.140% (c) `9600 (d) Data inadequate
[Based on MAT (Dec), 2008] [Based on MAT (Feb), 2010]
127.
In an examination, 40% marks are required to pass. A 133.
The tax on a commodity is diminished by 10% and its
obtains 10% less than the number of marks required to consumption increased by 10%. The effect on the revenue
1 3 derived from it is K% change. Find the value of K.
pass. B obtains 11 % less than A and C obtained 41 %
9 17 (a) 1 (b) 2
less than the number of marks obtained by A and B (c) –1 (d) –2
together. What marks did C get? [Based on MAT (Sept), 2009]
(a) 50 (b) 40 134.
A salesman’s terms were changed from a flat commission
(c) 35 (d) 45 of 5% on all his sales to a fixed salary of `1000 plus
2.5% commission on all sales exceeding `4000. If his
[Based on MAT (Feb), 2011]
remuneration as per the new scheme was `600 more than
128.
Mohan spends 40% of his salary on food items, 50% of by the first scheme, what were his sales worth?
the remaining on transport, 30% of the remaining, after (a) `11000 (b) `17000
spending on food and transport, he spends on clothes and (c) `16000 (d) `12000
saves the balance. If he saves `630 every month, what is
[Based on MAT (Sept), 2009]
his monthly salary?
135.
If a bucket is 80% full, then it contains 2 litres more water
(a) `1500 (b) `3000
2
(c) `5000 (d) `6500 than when it is 66 % full. What is the capacity of the
3
[Based on MAT (Feb), 2011] bucket?

Chapter_05.indd 106 1/30/2016 12:43:45 PM


107
Percentage 

(a) 10 l (b) 15 l 142.


In a medical certificate by mistake a candidate gave his
2 height as 25% more than actual. In the interview panel,
(c) 16 l (d) 20 l he clarified that his height was 5 ft 5 inches. Find the
3
[Based on MAT (Sept 2009, Dec), 2007] percentage correction made by the candidate from his
stated height to his actual height.
136. A salesman averages `240 during a normal 40-hour week.
(a) 28.56 (b) 20
During a sale, his rates are increased by 50%. What is his
commission if he puts in 60 hours during the sale? (c) 25 (d) None of these
(a) 390 (b) 540 [Based on MAT (Feb), 2009]
(c) 600 (d) 640 143.
In a Mathematics exam, a student scored 30% in the first
[Based on MAT, 2000] paper out of a total of 180. How much should he score in
137.
In an examination, it is required to get 296 of the total the second paper (out of 150) if he is to get at least 50%
maximum aggregate marks to pass. A student gets 259 marks overall?
marks and is declared failed. The difference of marks (a) 75% (b) 80%
obtained by the student and that required to pass is 5%. (c) 74% (d) 84%
What are the maximum aggregate marks a student can
[Based on MAT (Dec 2008, Sept), 2007]
get?
(a) 690 (b) 780 144.
7% of the total quantity of wheat is lost in grinding when
(c) 740 (d) Cannot be determined a country has to import 6 million tonnes, but when only
1
[Based on MAT (May), 2009] 5 % is lost, it can import 3 million tonnes. Find the
5
138.
In an examination, Mohit obtained 20% more than quantity of wheat grown in the country.
Sushant but 10% less than Rajesh. If the marks obtained
by Sushant are 1080, find the percentage marks obtained (a) 500 million tonnes
by Rajesh if the full marks are 2000. (b) 400 million tonnes
(a) 72% (b) 86.66% (c) 600 million tonnes
(c) 78.33% (d) None of these (d) 700 million tonnes
[Based on MAT (Feb), 2009] [Based on MAT (Dec), 2008]
139.
Ram spends 20% of his monthly income on his household
145.
Water tax is increased by 20% but its consump­tion is
expenditure, 15% of the rest on books, 30% of the rest on
decreased by 20%. Then, the increase or decrease in the
clothes and saves the rest. On counting, he comes to know
expenditure of the money is
that he has finally saved `9520. Find his monthly income.
(a) no change (b) 5% decrease
(a) `15000 (b) `10000
(c) `20000 (d) None of these (c) 4% increase (d) 4% decrease
[Based on MAT (Feb), 2009] [Based on MAT, 2000]

140.
Out of the total production of iron from haematite, an ore 146.
In a co-educational school there are 15 more girls than
of iron, 20% of the ore gets wasted. Out of the remaining boys. If the number of girls is increased by 10% and the
iron, only 25% is pure iron. If the pure iron obtained in number of boys is also increased by 16%, there would be
a year from a mine of haematite was 80000 kg, then the nine more girls than boys. What is the number of students
quantity of haematite mined in the year is in the school?
(a) 400000 kg (b) 500000 kg (a) 140 (b) 125
(c) 450000 kg (d) 600000 kg (c) 265 (d) 255
[Based on MAT (Feb), 2009] [Based on MAT, 1999]
141.
There are two candidates Bhiku and Mhatre for an 147.
The digit at unit’s place of a two-digit number is increased
election. Bhiku gets 65% of the total valid votes. If the by 100% and the ten’s digit of the same number is increased
total votes were 6000, what is the number of valid votes by 50%. The new number thus formed is 19 more than the
that the other candidate Mhatre gets if 25% of the total original num­ber. What is the original number?
votes were declared invalid?
(a) 22 (b) 63
(a) 1575 (b) 1625
(c) 44 (d) None of these
(c) 1675 (d) 1525
[Based on MAT, 1999]
[Based on MAT (Feb), 2009]

Chapter_05.indd 107 1/30/2016 12:43:45 PM


108  Chapter 5

148.
At a college entrance examination, each candidate is 154.
Sharma’s expenditure and savings are in the ratio of
admitted or rejected according to whether he has passed 3:2. Her income increases by 10%. Her expenditure also
or failed the test. Of the candidates who are really capable, increases by 12%. How much per cent does her savings
80% pass the tests and of the incapable, 25% pass the increase?
tests. Given that 40% of the candidates are really capable, (a) 7% (b) 6%
then the proportion of capable college students is about (c) 13% (d) 11%
(a) 73% (b) 70% [Based on MAT (May), 2010]
(c) 68% (d) 75%
155.
There are four friends. The average score in unit test of the
[Based on MAT (May), 2007] first three is 15 and that of the last three is 16. If the score
149.
In an examination, there were 2000 candidates, out of of the last friend is 19, then first friend’s score is what per
which 900 candidates were boys and the rest were girls. If cent of average of the last three?
32% of the boys and 38% of the girls passed, then the total 2
percentage of failed candidates is (a) 66 % (b) 300%
3
(a) 68.5% (b) 64.7%
1
(c) 35.3% (d) 70% (c) 33 % (d) None of these
3
[Based on MAT (May), 2007] [Based on MAT (Sept), 2010]
150.
If the price of gold increases by 30%, find by how much 156.
A monthly return railway ticket costs 25% more than a
the quantity of ornaments must be reduced so that the single ticket. A week’s extension can be had for the former
expenditure may remain the same as before? by paying 5% of the monthly ticket’s cost. If the money
1 paid for the monthly ticket (with extension) is `84, the
(a) 30% (b) 23 % price of the single ticket is
13
(a) `64 (b) `80
2
(c) 27 % (d) 19% (c) `48 (d) `72
13
[Based on MAT (May), 2007] [Based on MAT (May), 2007]
151.
In a company, there are 75% skilled workers and remaining 157.
When the price of an audio system was reduced by 20%,
ones are unskilled. 80% of skilled workers and 20% of the sale increased by 80%. What was the net effect on the
unskilled workers are permanent. If number of temporary sale?
workers is 126, then what is the number of total workers? (a) 44% increase (b) 44% decrease
(a) 480 (b) 510 (c) 66% increase (d) 60% increase.
(c) 360 (d) 377 [Based on MAT, 1998]
[Based on MAT (Dec), 2006]
158.
If two numbers are respectively 20% and 50% of a third
152.
An MBA student gets a fellowship from which he spends number, what is the percentage of the first number to the
70% on personal expenses and 20% on books, fees, etc. second?
The remaining amount is saved and it amounts to `4800 (a) 10 (b) 20
in a year. The value of the monthly fellowship is
(c) 30 (d) 40
(a) `3000 (b) `3500 [Based on MAT, 1998]
(c) `5000 (d) `4000
[Based on MAT, 1998]
159.
Given two positive integers x and y with x < y. The per
cent that x is less than y is
153.
Population of a district is 296000 out of which 166000 are
males. 50% of the population is literate. If 70% males are 100( y  x) 100( x  y )
(a) (b)
literate, then the number of women, who are literate, is x x
(a) 32200 (b) 31800 100( y  x)
(c) (d) 100(y – x)
(c) 66400 (d) 48000 y
[Based on FMS, 2011]
[Based on MAT (Feb), 2006]

Chapter_05.indd 108 1/30/2016 12:43:45 PM


109
Percentage 

EXPLANATORY ANSWERS

1.
(d) Let total money be `X  27  119
Then, X = 0.25X + 0.1X + 0.5 [1 – 0.25 – 0.1] X + 26 =  46   % X = % of X
 2 2
⇒ X = `80 Number of students failing – Number of students
2.
(b) 100 → 105 → 110.25, i.e., 10.25%. passing = 570 (Given)
3.
(d) Ratio of maximum marks = 1:2:2  119 81
i.e.,     % of X = 570 ⇒ 19% of X = 570
Ratio of marks obtained  2 2
= (0.5 × 1):(0.6 × 2):(0.65 × 2) 570  100
⇒  Thus, X = = 3000
= 0.5:1.2:1.3 19
⇒ Overall percentage Hence, the total number of students are 3,000
0.5  1.2  1.3 6.
(b) 100 – 20% = 80
= × 100
1 2  2 80 – 30% = 56
= 60% \ Single discount = 44%.
4.
(d) Required ratio = 4VA dA:7VBdB 7.
(b) If the weight of Manu is 50 kg then Ram’s weight will
be 70 kg. So, Mahesh’s weight is 100 kg and Tanu’s
4VAd A
= :7VB, weight is 90 kg. Hence, percentage of Ram’s weight
dB to Tanu’s weight
where d is density of the substance 70
= × 100 = 77.8%
Given 117dA = 151dB 90
dA 151
\ = 8.
(b) A = B + 25% of B
dB 117 B 5B
⇒ A = B + =
Now with 7VB of substance B, 4VA of substance A is 4 4
151 4 1
used in place of 4VA × ⇒ B = A = A − A = A – 20% of A.
117 5 5
34 117
⇒ % error = × × 100 ≈ 22%. 9.
(c) The businessman’s earning after five years
117 151
= `72,000
5.
(c) Let the total number of students = X
Let his earnings be `100
Number of students failing in first subject = 40% of X After 1st year →  125  (25% increase)
Number of students failing in second subject After 2nd year →  120  (4% decrease)
= 10% of rest = 10% (60%) of X = 6% of X After 3rd year →  150  (25% increase)
Therefore, total number of students failing in both the After 4th year →  144  (4% decrease)
subjects After 5th year →  180  (25% increase)
= (40 + 6)% of X = 46% of X ...(1) 100
∴ Earning today =  72,000 = `40,000
Therefore, students passing in two subjects 180
= 54% of X 10.
(c) Let the original price be `x per kg
  The students passing in remaining subject 4
\ Reduced price = ` x per kg
81 5
= 75% (54% of X) = % of X
2 5 160 1
Hence students failing in remaining subject ⇒ × 160 = +2
4x x 2
 81 27 200 160 5
=  54   % of X = % of X ...(2) ⇒ = +
 2 2 x x 2
Therefore, total number of students failing in all the 40 5
⇒ = ⇒ x = 16.
subjects equation (1) + (2) x 2

Chapter_05.indd 109 1/30/2016 12:43:46 PM


110  Chapter 5

(−20) × (−15) 17.


(c) Let reduced price of the radio be `x.
11.
(b) (–20) + (–15) + = –­32.
100 \ x + 7% of x = 2568
⇒ 107x = 256800
12.
(a) The man invests `1,200 at 10% p.a.
⇒ x = 2400
At the end of 1st year the amount = `1,320
\ Reduction needed in the price of radio = `168.
30 18.
(c) 600 – 25% = 450
Withdrawal  1320  24 = `420
100
450 – 10% = 405.
Amount at the end of second year (a) nth member contributed `(10 × 2n – 5)
19.
= 900 × 1.1 = `990 ⇒ 1st member contributed `15
30 2nd member contributed `35
Withdrawal =  990  93 = `390
100 3rd member contributed `75
4th member contributed `155
∴ Amount at the end of 3 years
And so on.
= 600 × 1.1 = `660
Since 4th member gets `62 as his share in the profit,
13.
(c) When expressed as a fraction, savings last month therefore we conlcude that 40% profit is earned by
2 each member.
were of the salary. Reduced this month by 50%
15 \ Total profit earned
1 = 40% of [15 + 35 + 75 + ... + upto 100 terms]
savings are of last month’s salary, which is
15 = 2 [3 + 7 + 15 + 31 + ... + upto 100 terms]
`667. New salary is `11,500. Expenditure this month = 2 [(4 + 8 + 16 + 32 + ... + upto 100 terms]
is 11,500 – `667 = `10,833.
= 8 [(1 + 2 + 4 + 8 + ... + upto 100 terms) – 25]
14.
(b) Let the amount of iron be x kg  2100 − 1  
= 8   − 25

 2 − 1 
 x   x – 25  
∴ 1.2  =  ⇒ 3.6x = 5x – 125
 500   300 
= 8 (2100 – 1 – 25)
or 1.4x = 125 = 8 (2100 – 26).
∴ x = 89.28 kg 20.
(c) Total no. of students = K, say
3
a b a 2  b2 \ No. of boys = K
15.
(c) Here the difference = – = 5
b a ab 3 9
No. of boys who passed = 75% of K = K
5 20
 a 2  b2 
b a  ab  No. of boys who got 1st Division
Now, < by   100 % 9 9
a b  a  = 40% of K = K
 b  20 50
4
No. of students passed = K
a b  b2  2 2 5
= × 100 = 100 1  2 
a2  a  No. of students who get 1st division
2
16.
(b) Let X’s salary = `100 = K
5
\ Salary after increase = `120, i.e., `20 is the 4 9 7K
increase in salary on `100. No. of girls passed = K – K =
5 20 20
Let the tax on the original salary was 30%. No. of girls who got 1st division
Hence the tax on the increased salary (`20) will be 2 9 11K
33%, i.e., `6.60. = K− K =
5 50 50
6.60
\ Increase in tax liability = × 100 = 22%. 2K 7 K K
30 No. of girls failed = − = .
5 20 20

Chapter_05.indd 110 1/30/2016 12:43:47 PM


111
Percentage 

21.
(b) Suppose the shopkeeper had x eggs, originally. 26.
(d) Let 100 angstroms = x% of 10 microns
x ⇒ 100 angstroms = x% of 100000 angstroms
No. of broken eggs = 5% of x =
20 100 × 100 1
⇒ x = = = 0.1.
x 19 x 100000 10
Balance = x − =
20 20
27.
(a) Suppose side of the square = 1 metre
19 x
\ 7% of = 266 ⇒ x = 4000. \ Diagonal = 2m
20

22.
(b) Let the number be K
3 1
\ K = K + 23
5 2
⇒ K = 230
⇒ 80% of K = 80% of 230 = 184.

23.
(d) Let C.P. of 1 kg of tea be `x and 1 kg of sugar be `y.
\ x + y = 95 ...(1)
 x  y Distance saved by not walking along the edges
Also,  x −  +  y +  = 90
 10   5  = 2 − 2
⇒ 3x + 4y = 300 ...(2)
2− 2 
From (1) and (2) we, get i.e.,  × 100  % i.e., 29.3% ≈ 30%.
x = 80, y = 15.  2 

24.
(c) Let the third number be x. ptd ptd 5
28.
(a) = = d.
3x 7x  p 6t 2 p 4
\ 1st number = x – 30% of x = x − = (t + 20% of t )  p −  ×
10 10  3  5 3
4x 6x 29.
(d) Let total adult population of town = T
2nd number = x – 40% of x = x − =
10 10
⇒ Adult population that is male = 0.6T
Suppose 2nd number = K% of 1st number
Adult population that is female = 0.4T
6x K 7x a% of adult male population and b% of adult
\ = ×
10 100 10 female population is educated.
600 5 Given, educated adult males + uneducated adult
⇒ K = = 85 .
7 7 females
  = Sum of uneducated adult males
25.
(a) Number of packs bought by customer = 12 + Educated adult females
Number of gift soaps received a (100  b)
⇒  0.6 T   0.4T 
 12  100 100
= Integer part of   = 2
 5
0.6T (100  a ) 0.4 Tb
Total number of soaps received by the customer = 
100 100
= (4 × 12) + 2 = 50
⇒ 3a – 2b = 50
Total money paid by the customer = 12 × 3 × s, where
Only choice (d) satisfies the above equation.
s is the listed sale price of each soap
For 50 soaps, the listed sale price = 50s 30.
(c) 120 coats for full length. 500 shorter length coats are
removed.
Actual amount paid is 36s
\ Percentage of full length coats out of the remaining

Hence, discount is 14s
300 coats
14 s  120
  Discount percentage =   100 = 28% = × 100 = 40.
 50 s  300

Chapter_05.indd 111 1/30/2016 12:43:48 PM


112  Chapter 5

31.
(d) 5% of A = 15% of B ⇒ 5A = 15B ⇒ A = 3B 36.
(a) Suppose Tito’s salary = x,
10% of B = 20% of C ⇒ 10B = 20C ⇒ B = 2C Tom’s salary = y and Tina’s salary = z
If C = 2000, then B = 4000 5z
\ y = 125% of z =
\ A = 12000 4
Hence, the total income of A, B and C = 18000. 4 5
x = 80% of z = z ⇒z= x
5 4
32.
(c) 0.25x = 0.3y = 0.45z
5z 5 5 25
x  x y \ y = = × x = x
Now, × 100 =    × 100 4 4 4 16
z  y z
Also x + y + z = 61000
0.3 0.45 25 5
=  × 100 ⇒ x + x + x = 61000
0.25 0.3 16 4
9 61x
= × 100 = 180% ⇒ = 61000
5 16
⇒ x = 16000
p Also y = 25000, z = 20000.
33.
(b) Let the fraction be
q
37.
(c) Mayur = 2 (Shweta)
p + 20% of p 16 3
\ = Shweta = 60% of Deepika = of Deepika
q − 10% of q 21 5
p 1
p+ Rakesh = 50% of Vikas = of Vikas
⇒ 5 = 16 2
q 21
q−
10 19
Vikas = 190% of Mayur = of Mayur
10
6 p 10 16
⇒ × = \ M > D, S < D, R < V, V > M
5 9q 21
⇒ V > M > D > S, V > R
p 16 9 144 4 19
⇒ = × = = . Rakesh = of Mayur
q 21 12 252 7 10
34.
(c) Let the marked price be `x 20
⇒  Mayur = of Rakesh
Let the C.P. be `y 19
95 x 19 x 10
S.P. = x – 5% of x = = Shweta = of Rakesh
100 20 19
19 x ⇒ S < R ⇒ R > S
y + 33% of y =
20 \ Shweta weighs the least.
⇒ 7y = 5x 38.
(a) 1, 9, 11, 19, 21, 29, 31, 39, 41, 49, 51, 59, 61, 69:
7 2 These numbers have their squares ending in digit 1,
⇒ x = y = y+ y i.e., 14 out of 70, i.e., 20%.
5 5
= y + 40 % of y. 39.
(b) Expenditure by Ajay on batteries

35.
(c) No. of singers = 20 150
= 80% of 150 +
2
No. of dancers = 40
= 120 + 75 = `195.
\ 4 singers are less than 25 years old.
24 out of 60 members of the group are less than 25 40.
(c) 5% increase in X ⇒ 3% increase in Y
years old. 5% increase in Y ⇒ 2.5% increase in Z
\ 20 dancers are less than 25 years old, i.e., 50% of \ 3% increase in Y ⇒ 1.5% increase in Z

the dancers are less than 25 years old. \ 5% increase in X

Chapter_05.indd 112 1/30/2016 12:43:49 PM


113
Percentage 

⇒ 1.5% increase in Z i.e., 5X = 1.5Z


49.
(b) Time × Rate = Total charges

⇒ 25X = 7.5Z. 1 × 1 = 1
41.
(a) 30% of increase in Y x × 1.25 = 1.1
2 1.1
⇒ 15% of increase in Z i.e., 32.25% increase in Z . ∴ x =  × 100 = 88%
1.25
42.
(c) X is increased from 20 to 23, i.e., there is 15% increase
in X. Thus, decrease in time = 12%
⇒ There is 9% increase in Y i.e., Y will increase from 480 x
100 to 109. 50.
(d) Total land of Sukhiya =  = 800 x
0.6
43.
(a) X is increased from 10 to 15, i.e., there is 50% increase ∴ Cultivated land of village = 384000 x
in X. 800 x
\ There must be 15% increase in Z, i.e., Z must ∴ Required percentage = × 100
384000
increase from 30 to 34.5, i.e., 35 approx.
= 0.20833
44.
(c) Suppose salary in 1999 was `x 51.
(c) Let the price of the cooking oil
2
 20  = `100/unit
\ x 1 +  = 26640
 100  Let the family consumes 100 units
26640 × 25 \ Total expenditure on this account = `10000
⇒ x = = `18500.
36 If the price becomes `125 and the family consumes,
45.
(c) Percentage of those who were not certain say x units, then the total expenditure will become
`125x.
= 100 – (20 + 60) = 20%
In order that after increase in price, the total expenditure
Now, let the number of persons involved in the survey
of the family remains unaffected, therefore
be x
125x = 10000 ⇒ x = 80
Then x × 60% – x × 20% = 720
\ Reduction in consumption of units
⇒ x × 40% = 720
∴ x = 1,800 = 20, i.e., 20%.

1 1 6% of 1000000
46.
(c) R = B,G= R =G=Y 52.
(c) = `5000/month
3 2 12
Incidental Expenses + Taxes = `3000/month
Since B = 42, R = 14, G = 7 and Y = 7,
Total Rent per month = `8000.
42
\ Percentage of blue caps = × 100 = 60. 53.
(b) 100 + 20% = 120
70
120 – 10% = 108
47.
(c) No. of pens removed
\ Gain % = 8.
= 12% of 600 + 25% of 1200
= 72 + 300 = 372 5Y
54.
(a) X = Y + 25% of Y =
\ Percentage of total pens removed 4

372 6Z
= × 100 = 20.67 = 22. Y = Z + 20% of Z =
1800 5

48.
(b) A = 125% of B, C = 80% of B X Y Z
\ 4X = 5Y = 6Z ⇒ = =
5 5 4 4 16 15 12 10
⇒ A = B,C= B = × A = A
4 4 5 5 25 i.e., X, Y, Z share their profit in the ratio 15:12:10.
A + B + C = 61000 10
\ Z’s share = × 740 = `200.
4 16 37
⇒ A + A + A = 61000
5 25
55.
(d) Cannot be determined. We do not know whether there
⇒ A = 25000 are some male employees who have exactly `8,000
\ B = 20000, C = 16000. per month as their salary or not.

Chapter_05.indd 113 1/30/2016 12:43:50 PM


114  Chapter 5

56.
(d) Let the total number of votes polled = x
\ The winning candidate got 70% of the votes  P 
64.
(d) Required % =  × 100  %
polled.  100 + P 
Hence, 40% of x = 16000 ⇒ x = 40000.  20 
=  × 100  % = 16.66%
57. (b) Let the earlier price of groundnut be ` x/kg  120 
65.
(d) Passing marks are 0.6x
48
\ kg of groundnuts could be purchased for `48. So, 0.3x + 30 = 0.6x
x
⇒ x = 100
 48 3  5 x
⇒  − × = 48 ⇒ x = 6.40.
 x 2 4 66.
(d) Effective increase percentage
58.
(a) Let the quantity of paint purchased be x kg  20 × 10 
= 10 + 20 +  % = 32
then  (x – 15% of x) = 25  100 
⇒  x = 29.41 or 30 kg 132
Therefore, x × = 33
So, he must purchase 15 cans 100
Total cost = (16 × 15) = `240 32 × 100
⇒ x = = `25
132
59.
(d) X + Y + Z = 7400 ...(1)
67.
(a) Rainbow will occur once in 20 days
5Y
X = Y + 25% of Y = ...(2) Rest 19 days will not producer rainbow
4
19
6Z % = × 100 = 95%
Y = Z + 20% of Z = ...(3) 20
5
\ Eq. (1) gives
68.
(c) Let the amount of the bill be `x
5Y 6 Z 4x
+ + Z = 7400 Therefore, = 13
4 5 100
3Z 6 Z 4x = 1300
⇒ + + Z = 7400
2 5 1300
⇒ x = = `325
⇒ 37Z = 74000 ⇒ Z = 2000 4
\ Y = 2400, X = 3000.
69.
(d) Houses containing only one person
60.
(c) 8% of m = 4% of p ⇒ 2m = p = 100 – 40 = 60%
m p Houses containing only a male
\ 20% of m = = = 10% of p.
5 10 25
= 60 × = 15%
61.
(d) Since we do not have sufficient data. Further any 100
value is possible as the required income tax. \ Houses containing only one female
62.
(b) S = 150% of T = 60 – 15 = 45%.
150T 3 70.
(b) Let the original cost price of sugar be `x per kg.
⇒ S = ⇒S=  T
100 2 270 × 100 270
Q − =1
3 5T 90x x
⇒ S + T = T +T =
2 2 270  10 
 − 1 = 1
2 x 9 
(S + T )
⇒ T =
5 30
⇒ = 1
= 40% of (S + T). x
\ x = `30 per kg
63.
(b) Suppose total number of students = 100
71.
(d) Let the third number be 100
\ No. of seniors who attended the play = 20
Then, first number = 70
Total number of students who attended the play = 60 Therefore, second number = 63
\ No. of non-seniors who attended the play 70 − 63
Hence, required % = × 100
= 60 – 20 = 40 i.e., 40%. 70

Chapter_05.indd 114 1/30/2016 12:43:52 PM


115
Percentage 

7 78.
(a) Let the first and second numbers be x and y,
= × 100 = 10% respectively.
70
25 5
40 × 40  y – x × = y ×
 100 6
72.
(d) Required effect =  +40 − 40 − %
 100 
x 5 1 x
= –16%. or, y– = y  or,   y =
4 6 6 4
i.e., the area will decrease by 16%. \ x:y = 2:3.
73.
(a) In 1 kg mixture quantity of iron = 20 gm 79.
(a) In 1 l mixture quantity of unleaded petrol = 100 ml
Let x gm sand should be added, then 10% of (1000 + x) Let x ml leaded petrol be added, then 5% of (1000 + x)
= 200 = 100 ml
\ x = 1000 gm = 1 kg. or,   5(1000 + x) = 100 × 100
5000
7.50 − 6 × 100 ⇒ x = = 1000 ml.
74.
(b) Increase % = = 25 5
6
80.
(b) Let the number of boys = x
Therefore, decrease percentage in consumption
7x
25 Then, x+ = 85
= × 100 = 20% 10
125 ⇒ x = 50
No. of girls = 85 – 50 = 35.
a 5 4
75.
(a) = , b = a  (Given)
b 4 5
2
Given (40% of a) = a = 12
5
4
\ a = 5 × 6 and b = × 5 × 6 = 24
5
24 x
\ 50% of b = = 12. 81.
(d) Let the original fraction be
2 y
76.
(b) Let the first and the second number be x and y, x+2 5
Then, = or, 8x – 5y = – 11 ...(1)
respectively, then y +1 8
y + 30% of x = 140% of y x+3 3
Again, = or, 4x – 3y = –9 ...(2)
or, y + 0.3x = 1.4y y +1 4
or, 0.3x = 0.4y Solving, we get x = 3 and y = 7
\ x:y = 4:3 3
\ Fraction =
7
77.
(a) Suresh = Vinod + 30% of Vinod
82.
(c) Let the numbers be y and x, respectively.
= 1.3 Vinod
4x y 4x
Vinod = Vinay – 20% of Vinay x + 50% of y = or, = –x
3 2 3
= 80% of Vinay y x y 2
= 0.8 Vinay or, = or, = .
2 3 x 3
\ Suresh = 1.3 × 0.8 Vinay 83.
(a) Let the total marks of the exam be x.
= 1.04 Vinay Then,
Now, 54
x× = 456 – 24
Suresh – Vinay = 1.04 Vinay – Vinay 100
= 0.04 Vinay 54
⇒ x× = 432
= `800 (given) 100
\ Vinay = `20000 432 × 100
⇒ x × = 800
\ Vinod = 0.8 × 20000 = `16000. 54

Chapter_05.indd 115 1/30/2016 12:43:53 PM


116  Chapter 5

\ Minimum passing marks By solving this, we get x = 800. Number of candidates


34 appeared = 800.
= 800 ×
100 87.
(c) Let the number of fish be x then,
= 272 50 × 100 48 × 100
Hence, required more marks get by Raman =
x ( x − 50)
= 456 – 272
50 48
⇒ =
= 184 x x − 50
20 × 80 ⇒ 50x – 2500 = 48x
84.
(c) Successive discounts = 20% + ⇒ 50x – 48x = 2500
100
= 20 + 16 = 36% ⇒ 2x = 2500
Difference in discounts = 36 – 35 = 1% \ x = 1250
\ Bill amount = 22 × 100 88.
(c) Total fruit = 14 + 23 = 37
= `2200. x × 70
Again = 14
85.
(d) Let there be a gm of food X and (300 – a) g of food Y. 100
Then a × 10% + (300 – a) 15% = 38 x = 20
10a 15 Reduce oranges = 37 – 20 = 17
+ (33 − a )
⇒ = 38
100 100 89.
(d) Number of researchers prefer team A
⇒ 10a + 4500 – 15a = 3800 70
50 × = 35
⇒ – 5a = – 700 100
700 40
\ a = Researchers assigned to A = 50 × = 20
5 100
= 140 g of food X Difference = 15
86.
(b) Suppose that total candidates appeared be x. 30
Prefer team B = 50 × = 15
Then, number of candidates who answered all 100
5 60
= x × Assigned to B = 50 × = 30
100 100
Number of candidates who answered none Difference = 15
5 Hence, least possible number of researchers who will
= x ×
100 not be assigned to the team they prefer = 15 + 15 = 30
 5  4 × (− x)
\ Remaining = x − 2  x × 90.
(d) 4–x+ = 10
 100
 100 
x 9x 4x
= x − = ⇒ –x – = 6
10 10 100
Number of candidates answered only 1 question ⇒ –104x = 600
9x 25 9 x −600
= 25% of = × ⇒ x = %
10 100 10 104

9x 2 1 2 1
= 91.
(c) D = R    S = × R = R
40 5 4 5 10
Number of candidates answered 4 questions 1
and, R – 200 = 600
9x 9x 10
= × 20% =
10 50 1
\ R = 800
Candidates who answered 2 questions 10
49 49 \ R = `8000
= % of x = x
2 200
100
92.
(a) C.P. of the article = 960 × = `800
 5  9 x 9 x 49 x 120
Now 2  x × + + + + 200 = x
 100  40 50 200 \ C.P. of 5 articles = `800 × 5 = `4000

Chapter_05.indd 116 1/30/2016 3:29:19 PM


117
Percentage 

\ S.P. of 5 articles = `825 × 5 = `4125 \ Increase % in the production in year 1994


4125 − 4000 (1 ⋅ 368 x − x) × 100
\ Gain % = × 100 = 3.125%. =
4000 x
= 36.8%
93.
(b) Suppose there are 8x questions apart from 41
≈ 37%.
questions. Then,
37 + 5 x 4 35
= 80% = 99.
(c) Total migrants population = × 728400 = 254940
41 + 8 x 5 100
⇒ 185 + 25x = 164 + 32x 65
Local population = × 728400 = 473460
⇒ 7x = 21 ⇒ x = 3 100
\ Total number of questions = 41 + 8x 20
Total rural migrants = × 254940 = 50988
= 41 + 8 × 3 100
= 41 + 24 = 65. 80
Total urban migrants = × 254940 = 203952
100
94.
(a) Number of students who speak only English
\ Population of females
= 30% of 60 = 18
48 30 40
Number of students who speak Hindi and English = × 473460 + × 50988 + × 203952
100 100 100
= 20% of 60 = 12 = 227260.8 + 15296.4 + 81580.8 = 324138
\ Number of students who speak only Hindi 100.
(d) Number of full length coast = 15% of 800 = 120
= (60 – 30) = 30 Remaining coats = 800 – 500 = 300
\ Number of students who speak Hindi 120
\ Required percentage = × 100% = 40%
= 30 + 12 = 42. 300
101.
(b) Let company received x ball bearing in first shipment
100 and 2x ball bearing in 2nd shipment.
95.
(b) Number of females = 156800 × = 196000 \ 1% of x + 4.5% of 2x = 100
80
7 x 9x
\ Number of males = × 196000 = 171500 ⇒ + = 100
8 100 100
⇒ x = 1000
\ Total population = 196000 + 171500 = 367500.
102.
(d) Q Maximum marks in examination = 875
96.
(d) 97% = 23700 56
\ Ritu’s marks = 875 × = 490
23700 100
\ 93% = × 93 = `22722.679
97 92
and Smita’s marks = 875 × = 805
100
97.
(c) Let the investment of X in 1995 be `x and Rina’s marks = 634
x  x 6x Hence, required average marks
\ Profit = ` \ Income = `  x +  = `
5  5  5 490 + 805 + 634 1929
= = = 643
Investment of company X in 1996 would be (x – 5000) 3 3
From the question, 103.
(c) Q Candidate secured 336 marks in 700 total marks
700
126 6 \ Candidate secured 468 marks in × 468
(x – 5000) × = x ⇒ x = `105000. 336
100 5 = 975 total marks
98.
(a) Suppose the production of the company in the year 104.
(c) Let the second discount be x%
1990 be x \ 800 – 10% = 720
Then, production of the company in year 1994 720 – x% of 720 = 612
115 115 90 115 x
= x × × × × ⇒ × 720 = 108
100 100 100 100 100
108 × 100
= 1.368x ⇒ x = = 15.
720

Chapter_05.indd 117 1/30/2016 12:43:55 PM


118  Chapter 5

105.
(d) Q Total number of employs = 4800 63 z
y = z – 37% of z =
45 100
\ Males people = 4800 × = 2160
100 10 x 100 y 10 y
⇒ = ⇒x=
Hence, number of people, younger than 25 year 7 63 9
40 9x x
= 2160 × = 864 ⇒ y = =x–
100 10 10
⇒ y = x – 10% of x.
106.
(d) Q Third number = 2400
110.
(d) 8% votes are invalid.
1

\ Second number = × 2400 = 600 Winner got 48% of the total votes.
4
Loser will get
6
and first number × = 22% of 600 100 – (8 + 48) = 44% of the total votes
11
Now, the total number of voters in the election
11 22
⇒ First number = × 600 × 1100
6 100 = × 100 = 27500.
4
= 242
111.
(c) Suppose maximum marks = x
242 × 45
Hence, 45 of the first number = = 108.90 35
100 then x× = 40 + 30
100
107.
(b) Suppose the salary of Bhawna was `x. 35 70 × 100
⇒ x× = 70 ⇒ x =
 12  125 100 35
 of x  × = 2400 x = 200 marks
 100  100
112.
(d) According to passing percentage = 40% (boys)
12 125
x× × = 2400 According to question,
100 100
x × 40% = 483 + 117 (If total marks = x)
2400 × 100 × 100 40
x = x× = 600
12 × 125 100
= `16000 600 × 100
x =
40
x
108.
(d) Suppose the original fraction is . x = 1500
y
According to question, 35
Passing marks for girls = 1500 × = 525
100
400
x + x× 113.
(e) Let there are 100 employees in Sun Metals.
100 = 20
500 27 \ General graduates = 40 and engineers = 60
y + y×
100 No. of engineers having salary more than 5 lakh/year
x + 4x 20  75 
⇒ = =  60  45
y + 5y 27  100 
No. of employees having salary more than 5 lakh/year
5x 20
⇒ = = 50
6y 27
\ No of general graduates having salary less than 5
x 20 × 6 lakh/year = 35.
⇒ =
y 5 × 27 This is 7/8th of the number of general graduates.
x 8 114.
(d) We know that,
⇒ =
y 9 Exactly 1 + 2 Exactly 2 + 3 Exactly 3
= 61 + 46 + 29
109.
(b) Let the numbers be x, y and z
= 136% ...(1)
7z Exactly 1 + Exactly 2 + Exactly 3
\ x = z – 30% of z =
10 = 100 – 3 = 97% ...(2)

Chapter_05.indd 118 1/30/2016 12:43:56 PM


119
Percentage 

Adding Eqs. (1) and (2), 2


Exactly 2 + 2 Exactly 3 = 39 120.
(d) Number of girls =× 1000 = 400
5
Exactly 2 = 25 Boys = 600
⇒ 25 + 2 Exactly 3 = 39 According to question,
Exactly 3 = 7% 1 1
 400   600 = 230
So, 7% of people watched all the movies. 5 4
\ 12 year above = 1000 – 230 = 770
115.
(b) Let the C.P. of the article be `100, then M.P. of the 770
article = `110 \ Percentage = × 100 = 77%
1000
If the trader has a loss of 1%, it means that the trader
sold the article at `99 121.
(d) Let the original price of rice per kg be `x
110 − 99 385 385
\ % discount = × 100 = 10%. Now, – = 3.5  or  x = `27.50.
110 4 x/5 x
116.
(a) Total growth in sales 122.
(d) Let earlier there was consumption of 1 kg wheat
= (193.8 × 7.25%) + (79.3 × 8.2%) + (57.5 × 7.19%) \ Expenditure = 24
= 14.05 + 6.5 + 4.11 = 24.66 Now, 27 is price of 1 kg
Total sales from all the zones = 330.6 1
\ 1 is price of kg
\ overall percentage growth 27
24.66 24 8
=  100 
7.46% \ 24 is price of = kg
330.6 27 9
1
117.
(d) Bonus share of Sun Systems Limited received by \ Consumption should be reduced by or 11.1%
9
3
Raveendra in 2007 = 650 × = 150 and in 2008 123.
(b) Two successive discounts of 10%
13
(−10) × (−10)
1 = –10 – 10 + = –19%
it is 800  = 400 100
2
30 × (−19)
Hence, in 2009, when bonus share were announced he So, resultant = + 30 – 19 + = +5.3%
100
has 550 shares additionally.
124.
(c) Let C.P. of one article be `x
1200  12.5 \ C.P. of 16 articles = `16x
\ Percentage =  100 = 23%
650
 135 
S.P. of 16 articles = `16x  
118.
(d) Let the income = `x  100 
Given Let the marked price of the article be increased by y%
90 70 80 above the cost price.
× × x = 10080
100 100 100  100 + y   96   135 
⇒ 504x = 10080000 Then, 15x  ×  = 16x  
 100   100   100 
or, x = `20000. or, (100 + y)6 = 900 or y = 150
119.
(b) Let earlier `100 was cost of 1 kg cement. i.e., M.P. of the article is 50% above the cost price.
Now it is `70. 125.
(b) Smaller burner burns for 24 more hours.
Now `70 for 1 kg cement. 24
\ Percentage difference = × 100 = 30%
1 80
\ `1 for kg cement.
70 126.
(c) Total production of red tractors
100 = 294000 × 53% = 15582
\ `100 for kg cement.
70 Number of red tractors of Mahindra make
100 30 150000
\ Increased cement = 1 = = × 98 = 14700
70 70 1000
30 6 Number of non-Mahindra tractors
\ Percentage increase =  100 = 42 % = 294000 – 150000 =144000
70 7

Chapter_05.indd 119 1/30/2016 12:43:57 PM


120  Chapter 5

\ Number of Mahindra red tractors 132.


(a) Let his monthly salary be `x.
= 15582 – 14700 = 882 He spends `0.4x on educational expenses, `0.24x on
Hence, required percentage purchasing books and `0.08x on purchasing stationary
882 items.
= × 100 = 0.6125%
144000 Remaining amount = 0.4x – (0.24x + 0.08x)
127.
(b) Let total marks be 100. = `0.08x
Then, minimum marks to be passed 40. 1
Also, × 0.08x = 160
4
10
\ A obtained marks = 40 – 40 × 160 × 4
100 \ x = = `8000
= 36 marks 0.08

100 36 xy
\ B obtained marks = 36 – × 133.
(c) Using, formula K% = – x + y –
9 100 100

= 36 – 4 10 × 10
= – 10 + 10 – = – 1%
= 32 marks 100
134.
(d) Let his sales were x.
\ C obtained marks Then,
700
= (36 + 32) – (36 + 32) × 2.5 5
17 × 100 1000 + ( x − 4000) = x + 600
= 68 – 28 1000 100
= 40 marks ⇒ 100000 + 2.5x – 10000 = 5x + 60000
128.
(b) Let monthly salary be `x. ⇒ 2.5x = 30000
According to the given condition, ⇒ x = `12000
4 x 6 x 50 3 x 30 135.
(b) Let the capacity of bucket be x L.
+ × + × + saving money = x Then, 0.80x = 0.667x + 2
10 10 100 10 100
4 x 3x 9 x 2
⇒ + + + 630 = x ⇒ x = = 15 L
10 10 100 0.133
100 x − 70 x − 9 x 240
⇒ 630 = 136.
(b) In normal time rate = = `6 per hour
100 40
21x During sale rates are increased by 50% i.e.,
⇒ 630 =
100 Rates 40-hour week = 240 + 50% of 240
⇒ x = `3000 = `360
129.
(c) Let the price of sugar be `x per kg. 360
\ Rate per hour = = `9 per hour
\ Initial expenditure = `30x 40
New expenditure = `33x Now, according to the question,
\ New monthly consumption Required commission = 9 × 60 = `540
33 x 137.
(c) Let the maximum marks be x.
= = 25 kg
1.32 x Then, 296 – 259 = 5% of x
130.
(b) Let his increased income be x. 5
⇒ x = 37
80 12 80 10 100
( x − 12000) × + = x× ×
100 100 100 100 ⇒ x = 740
⇒ 12x – 14400 = 10x 138.
(a) Marks obtained by Sushant = 1080
⇒ x = `7200 Marks obtained by Mohit
131.
(c) Let his monthly rent be x. = 1.2 × 1080 = 1296
12.5 5.5 1296
12 x − 12 x × − 325 = 100000 × Marks obtained by Rajesh = = 1440
100 100 0.9
⇒ 12x – 1.5x – 325 = 5500 So, percentage of marks obtained by Rajesh
5500 + 325 1440
⇒ x = ≈ `554.76 = × 100 = 72%
10.5 2000

Chapter_05.indd 120 1/30/2016 12:43:58 PM


121
Percentage 

139.
(c) Let Ram’s monthly income be x. \ New expenditure on water
80 85 70 6 4 24
Total savings = x × × × = ` x × y=` xy
100 100 100 5 5 25
100 100 100 \ Decrease in expenditure on water
⇒ x = 9520 × × ×
80 85 70  24  1
= `  xy − xy  = ` xy
= `20000  25  25
140.
(a) Let the quantity of haematite mined be x kg. Hence, percentage decrease
80 25 1
Then, x × × = 80000 xy
100
100 100 = 25 × 100 = = 4%
100 100 xy 25
⇒ x = 80000 × ×
80 25 Quicker Method: If the value is first increased
= 400000 kg by x% and then decreased by y% then there is

141.
(a) Total number of votes = 6000  xy 
x − y −  % increase or decrease, according to
Total number of valid votes  100 
= 6000 × 0.75 = 4500 the +ve or –ve sign respectively.
Total valid votes that Bhiku gets Here x = 20% and y = 20%
= 4500 × 0.65 = 2925 20 × 20
\ % effect = 20 – 20 – =–4
Total valid voted that Mhatre gets 100
Thus, there is 4% decrease in the expenditure of the
= 4500 – 2925 = 1575
money.
142.
(b) Required percentage correction
146.
(c) G = B + 15 ...(1)
1.25 − 1
= × 100 = 20% G + 10% of G = B + 16% of B + 9
1.25
Note: This question can be solved, even if his height
11G 116 B
i.e., = +9
is not given because there is no need of his height. 10 100
143.
(c) Required marks i.e., 50% of (180 + 150) = 165 i.e., 110G – 116B = 900 ...(2)
Marks scored in first paper = 54 Using (2) in (1), we get
Marks required to be scored in second paper = 111 B = 125, G = 140
111 \ Total number of students = B + G = 265.
\ Required percentage = × 100 = 74%
150 147.
(d) Let the original number be 10x + y
144.
(a) Difference in quantity (percentage) lost Here unit’s digit is y.
= 7% – 5.2% = 1.8% According to the question, unit’s digit of new number
Let the total quantity of wheat grown be x million = y + 100% of y = 3y
tonnes. Ten’s digit of new number
Then, = 10x + 50% of 10x = 15x
1.8% x = (6 + 3) Now, 15x + 3y – 10x – y = 19
9 × 100 or, 5x + 2y = 19
⇒ x = = 500 million tonnes
1.8
By trial and error method, we have x = 3 and y = 2
145.
(d) Suppose water tax = `x Since 5 × 3 + 2 × 2 = 19
Consumption of water = y litres \ Original number = 10 × 3 + 2 = 32
\ Original expenditure on water = `xy
Increased water tax = `(x + 20% of x) 148.
(c) Suppose there are 100 candidates for entrance.
6
\ No. of capable candidates = 40
= ` x
5 and no. of incapable candidates
Decreased consumption of water = 100 – 40 = 60
4 Now, no. of capable candidates who pass the test
= y – 20% of y = y litre
5 = 80% of 40 = 32

Chapter_05.indd 121 1/30/2016 12:43:59 PM


122  Chapter 5

No. of incapable candidates who pass the test \ Monthly amount saved
= 25% of 60 = 15 = `100 – (70 + 20) = `10
(Since these successful candidates become college Actual yearly savings
students.) = `4800
Thus, there are 32 + 15 = 47 colleges students in all,
of which 32 are capable. Actual monthly savings

Hence, proportion of capable college students 4800
= ` = `400
32 12
= × 100 ≈ 68%
47 If the amount saved is `10, monthly fellowship
149.
(b) Total candidates = 2000 = `100
No. of boys = 900 \ If the amount saved is `400 monthly fellowship
No. of girls = 1100 100
= ` × 400 = `4000.

No. of students who passed 10
32 × 900 38 × 1100 153.
(b) Number of literate women
= +
100 100 50 70
= 296000 × – 166000 ×
= 288 + 418 = 706 100 100
No. of students who failed = 2000 – 706 = 1294 = 148000 – 116200
1294 = 31800
Required percentage = × 100 = 64.7 %
2000 154.
(a) Let initial expenditure and savings be 3x and 2x
respectively.
150.
(b) Required per cent decrease
So, initial income = 3x + 2x = 5x
30 300 1
= × 100 = = 23 % New income = 5.5x
130 13 13
New expenditure = 3x × 1.12 = 3. 36x
151.
(c) Let the total number of workers be 100.
New savings = 5.5x – 3.36x = 2.14x
Number of skilled workers = 75% of 100 = 75
Percentage increase in savings
Number of unskilled workers = 100 – 75 = 25
2.14 x − 2 x

Number of permanent workers = 80% of 75 + 20% of = × 100 = 7%
25 2x
80 20 155.
(c) Total score of first three friends = 15 × 3 = 45
= × 75 + × 25
100 100 and total score of last three friends = 16 × 3 = 48
= 60 + 5 = 65 \ Total score of four friends = 45 + 19 = 64
Number of temporary workers = 100 – 65 = 35 \ Score of first friend = 64 – 48 = 16
The number of temporary workers are 35, then total 16 1
workers = 100 \ Required percentage = × 100% = 33 %
48 3
⇒ Number of total workers when number of
156.
(a) Required price of the single ticket
temporary workers is 126
84 100 100 100 100
100 = × × = 84 × × = `64
= × 126 =
360 105 1 125 105 125
35
152.
(d) Suppose monthly fellowship 157.
(a) Suppose original price = `100

= `100 Reduction = 20%


100 − 20
Amount spent on monthly expenses \ Reduced price = `100 ×
100
= `70
80
Amount spent on books = `100 × = `80
100
= `20

Chapter_05.indd 122 1/30/2016 12:44:00 PM


123
Percentage 

Increased in sale = 80%


100 + 80 2 x y
\ Increased sale = `80 × ⇒ x = y ⇒ = = K
180 5 2 5
⇒ x = 2K, y = 5K
180
= `80 × = `144 2K
100 \ Percentage of x to y is ×100 = 40
5K
Net increase in sale = `144 – `100
= `44 on `100 = 44% 159. (c) Given, x < y
158.
(d) Suppose the numbers are x, y and z. The per cent by which x is less than y
\ x = 20% of z, y = 50% of z y−x
= × 100
2 1 y
\ x = y ⇒y= z
5 2

Chapter_05.indd 123 1/30/2016 12:44:00 PM


6 Average

INTRODUCTION
Whenever we are asked the marks scored by us in any The average or mean or arithmetic mean of a number of
examination, we usually tell the marks in percentage, quantities of the same kind is equal to their sum divided by
taking the percentage of total marks of all subjects. This the number of those quantities. For example, the average of
percentage is called average percentage. Also, in a class, 3, 9, 11, 15, 18, 19 and 23 is
if there are 100 students, instead of knowing the age of 3  9  11  15  18  19  23 98
individual student, we usually talk about average age. = = 14.
7 7

soMe BAsiC FoRMulAe

1000 + 1500 + 2700


Sum of quantities Average price of 1 toy =
1. Average = 20
Number of quantities
5200
= = `260.
2. Sum of quantities = Average × Number of quantities 20
Sum of quantities Illustration 2 The average marks obtained by 200 students
3. Number of quantities = in a certain examination is 45. Find the total marks.
Average
Solution: Total marks
Illustration 1 A man purchased 5 toys at the rate of `200 = Average marks × Number of students
each, 6 toys at the rate of `250 each and 9 toys at the rate of = 200 × 45 = 900.
`300 each. Calculate the average cost of one toy. Illustration 3 Total temperature for the month of September
is 840°C. If the average temperature of that month is 28°C,
Solution: Price of 5 toys = 200 × 5 = `1000
find of how many days is the month of September.
Price of 6 toys = 250 × 6 = `1500
Solution: Number of days in the month of September
Price of 9 toys = 300 × 9 = `2700
Total temperature 840
Total number of toys = 5 + 6 + 9 = 20. = = = 30 days.
Average temperature 28

soMe useFul shoRt-Cut MethoDs

1. Average of two or more groups taken together respectively, the combined average (average
of all of them put together) is
(a) If the number of quantities in two groups
be n1 and n2 and their average is x and y, n1 x  n2 y
n1  n2

Chapter_06.indd 124 1/30/2016 1:26:28 PM


Average 125

Explanation Illustration 5 Average salary of all the 50 employees


No. of quantities in first group = n1 including 5 officers of a company is `850. If the average
Their average = x salary of the officers is `2500, find the average salary of the
remaining staff of the company.
∴ Sum = n1 × x
Solution: Here n1 = 50, n2 = 5, x = 850 and y = 2500.
No. of quantities in second group = n2
∴ Average salary of the remaining staff
Their average = y
n x  n2 y 50  850  5  2500
∴ Sum = n2 × y = 1 =
n1  n2 50  5
No. of quantities in the combined group = n1 + n2
42500  12500 30000
Total sum (sum of quantities of first group and second = =
group) 45 45
= n1x + n2y. = `667
667 (approx.)
∴ Average of the two groups
n x  n2 y 2. If x is the average of x1, x2, ..., xn, then
= 1 .
n1  n2 (a) The average of x1 + a, x2 + a, ..., xn + a is x + a.
(b) The average of x1 – a, x2 – a, ... xn – a is x – a.
(b) If the average of n1 quantities is x and the average (c) The average of ax1, ax2, ..., axn is a x ,
of n2 quantities out of them is y, the average of provided a ≠ 0.
remaining group (rest of the quantities) is x x x x
n1 x  n2 y (d) The average of 1 , 2 , ..., n is , provided
. a a a a
n1  n2 a ≠ 0.

Explanation Illustration 6 The average value of six numbers 7, 12, 17,


24, 26 and 28 is 19. If 8 is added to each number, what will
No. of quantities = n1
be the new average?
Their average = x
Solution: The new average = x + a
∴ Sum = n1 x
= 19 + 8 = 27.
No. of quantities taken out = n2
Their average = y Illustration 7 The average of x numbers is 5x. If x – 2 is
∴ Sum = n2y subtracted from each given number, what will be the new
average?
Sum of remaining quantities = n1x – n2y
No. of remaining quantities = n1 – n2 Solution: The new average x = – a
n x  n2 y = 5x – (x – 2) = 4x + 2.
∴ Average of remaining group = 1 . Illustration 8 The average of 8 numbers is 21. If each of
n1  n2
the numbers is multiplied by 8, find the average of a new
Illustration 4 The average weight of 24 students of section set of numbers.
A of a class is 58 kg whereas the average weight of 26 Solution: The average of a new set of numbers
students of section B of the same class is 60.5 kg. Find the = a x = 8 × 21 = 168.
average weight of all the 50 students of the class.
Solution: Here n1 = 24, n2 = 26, x = 58 and y = 60.5. 3. The average of n quantities is equal to x. If one of
∴ Average weight of all the 50 students the given quantities whose value is p, is replaced
n x  n2 y by a new quantity having value q, the average
= 1 becomes y, then q = p + n(y – x)
n1  n2
24  58  26  60.5 Illustration 9 The average weight of 25 persons is
=
24  26 increased by 2 kg when one of them whose weight is
1392  1573 2965 60 kg is replaced by a new person. What is the weight of
= = = 59.3 kg.
50 50 the new person?

Chapter_06.indd 125 1/30/2016 1:26:29 PM


126 Chapter 6

Solution: The weight of the new person Illustration 13 What is the average of squares of the
= p + n(y – x) natural numbers from 1 to 41?
= 60 + 25(2) = 110 kg. Solution: The required average
(n  1)(2n  1) (41  1)(2  41  1) 42  83
4. (a) The average of n quantities is equal to x. When = = =
a quantity is removed, the average becomes y. 6 6 6
The value of the removed quantity is n(x – y) + y. 3486
= = 581.
6
(b) The average of n quantities is equal to y. When
a quantity is added, the average becomes y. Illustration 14 Find the average of cubes of natural
The value of the new quantity is n(y – x) + y. numbers from 1 to 27.
Solution: The required average
Illustration 10 The average age of 24 students and the class n(n  1) 2 27  (27  1) 2 27  28  28
teacher is 16 years. If the class teacher’s age is excluded, the = = =
4 4 4
average age reduces by 1 year. What is the age of the class
21168
teacher? = = 5292.
4
Solution: The age of class teacher
= n(x – y) + y Illustration 15 What is the average of odd numbers from
= 25(16 – 15) + 15 1 to 40?
= 40 years. Solution: The required average
last odd number + 1 39 + 1
Illustration 11 The average age of 30 children in a class = = = 20.
is 9 years. If the teacher’s age be included, the average age 2 2
becomes 10 years. Find the teacher’s age. Illustration 16 What is the average of even numbers from
Solution: The teacher’s age 1 to 81?
= n(y – x) + y Solution: The required average
= 30(10 – 9) + 10 last even number + 2 80 + 2
= = = 41.
= 40 years. 2 2

5. (a) The average of first n natural numbers is 6. (a) If n is odd: The average of n consecutive
numbers, consecutive even numbers or
n 1
. consecutive odd numbers is always the
2 middle number.
(b) The average of square of natural numbers till (b) If n is even: The average of n consecutive
(n  1)(2n  1) numbers, consecutive even numbers or
n is .
6 consecutive odd numbers is always the
(c) The average of cubes of natural numbers till average of the middle two numbers.
n(n  1) 2 (c) The average of first n consecutive even
n is .
4 numbers is (n + 1).
(d) The average of odd numbers from 1 to n is (d) The average of first n consecutive odd
last odd number + 1 numbers is n.
. (e) The average of squares of first n consecutive
2
(e) The average of even numbers from 1 to n is 2(n +1)(2n  1)
even numbers is .
last even number + 2 3
. (f) The average of squares of consecutive even
2
(n  1)(n  2)
numbers till n is .
Illustration 12 Find the average of first 81 natural numbers.. 3
Solution: The required average (g) The average of squares of consecutive odd
n  1 81  1 n(n  2)
= = = 41. numbers till n is .
2 2 3

Chapter_06.indd 126 1/30/2016 1:26:31 PM


Average 127

Solution: The required difference


(h) If the average of n consecutive numbers is
= 2(n – 1) = 2(6 – 1) = 10.
m, then the difference between the smallest
and the largest number is 2(n – 1).
7. Geometric mean or geometric average
Geometric mean of x1, x2, ..., xn is denoted by
Illustration 17 Find the average of 7 consecutive numbers
3, 4, 5, 6, 7, 8, 9. G.M. = n x1  x2  ...  xn .
Solution: The required average = middle number = 6. Geometric mean is useful in calculating averages
Illustration 18 Find the average of consecutive odd of ratios such as average population growth rate,
numbers 21, 23, 25, 27, 29, 31, 33, 35. average percentage increase and so on.
Solution: The required average
Illustration 25 The production of a company for three
= average of middle two numbers
successive years has increased by 10%, 20% and 40%,
= average of 27 and 29 respectively. What is the average annual increase of
27  29 production?
= = 28.
2 Solution: Geometric mean of x, y and z = (x × y × z)1/3.
Illustration 19 Find the average of first 31 consecutive ∴ Average increase = (10 × 20 × 40)1/3% = 20%.
even numbers.
Solution: The required average = (n + 1) = 31 + 1 = 32. Illustration 26 The population of a city in two successive
years increases at the rates of 16% and 4%, respectively.
Illustration 20 Find the average of first 50 consecutive Find the average increase of two years.
odd numbers. Solution: In case of population increase, the geometric
Solution: The required average = n = 50. mean is required.
Illustration 21 Find the average of squares of first 19 ∴ Geometric mean of 16% and 4% is
consecutive even numbers. = (16 × 4)1/2 %, i.e., 8%.
Solution: The required average
2(n  1)(2n  1) 2(19  1)(2  19  1) 8. Harmonic mean or harmonic average
= = Harmonic means of x1, x2, ..., xn is denoted by
3 3
2  20  39 1560 1
= = = 520. H.M. =
3 3 1 1 1 1
  ...  
Illustration 22 Find the average of squares of consecutive n  x1 x2 xn 
even numbers from 1 to 25. Harmonic mean is useful for finding out average
Solution: The required average speed of a vehicle, average production per day
(n  1)(n  2) (25  1)(25  2) and so on.
= =
3 3
26  27 702 Illustration 27 A man runs 1 km at 15 km per hour and
= = another 1 km he walks at 5 km per hour. Find his average
3 3
speed for the whole distance in covering 2 km.
= 234.
Solution: When the distance is constant and the speed
Illustration 23 Find the average of squares of consecutive
varies, harmonic mean is used. Harmonic mean of x and y
odd numbers from 1 to 31.
2 2xy
Solution: The required average is or .
1 1 x y
n(n  2) 31  (31  2) 31  33 
= = = = 341. x y
3 3 3 ∴ Average speed for the whole distance
Illustration 24 If the average of 6 consecutive numbers
is 48, what is the difference between the smallest and the 2  15  5
= = 7.5 km/hr.
largest number? 15  5

Chapter_06.indd 127 1/30/2016 1:26:32 PM


128 Chapter 6

Illustration 31 A person covers 9 km at a speed of 3 km/hr,


9. If a certain distance is covered at a speed of x km/ 25 km at a speed of 5 km/hr and 30 km at a speed of 10 km/
hr and the same distance is covered at a speed of y hr. Find the average speed for the entire journey.
km/hr, the average speed during the entire journey is
 A BC 
 2xy  Solution: The average speed =  A B C 
 x  y  km/hr.    
 x y z
 9  25  30 
Illustration 28 If half of the journey is travelled at a speed =  9 25 30 
of 15 km/hr and the next half at a speed of 12 km/hr, find the    
3 5 10 
average speed during the entire journey.
64 9
Solution: The average speed = = 5 km/hr.
11 11
 2xy   2  15  12 
=  =  
 x  y   15  12  12. If a person covers Ath part of the distance at x
km/hr, Bth part of the distance at y km/hr and the
360 1
= = 13 km/hr. remaining Cth part at z km/hr, then the average
27 3 speed during the entire journey is
Illustration 29 A man goes to a certain place at a speed of
 1 
30 km/hr and returns to the original place at a speed of 20 km/
 A B C  km/hr.
hr, find the average speed during up and down journey.    
Solution: The average speed  x y z
 2xy   2  30  20  1200
=  =   = = 24 km/hr.
 x  y   30  20  50 Illustration 32 A person covers the first one-forth of the
distance at 8 km/hr, the next three-fifths at 6 km/hr and the
10. If a person or a motor car covers three equal remaining distance at 15 km/hr. Find the average speed
distances at the speed of x km/hr, y km/hr and z during the entire journey.
km/hr, respectively, then for the entire journey Solution: The average speed
average speed of the person or motor car is 1  1 
 3xzy  = =  1/4 3/5 3/20 
 A B C
 xy  yz  zx  km/hr.  x  y  z     
8 6 15 

Illustration 30 A train covers the first 160 kms at a speed  1 3  1 3 3 


 Here, A 4 , B  5 and C = 1   4  5   20 
of 120 km/hr, another 160 kms at 140 km/hr and the last 160  
kms at 80 km/hr. Find the average speed of the train for the 1 3200 9
entire journey. = 1 1 1 = 452 = 7 113 km/hr.
 
Solution: Average speed 32 10 100
3xyz 3  120  140  80
= = Illustration 33 A train covers 50% of the journey at 30 km/
xy  yz  zx 120  140  140  80  80  120
hr, 25% of the journey at 25 km/hr and the remaining at
360  140  80 20 km/hr. Find the average speed of the train during the
=
16800  11200  9600 entire journey.
4032000 11 Solution: The average speed
= = 107 km/hr.
37600 47
 100   100 
11. If a person covers A km at a speed of x km/hr, B km =  A B C  =  50 25 25 
       
at a speed of y km/hr and C km at a speed of z km/  x y z 30 25 20 
hr, the average speed during the entire journey is
[Here A = 50, B = 25 and C = 25]
 A B C 
 A B C  km/hr. 100 1200 25
    = = = 25 km/hr.
 x y z 47/12 47 47

Chapter_06.indd 128 1/30/2016 1:26:33 PM


129
Average 

Multiple Choice QuestionS

1. The average of 11 numbers is 10.9. If the average of the (a) 47 (b) 51


first six numbers is 10.5 and that of the last six numbers is (c) 53 (d) None of the above
11.4, then the middle number is [Based on FMS (Delhi), 2004]
(a) 11.5 (b) 11.4
8. There is a sequence of 11 consecutive odd numbers. If the
(c) 11.3 (d) 11.0 average of first 7 numbers is X, then find the average of all
[Based on MAT, 2003] the 11 integers.
2.
An officer’s penson on retirement from service is equal to (a) X + 3 (b) X + 4
half the average salary during last 36 months of his service. (c) X + 5 (d) X + 7
His salary from 1 January, 1954 is `380 per month with [Based on FMS (Delhi), 2004]
increment of `40 on 1 October, 1954, 1 October, 1955 and
9. There are 197 boys and 591 girls in a college. If the
1 October, 1956. If he retires on 1 January, 1957, what
average weight of boys is 63 kg and average weight of
pension does he draw?
girls is 51 kg, then what is the average weight of the
(a) `210 (b) `215 students in the college?
(c) `220 (d) `225 (a) 56 kg (b) 57 kg
3.
A hiker walked for two days. On the second day, the hiker (c) 54 kg (d) 60 kg
walked 2 hrs longer and at an average speed of 1 km per [Based on IIT Joint Man. Ent. Test, 2004]
hour faster than he walked on the first day. If during the
10. A student on his birthday distributed on an average 5
two days he walked a total of 64 km and spent a total of 18
chocolates per student. If on the arrival of the teacher
hrs walking, what was his average speed on the first day?
and the headmaster to whom the student gives 10 and 15
(a) 2 km/h (b) 3 km/h chocolates respectively, the average chocolate distributed
(c) 4 km/h (d) 5 km/h per head increases to 5.5, then what is the strength of the
4.
In a class of 100 students the average marks obtained by class?
a student in Maths is 44. If we remove the highest and (a) 28 (b) 30
the lowest the average of the class becomes 43.92. A total (c) 32 (d) None of these
of three students get either highest or lowest. What is the [Based on IIT Joint Man. Ent. Test, 2004]
average of these three students?
11. The average temperature of the town in the first four
(a) 73.33 (b) 46.66 days of a month was 58 degrees. The average for the
(c) 59.99 (d) Cannot be determined second, third, fourth and fifth days was 60 degrees. If the
5.
Neeta’s attendance for first two semesters out of four temperatures of the first and fifth days were in the ratio
was 60% and 70%, respectively. What is the minimum 7:8, then what is the temperature on the fifth day?
attendance required in third semester so that her average (a) 240 degrees (b) 232 degrees
attendance will be 80% throughout four semesters? (c) 64 degrees (d) None of these
(Assume equal number of days among the four semesters) [Based on Narsee Manjee Inst. of Man. Studies, 2003]
(a) 70% (b) 80%
12. If the mean of a, b, c is M and ab + bc + ca = 0, then the
(c) 90% (d) None of these mean of a2, b2, c2 is
6. A car owner buys petrol at `7.50, `8.00 and `8.50 per (a) M2 (b) 3M2
litre for three successive years. What approximately is the (c) 6M2 (d) 9M2
average cost per litre of petrol if he spends `4000 each [Based on IITTM, Gwalior, 2003]
year?
13.
A is the set of first ten consecutive natural numbers.
(a) `8 (b) `9
Find the number of ways in which a subset B can be
(c) `7.98 (d) `8.50 formed out of set A, such that the sum of all the elements
[Based on MAT, 2001] in B is odd.
7. In a one-day cricket match, Agarkar, Sehwag, Sachin, (a) 761 (b) 763
Dravid and Ganguly scored an average of 39 runs. Dravid (c) 765 (d) 767
scored 7 more than Ganguly. Ganguly scored 9 fewer than
Agarkar. Sehwag scored as many as Dravid and Ganguly 14. An aeroplane flies along the four sides of a square field at
combined; and Sehwag and Sachin scored 110 runs the speeds of 200, 400, 600 and 800 km/hr. The average
between them. How many runs did Sachin score? speed of the plane around the field in km/hr is

Chapter_06.indd 129 1/30/2016 1:26:33 PM


130  Chapter 6

(a) 384 (b) 400 22. For 10 hrs, a train travels at a constant speed of 20 miles
(c) 500 (4) 284 per hour and during the next 15 hrs, it travels 240 miles.
[Based on FMS (Delhi), 2003] What is the average speed of the train for the whole
journey?
15. The average of marks obtained by 120 candidates was 35.
(a) 17.6 miles/hr (b) 20.8 miles/hr
If the average of the passed candidates was 39 and that
of the failed candidates was 15, then the number of those (c) 130 miles/hr (d) 176 miles/hr
candidates, who passed the examination, was [Based on IMT Ghaziabad, 2002]

(a) 100 (b) 110 23. Average of 10 positive numbers is X . If each number
(c) 120 (d) 150 increases by 10%, then X
[Based on FMS (Delhi), 2003] (a) Remains unchanged
16. Three years ago, the average age of A and B was 18 years. (b) Is increased by 10%
With C joining them, the average age becomes 22 years. (c) May decrease
How old is C now? (d) May either increase or decrease
(a) 24 years (b) 27 years [Based on IMT Ghaziabad, 2002]
(c) 28 years (d) 30 years 24.
On an 800 miles trip, car W travelled half the distance at
[Based on FMS (Delhi), 2003] 80 miles per hour and the other half at 100 miles per hour.
What was the average speed of the car?
17. The average of three numbers is 135. The largest number
is 180 and the difference of the other numbers is 25. The (a) 18.00 (b) 180.00
smallest number is 8
(c) 90.00 (d) 88
9
(a) 130 (b) 125
[Based on Narsee Monjee Inst. of Man. Studies, 2002]
(c) 120 (d) 100
25. Having scored 98 runs in the 19th innings, a cricketer
[Based on IIFT, 2003]
increases his average score by 4. What will be his average
18. The ratio of the arithmetic mean of two numbers to one of score after the 19th innings?
the numbers is 3:5. What is the ratio of the smaller number (a) 28 (b) 26
to the larger one?
(c) 24 (d) 22
(a) 1:5 (b) 1:4 [Based on I.P. Univ., 2002]
(c) 1:3 (d) 1:2 26. Angad was conducting an experiment in which the average
[Based on IIFT, 2003] of 11 observations came to be 90, while the average of
19. A person travels from X to Y at a speed of 40 kmph and first five observations was 87, and that of the last five was
returns by increasing his speed by 50%. What is his 84. What was the measure of the 6th observation?
average speed for both the trips? (a) 165 (b) 150
(a) 36 kmph (b) 45 kmph (c) 145 (d) 135
(c) 48 kmph (d) 50 kmph [Based on I.P. Univ., 2002]
[Based on IIFT, 2003] 27. A student who scored 30% marks in the first paper of
Physics out of 180 marks, has to get an overall score of
20.
A painter is paid x rupees for painting every 10 metres of
at least 50% in two papers, the second paper carrying 150
a wall and y rupees for painting every extra metre. During
marks. What percentage of marks should he score in the
one week, he painted 10 metres on Monday, 13 metres on
second paper to get the overall average score?
Tuesday, 12 metres on Wednesday, 11 metres on Thursday
and 12 metres on Friday. What is his average daily earning (a) 80% (b) 76%
in rupees for the five day week? (c) 74% (d) 70%
(a) x + (8/5) y (b) (5x + 9y)/5 [Based on I.P. Univ., 2002]

(c) 10x + (8/5)y (d) 5x + 8y 28.


The captain of a cricket team of 11 players is 25 years
[Based on SCMHRD Ent. Exam., 2003]
old and the wicket keeper is 3 years elder to the captain.
If the ages of these two are excluded, the average age of
21.
The average of 5 consecutive numbers is n. If the next two the remaining players is 1 year less than the average age
numbers are also included,. the average will of the whole team. What is the average age of the whole
(a) Increase by 1 (b) Remain the same team?
(c) Increase by 1.4 (d) Increase by 2 (a) 21.5 years (b) 22 years
[Based on MAT, 2005] (c) 22.5 years (d) 32 years

Chapter_06.indd 130 1/30/2016 1:26:33 PM


131
Average 

29. A batsman has a certain average of runs for 12 innings. 37.


The average income of A, B and C is `12,000 per month
In the 13th innings he scores 96 runs, thereby increasing and the average income of B, C and D is `15,000 per
his average by 5 runs. What is his average after the 13th month. If the average salary of D be twice that of A, then
innings? the average salary of B and C is (in `)
(a) 64 (b) 48 (a) 8,000 (b) 18,000
(c) 36 (d) 72 (c) 13,500 (d) 9,000
[Based on I.P. Univ., 2002]
38.
There were 35 students in a hostel. If the number of
30.
Average of n numbers is 36 where n is a multiple of 4. students increases by 7, the expenses of mess increase
If three-fourth of the numbers are increased by 4 and the by `42 per day while the average expenditure per head
remaining decreased by 4, what is the new average? diminshes by Re 1. Find the original expenditure of the
(a) 40 (b) 37.5 mess.
(c) 38 (d) None of these (a) `320 (b) `420
31. The average of 7 consecutive numbers is 20. The largest (c) `160 (d) `158
of these numbers is
39. The average of marks of 14 students was calculated as 71.
(a) 24 (b) 23 But it was later found that the marks of one student had
(c) 22 (d) 20 been wrongly entered as 42 instead of 56 and of another
[Based on SSC (GL) Prel. Exam, 2000] as 74 instead of 32. The correct average is
32. The average age of 14 girls and their teacher is 15 years. (a) 67 (b) 68
If the teacher’s age is excluded, the average reduces by 1. (c) 69 (d) 71
What is the teacher’s age? [Based on SSC (GL) Prel. Exam, 2000]
(a) 35 years (b) 32 years
40.
Of the four numbers whose average is 60, the first is
(c) 30 years (d) 29 years one-fourth of the sum of the last three. The first number is
[Based on SSC (GL) Prel. Exam, 2000]
(a) 15 (b) 45
33. The average age of four brothers is 12 years. If the age of
(c) 48 (d) 60.25
their mother is also included, the average is increased by
5 years. The age of the mother (in years) is [Based on SSC (GL) Prel. Exam, 2000]

(a) 37 years (b) 43 years 41. Average age of father and his two sons is 27 years. Five
(c) 48 years (d) 53 years years ago, the average age of the two sons was 12 years. If
[Based on SSC (GL) Prel. Exam, 2000]
the difference between the ages of the two sons is 4 years,
then the present age of the father is
34. The average age of A and B is 20 years, that of B and C is
(a) 34 years (b) 47 years
19 years and that of A and C is 21 years. What is the age
(in years) of B? (c) 64 years (d) 27 years
(a) 39 (b) 21 [Based on SSC (GL) Prel. Exam, 2000]
(c) 20 (d) 18 42. The average age of 30 boys in a class is 15 years. One boy,
[Based on SSC (GL) Prel. Exam, 2000] aged 20 years, left the class, but two new boys came is his
place whose ages differ by 5 years. If the average age of
35. There are in all 10 balls; some of them are red and others
all the boys now in the class becomes 15 years, the age of
white. The average cost of all balls is `28. If the average
the younger newcomer is
cost of red balls is `25 and that of white balls is `30, then
the number of white balls is (a) 20 years (b) 15 years
(a) 3 (b) 5 (c) 10 years (d) 8 years
(c) 6 (d) 7 [Based on SSC (GL) Prel. Exam, 2002]
[Based on SSC (GL) Prel. Exam, 2000]
43. Out of three numbers, the first is twice the second and is
36.
The average age of a group of 14 persons is 27 years and half of the third. If the average of the three numbers is 56,
9 months. Two persons, each 42 years old, left the group. then difference of first and third numbers is
What will be the average age of the remaining persons in (a) 12 (b) 20
the group?
(c) 24 (d) 48
(a) 26.875 years (b) 26.25 years
[Based on SSC (GL) Prel. Exam, 2002]
(c) 25.375 years (d) 25 years

Chapter_06.indd 131 1/30/2016 1:26:33 PM


132  Chapter 6

44. Of the three numbers second is twice the first and is also 51.
Manmohan calculated the average of 10, ‘three digit
twice the third. If the average of the three numbers is 44, numbers’. But due to mistake he reversed the digits of
the largest number is a number and thus his average increased by 19.8. The
(a) 24 (b) 72 difference between the unit digit and hundreds digit of
that number is
(c) 36 (d) 108
(a) 8 (b) 4
[Based on SSC (GL) Prel. Exam, 2002]
(c) 2 (d) Cannot be determined
45. The average of 8 men is increased by 2 years when 2 of
52. The average marks scored by Ganesh in English, Science,
them whose ages are 21 and 23 years are replaced by two
Mathematics and History is less than 15 from that scored
new men. The average age of two new men is
by him in English, History, Geography and Mathematics.
(a) 22 years (b) 24 years What is the difference of marks in Science and Geography
(c) 28 years (d) 30 years secured by him?
[Based on SSC (GL) Prel. Exam, 2002] (a) 40 (b) 50
46. The average age of A and B is 30 years, that of B and C is (c) 60 (d) Data inadequate
32 years, and the average age of C and A is 34 years. The [Based on BSRB Chennai PO, 2000]
age of C is 53. A Mathematics teacher tabulated the marks secured by 35
(a) 33 years (b) 34 years students of 8th class. The average of their marks was 72. If
the marks secured by Reema was written as 36 instead of
(c) 35 years (d) 36 years
86 then find the correct average marks up to two decimal
[Based on SSC (GL) Prel. Exam, 2002] places.
47. In a certain year, the average monthly income of a person (a) 73.41 (b) 74.3
is `3400 and that for the first eight months is `3160 and (c) 72.43 (d) 73.43
for the last five months is `4120. The income in the eight [Based on BSRB Bangalore PO, 2000]
month of the year is
54.
The average age of a husband and wife was 23 when
(a) `5080 (b) `6080 they were married 5 years ago. The average age of the
(c) `5180 (d) `3880 husband, the wife and a child who was born during the
[Based on SSC (GL) Prel. Exam, 2003] interval, is 20 years now. How old is the child now?
(a) 9 months (b) 1 year
48. Average age of 6 sons of a family is 8 years. Average age
of sons together with their parents is 22 years. If the father (c) 3 years (d) 4 years
is older than the mother by 8 years, the age of the mother 55. The average of four consecutive even numbers is one-
(in years) is fourth of the sum of these numbers. What is the difference
(a) 44 (b) 52 between the first and the last number?
(c) 60 (d) 68 (a) 4 (b) 6
[Based on SSC (GL) Prel. Exam, 2003] (c) 2 (d) Cannot be determined
[Based on BSRB Delhi PO, 2000]
49. A grocer has a sale of `6435, `6927, `6855, `7230 and
`6562 for 5 consecutive months. How much sale must he 56. Of the three numbers, the average of the first and the
have in the sixth month so that he gets an average sale of second is greater than the average of the second and the
`6500? third by 15. What is the difference between the first and
the third of the three numbers?
(a) `4991 (b) `5991
(a) 15 (b) 45
(c) `6991 (d) `6001
(c) 60 (d) None of these
[Based on SSC (GL) Prel. Exam, 2003]
[BABARD Asst. Manager Exam, 2002]
50. The average weight of three men A, B and C is 84 kg. D 57. The average of 25 results is 18, that of first 12 is 14 and of
joins them and the average weight of the four becomes the last 12 is 17. Thirteenth result is
80 kg. If E, whose weight is 3 kg more than that of D,
(a) 72 (b) 78
replaces A, the average weight of B, C, D and E becomes
79 kg. The weight of A is (c) 85 (d) 28
[Based on CANARA BANK PO, 2003]
(a) 65 kg (b) 70 kg
(c) 75 kg (d) 80 kg 58. Average age of seven persons in a group is 30 years. The
average age of five persons of this group is 31 years. What
[Based on SSC (GL) Prel. Exam, 2003]
is the average age of the other two persons in the group?

Chapter_06.indd 132 1/30/2016 1:26:34 PM


133
Average 

(a) 55 years (b) 26 years 65.


The average of 5 consecutive odd numbers A, B, C, D and
(c) 15 years (d) None of these E is 41. What is the product of A and E?
[Based on PNB Management Trainee Exam, 2003] (a) 1677 (b) 1517
1 (c) 1665 (d) 1591
59. Average weight of three boys P, T and R is 54 kg while [Based on MAT (May), 2009]
3
the average weight of three boys T, F and G is 53 kg. 66.
The average age of a woman and her daughter is 42 year.
What is the average weight of P, T, R, F and H? The ratio of their ages is 2:1 respectively. What is the
(a) 53.8 kg (b) 52.4 kg daughter’s age?
(c) 53.2 kg (d) Data inadequate (a) 28 year (b) 48 year
[Based on Bank of Maharashtra, 2003] (c) 52 year (d) 32 year
[Based on MAT (May), 2009]
60. In a class of 52 students the number of boys is two less
than the number of girls. Average weight of the boys is 67.
The average weight of 29 students in a class is 48 kg. If
42 kg, while the average weight of all the 52 students is the weight of the teacher is included, the average weight
40 kg. Approximately what is the average weight of the rises by 500 g. Find the weight of the teacher.
girls? (a) 57 kg (b) 60 kg
(a) 41 kg (b) 29 kg (c) 65 kg (d) 63 kg
(c) 40 kg (d) 38 kg [Based on MAT (Feb), 2009]
[Based on IBPS, 2003]
68.
The average of marks obtained by 120 candidates was
61.
While calculating the average of a batsman as 36 in 100 35. If the average of the passed candidates was 39 and
matches that he played, one of the score 90 was incorrectly that of the failed candidates was 15, then the number of
noted as 40. The percentage error is candidates who passed the examination was
(a) 0.6% (b) 1.36% (a) 100 (b) 110
(c) 1.34% (d) 1.21% (c) 120 (d) 150
[Based on MAT (Sept), 2010] [Based on MAT (May), 2008]
62.
The average sale of a car dealership was 15 cars per week. 69.
There are twice the number of two wheelers as there are
After a promotional scheme, the average sale increased to three wheelers and the number of 4 wheelers are equal
21 cars per week. The percentage increase in the sale of to the number of two wheelers. The average number of
cars was wheels per vehicle is
(a) 40% (b) 140% (a) 3 (b) 4
6 (c) 5 (d) None of these
(c) 42 % (d) 39.33%
7 70.
The mean of five observations is 4 and their variance 5.2.
[Based on MAT (Feb), 2010] If three of these observations are 1, 2 and 6, then the other
63.
In a class with a certain number of students, if one two are
new student weighing 50 kg is added, then the average (a) 2 and 9 (b) 3 and 8
weight of the class increased by 1 kg. If one more student (c) 4 and 7 (d) 5 and 6
weighing 50 kg is added, then the average weight of the
[Based on MAT (Feb), 2008, (Sept), 2007]
class increases by 1.5 kg over the original average. What
is the original weight (in kg) of the class? 71.
The average of 11 results in 50. If the average of
(a) 46 (b) 42 first six results is 49 and that of last six 52, the sixth
result is
(c) 27 (d) 47
[Based on MAT (Sept), 2009] (a) 60 (b) 56
(c) 64 (d) 70
64. The average marks of a student in 8 subjects is 87. Of
[Based on MAT (Feb), 2008]
these, the highest marks is 2 more than the one next in
value. If these two subjects are eliminated, the average 72.
The average temperature from Monday to Thursday is 48°
marks of the remaining subjects is 85. What is the highest and from Tuesday to Friday is 52°. If the temperature on
marks obtained by him? Monday is 42°, what was it on Friday?
(a) 94 (b) 91 (a) 55° (b) 52°
(c) 89 (d) 96 (c) 58° (d) 51°
[Based on MAT (Sept), 2009] [Based on MAT (Feb), 2008]

Chapter_06.indd 133 1/30/2016 1:26:34 PM


134  Chapter 6

73. An investor earns 3% returns on one-fourth of his capital, 80. The average monthly expenditure of a family was `2200
5% on two-thirds and 11% on the remainder. What is the during the first 3 months; `2250 during the next 4 months
average rate of return he earns on his total capital? and `3120 during the last 5 months of a year. If the total
(a) 10% (b) 5% savings during the year were `1260, then the average
monthly income was
(c) 5.5% (d) 10.5%
(a) `2605 (b) `2805
[Based on MAT (Feb), 2008]
(c) `2705 (d) `2905
74.
A batsman has a certain average of runs for 12 innings. [Based on MAT (May), 2006]
In the 13th inning, he scores 96 runs thereby increasing
his average by 5 runs. What is his average after the 13th 81.
Ram spends `3620 for buying pants at the rate of
innings? `480 each and shirts at the rate of `130 each. What will
be the ratio of pants to shirts when maximum number of
(a) 64 (b) 48
pants are to be bought?
(c) 36 (d) 72
(a) 7:2 (b) 7:3
[Based on MAT (Feb), 2008]
(c) 2:7 (d) None of these
75.
There was one mess for 30 boarders in a certain hostel. If [Based on MAT, 1999]
the number of boarders was increased by 10, the expenses
of the mess increased by `40 per month, while the average 82.
The average weight of 45 students in a class is
expenditure per head diminished by `2. Find the original 52 kg. 5 of them whose average weight is 48 kg leave the
monthly expenses. class and other 5 students whose average weight is 54 kg
join the class. What is the new average weight (in kg) of
(a) `390 (b) `360 the class?
(c) `410 (d) `480 2
(a) 52.6 (b) 52
[Based on MAT (Dec), 2007] 3
76.
The mean of 25 observations was found to be 78.4. But 1
(c) 52 (d) None of these
later on it was found that 96 was misread as 69. The 3
correct mean is [Based on MAT, 1999]
(a) 79.48 (b) 76.54 83.
If p, q, r be three positive numbers such that p > q > r
(c) 81.32 (d) 78.4 when the smallest number is added to the difference of the
rest two numbers, then the average of the resultant number
[Based on MAT (Sept), 2007]
and the original numbers except to the smallest number is
77.
The average age of a family of 6 members is 22 years. 21 more than the average of all the three original numbers.
If the age of the youngest member be 7 years, what was The value of (p – q) is
the average age of the family at the birth of the youngest (a) 7 (b) 14
members?
(c) 63 (d) 42
(a) 15 (b) 18
84.
Of the three numbers, the first is twice the sec­ond and
(c) 21 (d) 22
the second is twice the third. The average of these three
78.
The average score of boys in an examination in a school numbers is 21. Find the largest number.
is 71 and that of the girls is 73. The average score of the (a) 36 (b) 38
school is 71.8. The ratio of the number of boys to that of (c) 47 (d) 48
the girls that appeared in the examination is
[Based on MAT, 1999]
(a) 1:2 (b) 3:2
85.
The average of three numbers is 135. The largest number
(c) 2:3 (d) 4:2 is 180 and the difference of the others is 25. The smallest
[Based on MAT (May), 2006] number is x
79.
The mean monthly salary paid to 75 workers in a factory (a) 130 (b) 125
is `5680. The mean salary of 25 of them is `5400 and that (c) 120 (d) 100
of 30 others is `5700. The mean salary of the remaining [Based on MAT, 1999]
workers is
86.
The batting average for 40 innings of a cricket player is
(a) `5000 (b) `7000
50 runs. His highest score exceeds his lowest score by 172
(c) `6000 (d) `8000 runs. If these two innings are excluded, the average of the
[Based on MAT (May), 2006] remaining 38 innings is 48. His highest score was

Chapter_06.indd 134 1/30/2016 1:26:34 PM


135
Average 

(a) 172 (b) 173 94.


The average of nine numbers is M and the average of three
(c) 174 (d) 176 of these is P. If the average of remaining numbers is N,
[Based on MAT, 1999] then
(a) M = N + P (b) 2M = N + P
87.
The average marks of a students in 10 papers are 80. If
the highest and the lowest scores are not considered, the (c) 3M = 2N + P (d) 3M = 2 P + N
average is 81. If the highest score is 92, find the lowest? [Based on SNAP, 2007]
(a) 55 (b) 60 95.
The average of any 5 consecutive odd natural numbers is
(c) 62 (d) Cannot be determined k. If two more such numbers, just next to the previous 5
numbers are added, the new average becomes
88.
The average age of Sachin and Ganguly is 35 years. If Kaif
replaces Sachin, the average age becomes 32 years and 2
(a) (k  1) (b) 2k – 3
if Kaif replaces Ganguly, then the average age becomes 7
38 years. If the average age of Dhoni and Irfan be half (c) 2k + 1 (d) k + 2
of the average age of Sachin, Ganguly and Kaif, then the 96.
The average age of A and B is 20 years. If C were to replace
average age of all the five people is A, the average would be 19 and if C were to replace B, the
(a) 28 years (b) 32 years average would be 21. What are the ages of A, B and C?
(c) 25 years (d) None of these (a) 22, 18, 20 (b) 18, 19, 20
89.
Nine persons went to a hotel for taking their meals. Eight (c) 22, 20, 17 (d) Cannot be determined
of them spent `12 each over their meals and the ninth
97.
If the algebraic sum of deviations of 20 observations
spent `8 more than the average expenditure of all the nine.
measured from 23 is 70, mean of these observations
Total money spent by them was
would be
(a) `104 (b) `105
(a) 24 (b) 25
(c) `116 (d) `117
(c) 26 (d) None of these
[Based on MAT, 1999]
[Based on SNAP, 2010]
90.
Out of three numbers, the first is twice the sec­ond and is
half of the third. If the average of the three numbers is 56, 98.
A cricketer has completed 20 innings and his average
the three numbers in order are is 44.5 runs. How many runs must be make in his next
innings so as to raise his average to 45?
(a) 48, 96, 24 (b) 48, 24, 96
(a) 45 (b) 60
(c) 96, 24, 48 (d) 96, 48, 24
[Based on MAT, 2000] (c) 40 (d) 55

91.
If a, b, c, d, e are five consecutive odd numbers, their 99.
The average of 6 numbers is 30. If the average of first four
average is is 25 and that of the last three is 35, the fourth number is
(a) 5 (a + 4) (b) a b c d e/5 (a) 35 (b) 30
(c) 5 (a + b + c + d + e) (d) None of these. (c) 25 (d) 20
[Based on MAT, 2000] [Based on FMS, 2006]

92.
The average weight of three men A, B and C is 84 kg. 100.
The average of three consecutive odd numbers is 12
Another man D joins the group and the average now more than one-third of the first of these numbers.
becomes 80 kg. If another man E, whose weight is 3 kg What is the last of the three numbers?
more than that of D, replaces A, then the average weight (a) 15 (b) 17
of B, C, D and E becomes 79 kg. The weight of A is
(c) 19 (d) Data inadequate.
(a) 70 kg (b) 72 kg [Based on SSC (GL), 2011]
(c) 75 kg (d) 80 kg.
[Based on MAT, 2000] 101.
Given the set of n numbers, n > 1, of which one is 1 –
(1/n), and all the others are 1. The arithmetic mean of the
93.
In a mathematics exam, a student scored 30% marks in the
n numbers is
first paper out of a total of 180. How much should he score
in the second paper out of a total of 150, if he is to get an 1
(a) 1 (b) n 
overall average of at least 50%? n
(a) 74% (b) 76% 1 1
(c) n  2
(d) 1 
(c) 70% (d) 80% n n2
[Based on MAT, 2000] [Based on FMS, 2010]

Chapter_06.indd 135 1/30/2016 1:26:34 PM


136  Chapter 6

102.
In a B-School, there are three levels of faculty positions 107. The average weight of 45 students in a class was calculated
i.e., Professor, Associate Professor and Assistant as 36 kg. It was later found that the weight of two students
Professor. It is found that the sum of the ages of all faculty in the class was wrongly calculated. The actual weight of
present is 2160, their average age is 36; the average age one of the boys in the class was 32 kg but it was calculated
of the Professor and Associate Professor is 39; of the as 34 kg, and the weight of another boy in the class was 45
8 kg whereas it was calculated as 40 kg. What is the actual
Associate Professor and Assistant Professor is 32 ; of
11 average weight of the 45 students in the class? (Rounded
2 off to two-digits after decimal)
the Professor and Assistant Professor is 36 . Had each (a) 36.07 kg (b) 36.16 kg
3
Professor been 1 year older, each Associate Professor 6 (c) 35.84 kg (d) Cannot be determined
year older and each Assistant Professor 7 year older, then [Based on Punjab National Bank PO, 2010]
their average age would increase by 5 years. What will be
108. The cost of 5 kg of apples is `450. The cost of 12 dozen
the number of faculty at each level and their average ages?
mangoes is `4320 and the cost of 4 kg of oranges is
(a) (16, 24, 20:45, 35, 30 year) `240. What is the total cost of 8 kg of apples, 8 dozens of
(b) (18, 24, 20:42, 38, 30 year) mangoes and 8 kg of oranges?
(c) (16, 20, 24:50, 30, 30 year) (a) `4020 (b) `4080
(d) None of these (c) `4000 (d) `4050
[Based on IIFT, 2010] [Based on Punjab National Bank PO, 2010]
103.
Out of 4 numbers, whose average is 60, the first one 109. 12% of Kaushal’s monthly salary is equal to 16% of
is one-fourth of the sum of the last three. The first number Nandini’s monthly salary. Suresh’s monthly salary is half
is that of Nandini’s monthly salary. If Suresh’s annual salary
(a) 15 (b) 45 is `1.08 lacs. What is Kaushal’s monthly salary?
(c) 48 (d) 60 (a) `20000 (b) `18000
[Based on SSC (GL), 2011] (c) `26000 (d) `24000
104. a, b, c, d and e be non-negative real numbers such that a + [Based on CBI (PO), 2010]
b + c + d + e = 10. Let X be the maximum of the numbers
110.
In a test, a candidate secured 336 marks out of maximum
a + b, b + c, c + d and d + e. The least possible value of X
marks ‘x’. If the maximum marks ‘x’ were converted into
lies in the “interval”
400 marks, he would have secured 192 marks. What were
(a) [0, 2] (b) [2, 3] the maximum marks of the test?
(c) [3, 4] (d) [4, 5] (a) 700 (b) 750
[Based on XAT, 2007]
(c) 500 (d) 650
105.
There are three baskets of fruits. First basket has twice the  [Based on Corporation Bank PO, 2009]
number of fruits in the 2nd basket. Third basket has three-
fourths of the fruits in the first. The average of the fruits 111.
The average marks in Science subject of a class of 20
in all the baskets is 30. What is the number of fruits in the students is 68. If the marks of two students were misread
first basket? as 48 and 65 of the actual marks 72 and 61, respectively,
(a) 20 (b) 30 then what would be the correct average?
(c) 35 (d) 40 (a) 68.5 (b) 69
[Based on SSC (GL), 2011] (c) 69.5 (d) 70
106.
Professor Bee noticed something peculiar while entering  [Based on Corporation Bank PO, 2009]
the quiz marks of his five students into a spreadsheet. The
112.
The average age of the family of five members is 24. If the
spreadsheet was programmed to calculate the average
present age of youngest member is 8 years, then what was
after each score was entered. Professor Bee entered the
the average age of the family at the time of the birth of the
marks in a random order and noticed that after each
youngest member?
mark was entered, the average was always an integer. In
ascending order, the marks of the students were 71, 76, (a) 20 years (b) 16 years
80, 82 and 91. What were the fourth and fifth marks that (c) 12 years (d) 18 years
Professor Bee entered?  [Based on Corporation Bank PO, 2009]
(a) 71 and 82 (b) 71 and 76 113.
In a family, the average age of a father and a mother is 35
(c) 71 and 80 (d) 76 and 80 years. The average age of the father, mother and their only
[Based on XAT, 2011] son is 27 years. What is the age of the son?

Chapter_06.indd 136 1/30/2016 1:26:34 PM


137
Average 

(a) 12 years (b) 11 years 118. Sum of eight consecutive numbers of Set A is 376. What
(c) 10.5 years (d) 10 years is the sum of five consecutive numbers of another set if its
[Based on SSC (GL), 2010] minimum number is 15 ahead of average of Set A?
114. The average marks in English subject of a class of 24 (a) 296 (b) 320
students is 56. If the marks of three students were misread (c) 324 (d) 284
as 44, 45 and 61 of the actual marks 48, 59 and 67 [Based on Union Bank of India PO, 2011]
respectively, then what would be the correct average?
119. In a class, the average height of 35 girls was measured
(a) 56.5 (b) 59 160 cm. Later on, it was discovered that the height of one
(c) 57.5 (d) None of these of the girl was misread as 144 cm, while her actual height
 [Based on IBPS Bank PO, 2011] was 104 cm. What was the actual average height of girls
in the class? (rounded off to two digits after decimal)
115.
The sum of five numbers is 290. The average of the first
two numbers is 48.5 and the average of last two numbers (a) 159.86 cm (b) 158.54 cm
is 53.5. What is the third number? (c) 159.56 cm (d) None of these
(a) 72 (b) 84 [Based on Syndicate Bank PO, 2010]
(c) 96 (d) None of these 120. ‘Mr. Haque’s total annual gross salary, which was `10
[Based on Indian Overseas Bank PO, 2009] lakhs per year in 2007, has been reduced by 10% in 2008.
116.
The average of the first 100 positive integers is In 2007 his family expenditure for food items was 40%
(a) 100 (b) 51 of the total annual gross salary. The prices of average
food items have increased by 5% between 2007 and 2008.
(c) 50.5 (d) 49.5
Assuming that the family consumed the same amount
[Based on SSC (GL), 2010]
of food in 2008, the percentage expenditure on food
117.
The average contribution of 5 men to a fund is `35. Sixth items, calculated on total annual gross salary in 2008, is
man joins and pays `35 more than the resultant average of approximately
six men. The total contribution of all the six men is (a) 43% (b) 45%
(a) `210 (b) `245
(c) 47% (d) 49%
(c) `250 (d) `252
[Based on JMET, 2009]
[Based on U.P. P.C.S., 2012]

EXPLANATORY ANSWERS

1.
(a) 6 × 10.5 + 6 × 11.4 – 11 × 10.9 Therefore, 8s + 10 (s + 1) = 64
= 63 + 68.4 – 119.9 ⇒ 18s = 54  or  s = 3 km/h
= 131.4 – 119.9 = 11.5 4. (b) Total marks of those 3 students must be
2. (b) For first nine months his salary is `380 per month. For (44 × 100 – 97 × 43.92) = 139.96.
next 12 months, it was `420. For next 12 months it 139.76
was `460 and for the last three months it was `500. ∴ Average of those 3 students = = 46.6
3
∴ His average salary of last 36 months
5.
(c) Since, we want to find the minimum attendance in
[9(380) + 12(420 + 460) + 3(500)]
= = `430 third semester, we will take the attendance in fourth
36 semester 100%.
∴ His pension is `215 per month. Let the required % of attendance be x
3. (b) If t is the number of hrs the hiker walked on the first 60  70  x  100
= 80
day, then t + 2 is the number of hrs he walked on the 4
second day. ⇒ x = 320 – 230 = 90
∴ t + t + 2 = 18, or t = 8. If s was the hiker’s average 6.
(c) Let average cost of petrol per litre be `x.
speed in kmph on the first day, then s + 1 was his
12000
average speed on the second day. So, the total \ x = = 7.98.
4000 4000 4000
distance hiked in 2 days was (8) × (s) + (10) + +
× (s + 1). 7.50 8 8.50

Chapter_06.indd 137 1/30/2016 1:26:34 PM


138  Chapter 6

7. (d) (Agarkar + Sehwag + Sachin + Dravid + Ganguly) ∴ The number of odd numbers in all the sets from
make = 39 × 5 = 195 runs. A1 to A20 is = 400(starting from 3). Therefore, the
With respect to scoring runs 400th odd number (starting from 3) is 2(400) + 1 =
Dravid = Ganguly + 7 801. This is the last number in A20
Ganguly = Agarkar – 9 Similarly, the last odd number in A19 is the 361st odd
Sehwag = Dravid + Ganguly number starting from 3 = (361) (2) + 1 = 723
Sehwag + Sachin = 110 ∴ The first odd number in A20 is 725
⇒ Agarkar, Dravid, Ganguly, Sehwag and Sachin ∴ The average of the numbers of

scored 32, 30, 23, 53 and 57 runs respectively. 725  801
A20 = = 763
8.
(b) Average of first 7 numbers will be the 4th number = X 2
(Given) 14.
(a) Let each side of the square field be x km
Average of all the 11 numbers will be the 6th number,
\ Average speed of the plane
i.e., X + 4.
4x
197 × 63 + 591 × 51 197 × 63 + 591 × 51 =
9.
(c) = x x x x
197 + 591 788 + + +
800 600 400 200
63 3 63 + 153 216
= + × 51 = = 4 × 2000 9600
4 4 4 4 = =
3 + 4 + 6 + 12 25
= 54 kg.
10.
(a) Suppose strength of the class = x = 384 km/hr.
\ 5x + 10 + 15 = 5.5 (x + 2) 15.
(a) Suppose the number of candidates passed =  x
⇒ 0.5x = 14 ⇒ x = 28. \ 39x + 15 (120 – x) = 120 × 15
11.
(c) Suppose temperature on 1st day = 7K ⇒ 24x = 120 × 35 – 120 × 35
Suppose temperature on 5th day = 8K = 120 (35 – 15) = 120 × 20
\ Temperature on ⇒ x = 100.
M + T + W + Th = 232
16.
(a) (A – 3) + (B – 3) = 36 ⇒ A + B = 42
Average temperature on
Also A + B + C = 66 ⇒ C = 24.
T + W + Th + F = 240
\ 232 – 7K = 240 – 8K 17.
(d) Let the three numbers be X, Y and Z.
⇒ K = 8 X +Y +Z
\ = 135 ⇒ X + Y + Z = 405
\ Temperature on 5th day = 64º. 3
12.
(b) a + b + c = 3M Let X be the largest number
(a + b + c)2 = a2 + b2 + c2 + 2 (ab + bc + ca) \ X = 180 ⇒ Y + Z = 225
= a2 + b2 + c2 Y – Z = 25
⇒ a2 + b2 + c2 = (3M)2 = 9M2 \ Y = 125,
2
9M Z = 100 (smallest number).
⇒ Mean of a2, b2 and c2 = = 3M2.
3 18.
(a) Let X and Y be the two numbers
13. (b) The given sets are formed by taking 1, 3, 5, ...... X +Y
odd numbers starting from the second odd number
\ 2 = 3
onwards (i.e., 3 onwards), in the successive sets. So,
X 5
there are (2n – 1) successive odd numbers in An
∴ Total number of odd numbers in all the sets from X +Y 3
A1 to An (both inclusive) ⇒ =
2X 5
n n n
⇒ 5X + 5Y = 6X ⇒ X = 5Y
=  (2i – 1) = 2 i – 1
1 1 1 X 1
⇒ = .
= n(n + 1) –n = n2 Y 5

Chapter_06.indd 138 1/30/2016 1:26:35 PM


139
Average 

19.
(c) Let the distance between X and Y be x km. Total age of 9 players = 11x – (28 + 25)
x 11x  3
\ Time taken from X to Y @ 40 km/hr = hrs Average of 9 players = =x–1
40 9
⇒ x = 22 years
x
Time taken from Y to X @ 60 km/hr = hrs 29. (c) Suppose his average after 12 inning = x
60
12 + 96
2x Then = x + 5
\ Average speed = = 48 km/hr. 13
x x x = 31
+
40 60 \ Required average = x + 5 = 31 + 5 = 36.
20. (a) On Monday, the payment = `x 3n n
30.
(c) For numbers average is 36 + 4 and for numbers
On Tuesday, the payment = `(x + 3y) 4 4
On Wednesday, the payment = `(x + 2y) average is 36 – 4
On Thursday, the payment = `(x + y) 3n n
 40   32
On Friday, the payment = `(x + 2y) New average = 4 4 = 30 + 8 = 38
\ Average daily earnings during the five-day week n
5x + 8 y 8y 31.
(b) Let the 7 consecutive numbers be x – 3, x – 2, x – 1, x,
= = x+ . x + 1, x + 2 and x + 3
5 5
21.
(a) Let the consecutive numbers be x, x + 1, x + 2, x + 3, ( x − 3) + ( x − 2) + ( x − 1) + x + ( x + 1) + ( x + 2) + ( x + 3)
\
x+4 7
5 x + 10 = 20
Average = =x+2
5 7x
⇒ = 20 ⇒ x = 20
5 x + 10 + x + 5 + x + 6 7
Average of 7 numbers = \ largest number = x + 3 = 20 + 3 = 23.
7
7 x + 21 32.
(d) Let the teacher’s age be x years
= =x+3 15 × 15 − x
7 \ = 14
\ The average increased by 1. 14
⇒ 225 – x = 196
20 × 10 + 240 200 + 240 440
22.
(a) = = ⇒ x = 29.
25 25 25
33.
(a) Let the age of the mother be x years
= 17.6 miles/hr.
14 × 12 + x
23. (b) \ = 17 ⇒ x = 37.
5
24. (d) The distance of 800 miles was covered in 5 + 4 = 9
hrs. 34.
(d) Total age of A and B = 40
800 8 Total age of B and C = 38
\ Average speed of W = = 88 miles per hour.
9 9 Total age of A and C = 42
Total age of A + B + C = 60
25. (b) Let the average score of the 1st 18 innings be x
\ Age of B = 60 – 40 = 18.
\ 18x + 98 = 19(x + 4)
⇒ x = 22 35.
(c) Total cost of all the balls = `280
Average score after 19th innings = x + 4 = 26. Let no. of red balls = x
26. (d) 11 × 90 – 5 × 87 – 5 × 84 \ No. of white balls = 10 – x
= 990 – 435 – 420 = 135. \ 25x + 30 (10 – x) = 280
27. (c) Out of 330 marks, the student is required to obtain 165 ⇒ x = 4
marks, i.e., 50% \ No. of white balls = 10 – 4 = 6.
\ In first paper, because of scoring 54 marks only, 111
he has to score 111 marks out of 150 in the 2nd paper, 36.
(c) Total age of 14 persons =  × 14 years
4
i.e., 74% Total age of 12 persons
28.
(b) Let the average age of the whole team be x  1554  1218
 
 84 =  = 304.5
Total age = 11x  4  4

Chapter_06.indd 139 1/30/2016 3:38:56 PM


140  Chapter 6

Average age of 12 persons 43.


(d) 2x + x + 4x = 56 × 3
304.5 \ 2x = 48.
  = 25.375 years
12 44.
(b) Let the first number be x
\ Second number = 2x
37.
(c) A + B + C = 12,000 × 3
2
B + C + D = 15,000 × 3 and third number = x
3
⇒ D – A = 3000 × 3
2
D – A = 9000 x + 2x + x
3 11x
\ = 44 or, = 44
also D = 2A 3 9
⇒ D = 18,000 and A = 9,000 \ x = 36
Therefore average salary of B and C \ Second no. is the largest = 2x = 2 × 36 = 72.
(45,000  18,000) 45.
(d) When 2 new men are replaced, average age is
= = 13,500 increased by 2 years.
2
\ Increase in the total age of 8 men
38.
(b) Let the average expenditure per student be `x = 8 × 2 = 16 years
Therefore, original total expenses = `35x \ Total age of 2 new men
New average expenditure per student = `(x – 1) = 16 + 44 = 60 years
New average expenditure per student = `(x – 1) \ Average age of 2 new men
New total expenses = `(35x + 42) 60
= = 30 years.
35 x  42 2
∴ = (x – 1) 46.
(d) A + B = 60, B + C = 64, A + C = 68
42
\ C – A = 4, C + A = 68
⇒ 35x + 42 = 42x – 42
⇒ C = 36, A = 32, B = 28.
or x = 12, therefore original expenditure of the mess
47.
(a) Income in the eighth month
= 35 × 12 = `420
= (8 × 3160 + 5 × 4120) – (12 × 3400)
39.
(c) Marks obtained by 14 students
= (25280 + 20600 – 40800)
= 14 × 71 = 994 = `5080.
Exact marks of 14 students 48.
(c) Age of mother + father = 22 × 8 – 8 × 6
= 994 + {(56 – 42) + (32 – 74)} = 176 – 48
= 994 + {14 + (–42)} = 994 + {–28} = 128
= 994 – 28 = 966 father – mother = 8
966 128 − 8

\ Correct average = = 69. \ Age of mother = = 60 years.
14 2
40.
(c) Let the numbers be x, y, z and t 49.
(a) Sale amount of grocer in the sixth month
1 = 6 × 6500 – (6435 + 6927 + 6855 + 7230 + 6562)
\ x + y + z + t = 240 and x = (y + z + t)
4 = 39000 – 34009
\ x = 48 = `4991.
\ First no. = 48. 50.
(c) Sum of weights of A, B and C = 84 × 3 kg
41.
(b) F + S 1 + S2 = 81 = 252 kg
(S1 – 5) + (S2 – 5) = 24 Weight of D = 4 × 80 – 252
⇒ S1 + S2 = 34 = 320 – 252
= 68 kg
Also S1 – S2 = 4
Weight of E = 68 + 3
\ S1 = 19, S2 = 15
= 71 kg
\ Age of father = 47.
A + B + C + D = 320
42.
(b) 31 × 15 – (30 × 15 – 20) = x + x + 5
B + C + D + E = 79 × 4
\ x = 15
= 316

Chapter_06.indd 140 1/30/2016 1:26:36 PM


141
Average 

⇒ A – E = 4 kg Obviously, this cannot be determined as we do not


\ A = 4 + E = 4 + 71 = 75 kg. know the weight of H.
abc 60.
(d) Let number of boys = x
– cba
51.
(c) Remember and number of girls = y
99(a  c)
Now, y – 2 = x and x + y = 52
where abc and cba are the three digit numbers and
y – 2 + y = 52
(a, c) ≠ 0
Again since the difference in average = 19.8 2y = 54
Therefore, the difference in total = 19.8 × 10 = 198 \ y = 27
Thus, 99 × (a – c) = 198 \ x = 25
⇒ (a – c) = 2 Let the average weight of the girls be ‘a’.
(25 × 42) + (27 × a )
E+S+M +H E + H +G+M Then, = 40
52.
(c) – = 15 52
4 4
or, 27 × a = (52 × 40) – (25 × 42)
or, E + S + M + H – E – H – G – M = 60
or, 27 × a = 2080 – 1050
\ S – G = 60.
1030
35 × 72 + (86 − 36) \ a = = 38.148 ≈ 38 kg.
53.
(d) Correct average = 27
35
61.
(b) The total score by a batsman = 100 × 36
≈ 72 + 1.43
= 3600
= 73.43.
\ The correct total score = 3600 – 40 + 90 = 3650
54. (d) Present total age of husband and wife
3650 − 3600 5000
= (23 × 2 + 5 × 2) = 56 years \ Percentage error = × 100 =
3650 3650
Present total age of husband, wife and child
= 20 × 3 = 60 years = 1.36%
∴ age of child = (60 – 56) = 4 years 21 − 15
62.
(a) \ Required percentage = × 100
15
55.
(b) Let the four consecutive even numbers be 2x, 2x + 2,
2x + 4 and 2x + 6, respectively. 600
= = 40%
Required difference = 2x + 6 – 2x = 6. 15
56.
(d) Let the three numbers be x, y and z. 63.
(d) Let number of students be n and average weight w.
x+ y y+z nw + 50
or, − = 15 According to the given condition, =w+1
2 2 n +1
x+ y− y−z ⇒ n + w = 49 …(1)
or, = 15  or,  x – z = 30.
2 nw + 50 + 50
and = w + 1.5
57.
(b) Total of 25 results = 25 × 18 = 450 n+2
Total of first 12 results = 12 × 14 = 168 ⇒ 1.5n + 2w = 97 …(2)
Total of last 12 results = 12 × 17 = 204 On solving Eqs. (1) and (2), we get
\ Thirteenth result = 450 – 168 – 204 w = 47
= 78. 64.
(a) Total marks of a student in 8 subjects
58.
(d) Total age of seven persons = 30 × 7 = 210 years. = 8 × 87 = 696
Total marks of a student in 6 subjects = 6 × 85 = 510
Total age of five persons = 31 × 5 = 155 years.
\ Remaining marks of 2 subjects =186
\ Total age of two persons = (210 – 155) years
Let the second highest marks be x, then highest marks
= 55 years. is x + 2.
55 \ x + x + 2 = 186
\ Average age of two persons = = 27.5 years.
2 ⇒ 2x = 184
59.
(d) We are to determine the average weight of P, T, R, F ⇒ x = 92
and H. \ Highest marks is x + 2 = 94

Chapter_06.indd 141 1/30/2016 1:26:37 PM


142  Chapter 6

65.
(a) Let first number be x, then ⇒ x22 – 11x2 + 28 = 0
x + ( x + 1) + ( x + 2) + ( x + 3) + ( x + 4) ⇒ (x2 – 4) (x2 – 7) = 0
= 41
5 ⇒ x2 = 4, 7 ⇒ x1 = 7, 4
     
⇒ 5x + 10 = 205 Hence, other observations are 4, 7
⇒ 5x = 195 71.
(b) Total of eleven results = 11 × 50 = 550
⇒ x = 39 Total of first six results = 49 × 6
\ Product of A and E = x × (x + 4) = 294
= 39 × 43 = 1677 Total of last six results = 52 × 6
66.
(a) Let woman and her daughter ages be 2x and x. = 312
2x + x \ Required six results = 294 + 312 – 550
Also, = 42 = 56
2
\ = 3x – 84 Mon + Tue + Wed + Thu
72.
(c) Given = 48°
⇒ x = 28 4
Hence, daughter’s age be 28 years. \ 42° + Tue + Wed +Thu = 192°
67.
(d) Total weight of 29 students = 29 × 48  = 1392 kg ⇒ Tue + Wed + Thu = 150° (1)
If teachers weight is included, then total weight Tue + Wed + Thu + Fri
and = 52º
= 30 × 48.5 = 1455 kg 4
\ Weight of teacher ⇒ 150° + Fri = 208°
= 1455 – 1392  [from Eq. (1)]
= 63 kg ⇒ Fri = 58°
68.
(a) Total marks obtained by the candidates 73.
(b) Let total capital be `x.
= 120 × 35 = 4200 \ Required average
Let passed students be x, then x 2x x
× 3% + × 5% + × 11%
4200 = x × 39 + (120 – x)15
= 4 3 12
⇒ 4200 = 24x + 1800 x
⇒ 2400 = 24x
 3 10 11 
⇒ x = 100 =  + + %
 4 3 12 
69.
(a)
 9 + 40 + 11 
No. of 2 No. of 3 No. of 4 =  %
 12 
wheelers wheelers wheelers
2x x 2x  60 
=   % = 5%
No. of wheels 2 × 2x 3×x 2x × 4  12 
74.
(c) Let the average of 12 innings be x.
= 4x   = 3x   = 8x
12 x + 96

Therefore average number of wheels Also, =x+5
13
4 x  3x  8 x
=  = 3 ⇒ 12x + 96 = 13x + 65
5x
⇒ x = 31
70.
(c) Let other two observations be x1 and x2. 12 × 31 + 96
\ Required average =
1 + 2 + 6 + x1 + x2 13
\ 4 =
5 468
= = 36
⇒ x1 + x2 = 11 13
(4 − 1) 2 + (4 − 2) 2 + (4 − 6) 2  75.
(b) Let the original average expenditure be `x.
 
 + (4 − x1 ) 2 + (4 − x2 ) 2  Then,
and 5.2 =
5 40(x – 2) – 30x = 40
⇒ 26 = 9 + 4 + 4 + (x2 – 7)2 + (4 – x2)2 ⇒ 10 x = 120 ⇒ x = 12
⇒ 9 = 2x22 – 22x2 + 49 + 16 \  Original expenditure = 30 × 12 = `360

Chapter_06.indd 142 1/30/2016 1:26:37 PM


143
Average 

78.4 × 25 + 96 − 69 Total weight of 5 students who join


76.
(a) Required correct mean =
25 = 5 × 54 = 270
1960 + 27 1987 Hence new total weight of 45 students
   = = = 79.48
25 25 = 2340 – 240 + 270 = 2370
77. (b) Total present age of the family of 6 members 2370 2
\ Average = = 52 kg
= 6 × 22 = 132 years 45 3
Total age of the family of 6 members 7 years ago
[r  ( p  q )]  p  q pqr
= (132 – 7 × 6) = 90 years 83.
(c) = 21 
3 3
∴ Average age of the family at the birth of the
90 2p  r pqr
youngest member =  = 18 years ⇒  21 =
5 3 3
78.
(b) Let the total number of boys and girls be B and G pq
respectively. ⇒ = 21
3
\ Total score of boys = 71 B
⇒ p – q = 63
Total score of girls = 73 G
Total score of the class = 71.8(B + G) 84.
(a) Let the numbers be F, S and T.
\ 71B + 73G = 71.8(B + G) According to the question,
⇒ 0.8B = 1.2G F + S +T
F = 2S, S = 2T and = 21
B 1.2 3 3
⇒   = =
G 0.8 2 or, F + S + T = 63 ... (1)
79.
(c) Total salary of 75 workers = `426000 Now, putting the value of F and T in (1), we have
Total salary of 25 workers = `135000 S
28 + S + = 63
Total salary of 30 workers = `171000 2
\ Total salary of remaining 20 workers 63 × 2
or S = = 18
= 426000 – (135000 + 171000) 7
= 120000 \ F = 2S
120000 = 2 × 18 = 36
\ Mean salary of 20 workers = = `6000
20
S 18
and T = = = 9
80. (c) Total expenditure for the year 2 2
= [2200 × 3 + 2250 × 4 + 3120 × 5] \ largest number is 36.
= 6600 + 9000 + 15600 = `31200
85.
(d) Let the three numbers be x, y and z.
  Total saving = `1260
x+ y+z
Total income = expenses + savings \ = 135
3
= 31200 +1260 = `32460
i.e., x + y + z = 405 ...(1)
Average income
32460 Let z be the largest number
= = `2705
12 \ z = 180
81.
(a) 480 × 7 = 3360 ⇒ x + y = 225 ...(2)
480 × 8 = 3840 Also x – y = 25 ...(3)
\ Maximum number of pants that can be purchased Solving (2) and (3), we get x = 125, y = 100
is 7.
86.
(c) Let x be the highest score and y be the lowest score.
Balance Amount = 3620 – 3360 = 260
\ x + y = 40 × 50 – 38 × 48
In `260, two shirts can be purchased.
= 2000 – 1824 = 176
82.
(b) Total weight of 45 students = 45 × 52 = 2340
x – y = 172
Total weight of 5 students who leave
\ x = 174, y = 2
= 5 × 48 = 240

Chapter_06.indd 143 1/30/2016 3:41:17 PM


144  Chapter 6

87.
(b) Total marks in 10 papers = 800 or, A + B + C = 252 …(1)
Total marks in 8 papers = 648 A+ B+C + D
= 80
Total of highest and lowest marks = 152 4
lowest marks = 152 – 92 or, A + B + C + D = 320 ...(2)
= 60
B + C + D + ( D + 3)
88.
(a) Average Total and = 79
4
S + G → 35 70
or B + C + 2D + 3 = 316
K + G → 32 64
or B + C + 2D = 313 ...(3)
S + K → 38 76
Subtracting (1) from (2),
S + K + G → 35 105
D = 320 – 252 = 68 kg
35
D + I → 35 Subtracting (3) for (2),
2
A – D = 7
S  K  G  D  I 105  35 A = D + 7 = 68 + 7 = 75 kg.
∴     28
S 5
93.
(a) Max. marks = 180 + 150 = 330
89.
(d) Suppose ninth person spent `x. If the required percentage of marks is x% in the
Total money spent by nine persons = 96 + x second paper, then according to the question, 30% of
96 + x 180 + x% of 150 = 50% of 330
\ x = +8
9 30 x 50
or × 180 + × 150 = × 330 = 165
⇒ 9x = 96 + x + 72 100 100 100
⇒ x = 21 3
or 54 + x = 165
\ Total money spent = 96 + x = 117. 2
90.
(b) Suppose the third number is x. 3
or x = 111
x 2
\ First number = x = 74 %
2
94.
(c) Let A1, A2, …, A9 be the numbers.
1 x x
and second number =  = Let A1 + A2 + … + A9 = 9 M
2 2 4
A1 + A2 + A3 = 3P
According to the question,
A4 + A5 + … + A9 = 6 N
x x
+ +x Then, 9M = 3P + 6N
2 4 x x
= 56 or + + x = 168 3M = P + 2N
3 2 2
or 7x = 672
\ x = 96 95.
(d) The 5 consevtice odd numbers whose average is k are
(k – 4) (k – 2), k, (k + 2) (k + 4)
96 96
Hence, the numbers are , , 96 Again the average of (k – 4), (k – 2), (k), (k + 2),
2 4 (k + 4), (k + 6), (k + 8) is (k + 2)
91.
(d) Suppose
Alternatively: Consider some appropriate numbers.
a = 2n – 5, b = 2n – 3, c = 2n – 1, d = 2n + 1,
e = 2n + 3 96.
(a) Total age of A and B = 20 × 2 = 40 years
\ Their average Total age of B and C = 19 × 2 = 38 years
(2n − 5) + (2n − 3) + (2n − 1) Total age of A and C = 21 × 2 = 42 years
+ (2n + 1) + (2n + 3) 40  38  42
= Total age of (A + B + C) =
5 2
= 60 years.
10n − 5
= = 2n – 1 = c. Therefore, ages of A, B and C = 22 years, 18 years and
5 20 years, respectively.
92.
(c) According to the question,
97.
(d) Let a, b, c, d, e and f be six numbers, such that
A+ B+C a = e + m
= 84
3 b = e – k

Chapter_06.indd 144 1/30/2016 1:26:38 PM


145
Average 

c = e – l \ Arithmetic mean of n numbers



d = e – j 1
n 2
f = e + p = n  n  1 1  1
Then, the sum of deviations of a, b, c, d, e and f from n n2 n2
e is (m – k + l – j + p) 102.
(a) Let the number of professors, associates and assistant
The mean of a, b, c, d, e and f is given by professors be a, b and c, respectively.
abcd e f Let their average ages = p, q and r, respectively.
So, ap + bq + cr = 2160 ...(1)
6
ap  bq
emek el e j ee p = 39 ...(2)
= ab
6
bq  cr 328
6e  (m  k  l  j  p ) = …(3)
= b  c 11
6
ap  cr 362
6e  Sum of deviations = …(4)
= ac 3
6
   a(p + l) + b(q + 6) + c(r + 7) = 2460 ...(5)
Using the similar logic, the required mean Solving these,
20  23  70 a = 16, b =24, c =20, p = 45, q = 35, r = 30
= = 26.5
20 103.
(c) let the number be x, then
Hence, option (d).
240 − x
x =
98.
(d) Total runs upto 20 innings = 20 × 44.5 = 890 runs 4
Let he scores x runs in the 21st innings then ⇒ 4x = 240 – x
890  x ⇒ 4x + x = 240
Average after 21 innings =
21 ⇒ 5x = 240
890  x 240
⇒ 45 = ⇒ 945 = 890 + x ⇒ x = = 48
21 5
or x = 55 runs
104.
(a) a + b + c + d + e = 10
99.
(c) Total 6 numbers → 6 × 30 = 180 10
Average = =2
1st 4 numbers → 4 × 25 = 100 5
last 3 numbers → 3 × 35 = 105 a = 0 to 4
4th number = 205 – 180 = 25 Least value of x:0 ≤ x ≤ 2
100. (c) Let the smallest number be x, then b = 0 to 4 [Q average = 2]
x c = 0 to 4
+ 12 = x + 2
3 d = 0 to 4
e = 0 to 4
x + 36
⇒ = x + 2
3 105. (d)
⇒ x + 36 = 3(x + 2)
⇒ x + 36 = 3x + 6
⇒ 3x – x = 36 – 6
⇒ 2x = 30
⇒ x = 15 Let the no. of fruits in the second basket be x.

Hence, third number = 15 + 4 = 19
Therefore, the no. of fruits in the first basket = 2x

So, no. of fruits in the third basket
 1 3 3
101.
(d) Sum of numbers = 1   + 1 + 1 + 1 ... (n – 1) times = 2x × = x
 n 4 2
1 1 3
= 1   (n  1)  n  2x + x + x = 30 × 3
n n 2

Chapter_06.indd 145 1/30/2016 1:26:39 PM


146  Chapter 6

4 x + 2 x + 3x 110. (a) x:336 = 400:192


⇒ = 90
2 x × 190 = 336 × 400
9x 336 × 400
= 90 x =
2 192
9x = 180 x = 700 marks
180 111. (b) Difference of marks = 72 + 61 – 48 – 65 = 20
x = = 20 20
9 Correct average marks = 68 + = 68 + 1 = 69
Hence, no. of fruits in the first 20
basket = 2x = 2 × 20 = 40 112. (b) Total age of the family of five members
= 24 × 5 = 120
106.
(c) The marks of the 5 students and the remainder that
they leave when divided by 3 are tabulated below. Total age of the family of five members before 8 years
= 120 – 5 × 8
Marks 71 76 80 82 91 = 120 – 40 = 80
Remainder 2 1 2 1 1 80
So, required average age = = 16 years
After the first 3 marks are entered, the total has to be 5
a multiple of 3. The remainders can only be 1, 1, 1.
(2, 1, 1 or 2, 2, 1 would not produce a multiple of 3) 113.
(b) Age of the son = 3 × 27 – 2 × 35
= 81 – 70 = 11 years
\ The fourth and fifth marks to be entered were 71
and 80. 114. (d) Q Total marks of 24 students = 24 × 56
(45 × 36) + 32 − 34 + 45 − 40     = 1344
107. (a) Average =
45 New total marks of 24 students
1620 + 3 = 1344 – 44 – 45 – 61 + 48 + 59 + 67 = 1368
= = 36.07 kg
45 1368
Hence, required average = = 57
24
108. (b) Cost of 8 kg of apples + 8 dozens of mangoes + 8 kg
of oranges 115. (d) Average of first two numbers = 48.5
450 4320 240 Average of last two number = 53.5
×8 + ×8 + ×8
5 12 4 Sum of five numbers = 290
720 + 2880 + 480 = `4080 Third number = x
109. (d) Annual salary of Suresh = 108000 \ 2 × (48.5) + x + 2 × (53.5) = 290
108000 97 + x + 107 = 290
\ Monthly salary = x = 290 – 204
12
= 9000 x = 86

Nandini’s monthly salary = 18000 1 + 2 + ...100


116.
(c) Required average =
According to question, 100
12 100 × 101
Kaushal’s monthly salary × = = 50.5
100 2 × 100
16
= Nandini’s monthly salary × 117.
(d) Let the contribution of sixth man is `x then,
100
5 × 35 + x
12 16 = x – 35
K× = 18000 × 6
100 100
⇒ 175 + x = 6x – 210
12
K× = 2880 ⇒ 5x = 385
100
2880 × 100 \ x = 77
K =
12 \ Total contribution = `(175 + 77)
K = 24000 = `252

Chapter_06.indd 146 1/30/2016 1:26:40 PM


147
Average 

376 120. (c) Haque’s salary in 2008 = 900000 rupees


118. (b) Average of Set A = = 47
8 Expenditure on food in 2007 = 400000 rupees
Minimum number of second set = 47 + 15 = 62 Expenditure on food in 2008
Hence, required sum = 62 + 63 + 64 + 65 + 66 = 320 5
= 400000 + 400000 ×
100
(35 × 160) − 144 + 104
119. (d) = 420000 rupees
35
\ Percentage expenditure on food items in 2008
5600 − 144 + 104
= 420000
35 = × 100
900000
5560
= = 158.85 cm = 46.67 ≈ 47%
35
144 − 104
Short cut:160 – = 158.85
35

Chapter_06.indd 147 1/30/2016 3:42:20 PM


7 Ratio and Proportion

Ratio 2. Triplicate Ratio  The ratio of the cubes of two numbers


is called the triplicate ratio of the two numbers.
A ratio is a comparison of two quantities by division. It is
a relation that one quantity bears to another with respect 33 27 3
For example, or is triplicate ratio of .
to magnitude. In other words, ratio means what part one 43 64 4
quantity is of another. The quantities may be of same kind
3. Sub-duplicate Ratio  The ratio of the square roots of
or different kinds. For example, when we consider the ratio
two numbers is called the sub-duplicate ratio of two
of the weight 45 kg of a bag of rice to the weight 29 kg of a
numbers.
bag of sugar we are considering the quantities of same kind 3 9
but when we talk of allotting 2 cricket bats to 5 sportsmen, For example, is the sub-duplicate ratio of .
4 16
we are considering quantities of different kinds. Normally,
we consider the ratio between quantities of the same kind. 4. Sub-triplicate Ratio  The ratio of the cube roots of
two numbers is called the sub-triplicate ratio of two
a numbers.
If a and b are two numbers, the ratio of a to b is or
b 2 8
a ÷ b and is denoted by a:b. The two quantities that are being For example, is the sub-triplicate ratio of .
3 27
compared are called terms. The first is called antecedent
and the second term is called consequent. 5. Inverse Ratio or Reciprocal Ratio  If the antecedent
and consequent of a ratio interchange their places, the
3 new ratio is called the inverse ratio of the first. Thus,
For example, the ratio 3:5 represents with antecedent
5 1 1
3 and consequent 5. if a:b be the given ratio, then : or b:a is its inverse
a b
ratio.
Notes:
3 5
1. A ratio is a number, so to find the ratio of two quantities, For example, is the inverse ratio of .
5 3
they must be expressed in the same units.
2. A ratio does not change if both of its terms are multiplied 6. Compound Ratio  The ratio of the product of the
or divided by the same number. Thus, antecedents to that of the consequents of two or more
given ratios is called the compound ratio. Thus, if
2 4 6
= = etc. a:b and c:d are two given ratios, then ac:bd is the
3 6 9 compound ratio of the given ratios.
Types of Ratios 3 4 5
For example, if , and be the given ratios, then
1. Duplicate Ratio  The ratio of the squares of two 4 5 7
numbers is called the duplicate ratio of the two 3× 4 × 5 3
numbers. their compound ratio is , that is, .
4× 5× 7 7
32 9
For example, 2 or is called the duplicate ratio
4 16 Proportion
3
of . The equality of two ratios is called proportion.
4

Chapter_07.indd 148 1/30/2016 2:54:28 PM


Ratio and Proportion 149

a c a c
If = , then a, b, c and d are said to be in proportion Each term of the ratio and is called a proportional. a,
b d b d
and we write a:b:: c:d. This is read as “a is to b as c is to d”. b, c and d are, respectively, the first, second, third and fourth
3 6 roportionals.
For example, since = , we write 3:4:: 6:8 and say
4 8 Here a, d are known as extremes and b, c are known
3, 4, 6 and 8 are in proportion. as means.

soMe BAsiC FoRMulAe

Illustration 2 Find a third proportional to the numbers


1. If four quantities are in proportion, then 2.5, 1.5.
Product of means = Product of extremes Solution: Let x be the third proportional, then
For example, in the proportion a:b:: c:d, we have 2 ⋅ 5 1⋅ 5
bc = ad. 2×5:1×5:: 1×5:x or = .
1⋅ 5 x
1⋅ 5 ×1⋅ 5
\ x = = 0.9.
2⋅5

4. Mean Proportional If a:x:: x:b, x is called the


mean or second proportional of a, b.

From this relation, we see that if any three of a x


We have = or x2 = ab or x = ab .
the four quantities are given, the fourth can be x b
determined. \ Mean proportional of a and b is ab .
2. Fourth proportional If a:b:: c:x, x is called the We also say that a, x, b are in continued proportion.
fourth proportional of a, b, c.
a c b×c Illustration 3 Find the mean proportional between 48
We have = or, x = .
b x a and 12.
Solution: Let x be the mean proportional. Then,
b×c
Thus, fourth proportional of a, b, c is .
a 48 x
48:x:: x:12 or, =
x 12
Illustration 1 Find a fourth proportional to the numbers
or, x2 = 576 or, x = 24.
2, 5, 4.
Solution: Let x be the fourth proportional, then
2 4 a c
2:5:: 4:x or = . 5. If = , then
5 x b d
5× 4 a+b c+d
\ x = = 10. (i) = (Componendo)
2 b d

3. Third Proportional If a:b:: b:x, x is called the a −b c−d


(ii) = (Dividendo)
third proportional of a, b. b d
a b b2 a+b c+d
We have = or x = . (iii) = (Componendo and dividendo)
b x a a −b c−d

b2 a a+c a−c
Thus, third proportional of a, b is . (iv) = = .
a b b+d b−d

Chapter_07.indd 149 1/30/2016 2:54:29 PM


150 Chapter 7

Illustration 4 The sum of two numbers is c and their x p


p \ =
quotient is . Find the numbers. x+ y p+q
q
x p
Solution: Let the numbers be x, y. ⇒ = [Using (1)]
c p+q
Given x+y=c ...(1)
pc
x p ⇒ x= .
and, = ...(2) p+q
y q

soMe useFul shoRt-Cut MethoDs

Illustration 5 Two numbers are in the ratio of 4:5 and the


1. (a) If two numbers are in the ratio of a:b and the sum of these numbers is 27. Find the two numbers.
sum of these numbers is x, then these numbers
Solution: Here a = 4, b = 5 and x = 27.
ax bx
will be and , respectively. ax 4 × 27
a+b a+b \ The first number = = = 12
a+b 4+5
or
bx 5 × 27
If in a mixture of x litres, two liquids A and B and, the second number = = = 15.
a+b 4+5
are in the ratio of a:b, then the quantities of
ax Illustration 6 Three numbers are in the ratio of 3 : 4 : 8 and
liquids A and B in the mixture will be the sum of these numbers is 975. Find the three numbers.
bx a +b
litres and litres, respectively. Here a = 3, b = 4, c = 8 and x = 975.
a+b
ax 3 × 975
\ The first number = = = 195.
(b) If three numbers are in the ratio of a:b:c and a+b+c 3+ 4+8
the sum of these numbers is x, then these
bx 4 × 975
ax bx The second number = = = 260.
numbers will be , and a+b+c 3+ 4+8
cx a+b+c a+b+c
, respectively. cx 8 × 975
a+b+c and, the third number = = = 520.
a+b+c 3+ 4+8

2. If two numbers are in the ratio of a:b and


Explanation difference between these numbers is x, then these
Let the three numbers in the ratio a:b:c be A, B and C. numbers will be
ax bx
Then, (a) and , respectively (where a > b)
a −b a −b
A = ka, B = kb, C = kc
and, A + B + C = ka + kb + kc = x ax bx
(b) and , respectively (where a < b).
b−a b−a
x
⇒ k(a + b + c) = x ⇒ k = .
a+b+c Explanation
ax Let the two numbers be ak and bk.
\ A = ka = Let a > b.
a+b+c
Given ak – bk = x
bx
B = kb = x
a+b+c ⇒ (a – b)k = x or k = .
a −b
cx ax bx
C = kc = . Therefore, the two numbers are and .
a+b+c a −b a −b

Chapter_07.indd 150 1/30/2016 2:54:31 PM


Ratio and Proportion 151

Illustration 7 Two numbers are in the ratio of 4:5. If


the difference between these numbers is 24, then find the Therefore, the two numbers are ax(c − d ) and
ad − bc
numbers. bx(c − d )
Solution: Here a = 4, b = 5 and x = 24. .
ad − bc
ax 4 × 24
\ The first number = = = 96
b−a 5−4 (b) The ratio between two numbers is a:b. If x
is subtracted from each of these numbers, the
bx 5 × 24
and the second number = = = 120. ratio becomes c:d.
b−a 5−4
The two numbers are given as:
3. (a) If a:b = n1:d1 and b:c = n2:d2, then ax(d − c) bx(d − c)
and .
a:b:c = (n1 × n2):(d1 × n2):(d1 × d2). ad − bc ad − bc
(b) If a:b = n1:d1, b:c = n2:d2
Explanation
and c:d = n3:d3, then
Let the two numbers be ak and bk.
a:b:c:d = (n1 × n2 × n3):(d1 × n2 × n3)
ak − x c
: (d1 × d2 × n3):(d1 × d2 × d3). Given = ⇒ akd – xd = bck – xc
bk − x d
⇒ k(ad – bc) = x(d – c)
Illustration 8 If A:B = 3:4 and B:C = 8:9, find A:B:C.
x(d − c)
Solution: Here n1 = 3, n2 = 8, d1 = 4 and d2 = 9. ⇒k= .
ad − bc
\ a : b : c = (n1 × n2):(d1 × n2):(d1 × d2)
ax(d − c)
= (3 × 8):(4 × 8):(4 × 9) Therefore, the two numbers are and
ad − bc
= 24:32:36 or, 6:8:9.
bx(d − c)
Illustration 9 If A:B = 2:3, B:C = 4:5 and C:D = 6:7, find .
ad − bc
A:D.
Solution: Here n1 = 2, n2 = 4, n3 = 6, d1 = 3, d2 = 5 and Illustration 10 Given two numbers which are in the ratio
d3 = 7. of 3:4. If 8 is added to each of them, their ratio is changed
\ A:B:C:D = (n1 × n2 × n3):(d1 × n2 × n3) to 5:6. Find the two numbers.
: (d1 × d2:n3):(d1 × d2 × d3) Solution: We have
a:b = 3 : 4, c : d = 5:6 and x = 8.
= (2 × 4 × 6) : (3 × 4 × 6):(3 × 5 × 6)
ax(c − d )
: (3 × 5 × 7) \ The first number =
ad − bc
= 48:72:90 : 105 or, 16:24:30:35.
3 × 8 × (5 − 6)
Thus, A:D = 16:35. = = 12
(3 × 6 − 4 × 5)
4. (a) The ratio between two numbers is a:b. If x bx(c − d )
is added to each of these numbers, the ratio and, the second number =
ad − bc
becomes c:d. The two numbers are given as: 4 × 8 × (5 − 6)
ax(c − d ) bx(c − d ) = = 16.
and . (3 × 6 − 4 × 5)
ad − bc ad − bc
Illustration 11 The ratio of two numbers is 5:9. If each
Explanation number is decreased by 5, the ratio becomes 5:11. Find the
Let two numbers be ak and bk. numbers.
Solution: We have a:b = 5:9, c:d = 5:11 and x = 5.
ak + x c
Given = ⇒ akd + dx = cbk + cx ax(d − c)
bk + x d \ The first number =
⇒ k(ad – bc) = x(c – d) ad − bc
x (c − d ) 5 × 5 × (11 − 5)
⇒k= . = = 15
ad − bc (5 × 11 − 9 × 5)

Chapter_07.indd 151 1/30/2016 2:54:32 PM


152 Chapter 7

bx(d − c) 6. There are four numbers a, b, c and d.


and the second number =
ad − bc (i) The number that should be subtracted from
9 × 5 × (11 − 5) each of these numbers so that the remaining
= = 27. numbers may be proportional is given by
(5 × 11 − 9 × 5)
ad − bc
5. (a) If the ratio of two numbers is a:b, then the .
(a + d ) − (b + c)
numbers that should be added to each of the
numbers in order to make this ratio c:d is
given by Explanation
ad − bc Let x be subtracted from each of the numbers.
.
c−d The remainders are a – x, b – x, c – x and d – x.
a−x c−x
Explanation Given =
b−x d−x
Let the required number be x.
a+x c ⇒ (a – x) (d – x) = (b – x) (c – x)
Given = ⇒ ad + xd = bc + xc ⇒ ad – x(a + d) + x2 = bc – x(b + c) + x2
b+ x d
⇒ x(d – c) = bc – ad ⇒ (b + c)x – (a + d)x = bc – ad
ad − bc bc − ad bc − ad
or x= . \ x= or
c−d (b + c) − (a + d ) (b + c) − (a + d )
(b) If the ratio of two numbers is a:b, then the
(ii) The number that should be added to each of
number that should be subtracted from each
these numbers so that the new numbers may
of the numbers in order to make this ratio c:d
be proportional is given by
is given by
bc − ad bc − ad
. .
c−d (a + d ) − (b + c)

Explanation Explanation
Let the required number be x. Let x be added to each of the numbers.
a−x c The new numbers are a + x, b + x, c + x and d + x.
Given = ⇒ ad – xd = bc – xc
b−x d
a+x c+x
⇒ x(c – d) = bc – ad Given = .
b+ x d+x
bc − ad
or x= . ⇒ (a + x) (d + x) = (b + x) (c + x)
c−d
⇒ ad + x(a + d) + x2 = bc + x(b + c) + x2
Illustration 12 Find the number that must be subtracted
⇒ (a + d)x – (b + c)x = bc – ad.
from the terms of the ratio 5:6 to make it equal to 2 : 3.
Solution: We have a:b = 5:6 and c:d = 2:3. bc − ad
\ x= .
\ The required number (a + d ) − (b + c)
bc − ad 6× 2 − 5× 3 Illustration 14 Find the number subtracted from each of
= = = 3. the numbers 54, 71, 75 and 99 leaves the remainders which
c−d 2−3
are proportional.
Illustration 13 Find the number that must be added to the
terms of the ratio 11:29 to make it equal to 11:20. Solution: We have a = 54, b = 71, c = 75 and d = 99.
Solution: We have a:b = 11:29 and c:d = 11:20. ad − bc
The required number =
\ The required number (a + d ) − (b + c)
ad − bc 11× 20 − 29 × 11 54 × 99 − 71× 75
= = = 11. = = 3.
c−d 11 − 20 (54 + 99) − (71 + 75)

Chapter_07.indd 152 1/30/2016 2:54:33 PM


Ratio and Proportion 153

Illustration 16 The incomes of Mohan and Sohan are in


7. The incomes of two persons are in the ratio of
the ratio 7:2 and their expenditures are in the ratio 4:1. If
a : b and their expenditures are in the ratio of
each saves `1000, find their expenditures.
c : d. If the saving of each person be `S, then their
incomes are given by Solution: We have a:b = 7:2, c:d = 4:1 and S = 1000.
cS (b − a ) 4 × 1000 × (2 − 7)
aS (d − c) bS (d − c) \ A’s expenditure = =
` and ` . ad − bc (7 × 1 − 2 × 4)
ad − bc ad − bc
and their expenditures are given by = `20000
dS (b − a ) 1× 1000 × (2 − 7)
cS (b − a ) dS (b − a ) B’s expenditure = =
` and ` . ad − bc (7 × 1 − 2 × 4)
ad − bc ad − bc
= `5000.
Explanation
8. (a) If in a mixture of x litres of two liquids A and
Let their incomes be `ak and `bk, respectively. Since
B, the ratio of liquids A and B is a:b, then the
each person saves `S,
quantity of liquid B to be added in order to
\ expenditure of first person = `(ak – S) make this ratio.
and expenditure of second person = `(bk – S). x(ad − bc)
c:d is .
ak − S c c ( a + b)
Given =
bk − S d
⇒ akd – Sd = bkc – Sc Explanation
(d − c) S ax
⇒ k(ad – bc) = (d – c)S or, k = . Quantity of liquid A in the mixture = .
ad − bc a+b
Therefore, the incomes of two persons are bx
a(d − c) S b( d − c ) S Quantity of liquid B in the mixture = .
and a+b
ad − bc ad − bc
Let litres of liquid B to be added in order to make this
and their expenditures are
ratio as c:d.
ak – S and bk – S
ax bx
a(d − c) S b( d − c ) S Then, : + l = c:d
that is, – S and –S a+b a+b
ad − bc ad − bc
cS (b − a ) dS (b − a ) ax bx + l (a + b)
or and . or : = c:d
ad − bc ad − bc a+b a+b

Illustration 15 Annual income of A and B is in the ratio of ax c


or =
5:4 and their annual expenses bear a ratio of 4 : 3. If each bx + l (a + b) d
of them saves `500 at the end of the year, then find their or axd = bcx + cl(a + b)
annual income.
Solution: We have a:b = 5:4, c:d = 4:3 and S = 500. x(ad − bc)
or l= .
aS (d − c) ( a + b )c
\ Annual income of A =
ad − bc
5 × 500 × (3 − 4) (b) In a mixture of two liquids A and B, the
=
(5 × 3 − 4 × 4) ratio of liquids A and B is a:b. If on adding
= `2500. x litres of liquid B to the mixture, the ratio of
bS (d − c) A to B becomes a:c, then in the beginning the
and annual income of B = ax
ad − bc quantity of liquid A in the mixture was
4 × 500 × (3 − 4) c−b
= bx
(5 × 3 − 4 × 4) litres and that of liquid B was litres.
c−b
= `2000.

Chapter_07.indd 153 1/30/2016 2:54:34 PM


154 Chapter 7

Explanation Illustration 19 In what ratio the two kinds of tea must be


Let the quantity of mixture be M litres. mixed together into one at `9 per kg and another at `15 per
kg, so that mixture may cost `10.2 per kg?
aM
Then, the quantity of liquid A= litres
a+b Solution: We have c1 = 9, c2 = 15, cm = 10.2
bM q1 c −c
and the quantity of liquid B= litres. ∴ = 2 m
a+b q2 cm − c1
If x litres of liquid B is added, then 15 − 10.2 4.8
= =
a M bM 10.2 − 9 1.2
: + x = a:c
a+b a+b 4
= .
a M bM + x(a + b) 1
or : = a:c
a+b a+b Thus, the two kinds of tea are mixed in the ratio 4:1.
aM a
or = Illustration 20 In a mixture of two types of oils O1 and
bM + x(a + b) c
O1, the ratio O1:O2 is 3:2. If the cost of oil O1 is `4 per litre
or cM = bM + x(a + b)
and that of O2 is `9 per litre, then find the cost/litre of the
x ( a + b)
or M= . resulting mixture.
c−b
We have q1 = q2 = 2, c1 = 4 and c2 = 9.
∴ Quantity of liquid
∴ The cost of resulting mixture
ax(a + b) ax
A= = litres c1 × q1 + c2 × q2
(c − b)(a + b) c−b =
and quantity of liquid q1 + q2
bx(a + b) bx 4×3 + 9× 2 30
B= = litres. = = = `6.
(c − b)(a + b) c−b 3+ 2 5
Illustration 17 729 ml of a mixture contains milk and water
in the ratio 7:2. How much more water is to be added to get 10. (a) If a mixture contains two ingredients A and
a new mixture containing milk and water in the ratio of 7:3. B in the ratio a:b, then
a
Solution: Here x = 729, a : b = 7 : 2 and c:d = 7:3. percentage of A in the mixture = ×
a+b
∴ The quantity of water to be added 100% and percentage of B in the mixture =
x(ad − bc) 729 × (7 × 3 − 2 × 7) b
= = = 81 ml. × 100%.
c ( a + b) 7(7 + 2) a+b
(b) If two mixtures M1 and M2 contain ingredients
Illustration 18 A mixture contains alcohol and water in A and B in the ratios a:b and c:d, respectively,
the ratio of 6:1. On adding 8 litres of water, the ratio of then a third mixture M3 obtained by mixing
alcohol to water becomes 6:5. Find the quantity of water in M1 and M2 in the ratio x:y will contain
the mixture.
 a× x c× y 
Solution: We have a : b = 6 : 1, a : c = 6 : 5 and x = 8. a+b + c+d 
∴ The quantity of water in the mixture   × 100% ingredient A, and
 x+ y 
bx 1× 8
= = = 2 litres.   ax cy 
c−b 5 −1   a + b + c + d 
100% −  
9. When two ingredients A and B of quantities q1   x+ y 
and q2 and cost price/unit c1 and c2 are mixed to   
get a mixture c having cost price/unit cm, then
 bx dy 
q1 c2 − cm c ×q +c ×q a+b + c+d 
(a) = and (b) cm = 1 1 2 2 . or,   × 100% ingredient B.
q2 cm − c1 q1 + q2  x+ y 

Chapter_07.indd 154 1/30/2016 4:36:57 PM


Ratio and Proportion  155

Illustration 21  If a mixture contains water and alcohol in Then, percentage quantity of sliver in the new alloy
the ratio 2:3, what is the percentage quantity of water in the
mixture?  ax cy   3x 5 y 
a+b + c+d   4 + 8 
Solution: Here a = 2, b = 3. =   × 100% =   × 100%
 x+ y   x+ y 
\ Percentage quantity of water in the mixture
6x + 5 y
a = × 100% ...(1)
= × 100% 8( x + y )
a+b
2 Since the ratio of silver and copper in the new alloys
= × 100% is 2:1.
2+3
\ Percentage quantity of sliver in the new alloy
2
= × 100% 2 200
5 = × 100% = %...(2)
2 +1 3
200
= or 40%. From (1) and (2), we get
5
6x + 5 y 2
Illustration 22  Two alloys contain silver and copper in the =
8( x + y ) 3
ratio 3:1 and 5:3. In what ratio the two alloys should be
added together to get a new alloy having silver and copper or 18x + 15y = 16x + 16y
in the ratio of 2:1? or, 2x = y
Solution: We have a:b = 3:1, c:d = 5:3 or, x:y = 1:2.
Let the two alloys be mixed in the ratio x:y. Hence, the two alloys should be mixed in the ratio 1:2.

Multiple Choice QuestionS

1.
Let a, b, c, d and e be integers such that a = 6b = 12c, and 4. A man fell in love with a woman who lived 63 miles
2b = 9d = 12e. Then which of the following pairs contain away. He decided to propose his beloved and invited her
a number that is not an integer? to travel to his place and offered to meet her en route and
bring her home. The man is able to cover 4 miles per hour
 a b  a c to the woman’s 3 miles per hour. How far will each have
(a)  ,  (b)  , 
 27 e   36 e  travelled upon meeting?
(a) Man = 27 miles; woman = 36 miles
 a bd  a c (b) Man = 36 miles; woman = 27 miles
(c)  ,  (d)  , 
 12 18  6 d (c) Man = 40 miles; woman = 23 miles
[Based on CAT, 2004] (d) Man = 45 miles; woman = 18 miles
2. The ratio of the rate of flow of water in pipes varies
5. The price of branded PC (personal computer) and
inversely as the square of the radius of the pipes. What
assembled PC is in the ratio of 64:27. If from now on,
is the ratio of the rates of flow in two pipes of diameters
every year price of branded PC goes on decreasing by 10
2 cm and 4 cm?
per cent and price of assembled PC goes on increasing by
(a) 1:2 (b) 2:1 20 per cent, how after many years the price of both will be
(c) 1:8 (d) 4:1 equal?
[Based on MAT, 2004]
(a) 2 years (b) 3 years
3. Half the girls and one-third of the boys of a college reside 1 1
in the hostel. What fractional part of the student body is (c) 3 years (d) 2 years
3 2
hostel dwellers if the total number of girls in the college is
100 and is one-fourth of the total strength? 6. In a class room, three-fourths of the boys are above 160
(a) Two-fifths (b) Five-twelfths cm in height and they are 18 in number. Also out of the
(c) One-fifth (d) Three-eighths total strength, the boys form only two-thirds and the rest
[Based on MAT, 2004] are girls. The total number of girls in the class is

Chapter_07.indd 155 1/30/2016 2:54:36 PM


156  Chapter 7

(a) 18 (b) 24 14.


If a:b = 2:5, then the value of (2a + 3b):(7a + 5b) is
(c) 12 (d) 20 19 99
(a) (b)
 [Based on MAT, 2004] 39 13
7. A box filled with paper bundles weighs 36 kilograms. If
31 19
the weight of the box and paper bundles, respectively, are (c) (d)
in the ratio of 3:22, then find the weight of the papers in 19 31
[Based on MAT, 1999]
grams.
(a) 30,680 (b) 30,710 15.
If two numbers are in the ratio 6:13 and their least common
multiple is 312, the sum of the numbers is
(c) 31,500 (d) 31,680
(a) 75 (b) 57
8. A, B and C scored 581 runs such that four times A’s runs
are equal to 5 times B’s runs which are equal to seven (c) 76 (d) 67
times C’s runs. Determine the difference between A’s runs [Based on MAT, 1999]
and C’s runs. 16. The ratio of boys to girls in an engineering college is
(a) 125 (b) 120 20:1. How many girls need to be added to make this ratio
(c) 105 (d) 90 8:3?
(a) 26 (b) 43
9. A person spends one-third of the money with him on clothes,
one-fifth of the remaining on food and one-fourth of the (c) 20 (d) Cannot be determined
remaining on travel. Now, he is left with `100. How much 17. A certain product C is made of two ingredients A and B in
did he have with him in the beginning? the proportion of 2:5. The price of A is three times that of
(a) `200 (b) `250 B. The overall cost of C is `5.20 per kg including labour
(c) `300 (d) `450 charges of 80 paisa per kg. Find the cost of B per kg?
[Based on MAT, 2003] (a) `8.40 (b) `4.20
10. One-third of the contents of a container evaporated on the (c) `4.80 (d) `2.80
1st day. three-fourths of the remaining evaporated on the 18.
Two numbers are in the ratio 2:3. If eight is added to both
second day. What part of the contents of the container is the numbers, the ratio becomes 3:4. The numbers are
left at the end of the second day?
(a) 15 and 20 (b) 16 and 24
(a) One-fourth (b) One-half
(c) 13 and 17 (d) 17 and 9
(c) One-eighteenths (d) One-sixth
[Based on MAT, 1999]
[Based on MAT, 2003]
19. A sum of `430 has been distributed among 45 people
11. The monthly incomes of two persons are in the ratio of
consisting of men, women and children. The total
4:5 and their monthly expenditures are in the ratio of 7:9.
amounts given to men, women and children are in the
If each saves `50 a month, then what are their monthly
ratio 12:15:16. But, the amounts received by each man,
incomes?
woman and child are in the ratio 6:5:4. Find, what each
(a) `100, `125 (b) `200, `250 man, woman and child receives (in `).
(c) `300, `375 (d) `400, `500 (a) 12, 10, 8 (b) 18, 15, 12
[Based on MAT, 2002]
(c) 120, 150, 160 (d) 60, 75, 80
12. When a bus started from the first stop, the number of
male passengers to the number of female passengers was 20. The total salary of A, B, C is `444. If they spend 80%,
3:1. At the first stop, 16 passengers got down and 6 more 85%, 75% of their salaries, respectively, their savings are
female passengers got in. The ratio of the male to female as 7:6:9. The salary of B is
passengers now became 2:1. What was the total number (a) `140 (b) `160
of passengers in the bus when it started from the first stop? (c) `144 (d) None of the above
(a) 64 (b) 48
(c) 54 (d) 72 21. A factory employs skilled workers, unskilled workers and
[Based on MAT, 2002] clerks in the proportion 8:5:1 and the wage of a skilled
worker, an unskilled worker and a clerk are in the ratio
13. If the ratio of boys to girls in a class is B and the ratio of
5:2:3. When 20 unskilled workers are employed, the total
girls to boys is G, then 3 (B + G) is
daily wages of all amount to `3180. Find the daily wages
(a) Equal to 3 (b) Less than 3 paid to each category of employees.
(c) More than 3 (d) Less than one-third
(a) 2100, 800, 280 (b) 2400, 480, 300
[Based on MAT, 2001]
(c) 2400, 600, 180 (d) 2200, 560, 420

Chapter_07.indd 156 1/30/2016 2:54:36 PM


Ratio and Proportion  157

22.
The soldiers in two armies when they met in a battle were (a) 6:5 (b) 5:6
in the ratio of 10:3. Their respective losses were as 20:3 (c) 5:4 (d) 4:5
and the survivors as 40:13. If the number of survivors in [Based on Narsee Monjee Inst. of Man. Studies, 2003]
the larger army be 24,000, find the original number of
soldiers in army. 31. The intensity of illumination on a surface from a source of
light varies inversely as the square of the distance of the
(a) 28000, 8400 (b) 25000, 7500
surface from the source. The effect of moving a piece of
(c) 29000, 2750 (d) 26000, 7800 paper 3 times as far from the source is to
23.
What must be added to each of the numbers 7, 11 and (a) Divide the intensity by 3
19, so that the resulting numbers may be in continued (b) Multiply the intensity by 3
proportion? (c) Divide the intensity by 9
(a) 3 (b) 5 (d) Multiply the intensity by 9
(c) 4 (d) –3 [Based on REC Tiruchirapalli, 2003]

24.
`770 have been divided among A, B and C such that A 32.
Suppose y varies as the sum of two quantities of which
receives two-ninths of what B and C together receive. one varies directly, as x and the other varies inversely as
Then A’s share is 1
x. If y = 6 when x = 4 and y = 3 when x = 3, then the
(a) `140 (b) `154 3
relation between x and y is
(c) `165 (d) `170
[Based on MAT, 2001] 4 4
(a) y = x + (b) y = –2x +
x x
25.
What least number must be subtracted from each of the
numbers 14, 17, 34 and 42 so that the remainders are 8 8
(c) y = 2x + (d) y = 2x –
proportional? x x
(a) 0 (b) 1 [Based on FMS (Delhi), 2003]
(c) 2 (d) 7 33. The ratio of the age of a man and his wife is 4:3. After 4
[Based on MAT, 2001] years, this ratio will be 9:7. If at the time of the marriage,
the ratio was 5:3, then how many years ago they were
26.
The mean proportional between 45 and a certain number
married ?
is three times the mean proportional between 5 and 22.
The number is (a) 12 years (b) 8 years
(a) 24 (b) 49 (c) 10 years (d) 15 years
[Based on IIFT, 2003]
(c) 22 (d) 9
34. Pressure varies inversely with volume while temperature
27.
If the mean proportional between X and Y is n times the varies directly with volume. At a time, Volume = 50 m3,
third proportional, then the ratio X:Y will be Temperature = 25º K and Pressure = 1 atmosphere. If the
(a) n2:1 (b) 1:n2 volume is increased to 200 m3, then the temperature will
(c) n2/3:1 (d) 1:n2/3 be
(a) 100º K (b) 50º K
28. At Narmada Sarovar Bachao demonstration, supporters 1º
of Ms. Patkar outnumbered the police by 9:1. The police (c) 12 K (d) 0º K
2
arrested 135 NSB supporters averaging 5 for every 3 [Based on IIFT, 2003]
policemen. How many supporters of NSB were there in
35.
X’s income is three-fourths of Y’s income, and X’s
the demonstration?
expenditure is four-fifths of Y’s expenditure. If X’s income
(a) 405 (b) 665 is nine-tenths of Y’s expenditure, find the ratio of X’s
(c) 1215 (d) None of the above savings to Y’s savings.
[Based on FMS (Delhi), 2004] (a) 1:2 (b) 2:1
2
29.
If x varies inversely as (y – 1) and is equal to 24 when (c) 1:4 (d) 2:3
y = 10, the value of x when y = 5 will be
36. x2 varies directly as y3 and when x = 6, y = 3. Which of the
(a) 100 (b) 101
following equations correctly represents the relationship
(c) 99 (d) None of the above between x and y?
30. Which of the following is the ratio between a number and (a) 6x2 = 3y3 (b) 3y2 = 6x3
the number obtained by adding one-fifth of that number (c) 3x2 = 2y3 (d) 3x2 = 4y3
to it? [Based on IMT Ghaziabad, 2002]

Chapter_07.indd 157 1/30/2016 2:54:37 PM


158  Chapter 7

1 (a) 13:23 (b) 26:79


1
It is given that y ∝
37. . For x = 2, value of y is . If (c) 8:13 (d) 23:36
x3 − x 6
[Based on IRMA, 2002]
x = 1, then the value of y will be
45.
The cost of a bat increased by 10 per cent and the cost of
(a) 1 (b) 0
a ball increased by 18 per cent. Before the price rise, the
(c) –1 (d) None of these ratio of the cost of the bat to the cost of the ball was 9:2.
[Based on IMT Ghaziabad, 2002] If the cost of 12 bats and 54 balls before the price rise was
38.
Between two stations, the first, second and third class `C, what is their cost (in `) now?
fares are in the ratio 9:7:2. The number of passengers (a) 1.12 C (b) 1.13 C
travelling in a day are in the ratio 5:3:2, respectively, in (c) 1.14 C (d) 1.15 C
the above classes. If the sale of tickets generated revenue
of `98,000 that day and if 200 passengers travelled by 46.
Radhika purchased one dozen bangles. One day she
third class, what was the fare for a first class ticket? slipped on the floor fell down. What cannot be the ratio of
(a) `84 (b) `92 broken to unbroken bangles?
(c) `106 (d) `126 (a) 1:2 (b) 1:3
(c) 2:3 (d) 1:5
39.
If x:y = 1:2, y:z = 1:3, z:w = 3:8 and u:w = 2:5, find the
value of (xyu):(w2z). 47.
A precious stone worth `6,800 is accidently dropped and
(a) 1:90 (b) 3:80 breaks into three pieces. The weight of three pieces are in
(c) 1:120 (d) 3:160 the ratio 5:7:8. The value of the stone is proportional to
the square of its weight. Find the loss.
40.
If (x – y + z):(y – z + 2w):(2x + z – w) = 2:3:5, find the (a) `4,260 (b) `4,273
value of S, where S is (3x + 3z – 2w):w. z:w = 3:8 and
(c) `4,454 (d) `3,250
u:w = 2:5, find the value (xyu):(w2z).
(a) 7:1 (b) 6:1 48.
A man spends `8,100 in buying tables at `1,200 each
(c) 13:2 (d) None of these chairs at `300 each. The ratio of chairs to tables when the
maximum number of tables is purchased is
41. A man spends `500 in buying 12 tables and chairs. The (a) 1:4 (b) 5:7
cost of one table is `50 and that of one chair is `40. What
(c) 1:2 (d) 2:1
is the ratio of the numbers of the chairs and the tables
purchased ? 49. From a number of mangoes, a man sells half the number
(a) 5:4 (b) 3:2 of existing mangoes plus 1 to the first customer, then sells
(c) 1:5 (d) None of these one-third of the remaining number of mangoes plus 1 to
[Based on IMT Ghaziabad, 2002] the second customer, then one-fourth of the remaining
number of mangoes plus 1 to the third customer and
42. If x varies directly as 3y + 1 and x = 9 when y = 1, then one-fifth of the remaining number of mangoes plus 1 to the
what is the value of x when y = 5? fourth customer. He then finds that he does not have any
(a) 11 (b) 10 mango left. How many mangoes did he have originally?
(c) 20 (d) 36 (a) 12 (b) 14
[Based on IMT Ghaziabad, 2002] (c) 15 (d) 13
Directions (Qs. 43 to 44): These questions are based on the [Based on FMS (Delhi), 2002]
following information:
50. Two numbers are in the ratio of 1:2. If 7 be added to both,
Out of the total 390 students studying in a college of their ratio changes to 3:5. The greater number is
Arts and Science, boys and girls are in the ratio of (a) 20 (b) 24
7:6 respectively and the number of students studying Arts
(c) 28 (d) 32
and Science are in the ratio of 3:7 respectively. The boys
[Based on FMS (Delhi), 2002]
and girls studying Arts are in the ratio of 4:5 respectively.
51.
The prices of a room air conditioner and an automatic
43. How many boys are studying Science?
washing machine are in the ratio of 3:2. What would be
(a) 52 (b) 65 the price of the washing machine if it costs `6000 less
(c) 115 (d) None of these than the air conditioner?
[Based on IRMA, 2002] (a) `18000 (b) `10000
44. What is the ratio between the girls studying Arts & (c) `12000 (d) `6000
Science respectively? [Based on I.P. Univ., 2002]

Chapter_07.indd 158 1/30/2016 2:54:37 PM


Ratio and Proportion  159

52. The ratio between the number of passengers travelling by Directions (Questions 58 to 60): Answer the questions based on
I and II class between the two railway stations is 1:50, the following information.
whereas the ratio of I and II class fares between the same Alphonso, on his death bed, keeps half his property for
stations is 3:1. If on a particular day, `1325 were collected his wife and divides the rest equally among his three sons:
from the passengers travelling between these stations, Ben, Carl and Dave. Some years later, Ben dies leaving
then what was the amount collected from the II class half his property to his widow and half to his brothers Carl
passengers? and Dave together, sharing equally. When Carl makes his
(a) `750 (b) `850 will, he keeps half his property for his widow and rest he
(c) `1000 (d) `1250 bequeaths to his younger brother Dave. When Dave dies
some years later, he keeps half his property for his widow
[Based on I.P. Univ., 2002]
and the remaining for his mother. The mother now has
53.
A sporting goods store ordered an equal number of `15,75,000.
white and yellow tennis balls. The tennis ball company 58.
What was the worth of the total property?
delivered 45 extra white balls, making the ratio of white
(a) `30 lakh (b) `8 lakh
1 1
balls to yellow balls : . How may white tennis balls (c) `18 lakh (d) `24 lakh
5 6
did the store originally order for? 59.
What was Carl’s original share?
(a) 450 (b) 270 (a) `4 lakh (b) `12 lakh
(c) 225 (d) None of these (c) `6 lakh (d) `5 lakh

54.
A dog takes 6 leaps for every 4 leaps of a hare and 2 leaps 60.
What was the ratio of the property owned by the widows
of the dog are qual to 5 leaps of the hare. What is the ratio of the three sons in the end?
of their speed? (a) 7:9:13 (b) 8:10:15
(a) 8:13 (b) 24:15 (c) 5:7:9 (d) 9:12:13
(c) 12:7 (d) 15:4 61. To win an election, a candidate needs three-fourths of
the votes cast. If, after two-thirds of the votes have been
55. A jar contains black and white marbles. If there are ten
counted, a candidates has five-sixths of what he needs,
marbles in the jar, then which of the following could NOT
then what part of the remaining ratio does he still need?
be the ratio of black to white marbles?
(a) One-eighth (b) One-tenth
(a) 9:1 (b) 7:3
(c) One-fourth (d) Three-eights
(c) 1:10 (d) 1:4
[Based on MAT, 2008]
[Based on REC Tiruchirapalli, 2002] 62. Fresh grapes contain 90 per cent water by weight while
56. Eight people are planning to share equally the cost of a dried grapes contain 20 per cent water by weight. What
rental car. If one person withdraws from the arrangement is the weight of dry grapes contain 20 per cent water by
and the others share equally the entire cost of the car, then weight. What is the weight of dry grapes available from
the share of each of the remaining persons increased by 20 kg of fresh grapes?
(a) One-ninth (b) One-eighth (a) 2 kg (b) 2.4 kg
(c) One-seventh (d) Seven-eighths (c) 2.5 kg (d) None of these
[Based on REC Tiruchirapalli, 2002] 63.
When a number is added to another number the total
1
57. Determine the ratio of the number of people having becomes 3 per cent of the second number. What is the
characteristic X to the number of people having 3
ratio between the first and the second number?
characteristic Y in a population of 100 subjects from the
following table: (a) 3:7 (b) 7:4
(c) 7:3 (d) Data inadequate
Having X and Y 10
[Based on BSRB Mumbai PO, 1998]
Having X but not Y 30 64.
An amount of money is to be distributed among P, Q and
Having Y but not X 20 R in the ratio 6:19:7, respectively. If R gives `200 of his
Having neither X nor Y 40 share to Q the ratio among P, Q and R becomes 3:10:3,
respectively. What was the total amount?
(a) 4:3 (b) 3:2 (a) `6400 (b) `12800
(c) 1:2 (d) 2:3 (c) `3200 (d) Data inadequate
[Based on REC Tiruchirapalli, 2002] [Based on Bank of Baroda PO, 1999]

Chapter_07.indd 159 1/30/2016 2:54:37 PM


160  Chapter 7

65. A man in his will distributed his money in such a way 73.
Mohan is younger than Sohan by 10 years. If 5 years back
that half of it is for his wife, two-thirds of the remaining their ages were in the ratio 1:2, how old is Sohan?
equally to three of his sons and the remaining amount (a) 20 (b) 15
equally to four of his daughters. If each of the daughters
(c) 25 (d) Data inadequate
receives `20000, how much money will each of his sons
[Based on BSRB Chennai PO, 2000]
receive?
(a) `50333.33 (b) `48233.33 74.
Ratio of present ages of P and Q is 7:3. After four years
(c) `53333.33 (d) Data inadequate their ages are in the ratio 2:1. What is the present age of P?
[Based on SBI Associates PO, 1999] (a) 24 years (b) 28 years
66.
The ratio between the present ages of P and Q is 5:8. After (c) 32 years (d) Data inadequate
4 years, the ratio between their ages will be 2:3. What is [Based on BSRB Chennai PO, 2000]
Q’s age at present?
75.
An amount of money is to be distributed among A, B and
(a) 36 years (b) 20 years
C in the ratio 5:8:12, respectively. If the total share of B
(c) 24 years (d) None of these and C is our times that of A. What is A’s share?
[Based on Guwahati PO, 1999]
(a) `3,000 (b) `5,000
67.
The ratio of P’s and Q’s ages is 5:7. If the difference between
(c) Cannot be determined (d) None of these
the present age of Q and the age of P six years hence is 2,
then what is the total of present ages of P and Q? 76.
An amount of `125000 is to be distributed among Raju,
(a) 52 years (b) 48 years Monu and Sonu in the respective ratio of 2:3:5. What will
(c) 56 hears (d) Data inadequate be the difference between Monu and Raju’s share?
[Based on Guwahati PO, 1999] (a) `25000 (b) `12500
68.
An amount of money is to be distributed among P, Q and (c) `18750 (d) `2500
R in the ratio 5:8:12, respectively. If the total share of Q [Based on BSRB Bangalore PO, 2000]
and R is four times that of P, what is definitely P’s share?
77.
The ratio of present ages of Ram and Shyam is 7:8,
(a) `3000 (b) `5000
respectively. Four years hence this ratio becomes 9:10,
(c) `8000 (d) Data inadequate
respectively. What is Ram’s present age in years?
[Based on BSRB Mumbai PO, 1999]
(a) 18 (b) 14
69.
In a business A and C invested amounts in the ratio 2:1,
whereas the ratio between amounts invested by A and B (c) 17 (d) Data inadequate
was 3:2. If `1,57,300 was their profit, how much amount [Based on BSRB Bangalore PO, 2000]
did B receive? 78.
Salaries of A, B and C were in the ratio 3:5:7, respectively.
(a) `72600 (b) `48400 If their salaries were increased by 50 per cent, 60 per cent
(c) `36300 (d) `24200 and 50 per cent, respectively, what will be the new ratio
[Based on BSRB Calcutta PO, 1999] of their respective salaries?
70.
An amount of money is to be divided among P, Q and R (a) 3:6:7 (b) 4:5:7
in the ratio 4:9:16. If R gets 4 times more that P, What is (c) 4:5:8 (d) None of these
Q’s share in it? [Based on BSRB Delhi PO, 2000]
(a) `1800 (b) `2700
79.
The incomes of A, B and C are in the ratio 7:9:12 and
(c) `3600 (d) Data inadequate
their spending are in the ratio 8:9:15. If A saves one-
[Based on BSRB Hyderabad PO, 1999]
fourth of his income, then the savings of A, B and C are
71.
When 30 per cent of a number is added to another number in the ratio of
the second number increases by its 20 per cent. What is
(a) 69:56:48 (b) 47:74:99
the ratio between the first and the second number?
(c) 37:72:49 (d) 56:99:69
(a) 3:2 (b) 2:3
[Based on SSC (GL), 2011]
(c) 2:5 (d) Data inadequate
[Based on NABARD, 1999] 80.
The third proportional of 38 and 15 is
72.
The ratio of A’s and B’s salary is 9:4. If A’s salary is 38 × 38 15
increased by 15%, then his total salary becomes `5175. (a) (b)
15 38 × 38
What is the salary of B?
(a) `2000 (b) `4000 15 × 15 38 × 15
(c) (d)
38 2
(c) `4500 (d) `2500
[Based on BSRB Chennai PO, 2000] [Based on BSRB Patna PO, 2001]

Chapter_07.indd 160 1/30/2016 2:54:37 PM


Ratio and Proportion  161

81.
An amount of money is to be divided among P, Q and R 88.
Three pots have the same volume. The ratio of milk and
in the ratio of 3:5:7, respectively. If the amount received water in first, second and third pots are 3:2, 7:3 and 11:4,
by R is `4,000 more than the amount received by Q, what respectively. If the liquid of three pots are mixed, then the
will be the total amount received by P and Q together? ratio of milk and water in the mixture is
(a) `8,000 (b) `12,000 (a) 61:29 (b) 61:30
(c) `16,000 (d) Cannot be determined (c) 5:4 (d) 29:61
[Based on Gramin Bank U.P. (SO) Exam, 2012] [Based on SI of Police Rec. Exam, 1997]
89. A sum of money is divided among A, B, C and D in the
82. The ratio of students in school A, B and C is 5:4:7
ratio of 3:7:9:13 respectively. If the share of B is `9180
respectively. If number of students in schools are increased
more than the share of A, then what is the total amount of
by 20 per cent, 25 per cent and 20 per cent respectively
money of A and C together?
then what will be the ratio of students in school A, B and
C, respectively? (a) `27540 (b) `27560
(a) 5:5:7 (b) 30:25:42 (c) `26680 (d) `24740
[Based on NMAT, 2008]
(c) 30:20:49 (d) Cannot be determined
[Based on Syndicate Bank PO, 2010] 90.
One year ago, the ratio between A’s and B’s salary was
4:5. The ratio of their individual salaries of last year and
83. On Republic Day sweets were to be equally distributed present year are 3:5 and 2:3 respectively. If their total
among 450 children. But on that particular day, 150 salaries for the present year is `680, the present salary
children remained absent. Thus, each child got 3 sweets of A is (`)
extra. How many sweets did each child get?
(a) 4080.00 (b) 3200.00
(a) 6 (b) 12 (c) 4533.40 (d) 2720.00
(c) 9 (d) Cannot be determined [Based on ATMA, 2008]
[Based on Bank of India PO, 2010]
91. In a certain company, the ratio of the number of managers
84. A and B are two different alloys of gold and copper to the number of production-line workers is 5 to 72. If 8
prepared by mixing metals in the proportion 7:2 and 7:11, additional production-line workers were to be hired, the
respectively. If equal quantities of the alloys are melted to ratio of the number of managers to the production-line
form a third alloy C, find the ratio of gold and copper in C. workers would be 5 to 74. How many managers does the
(a) 5:7 (b) 6:6 company have?
(c) 7:5 (d) 14:13 (a) 10 (b) 20
(c) 30 (d) 25
x 6 x − y 14 [Based on ATMA, 2005]
85. If = , the value of + equals
2y 7 x + y 19 92. A shopkeeper mixes two kinds of flour, one costing `3.50
13 15 per kg and the other at `2.75 per kg. The ratio of first kind
(a) (b) of flour to that of the second is
19 19
(a) 1:2 (b) 1:3
1 (c) 3:4 (d) None of these
(c) 1 (d) 1
19 [Based on NMAT, 2006]
[Based on SI of Police Rec. Exam, 1997] 93.
An AC consumes 8 units of electricity in 30 minutes
86.
The ratio between the annual incomes of A and B is 4:3 and a bulb consumes 18 units of electricity in 6 hours.
and between their annual expenditure is 3:2. If at the end How much total unit of electricity will both AC and bulb
of a year both save `600 each, find the difference in their consume in 8 days if they run 10 hours a day?
incomes. (a) 1280 unit (b) 1528 unit
(a) `450 (b) `500 (c) 1520 unit (c) 1520 unit
(c) `600 (d) `750 [Based on Corporation Bank PO, 2009]

[Based on SI of Police Rec. Exam, 1997] 94.


Monthly income of Anil and Sunil are in the ratio 5:4 and
their expenses are in the ratio 4:3. If each of them saves
87. If a, b, c and d are real numbers such that a:b = b:c = c:d `1200 at the end of the month, their monthly incomes,
and a:d = 8:125, then the value of a:c is respectively are
(a) 25:4 (b) 125:8 (a) `6000, `4800 (b) `8000, `6400
(c) 4:25 (d) 8:25 (c) `8000, `7200 (d) `2000, `1600
[Based on SI of Police Rec. Exam, 1997] [Based on SI of Police Rec. Exam, 1999]

Chapter_07.indd 161 1/30/2016 2:54:38 PM


162  Chapter 7

95. The respective ratio between the speeds of a car, a jeep (a) 88 (b) 96
and a tractor is 3:5:2. The speed of the jeep is 250 per cent (c) 102 (d) 112
the speed of the tractor which covers 360 km in 12 hr. [Based on XAT, 2008]
What is the average speed of car and jeep together?
(a) 60 km/hr (b) 75 km/hr 102.
The number of students in three rooms is 138. The ratio
of the number of students in 1st and the 2nd room is 3:4.
(c) 40 km/hr (d) Cannot be determined
The ratio of the number of students in 2nd and 3rd room is
[Based on CBI (PO), 2010]
7:5. The number of students in the 1st, 2nd and 3rd room
96.
A sum of money is divided among 160 males and some respectively is
females in the ratio 16:21. Individually each male gets `4 (a) 56, 40, 42 (b) 42, 56, 40
and a female `3. The number of females is
(c) 40, 56, 42 (d) 56, 42, 40
(a) 280 (b) 198
[Based on IIFT, 2005]
(c) 284 (d) 270
[Based on SI of Police Rec. Exam, 1999] 103.
Indiacate in which one of the following equations y is
neither directly nor inversely proportional to x
97.
Tea at `126 per kg. and at `135 per kg are mixed with
a third variety in the ratio 1:1:2. If the mixture is worth (a) x + y = 0 (b) 3xy = 10
`153 per kg, the price of the third variety (per kg) is (c) x = 5y (d) 3x + y = 10
(a) `169.50 (b) `175 [Based on FMS, 2011]
(c) `175.50 (d) `185 104.
If p:q = r:s:t:u = 2:3, then
[Based on SI of Police Rec. Exam, 1999] (mp + nr + ot):(mq + ns + ou) is equal to
98. Mr. Pandit owned 950 gold coins all of which he (a) 1:3 (b) 1:2
distributed amongst his three daughters Lalita, Amita and (c) 2:3 (d) 3:2
Neela. Lalita gave 25 gold coins to her husband, Amita [Based on SSC (GL) Prel. Exam, 2000]
donated 15 gold coins and Neeta made jewellery out of
30 gold coins. The new respective ratio of the coins left 105.
The numbers x, y, z are proportional to 2, 3, 5. The sum of
with them was 20:73:83. How many gold coins did Amita x, y and z is 100. The number y is given by the equation y
receive from Mr. Pandit? = ax – 10. Then a is
(a) 380 (b) 415 3
(a) 2 (b)
(c) 400 (d) 350 2
[Based on Punjab National Bank PO, 2010] 5
(c) 3 (d)
99. When 30 per cent of one number is subtracted from 2
another number, the second number reduces to its four- [Based on FMS, 2011]
fifth. What is the ratio between the first and the second 106.
If x varies as the cube of y, and y varies as the fifth root of
number respectively? z, then x varies as the nth power of z, where n is
(a) 4:7 (b) 3:2 1 5
(a) (b)
(c) 2:5 (d) Cannot be determined 15 3
[Based on Allahabad Bank PO, 2010]
3
(c) (d) 15
100.
A sum of `370 is to be divided among A, B and C such that 5
[Based on FMS, 2011]
A's share B's share 3
= =
B's share C's share 4 107.
Instead of walking along two adjacent sides of a
Then A’s share (in rupees) is rectangular field, a boy took a short-cut along the diagonal
of the field and saved a distance equal to half of the longer
(a) 240 (b) 120
side. The ratio of the shorter side of the rectangle to the
(c) 100 (d) 90 longer side is
[Based on Assistant’s Grade Exam, 1997]
1 2
101.
In a cricket match, Team A scored 232 runs without losing (a) (b)
2 3
a wicket. The score consisted of byes, wides and runs
scored by two opening batsmen: Ram and Shyam. The 1 3
(c) (d)
runs scored by the two batsmen are 26 times wides. There 4 4
are 8 more byes than wides. If the ratio of the runs scored [Based on FMS, 2011]
by Ram and Shyam is 6:7, then the runs scored by Ram is

Chapter_07.indd 162 1/30/2016 2:54:38 PM


Ratio and Proportion  163

108.
In counting n coloured balls, some red and some black, 114. A man spends `1810 for buying bags at `200 each and
it was found that 49 of the first 50 counted were red. bottles at `70 each. What will be the ratio of bags to
Thereafter, 7 out of every 8 counted were red. If, in all, bottles when maximum number of bags are bought?
90 per cent or more of the balls counted were red, the (a) 3:8 (b) 8:3
maximum value of n is
(c) 9:1 (d) 1:9
(a) 225 (b) 210
[Based on IBPS Jr. Executive Exam, 2000]
(c) 200 (d) 180
[Based on FMS, 2010] If a:b = 2:3 and b:c = 4:5, find a2:b2:bc
115.
109.
The income distribution in the villages of Delhi (a) 4:9:45 (b) 16:36:45
is symmetrical. Two surveys estimated that the (c) 16:36:20 (d) 4:36:20
implementation of Gramin Rozgar Yozna of central [Based on SSC (GL) Prel. Exam, 2002]
government in a village of Delhi will increase the income
of every villager either 116.
A driver’s income consists of his salary and tips. During
one week, his tips were five-fourths of his salary. What
(i) by a certain proportion or
fraction of his income came from tips?
(ii) by `3650.
(a) Four-ninths (b) Five-ninths
Will the symmetry of income distribution be affected?
(c) Five-eighths (d) Five-fourths
(a) Change in income distribution in case of (i) [Based on MAT, 2000]
(b) Change in income distribution in case of (ii)
117.
A man ordered 4 pairs of black socks and some pairs of
(c) Change in income distribution in both the cases (i) brown socks. The price of a black pair is double that a
and (ii) brown pair. While preparing the bill, the clerk interchanged
(d) No change in income distribution in both the cases (i) the number of black and brown pairs by mistake
and (ii) which increased the bill by 50 per cent. The ratio of the
[Based on FMS, 2009] number of black and brown pairs of socks in the original
order was
110.
Four milkmen rented a pasture. A grazed 18 cows for 4
months, B 25 cows for 2 months, C 28 cows for 5 months (a) 4:1 (b) 2:1
and D 21 cows for 3 months. If A’s share of rent is `360, (c) 1:4 (d) 1:2
the total rent of the field (in rupees) is [Based on MAT, 1999]
(a) 1500 (b) 1600
118.
If P varies as QR and the three corresponding values of
(c) 1625 (d) 1650 P, Q, R be 6 ,9, 10 respectively, then the value of P, when
[Based on FMS, 2006] Q = 5 and R = 3, is
111.
What is the ratio whose terms differ by 40 and the measure (a) 3 (b) 2
of which is two-sevenths?
(c) 1 (d) 4
(a) 6:56 (b) 14:56
[Based on MAT, 1999]
(c) 16:56 (d) 16:72
[Based on FMS, 2005]
119.
Two numbers are such as the square of one is 224 less than
8 times the square of the other. If the numbers be in the
112.
Two persons are climbing up on two moving escalators ratio of 3:4, their values are
which have 120 steps. The ratio of 1st person’s speed
(a) 12, 16 (b) 6, 8
to that of 1st escalator is 2:3 (steps). The ratio of 2nd
person’s speed to that of 2nd escalator is 3:5 (steps). Find (c) 9, 12 (d) None of these
the total number of steps they both have taken together. [Based on MAT, 1999]
(a) 85 (b) 93
(c) 80 (d) 75 x2 + 7 y 2 + 9 z 2
120.
If x:y:z:: 1:3:5, then the value of is
[Based on SNAP, 2007] x
(a) 7 (b) 17
113.
Neeta bought a book at 30 per cent discount on the listed
price. Had she not got the discount, she would have paid (c) 13 (d) 1
`82.50 extra. At what price did she buy the book? [Based on MAT, 1999]
(a) `192.50 (b) `275 121.
Zinc and copper are in the ratio of 5:3 in 200 gm of an
(c) `177.85 (d) Cannot be determined alloy. How many grams of copper should be added to
[Based on Canara Bank PO, 2003] make the ratio 3:5?

Chapter_07.indd 163 1/30/2016 2:54:38 PM


164  Chapter 7

1 1 129.
A, B, C and D have `40, 50, 60 and 70 respectively when
(a) 133 (b) they go to visiit a fair. A spends `18, B spends `21, C
3 200
spends `24, and D spends `27. Who has done the highest
(c) 72 (d) 66 expenditure proportionate to his resources?
[Based on SSC (GL) Prel. Exam, 2002]
(a) A (b) B
122.
Divide `7500 among A, B and C such that A’s share to B’s (c) C (d) D
share is in the ratio 5:2 and B’s share to C’s share is in the
ratio 7:13. How much will B receive? [Based on MAT, 1998]

(a) `1400 (b) `3500 130.


An equilational triangle is described on the diagonal of a
(c) `2600 (d) `7000 square. What is the ratio of the area of the triangle to that
[Based on SSC (GL) Prel. Exam, 2002]
of the square?

123.
An amount of money is to be distributed among A, B and (a) 3 :2 (b) 2: 3
C in the ratio 3:1:5. The difference between B’s and C’s
(c) 3 :4 (d) 4: 3
shares is `3600. What is the total of A’s and B’s shares?
[Based on SSC (GL) Prel. Exam, 2002]
(a) `5400 (b) `3600
(c) `2700 (d) `1800 131.
The total emoluments of A and B are equal. However, A
[Based on MAT, 1999] gets 65 per cent of his basic salary as allowances and B
gets 80 per cent of his basic salary as allowances. What is
124.
The ratio of the prices of two houses A and B was 4:5 last the ratio of the basic sala­ries of A and B?
year. This year, the price of A is increased by 25 per cent
and that of B by `50,000. If their prices are now in the (a) 16:13 (b) 5:7
ratio 9:10, the price of A last year was (c) 12:11 (d) 7:9
(a) `3,60,000 (b) `4,50,000 [Based on MAT, 1997]
(c) `4,80,000 (d) `5,00,000 132.
A contractor employed 25 labourers on a job. He was
[Based on MAT, 1998] paid `275 for the work. After retaining 20 per cent of
125.
Two numbers are in the ratio 5:4 and their difference is 10. this sum, he distributed the remaining amount amongst
What is the larger number? the labourers. If the number of men to women labourers
(a) 30 (b) 40 was in the ratio 2:3 and their wages in the ratio 5:4, what
wages did a woman labourer get?
(c) 50 (d) 60
[Based on MAT, 1998]
(a) `10 (b) `8
(c) `12 (d) `15
1 2
126.
Two numbers are in the ratio 1 :2 . When each of [Based on MAT, 1997]
2 3
2 1 133.
Railway fares of 1st, 2nd and 3rd classes between two
these is increased by 15, they become in the ratio 1 :2 .
3 2 stations were in the ratio of 8:6:3. The fares of 1st and
The greater of the numbers is 2nd class were subsequently reduced by 1/6 and 1/12,
respectively. If during a year, the ratio between the
(a) 27 (b) 36 passengers of 1st, 2nd and 3rd classes was 9:12:26 and
(c) 48 (d) 64 total amount collected by the sale of tickets was `1088,
[Based on SSC (GL) Prel. Exam, 2002] then find the collection from the passengers of 1st class.
127.
The students in three classes are in the ratio 2:3:5. If 40 (a) `260 (b) `280
students are increased in each class, the ratio changes to (c) `300 (d) `320
4:5:7. Originally, the total number of students was [Based on MAT, (Dec), 2006]
(a) 100 (b) 180
1 2 3
(c) 200 (d) 400 134.
The three numbers are in the ratio : : . The difference
2 3 4
[Based on SSC (GL) Prel. Exam, 2002]
between the greatest and the smallest numbers is 36. Find
128.
If A gets 25 per cent more than B and B gets 20 per cent the numbers.
more than C, the share of C out of a sum of `7400 is
(a) 72, 84, 108 (b) 60, 72, 96
(a) `3000 (b) `2000
(c) 72, 84, 96 (d) 72, 96, 108
(c) `2400 (d) `3500
[Based on SSC (GL) Prel. Exam, 2002]
[Based on MAT, 1998]

Chapter_07.indd 164 1/30/2016 2:54:39 PM


Ratio and Proportion  165

135.
Ajay, Aman, Suman and Geeta rented a house and agreed (a) 6 (b) 4
to share the rent as follows (c) 2 (d) None of these
Ajay:Aman = 8:15,
[Based on MAT (May), 2009, (Feb), 2008]
Aman:Suman = 5:8 and
Suman:Geeta = 4:5. 142.
The sum of money is to be divided amongst A, B and C in
The part of rent paid by Suman will be the respective ratio of 3:4:5 and another sum of money is
to be divided between E and F equally. If F got `1050 less
(a) 24/77 (b) 13/66
than A, how much amount did B receive?
(c) 12/55 (d) 13/77 (a) `750 (b) `2000
[Based on MAT (Dec), 2007]
(c) `1500 (d) Cannot be determined
136.
Mira’s expenditure and savings are in the ratio 3:2. Her [Based on MAT (May), 2009]
income increases by 10%. Her expenditure also increases
by 12 per cent. By how much per cent do her savings 143.
Instead of dividing `117 among P, Q, R in the ratio
increase? 1 1 1
: : , it was divided in the ratio 2:3:4 by mistake.
(a) 7% (b) 9% 2 3 4
(c) 10% (d) 13% Who gained in this transaction?
[Based on MAT (Dec), 2008] (a) Only P (b) Only Q
(c) Only R (d) Both Q and R
2 1 2 5 7 3
137. If a:b = : , b:c = : and d:c = : , then a:b:c:d [Based on SSC (GL) Prel. Exam, 2000]
9 3 7 14 10 5
is 144.
The number that must be added to each of the numbers 8,
21, 13 and 31 to make the ratio of first two numbers equal
(a) 4:6:7:9 (b) 16:24:30:35 to the ratio of last two numbers is
(c) 8:12:15:7 (d) 30:35:24:16 (a) 7 (b) 5
[Based on SSC (GL) Prel. Exam, 2003] (c) 9 (d) None of these
138.
The incomes of A, B and C are in the ratio 7:9:12 and [Based on MAT (Sept), 2009]
their spendings are in the ratio 8:9:15. If A saves 145.
Men, women and children are employed to do a work
one-fourth of his income, then the savings of A, B and in the proportion of 1:2:3 and their wages are in the
C are in the ratio of proportion of 6:3:2. When 50 men are employ­ed, total
(a) 56:99:69 (b) 69:56:99 wages of all amount to `4500. What is the weekly wages
(c) 99:56:69 (d) 99:69:56 paid to a man, a woman and a child, in rupees?
[Based on SSC (GL) Prel. Exam, 2003] (a) 210, 105, 80 (b) 210, 105, 70
(c) 210, 105, 90 (d) 200, 105, 70
139.
In an alloy, the ratio of copper and zinc is 5:2. If 1.250 kg
of zinc is mixed in 17 kg 500 gm of alloy, then the ratio of [Based on MAT (Feb), 2011]
copper and zinc will be 146.
One year ago, the ratio between Mahesh’s and Suresh’s
(a) 2:1 (b) 2:3 salaries was 3:5. The ratio of their individual salaries of
(c) 3:2 (d) 1:2 last year and present year are 2:3 and 4:5 respectively. If
their total salaries for the present year are `43000, what is
[Based on SSC (GL) Prel. Exam, 2003]
the present salary of Mahesh?
140.
The ratio between the number of passengers travelling by (a) `19000 (b) `18000
I and II classes between the two railway stations is 1:50,
(c) `16000 (d) `15500
whereas the ratio of I and II classes fares between the
same stations is 3:1. If on a particular day, `1325 were [Based on MAT (Feb), 2011]
collected from the passengers travelling between these 147.
The first, second and third class fares between two stations
stations, then what was the amount collected from the II were 10:8:3 and the number of first, second and third class
class passengers? passengers between the two stations in a day was 3:4:10.
(a) `750 (b) `1000 The sale of tickets to passengers running between two
(c) `850 (d) `1250 stations on that day was `8050. How much was realized
by the sale of second class tickets?
[Based on MAT (Dec), 2008, (May), 2007]
(a) `3000 (b) `2800
141.
What should be subtracted from 15, 28, 20 and 38 so that
(c) `4500 (d) `3500
the remaining numbers may be proportional?
[Based on MAT (Feb), 2011]

Chapter_07.indd 165 1/30/2016 2:54:39 PM


166  Chapter 7

148.
A person distributes his pens among four friends A, B, C 152.
Three numbers A, B and C are in the ratio of 12:15:25. If
and D in the ratio 1/3:1/4:1/5:1/6. What is the minimum sum of these numbers is 312, ratio between the difference
number of pens that the person should have? of B and A and the difference of C and B is
(a) 65 (b) 55 (a) 3:7 (b) 10:3
(c) 23 (d) 57 (c) 3:10 (d) 5:1
[Based on MAT (Sept), 2010]
[Based on MAT (Dec), 2010]
153.
A man left one-half of the property for his wife. One-third
149.
At the start of a seminar, the ratio of the number of male to his son and the remainder to his daughter and her share
participants to the number of female participants was was worth `45000, how much money did the man leave?
3:1. During the tea break, 16 male participants left and 6
(a) `245000 (b) `260000
more female participants registered. The ratio of the male
to the female participants became 2:1. The total number of (c) `270000 (d) `275000
participants at the start of the seminar was [Based on MAT (Sept), 2009]

(a) 112 (b) 48 154.


A man ordered 4 pairs of black socks and some pairs of
brown socks. The price of a black pair is double that of
(c) 54 (d) 72
a brown pair. While preparing the bill, the clerk did a
[Based on MAT (Sept) 2009, (May) 2003, (Dec) 2002] mistake and interchanged the number of black and brown
150.
The prime cost of an article is three times the value of the pairs. This increased the bill by 50 per cent. The ratio
raw material used. The cost of raw materials increases in of the number of black and brown pairs of socks in the
the ratio of 5:12 and manufacturing expenses in the ratio original order was
4:5. The article, which originally cost `6, will new cost (a) 4:1 (b) 2:1
(a) `10 (b) `17 (c) 1:4 (d) 1:2
[Based on MAT (Feb), 2010, (Jan), 1999]
(c) `20.50 (d) None of these
[Based on MAT (Dec), 2010] 155.
Three friends Anita, Bindu and Champa divided `1105
amongst them in such a way that if `10, `20 and `15 are
151.
The sum of the reciprocals of the ages of two brothers is removed from the sums that Anita, Bindu and Champa
five times the difference of the reciprocals of their ages. received respectively, then the share of the sums that
If the ratio of the product of their ages to the sum of their they got will be in the ratio of 11:18:24. How much did
ages is 14.4:1, find their ages. Champa receive?
(a) 36 and 24 year (b) 24 and 20 year (a) `495 (b) `510
(c) 18 and 15 year (d) 12 and 9 year (c) `480 (d) `375
[Based on MAT (Dec), 2010] [Based on MAT (Feb), 2010]

EXPLANATORY ANSWERS

1.
(d) a = 6b = 12c Now, going through the options, the pair in option
1 1 (d) is not an integer:

⇒ a:b:c ≡ 1: :
6 12 a c
is an integer but is not an integer.
⇒ a:b:c = 12:2:1 ...(1) 6 d
It is also given 2.
(d) Radii of the two pipes are 1cm and 2cm.
2b = 9d = 12e
Squares of the radii of the two pipes are 1cm and 4cm.
1 1 1
⇒ b:d:e ≡ : : \ Rates of flow of the two pipes are in the ratio
2 9 12
1
⇒ b:d:e = 18:4:3 ...(2) 1: , i.e., 4:1.
4
From (1) and (2),
a:b:c:d:e = 108:18:9:4:3 3.
(d) Number of girls = 100
⇒ a = 108 K, b = 18K, \ Number of boys = 300
c = 9K, d = 4K, e = 3K \ Number of hostel dwellers = 50 + 100 = 150

Chapter_07.indd 166 1/30/2016 2:54:39 PM


Ratio and Proportion  167

⇒ Required ratio = 150:400



6x 2x
3 = =`
= 3:8 = . 15 5
8
2x
4.
(b) Ratio of speed = Ratio of distance. Therefore, ratio of \ = 100
5
distance = 4:3. Hence, the man travels 36 miles and
⇒ x = 250.
the woman 27 miles.
5.
(b) Let, the price of branded PC be = 64x and that of 2
10.
(d) After first day, rd of the contents remain.
assembled PC = 27x. 3
Let, after n years the prices of both will become equal. 2 3 2 2 1 1
After second day, −   = – = of the
Then, (0, 9)n 64x = (1.2)n × 27x 3 4 3 3 2 6
n contents remain.
64  4 
⇒  
27  3  11.
(d) Let the incomes of the two persons be 4I and 5I
Hence, n = 3 years respectively.
Let the expenditure of the two persons be 7E and 9E
6.
(c) Total number of boys in the class = 24 respectively.
Total number of girls in the class = 12 \ 4I – 7E = 50 and 5I – 9E = 50
Total strength of the class = 36. ⇒ 20I – 35E = 250
7.
(d) The box to paper ratio is 3:22. To arrives the weight 20I – 36E = 200
of the box, we first divide 36000 by 25 to get 1440. ⇒ E = 50 and I = 100
Multiplying this by 3 we get 4320 which is the weight \ Monthly incomes of the two persons are `400 and
of the box. Therefore, weight of the paper is `500 respectively.
36000 – 4320 = 31,680 kg
12.
(a) Let, initially, the number of males and females in the
8.
(c) Given 4A = 5B = 7C bus be 3x and x respectively.
A B C At the first stop, suppose m males and f females left
⇒ = =
35 28 20 the bus.
So, A:B: = 35:28:20 \ At the first stop: No. of the Males is 3x – m
35 and No. of females
∴ A’s Runs = × 581 = 245 (x – f) + 6
(35  28  20)
20 3x − m 2
and C’s runs = × 581 = 140 \ =
83 (x − f ) + 6 1
⇒ 3x – m = 2x – 2f + 12
Thus, difference between A’s score and B’s score
Also m + f = 16
= (245 – 140) = 105
⇒ x = 28 – 3f
9.
(b) Suppose the amount in the beginning was `x
\ f = 4, x = 16
1 \ Total number of passengers in the beginning
Money spent on clothes = ` x
3 = 4x = 64
2 For no other value of f any of the other alternatives
Balance = ` x
3 holds good.

1 2 2 13.
(c) Let number of boys = x
Money spent on food = of x = ` x Let number of girls = y
5 3 15
x y
2 2 8x \ = B and =G
Balance = x− x =` y x
3 15 15
1 8x 2x x y
Money spent on travel = of =` \ 3 (B + G) = 3  + 
4 15 15 y x
8x 2x 3( x 2 + y 2 )
= − = >3.
15 15 xy

Chapter_07.indd 167 1/30/2016 2:54:40 PM


168  Chapter 7

and, the number of children = 45 – (10 + 15) = 20


a 2
14.
(a) = Now, dividing `430 in the ratio 12:15:16
b 5
Total amount of men’s share
a 430  12 
2a + 3b
2 +3 = ` 
= b  43 
7 a + 5b a
7 +5
b = `120

4 Total amount of women’s share


+3
19 430  15 
= 5 = . = ` 
14
+5 39  43 
5
= `150
15.
(c) Let the two numbers be 6K and 13K. Total amount of children’s share
L.C.M. of 6K and 13K = 78K = `[430 – (120 + 150)]
\ 78K = 312 = `160
⇒ K = 4  120 
∴ Each man’s share = `  = `12
\ Sum of the numbers  10 
= 6K + 13K = 19K = 76.  150 
Each woman’s share = `  = `10
16.
(d) Since the ratio is 20:1, the number of boys can be 20,  15 
40, 60, ... and number of girls can be 1, 2, 3. .... .
 150 
Thus, as the number of students is not known, we Each child’s share = `  = `8
 15 
cannot say for sure the number of girls to be required.
Hence, data is insufficient. 20.
(b) Let their respective salaries be X, Y, Z.
17.
(d) Let the price of B per kg be `X. Then the price of A per Then, A saves 0.2 X, B saves 0.15 Y, and C saves
kg = `3X 0.25 Z.
1kg of C contains 2/7 kg of A and 5/7 kg of B We have X + Y + Z = 444 ...(1)
Price of 1 kg of C = (2/7) × 3X + (5/7) X = (11/7) X 0.2 X:0.15 Y:0.25 Z = 7:6:9
By the given condition, 11X/7 ⇒ 4X:3Y:5Z = 7:6:9
= 5.20 – 0.80 Or X = 7Y/8, Z = 9Y/10
= `4.40 Putting in (1), we have Y = 160
⇒ X = 4.40 × (7/11) = `2.80 8 
Hence the price of B per kg = `2.80 (c) Skilled workers =   20 = 32
21.
5 
18.
(b) Let x and y be the two numbers 1 
Number of clerks =   20 = 4
x 2 x+8 3 5 
\ = , =
y 3 y +8 4 Ratio of amount of 32 skilled workers, 20 unskilled
⇒ x = 16, y = 24. workers and 4 clerks
= 5 × 32:2 × 20 : 3 × 4
19.
(a) Ratio of personal shares = 6:5:4
= 160:40:12 or 40:10:3
Ratio of the amounts = 12:15:16
Now, divide `3,180 in the ratio 40:10:3
Ratio of men, women and children 22.
(a) Let the soldiers in the two armies be 10X, 3X and
12 15 16 losses be 20Y, 3Y, then we have
= : :
6 5 4 10X – 20Y = 24000 ...(1)
= 2:3:4 and 3X – 3Y = 24000 × 13/40 = 7800 ...(2)
Sum of these ratios = (2 + 3 + 4) = 9 Solving, we have
 45  2  10X = 28000, 3X = 8400
Number of men =  = 10
 9  23.
(d) Let X be the required number, then
 49  3  (7 + X):(11 + X) = (11 + X):(19 + X)
Number of women =  = 15
 9  ⇒ (7 + X) (19 + X) = (11 + X)2

Chapter_07.indd 168 1/30/2016 2:54:41 PM


Ratio and Proportion  169

⇒ X2 + 26X + 133 = X2 + 22X + 121 N



⇒ 4X = –12 or X = –3 ⇒ y = K  Mx + 
 x
2 10
24.
(a) A = (B + C ) y = ,
9 3
A + B + C = 770
when x = 3
9A
⇒ A+ = 770  N 10
2 ⇒ K  3M +  =
⇒ 11A = 770 × 2  3 3
⇒ A = 140. \ MK = 2 and NK = –8
2x −8
14 − x 34 − x \ p = ,q=
25.
(c) = K Kx
17 − x 42 − x
8
⇒ 588 – 56x + x2 = 578 – 51x + x2 and so y = 2x − .
x
⇒ x = 2.
33.
(a) Man’s age = 4k, say
26.
(c) If X be the required number, then
Wife’s age = 3k, say
(45  X ) = 3  (5  22)
4k + 4 9
or 45X = 9 × 110 or X = 22 \ = ⇒ k = 8.
3k + 4 7
27.
(c) If A and B are mean & third proportional, then
\ Man’s age = 32 years
X/A = A/Y
Wife’s age = 24 years
or A = ( XY ) and X/Y = Y/B Suppose they were married x years ago.
2
or B = Y /X 32 − x 5
2 \ = ⇒ x = 12.
Now ( XY ) = n × Y /X 24 − x 3
or (X/Y)3/2 = n/1 34.
(a) Q Temperature ∝ Volume
or X/Y = n2/3:1 If Temperature = 25º K,
Volume = 50 m3
3
28.
(d) No. of policemen = × 135 = 81 If Volume = 200m3, then
5
Temperature = 100º K.
Since number of supporters: number of policemen =
9:1, therefore number of supporters must be 729. 35.
(a) Let X’s income be 3k then Y’s income is 4k. Let X’s
29.
(c) Let X = K/(Y2 – 1) expenditure be 4g then Y’s expenditure is 5g.
Now 24 = K/(100 – 1) But 3k = 9/10 (5g) or k = 3/2g
or K = 24 × 99. X = K/(Y2 – 1) ⇒ X’s saving/Y’s saving = (3k – 4g)/(4k – 5g)
3(3 / 2 g )  4 g 1
When Y = 5, X = 24 × 99/(25 – 1) = 99 ⇒ =
4(3 / 2 g )  5 g 2
30.
(b) Let the number be x
⇒ X’s saving:Y’s saving = 1:2
x x 5
\ Required ratio = = = . 36.
(d) x2 ∝ y3 ⇒ x2 = Ky3
1 6x 6
x+ x
5 5 \ x = 6, y = 3
4
1 ⇒ K =
31.
(c) I a . 3
R2
\ 3x2 = 4y3.
1
32.
(d) Let y ∝ p + q, where p ∝ x and q ∝ 1
x 37.
(d) y ∝ 3
x −x
\ y = K (p + q),
N K
p = Mx, q = ⇒ y =
x x3 − x

Chapter_07.indd 169 1/30/2016 2:54:42 PM


170  Chapter 7

1 9
When x = 2, y = \ x =  (3y + 1)
6 4
1 K \ When y = 5, x = 36.
\ =
6 8−2 43.
(d) Number of boys = 210
⇒ K = 1 Number of girls = 180
1 Number of students studying Arts and Science are in
\ y = 3
x −x the ratio of 3:7
\ x = 1 \ No. of student studying Arts = 117
1 No. of students studying Science = 273
⇒ y = = ∞.
0 No. of boys studying Arts = 52
38.
(d) Ratio of number of passengers is 5:3:2 No. of girls studying Arts = 65
∴ If 200 passengers travelled by third class, 500 \ No. of boys studying Science = 210 – 52 = 158.
must have travelled by first class. 44.
(a) No. of girls studying Arts = 65
Sum of ratio of amount collected = (5 × 9 + 3 × 7 No. of girls studying Science = 115
+ 2 × 2) = 45 + 21 + 4 = 70. \ Required ratio = 65:115 = 13:23
∴ Amount collected from 1st class 45.
(c) Let the prices of a bat and a ball be 9x and 2x
45 respectively.
=  98000 = `63,000
70 ⇒ The new prices are 9.9x and 2.36x
∴ Fare for the first class Given: 12 × 9x + 54 × 2x = C.
63000  ⇒ 108x + 108x = C
= `  = `126
 500  ⇒ 108x =
C
2
39.
(c) Given x:y = 1:2,  therefore y = 2x
C C
Hence, new price =  1.10   1.18
 8 2 2
Similarly z = 3y = 6x and w =   z = 16x
 3
C
= (2.28) = 1.14 C
 2  32  2
and x =   w    x
 5  5
46.
(c) Since there are 12 bangles, then the number of broken
to unbroken bangles can not be 2 : 3, since 5x = (2x
Therefore,  xyu   1
 w2 z  120 + 3x) can not divide 12 for any integral value of x i.e.,
all the sum of ratios which are the factors of 12 can
40.
(a) Let (x – y + z) = 2k, (y – z + 2w) = 3k possibly be the ratio of broken to unbroken.
and (2x + z – w) = 5k
47.
(c) Total weight of the stone
Then, (x – y + z) + (y – z + 2w)
= 5 + 7 + 8 = 20
= 2k + 3k
Given value α (weight)2
= 5k = 2x + z –w
6800 = K (20)2 ⇒ K = 17
or x + z = 3w
Therefore, value of the pieces
∴ S = (3x + 3z – 2w):w = 7:1
= K (5)2 + K (7)2 + K (8)2
41.
(d) 50T + 40C = 500 = 17 × 25 + 17 × 49 + 17 × 64
T + C = 12 = `2,346
⇒ C = 10, T = 2 ∴ Loss in value = `(6800 – 2346) = `4,454
\ Ratio of the number of chairs and tables = 5:1.
48.
(c) Number of maximum number of tables costing
42.
(d) x ∝ 3y + 1 `1200 each = 6 and from remaining money i.e., (8100
⇒ x = K (3y + 1) – 7200) = `900 he can buy 3 chairs costing `300 each.
Put x = 9, y = 1, Hence ratio of chairs to tables = 3 : 6 = 1 : 2
9 49.
(b) Let the number of mangoes that the man had
we get K =
4 originally = x

Chapter_07.indd 170 1/30/2016 2:54:42 PM


Ratio and Proportion  171

No. of Mangoes sold to Balance \ Increase in the share of each person


x x−2 1 1 1 1 1 1
1st customer = + 1 = − = , i.e., of , i.e.,
2 2 7 8 56 7 8 7
x−2 x−5 of the original share of each person.
2nd customer = +1
6 3 57.
(a) No. of people having characteristic X
= 10 + 30 = 40
x−5 x−9
3rd customer = +1 No. of people having characteristic Y
12 4
= 10 + 20 = 30
x−9
4th customer = + 10 Required ratio = 40:30 = 4:3.
20
58.
(d) Let the total property of Alphonso be `x
x−9 x−9 After first distribution, money possessed by the family
\ + 1 =
20 4 members would be
⇒ x = 14.  x  x  x  x
wife =    , Ben   , Carl   , Dave  
x 1 x+7 3  2  6  6  6
50.
(c) = , =
y 2 y+7 5 After second distribution, money possessed by each
⇒ x = 14, y = 28.  x
of them would be Alphonso’s wife =   , Ben (0),
 12 
AC 3
51.
(c) =
W 2  x x x  x x 
Ben’s wife   , Carl    , Dave   
W = AC – 6000  12   6 24   6 24 
3W After third distribution, money possessed by them
⇒ W = − 6000
2  x
would be Alphonso’s wife   , Ben (0),
⇒ 2W = 3W – 12000  2
⇒ W = 12000.  x  5x 
Ben’s wife   , Carl (0), Carl’s wife   ,
52.
(d) Ratio of the amount collected from the 1st class and  12   48 
the 2nd class passengers = 3:50
 x x 5x  15 x
\ Amount collected from the 2nd class passengers Dave     =
6 24 48  48
50
= × 1325 = `1250. After last distribution, money possessed by them
53
 x 15 x 
53.
(c) Ratio of white to yellow balls = 6 : 5 Alphonso’s wife   , Ben (0),
 2 96 
Difference in the number of white and yellow balls
= 6x – 5x = x = 45  x  5x 
Ben’s wife   , Carl (0), Carl’s wife   ,
 12  48
Therefore, number of white balls now available
= 45 × 6  15 x 
Dave (0), Dave’s wife 
Number of white balls ordered  96 
Now, given that
= (45 × 6) – 45 = 225
 x 15 x 
54.
(d) 2 leaps of the dog = 5 leaps of the hare, or 1 leap of the    = 157500
2 96 
dog = 2.5 leaps of hare
⇒ x = 24,00,000
∴ 6 leaps of dog = 15 leaps of hare. Hence, ratio of
leaps of dog to hare = 15 : 4  x  24 
59.
(a) Carl’s original share =   =   = `4 lakh
 6  6
55.
(c) 1:10 ⇒ There are atleast 11 marbles in the jar.
56.
(c) When there are eight people, the share of each person 60.
(b) Ratio of property owned by the widows of three sons
1 x 5 x 15 x
is of the total cost. = : : = 8:10:15
8 12 48 96
When there are seven people, the share of each person 61.
(d) Suppose total votes = x
1 3 3
is of the total cost. To win a candidate required = ×x= x
7 4 4

Chapter_07.indd 171 1/30/2016 2:54:44 PM


172  Chapter 7

2 2 \ Amount gives to each son


When × x = x votes were counted, a candidate has 1
3 3 = 480000 × 
9
5 3 5
× x = x = `53333.33.
6 4 8
So now he needed P 5 5Q
66.
(d) = or P = ...(1)
3 5 6x − 5x x Q 8 8
x − x = =
4 8 8 8 P+4 2
=
Votes out of remaining Q+4 3
2 x or, 3P + 12 = 2Q + 8
x– x =
3 3 or, 2Q – 3P = 4 ...(2)
x 3 3 Putting value of P from equation (1), we get
\ required ratio = × =
8 8 8
 5Q 
2Q – 3   = 4
62.
(c) Fresh grapes contain 10% pulp.  8 
\ 20 kg fresh grapes contain 2 kg pulp. ⇒ Q = 32 years.
Dry grapes contain 80% pulp. P 5 7P
2 kg pulp would contain 67.
(b) = or, Q =
Q 7 5
2 20 Case I: Q – (P + 6) = 2 or, Q = P + 8
= = 2.5 kg dry grapes.
0.8 8 7P
\ = P + 8 or, 7P = 5P + 40
63.
(c) Let the first and second numbers be x and y, 5
respectively 7
\ P = 20 and Q = × 20
10 7 5
Then, x + y = y or, x = y Q = 28
3 3
\ x:y = 7:3. \ P + Q = 20 + 28 = 48 years.
Case II: (P + 6) – Q = 2
64.
(a) Let the sums of P, Q and R be 6x, 19x and 7x
7
\ Total sum = 6x + 19 + 7x = 32x or, P+6– P = 2 or, P = 10 and Q = 14
5
From the question
\ P + Q = 10 + 14 = 24 years.
6x:19x + 200:7x – 200 = 3:10:3
that is, 6x = 7x – 200 68.
(d) P:Q:R = 5:8:12

\ x = 200 Total share of Q and R 8 + 12 20


= = = 4.
\ Total sum = 32 × 200 = `6400. share of P 5 5
Thus, we see that no new information has been given
65.
(c) Let the man have `x in the question and therefore P’s share cannot be
x determined.
Amount given to his wife =
2 69.
(b) Ratio A:B = 3:2
and A:C = 2:1
x x
Remaining = x – = \ A:B:C = 6:4:3
2 2
4
x 2 1 x Profit share of B = × 1,57,300 = `48400.
\ Amount given to each son = × × = 13
2 3 3 9
70.
(d) Here neither the total amount nor the individual
\ Amount given to each daughter amount is given. So the share of Q cannot be
 x x 1 x determined.
=  −  × =
 2 3  4 24 120
71.
(b) 30% of I + II = II ×
x 100
⇒ = 20000
24 3 12
or, I + II = II
\ x = `480000 10 10

Chapter_07.indd 172 1/30/2016 2:54:45 PM


Ratio and Proportion  173

3 2 79.
(d) let A’s income, B’s income, C’s income be `7x, `9x
or, I = II and `12x, respectively and their expenditures be `8y,
10 10
`9y and `15y respectively
⇒ I:II = 2:3.
Therefore,
72.
(a) Let the salaries of A and B be 9x and 4x
7x
115 7x – 8y =
9x × = 5175 4
100
\ x = 500 ⇒ 4(7x – 8y) = 7x
\ Salary of B = 500 × 4 = `2000 ⇒ 28x – 32y = 7x

73.
(c)
Sohan – Mohan = 10 ...(1) ⇒ 28x – 7x = 32y
⇒ 21x = 32y
Mohan − 5 1
=
Sohan − 5 2 21x
⇒ y =
⇒ 2 Mohan – Sohan = 5 ...(2) 32
On solving (1) and (2), we get 7x
A’s saving =
Mohan = 15 years and Sohan = 25 years. y
P P B’s saving = 9x – 9y
74.
(b) =
Q Q
 21x   32 x 
= 9  x −  = 9 − 21x 
3P  32   32 
⇒ Q = ...(1)
7
9 × 11x 99 x
= =
P+4 2 32 32
and, =
Q+4 1
C’s saving = 12x – 15y
or, P + 4 = 2Q + 8
15 × 21x 69 x
or, 2Q = P – 4 ...(2) = 12 x – =
32 32
Solving equations (1) and (2), we get
Hence, the required ratio
P = 28 years.
7 x 99 x 69 x
75. (c) It cannot be determined because the total money to be = ; ;
distributed is not given. 4 32 32
∴  Number of 50 paise coins = 3K = 144 = 56; 99; 69
76.
(b) Required difference between Monu’s and Raju’s 80.
(c) Let k be the third proportional of 38 and 15
shares 38:15:: 15:k
3− 2 15 × 15
= × 125000 = `12500. ⇒ k = .
2+3+5 38
77.
(b) Ram Shyam
7 8 81.
(c) Let the amount of P, Q, R be `3x, `5x and `7x
4 9 10 respectively.
4 × (10 − 9) Q 7x – 5x = 4000
Present age of Ram = ×7
9 × 8 − 10 × 7 x = 2000
= 14 years. \ Total amount received by P and Q together
78.
(d) Suppose the salaries of A, B and C were 300 k, 500 k = (3 + 5) × `2000
and 700 k, respectively.
= `16000
After increment salary of
A = 300 k + 50% of 300 k = 450 k 82. (b) A:B:C = 5:4:7
B = 500 k + 50% of 500 k = 800 k 120 125 120
After increasing ratio = 5 × :4 × :7 ×
C = 700 k + 50% of 700 k = 1050 k 100 100 100
Hence, new ratio of the respective salaries of A, B and = 600:500:840
C = 450 k:800 k:1050 k = 9:16:21. = 30:25:42

Chapter_07.indd 173 1/30/2016 4:38:41 PM


174  Chapter 7

83. (c) Suppose the number of sweets is = x. 18 + 21 + 22 12 + 9 + 8


= :
x x 30 30
\ – = 3
450 − 150 450
= 61:29.
x x
– = 3 89. (a) Total amount of A and C
300 450
2580
3x − 2 x = × (3 + 9)
= 3 (7 − 3)
900
x = 2700 2580
= × 12 = 27540
4
Number of sweets to each children
2700 90.
(b) Ratio of A’s last year and present year Salary = 3:5 let
= =9 salary be 3x and 5x.
300
Ratio B’s is last year and present year. Salary = 2:3
84.
(c) Suppose 18 kg each is melted. Ratio of gold and i.e., salary be 2y and 3y respectively
copper in one alloy will be 14:4 and in another 7 : 11. 3x 4
\ Ratio of gold and copper in the new alloys Given that, =
2y 5
C = 14 + 7:4 + 11 = 21:15 = 7:5.
⇒ 15x = 8y ... (1)
x 6 x 12 Also, given
85.
(c) = ⇒ =
2y 7 y 7 5x + 3y = 6800 ... (2)
x From Eq. (1) 15x = 8y
−1 12
−1
x− y 14 y 14 14 8
\ + = + = 7 + \ 5x = y
x+ y 19 x 19 12 19 3
+1
y 7
Putting this value in Eq. (2), we have
5 14
= + = 1. 8
19 19 y + 3y = 6800
3
86.
(c) Annual incomes of A and B are 4k and 3k, say. \ y = 1200
Annual expenditure of A and B are 3L and 2L, say.
Put the value of y in Eq. (1), we get 5x = 3200
\ 4k – 3 L = 600
\ A’s present salary = 5x = 3200
3k – 2 L = 600
⇒ k = 600, L = 600 91. (b) According to question,
\ Difference in incomes of A and B 5x 5
=
= 4k – 3k = k = 600. 72 x + 8 74
⇒ x = 4
a b c
87.
(c) = = = k, say. Managers = 5 × 4 = 20
b c d
a 8 92. (d) We do not know the average price of two flours.
Given: =
d 125 93. (c) Total consumption of electricity
a b c = (10 × 16 × 8 + 3 × 10 × 8) units
= × × = k3
b c d = (1280 × 240) units
2 = 1520 units
⇒ k =
5
94.
(a) Suppose income of Anil and Sunil are 5k and 4k,
a a b 4 respectively.
Now, = × = k2 = .
c b c 25
Expenses of Anil and Sunil are 4L and 3L.
88.
(a) Ratio of milk and water \ 5k – 4L = 1200
3 7 11 2 3 4 4k – 3L = 1200
= + + : + +
5 10 15 5 10 15 ⇒ k = 1200, L = `1200

Chapter_07.indd 174 1/30/2016 2:54:47 PM


Ratio and Proportion  175

360 101.
(b)
95. (a) Speed of the Tractor = = 30 km/h
12
250
Speed of the Jeep = 30 × = 75 km/h
100
3
Speed of the Car = × 75 = 45 km/h
5
Average speed of Car and Jeep together
75 + 45
= = 60 km/h
2
96.
(a) Let number of females = F
Suppose 160 males get `16 k and F females get `21 k
\ x + 8 + x + 26x = 232
\ 160 × 4 = 16k
⇒ 28x = 232 – 8 = 224
⇒ k = 40
x = 8
\ F females get `840
\ Ram runs scored = 12x = 96.
840
\ Number of females = = 280.
3 1st 2nd 3rd
102.
(b) : :
3 4 5
97.
(c) Let k kg of 1st variety, k kg of 2nd variety and 2k kg
of 3rd variety of tea are mixed. \ 1st:2nd:3rd = 21:28:20
Let price of the third variety = `x per kg 1st = 42 
\ 126k + 135k + x(2k) = 153(k + k + 2k) 
2nd = 56  By option it can easily be found. 3rd
⇒ x = 175.50. 3rd = 40 

98. (a) Total = 950 coins 103.


(d) Option (a) , y = – x. So, y is directly proportional to x.
Ratio of coins before = (20x + 25):(73x + 15): 10
Option (b) , y = . So, y is inversely proportional to x.
(83x + 30) (Lalita:Amita:Neeta) 3x
Now,  20x + 25 + 73x + 15 + 83x + 30 = 950
1
\ 176x = 880 \ x = 5 Option (c) , y = x . So, y is directly proportional to x.
5
\ Amita = 73x + 15
Option (d) , y varies with x, but it is neither directly nor
inversely proportional to x.
= 73 × 5 + 15 = 380 coins
p r t 2
99. (e) Suppose first number is x and second number is y. 104.
(c) = = =
q s u 3
4
y = 0.3x = y 2q 2s 2u
5 m +n +0
mp + nr + ot 3 3 3 = 2.
y – 0.8 y = 0.3x
\ =
mq + ns + ou mq + ns + ou 3
0.2 y = 0.3 x
105.
(a) Let the value of x, y and z be 2k, 3k and 5k respectively.
x 2
= Sum of x, y and z will be
y 3
2k + 3k + 5k = 100
100.
(d) Suppose A’s share = `3k
10k = 100
B’s share = `4k
⇒ k = 10
16
C’s share = ` So, the numbers are 20, 30 and 50.
3
y = ax – 10
\ `370 are divided among A, B and C in the ratio
9:12:16 ⇒ 30 = a × 10 – 10
9 ⇒ 10a = 20
\ A’s share = × 370 = `90.
37 a = 2

Chapter_07.indd 175 1/30/2016 2:54:47 PM


176  Chapter 7

106.
(c) X varies as the cube of Y and Y varies as the fifth 109.
(b) If income is increased in proportion, then the similarity
root of Z. will be unaltered but if it is increased by a fixed price
it will change.
X ∝ Y3 and Y ∝ Z1/5
X = K1 Y 3 and Y = K2 Z1/5
110.
(c) Ratio of rent = 18 × 4:25 × 2:28 × 5:21 × 3
X = K1(K2 Z1/5)3 72x = 360
⇒ X = K1(K2)3 Z 3/5 x = 5
X = K3 Z  3/5 Total rent = 325 × 5 = `1625
3 111.
(c) By option method 16:56,
As X varies as the nth power of Z, so, n =
5 Now, (56 – 16) = 40
107.
(d) Let the longer and shorter side of the rectangle = L and 16 2
and =
B respectively. 56 7
When the boy took the short cut, the distance covered
 2 120 
by him was = L2  B 2 112.
(b) Steps for 1st person =  
3 1 2 
 
L  3
Given, boy covered L  B  as he saved a distance
2 3 2
= 120   = 48
equal to half the longer side. 5 3
L Steps for 2nd person
L2  B 2 =  B
2
3 120
Taking the square on both sides, = 
5 1 3
L2 5
L2 + B2 =  B 2  LB
4 3 5
= × 120 × = 45
3 2 3 5 8
L = LB ⇒ LB – L2  0
4 4
\ Total steps taken together = 48 + 45 = 93
 3  82 ⋅ 50
L  B  L = 0 113.
(a) List price of book = × 100 = `275
 4  30

3 Neeta bought the book in
Q L ≠ 0, B = L
4 275 × 0.70 = `192.50.
L 4 114.
(b) The man cannot purchase more than 8 bags
=
B 3
\ Cost of 8 bags = 8 × 200 = `1600
3 Remaining amount = `(1810 – 1600) = `210
Thus, the ratio of the shorter side to the longer side =
4 In `210, the man can purchase
108.
(b) In the first 50 balls, 49 are red. If 8x balls are counted 210
after the first 50 balls, then 7x out of them are red. = 3 bottles
70
7 x  49 \ Required ratio = 8:3.
So, the proportion of red balls is
8 x  50
a 2 b 4
Here n = 8x + 50 115.
(b) We have = and =
b 3 c 5
7 x  49 90
≥ a b b c
8 x  50 100 ⇒ = and =
2 3 4 5
\ 700x + 4900 ≥ 720x + 4500
\ 20x ≤ 400 a b c
⇒ = =
\ x ≤ 20 8 12 15
Q n = 8x + 50 \ a:b:c = 8:12:15
\ n ≤ 210 \ a2:b2:bc = 64:144:180
The maximum value of n is 210. = 16:36:45.

Chapter_07.indd 176 1/30/2016 2:54:49 PM


Ratio and Proportion  177

116.
(b) Let salary of the driver be `k. Therefore, income from 122.
(a) A B C
5 5 2
tips = ` k
4 7 13
5 9 5 × 7 : 2 × 7 : 2 × 13
Therefore, total income = k + k= k
4 4 or, 35 : 14 : 26
5 9k 5k 14 × 7500
⇒ of = Hence, share of B =
9 4 4 35 + 14 + 26
5 = `1400.
⇒ of total income = Income from tips
9 123.
(b) Suppose A, B and C get `3K, `K and `5K, respectively.
117.
(c) Let x pairs of brown socks were ordered. \ 5K – K = 3600
Let p be the price of a brown pair. ⇒ K = 900
\ 2p is the price of a black pair. \ A’s share = `2,700
\ (2p × 4 + px) + 50% of (2p × 4 + px) B’s share = `900
= 2p × x + 4p C’s share = `4,500
⇒ x = 16 \ Required total = `3,600

\ Required ratio = 1:4 124.


(a) Let the price of the two houses A and B be 4k and 5k,
respectively.
118.
(c) P a QR, P = 6, Q = 9, R = 10.
According to the question,
⇒ P = KQR,
where K is the constant of proportionality 4k + 25% of 4k 9
=
50000 10
⇒ 6 = K × 9 × 10
1 5k 9
⇒ K = ⇒ =
15 5k + 50000 10

⇒ 15P = QR ⇒ k = 90000
When Q = 5 and R = 3, then P = 1 \ Price of house A = `360000

119.
(b) Let the numbers be x and y. 125.
(c) Let the number be 5k and 4k.
x 3 x y \ 5k – 4k = 10
\ = ⇒ = = K, say
y 4 3 4 ⇒ k = 10

\ x = 3K, y = 4K. ⇒ Largest number = 50
Also y2 = 8x2 – 224 3 8
126.
(c) Let the numbers be x and x
⇒ 16K2 = 72K2 – 224 2 3
⇒ K = 2
3 5 3 x + 30
Therefore, the numbers are 6 and 8. x + 15
2 3 , or, 2 2
= =
8 5 8 x + 45 3
x y z x + 15
120.
(b) = = = k, say 3 2 3
1 3 5
\ x = k, y = 3k, z = 5k 3 x + 30 3 2 3 x + 30 4
or, × = or, = or, x = 18.
8 x + 45 2 3 8 x + 45 9
x2 + 7 y 2 + 9 z 2 k 2 + 7 × 9k 2 + 9 × 25k 2
\ = 8
x k \ greater of the numbers = × 18 = 48.
3
= 289 = 17 127.
(c) Let the number of students in three classes be 2x, 3x
and 5x.
200 × (5 × 5 − 3 × 3) 200 × 16 Now, according to the question
121.
(a) =
3(5 + 3) 3× 8 2x + 40:3x + 40:5x + 40 = 4:5:7
400 2 x + 40 4
= gram. or, =
3 3 x + 40 5

Chapter_07.indd 177 1/30/2016 2:54:50 PM


178  Chapter 7

or, 10x + 200 = 12x + 160 Area of the square with side 1 = 1
\ x = 20 \ Required ratio = 3:2 .
\ total number of students
= 2x + 3x + 5x = 10x 131.
(c) Suppose that the basic salaries of A and B be x and y
respectively.
= 10 × 20 = 200.
\ x + 65% of x = y + 80% of y
128.
(b) Suppose the share of C = `100
65 80
100 + 20 ⇒ x+ x = y+ y
\ Share of B = `100 × 100 100
120 ⇒ 165 x = 180 y
120 x 180 12
= `100 × = `120 ⇒ = =
100 y 165 11

100 + 25 132.
(b) Suppose the wages of each man = `5K and wages of
Share of A = `120 ×
100 each woman = `4K
125 2
`120 ×= `150 Number of men = × 25 = 10
100 5
\ Ratio of shares of A, B, C = 150:120:100 3
Number of women = × 25 = 15
= 15:12:10 5
Sum of ratios = 15 + 12 + 10 = 37 Now `220 are to be divided among 10 men and 15
Total amount = `7400 women.
7400 \ 10 × 5K+ 15 × 4K = 220
Hence, share of C = ` × 10 = `2000
37 ⇒ 110K = 220
129.
(a) Percentage of A’s expenditure ⇒ K = 2.
18 133.
(d) New ratio of fares (1st, 2nd and 3rd)
= × 100 =45%
40
5 11
Percentage of B’s expenditure = 8 × : 6 × : 3 ×1
6 12
21 = 80:66:36 = 40:33:18
= × 100 = 42%
50 Ratio of passengers = 9:12: 26
Percentage of C’s expenditure
⇒ Ratio of amount collected
24 = 40 × 9:12 × 33:26 × 18
= × 100 = 40%
60 = 90:99:117
Percentage of D’s expenditure
Amount collected from 1st class fares
27 4 90
= × 100 = 38 % = × 1088
70 7
306
Hence, A’s expenditure is the highest.
= `320
130.
(a) Area of equilateral triangle with side 2
1 2 3
134.
(d) Let the number be k , k and k
2 3 4
3 1
\ k − k = 36 ⇒ k = 144
4 2
\ The numbers are 72, 96 and 108.

135.
(a) Ajay:Aman = 8:15
Aman:Suman = 5:8
Suman:Geeta = 4:5
3 3 \ Ajay:Aman:Suman:Geeta
= × ( 2) 2 =
4 2 = 8:15:24:30

Chapter_07.indd 178 1/30/2016 2:54:51 PM


Ratio and Proportion  179

\ Part of rent paid by Suman 141.


(c) Let x should be subtracted from each number.
24 24 15 − x 20 − x
= = Then, =
8 + 15 + 24 + 30 77 28 − x 38 − x

⇒ 570 – 38x – 15x + x2 = 560 – 28x – 20x + x2
136.
(a) Let expenditure be `60 and savings be `40.
⇒ 570 – 53x = –48x + 560
Total income = `100
⇒ x = 2
New income = `110
142.
(d) As the sum of money that are to be divided among A,
New expenditure = `67.2
B and C and between E and F are not given. So, the
New savings = 110 – 67.2 = `42.8 amount that B receive cannot be determined.
\ Percentage increase in savings
2.8 1 1 1
= × 100 =7% 143.
(c) If `117 are divided in the ratio : : that is, 6:4:3
40 2 3 4
among P, Q and R, then
2 1 Share of P = `54
137.
(b) a:b = : = 2:3
9 3 Share of Q = `36
2 5 Share of R = `27
b:c = : = 4:5
7 14
If `117 are divided in the ratio of 2:3:4 among, P, Q
3 7 and R, then
c:d = : = 6:7
5 10 Share of P = `26
Now, a:b = 2:3 Share of Q = `39
b:c = 4:5 Share of R = `52
c:d = 6:7
144.
(b) Let that number be x.
\ a:b:c:d = 48:72:90:105
8+ x 13 + x
= 16:24:30:35. Then, =
138.
(a) Let the incomes of A, B and C be 7x, 9x and 12x 21 + x 31 + x

respectively, and the expenditures 8y, 9y and 15y, ⇒ 248 + 31x + 8x + x 2
respectively. We have to find the value of = 273 + 21x + 13x + x2
(7x – 8y):(9x – 9y):(12x – 15y)
⇒ 5x = 25
7x 7x ⇒ x = 5
Also, 7x – 8y = or, 7x – = 8y
4 4
145.
(b) Since, the total number of men employed is 50, then
x 32
or, = women and children are employed 100 and 150
y 21 respectively.
Now, (7 × 32 – 8 × 21):(9 × 32 – 9 × 21):(12 × 32 – 15
× 21) = 56:99:69. Let the men, women and children wages are 6x, 3x
and 2x respectively.
5 \ 50 × 6x + 100 × 3x + 150 × 2x = 4500
139.
(a) Amount of copper = × 17.5 = 12.5 kg
7 ⇒ 900x = 4500
2 ⇒ x = 5
× 17.5 = 5
Amount of zinc =
7 \ Per day wages of men, women and children are
Now, the amount of zinc = 5 + 1.25 `30, `15 and `10.
= 6.25 = 6.25 kg \ Weekly wages of men, women and children are
\ Required ratio = 12.5:6.25 = 2:1. `210, `105 and `70.
140.
(d) Let the number of passengers travelling by Class I and 146.
(b) Let the present and last year salary of Mahesh’s and
Class II be x and 50 x respectively. Suresh’s be x, x′ and y, y′ respectively.
Then, amount collected from Class I and Class II will According to the given condition,
be `3x and `50 x respectively.
x′ 3 x′ 2
Given, 3x + 50x = 1325 = , =
y ′ 5 x 3
⇒ 53x = 1325
⇒ x = 25 y′ 4
and =
\ Amount collected from Class II = 50 × 25 = `1250 y 5

Chapter_07.indd 179 1/30/2016 2:54:51 PM


180  Chapter 7

x′/x 2/3 ⇒ xy = 14.4(x + y) …(2)


\ =
y ′ /y 4/5 From Eqs. (1) and (2),

x = 24 year and y = 36 year
x′ y 10 3 y 10
⇒ × = ⇒ × = 152.
(c) Let three numbers A, B and C are 12x, 15x and 25x
y′ x 12 5 x 12
respectively.
y 50
⇒ = \ 12x + 15x + 25x = 312
x 36
312
Also, x + y = 43000 ⇒ x = =6
52
50
⇒ x+ x = 43000 15 × 6 − 12 × 6
36 \ Required ratio =
25 × 6 – 15 × 6
43000 × 36
⇒ x = = `18000 3× 6 3
86 = =
10 × 6 10
147.
(b) The sale of second class ticket
Short-cut method
8× 4 32
= × 8050 = × 8050 = `2800 There is no need to calculate the value of x.
30 + 32 + 30 92
15 x − 12 x
Required ratio =
1 1 1 1 25 x − 15 x
148.
(d) : : : = 20:15:12:10
3 4 5 6
3x 3
So, the minimum number of pens that the person = =
10 x 10
should have = 20 + 15 + 12 + 10 = 57
149.
(a) Let number of male and female participants at the 153.
(c) Let the man left `x.
start of seminar be 3x and x, respectively. x x
\ Share of daughter = x − −
3 x − 16 2 2 3
Then, =
x+6 1 x

⇒ 45000 =
⇒ 3x – 16 = 2x + 12 6
⇒ x = 28 ⇒ x = `270000
\ Total number of participants at the start of seminar
= 3x + x = 4 × 28 = 112 154.
(c) Let he purchase x pairs of brown socks.
Price of black socks and brown socks be `2a and `a
150.
(d) If the original cost of article is `6.
per pair respectively.
Then, original raw material cost = `2
3
12 \ (4 × 2a + x × a) = x × 2a + 4 × a
New cost of raw material = 2 × = `4.80 2
5
3
Original manufacturing expenses = (6 – 2) = `4 ⇒ 12a + xa = 2xa + 4a
2
5 3
New manufacturing expenses = 4 × = `5 ⇒ 12 + x = 2x + 4
4 2
\ New cost of article = 4.80 + 5 = `9.80 x
⇒ =8
151.
(a) Let their ages be x and y. 2
1 1 1 1 ⇒ x = 16
\ + = 5 − 
x y x y 4 1
\ Required ratio = =
⇒ y + x = 5(y – x) 16 4
⇒ 6x = 4y 155.
(a) 1105 =11x + 10 + 18x + 20 + 24x + 15
x 2 ⇒ 1105 = 53x + 45
⇒ = …(1)
y 3 ⇒ x = 20

xy 14.4 \ Amount with Champa = 24x + 15
Now, =
x+ y 1 = 24 × 20 + 15 = `495

Chapter_07.indd 180 1/30/2016 2:54:53 PM


8 Partnership

INTRODUCTION Compound partnership is one in which the capitals of


partners are invested for different periods.
In partnership, two or more persons carry on a business and
share the profi ts of the business at an agreed proportion. Again partner can be working partner or sleeping
Persons who have entered into partnership with one another partner.
are individually called partners and collectively called a Sleeping partner is one who invests the capital in the
firm and the name under which their business is carried on business but does not actively participate in the conduct of
is called the firm name. The partnership may be simple or business.
compound. Working partner besides investing capital, takes part
Simple partnership is one in which the capital of each in running the business. For his work, he is either paid some
partner is in the business for same time. salary or given a certain per cent of profi t, in addition.

SOME BASIC FORMULAE

Therefore, profi t share of A


1. (a) I f capitals of two partners be `C1 and `C2
for the same period and the total profi t be `P, C1 × P
=
then shares of the partners in the profi ts are C1 + C2 + C3

 C ×P   C2 × P  20000 × 12100
` 1 =
 and `  . 20000 + 50000 + 40000
 C1 + C2   C1 + C2 
2
(b) If the capitals of three partners be `C1, `C2 and = × 12100 = `2200
11
`C3 for the same period and the total profi t be
`P, then shares of the partners in the profi ts are Profi t share of B
C2 × P
 C1 × P   C2 × P  =
` , `   C1 + C2 + C3
 C1 + C2 + C3   C1 + C2 + C3 
50000 × 12100
 C3 × P  =
and ` . 20000 + 50000 + 40000
 C1 + C2 + C3 
5
= × 12100 = `5500.
11
Illustration 1 A, B and C invested `20000, `50000 and and, profi t share of C
`40000, respectively, in a business. The net profi t for C3 × P
=
the year was `12100 which was divided in proportion to C1 + C2 + C3
investments. Find the amount of profi t each partner earned.
40000 × 12100
Solution: We have, C1 = 20000, C2 = 50000, C3 = 40000 =
and P = 12100. 20000 + 50000 + 40000

Chapter_08.indd 181 1/30/2016 3:10:07 PM


182 Chapter 8

4 Illustration 2 A, B are two partners in a business.


= × 12100 = `4400. A contributes `1200 for 5 months and B `750 for 4 months.
11
If total profi t is `450, fi nd their respective shares.
2. (a) If the capitals of two partners be `C1 and Solution: We have C1 = 1200, C2 = 750, t1 = 5, t2 = 4 and
`C2 for the periods t1 and t2, respectively, P = 450.
and the total profi t be `P, then shares of the \ Profi t share of A
partners in the profi ts are: C ×t × P 1200 × 5 × 450
= 1 1 =
 C × t ×P   C2 × t 2 × P  C1t1 + C2t2 1200 × 5 + 750 × 4
` 1 1  and `  
 C1t1 + C2t2   C1t1 + C2t2  2700000
= = ` 300
9000
(b) If the capitals of three partners be `C1, and profi t share of B
`C2 and `C3 for the periods t1, t2 and t3,
respectively, and the total profi t be `P, then C2 × t 2 × P
=
shares of the partners in the profi ts are C1t1 + C2t2

 C1 × t1 × P   C2 × t 2 × P  750 × 4 × 450
` , `   =
1200 × 5 + 750 × 4
 C1t1 + C2t2 + C3t3   C1t1 + C2t2 + C3t3 
 C3 × t3 × P  1350000
and, `   = = `150.
C t +
 11 2 2C t + C t
33
9000

SOME USEFUL SHORT-CUT METHODS

1. (a) If the capitals of two partners be `C1 and 2. If the capitals of three partners are invested
`C2 for the periods t1 and t2 respectively, then in the ratio C1:C2:C3 and their profi ts are in the
ratio P1:P2:P3, then the ratio of timing of their
Profit of A C ×t
= 1 1 . P P P
Profit of B C2 × t 2 investments = 1 : 2 : 3 .
C1 C2 C3
(b) If the capitals of three partners be `C1, `C2 and
`C3 for the periods t1, t2 and t3 respectively,
then profi t of A:profi t of B:profi t of C = C1 × Illustration 4 Anu, Manu and Tanu invested capitals in the
t1:C2 × t2:C3 × t3. ratio 4:6:9. At the end of the business term, they received
the profi ts in the ratio 2:3:5. Find the ratio of time for which
Note: If there is a loss in the business, then they invested their capitals.
Loss of A:Loss of B:Loss of C
Solution: We have C1:C2:C3 = 4:6:9
= C1 × t1:C2 × t2:C3 × t3.
and P1:P2:P3 = 2:3:5
Illustration 3 There are three partners A, B and C in Therefore, the ratio of time for which Anu, Manu and
a certain business. A puts in `2000 for 5 months, B Tanu invested their capitals
`1200 for 6 months and C `2500 for 3 months. Find the
P P P 2 3 5
ratio of their shares in the profi t. = 1: 2 : 3 = : :
C1 C2 C3 4 6 9
Solution: Here C1 × t1 = 2000 × 5 = 10000, C2 × t2 = 1200
× 6 = 7200 and C3 × t3 = 2500 × 3 = 7500. 1 1 5
\ Profi t of A:Profi t of B:Profi t of C or, : :
2 2 9
= C1 × t1:C2 × t2:C3 × t3
or, 9:9:10.
= 10000:7200:7500 or 100:72:75.

Chapter_08.indd 182 1/30/2016 3:10:08 PM


Partnership 183

Solution: We have P1:P2:P3 = 3:4:5


3. Three partners invested their capitals in a business.
If the timing of their investments is in the ratio t1:t2:t3 and t1:t2 : t3 = 1:2:5.
and their profi ts are in the ratio P1:P2:P3, then the P1 P2 P3
P P P \ Required ratio = : :
ratio of their capitals invested is 1 : 2 : 3 . t1 t2 t3
t1 t2 t3
3 4 5
= : : or 3:2:1.
Illustration 5 Gupta, Singhal and Kansal start a business. 1 2 5
If the ratio of their periods of investments are 1:2:5 and their Thus, Gupta, Singhal and Kansal invested their capitals
profi ts are in the ratio of 3:4:5, fi nd the ratio of capitals of in the ratio 3:2:1.
Gupta, Singhal and Kansal.

MULTIPLE CHOICE QUESTIONS

1. A, B, C subscribe `50000 for business. A subscribes 4000 6. Rahul started a business with a capital of `8,000. After
more than B and B `5000 more than C. Out of total profi t six months, Sanjay joined him with an investment of
of `35000 A receives: some capital. If at the end of the year each of them gets
(a) `11900 (b) `8400 equal amount as profi t, how much did Sanjay invest in the
(c) 14700 (d) 13600 business?
[Based on MAT, 2005] (a) `17,500 (b) `18,000
(c) `16,000 (d) `16,500
2. A and B enter into a partnership with `50,000 and `60,000,
[Based on MAT, 2008]
respectively. C joins them after x months contributing
`70,000 and B leaves x months before the end of the year. 7. Three friends X, Y and Z started a partnership business
If they share the profi t in the ratio of 20:18:21, then fi nd investing money in the ratio of 5:4:2, respectively, for a
the value of x. period of 3 years. What is the amount received by X as the
(a) 9 (b) 3 share in the total profi t?
(c) 6 (d) 8 I. Total amount invested in the business is `22000.
[Based on IIT Joint Man. Ent. Test, 2004] II. Profi t was distributed after a period of 2 years.
3. `120 are divided among X, Y and Z so that X’s share is III. The average amount of profi t earned per year is
`20 more than Y’s share and `20 less than Z’s share. What `2750.
is Y’s share? (a) I only (b) II only
(a) `40 (b) `30 (c) III only (d) I or III only
(c) `25 (d) `20 [Based on SBI PO, 1999]
[Based on IIFT, 2003]
8. A and B started a business together, but they both were
4. Surendra, Rajendra and Manindra invested some amount working on different units. The ratio of investment of A
in a business in the ratio of 5:7:6, respectively. In the next and B is 7:5. If A has a loss which is 3 times the profi t
year, they increased their investments by 26 per cent, of B and they get total `6,000 amount (net profi t) back,
20 per cent and 15 per cent respectively. The profi t earned then how much money does both A and B invest if sum of
during the second year should be distributed in what ratio investment of B and 5 times his profi t is 9,500?
among Surendra, Rajendra and Manindra. (a) `8,400 (b) `7,400
(a) 31:27:21 (b) 21:28:23 (c) `9,600 (d) None of these
(c) 26:20:15 (d) Cannot be determined
9. Three friends A, B and C started a business by investing
[Based on IRMA, 2002]
a sum of money in the ratio 5:7:6. After 6 months C
5. X and Y start a business. X invests `3000 for 4 months withdraws half of his capital. If the sum invested by ‘A’
and Y invests `2000 for 6 months. How much should X be is `40000, out of total annual profi t of `33000. C’s share
paid out of a total profi t of `500? will be
(a) `200 (b) `300 (a) `9000 (b) `12000
(c) `250 (d) `350 (c) `11000 (d) `10000
[Based on FMS (Delhi), 2002] [Based on BSRB Hyderabad PO, 1999]

Chapter_08.indd 183 1/30/2016 3:10:08 PM


184  Chapter 8

10. Mr Saxena started a business investing `5000. Four 17. A, B and C invested `10000, `14000 and `12000,
months later Mr Jain joined the business by investing respectively, in a business. If at the end of a year, they
`9000. If the profit at the end of the year was `22000 how got a profit of `5400, the share of B in profit (if profit is
much amount would Mr Jain have received as the profit? divided in proportion to the investment of each) is
(a) `16000 (b) `14000 (a) `1500 (b) `2100
(c) `12000 (d) `11000 (c) `1800 (d) `150
 [Based on BSRB Delhi PO, 2000] [Based on Assistant’s Grade Exam., 1998]
11. Vinod and Ankit started a business, investing amounts in 18. A starts business with `3500 and after 5 months B joins
the ratio 2:3. If Vinod had invested an additional amount with A as his partner. After a year, the profit is divided in
of `10000, the ratio of Vinod’s investment to Ankit’s the ratio 2:3. What is B’s contribution in the capital?
investment would have been 3:2. What was the amount
invested by Vinod? (a) `8000 (b) `8500
(a) `8000 (b) `12000 (c) `9000 (d) `7500
(c) `9000 (d) Data inadequate [Based on SSC (GL) Prel. Exam., 2000]
[Based on BSRB Patna PO, 2001] 19. A, B and C rent a pasture. A puts 10 oxen for 7 months,
12. Vinay started a business investing `50000. After one year B puts 12 oxen for 5 months and C puts 15 oxen for 3
he invested another `30000 and Aditya also joined him months for grazing. If the rent of the pasture is `175, how
with a capital of `70000. If the profit earned in three years much must C pay as his share of rent?
from the starting of business was `87500, find the share of (a) `45 (b) `50
Aditya in the profit. (c) `55 (d) `60
(a) `37500 (b) `32500 [Based on SSC (GL) Prel. Exam., 2000]
(c) `38281 (d) None of these
20. A, B and C enter into a partnership and their shares are in
[Based on Corporation Bank PO, 2002]
1 1 1
13. A started a business with `10000 and B joined him later the ratio : : . After 2 months, A withdraws half of
2 3 4
with a capital of `40000. If at the end of the year, they
both get an equal share of the profit, how many months its capital and after 10 months a profit of `378 is divided
after the business started did B join it? among them. What is B’s share?
(a) 4 months (b) 6 months (a) `129 (b) `144
(c) 8 months (d) 9 months (c) `156 (d) `168
[Based on SI of Police Rec. Exam., 1997] [Based on SSC (GL) Prel. Exam., 2000]
14. A, B, C enter into a partnership investing `35000, `45000 21. A and B are partners in a business. A contribute one-fourth
and `55000, respectively. The respective shares of A, B, C of the capital for 15 months and B received one-fourth of
in an annual profit of `40500 are the profit. Find for how long B’s money was used?
(a) `13500, `16500, `10500 (a) 1 year (b) 9 months
(b) `10500, `13500, `16500 (c) 6 months (d) 10 months
(c) `13500, `10500, `16600 [Based on SSC (GL) Prel. Exam., 2000]
[Based on SI of Police Rec. Exam., 1997]
22. A, B and C are partners of a company. During a particular
15. Three men rent a pasture for `660. The first uses it to year A received one-third of the profit, B received one-
graze 50 sheep for 4 months, the second uses it to graze fourth of the profit and C received the remaining `5000.
40 sheep for 3 months and the third 46 sheep for 5 months. How much did A receive?
How much should the first man pay as rent?
(a) `5000 (b) `4000
(a) `276 (b) `220
(c) `3000 (d) `1000
(c) `144 (d) `240
[Based on SSC (GL) Prel. Exam., 2000]
[Based on SI of Police Rec. Exam., 1997]
16. A began business with `45000 and B joined afterwards 23. A began a business with `4500 and was joined afterwards
with `30000. At the end of a year, the profit is divided in by B with `5400. When did B join if the profits at the end
the ratio 2:1. When did B join? of the year were divided in the ratio 2:1?
(a) 3 months after (b) 6 months after (a) 4 months (b) 5 months
(c) 8 months after (d) 9 months after (c) 6 months (d) 7 months
[Based on Assistant’s Grade Exam., 1997] [Based on SSC (GL) Prel. Exam., 2000]

Chapter_08.indd 184 1/30/2016 3:10:09 PM


185
Partnership 

24. Vinay started a business investing `70000. Ashok joined (a) `336 (b) `366
him after six months with an amount of `105000 and Sunil (c) `633 (d) `663
joined them with `1.4 lakhs after another six months. The
[Based on MAT (Sept), 2009, 2008)]
amount of profit earned should be distributed in what ratio
among Vinay, Ashok and Sunil, respectively, three years 31.
Sita and Gita enter into a partnership, Sita contributes
after Vinay started the business? `5000 while Gita contributes `4000. After 1 month, Gita
(a) 42:45:56 (b) 7:6:10 withdraws one-fourth part of her contribution and after 3
months from the starting, Sita puts `2000 more. When Gita
(c) 12:15:16 (d) Cannot be determined
withdraws her money Rita also joins them with `7000.
[Based on PNB Management Trainee, 2003] If at the end of 1 year, there is profit of `1218, what will
25.
`300 is divided among A, B and C. The ratio of A’s share be the share of Rita in the profit?
to the sum of B and C’s share equals ratio of B’s share to (a) `844.37 (b) `488.47
the sum of A and C’s share and also equals the ratio of C’s
(c) `588.47 (d) None of these
share to the sum of A and B’s share. Then, A, B and C’s
share, respectively, is [Based on MAT (Feb), 2009]

(a) 150, 100, 150 (b) 100, 150, 50 32.


Three partners invested capital in the ratio 2:7:9. The time
(c) 100, 100, 100, (d) 150, 50, 100 period for which each of them invested was in the ratio of
the reciprocals of the amount invested. Find the share of
26.
A and B enter into a partnership. A puts in the whole the partner who brought in the highest capital, if profit is
capital of `45,000 on the condition that the profits will be `1080.
equally divided after which B will pay A interest on half
(a) `120 (b) `360
the capital at 10 per cent p.a. and receive `60 per month
from A for carrying on the concern. What is the yearly (c) `540 (d) `420
profit, if B’s income is half of A’s income? [Based on MAT (Sept), 2008]
(a) `8,190 (b) `9,180 33.
A, B, C start a business. A invests three times as much as B
(c) `3,600 (d) `6,900 invests and B invests two-thirds of what C invests. Then,
the ratio of capitals of A, B and C is
27.
Anu is a working partner and Bimla is a sleeping partner
in a business. Anu puts in `5000 and Bimla puts in (a) 3:9:2 (b) 6:10:15
`6000. Anu receives 12.5 per cent of the profit for (c) 5:3:2 (d) 6:2:3
managing the business and the rest is divided in proportion
34.
A, B, C enter into a partnership with shares in the ratio
to their capital. What does each get out of a profit of `880?
7 4 6
(a) `400 and `480 (b) `460 and `420 : : . After 4 months, A increases his share by
2 3 5
(c) `450 and `430 (d) `470 and `410 50 per cent . If the total profit at the end of one year be
[Based on MAT (Sept), 2010, (Dec), 2009] `21,600, the B’s share in the profit is
28.
A started a business with a capital of `2,100. After (a) `2,100 (b) `2,400
4 months he admitted another partner B. What amount
(c) `3,600 (d) `4,000
should B put in so that the profit may be divided equally
at the end of the year? 35.
A, B and C enter into partnership in a business with
(a) `3,000 (b) `4,120 capitals of `5,000, `6,000 and `4,000 respectively. A gets
30 per cent of the profit for managing the business and
(c) `3,150 (d) `3,600
balance is divided in proportion to their capitals. At the
29.
X and Y are partners in a business. X contributed one-third end of the year, A gets `200 more than B and C together.
of the capital for 9 months and Y received two-fifths of the Find the total profit.
profits. For how long was Y’s money used in the business? (a) `3,600 (b) `3,000
(a) 4 months (b) 3 months (c) `2,875 (d) `2,550
(c) 2 months (d) 5 months
36.
A and B enter into partnership. A supplies whole of the
[Based on MAT (Sept), 2010, (Dec) 2009] capital amounting to `45,000 with the condition that the
30.
X and Y entered into partnership with `700 and `600 profit are to be equally divided and that B pays the interest
respectively. After 3 months, X withdrew two-sevenths of on half the capital to A at 10 per cent per annum, but receives
his stock but after 3 months, he puts back three-fifths of `120 per month for carrying on the concern. Find their
what he had withdrawn. The profit at the end of the year is total yearly profit when B’s income is one half of A’s
`726. How much of this should X receive? income.

Chapter_08.indd 185 1/30/2016 3:10:09 PM


186  Chapter 8

(a) `9,215 (b) `9,000 a fourth of the profit for this services and the remaining
(c) `9,227.5 (d) `9,180 profit is divided amongst the three in the ratio of their
investments. What is the amount of profit that B gets if A
37.
Three shepherds A, B and C rented a pasture for a year. gets a total of ` 4995?
A grazed 22 sheep for 4 months, B grazed 16 sheep for 8
(a) `1665 (b) `2960
months and C grazed 32 sheep for 6 months. If C’s share
of rent is `600, the total rent for the year was (c) `2590 (d) Cannot be determined
[Based on MHT-CET MBA, 2010]
(a) `3,550 (b) `2,000
(c) `1,275 (d) `675 44. Firoz invested `650000 to start a business. Dhruv joined
him six months later by investing `800000. At the end of
38.
A, B and C enter into partnership. A invests some money two years from the commencement of the business, they
at the beginning, B invests double the amount after 6 earned a profit of `435000. What is Firoz’s share of the
months and C invests thrice the amount after 8 months. profit?
If the annual profit be `27000, C’s share is
(a) `195000 (b) `185000
(a) `9000 (b) `11250
(c) `240000 (d) None of these
(c) `10800 (d) `8625  [Based on IRMA, 2006]
[Based on FMS (MS), 2006]
45.
A, B and C invested their capital in the ratio 5:6:8. At the
39.
John, Mona and Gordon, three US based business end of the business they received the profits in the ratio
partners, jointly invested in a business project to supply 5:3:1. Find the ratio of time for which they contributed
nuclear fuel to India. As per their share in the investment, their capital.
Gordon will receive 2/3 of the profits whereas John and
(a) 12:9:7 (b) 25:18:8
Mona divide the remainder equally. It is estimated that the
income of John will increase by $60 million when the rate (c) 5:6:8 (d) 8:4:1
of profit rises from 4 per cent to 7 per cent . What is the 46.
Three bachelors, Amar, Akbar and Anthony rented a
capital of Mona? house for a year. Amar left after 4 months, Akbar stayed
(a) $ 2000 million (b) $ 3000 million for 8 months and only Anthony stayed for the entire year.
(c) $ 5000 million (d) $ 8000 million If the annual rent was `6,000, find the share of Akbar.
[Based on FMS, 2009] (a) `4,000 (b) `2,000
40.
Amber Chew opened a departmental store at Great India (c) `300 (d) `2,500
Palace in Noida by investing `20 million. After a few 47. Average score of Rahul, Manish and Suresh is 63. Rahul’s
months her brother Sheesh Chew joined the business and score is 15 less than Ajay and 10 more than Manish. If
invested `30 million. At the end of the year, the profit was Ajay scored 30 marks more than the average scores of
shared in the ratio of 3:2. After how many months did Rahul, Manish and Suresh, what is the sum of Manish’s
Amber’s brother join the business? and Suresh’s scores?
(a) 4 months (b) 6 months (a) 120 (b) 111
(c) 7 months (d) 8 months (c) 117 (d) Cannot be determined
[Based on FMS, 2009] [Based on Corporation Bank PO, 2011]
41.
A and B invest `60,000 and `80,000 in a business. A 48.
A and B enter into partnership, investing `12,000 and
receives `100 per month out of profit for running the `16,000, respectively. After 8 months, C joins them with
business and the rest of the profit is divided in the ratio of a capital of `15,000. The share of C in a profit of `45,600
investments. If A receives `3,900 annually, B receives after 2 years will be
(a) `3,200 (b) `2,700 (a) `21,200 (b) `19,200
(c) `3,600 (d) `2,925 (c) `14,400 (d) `12,000
42. `1950 is divided amongst three workers A, B and C such 49. Sumant started a business investing `48000. After
that 6 times A’s share is equal to 4 time B’s share which is 6 months Maurya joined him with a captial `56000.
equal to 8 times C’s share. How much did A get? At the end of the year the total profit was `24529. What
(a) `600 (b) `550 is the difference between the share of profits of Sumant
and Maurya?
(c) `900 (d) `450
(a) `6455 (b) `7775
[Based on MHT-CET MBA, 2010]
(c) `5545 (d) `4875
43. A, B and C enter into a partnership by investing ` 28000,
[Based on NMAT, 2008]
` 32000 and `  18000. A is a working partner and gets

Chapter_08.indd 186 1/30/2016 3:10:09 PM


187
Partnership 

50. Ram and Shyam form a partnership (with Shyam as (a) 520,000 (b) 530,000
working partner) and start a business by investing `4,000 (c) 540,000 (d) 550,000
and `6,000, respectively. The conditions of partnership
[Based on XAT, 2012]
are as follows:
• In case of profits till `200,000 per annum, profit 51.
A, B and C enter into a partnership by making investments
would be shared in ratio of the invested capital. in the ratio 3:5:7. After a year, C invests another `3,37,600
while A withdraws `45,600. The ratio of investments then
• Profits from `200,001 till `. 400,000, Shyam would changes to 24:59:167. How much did A invest initially?
take 20% out of the profit, before the division of
(a) `45,600 (b) `96,000
remaining profits, which will then be based on ratio of
invested capital. (c) `1,41,600 (d) None of these
• Profits in excess of `400,000 Shyam would take 35% 52.
Eight people enter into a partnership; 6 of them bring in
out of the profits beyond `400,000, before the division `30 each. The seventh person brings in `10 more than the
of remaining profits, which will then be based on ratio average of eight persons, and the eighth person brings in
of invested capital. `55. What is the total sum brought in?
If Shyam’s share in a particular year was `367,000, which (a) `40 (b) `240
option indicates the total business profit (in `) for that (c) `280 (d) `250
year?

EXPLANATORY ANSWERS

1. (c) Let C = x 4. (b) 5K + 26% of 5K:7K + 20% of 7K:6K + 15% of 6K


B = x + 5000, 630 840 690
⇒ K: K: K
A = x + 5000 + 4000 100 100 100
\ x + x + 5000 + x + 9000 ⇒ 63:84:69
= 50000 ⇒ 21:28:23.
\ x = 12000 5. (c) The ratio in which X and Y should be paid the profit
out of `500.
\ A:B:C = 21000:17000:12000
= 3000 × 4:2000 × 6 = 12:12 = 1:1
= 21:17:12
\ Each of X and Y should be paid `250.
21
A’s share = 35000 × = `14700. 8000 × 12 1
50 6.
(c) = ⇒ x = 16000
x×6 2
2. (b) Let B leaves x months before the end of the year.
7.
(c) When investment ratio is given, the amount of profit
\ B stays for (12 – x) months. can be found out with the help of III only.
C joins after x months. 8. (a) Let 7x and 5x be the investments of A and B,
\ C also remains for (12 – x) months. respectively
\ Profits are shared among A, B and C in the ratio. Let 3y and y be the loss of A and profit of B,
50000 × 12:60000 (12 – x):70000 (12 – x) respectively, then, 7x – 3y + 5x + y = 6000
⇒ 6x – y = 3000 ...(1)
= 20:18:21 (Given)
60 20 and 5x + 5y = 9500
\ = ⇒ x = 3. ⇒ x + y = 1900 ...(2)
6(12 − x) 18
(1) + (2) ⇒ x = 700
3. (d) X + Y + Z = 120 Hence, total money invested = 7x + 5x = `8,400
X = Y + 20, X = Z – 20
9. (a) Sum invested by A, B and C is
⇒ (Y + 20) + Y + (X + 20) = 120 5 × 12:7 × 12:6 × 6 + 3 × 6
⇒ X + 2Y = 80 or, 60:84:54
⇒ (Y + 20) + 2Y = 80 or, 10:14:9
⇒ 3Y = 60 9
\ Share of C = × 33000 = `9000.
⇒ Y = 20. 33

Chapter_08.indd 187 1/30/2016 3:10:09 PM


188  Chapter 8

10. (c) Ratio of their investment 17.


(b) Profit is divided between A, B and C in the ratio
= 50000 × 12:90000 × 8 = 5:6 10:14:12
i.e., 5:7:6
\ Amount received by Mr Jain

7
6 \ B’s share = × 5400 = `2100.
= × 22000 = `12000. 18
11
18.
(c) Let B’s contribution be `x
11. (a) Let the initial investments of Vinod and Ankit be 2x
and 3x, respectively. 3500 × 12 2
\ =
According to question, 7x 3

2 x + 10000 3 3500 × 12 × 3
= or, x =
3x 2 7×2
or, 4x + 20000 = 9x \ x = 9000.
\ x = 4000 19.
(a) A, B and C pay in the ratio of 10 × 7:12 × 5:15 × 3
\ Amount invested by Vinod
i.e., 70:60:45
= 2x = `8000.
i.e., 14:12:9
12. (d) Ratio of Vinay and Aditya for one month
9
= (50000 × 12) + (80000 × 24):(70000 × 24) C pay `
\ × 175 = `45.
35
= 60000 + 1920000:1680000 = 3:2
Hence, share of Aditya in the profit earned from the 20.
(b) Suppose, shares of A, B and C are 6K, 4K and 3K,
87,500 respectively.
business = × 2 = `35000.
3+ 2 \ Profit of 378 is divided among them in the ratio of
6K × 2 + 3K × 10:4K × 12:3K × 12
13. (d) Suppose, B joins A after K months. i.e., 42:48:36
\ Investments of A and B are in the ratio of
i.e., 7:8:6
10000 × 12:40000 × (12 – K) 8
\ B’s share = × 378 = `144.
\ 10000 × 12 = 40000 × (12 – K) 21
i.e., 4 (12 – K) = 12 ⇒ K = 9.
1
14. (b) A, B, C invest in the ratio 35000:45000:55000 21.
(d) A’s capital = K
4
i.e., 7:9:11
7 3
\ A’s share in the profit = × 40500 B’s capital = K
27 4
= 7 × 1500 = `10500 A and B share the total profit in the ratio 1:2.
B’s share in the profit = 9 × 1500 = `13500 Let B contribute for x months
C’s share in the profit = 11 × 1500 = `16500. 1
K × 15
4 1
15. (d) They pay in the ratio 50 × 4:40 × 3:46 × 5 \ = ⇒ x = 10.
3 2
i.e., 200:120:230 K×x
4
i.e., 20: 12:23
22.
(b) Profit earned by
20
\ 1st man should pay ` × 660 = `240.
55 1 1 7 5
C = 1 −  +  = 1 − =
16. (d) Suppose B joined after K months 3 4 12 12
\ Profit is divided in the ratio of 5
So, = 5000
45000 × 12:30000 × K 12
45000 × 12 2 \ 1 → 12000
\ =
30000 × K 1 \ Profit received by
⇒ 60000 K = 540000 1
A = × 12000 = `4000.
\ K = 9. 3

Chapter_08.indd 188 1/30/2016 3:10:10 PM


189
Partnership 

23. (d) Suppose, B joined after K months Let Y’s money was used for n months.
\ 4500 × 12:5400 × (12 – K) = 2:1 \ (1 × 9):(2 × n) = 3:2
4500 × 12 2 ⇒ n = 3 months
⇒ =
5400 × (12 − K ) 1 30.
(b) X’s investment = (700 × 3)
60 2  5   3
⇒ = +  700 × × 3  +  500 + 200 ×  × 6
6(12 − K ) 1  7   5
⇒ 12(12 – K) = 60 = `7320
⇒ K = 7. Y’s investment = 600 × 12 = `7200.
24. (c) Ratio of their investments \ X’s share from profit
= 70 × 36:105 × 30:140 × 24 7320
= × 726 = `366
= 12:15:16. (7320 + 7200)
25. (c) `300 is to be divided among A, B and C 31. (b) Ratio of investment of Sita, Gita and Rita is (5000 ×
Let A be A’s share, B be B’s share and C be C’s share 3 + 7000 × 9):(4000 × 1 + 3000 × 11):(7000 × 11)
A B C = 78000:37000:77000
∴ = =
BC AC AB = 78:37:77
ABC 1 \ Share of Rita in profit
= = 77
2( A  B  C ) 2 = × 1218
78 + 37 + 77
∴ A = B = C = `100
≈ `488.47
26. (b) Interest on `22,500 = 0.1 × 22,500 = `2,250
Charges for managing the concern = 60 × 12 32. (b) Ratio of capital = 2:7:9
= `4,720 1 1 1
Ratio of time = : :
If yearly profit is `x, then B’s share and A’s share  2 7 9
x \ Ratio of investment
= 
2 1 1 1
= 2 × : 7 × : 9 × = 1:1:1
x 1  x  2 7 9
∴  – 2250 + 720 =   720  2250
2 
2 2  \ Share of each partner
∴ x = `9,180 1
= × 1080 = `360
3
12.5
27. (b) 12.5% of profit = × 880 = `110 33. (d) Let C invests `x, then ratio of investments of A, B
100
and C.
Remaining `770 is divided in the ratio
2x
= 5000:6000 = 5:6 = 2x : : x  = 6 : 2 : 3
3
5
Profit of Anu = × 770 + 110 = `460 34. (d) Ratio of investments of A, B and C
11
6 7 105  4  12 6  12
Profit of Bimla = × 770 = `420   4  8 : :
11  2 2  3 5

28. (c) Initial Capital = `2,100. A’s capital continues for 12 ⇒ 56 : 16 : 14.4. Therefore, B’ share

months. 16
= × 21600 = `4,000
B’s capital continues (12 – 4) = 8 months. Let B puts 86.4
in ` x
35.
(b) Let the total profit be `x
⇒ 2100 × 12 = x × 8
⇒ x = 2100 × 12/8 = `3,150  3x   7x 
Amount of profit = `  x   = `  
 10   10 
1 2
29.
(b) Ratio of capital = : = 1:2 Ratio of capitals = 5000:6000:4000 or 5:6:4
3 3
3 2 ∴ A’s share = `   7x  5   3 x 
Ratio of profit = = 3:2
5 5   10 15  10 

Chapter_08.indd 189 1/30/2016 3:10:11 PM


190  Chapter 8

Similarly, rent paid by


 8x 
= `   22  4  60
 15  A =  = `275
32  6
7x 6  7x
B’s share = `   = `   ∴ Total rent = 400 + 600 + 275 = `1,275
 10 15   25 
38.
(a)
7x 4  14 x 
C’s share = `   = `  A B C
 10 15   75 
x  12 : 2x  6 : 3x  4
7 x 14 x 8x  12 x : 12 x : 12 x
∴   200 = or x = 3000
25 75 15 \ 3x = 27000
Thus, the total profit is `3,000 ⇒ x = `9000
36.
(d) Let the yearly total profit be `x 39.
(a) Ratio of the shares of investment of Gordon, John and
Amount paid to B as salary 2 1 1
Mona = 4:1:1 = : :
= `(120 × 12) = `1,440 3 6 6
 x – 1440  Let the capital be x.
Share of each = `  
 2 (7  4) x 3 x
Total profit increased = 
100 100
 22500  10 
Interest paid by B = `   = `2,250
 100 3x
\ John’s increased share =
Total money received by A 600
3x
= `  x  1440  2250  x  3060 
` \ = 60 million
 2   2  600
⇒ x = 12000 million
Total money recevied by B
12000
  x  1440   \ Capital of Mona = = $ 2000 million
= `    1440  2250  6
 2  
x  3060  40.
(b) Let x month be the period of investment of Sheesh
= `   Chew.
 2
\ Ratio of the profits of Amber Chew and Sheesh
1  x  3060   x  3060  20  12
Given:   =   Chew =
2 2 2 30  x
x  3060 x  3060 20  12 3
or = =x ⇒ =
4 2 30  x 2

or x = 9180 ⇒ x = 5.33 = 6 months
Hence, the total profit = `9,180
41.
(c) A’s profit + 1200 = 3900
37.
(c) A’s Monthly Equivalent Rent = 22 × 4
⇒ A’s profit = `2,700
B’s Monthly Equivalent Rent = 16 × 8
B’s profit = (80000/60000) × 2,700
C’s Monthly Equivalent Rent = 32 × 6
= (4/3) × 2,700
B’s Monthly Equivalent Rent
= `3,600
C ’s Monthly Equivalent Rent
42. (a) Let, A × 6 = B × 4 = C × 8 = l
Rent paid by B
= λ λ λ
Rent paid by C So, A = ,B= ,C=
6 4 8
16  8
∴ = Rent paid by B Amount ratio among them
32  6 600 λ λ λ
= : :
16  8  600 6 4 8
∴ Rent paid by B =  = `400
32  6 = 4:6:3

Chapter_08.indd 190 1/30/2016 3:10:12 PM


191
Partnership 

4 47. (b) Let the score of Ajay = x


Hence, A’s share = × `1950 Rahul = x – 15
(4 + 6 + 3)
Manish = x – 25
4 According to question, x = 63 + 30
= × `1950
13 \ x = 93
= `600 \ Score of Ajay = 93
43. (b) Investment ratio among A, B and C then Rahul = 93 – 15 = 78
= 28000:32000:18000 then Manish = 93 – 25 = 68
= 14:16:9 Total marks of Rahul, Manish and Suresh
Suppose total profit = `x = 3 × 63 = 189
1 x \ Suresh = 189 – (78 + 68) = 43
A’s profit for his services = `x × =`
4 4 \ Manish + Suresh = 68 + 43 = 111

x 3x 48. (d) Ratio of investments of A, B and C


Remaining profit = x –
=` = 12000 × 24 : 16000 × 24 : 15000 × 16
4 4
A’s profit according to his investment = 18 : 24 : 15
3x 14 15
= ` × So share of C = × 45600 = `12,000
4 (14 + 16 + 9) 57

3 x 14 49. (a) Sumant : Maurya


= ` × 12 × 48000 : 6 × 56000
4 39
12 × 48 : 6 × 56
7x
⇒ =` 576 : 336
26
12 : 7
 x 7x 
Then,  +  = `4995
\ Share of profit of Sumant
 4 26 
12
13 x + 14 x = × 24529
⇒ = `4995 19
52
4995 = 12 × 1291 = `15492
⇒ x = 52 × = `9620
27 Share of profit of Maurya = 24529 – 15492 = `9037
3 x 16 Difference between the share of profit of Sumant and
Hence, B’s profit = ` × Maurya = 15492 – 9037 = `6455
4 39
3 × 9620 16 50.
(d)
= ` × = `2960
4 39 51. (c) Let the initial investments of A, B and C be 3x, 5x and
7x respectively. Investment after one year.
44. (d)
Firoz Dhruv = 3x – 45600, 5x, 7x + 337600

650000 × 24 : 800000 × 18 Given 3x – 45600 : 5x : 7x + 337600
13 : 12 = 24 : 59 : 167
or x = `47,200, therefore, investment of A = 3
13
\ Share of Firoz = × 435000 × 47200 = `1,41,600
25
52. (c) Six people contribute a total of `180. Let the seventh
= `226200
person contributes `x
45. (d) Let they invest money for x, y and z months Eighth person contributes `55
then 5x : 6y : 8z = 5 : 3 : 1 Total contributions of these eight persons = 235 + x
1 1 235  x
or, x : y : z = 1: :  = 8 : 4 : 1
2 8 Now x =  10
8
46. (b) Rate in which the rent is to be divided = 4 : 8 : 12 7 1
⇒ x  (235) + 10 ⇒ x = 45
8 8 8
∴ Akbar’s share of rent =  6000  = `2,000
24 ∴ Total collection = 235 + 45 = `280

Chapter_08.indd 191 1/30/2016 3:10:13 PM


9 Profit and Loss

INTRODUCTION
Business transactions have now-a-days become common Loss If the cost price of an article is greater than the selling
feature of life. When a person deals in the purchase and sale price, the seller suffers a loss.
of any item, he either gains or loses some amount generally.
Thus, Loss = C.P. – S.P.
The aim of any business is to earn profit. The commonly
used terms in dealing with questions involving sale and Note that profit and loss are always calculated with
purchase are: respect to the cost price of the item.
Cost price The cost price of an article is the price at which Illustration 1
an article has been purchased. It is abbreviated as C.P.
(i) If C.P. = `235, S.P. = `240, then profit = ?
Selling price The selling price of an article is the price at
(ii) If C.P. = `116, S.P. = `107, then loss = ?
which an article has been sold. It is abbreviated as S.P.
Profi t or gain If the selling price of an article is more than Solution:
the cost price, there is a gain or profit. (i) Profit = S.P. – C.P. = 240 –235 = `5.
Thus, Profit or gain = S.P. – C.P. (ii) Loss = C.P. – S.P. = 116 – 107 = `9.

SOME BASIC FORMULAE

Illustration 3 Anu bought a necklace for `750 and sold it


1. Gain on `100 is gain per cent
for `675. Find her percentage loss.
Gain × 100 Solution: Here C.P. = `750, S.P. = `675.
Gain% =
C.P. Loss = C.P. – S.P. = 750 – 675 = `75.
Loss on `100 is loss per cent Loss × 100
\ Loss% =
Loss × 100 C.P.
Loss% =
C.P. 75 × 100
= = 10%.
750
Illustration 2 The cost price of a shirt is `200
200 and selling
price is `250. Calculate the % profit. 2. When the selling price and gain pre cent are given:
Solution: We have C.P. = `200, S.P. = `250.  100 
C.P. =   × S.P.
Profit = S.P. – C.P = 250 – 200 = `50.  100 + Gain% 
Profit × 100 3. When the cost and gain per cent are given:
\ Profit% =
C.P.
 100 + Gain% 
50 × 100 S.P. =   × C.P.
= = 25%.  100 
200

Chapter_09.indd 192 1/30/2016 3:50:22 PM


Profit and Loss 193

Explanation Illustration 4 Mr. Sharma buys a cooler for `4500. For how
Profit × 100 much should he sell so that there is a gain of 8%?
Since Profit% =
C.P. Solution: We have C.P. = `4500, gain% = 8%.
 (S.P. − C.P.) × 100   100 + Gain% 
=  \ S.P. =   × C.P.
 C.P.  100 
Profit% S.P.  100 + 8 
\ = –1 =
100 C.P.  × 4500
 100 
S.P. Profit%
or, =1+ 108
C.P. 100 = × 4500
100
 100 + Profit% 
\ S.P. =   × C.P. = `4860.
 100 
Illustration 5 By selling a fridge for `7200, Pankaj loses
 100 
and C.P. =   × S.P. 10%. Find the cost price of the fridge.
 100 + Profit% 
Solution: We have, S.P. = `7200, loss = 10%.
4. When the cost and loss per cent are given:
 100 
\ C.P. =   × S.P.
 100 − Loss%   100 − Loss% 
S.P. =   × C.P.
 100 
 100 
5. When the selling price and loss per cent are given: =  × 7200
 100 − 10 
 100 
C.P. =   × S.P. 100
 100 − Loss%  = × 7200
90

Explanation = `8000.
Loss × 100 Illustration 6 By selling a pen for `99, Mohan gains
Since Loss% =
C.P.
1
12 %. Find the cost price of the pen.
 (C.P. − S.P.) × 100  2
= 
 C.P. 1 25
Solution: Here S.P. = `99, gain% = 12 % or %.
Loss% S.P. 2 2
\ =1–
100 C.P.  100 
\ C.P. =   × S.P.
or
S.P.
=1–
Loss%  100 + Gain% 
C.P. 100
 100 
= × 99
 100 − Loss%  25 
\ S.P. =   × C.P.  100 + 
 100   2 

100  100 × 2 
and C.P. =   × S.P.
 =  × 99
 100 − Loss%   225 
= `88.

Chapter_09.indd 193 1/30/2016 3:50:23 PM


194 Chapter 9

SOME USEFUL SHORT-CUT METHODS

1. If a man buys x items for `y and sells z items for  xw 


% profit =  − 1 × 100%
`w, then the gain or loss per cent made by him is  zy 
 xw   11 × 11 
 − 1 × 100% =  − 1 × 100%
 zy   10 × 10 
Explanation 21
= × 100% = 21%.
S.P. of z items = `w 100
w Illustration 8 A fruit seller buys apples at the rate of `12
S.P. of x items = ` x
z per dozen and sells them at the rate of 15 for `12. Find his
w percentage gain or loss.
Net profit = x− y. Solution:
z
w Quantity Price
x− y 12 12
∴ % Profit = z × 100%
y
 xw  15 12
i.e.,  − 1 × 100% ,
 zy   xw 
% gain or loss =  − 1 × 100%
which represents loss, if the result is negative.  zy 
Note: In the case of gain per cent the result obtained bears  12 × 12 
positive sign whereas in the case of loss per cent the result =  − 1 × 100%
 15 × 12 
obtained bears negative sign.
36
How to remember: =– × 100% = –20%.
180
Quantity Price (C.P. or S.P.)
Since the sign is –ve, there is a loss of 20%.
x y
2. If the cost price of m articles is equal to the
z w selling price of n articles, then
m−n
1. Cross-multiply the numbers connected by the arrows % gain or loss =   × 100
(xw and zy)  n 
2. Mark the directon of the arrows for cross- [If m > n, it is % gain and if m < n, it is % loss]
multiplicaton. The arrow going down forms the
numerator while the arrow going up forms the Explanation
 xw  Let the C.P. of one article be `1
denominator  .
 zy  ∴ C.P. of m articles = `m × 1 = `m
 S.P. of n articles = `m
Illustration 7 If 11 oranges are bought for `10 and sold at
m
10 for `11, what is the gain or loss%? ∴ S.P. of 1 article = `
n
Solution:
m  m−n
Quantity Price ∴ Profit on 1 article = `  − 1 i.e., `  
 n   n 
11 10
m − n 100 m−n
∴ % profit = × i.e.,   × 100 .
10 11 n 1  n 

Chapter_09.indd 194 1/30/2016 3:51:28 PM


Profit and Loss 195

Illustration 9 A shopkeeper professes to sell his goods on


cost price but uses 800 gm, instead of 1kg. What is his gain 4. If ‘A’ sells an article to ‘B’ at a gain/loss of m%
%? and ‘B’ sells it to ‘C’ at a gain/loss of n% If ‘C’
pays `z for it to ‘B’ then the cost price for ‘A’ is
Solution: Here cost price of 1000 gm is equal to selling
price of 800 gm  1002 z 
 
m−n  (100 + m) (100 + n) 
∴ % gain =   × 100
 n  where m or n is –ve, of it indicates a loss, otherwise
 1000 − 800  it is +ve.
=   × 100
 800 
Illustration 12 Mohit sells a bicycle to Rohit at a gain
200
= × 100 = 25%. of 10% and Rohit again sells it to Jyoti at a profit of 5%.
800 If Jyoti pays `462 to Rohit, what is the cost price of the
Illustration 10 If the selling price of 12 articles is equal to bicycle for Mohit?
the cost price of 18 articles, what is the profit %? Solution: Here m = 10, n = 5, z = `462.
Solution: Here m = 18, n = 12 Using the formula,
m−n  18 − 12   1002 z 
∴ Profit% =   × 100 =   × 100 C.P. =  ,
 n   12   (100 + m) (100 + n) 
6
= × 100 = 50%.  1002 × 462 
12 we get C.P. for Mohit =  
 (100 + 10) (100 + 5) 
3. If an article is sold at a price S.P.1, then % gain or
% loss is x and if it is sold at a price S.P.2, then % 462 × 10000
= = `400.
gain or % loss is y. If the cost price of the article is 110 × 105
C.P., then
Illustration 13 ‘A’ sells a DVD to ‘B’ at a gain of 17% and
S.P1 S.P2 C.P. S.P1 − S.P2 ‘B’ again sells it to ‘C’ at a loss of 25%. If ‘C’ pays `1053 to
= = = ,
100 + x 100 + y 100 x− y ‘B’, what is the cost price of the DVD to ‘A’?
where x or y is –ve, if it indicates a loss, otherwise Solution: We have, m = 17, n = –25, z = `1053.
it is +ve. ∴ Cost price of DVD to ‘A’
Illustration 11 By selling a radio for `1536, Suresh lost 20%.  1002 z 
What per cent shall he gain or lose by selling it for `2000? =  
 (100 + m) (100 + n) 
Solution: Here S.P1 = 1536, x = – 20 100 × 100 × 1053
(–ve sign indicates loss) =
(100 + 17) (100 − 25)
S.P2 = `2000, y = ?
100 × 100 × 1053
Using the formula, = = `1200.
117 × 75
S.P1 S.P2
=
100 + x 100 + y 5. If ‘A’ sells an article to ‘B’ at a gain/loss of m%
1536 2000 and ‘B’ sells it to ‘C’ at a gain/loss of n%, then the
we get = resultant profit/loss per cent is given by
100 − 20 100 + y
2000 × 80 1  mn 
⇒ 100 + y = = 104 m + n +  ...(1)
1536 6  100 
1 where m or n is –ve, if it indicates a loss, otherwise
⇒ y = 4 %. it is +ve.
6
1 Note: The expression given by (1) represents resultant
Thus, Suresh has a gain of 4 % by selling it for
6 profit or loss accordingly as it is +ve or –ve.
`2000.

Chapter_09.indd 195 1/30/2016 3:51:29 PM


196 Chapter 9

Illustration 14 ‘A’ sells a horse to ‘B’ at a profit of 5% and Since gain/loss of x% is made on the first, cost price of the
‘B’ sells it to ‘C’ at a profit of 10%. Find the resultant profit first article
per cent.  100 
= `z  .
Solution: We have, m = 5 and n = 10  100 + x 
 mn  Also, gain/loss of y% is made on the second, therefore
\ Resultant profit% =  m + n + 
 100  cost price of the second article
 5 × 10   100 
=  5 + 10 +  = `z  .
 100   100 + y
31 1  100   100 
= % or 15 %. \ Total C.P. = z   + z 
2 2  100 + x   100 + y 
Illustration 15 Manoj sells a shirt to Yogesh at a profit of 100 (100 + y ) + 100 (100 + x) 
15% and Yogesh sells it to Suresh at a loss of 10%. Find the =z  
 (100 + x) (100 + y ) 
resultant profit or loss.
Total S.P. = 2z.
Solution: Here m = 15, n = –10 S.P. − C.P.
\ Overall % gain or loss = × 100
 mn  C.P.
\ Resultant profit/loss%=  m + n + 
 100  100 z [100 + x + 100 + y ]
2z −
 15 × − 10  (100 + x)(100 + y )
= 15 − 10 + = × 100
 100 z[100 + x + 100 + y ]
 100 
(100 + x)(100 + y )
 150 
= 15 − 10 −  2(100 + x)(100 + y ) − 100(200 + x + y )
 100  = × 100
100(200 + x + y )
1
= 7/2% or 3 %,
2 100 x + 100 y + 2 xy
= %
which represents profit as the sign is +ve. (100 + x) + (100 + y )

6. When two different articles are sold at the same  100( x + y ) + 2 xy 


=
 (100 + x) + (100 + y )  %.
selling price, getting gain/loss of x% on the first and  
gain/loss of y% on the second, then the overall% Note: In case y = –x, we have
gain or % loss in the transaction is given by
x2
 100( x + y ) + 2 xy  Overall% gain or loss = − %
 (100 + x) + (100 + y )  % 100
  Since the sign is –ve, there is always a loss.
The above expression represent overall gain or Illustration 16 Mahesh sold two scooters, each for `24000.
loss accordingly as its sign is +ve or –ve. If he makes 20% profit on the first and 15% loss on the
7. When two different articles are sold at the same second, what is his gain or loss per cent in the transaction?
selling price getting a gain of x% on the first and Solution: Here x = 20 and y = –15.
loss of x% on the second, then the overall% loss \ Over all gain/loss%
in the transaction is given by  100( x + y ) + 2 xy 
=  (100 + x) + (100 + y )  %
2
 x  
  %
 10  100(20 − 15) + 2 × 20 × − 15 
=
 (100 + 20) + (100 − 15)  %
Note that in such questions there is always a loss.  
100 20
= – % = – %,
Explanation 205 41
Let each article be sold at `z. which represents loss, being a –ve expression.

Chapter_09.indd 196 1/30/2016 3:50:25 PM


Profit and Loss 197

Illustration 17 Rajesh sold two horses for `990 each; Illustration 19 A dis-honest shopkeeper professes to sell
gaining 10% on the one and losing 10% on the other. Find his goods at cost price but he uses a weight of 800 g for the
his total gain or loss per cent. kg weight. Find his gain per cent.
Solution: Here x = 10. Solution: True measure = 1000 g
2 False measure = 800 g
 x
\ Overall loss% =   %
 10  Also, x = 0.
2 \ Overall gain% is given by
 10 
=   % = 1%. 100 + g True measure
 10  =
100 + x False measure
8. A merchant uses faulty measure and sells his 100 + g 1000
goods at gain/loss of x% The overall % gain/ ⇒ =
100 800
loss(g) is given by
1000 × 100
100 + g True measure ⇒ 100 + g =
= 800
100 + x Faulty measure
Note: If the merchant sells his goods at cost price, 1000
⇒ g= – 100 = 25%.
then x = 0. 8
9. A merchant uses y% less weight/length and sells Illustration 20 A shopkeeper sells the goods at 44% loss on
his goods at gain/loss of x% The overall % gain/ cost price but uses 30% less weight. What is his percentage
loss is given by profit or loss?
 y + x   Solution: Here x = –44 and y = 30.
  × 100  %
 100 − y    y+x 
Overall gain/loss% = 
\  × 100%
 100 − y 
Illustration 18 A dishonest shopkeeper professes to sell  30 − 44 
cloth at the cost price but he uses faulty metre rod. His = × 100  %
metre rod measures 95 cm only. Find his gain per cent.  100 − 30 

Solution: Here True measure = 100 cm  −14 


= × 100  % = –20%,
 70 
False measure = 95 cm.
which represents loss being a negative expression.
Since the shopkeeper sells the cloth at cost price,
\ x = 0. 10. A person buys two items for `A and sells one at a
\ Over all gain% is given by loss of l% and other at a gain of g% If each item
was sold at the same price, then
100 + g True measure (a) The cost price of the item sold at loss
=
100 + x Faulty measure A (100 + %gain)
=
100 + g 100 (100 − %loss) + (100 + % gain)
⇒ =
100 95 (b) The cost price of the item sold at gain
100 × 100 A (100 − % loss)
⇒ 100 + g = =
95 (100 − %loss) + (100 + % gain)
10000
⇒ g= – 100 Illustration 21 Ramesh buys two books for `410 and sells
95
one at a loss of 20% and the other at a gain of 25%. If both
5 the books are sold at the same price, find the cost price of
=5 %.
19 two books.

Chapter_09.indd 197 1/30/2016 3:50:25 PM


198 Chapter 9

Solution: Cost price of the book sold at a loss of 20% 100 (100 − l ) − (100 − l ) m
= `
410 (100 + 25) 100
=
(100 − 20) + (100 + 25) (100 − l ) ⋅ (100 − m)
= `
410 × 125 100
= = `250.
80 + 125 (100 − l ) (100 − m)
Third discount at n% on `
Cost price of the book sold at a profit of 25% 100
410 (100 − 20) 410 × 80 (100 − l ) (100 − m) n
= = =`
(100 − 20) + (100 + 25) 80 + 125 100 × 100
=
`160. \ S.P. after third discount
(100 − l ) (100 − m) (100 − l ) (100 − m) n
11. If two successive discounts on an article are =
` –
m% and n%, respectively, then a single discount 100 100 × 100
equivalent to the two successive discounts will be (100 − l ) (100 − m) (100 − n)
=
`
 mn  100 × 100
m + n − %
 100 
 (lm + ln + mn) lmn 
=  l + m + n − + 
Explanation  100 (100) 2 
Let the marked price of the article be `100. \ Single equivalent discount
\ S.P. after the first discount = `(100 – m) and  (lm + ln + mn) lmn 
(100 − m) × n  l + m + n −
=
100
+  %.
(100) 2 
discount at n% on `(100 – m) = ` . 
100
\ Single equivalent discount Illustration 22 Find a single discount equivalent to two
 (100 − m) × n  successive discounts of 10% and 20%.
= m +  %
 100 Solution: The equivalent single discount is given by

 100m + 100n − mn   10 × 20 
=
 % 10 + 20 −  % i.e., 28%.
 100   100 

 mn  Illustration 23 Find a single discount equivalent to three


m + n −
=  %. successive discounts of 10%, 20% and 30%.
 100 
Solution: The equivalent single discount is given by
12. If three successive discounts on an article are l%,
m% and n%, respectively, then a single discount  (10 × 20 + 10 × 30 + 20 × 30) 10 × 20 × 30 
10 + 20 + 30 − + %
equivalent to the three successive discounts will be  100 1002 
 (lm + ln + mn) lmn   6 496
l + m + n − + % i.e.,  60 − 11 +  % = % or 49.6%.
 100 1002   10  10

Explanation Illustration 24 Two shopkeepers sell machines at the same


list price. The first allows two successive discounts of 30%
Let the marked price of the article be `100
and 16% and the second 20% and 26%. Which discount
\ S.P. after the first discount = `(100 – l). series is more advantageous to the purchaser?
Second discount at m% on `(100 – l) Solution: A single discount equivalent to the two successive
(100 − l ) × m  30 × 16 
=` . discounts of 30% and 16% is  30 + 16 −
100 %
 100 
\ S.P. after second discount
(100 − l ) m  24  1
`(100 – l) –
= or,  46 −  % or 41 %
100  5  5

Chapter_09.indd 198 1/30/2016 3:50:26 PM


Profit and Loss 199

Also, a single discount equivalent to the two successive Illustration 25 A shopkeeper sold sarees at `266 each after
giving 5% discount on labelled price. Had he not given the
 20 × 26 
discounts of 20% and 26% is  20 + 26 − % discount, he would have earned a profit of 12% on the cost
 100  price. What was the cost price of each saree?
 26  4 Solution: We have, labelled price z = `266, discount
or,  46 −  % or 40 %.
 5  5 d = 5% and profit p = 12%.
Clearly, the discount series being offered by the first Using the formula
shopkeeper is more advantageous to the purchaser.
 1002 z 
13. A shopkeeper sells an item at `z after giving a C.P. =  
discount of d% on labelled price. Had he not given  (100 − d ) (100 + p ) 
the discount, he would have earned a profit of p% we get the cost price of each saree
on the cost price.
 100 × 100 × 266 
The cost price of each item is given by =
 (100 − 5) (100 + 12) 
 
 1002 z 
C.P. =   100 × 100 × 266
 (100 − d ) (100 + p )  = = `250.
95 × 112

MULTIPLE CHOICE QUESTIONS

1. The cost price of 20 articles is the same as the selling price 5. My friend collects antique stamps. She purchased two, but
of x articles. If the profit is 25%, then the value of x is found that she needed to raise money urgently. So she sold
them for `8000 each. On one she made 20% and on the
(a) 25 (b) 18
other she lost 20%. How much did she gain or lose in the
(c) 16 (d) 15 entire transaction?
[Based on MAT, 2004] (a) No loss/profit (b) `667 loss
2. The marked price for a pair of shoes is 60% above the cost (c) `667 profit (d) None of these
price. A shopkeeper sells the pair of shoes after giving a [Based on MAT, 2003]
discount of x%. Now he raises the marked price by x%
6. A trader wants 10% profit on the selling price of a product
and gives a discount of (x + 5)%. If the profit earned in the
whereas his expenses amount to 15% on sales. What
second case is twice that of the first case, find the value
should his rate of mark up be on an article of `9?
of x.
(a) 20% (b) 25%
(a) 20 (b) 25 100
(c) 10 (d) 15 (c) 30% (d) %
3
[Based on MAT, 2003]
3. A man sells an article at 5% profit. If he had bought it at
5% less and sold it for `1 less, he would have gained 10%. 7. On selling a pen at 5% loss and a book at 15% gain, Karim
The cost price of the article is gains `7. If he sells the pen at 5% gain and the book at
10% gain, then he gains `13. The actual price of the book
(a) `200 (b) `150
is
(c) `250 (d) `240
(a) `100 (b) `80
[Based on MAT, 2003]
(c) `10 (d) `400
4. A shopkeeper sold a TV set for `17,940, with a discount [Based on MAT, 2003]
of 8% and gained 19.6%. If no discount is allowed, then
8. By selling 33 metres of cloth, a shopkeeper gains the price
what will be his gain per cent?
of 11 metres of cloth. His gain per cent is
(a) 25% (b) 26.4% (a) 7% (b) 50%
(c) 24.8% (d) None of these (c) 20% (d) 22%
[Based on MAT, 2003] [Based on MAT, 2002]

Chapter_09.indd 199 1/30/2016 3:50:27 PM


200  Chapter 9

9. If 7% of the sale price of an article is equivalent to 8% of (a) `175 (b) `200


its cost price and 9% of its sale price exceeds 10% of its (c) `225 (d) `250
cost price by `1, then what is the cost price of the article? [Based on MAT, 2000]
(a) `400 (b) `350
17. A house costs C rupees. Later it was sold for a profit
(c) `300 (d) `280 of 25%. What is the capital gains tax if it is 50% of the
[Based on MAT, 2002] profit?
10. A cycle agent buys 30 bicycles, of which 8 are first grade (a) C/24 (b) C/8
and the rest are second grade, for `3150, Find at what (c) C/4 (d) C/2
price he must sell the first grade bicycles so that if he sells [Based on MAT, 2000]
the second grade bicycles at three quarters of the price, he
18. If selling price is doubled, the profit triples. Find the profit
may make a profit of 40% on his outlay?
per cent.
(a) `200 (b) `240
2
(c) `180 (d) `210 (a) 66 (b) 100
3
[Based on MAT, 2002]
1
11. The retail price of a water geyser is `1265. If the (c) 105 (d) 120
3
manufacturer gains 10%, the wholesale dealer gains 15% [Based on MAT, 2001]
and the retailer gains 25%, then the cost of the product is
19. A showroom owner sells a leather jacket for `X and claims
(a) `800 (b) `900
to make a profit of 10%. He plans to have a stall in the
(c) `700 (d) `600 trade fair and marks the same jacket at `2X. At the stall,
[Based on MAT, 2002] he allows a discount of 20%. What will be the percentage
12. If the cost of 12 pencils is equal to the selling price of 10 profit that he will make at the trade fair?
pencils, the profit per cent in the transaction is (a) 80% (b) 60%
2 (c) 76% (d) None of these
(a) 16 % (b) 18%
3 [Based on IIT Joint Man. Ent. Test, 2004]
(c) 20% (d) 25% 20. If a person makes a profit of 10% on one-fourth of the
[Based on MAT, 2001] quantity sold and a loss of 20% on the rest, then what is
his average per cent profit or loss?
13. Two motor cars were sold for `9,900 each, gaining 10%
on one and losing 10% on the other. The gain or loss per (a) 12.5% profit (b) 11.25% loss
cent in the whole transaction is (c) 11.75% profit (d) 12.5% loss
(a) Neither loss nor gain (b) 1% profit [Based on IIT Joint Man. Ent. Test, 2004]
100 21. Rawat sold a book at 10% loss. Had he sold it for `85
(c) % profit (d) 1% loss more, then he would have made a profit of 7%. What was
99
[Based on MAT, 2001] the cost price of the book?
(a) `500 (b) `850
14. There would be 10% loss if a toy is sold at `10.80 per
piece. At what price should it be sold to earn a profit of (c) `615 (d) `585
20%? [Based on IIT Joint Man. Ent. Test, 2004]
(a) `12 (b) `1296 22. The initial profit percentage for a shirt is 87.5%, which is
(c) `14.40 (d) None of these sold after giving a discount of 20%. Due to some reason
[Based on MAT, 2005]
the cost price of making a shirt increases by 25%. Now
a shirt is sold after raising the marked price by `500 and
1 giving a discount of 25%. If the percentage profit still
15. Ravi sells an article at a gain of 12 % . If he had sold it at
2 remains the same, find the new selling price of the shirt.
`22.50 more, he would have gained 25%. The cost price (a) `800 (b) `1,000
of the article is (c) `1,200 (d) `1,500
(a) `162 (b) `140
23. What is the percentage profit made by selling an umbrella
(c) `196 (d) `180 at a certain price, if by selling at two-thirds of that price,
[Based on MAT, 1999] there would be a loss of 10%?
16. A machine is sold at a profit of 10%. Had it been sold for (a) 25% (b) 30%
`40 less, there would have been a loss of 10%. What was (c) 35% (d) None of these
the cost price? [Based on Narsee Monjee Inst. of Man. Studies, 2003]

Chapter_09.indd 200 1/30/2016 3:50:27 PM


Profit and Loss  201

24. A manufacturer of a certain item can sell all he can (a) 200 – Z/2 (b) 50Z + Z2/4
produce at the selling price of `60 each. It costs him `40 (c) 150Z – Z2/4 (d) 200Z – Z2/2
in materials and labour to produce each item and he has
[Based on SCMHRD Ent. Exam., 2003]
overhead expenses of `3000 per week in order to operate
that plant. The number of units he should produce and sell 31. To make an article, it takes 40 h for a workman who is
in order to make a profit of at least `1000 per week is paid `1.80 per hour. The material, 20 per cent of which is
(a) 400 (b) 300 wasted in the course of working, costs `22.5 per kilogram.
The final weight of the article is 8 kg. At what price it
(c) 250 (d) 200 must be sold so as to yield a profit of 33.33 per cent?
[Based on FMS (Delhi), 2003]
(a) `360 (b) `404
25. Profit after selling an article for `425 is the same as the (c) `396 (d) `384
loss after selling it for `355. The cost of the article is
(a) `385 (b) `390 32. A shopkeeper sold an article for `6,750 after giving a
discount of 10% on the labelled price. He would have
(c) `395 (c) `400
earned a profit of 50%, had there been no discount. What
[Based on FMS (Delhi), 2003] was the actual percentage of profit earned?
26. A tradesman by means of false balance defrauds to the (a) 36 (b) 40
extent of 8% in buying goods and also defrauds to the
(c) 35 (d) None of these
extent of 8% in selling. His gain per cent is
[Based on IRMA, 2002]
(a) 16% (b) 15.48%
33. The evergreen shrubs at Ravi’s nursery are planted in rows
(c) 16.64% (d) 36%
on a square plot of land measuring 2,401 square ft. The
[Based on FMS (Delhi), 2003]
shrubs are planted in such a manner that the centres of the
27. If 7 kg of tea of price `72/kg is blended with 33 kg of tea shrubs are 7 ft apart and the outer shrubs are planted along
of `87/kg and 35 kg of `85/kg and the mixture is sold at the edges of the plot, with a shrub at each corner. Ravi
15% profit, then what is the selling price of the mixture? spent `896 to cover all the costs necessary for raising this
(a) `80.66/kg (b) `102.33/kg crop of the evergreen shrubs. If Ravi succeeds in selling
each shrub for `35, his profit will be what percentage of
(c) `91.22/kg (d) `97.37/kg his total cost?
[Based on IIFT, 2003]
(a) 100% (b) 50%
28. A property dealer sells a house for `6,30,000 and in (c) 125% (d) 150%
the bargain makes a profit of 5%. Had he sold it for
`5,00,000, then what percentage of loss or gain he would 34. A wealthy man bought two flats in a posh locality to have
have made? a fixed return. But a financial crunch at his home forced
(a) 15% loss (b) 15% gain him to sell both the flats soon, and he sold both at the
rate of `3,00,000 each. He thereby incurred a loss of 20%
2 2 on one of the flats and a gain of 20% on the other. What
(c) 16 % gain (d) 16 % loss
3 3 was the overall profit or loss he incurred in the entire
[Based on IIFT, 2003] transaction?
29. In a game show, each participation costs `150. The 1st (a) No gain, no loss (b) `25,000 profit
person to play wins `10, the second person wins `50, the (c) `25,000 loss (d) `10,000 loss
3rd person wins `100 and the 4th person wins `250. This
cycle is repeated with 5th person winning `10. After 83 35. Sita buys a fridge at 15/16 of its original value and sells it
people have played, how much profit has the game show for 10% more than its value. Then the gain % is
made for itself? (a) 15.55 (b) 11.67
(a) 4090 (b) 3990 (c) 16.67 (d) None of these
(c) 3900 (d) 8180 [Based on MAT, 2008]
[Based on SCMHRD En. Exam., 2003] 36. A cash payment that will settle a bill for 250 chairs are
30. A furniture store owner has determined that he can sell `50 per chair less 20% and 15% with a further discount of
100 chairs a month at a selling price of `200 each. For 5% on cash payment is
each rise of `4 in the selling price, he will sell 2 less chairs (a) `8075 (b) 7025
a month. If he sells the chairs for `Z each, then how much
(c) `8500 (d) None of these
money a month will he receive from the sale of chairs ?
[Based on MAT, 2008]

Chapter_09.indd 201 1/30/2016 3:50:27 PM


202  Chapter 9

37. A shopkeeper marked price of an article as 150% of cost (a) `48 (b) `48.6
price and he has two schemes (c) `49.2 (d) `49.8
(i) Scheme A: You can purchase one article at a discount 44. A sweet seller sells three-fifths part of sweets at a profit
of 20%. of 10% and remaining at a loss of 5%. If the total profit is
(ii) Scheme B: Purchase two articles at a discount of `1500, then what is the total cost price of sweets?
30% each. (a) `36,500 (b) `37,000
He sold 80% of articles under the scheme B and rest under (c) `37,500 (d) None of these
the scheme A. Then, his overall profit is 45. A sold an article to B at a profit of 20%. B sold the same
(a) 8% (b) 12% article to C at a loss of 25% and C sold the same article to
D at a profit of 40%. If D paid `252 for the article, then
(c) 16% (d) 6%
find how much did A pay for it?
38. A shopkeeper is giving 6 kg of rice at the price of `5 per (a) `175 (b) `200
kg. What should be the markup on cost price if he wants (c) `180 (d) `210
to make a profit of 20 per cent?
46. If the absolute difference between the selling price of the
(a) 25% (b) 50%
article when there is 15% loss and 15% gain in selling a
(c) 44% (d) 20% article is `450, then what is the cost price of the article?
39. The amount of wheat at the rate of `610 per quintal which (a) `1,200 (b) `1,500
should be added to 126 quintals of wheat costing `285 per (c) `2,000 (d) `2,200
quintal so that 20% may be gained by selling the mixture 47. On selling an article at successive discounts of 20% and
at `480 per quintal will be 25%, a dealer makes a net profit of 20%. Find the net profit
(a) 38 quintals (b) 49 quintals per cent if the dealer sells the same article at a discount of
(c) 69 quintals (d) None of the above 25%.
(a) 50% (b) 40%
40. Amit went to Mumbai and bought a pair of watches
costing `360 at 25% discount on each but on the way (c) 66.66% (d) 60%
back he loses one of these watches and had to buy them 48. If the selling price of a mat is five times the discount
(pair) again home. How much did he totally spend on the offered and if the percentage of discount is equal to the
watches? percentage profit, find the ratio of the discount offered to
(a) `620 (b) `720 the cost price.
(c) `540 (d) `630 (a) 11:30 (b) 1:5
(c) 1:6 (d) 7:30
41. The cost price of 16 apples is equal to the selling price
of 10 apples. The cost price of 12 oranges is equal to the 49. A sells his house to B at a profit of 10% who in turn sells it
selling price of 16 oranges and the cost price of 6 mangoes to C at a profit of 15% who in turn sells it to D at a profit of
is equal to the selling price of 4 mangoes. If the ratio of 25% and D sells it to E at 35% profit. If cost price of E’s
the cost price of 1 apple, 1 orange and 1 mango is in the house is `35,00,000, what is the approximate cost price of
ratio of 1:1:2, then find the net profit per cent on the sale A’s house?
of 1 apple, 2 oranges and 2 mangoes. (a) `15,40,000 (b) `15,10,000
(a) 25% (b) 30% (c) `15,00,000 (d) `16,40,000
(c) 35% (d) 40% 50. The market price of an article was 40% more than its cost
price. I was going to sell it at market price to a customer,
42. The market price of an article is `100. If it is sold at a but he showed me some defects in the article, due to
discount of 10%, a profit of 35% is made. How much loss which I gave him a discount of 28.57%. Next day he came
of profit will be made if it is sold for ` less then the market again and showed me some more defects, hence I gave
price? him another discount that was equal to 12.5% of the cost
(a) 5% loss (b) 8% gain price. What was the approximate loss to me?
(c) 5% gain (d) 8% loss (a) Loss of 10% (b) Loss of 12.5%
(c) Loss of 15% (d) None of these
43. The cost price of three varieties of apples namely A, B and
C is `20/kg, `40/kg and `50/kg. Find the selling price of 51. A shopkeeper buys a toy at `100 and sells it at `120.
one kg of apple in which these three varieties of apples are Another shopkeeper buys the same toy at `120 but sells it
mixed in the ratio of 2:3:5 such that there is a net profit of at `100. What are the respective profit/loss per cent for the
20%? two shopkeepers?

Chapter_09.indd 202 1/30/2016 3:50:27 PM


Profit and Loss  203

(a) 20%, 20% (b) 20%, 16.7% (a) 8.33% (b) 8.25%
(c) 16.7%, 16.7% (d) 16.7% 10% (c) 10% (d) 12.5%
52. A book vendor sold a book at a loss of 10%. Had he sold 60. A man buys 6 dozen eggs for `10.80, and 12 eggs are
it for `108 more, he would have earned a profit of 10%. found rotten and the rest are sold at 5 eggs per rupee. Find
Find the cost price of the book. his percentage gain or loss.
(a) `432 (b) `540 1 1
(a) 11 % gain (b) 11 % loss
(c) `648 (d) `740 9 9
1 1
53. Cost price of 12 oranges is equal to the selling price of (c) 9 % gain (d) 9 % loss
9 oranges and the discount on 10 oranges is equal to the 11 11
profit on 5 oranges. What is the percentage point difference 61. If an article is sold at 8% profit instead of 8% loss, it
between the profit percentage and discount percentage? would have brought `12 more. Find out the cost price of
(a) 20 (b) 22.22 the article.
(c) 16.66 (d) 15 (a) `75 (b) `72
(c) `60 (d) `70
54. If books bought at prices ranging from `200 to `350 are
sold at prices ranging from `300 to `425. What is the 62. 20% loss on selling price is what per cent loss on the cost
greatest possible profit that might be made in selling eight price?
books? 2
(a) 25% (b) 16 %
(a) `800 (b) `1,800 3
(c) `1,800 (d) None of these 1
(c) 15% (d) 16 %
55. A dishonest dealer marks up the price of his goods by 3
20% and gives a discount of 10% to the customer. He also [Based on SSC (GL), 2011]
uses a 900 g weight instead of 1 kilogram weight. Find his
profit percentage. 63. X sells two articles for `4,000 each with no loss and no
gain in the interaction. If one was sold at a gain of 25%
(a) 8% (b) 12% the other is sold at a loss of
(c) 20% (d) None of these 2
(a) 25% (b) 18 %
56. A businessman marked the price of his goods 30% more 9
1
than his C.P. He then sells th of his stock at a discount 2
4 (c) 16 % (d) 20%
3
of 15%, and half of the stock at the marked price, and the
rest at a discount of 30%. Find his gain percentage. [Based on SSC (GL), 2011]

(a) 16.5% (b) 15.375% 64. A man purchased some eggs at 3 for `5 and sold them at 5
(c) 14.20% (d) 13.37% for `12. Thus, he gained `143 in all. The number of eggs
he bought is
57. When a bicycle manufacturer reduced its selling price by (a) 210 (b) 200
50%, the number of bicycles sold radically increased by
600%. Initially the manufacturer was getting only 140% (c) 195 (d) 190
profit. What is the percentage increase of his profit? [Based on SSC (GL), 2011]
(a) 10% (b) 14% 65. The cost price of an article is 64% of the marked price.
(c) 0% (d) Cannot be determined The gain percentage after allowing a discount of 12% on
the marked price is
58. The marked price of a watch is `1,600. The shopkeeper
(a) 37.5% (b) 48%
gives successive discount of 10%, r% to the customer.
If the customer pays `1,224 for the watch, find the value (c) 50.5% (d) 52%
of r. [Based on SSC (GL), 2011]
(a) 10% (b) 20% 66. By selling an article for `144, a person gained such that
(c) 25% (d) 15% the percentage gain equals the cost price of the article.
The cost price of the article is
59. A trader sells goods to a customer at a profit of k%
(a) `90 (b) `80
over the cost price, besides it he cheats his customer by
giving 880g only instead of 1 kg. Thus his overall profit (c) `75 (d) `60
percentage is 25%. Find the value of k. [Based on SSC (GL), 2011]

Chapter_09.indd 203 1/30/2016 3:50:28 PM


204  Chapter 9

67. A man sells two article for `5000 each neither losing nor 74. A dishonest shopkeeper pretends to sell his goods at cost
gaining in the deal. If he sold one of them at a gain of 1
25%, the other article is sold at a loss of price but using false weights and gains 11 % . For a
9
2 2 weight of 1 kg he uses
(a) 15 % (b) 16 %
3 3 (a) a weight of 900 g (b) a weight of 950 g
1 1 (c) a weight of 875 g (d) None of these
(c) 17 % (d) 18 %
3 3 [Based on NMAT, 2006]
[Based on SSC (GL), 2011] 75. A farmer sold a cow and an ox for `800 and got a profit
68. A man bought orange at the rate of 8 for `34 and sold of 20% on the cow and 25% on the ox. If he sells the cow
them at the rate of 12 for `57. How many oranges should and the ox for `820 and gets a profit of 25% on the cow
be sold to earn a net profit of `45? and 20% on the ox, the individual cost price of the cow
(a) 90 (b) 100 and the ox is
(c) 135 (d) 150 (a) `515.60, `115.60 (approx.)
[Based on SSC (GL), 2011] (b) `531.50, `135.50 (approx.)
69. A shopkeeper allows 23% commission on his advertised (c) `530.60, `131.60 (approx.)
price and still makes a profit of 10%. If he gains `56 on (d) Cannot be determined
one item. His advertised price of the item, in `, is [Based on NMAT, 2005]
(a) 820 (b) 780 76. A man sells two horses for `1485. The cost price of the
(c) 790 (d) 800 first is equal to the selling price of the second. If the first
[Based on SSC (GL), 2011] is sold at 20% loss and the second at 25% gain, what is his
70. A shopkeeper earns a profit of 12% on selling a book at total gain or loss (in rupees)?
10% discount on the printed price. The ratio of the cost (a) `80 gain (b) `60 gain
price and the printed price of the book is (c) `60 loss (d) Neither gain nor loss
(a) 45:56 (b) 45:51 [Based on NMAT, 2005]
(c) 47:56 (d) 47:51
77. Vineet calculates his profit percentage on the selling price
[Based on SSC (GL), 2010]
whereas Roshan calculates his profit on the cost price.
71. A shopkeeper bought 30 kg of wheat at the rate of `45 They find that the difference of their profit is `275. If the
per kg. He sold 40% of the total quantity at the rate of selling price of of both them are the same and Vineet gets
`50 per kg. Approximately, at what price per kg would 25% profit and Roshan gets 15% profit, then find their
he sell the remaining quantity to make 25 per cent overall selling price.
profit? (a) `2100 (b) `2300
(a) `54 (b) `52 (c) `2350 (d) `2250
(c) `50 (d) `60 [Based on NMAT, 2005]
(e) `56 [Based on Gramin Bank U.P. (SO) Exam, 2012)]
78. Padam purchased 30 kg of rice at the rate of `17.50 per kg
72. Seema purchased an item for `9600 and sold it for loss of
and another 30 kg of rice at a certain rate. He mixed the
5 per cent. From that money she purchased another item
two and sold the entire quantity at the rate of `18.60 per
and sold it for gain of 5 per cent. What is her overall gain/
kg and made 20% overall profit. At what price per kg did
loss?
he purchase the lot of another 30 kg rice?
(a) Loss of `36 (b) Profit of `24
(a) `14.50 (b) `13.50
(c) Loss of `54 (d) None of these
(c) `12.50 (d) `15.50
[Based on Bank of Baroda PO Exam, 2011]
[Based on NMAT, 2005]
73. Ms. Priyanka sold two properties X and Y for `30000
each. She sold property X for 20% more than she paid for 79. A trader purchases a watch and a wall clock for `390.
it and sold property Y for 20% less than she paid for it. If He sells them marking a profit of 10% on the watch and
the expenses are disregarded, what was her gain or loss, if 15% on the wall clock. He earns a profit of `51.50. The
any, on the two properties? difference between the original prices of the wall clock
(a) Gain of `2500 (b) Loss of `2500 and the watch is equal to
(c) Gain of `1250 (a) `110 (b) `100
(d) There was neither a net gain or a net loss (c) `80 (d) `120
[Based on ATMA, 2005] [Based on NMAT, 2005]

Chapter_09.indd 204 1/30/2016 3:50:28 PM


Profit and Loss  205

80. Rehaan purchased a bike for `54000. He sold it at a loss 87. By selling an article at 80% of its marked price, a merchant
of 8 per cent. With that money he again purchased another makes a loss of 12%. What will be the per cent profit or
bike and sold it at a profit of 10 per cent. What is his loss made by the merchant if he sells the article at 95% of
overall loss/profit? its marked price?
(a) Loss of `657 (b) Profit of `567 (a) 5.5% profit (b) 1% loss
(c) Loss of `648 (d) Profit of `648 (c) 5% profit (d) 4.5% profit
[Based on Corporation Bank PO, 2011] [Based on MAT (May), 2010]

81. A retailer bought a certain number of CDs for `1800. 88. What is the maximum percentage discount that a merchant
Keeping one to himself, he sold the rest at a profit of `6 can offer on her marked price so that she ends up selling at
each. In total, he earned a profit of `114. The number of no profit or loss, if she had initially marked her goods up
CDs he bought is by 50%?
(a) 20 (b) 28 (a) 16.67% (b) 20%
(c) 50% (d) 33.33%
(c) 32 (d) 30
[Based on MAT (May), 2010]
[Based on MAT (Sept), 2010]
89. A tradesman gives 4% discount on the marked price and
82. A garment company declared 15% discount for wholesale 1 article free with every 15 articles bought and still gains
buyers. Mr Sachdev bought garments from the company 35%. The marked price is more than the cost price by
for `25000 after getting discount. He fixed up the selling
(a) 40% (b) 39%
price of garments in such a way that he earned a profit of
8% on original company price. What is the approximate (c) 20% (d) 50%
total selling price? [Based on MAT (Dec), 2009]

(a) `28000 (b) `29000 90. Even after reducing the marked price of a pen by `32, a
shopkeeper makes a profit of 15%. If the cost price be
(c) `31000 (d) `29500
`320, what percentage of profit does he make if he sells
[Based on MAT (Sept), 2010] the pen at the marked price?
83. A person bought two tables for `2200. He sells one at 5% (a) 25% (b) 20%
loss and the other at 6% profit and thus on the whole he (c) 10% (d) 30%
neither gains nor loses. Find the cost price of each table. [Based on MAT (Dec), 2009]
(a) `1500, `700 (b) `2000, `200 91. Tarun got 30% concession on the labelled price of an
(c) `1200, `1000 (d) `1100, `1100 article and sold it at `8750 with 25% profit on the price he
[Based on MAT (Sept), 2010, (Dec), 2009] bought. What was the labelled price?
84. A shopkeeper sold an article at a profit of 17.5%. If he had (a) `16000 (b) `12000
bought it at 8% less and sold it at 30% profit, he would (c) `10000 (d) `14000
have earned `11.55 more as profit. Cost price of the article [Based on MAT (Dec), 2009]
is 92. The retail price of a water geyser is `1265. If the
(a) `550 (b) `675 manufacturer gain 10%, the wholesale dealer gains 15%
(c) `750 (d) `1475 and the retailer gains 25%, then the cost of the product is
[Based on MAT (Sept), 2010] (a) `800 (b) `900
(c) `700 (d) `600
85. What profit/loss per cent did Ravi earn if he purchased an
[Based on MAT (Sept), 2009]
item of `5600 and sold it at three-fourths of its cost price?
(a) Loss of 20 per cent (b) Gain of 25 per cent 93. A machine is sold at a profit of 10%. Had it been sold for
`40 less, there would have been a loss of 10%. What was
(c) Neither gain nor loss (d) None of these
the cost price?
[Based on OBC PO, 2010]
(a) `175 (b) `200
86. A man invested `8000 for a year in the share market. (c) `225 (d) None of these
At the end of the year, he gained 15% and he invested the [Based on MAT (Sept), 2009, (Dec), 2000, (Sept), 1999]
amount with profit again for the second year. At the end of
the second year, he suffered a loss of 15%. Find the gain 94. What per cent profit would be if 34% of cost price is
or loss per cent in the investment after two years 26% of the selling price?
(a) Gain 3.25% (b) Loss 2.25% (a) 30.77% (b) 74%
(c) No loss no gain (d) Loss 5% (c) 25.16% (d) 88.40%
[Based on MAT (May), 2010] [Based on MAT (Sept), 2009]

Chapter_09.indd 205 1/30/2016 3:50:28 PM


206  Chapter 9

95. What per cent selling price would be 34% of cost price, if 102. A shopkeeper marks the prices of his goods at 25% higher
gross profit is 26% of the selling price? than the original price. After that, he allows a discount of
(a) 25.16% (b) 74.00% 12%. What profit or loss did he get?
(a) 15% profit (b) 10% profit
(c) 17.16% (d) 88.40%
(c) 10% loss (d) 15% loss
[Based on MAT (Sept), 2009]
[Based on MAT (Feb), 2009]
96. A student purchased a computer system and a colour
103. Ramesh purchased a bicycle for `5200 and spent `800 on
printer. If he sold the computer system at 10% loss and the
its repairs. He had to sell it for `5500. Find his profit or
colour printer at 20% gain, he would not lose anything.
loss per cent.
But if he sells the computer system at 5% gain and the
colour printer at 15% loss, he would lose `800 in the 1 1
(a) 7 % gain (b) 8 % loss
bargain. How much did he pay for the colour printer? 2 3
(a) `8000 (b) `16000 (c) 9% loss (d) None of these
(c) `9000 (d) `5334 [Based on MAT (Feb), 2009]

[Based on MAT (Sept), 2009, 2008] 104. A milkman buys milk contained in 10 vessels of equal
size. If he sells his milk at `5 a litre, he loses `200; if he
97. The profit earned after selling an article for `996 is the sells it at `6 a litre, he would gain `150 on the whole. Find
same as loss incurred after selling the article for `894. the number of litres contained in each vessel
What is the cost price of the article?
(a) 20 L (b) 30 L
(a) `935 (b) 905
(c) 25 L (d) 35 L
(c) `945 (d) `975 [Based on MAT (Dec), 2008]
(e) None of these 105. A man buys apples at a certain price per dozen and sells
[Based on SBI PO, 2008] them at eight times per hundred. What is his gain or loss
98. A man sells a book at a profit of 20%. If he had bought it per cent?
at 20% less and sold it for `18 less, he would have gained 1
(a) 4% loss (b) 8 % loss
25%. The cost price of the book is 4
(a) `80 (b) `70 1
(c) 4% gain (d) 6 % gain
(c) `60 (d) `90 4
[Based on MAT (May), 2009] [Based on MAT (Dec), 2008]

99. Two-third of a consignment was sold at a profit of 5% and 15


106. Sita buys a fridge at of its original value and sells it
the remainder at a loss of 2%. If the total profit was `400, 16
the value of the consignment was for 10% more than its value. Then, gain per cent is
(a) `15000 (b) `12000 (a) 15.55 (b) 11.67
(c) `10000 (d) `20000 (c) 16.67 (d) None of these
[Based on MAT (May), 2009] [Based on MAT (Feb), 2008]

100. Even after reducing the marked price of a transistor by 107. A trader has 50 kg of rice, a part of which he sells at 10%
`32, a shopkeeper makes a profit of 15%. If the cost price profit and the rest at 5% loss. He gain 7% on the whole.
be `320, what percentage of profit would he have made if How much was sold at 10% gain and how much was sold
he had sold the transistor at the marked price? at 5% loss?
(a) 40 kg and 15 kg (b) 30 kg and 10 kg
(a) 25% (b) 20%
(c) 35 kg and 40 kg (d) 40 kg and 10 kg
(c) 10% (d) None of these
[Based on MAT (Feb), 2008]
[Based on MAT (May), 2009]
108. Naresh purchased TV set of `11250 after getting discount
101. By selling 12 notebooks, the seller earns a profit equal to
of 10% on the labeled price. He spent `150 on transport
the selling price of 2 notebooks. What is his percentage
and `800 on installation. At what price should it be sold so
profit?
that the profit earned would have been 15% if no discount
(a) 25% (b) 20% was offered?
2 (a) `12937.50 (b) `14030
(c) 16 % (d) Data inadequate
3 (c) `13450 (d) `15467.50
[Based on MAT (Feb), 2009] [Based on United Bank of India PO, 2009]

Chapter_09.indd 206 1/30/2016 3:50:28 PM


Profit and Loss  207

109. A manufacturer of a certain item can sell all he can 116. A loss of 19% gets converted into a profit of 17% when
produce at the selling price of `60 each. It costs him the selling price is increased by `162. Find the cost price
`40 in materials and labour to produce each item and he of the article.
has overhead expenses of `3000 per week in order to (a) `450 (b) `600
operate that plant. The number of units he should produce (c) `360 (d) `540
and sell in order to make a profit of at least `1000 per
[Based on MAT, 1999]
week is
(a) 250 (b) 300 117. When a commission of 36% is given on the retail price,
profit is 8.8%. Find the profit when the commission is
(c) 400 (d) 200 decreased by 24%.
[Based on MAT (May), 2007]
(a) 76% (b) 54%
110. If the selling price of a product is increased by `162, then (c) 58% (d) 49.6%
the business would make a profit of 17% instead of a loss [Based on MAT, 1999]
of 19%. What is the cost price of the product?
118. A horse and a carriage together cost `8,000. If by selling
(a) `540 (b) `450
the horse at a profit of 10%, and the carriage at a loss of
(c) `360 (d) `600 10%; a total profit of 2.5% is made, then what is the cost
[Based on MAT (May), 2007] price of the horse?
111. A firm of readymade garments makes both men’s and (a) `3,000 (b) `3,500
women’s shirts. Its average profit is 6% of the sales. Its (c) `4,000 (d) `5,000
profit in men’s shirts average 8% of the sales and women’s [Based on MAT, 1999]
shirts comprise 60% of the output. The average profit per
shirt in women’s shirts is 119. A machine is sold at a profit of 10%. Had it been sold for
`80 less, there would have been a loss of 10%. The cost
(a) 0.0466 (b) 0.0666 price of the machine is
(c) 0.0166 (d) None of these (a) `350 (b) `400
[Based on MAT (May), 2006]
(c) `450 (d) `520
112. A trader charges 20% over the cost price. He allowed a [Based on MAT, 1999]
10% discount on account of Diwali on the listed price.
120. At what percentage above the cost price must an article
What is the net percentage gain?
be marked so as to gain 33% after allow­ing a customer a
(a) 15% (b) 12% discount of 5%?
(c) 8% (d) 10% (a) 48% (b) 43%
[Based on MAT, 1998] (c) 40% (d) 38%
113. A shopkeeper sells two radios at `1540 each. On one he [Based on MAT, 1999]
gains 12% and on the other he loses 12%. What was the 121. A man gains 10% by selling a certain article for a certain
net result of the sale of both the radios? price. If he sells it at double the price, then the profit made
(a) No loss no profit (b) Gain of `165 is
(c) Loss of `45 (d) Gain of `45 (a) 120% (b) 60%
[Based on MAT, 1998] (c) 100% (d) 80%
114. By selling 12 marbles for a rupee, a shopkeeper loses [Based on MAT, 1999]
20%. In order to gain 20% in the transaction, he should 122. The sale price of an article including the Sales Tax is
sell the marbles at the rate of how many marbles for a `616. The rate of Sales Tax is 10%. If the shopkeeper has
rupee? made a profit of 12%, find the cost price.
(a) 8 (b) 6 (a) `500 (b) `515
(c) 4 (d) 3 (c) `550 (d) `600
[Based on MAT, 1998] [Based on MAT, 1999]
115. In a certain store, the profit is 320% of the cost. If the cost 123. A shopkeeper sold an air-conditioner for `25935 at a
increases by 25% but the selling price remains constant, discount of 9% and earned a profit of 3.74%. What would
approximately what per cent age of the selling price is the have been the percentage of profit earned if no discount
profit? had been offered?
(a) 30% (b) 70% (a) 15.6% (b) 16%
(c) 100% (d) 250% (c) 12.3% (d) None of these
[Based on MAT, 1998]  [Based on IRMA, 2005]

Chapter_09.indd 207 1/30/2016 3:50:28 PM


208  Chapter 9

124. There would be 10% loss if rice is sold at `5.40 per kg. (a) 16785 (b) 36165
At what price per kg should it be sold to earn a profit of (c) 17165 (d) 28125
20%? [Based on JMET, 2011]
(a) `12 (b) `12.96
132. Pure Ghee costs `100 per kg. After adulterating it with
(c) `14.40 (d) `7.20 vegetable oil costing `50 per kg, a shopkeeper sells the
[Based on MAT, 2000] mixture at the rate of `96 per kg, thereby making a profit
125. If I purchased 11 books for `10 and sold all the books at of 20% . In what ratio does he mix the two?
the rate of 10 books for `11, the profit per cent is (a) 1:2 (b) 3:2
(a) 10% (b) 11% (c) 3:1 (d) None of these
(c) 21% (d) 100% [Based on FMS (MS), 2006]
[Based on MAT, 2000] 133. A fruit vendor professes to sell fruits at the cost price, but
126. A owns a house worth `10,000. He sells it to B at a profit uses false weights. He gains 30% in this manner. What
of 10% based on the worth of the house. B sells the house weight does he substitute for one kilogram?
back to A at a loss of 10%. In this transaction A gets 1
(a) 645 g (b) 750 g
(a) No profit no loss (b) Profit of `100 3
(c) Profit of `1,100 (d) Profit of `2,000 3
[Based on MAT, 2000] (c) 769 g (d) 800 g
13
127. A horse and a cow were sold for `12,000 each. The horse [Based on JMET, 2011]
was sold at a loss of 20% and the cow at a gain of 20%.
The entire transaction resulted in 134. A dishonest dealer sells his goods at the cost price and still
(a) No loss or gain (b) Loss of `1,000 earns a profit of 25% by under weighing. What weight
does he use for a kilogram?
(c) Gain of `1,000 (d) Gain of `2,000
(a) 750 g (b) 800 g
[Based on MAT, 2000]
(c) 825 g (d) 850 g
128. A machine is sold at a profit of 10%. Had it been
[Based on FMS, 2005]
sold for `40 less, there would have been a loss of 10%.
What was the cost price?
Directions (Q. 135-136): Based on the information given below.
(a) `175 (b) `200
(c) `225 (d) `250 Books and More sells books, music CDs and film DVDs. In
December 2009, they earned 40% profit in music CDs and 25%
[Based on MAT, 2000]
profit in books. Music CDs contributed 35% towards their total
129. A shopkeeper sells notebooks at the rate of `45 each and sales in rupees. At the same time total sales in rupees from books
earns a commission of 4%. He also sells pencil box at the is 50% more than that of music CDs.
rate of `80 each and earns a commission of 20%. How
135. If Books and More have earned 20% profit overall, then in
much amount of commission will he earn in two weeks if
film DVDs they made
he sells 10 notebooks and 6 pencil boxes a day?
(a) `1956 (b) `1586 (a) 15.2% profit (b) 10.0% profit
(c) `1496 (d) `1596 (c) 10.0% loss (d) 23.4% loss
[Based on XAT, 2010]
(e) None of these [Based on CBI (PO), 2010)]
130. A shopkeeper bought 30 kg of wheat at the rate of `45 per 136. If Books and More made 50% loss in film DVDs, then
kg. He sold forty per cent of the total quantity at the rate overall they made
of `50 per kg. Approximately, at what price per kg should (a) 12.3% profit (b) 8.7% profit
he sell the remaining quantity to make 25 per cent overall (c) 0.4% loss (d) 6.25% loss
profit ?
[Based on XAT, 2010]
(a) `54 (b) `52
(c) `50 (d) `60 137. A dealer sold a radio at a loss of 2.5%. Had he sold it for
[Based on Allahabad Bank PO, 2010] 1
`100 more, he would have gained 7 %. In order to gain
131. Ajay loses 20% of his money. After spending 80% of the 2
remainder, he is left with `4500. How much money (in 1
12 %, he should sell it for
rupees) did he have initially? 2

Chapter_09.indd 208 1/30/2016 3:50:29 PM


Profit and Loss  209

(a) `850 (b) `925 142. A Techno Company has 14 machines of equal efficiency
(c) `1080 (d) `1125 in its factory. The annual manufacturing costs are `42000
and establishment charges are `12000. The annual
[Based on FMS, 2006]
output of the company is `70000. The annual output
138. A space research company wants to sell its two products A and manufacturing costs are directly proportional to the
and B. If the product A is sold at 20% loss and the product number of machines. The shareholders get 12.5% profit,
B at 30% gain, the company will not lose anything. If the which is directly proportional to the annual output of the
product A is sold at 15% loss and the product B at 15% company. If 7.14% machines remain closed throughout
gain, the company will lose `6 million in the deal. What is the year, then the percentage decrease in the amount of
the cost of product B? profit of the shareholders would be
(a) `140 million (b) `120 million (a) 12% (b) 12.5%
(c) `100 million (d) `80 million (c) 13% (d) None of these
[Based on FMS, 2009] [Based on IIFT, 2010]

1 143. A small and medium enterprise imports two components A


139. A jobber buys an article at “`24 less 12 %". He then and B from Taiwan and China respectively and assembles
2
them with other components to form a toy. Component
1 A contributes to 10% of production cost. Component B
wishes to sell the article at a gain of 33 % of his cost
3 contributes to 20% of the production cost. Usually, the
after allowing a 20% discount on his marked price. At company sells this toy at 20% above the production cost.
what price, in rupees, should the article be marked? Due to increase in the raw material and labour cost in
(a) 30.00 (b) 33.60 both the countries, component A became 20% costlier
and component B became 40% costlier. Owing to these
(c) 40.00 (d) None of these reasons the company increased its selling price by 15%.
[Based on FMS, 2010] Considering that cost of other components does not
140. A farmer bought 749 sheep. He sold 700 of them for the change, what will be the profit percentage, if the toy is
price paid for the 749 sheep. The remaining 49 sheep were sold at the new price?
sold at the same price per head as the other 700. Based on (a) 15.5% (b) 25.5%
the cost, the per cent gain on the entire transaction is (c) 35.5% (d) 40%
(a) 6.5 (b) 6.75 [Based on IIFT, 2010]
(c) 7.0 (d) 7.5 144. A salesman sells two kinds of trousers—cotton and
[Based on FMS, 2010] woollen. A pair of cotton trousers is sold at 30% profit
141. Sumit works as a state contractor for PWD and supplies and a pair of woollen trousers is sold at 50% profit.
bitumen mix for road construction. He has two varieties The salesman has calculated that if he sells 100% more
of bitumen, one at `42 per kg and the other at `25 per kg. woollen trousers than cotton trousers, his overall profit
How many kg of first variety must Sumit mix with 25 kg will be 45%. However he ends up selling 50% more
of second variety, so that he may, on selling the mixture at cotton trousers than woollen trousers. What will be his
40 kg, gain 25% on the outlay? overall profit?
(a) 30 (b) 20 (a) 37.5% (b) 40%
(c) 25 (d) None of these (c) 41% (d) 42.33%
[Based on IIFT, 2007] [Based on XAT, 2009]

EXPLANATORY ANSWERS

1. (c) Let S.P. of x articles = `100 \ 6.25x = 100


= C.P. of 20 articles ⇒ x = 16.
\ C.P. of one article = `5 2. (b) Let the C.P. be `100.
Profit = 25%
\ S.P. of one article = 6.25   (100  x)  
Then, 2 160    100 
⇒ S.P. of x articles = 6.25x   100  

Chapter_09.indd 209 1/30/2016 3:50:29 PM


210  Chapter 9

6. (d) Let S.P. = `x


  (100  x)   (100  ( x  5))  
= 160     100  \ Expenses = 15% of x = 0.15x
  100   100  
Profit = 10% of x = 0.10x
Now, solve for x, we get x = 25 C.P. = `9 (Given)
3. (a) Let the C.P. of the article be `x. \ 9 + 0.15x + 0.1x = x
Profit = 5% ⇒ x = 12
21x \ The rate of mark up on the article should be
\ S.P. = x + 5% of x = `
20 100
%.
3
19 x
If C.P. would have been (x – 5% of x), i.e., ` and 7. (b) Let actual price of the book = `x
20
Let actual price of the pen = `y
 21x 
S.P. would have been `  − 1 , then gain % = 10 \ (x + 15% of x) + (y – 5% of y) = x + y + 7
 20 
⇒ 15x – 5y = 700 ...(1)
 21x  19 x Also (x + 10% of x) + (y + 5% of y) = x + y + 13
 − 1 −
\  20  20 × 100 = 10 ⇒ 10x + 5y = 1300 ...(2)
19 x Using (1) and (2), we get x = 80, y = 100
20 \ Actual price of the book = `80.
2 x − 20 8. (b) Suppose S.P. of 33 metres of cloth = `33
⇒ × 100 = 10
19 x \ Gain = `11 ⇒ C.P. = `22
⇒ 19x = 20x –­200 ⇒ Gain = 50%.
⇒ x = 200. 9. (b) Suppose C.P. = x and S.P. = y
4. (d) S.P. = `17940 ⇒ 7% of y = 8% of x
Discount = 8% and 9% of y = 10% of x + 1
17940 × 100 ⇒ 7y = 8x and 9y = 10x + 100
\ Marked Price = = `19500
100 − 8 8x
⇒ 9× = 10x + 100
Gain = 19.6% 7
17940 × 100 ⇒ x = 350.
C.P. = = `15000
100 + 19.6 10. (c) Suppose the price of first grade cycle = `x and the
If no discount is allowed on the Marked Price, then price of second grade cycle = `y.
S.P. = `19500 \ 8x + 22y = 3150
4500 Suppose he sells the first grade bicycles @ `z per
\ Gain % = × 100 = 30.
15000 bicycle
3z
8000 × 100 \ 8 z + 22 × = 3150 + 40% of 3150
5. (b) C.P. of one stamp = 4
100 + 20
i.e., 98z = 17640
8000 × 100 20000 ⇒ z = 180
= =
120 3 \ S.P. of the first grade bicycle = `180
8000 × 100 S.P. of the second grade bicycle = `135.
C.P. of another stamp =
100 − 20
11. (a) Suppose the cost = `x
8000 × 100
= = 10000 \ S.P. of the manufacturer
80
11x
= x + 10% of x =
50000 10
\ C.P. of both the stamps =
3 ⇒ S.P. of the wholesale dealer

S.P. of both the stamps =16000 11x 11x
= 15% of
2000 10 10
\ Loss =
3 11x 33 x 253
= + = x
= `666.67 ≈ `667. 10 200 200

Chapter_09.indd 210 1/30/2016 3:50:29 PM


Profit and Loss  211

⇒ S.P. of the retailer


C
253 253 If = 50% profit, then Capital Gains Tax = 50% of
= x + 25% of x 4
200 200 C C
= .
253 253 1265 4 8
= x+ x = x
200 800 800
18. (b) Let C.P. = `x
1265 \ S.P. = `y.
\ x = 1265
800 Profit = `(y – x)
⇒ x = 800. If S.P = 2y, then profit = 3(y – x)
12. (c) Let C.P. of 12 pencils = `12 \ 2y – x = 3(y – x)
\ S.P. of 10 pencils = `12 ⇒ y = 2k
C.P. of 10 pencils = `10 \ Profit on `x = `x; i.e., 100%.
\ Profit = 20%. 19. (c) S.P. at the stall at the trade fair
13. (d) C.P. of the 1st Motor Car 8x
= 2x – 20% of 2x =
9900 × 100 5
= = `9000
100 + 10 x × 100 10 x
C.P. of the jacket = =
C.P. of the 2nd Motor Car 100 + 10 11
9900 × 100 8 x 10 x 38 x
= = `11000 \ Profit =
− =
100 − 10 5 11 55
\ Total C.P. = `20000 \ Profit % made at the trade fair
Total S.P. = `19800 38 x
200 = 55 × 100
\ Loss % = × 100 = 1. 10 x
20000
11
14. (c) 90:10.80 = 120:x
38 1
90 120 = × × 100 = 76.
= 5 10
10.80 x
120 × 10.80 20. (d) Suppose C.P. = `100
\ x = \ Gain on `25 = `2.50
90
= 14.40. ⇒ S.P. = `27.50
1 Loss on `75 = `15
15. (d) 12 % = `22.50 ⇒ S.P. = `60
2
\ Total S.P. = `87.50
⇒ C.P. = `180.
⇒ Loss = `12.50.
16. (b) Let the C.P. be `k
11k 21. (a)  Let C.P. of the book = `x
\ S.P. = k + 10% of k = \ x – 10% of x + 85 = x + 7% of x
10
11k ⇒ 17% of x = 85
\ k – 10% of k = – 40 ⇒ x = 500.
10
9k 11k 22. (d) Let the C.P. be (d ) x
⇒ = − 40
10 10 Then, initial profit percentage
2k   15 x   4  

⇒ = 40   8   5   x 
10 =   × 100 ...(1)
⇒ k = 200. x
In the second case, profit percentage
17. (b) C.P. = `C
  15 x    3   5x  
C     500       
Profit = 25% of C = 8 4 4 
4 =    ...(2)
 5x 
C 5C  
S.P. = C + = 4
4 4

Chapter_09.indd 211 1/30/2016 3:50:30 PM


212  Chapter 9

Equate (1) and (2) to find the value of x as 800 If S.P. would have been `500000, then there would
And the new selling price 2
have been a loss of 16 %.
 5 3
=   × (1500 + 500)
 4
29. (a) Profit per cycle = `190
= `1,500 Total profit = 20 × 190 + 140 + 100 + 50
23. (c) Let C.P of the umbrella = `x = 3800 + 290 = `4090.
Let S.P. of the umbrella = `y 30. (d) If S.P. of each chair is `Z = 200 + 4K, then number of
2 chairs purchased
x− y
\ 3 × 100 = 10 2( Z − 200)
x = 100 – 2K = 100 –
4
3x − 2 y Z
⇒ × 100 = 10 = 100 − + 100
3x 2
 2 y Z
⇒ 1 − ×  × 100 = 10 = 200 −
 3 x 2
y \ Total money received from the sale of chairs
⇒ = 1.35
x Z Z2

= Z  200 −  = 200 Z − .
y−x  2 2
\ Profit % = × 100
x
31. (c) Workman’s wages = 1.8 × 40 = `72
y
= 100 − 100 Weight of material taken before working
x
= 135 – 100 = 35.  5
= 8 ×   =10 kg
24. (d) Suppose the manufacturer should produce x items  4
\ 60x – [40x + 3000] = 1000 Cost of 10 kg of material = `225
⇒ 20x = 4000 Therefore, C.P. of the material = 225 + 72 = `297
⇒ x = 200. Hence, S.P. = 1.33 × 297 = `396
25. (b) Suppose C.P. = `x
\ 425 – x = x – 355 32. (c) Marked Price = `x, say
⇒ 2x = 780 9x
S.P. = x – 10% of x =
⇒ x = 390. 10
9x
26. (c) The tradesman pays for 100 kg and 108 kg of goods \ = 6750
by means of false balance. 10
Actually, he sells 108 kg of goods, but due to false ⇒ x = 7500
balance, he sells 116.64 kg of goods. If S.P. = `7500, then the C.P. would have been `5000,
27. (d) C.P./kg of the mixture due to 50% profit earned by the shopkeeper .
7 × 72 + 33 × 87 + 35 × 85 \ Actual percentage of profit by selling the article
= for `6750
75
504 + 2871 + 2975 1750
= = × 100 = 35%.
75 5000
6350 33. (d) By finding the square root of 2401, you can determine
= = `84.66
75 that the plot of land measures 49 ft × 49 ft.
Profit = 15% With shrubs planted along the edges and at the corners
\ S.P. per kg = `84.66 + 15% of `84.66 of the plot, with 7 ft between each shrub, there is room
for 8 rows, each with 8 shrubs, for a total of 64 shrubs.
= `97.37.
So Ravi’s total selling price is 64 × `35 = `2,240
28. (d) S.P. = `630000 His profit equals the total selling price less the total
Profit = 5% cost be produce the shrubs. So Ravi’s profit will be
\ C.P. = `600000 `2240 – `896 = `1,344.

Chapter_09.indd 212 1/30/2016 3:50:31 PM


Profit and Loss  213

To write `1,344 (his profit) as a percentage of `896 40. (c) After the discount of `25% each, the cost of watches
(his total cost); you can write it first as a fraction or as is `270.
a decimal, and then multiply by 100. He has to buy them again, hence total cost of the
1344 watches is 270 + 270 = `540
= 1.5 or 150%
896 41. (b) Let the cost price of 1 apple be ‘x’, therefore the cost
price of 1 orange and 1 mango would be ‘x’ and ‘2x’
34. (c) C.P.1 = `3,75,000
respectively.
and C.P.2 = `2,50,000
⇒ Σ C.P. – Σ S.P. = 625000 – 600000  16 x 
Selling price of 1 apple =   1.6 x
 10 
= `25,000
35. (d) Let original value = x  12 x 
Selling price of 1 orange =   0.75 x
 16 
15 15
C.P. = ×x= x
16 16  12 x 
Selling price of 1 mango =   3x
10 11  4 
S.P. = x + ×x= x
100 10 Total cost price of 1 apple, 2 oranges and 2 mangoes
11 15 = x + 2x + 4x = 7x.
x− x
10 16 52 Total selling price of 1 apple, 2 oranges and 2 mangoes
gain% = × 100% = = 17.33%
15 3
x = 1.6x + 1.5x + 6x = 9.1x
16
Net Profit = 9.1x – 7x = 2.1x
 80  85  95   2.1x 
36. (a) `250 × 50     = `8075 Net profit per cent =  100  30%
 100  100  100   7 x 

37. (a) Let number of articles = 100 200


42. (c) 135% of C.P. = `90 C.P. =
Let his C.P. = `x 3
Then, he sold 80 articles under scheme B and 20 under Profit on selling it for `70
scheme A.  200  10
80  30  20  20 = `  70   
`
Hence, overall discount = = 28%  3 3
100
Therefore, selling price = 1.5x × 0.72 = 1.08x, where 10
Profit% =
x is his cost price. 3
Hence, net profit is 8%. 200  100 
5%
3
38. (c) Let x be the cost price.
∴ cost of 6 kgs of rice = 6x 43. (c) Cost price of one kg of apple in which the three
With 20% profit margin, selling price = 6x × 1.2 varieties of apples are mixed in the ratio 2:3:5 is
equal to S where S = 0.2 × 20 + 0.3 × 40 + 0.5 × 50
This is the price charged for 5 kg
= 4 + 12 + 25 = `41
6 x  1.2
∴ Selling price of 1 kg = = 1.44x Selling price per kg of apples to ensure there is a net
5 proift of 20% = 1.2 × 41 = `49.2
∴ Markup = 44%
44. (c) Assume A be the cost price.
39. (c) We have C.P. of mixture
3 10 2 5 
100 ∴   A   A  = 1500
=  S.P.  5 100 5 100 
100  %profit
or,  A = `37,500
100 45. (b) Let the article costs ‘x’ to A
=  480 = 400
120 Cost price of B = 1.2x
Quantity of cheaper/Quantity of dearer Cost price of C = 0.75(1.2x) = 0.9x
= (610 – 400)/(400 – 285) = 42:23 Cost price of D = 1.4(0.9x) = 1.26x = 252
If cheaper wheat is 42 quintals, dearer one = 23 quintals Amount paid by A for the article = `200
⇒ If cheaper wheat is 126 quintals, dearer one 46. (b) 115% of C.P. – 85% of C.P. = `450
= 23/42 × 126 = 69 quintals ∴ C.P. = `1,500

Chapter_09.indd 213 1/30/2016 3:50:32 PM


214  Chapter 9

47. (a) Let the cost price and market price of the article be ‘x’ 120  100
and ‘y’ respectivley. 51. (b) Percentage profit = × 100 = 20%
100
Case 1: Successive discounts of 20% and 25%
120  100
Selling price of the aritcle = (0.6)0.75y) = 0.6y Percentage loss = × 100
120
Therefore, 0.6y = 1.2x  or,  y = 2x
Case 2: A single discount of 25% 20
       = × 100 = 16.7%
Selling price of the article = 0.75y = 1.5x 120

 (1.5 x  x)  52. (b) Given: 108 = 10% loss + 10% profit


Net profit per cent =  100 = 50% = 20% extra
 ( x) 
Now, 20% = 108
48. (d) Since S.P. = 5 (M.P. – S.P.)
∴ 100% = 540
⇒ 5 M.P. = 6 S.P.
53. (b) C.P. : S.P.
6
⇒ M.P. = S.P. 3 : 4
5
Profit on 3 apples = Re 1 (consider C.P. = Re 1)
Since the percentage discount = Percentage profit, Profit = 33.33%
6 and discount = 11.11%
S .P. – S .P. S.P. − C.P.
5 × 100 = × 100 Since C.P. S.P. M.P.
6 C.P.
S .P. 3 4 4.5
5
1 S.P. 7 (1) (0.5)
∴ = – 1 ⇒ S.P. =  C.P. Profit is double that of discount
6 C.P. 6
So, the percentage point difference
6 6 7 7
M.P. = S.P. = × C.P. =  C.P. = 33.33% – 11.11% = 22.22% point
5 5 6 5
54. (b) Profit would be maximum if books are bought for
∴ Ratio of discount to C.P.

`200 and sold for `425.
M.P. – S.P.
= Profit = `(425 – 200) = `225
C.P.
Profit of 8 books = `225 × 8 = `1,800
7 7
C.P. – C.P. 7 55. (c) Let the C.P. of 1,000 gm of goods be `1,000
= 5 6 =
C.P. 30 Marked price = `1,200
Selling price = `1,200 × 0.9
49. (d) Let the cost price of A’s house = `x
= `1,080
∴ Cost price of E’s house
(after a discount of 10%)
= 1.1 × 1.15 × 1.25 × 1.35 × x
`1,080 is the selling price of 900 g of goods (as he
∴ 3500000 = 1.1 × 1.15 × 1. 25 × 1.35 × x
cheats to the extent of 10% while selling)
3500000
∴ x = C.P. of 900 g = `900
1.1  1.15  1.25  1.35
Profit = `180
3500000 180

≈ ≈ `16,47,059 ∴ Profit % =  100  = 20%
1.26  1.7 900
The nearest option is `16,40,000 56. (b) If C.P. = 100, M.P. = 130
50. (b) Let the cost price be `100 1 1 1
S.P. =  110.5   130   91
Then, market price is `140 4 2 4
2 S.P. = 27.625 + 65 + 22.75
Now, the first discount is of 28.57% ≈ th of market
7 = 115.375
price.
Hence, profit = 15.375%
5
Hence, its selling price = 140 × = `100 57. (c) Let the C.P. of a bicycle = `100
7
Now since you are selling at cost price, any further Now, since profit is 140%
discount will be equal to loss percentage. ∴ S.P. = `240

Chapter_09.indd 214 1/30/2016 3:50:32 PM


Profit and Loss  215

Now, 7 bicycles are being sold instead if 1 bicycle, but 63. (d) S.P. of both the articles is same here. So, profit on one
the sale price of new bicycle = `120 article is equal to the loss on other let the loss % be x
Therefore total sale price of new sale of bicycles then
= 7 × 120 = `840 and the C.P. = 7 × 100 = 700 25 x
⇒ 25 – x – = 0
So the new profit = 840 – 700 = `140 100
Since the initial profit is same as the new so there is 0 2500 − 100 x − 25 x
increase in percentage. ⇒ = 0
100
58. (d) Marked price of the article = `1,600 2500 − 125 x
⇒ = 0
∴ Selling price = (100 – 10)% of (100 – r%) 100
  of 1600 ⇒ 2500 – 125x = 0
90 100  r ⇒ –125x = –2500
   1600
100 100 − 2500 x
⇒ x =
9 − 125
⇒ 1224 =  (100  r )  16
10 ⇒ x = 20
1224  10
⇒ = (100 – r) 64. (c) Let the no. of eggs bought by him be 15
9  16
Therefore,
25 120  k (Profit) C.P of 15 eggs = `25
59. (c) Profit % =  = ⇒ k = 100
100 880 (Sale) So, S.P. of 15 eggs = `36
100 Hence, gain = 36 – 25 = `11
Therefore, net profit % =  100 = 10%
1000 Thus, 15 eggs = `11
15
60. (a) 6 dozens eggs cost = `10.80 = × 143 = `143
11
Since one dozen is rotten, he sells only 5 dozen at 5
eggs per rupee. = 195 eggs
Hence, S.P. = `12
65. (a) Let the marked price of article be `100.
(12  10.8) 1 Therefore,
this gain % =  100  = 11 %
10.8 9 C.P. of article = `64
61. (a) Let the C.P. of the article be x. So, S.P. of article = `88
1.08x – 0.92x = 12 88 − 64
Thus, profit % = × 100 = 37.5%
12 64
⇒ 0.16x = 12 ⇒ x = = `75
0.16 66. (b) let the C.P of the article be `x.
62. (b) Let the C.P. be `100. Then,
Let the S.P be `x then 144 − x
× 100 = x
100 − x x
× 100 = 20
x ⇒ (144 – x) × 100 = x2
2
⇒ 100(100 – x) = 20x ⇒ x + 100x – 14400 = 0
⇒ 10000 – 100x = 20x ⇒ x2 + 180x + 80x – 14400 = 0
⇒ 10000 = 20x + 100x ⇒ x(x + 180) – 80(x + 180) = 0
⇒ 10000 = 120x ⇒ (x – 80) (x + 180) = 0
10000 Therefore, x = `80
⇒ =x
120
100
250 67. (b) C.P of first article = 5000 ×
⇒ ` = x 125
3
= `4000
Hence, required loss% Then, loss on second article
250 50 2
= 100 − = = 16 % = `1000
3 3 3

Chapter_09.indd 215 1/30/2016 3:50:33 PM


216  Chapter 9

Therefore, C.P of second article 71. (d) C.P. of 30 kg of wheat


= `6000. = 30 × 45 = `1350
Let the loss per cent be x% then 40
of 30 kg = 12 kg
6000 × x 100
= 1000
100 S.P. of 12 kg of wheat
1000 × 100 = 12 × 50 = `600
x =
6000 S.P. of 30 kg of wheat
50 2 125
= = 16 % = 1350 ×
3 3 100
68. (a) Let the man buy (LCM of 8 and 12) oranges = `1687.50
Therefore, \ Rate of selling the remaining quantity (30 – 12) kg
34 1687.5 − 600
C.P. of 24 oranges = × 24 =
8 30 − 12
= 34 × 3 = `102 1087.5
57 =
S.P. of 24 oranges = × 21 18
12 = 60/kg
= 57 × 2 = `114 ; 60.4/kg (Approx.)
Gain = `114 – `102
72. (d) Cost price of the item = `9600
Gain = `12
Selling price of the item,
Thus, `12 = 24 oranges.
95 105
24 = 9600 × ×
Hence, `45 = × 45 = 90 oranges. 100 100
12
= `9576
69. (d) Let the advertised price be `x Hence, required loss
77 x = 9600 – 9576
Then, S.P = `
100 = `24
 77 x 
Therefore, C.P = `  − 56 
 100  73. (b) If selling price of 2 article is same and sold at x%
more and x% less, then there will be always loss
77 x − 5600 110 77 x
\ = = x 2 202
100 100 100 = = = 4%
100 100
77 x − 5600 77 x 7 x \ Gain, 96% = 60000
⇒ = =
100 110 10
Loss, 4% = `2500
⇒ 77x – 5600 = 70x
74. (a) Let gain = x
⇒ 77x – 70x = 5600
100 x
⇒ 7x = 5600 % = × 100%
9 1000 −x
5600
⇒ x = = `800 ⇒ x = 100
7
\ He uses 1000 – 100 = 900 g
70. (a) Let the printed price be `100
75. (c) Let C.P. cow be x and C.P. of ox be y.
Selling price = `90
100 120 125
Cost price = × 90 \ x× + y× = 800
112 100 100

100 × 90 1 125 120


Required ratio = × x× + y× = 820
112 100 100 100
On solving, we get x = `530.6
45
= = 45:56 and y = `131.14
56

Chapter_09.indd 216 1/30/2016 3:50:34 PM


Profit and Loss  217

76. (d) 1st horse 2nd horse 82. (c) Cost price of garments = `25000
C.P. = 100 25000
Original company price = × 100
S.P. = 100 85
S.P. = 80 \ Selling price of garments
100 × 100 25000 108
C.P. = = 80 = × 100 ×
125 85 100
Hence, total C.P. = Total S.P. = `31764.71 ≈ `31000
Hence, neither gain nor loss. 83. (c) Let the cost price of one table be x.
77. (b) Let the S.P. of Vineet and Roshan be x. Then, cost price of other table will be (2200 – x).

15 300 95 106
15% profit on C.P. = × 100 = % profit on x× + (2200 – x) × = 2200
115 23 100 100
S.P. ⇒ 95x + 233200 – 106x = 220000
⇒ 11x = 13200
x × 25 300 x ⇒ x = `1200
\ − = 275
100 23 × 100 and 2200 – x = `1000
On solving x = 2300 84. (a) Let cost price of article be `x.
78. (b) Cost price of 30 kg of 1st rice = 30 × 17.5 Then,

30 kg of 2nd rise = 30 × x 117.5 92 130


x× + 11.55 = x × ×
Total cost price = 30 × 17.5 + 30x 100 100 100
11.55
30 × 17.5 + 30 x 120 ⇒ x = × 100
\ × = 18.6 (0.92 × 130 − 117.5)
60 100
On solving, we get x = `13.5 = `550

79. (a) Let cost of the watch be `x. 85. (d) Cost price = 5600
3
\ 10% of x + 15% of (390 – x) = 51.5 Selling price = 5600 ×
4
On solving, we get,
= 4200
x = 140
Loss = 5600 – 4200 = 1400
Hence, cost of clock = 390 – 140
1400
= `250 % loss = × 100 = 25%
5600
\ Difference = 250 – 140
86. (b) Total amount after one year
= `110
115
= 8000 × = `9200
80. (d) Cost price = 54000 100
(100 − 8) 85
Selling price = 54000 × Total amount after 2nd year = 9200 × = 7820
100 100
= 49680 8000 − 7820
\ Loss per cent = × 100 = 2.25%
Now, the cost price of another bike = 49680 8000
110 Short cut method
Selling price of another bike = 49680 × = 54648
100 When a value is increased and then decreased by same
Overall profit = 54648 – 54000 = 648 percentage, then the value is always decreased and it
81. (a) Let he bought x number of CDs. x2
is decreased by %
\ (x – 1) × 6 = 114 100
⇒ x – 1 = 19 (15) 2
So, loss per cent = = 2.25%
⇒ x = 20 100

Chapter_09.indd 217 1/30/2016 3:50:34 PM


218  Chapter 9

87. (d) Let the cost price of article be `x. 93. (b) Let the cost price of the machine be `x.
Then, selling price of article = 0.88x Then,
Marked price of article 110 90
x× − 40 = x ×
0.88 100 100
= × 100 × x = 1.1x
80
100
New selling price of article = 1.045x ⇒ x = 40 × = `200
20
1.045 x − x
\ Profit per cent = × 100 =
4.5% 94. (a) Let cost price and selling price be x and y, respectively.
x
88. (d) Let cost price = `100 34 26
\ x = y
Marked price = `150 100 100
50 17
\ Discount per cent = × 100 = 33.33% ⇒ y = x
150 13
89. (d) Discount on articles
y−x
1 \ Profit per cent = × 100
= × 100 = 6.25% x
16
Overall discount 17
x−x
= – 4 – 6.25 +
4 × 6.25
= – 10% = 13 × 100 ≈ 30.77%
100 x
Let cost price = `100, then 95. (a) Let the selling price be `100, then profit = `26
Selling price = `135
\ Cost price = 100 – 26 = `74
So, 90% of marked price = 135
\ Required per cent = (34% of 74)%
135 × 100
Marked price = = `150 = 25.16%
90
96. (b) Let the cost price of colour printer and computer

Marked price is increased by
system be x and y respectively.
150 − 100
= × 100 = 50% 120 90
100 x×
+ y× =x+y
100 100
115 115
90. (a) S.P. = C.P. = × 320 = `368 ⇒ 0.2x = 0.1y ...(1)
100 100
And S.P. = MP – 32 85 105
x× + y× = x + y – 800
So, MP = S.P. + 32 = 368 + 32 = `400 100 100

400 − 320 ⇒ 0.05y = 0.15x – 800 …(2)


Per cent profit = × 100 = 25%
320 From Eqs. (1) and (2),
x = `16000
70
91. (c) S.P. = MP
700
97. (c) The cost price of an article
S.P. of Tarun = `8750
996 + 894
125 70 =
Labelled price = × MP = 8750 2
100 100
⇒ MP = `10000 1890
= = `945
92. (a) Let the cost of the product be `x. 2
Then,
98. (d) Let the cost price of book be `x.
110 115 125
x× × × = 1265 Then, (1.2x – 18) – 0.8x = 0.25 × 0.8x
100 100 100
100 100 100 0.4x – 18 = 0.20x
⇒ x = 1265 × × ×
110 115 125 18
⇒ x = = `90
= `800 0.20

Chapter_09.indd 218 1/30/2016 3:50:35 PM


Profit and Loss  219

99. (a) Let the value of consignment be x.


2 1 100 25
Then, x × 1.05 + x × 0.98 = x + 400 \ C.P. of 1 apple = =`
3 3 12 3

1 800
⇒ x (3.08) = x + 400 and S.P. of 1 apple = = `8
3 100

0.08  25  1
⇒ x = 400 Loss =   8 = `
 3  3
3
⇒ x = `15000 1
Loss
100. (a) Cost price of transistor = `320 Loss per cent = × 100 = 3 × 100
CP 25
Selling price of transistor 3
= 320 × 1.15 = `368 1 3
= × × 100
Marked price of transistor 3 25
= 368 + 32 = `400 = 4 %
\ Required percentage of profit 106. (d) Let the original value of fridge be `x.
400 − 320 15
= × 100 = 25% Then, cost price = x
320 16
101. (b) Let the selling price of a notebook be `x. 110
Selling price = ×x
Then, cost price of 12 notebooks 100
= 12x – 2x = 10x 110 15
x− x
2x \ Gain per cent = 100 16 × 100
\ Profit per cent = × 100 = 20% 15
10 x x
16
102. (b) Profit per cent or loss per cent
= 17.33%
25 × 12
= + 25 – 12 – = + 10% 107. (d) By the rule of allegation,
100
As the sign is + ve. So, there is a profit of 10%.
103. (b) Total cost price = 5200 + 800 = `6000
Selling price = `5500
6000 − 5500 1
\ Loss per cent = × 100 =
8 %
6000 3
104. (d) Let the quantity of milk in each vessel be x L. \ Quantity of rice sold at 10% gain
\ Quantity of milk in 10 vessels will be 10x L. 12
= × 50 =
40 kg
Selling price of 10x L = 10x × 5 = `50x 12 + 3
\ C.P. = `(50 x + 200) Quantity of rice sold at 50% loss
New selling price of 10x L 3
= × 50 =
10 kg
=10x × 6 = `60x 12 + 3
\ C.P. = `(60x – 150)
108. (b) Total cost price = 11250 + 150 + 800
Now, (50x + 200) = (60x – 150)
⇒ 10x = 350 = `12200
\ x = 35 L 115
Selling price = 12200 ×
100
105. (a) Let the cost price of 12 apples be `100.
Then, selling price of 100 apples will be `800. = `14030

Chapter_09.indd 219 1/30/2016 3:50:36 PM


220  Chapter 9

109. (d) Required number of items 1 100 5


C.P. = × =
(3000 + 1000) 12 80 48
=
(60 − 40) If gain = 20%, then S.P. per marble
4000 5 5 1
= = 200 = + 20% of =
20 48 48 8
110. (b) (17 + 19) = 36% of the cost price = `162
115. (b) Suppose C.P. = `100
162 Profit = `320
\ 100% of the cost price = × 100 = `450
36 \ S.P. = `420
111. (a) Women’s shirts comprise 60% of the output. If C.P. becomes `125, S.P. remaining the same, then
\ Men’s shirts comprise 40% of the output. profit = `295
\ Average profit from men’s shirts = 8% of 40 295
\ Profit % on S.P. = × 100 ≈ 70
= 3.2 out of 40 420
Overall average profit = 6 out of 100 116. (a) Let the cost price of the article be `x.
Average profit from women’s shirts = 2.8 out of 60,
i.e., 0.0466 out of each shirt. At 19% loss, selling price
112. (c) Suppose C.P. = `100  100 − 19 
= ` x  
Profit = 20%  100 
100 + 20 81x
\ Listed price = ` 100 × = `
100 100
120 Now, according to the question,
= ` 100 × = `120
100
Discount = 10%  81   117 
  x + 162 = x  
100 − 10  100   100 
\ Discounted price = 120 ×
100 117 81
or, x− x = 162
90 100 100
= `120 × = `108
100 36
or, x = 162
\ Gain = `108 – `100 100
= `8 on `100 = 8% 162 × 100
\ Gain = `108 – `100 – 8 on `100 \ x = = `450
36
= 8%.
117. (d) Suppose retail price = `100
113. (c) S.P. = `1540, Gain = 12% Commission = 36% = `36
SP × 100 \ S.P. = `64
C.P. =
100 + Gain% Profit = 8.8%
1540 × 100 Let C.P. = `K
= = `1375
112 \ K + 8.8% of K = 64
S.P. = `1540, loss = 12% 64 × 100
⇒ K = = 58.8
SP × 100 1540 × 100 108.8
C.P. = =
100 − Loss% 88 If commission = 12% = `12,
= `1750 S.P. becomes `88
\ S.P. of both the radios = `3080 29.2
\ Profit % = × 100 = 49.6
C.P. of both the radios = `3125 58.8
\ Net loss = `45 118. (d) Suppose C.P. of Horse = `x
1 \ C.P. of Carriage = `(8000 – x)
114. (a) S.P. of each marble = Re
12 \ 10% of x – 10% of (8000 – x)
Loss = 20% = 2.5% of 8000

Chapter_09.indd 220 1/30/2016 3:50:37 PM


Profit and Loss  221

⇒ 20% of x = 1000
⇒ x = 5000 28500 − 25000
\ Profit percentage = × 100
25000
119. (b) Let C.P. = `x
= 14%
11x
\ S.P. = x + 10% of x = 124. (d) S.P. of 1 kg of rice = `5.40
10
11x Loss = 10%
If S.P. = – 80, then loss would have been 10%.
10 100
\ C.P. of 1 kg of rice = `5.40 ×
11x 100 − 10
\ – 80 = x – 10% of x
10 100
= `5.40 × = `6
⇒ 11x – 800 = 9x 90
⇒ x = 400
In order to have a profit of 20%
120. (c) Let C.P. = `x
S.P. of 1 kg of rice should be
Marked Price = `y 100 × 20
= `6 ×
133 100
\ S.P. = x + 33% of x = x
100 120
= `6 × = `7.20
Now, according to the question, 100
133 125. (c) C.P. of 11 books = `10
y – 5% of y = x
100 S.P. of 10 books = `11
x y 11
⇒ = = K, say \ S.P. of 11 books = ` × 11 = `12.10
95 133 10
⇒ x = 95K, y = 133K \ Profit = `12.10 – `11 = `2.10
\ x + p% of x = y, say 2.10
Hence, profit % = × 100 = 21%
⇒ 95K + p% of 95K = 133K 10
p Quicker Method:
⇒ 95 + × 95 = 133
100
⇒ 95p = 13300 – 9500 = 3800
⇒ p = 40
121. (a) Let the C.P. be `100, then
11 × 11 − 10 × 10
S.P. = `110 % profit = × 100 = 21%
10 × 10
New S.P. = `220
\ Profit % = 220 – 100 = 120 126. (c) A’s Cost price of house = `10,000
122. (a) Let the C.P. be `x 100 + 10
\ A’s Selling Price = `10,000 ×
112 56 28 100
\ S.P. = x + 12% of x = = x = x x
100 50 25 110
= `10,000 × = `11,000
28 28 100
\ x + 10% of x = 616
25 25 or B’s Cost Price = `11,000
208 100 − 10
⇒ x = 616 B’s Selling Price = `11,000 ×
250 100

⇒ x = 500 90
= `11,000 × = `9,900
100
25935 × 100 = `9,900
123. (d) Market price = = 28500 = S.P.
91 or A’s second Cost Price = `9,900
25935 × 100 Hence A’s profit in this transaction
Cost price = = 25000
103.74 = `(11,000 – 9,900) = `1,100

Chapter_09.indd 221 1/30/2016 3:50:37 PM


222  Chapter 9

127. (b) Total S.P. of horse and cow = `12000 × 2 6 pencil boxes in a day then in two weeks
= `24,000 (i.e., 14 days) = 14 × 6
C.P. of horse, sold at loss of 20% = 84 pencil boxes
100 20
= `12000 × Commission earned = 80 × × 84 = `1344
100 − 20 100
100 Total commission earned = 252 + 1344 = `1596
= `12000 × = `15,000
80
130. (d) Cost price of 30 kg wheat = 30 × 45 = `1350
C.P. of cow, sold at gain of 20%
Cost price of 30 kg wheat + 25% profit = Selling Price
100
= `12000 × = 1350 × 1.25 = `1687.50
100 + 20 40% of 30 kg wheat = 30 × 0.40 = 12 kg
100 Selling price of 12 kg wheat
= `12000 × = `10000
120 = 12 × 50 = `600
Total C.P. of horse and cow Remaining 18 kg wheat’s selling price
= `15000 + `10000 = `25000 = 1687.50 – 600
Hence, total loss = `25000 – `24000 = `1087.50
= `1000 1087.50
\ Selling price of 1 kg wheat = = ` 60
Quicker Method: When each of the two commodities 18
is sold at the same price, and a profit of x% is made on
131. (d) Let the amount after the loss = `x
one and a loss of x% is made on the other, then there
is always loss and the percentage value is given as So, x – 80% of x = 4500
20% of x = 4500
( % value )2 4500  100
100 x = = `22500
20
(20) 2 This amount is after the 20% loss.
Here required x% loss = =4%
100 So, initial amount will be
= 80% of initial amount
 100 
Cost price = 12000 × 2 ×   = 22500
 100 − 4 
22500  100
100 Initial amount = = `28125
= 24000 × = `25000 80
96
132. (b)
\ loss = `25000 – `24000 = `1000
128. (b) Let the C.P. be `k
11k
\ S.P. = k + 10% of k =
10
Now, according to the question, 96  100
11k \ C.P. = = 80
k – 10 % of k = − 40 120
10 \ Ratio = 3:2
9k 11k 133. (c) Let the weight substituted for 1 kg = x g
or, = − 40
10 10
100  x
So,  100 = 30
2k x
or, = 40
10
3
\ k = 200 Solving x = 769 g
13
129. (d) 10 notebooks in a day then in two weeks (i.e., 14
days) = 14 × 10 = 140 notebooks x
134. (b) 25% = × 100%
4 1000  x
Commission earned = 45 × × 140
10 ⇒ x = 200
= `252 Hence, he uses weight, 1000 – 200 = 800 g

Chapter_09.indd 222 1/30/2016 3:50:38 PM


Profit and Loss  223

135. (d) Let total sales be `100. Now, find the respective sales Profit = 1000 + 40x – 625 – 42x
of Music CDs and Books and Rest will be DVDs. = 375 – 2x
Since, profit is given, cost price of CDs and Books can Required profit = 25%
be calculated.
375  2 x
Further from overall profit, overall cost price can be  100 = 25
625  42 x
calculated.
⇒ 1500 – 8x = 625 + 42x
136. (b) Apply same concept as in the previous question.
⇒ 875 = 50x
137. (d) Let C.P. of radio be x.
⇒ x = 17.5 kg
 1 1  142. (b) Original profit = 70000 – 42000 – 12000
Total difference = x 107 %  97 %
 2 2  =16000
= x × 10% If 7.14% of 14 i.e., one of the machines remain closed
x  10 throughout the year, then change in profit will be
\ = 100
100 13
\ x = 1000 = (70000  42000)
14
12.5% gain on `1000 = `1125 i.e., 26000 – 12000 = 14000
138. (d) Let cost of A = x and B = y Thus, the decrease in the profit %
4x 2000
S.P. of A = =  100 
12.5%
16000
5
143. (b) Let the price of the product be 100. Then, the prices of
13 y
and B = the components A and B will be 10 and 20 respectively.
10 As the profit is 20%, the selling price = 120.
4 x 13 y Due to increase in the price of raw material, the
Now, x + y =  (because company doesn’t
5 10 new costs of components A and B will be 12 and 28
respectively.
lose anything)
The new selling price = 115% of 120 = 138
x 3y
⇒ = As, there is no change in the price of the other
5 10
components, new cost of the product = 110
⇒ 2x = 3y
28
Now, use this relation further. Thus, the new profit % =  100 = 25.45%
110
139. (d) Cost price = 24 × (100 – 12.5)% = `21
144. (b) Let the cost of cotton trouser be x and woollen trouser
 1 be y.
Selling price = 21 × 100  33  = `28
 3 13 x
Selling price of cotton trouser be and woolen
28 10
Marked price = × 100 = `35
(100  20) 3y
trouser be .
2
140. (c) Let the price of one sheep = `1
Earlier salesman sells 100 cotton and 200 woollen
So, cost price of 749 sheep = `749 trousers.
Selling price of 700 sheep = `749 \ C.P. = 100x + 200y
Selling price of 1 sheep = `1.07 and S.P. = 130x + 300y
1.07  1 Since, salesman gains profit of 45%.
\ Per cent gain =  100 
7%
1 \ S.P. = (100x + 200y)
141. (d) Let the amount of first variety be x kg. 45
+ (100x + 200y) ×
Cost of first variety = 42x 100
= 145x + 290 y
Cost of second variety
Now, 130x + 300y = 145x + 290y
= 25 × 25 = 625
⇒ 15x = 10y
Total cost price = 625 + 42x
2y
S.P. of both variety = (25 + x) × 40 ⇒ x =
3
= 1000 + 40x Use this relation for further calculation.

Chapter_09.indd 223 1/30/2016 3:50:39 PM


10 Time, Work and Wages

INTRODUCTION
n
In our daily life, we come across situations where we need complete the full work, ‘A’ will take hrs.
m
to complete a particular job in a reasonable time. We have
to complete the project earlier or later depending upon the 4. If ‘A’ does three times faster work than ‘B’, then
needs. Accordingly, the men on duty have to be increased or ratio of work done by A and B is 3:1 and ratio of
decreased, i.e., the time allowed and the men engaged for a time taken by A and B is 1:3.
project are inversely proportional to each other, i.e., the more 5. A, B and C can do a piece of work in T1, T2 and T3
the number of men involved, the lesser is the time required to days, respectively. If they have worked for D1, D2
finish a job. We also come across situations where time and and D3 days, respectively, then
work or men and work are in direct proportion to each other.
D1
For solving problems on time and work, we follow the Amount of work done by A =
T1
following general rules:
D2
Amount of work done by B =
1. If ‘A’ can do a piece of work in n days, then at a T2
1 D3
uniform rate of working ‘A’ will finish th work and, Amount of work done by C =
n T3
in one day.
Also, the amount of work done by A, B and C
1 together
2. If of a work is done by ‘A’ in one day, then ‘A’
n
will take n days to complete the full work. D1 D2 D3
= + + .
1 T1 T2 T3
3. If ‘A’ does th of a work in one hour then to
n which will be equal to 1, if the work is complete.

SOME USEFUL SHORT-CUT METHODS

1. If A can do a piece of work in X days and B can 1


B’s 1 day’s work =
do the same work in Y days, then both of them Y
working together will do the same work in 1 1 X +Y
Then, (A + B)’s 1 day’s work = + =
XY X Y XY
days.
X +Y
∴ A and B together can complete the work in
Explanation
XY
1 = days.
A’s 1 day’s work = X +Y
X

Chapter_10.indd 224 2/1/2016 9:49:08 AM


Time, Work and Wages 225

Illustration 1 A can finish a piece of work by working alone Explanation


in 6 days and B, while working alone, can finish the same A and B together can complete the work in X days.
work in 12 days. If both of them work together, then in how
1
many days, the work will be finished? ∴ (A + B)’s 1 day’s work =
X
Solution: Here X = 6 and Y = 12.
1
∴ Working together, A and B will complete the work in Similarly, A’s 1 day’s work =
Y
XY 6 × 12
= days = days, i.e., 4 days. 1 1 Y−X
X +Y 6 + 12 Therefore, B’s 1 day’s work = – = .
X Y XY
2. If A, B and C, while working alone, can complete  XY 
∴ B alone can complete the work in   days.
a work in X, Y and Z days, respectively, then they
Y − X 
XYZ
will together complete the work in ∴ B alone will complete the work in
days. XY + YZ + ZX
XY 15 × 20
= days = , i.e., 60 days.
Y−X 20 − 15
Explanation
Illustration 3 A and B working together take 15 days to
1
A’s 1 day’s work = complete a piece of work. If A alone can do this work in 20
X days, how long would B take to complete the same work?
1
B’s 1 day’s work = Solution: Here X = 15 and Y = 20.
Y
1 4. If A and B, working together, can finish a piece of
C’s 1 day’s work =
Z work in X days, B and C in Y days, C and A in Z
∴ (A + B + C)’s 1 day’s work days, then
1 1 1 XY + YZ + ZX (a) A, B and C working together, will finish the
= + + = .
X Y Z XYZ job in
So, A, B and C together can complete the work in  2XYZ 
  days.
 XYZ   XY + YZ + ZX 
=   days.
 XY + YZ + ZX  (b) A alone will finish the job in
Illustration 2 A, B and C can complete a piece of work in  2XYZ 
  days.
10, 15 and 18 days, respectively. In how many days would  XY + YZ − ZX 
all of them complete the same work working together? (c) B alone will finish the job in
Solution: Here X = 10, Y = 15 and Z = 18.  2XYZ 
Therefore, the work will be completed in   days.
 YZ + ZX − XY 
XYZ
= days
XY + YZ + ZX
10 × 15 × 18 Explanation
= days
10 × 15 + 15 × 18 + 18 × 10 1
(A + B)’s 1 day’s work =
i.e.,
2700 1
or, 4 days. X
600 2 1
(B + C)’s 1 day’s work =
Y
3. Two persons A and B, working together, can
1
complete a piece of work in X days. If A, working (C + A)’s 1 day’s work = .
alone, can complete the work in Y days, then B, Z
XY So, [(A + B) + (B + C) + (C + A)]’s 1 day’s work =
working alone, will complete the work in 1 1 1
days. Y−X + + .
X Y Z

Chapter_10.indd 225 2/1/2016 9:51:02 AM


226 Chapter 10

 1 1 1 7200
or 2 (A + B + C)’s 1 day’s work =  + +  or , i.e., 20 days.
X Y Z 360

1 1 1 1 C alone can do the work in


or (A + B + C)’s 1 day’s work =  + +  2XYZ
2X Y Z = days
ZX + XY − YZ
 XY + YZ + ZX 
i.e.,   2 × 12 × 15 × 20
 2 XYZ  = days
20 × 12 + 12 × 15 − 15 × 20
∴ A, B and C working together will complete the work
7200
 2XYZ  or , i.e., 60 days.
in   days. 120
 XY + YZ + ZX 
Also, A’s 1 day’s work = (A + B + C)’s
)’s 1 day’s work 5. (a) If A can finish a work in X days and B is k
– (B + C)’s 1 day’s work times efficient than A, then the time taken by
1 1 1 1 1 both A and B working together to complete
=  + + − x
2X Y Z Y the work is .
1+ k
1 1 1 1
=  − +  (b) If A and B working together can finish a work
2X Y Z
in X days and B is k times efficient than A,
XY + YZ − ZX then the time taken by
= .
2 XYZ (i) A, working alone, to complete the work is
(k + 1) X.
 2XYZ 
So, A alone can do the work in   days. (ii) B, working alone, to complete the work
 XY + YZ − ZX 
 k + 1
 2XYZ  is  X .
Similarly, B alone can do the work in   k 

 YZ + ZX − XY 
 2XYZ  Illustration 5 Harbans Lal can do a piece of work in 24
days and C alone can do the work in   days.
 ZX + XY − YZ  days. If Bansi Lal works twice as fast as Harbans Lal, how
long would they take to finish the work working together?
Illustration 4 A and B can do a piece of work in 12 days,
B and C in 15 days, C and A in 20 days. How long would Solution: Here X = 24 and k = 2.
each take separately to do the same work? ∴ Time taken by Harbans Lal and Bansi Lal, working
Solution: Here X = 12, Y = 15 and Z = 20. together to complete the work
∴ A alone can do the work in  X 
=   days.
=
2XYZ 1 + k 
XY + YZ − ZX
 24 
=   days, i.e., 8 days.
2 × 12 × 15 × 20 1 + 2 
= days
12 × 15 + 15 × 20 − 20 × 12
Illustration 6 A and B together can do a piece of work in
7200
or , i.e., 30 days. 3 days. If A does thrice as much work as B in a given time,
240 find how long A alone would take to do the work?
B alone can do the work in
Solution: Here X = 3 and k = 3.
2XYZ
= days ∴ Time taken by A, working alone, to complete the
YZ + ZX − XY work
2 × 12 × 15 × 20  k + 1  3 + 1
= days
15 × 20 + 20 × 12 − 12 × 15 =  X =   3 = 4 days.
 k   3 

Chapter_10.indd 226 2/1/2016 9:49:09 AM


Time, Work and Wages 227

6. If A working alone takes a days more than A and a


8. If A can complete part of work in X days, then
B working alone takes b days more than A and B b
together, then the number of days taken by A and B, c b×c× X
part of the work will be done in
working together, to finish a job is given by ab . d a×d
days.
Illustration 7 A alone would take 8 hrs more to complete
the job than if both A and B worked together. If B worked
Illustration 9 A can do three-fourths of a work in 12 days.
1 In how many days can he finish one-eighth of the work?
alone, he took 4 hrs more to complete the job than
2
Solution: Here a = 3, b = 4, X = 12, c = 1 and d = 8.
A and B worked together. What time would they take if both
A and B worked together? Therefore, number of days required to finish one-eighth
of the work
9
Solution: Here a = 8 and b =
.
2 b×c× X 4 × 1 × 12
= = = 2 days.
∴ Time taken by A and B, working together, to a×d 3×8
complete the job
= ab days 9. (a) There are two groups of people with same
efficiency. In one M1 persons can do W1
9
= 8× or 6 days. works in D1 time and in the other M2 persons
2 can do W2 works in D2 time. The relationship
between the two groups is given by
7. If A is k times more efficient than B and is therefore
able to finish a work in l days less than B, then M1D1W2 = M2D2W1

(a) A and B, working together, can finish the (b) There are two groups of people with same
kl efficiency. In one M1 persons can do W1
work in 2 days. works in D1 time working t1 hrs a day and M2
k −1 persons can do W2 works in D2 time working
(b) A, working alone, can finish the work in t2 hrs a day. The relationship between the two
l groups is given by
days.
k −1 M1D1t1W2 = M2D2t2W1.
(c) B, working alone, can finish the work in
kl
days. Illustration 10 If 10 persons can complete two-fifths of a
k −1
work in 8 days, then find the number of persons required to
complete the remaining work in 12 days.
Illustration 8 A is thrice as good a workman as B and takes
10 days less to do a piece of work than B takes. Find the 2
Solution: We have M1 = 10, W2 = ,D =8
time in which B alone can complete the work. 5 1
Solution: Here k = 3 and l = 10. 3
M2 = ?, W2 = , D = 12.
∴ Time taken by B, working alone, to complete the work 5 2
kl
= days ∴ M1D1W2 = M2D2W1
k −1
3 × 10 23 2
= days ⇒ 10 × 8 × = M2 × 12 ×
3 −1 5 5

i.e., 15 days. ⇒ M2 = 10.

Chapter_10.indd 227 2/1/2016 9:49:10 AM


228 Chapter 10

Illustration 11 If 10 persons can cut 20 trees in 3 days


working 12 hrs a day. Then, in how many days can 24  nab 
persons cut 32 trees working 4 hrs a day? work in   days
 bc + ad 
Solution: We have, M1 = 10, W1 = 20, D1 = 3, t1 = 12
Illustration 12 12 men or 15 women can do a work in
M2 = 24, W2 = 32, D2 = ?, t2 = 4
14 days. In how many days, 7 men and 5 women would
∴ M1D1t1W2 = M2D2t2W1 complete the work?
Solution: Here a = 12, b = 15, n = 14, c = 7 and d = 5.
⇒ 10 × 3 × 12 × 32 = 24 × D2 × 4 × 20
Required number of days
⇒ D2 = 6 days. nab  14 × 12 × 15 
= =   days
bc + ad  15 × 7 + 12 × 5 
10. If a men and b women can do a piece of work
in n days, then c men and d women can do the 168 3
= days or 15 days.
11 11

MULTIPLE CHOICE QUESTIONS

1. Running at the same constant rate, 6 identical machines 5. Lal Singh can eat 50 laddoos in 4 hrs and Pal Singh can
can produce a total of 270 bottles per min. At this rate, eat 42 laddoos in 6 hrs. If both of them start together, then
how many bottles could 10 such machines produce in 4 what is the total time required by them to eat 507 laddoos?
mins? (a) 20 hrs (b) 21 hrs
(a) 648 (b) 1,800 (c) 26 hrs (d) 25 hrs
(c) 2,700 (d) 10,800 6. The daily work of 2 men is equal to that of 3 women or
[Based on MAT, 2004] that of 4 youngsters. By employing 14 men, 12 women
2. Machine A produces bolts at a uniform rate of 120 every and 12 youngsters a certain work can be finished in 24
40 s, and machine B produces bolts at a uniform rate of days. If it is required to finish it in 14 days and as an
100 every 20 s. If the two machines run simultaneously, additional labour, only men are available, how many of
then how many seconds will it take them to produce a them will be required?
total of 200 bolts? (a) 20 men (b) 18 men
(a) 22 (b) 25 (c) 48 men (d) 28 men
(c) 28 (d) 32 7. Two men undertake to do a piece of work for `200. One
[Based on MAT, 2004] alone can do it in 6 days and the other in 8 days. With the
help of a boy, they finish it in 3 days. How much is the
3. A group of workers can complete a certain job in 9 days. share of the boy?
But it so happens that every alternate day starting from (a) `45 (b) `40
the second day, 2 workers are withdrawn from the job and
(c) `30 (d) `25
every alternate day starting from the third day, one worker
[Based on MAT, 2003]
is added to the group. In such a way, the job is finished by
the time there is no worker left. If it takes the double time 2
to finish the job now, find the number of workers who 8. A worker makes a basket in of an hour. If he works for
3
started the job?
(a) 5 (b) 10 1
7 hrs, then how many baskets will he make?
(c) 15 (d) 20 2

4. A can do a piece of work in 40 days and B in 60 days. 3 1


(a) 10 (b) 11
Both of them start working together and 4 days before the 4 4
scheduled completion, A drops out. By how many days is
the work extended beyond the normal schedule? 1
(c) 12 (d) 13
2
(a) 10 days (b) 8 days [Based on MAT, 2003]
(c) 8.33 days (d) 6 days

Chapter_10.indd 228 2/1/2016 9:49:11 AM


 Time, Work and Wages  229

9.
A and B require 10 days to complete a job. B and C require 16. A and B weave a carpet in 10 days and 15 days, respectively.
12 days to complete the same job. A and C require 15 days They begin to work together but B leaves after 2 days. In
to complete the same job. The number of days required, if what time will A complete the remaining work?
all are at work, to compete the job is 1 2
(a) 6 days (b) 6 days
(a) 8 days (b) 9 days 3 3
(c) 6 days (d) 7 days (c) 7 days (d) 8 days
10. Rohit, Harsha and Sanjeev are three typists who, working [Based on MAT, 2001]
simultaneously, can type 216 pages in four hrs. In one 17.
A can do a work in 18 days, B in 9 days and C in 6 days.
hour, Sanjeev can type as many pages more than Harsha A and B start working together and after 2 days C joins
as Harsha can type more than Rohit. During a period of them. What is the total number of days taken to finish the
rive hrs, Sanjeev can type as many pages as Rohit can work?
during seven hrs. How many pages does each of them (a) 4.33 (b) 4.5
type per hour? (c) 4.66 (d) None the these
(a) 16, 18, 22 (b) 14, 17, 20 [Based on MAT, 2000]
(c) 15, 17, 22 (d) 15, 18, 21
18.
If 18 persons can build a wall 140 m long in 42 days, the
[Based on MAT, 2003] number of days that 30 persons will take to complete a
11. A can do a work in 9 days. If B is 50% more efficient to A, similar wall 100 m long is
then in how many days can B do the same work? (a) 18 (b) 21
(a) 13.5 (b) 4.5 (c) 24 (d) 28
(c) 6 (d) 3 [Based on MAT, 2005]
[Based on MAT, 2003] 19.
A can do piece of work in 14 days which B can do in
12. Two men and 7 children complete a certain piece of work 21 days. They begin together but 3 days before the
in 4 days while 4 men and 4 children complete the same completion of the work A leaves off. The total number of
work in only 3 days. The number of days required by 1 days to complete the work is
man to complete the work is 3 1
(a) 60 days (b) 15 days (a) 6 (b) 8
5 2
(c) 6 days (d) 51 days
1 1
[Based on MAT, 2003] (c) 10 (d) 13
5 2
13. Shatabadi Express has a capacity of 500 seats of which [Based on MAT, 2005]
10% are in the Executive Class and the rest being Chair
cars. During one journey, the train was booked to 85% 20. A contractor undertook to do a certain work in 75 days
of its capacity. If Executive Class was booked to 96% and employed 60 men to do it. After 25 days he found
of capacity, then how many Chair Car seats were empty that only one-fourth of the work was done. How many
during that journey? more men must be employed in order that the work may
be finished in time?
(a) 75 (b) 73
(c) 71 (d) None of these (a) 34 (b) 38
[Based on MAT, 2002] (c) 35 (d) 30
21. A man can finish a work in 6 days, a woman in 10 days
14. ‘A’ can do a piece of work in 25 days and B in 20 days.
and a child in 12 days. The man starts the work and
They work together for 5 days and then ‘A’ goes away. In
works for one-third of a day, then the woman works for
how many days will ‘B’ finish the remaining work?
one-third of a day and finally the boy works for remaining
(a) 17 days (b) 11 days one-third of a day. This cycle is repeated till work finishes.
(c) 10 days (d) None of these Who was working when the work finished?
[Based on MAT, 2002] (a) Man (b) Woman
15. If Ajit can do one-fourth of a work in 3 days and Sujit can (c) Child (d) None
do one-sixth of the same work in 4 days, how much will
22.
A contractor undertook to do a piece of work in 9 days.
Ajit get if both work together and are paid `180 in all?
He employed certain number of labourers but 6 of them
(a) `120 (b) `108 being absent from the very first day, the rest could finish
(c) `60 (d) `36 the work in 15 days. Find the number of men originally
[Based on MAT, 2001] employed.

Chapter_10.indd 229 2/1/2016 9:49:11 AM


230  Chapter 10

(a) 15 (b) 6 (a) 26.25 days (b) 32 days


(c) 13 (d) 9 (c) 28.125 days (d) 30.12 days
[Based on MAT, 2001]
30. If four examiners can examine a certain number of answer
23. A man can do a job in 5 hrs. After 2 h 20 mins, the man books in 8 days by working 5 hrs a day, then for how
stops working. He is replaced by a woman to complete the many hrs a day, would two examiners have to work in
job. She does the remainder of the work in 1 h 40 mins. If order to examine twice the number of answer books in 20
the woman works alone, how much faster will she be than days?
the man? 1
(a) 1 h 25 mins (b) 1 h 55 mins (a) 6 hrs (b) 7 hrs
2
(c) 2 hrs (d) 1 h 17 mins
(c) 8 hrs (d) 9 hrs
24. If a man and a half can build a wall and a half in a day
[Based on FMS (Delhi), 2003]
and a half, then how many walls do six men build in six
days? 31. 15 men take 21 days of 8 hrs each to do a piece of work.
(a) 3 (b) 6 How many days of 6 hrs each would 21 women take, if 3
women do as much work as 2 men?
(c) 12 (d) None of these
(a) 20 (b) 18
[Based on FMS (Delhi), 2004]
(c) 25 (d) 30
25. Five engineers A, B, C, D and E can complete a process [Based on FMS (Delhi), 2003]
in 8 hrs, assuming that every engineer works with same
efficiency. They started working at 10.00 a.m. If after 4.00 32. A certain job was assigned to a group of men to do in
p.m., one engineer is removed from the group every hour, 20 days. But 12 men did not turn up for the job and the
what is the time when they will finish the work? remaining men did the job in 32 days. The original number
(a) 6.00 p.m. (b) 7.00 p.m. of men in the group was
(c) 4.00 p.m. (d) 8.00 p.m. (a) 32 (b) 34
(c) 36 (d) 40
26. 30 men can produce 1500 units in 24 days working 6 hrs
[Based on FMS (Delhi), 2003]
a day. In how many days, can 18 men produce 1800 units
working 8 hrs a day? 33. Niki types 150 words in 4 mins and Nishu types 150
(a) 36 days (b) 45 days words in 5 mins, then in how much time they can together
type 375 words?
(c) 18 days (d) None of these
[Based on IIT Joint Man. Ent. Test, 2004]
50 7
(a) mins (b) mins
9 2
27. If four boys consume x calories every y days, six girls
y 7
consume z calories every days. How many calories (c) mins (d) None of these
2 9
will 10 girls and 10 boys consume in 12 days? 34. Ram finishes a work in 7 days. Rahim finishes the same
10 job in 8 days and Robert in 6 days. They take turns to
(a) 3x + 4z (b) (3x + 4z) finish the work. Ram worked on the first day, Rahim on
y
1 the second day and Robert on the third and then again
(c) (30x + 4z) (d) y(3x + 4z) Ram and so on. Who was working on the last day when
y
work got finished?
28. A piece of work is done by three men A, B, C in 5 days in (a) Ram (b) Rahim
the following manner: (c) Robert (d) Rahim and Robert
A works for the whole time, B only on the first two days (e) Cannot be determined
and C only on the last three days. This work could have
[Based on IIFT, 2003]
been done by B and C in 6 days without involving A. If B
and C working together can do as much work in two days 35. Ritu and Somaya are working on an assignment. Ritu
as A can do in 3 days, then find, how long will it take for takes 6 hrs to type 32 pages on computer, while Somaya
each one to do this work alone? takes 5 hrs for typing 40 pages. How much time will they
(a) 10, 20, 10 days (b) 7, 9, 11 days take, working together on two different computers to type
an assignment of 110 pages?
(c) 8, 12, 9 days (d) 9, 18, 9 days
(a) 8 hrs (b) 7 hrs 30 mins
29. If a man can reap 80 hectares in 24 days and a woman can
reap 60 hectares in 30 days, then how much time they will (c) 8 hrs 15 mins (d) 8 hrs 25 mins
take together to reap 150 hectares? [Based on SCMHRD Ent. Exam., 2003]

Chapter_10.indd 230 2/1/2016 9:49:11 AM


 Time, Work and Wages  231

36. 16 men and 12 women can complete a work in 20 days. (a) `11.70; `12.00 (b) `10.80; `13.00
18 women can complete the same work in 40 days. In how (c) `11.25; `12.50 (d) `12.60; `11.00
many days will 12 men and 27 women complete the same [Based on SCMHRD, 2002]
work?
43. Eight children and 12 men complete a certain piece of
(a) 12 (b) 16
work in 9 days. If each child takes twice the time taken by
(c) 18 (d) 24 a man to finish the work, in how many days will 12 men
37. A boy read three-eighths of the book on one day and finish the same work?
four-fifths of the remainder on another day. If there were (a) 15 (b) 8
30 pages unread, then how many pages did the book
(c) 9 (d) 12
contain?
[Based on MAT, 2008]
(a) 600 (b) 300
44. Imran and Irfan are two tailors. Imran takes three hrs to
(c) 240 (d) None of these stitch 10 shirts and four hrs to stitch 12 pants. Irfan can
[Based on IMT Ghaziabad, 2002] stitch 12 pants in three hrs and 10 shirts in four hrs. They
38. Twenty men were employed to do some work in a get an order for the delivery of 200 shirts and 200 pants.
certain time. But when one-third of the scheduled time What is the quickest time in which they can deliver the
was over, it was found that only one-quarter of the total order?
work was completed. How many more men should now (a) 59 hrs 6 mins
be employed to complete the work in three-fourths of the (b) 66 hrs 6 mins
originally scheduled time? (c) 63 hrs 12 mins
(a) 20 (b) 40 (d) 55 hrs 43 mins
(c) 48 (d) 28
45. Mohan can complete a work in 25 days. He worked for
39. Three workers, working all days, can do a work in 10 days, 5 days and left the work; and then Bhim completed the
but one of them having other employment can work only remaining work in 30 days. Had Bhim started the work
half time. In how many days the work can be finished? and left it after 15 days, how much more time would
(a) 15 days (b) 16 days Mohan have taken to complete the remaining work?
(c) 12 days (d) 12.5 days (a) 7.5 days (b) 11.5 days
[Based on FMS (Delhi), 2002] (c) 12 days (d) 15 days
40. Sam, Bob and Kirm can do a job alone in 15 days, 10 Directions (Questions 46 and 47): Answer the questions based on
days and 30 days respectively. Sam is helped by Bob and the following information.
Kim every third day. In how many days will the job be
   Rakesh, Shiv and Vijay working alone can complete fencing a
completed?
wall in 12, 18 and 27 hrs respectively. All of them started fencing
1
(a) 9 (b) 8 the wall together as a team and after 2 hrs, Rakesh left the team.
3 Shiv and Vijay continued to fence until Vijay fell ill and hence he
1 had to leave the team. Then, Shiv finished fencing the wall in the
(c) 8 (d) 6 last 5 hrs.
3
[Based on I.P. Univ., 2002] 46. How many hrs did it take for the team to finish fencing the
wall?
41. Construction of a road was entrusted to a civil engineer.
He was to finish the work in 124 days for which he (a) 11 hrs (b) 13 hrs
employed 120 workmen. Two-thirds of the work was (c) 15.66 hrs (d) None of these
completed in 64 days. How many workmen can be
47.
Which pair among the following will be able to finish
reduced now without affecting the completion of the
fencing the wall in the least time?
work on time?
(a) Rakesh and Shiv (b) Shiv and Vijay
(a) 56 (b) 64
(c) Rakesh and Vijay (d) Either (b) or (c)
(c) 80 (d) 24
[Based on I.P. Univ., 2002] 48. A and B working separately can complete a job in 20 days
and 12 days, respectively. If they work on alternate days
42. Two workers earned `225. The first worked for 10 days in tandem with B alone starting the job on the first day,
and the second for 9 days. How much did each of them get then the job will get completed on
daily if the first worker got `15 more for working 5 days (a) 14th day (b) 15th day
than the second worker got for working 3 days?
(c) 16th day (d) 17th day

Chapter_10.indd 231 2/1/2016 9:49:11 AM


232  Chapter 10

49. A, B and C contract a work for `4,500. A and B together 56. A group of workers decided to finish a work in 10 days but
complete four-fifths of the work and then C takes over 5 of them could not join the team. If the rest of the crew
and finishes it. What is the amount that should be received completed the job in 12 days, the number of members
by C? present originally in the team were
(a) `600 (b) `900 (a) 45 (b) 30
(c) `1,200 (d) Cannot be determined (c) 50 (d) 35
57. A, B, and C, working together complete a job in 18
50. Robin, the cook, can cut either 120 carrots or 72 potatoes
days. A, and B, together work twice as C, A, and C,
or 60 beetroots in t mins. One day, during the first K mins,
together work thrice as much as B, A, alone can finish the
he spent an equal amount of time cutting each of the three
work in
types of vegetables. However, in the next K mins, he cut
exactly n carrots, n beetroots and n potatoes. If he cut a (a) 18 days (b) 43.2 days
total of 282 vegetables during the first 2K mins on that (c) 54 days (d) 72 days
day, what is the value of n? 58. A is twice as fast as B, and is therefore able to finish the
(a) 225 (b) 75 work in 30 days less than B. Find the time in which they
(c) 45 (d) 135 can do it working together.
(a) 18 days (b) 20 days
51. A team of 30 men is supposed to do a work in 38 days.
After 25 days, 5 more men were employed and the work (c) 24 days (d) 22 days
was finished one day earlier. How many days would it Directions (Questions 59 and 60): Answer the questions based on
have been delayed if 5 more men were not employed? the following information.
(a) 1 day (b) 4 days   The Weirodo Holiday Resort follows a particular system of
(c) 3 days (d) 5 days holidays for its employees. People are given holidays on the days
where the first letter of the day of the week is the same as the first
52. If 15 men or 24 women or 36 boys can do a work in letter of their names. All employees work at the same rate.
12 days working 8 hrs a day, how many men must be
associated with 12 women and 6 boys to do another work  59. Raja starts working on February 25, 1996 and finishes the
1 job on March 2, 1996. How much time would T and I
2  times as great in 30 days working 6 hrs per day ? take to finish the same job if both start on the same day as
4
Raja?
(a) 10 (b) 15
(a) 4 days (b) 5 days
(c) 8 (d) None of these
(c) Either 4 or 5 days (d) Cannot be determined
53. A contractor undertook to complete the work in 40 days
and he deployed 20 men for his work 8 days before the 60. Starting on February 25, 1996, if Raja had finished his
scheduled time he realised that one-third of the work was job on April 2, 1996, when would T and S likely to have
still to be done. How many more men were required to completed the job, had they started on the same day as
complete the work in stipulated time? Raja?
(a) 16 (b) 15 (a) March 15, 1996 (b) March 14, 1996
(c) 20 (d) 25 (c) March 22, 1996 (d) Data insufficient
54. A contractor undertook a work to complete in 60 days. But Directions (Questions 61 and 62): Answer the questions on the
just after 20 days he observed that only one-fifth of the basis of the information given below.
project work had been completed. To complete the work
in time (i.e., in rest days) minimum how many workers   Ram and Shyam run a race between points A and B, 5 km
he had to increase, if there were initially 75 workers were apart. Ram starts at 9 a.m. from A at a speed of 5 km/hr, reaches
deployed for the task? B, and returns to A at the same speed. Shyam starts at 9:45 a.m.
from A at a speed of 10 km/hr, reaches B and comes back to A at
(a) 25 (b) 50 the same speed.
(c) 75 (d) Cannot be determined
61. At what time do Ram and Shyam first meet each other?
55. A group of men decided to do a job in 4 days. But since
(a) 10.00 a.m. (b) 10.10 a.m.
20 men dropped out every day, the job completed at the
end of the 7th day. How many men were there at the (c) 10.20 a.m. (d) 10.30 a.m.
beginning?
62. At what time does Shyam overtake Ram?
(a) 240 (b) 140
(a) 10.20 a.m. (b) 10.30 a.m.
(c) 280 (d) 150
(c) 10.40 a.m. (d) 10.50 a.m.

Chapter_10.indd 232 2/1/2016 9:49:12 AM


 Time, Work and Wages  233

63. Cloth markers Inc. has p spindles, each of which can 70.
A does half as much work as B in three-fourth of the time.
produce q metres of cloth on an average in r mins. If If together they take 18 days to complete a work. How
the spindles are made to run with no interruption, then much time shall B take to do it alone?
how many hrs will it take for 20,000 m of cloth to be (a) 30 days (b) 35 days
produced?
(c) 40 days (d) 45 days
(a) 20,000 (pq)/r (b) 20,000 (rq)/p
[Based on SSC (GL), 2011]
(c) 20,000 (pr)/(pq) (d) 20,000 r/60 (pq)
71.
A and B working separately can do a piece of work in 9
64.
7 men can complete a piece of work in 12 days. How and 12 days respectively. If they work for a day alternately
many additional men will be required to complete double with A beginning, the work would be completed in
the work in 8 days?
2 1
(a) 28 (b) 21 (a) 10 days (b) 10 days
3 2
(c) 14 (d) 7
1 1
[Based on SSC (GL), 2010] (c) 10 days (d) 10 days
4 3
65.
X is three times as fast as Y and is able to complete the
[Based on SSC (GL), 2011]
work in 40 days less than Y. Then the time in which they
can complete the work together is 72.
A daily wage earner gets a daily wage at the rate of `150
(a) 15 days (b) 10 days per day subject to the condition that he will have to pay a
penalty at the rate of `25 per day for the days on which he
1 absents himself. At the end of 60 days he gets an amount
(c) 7 days (d) 5 days
2 of `7,600. The number of days he worked is
[Based on SSC (GL), 2011] (a) 54 days (b) 52 days
66.
A can do a work in 12 days. When he had worked for 3 (c) 51 days (d) 48 days
days, B joined him. If they complete the work in 3 more [Based on U.P. P.C.S., 2012]
days, in how many days can B alone finish the work?
73. 6 men can complete a piece of work in 12 days. 8 women
(a) 6 days (b) 12 days can complete the same piece of work in 18 days whereas
(c) 4 days (d) 8 days 15 children can complete the piece of work in 10 days. 4
[Based on SSC (GL), 2011] men, 12 women and 20 children work together for 2 days.
If only men were to complete the remaining work in 1 day
67.
A is thrice as good a workman as B and is, therefore, able
how many men would be required totally?
to finish a piece of work in 60 days less that B. The time
(in days) in which they can do it working together is (a) 36 (b) 24
(c) 18 (d) Cannot be determined
1
(a) 22 (b) 22 [Based on Bank of India PO, 2010]
2
74. Ramesh has two examinations on Wednesday-Engineering
1 Mathematics in the morning and Engineering Drawing in
(c) 23 (d) 23
4 the afternoon. He has a fixed amount of time to read the
[Based on SSC (GL), 2011] textbooks of both these subjects on Tuesday. During this
time he can read 80 pages of Engineering Mathematics
68.
A work can be completed by P and Q in 12 days, Q and R
and 100 pages of Engineering Drawing. Alternatively,
in 15 days, R and P in 20 days. In how many days P alone
he can also read 50 pages of Engineering Mathematics
can finish the work?
and 250 pages of Engineering Drawing. Assume that the
(a) 10 (b) 20 amount of time it takes to read one page of the textbooks
(c) 30 (d) 60 of either subject is constant. Ramesh is confident about
[Based on SSC (GL), 2011] Engineering Drawing and wants to devote full time
to reading Engineering Mathematics. The number of
69.
‘x’ number of men can finish a piece of work in 30 days. If Engineering Mathematics text book pages he can read on
there were 6 men more, the work could be finished in 10 Tuesday is
days less. The original number of men is
(a) 500 (b) 300
(a) 6 (b) 10
(c) 100 (d) 60
(c) 12 (d) 15
[Based on JMET, 2006]
[Based on SSC (GL), 2011]

Chapter_10.indd 233 2/1/2016 9:49:12 AM


234  Chapter 10

75. A started a work and left after working for 2 days. Then B (a) 24 days (b) 10 days
was called and he finished the work in 9 days. Had A left (c) 32 days (d) 60 days
the work after working for 3 days, B would have finished [Based on MAT (Feb), 2011]
the remaining work in 6 days. In how many days can each
of them, working alone, finish the whole work? 82.
Manoj and Ajita can do a job alone in 10 days and 12 days
(a) 5 days, 8.5 days (b) 2.5 days, 7.5 days respectively. Manoj starts the work and after 6 days Ajita
also joins to finish the work together. For how many days
(c) 5 days, 15 days (d) None of these
did Ajita actually work on the job?
[Based on NMAT, 2005]
3 1
76. 12 men can complete a piece of work in 36 days. 18 (a) 2 (b) 2
women can complete the same piece of work in 60 days. 11 11
8 men and 20 women work together for 20 days. If only 2 1
(c) 2 (d) 3
women were to complete the remaining piece of work in 11 12
4 days, how many women would be required? [Based on MAT (Dec), 2010]
(a) 70 (b) 28
83.
Ashokan is thrice as good a workman as Nitin and is
(c) 66 (d) 40 therefore able to finish a piece of work in 40 days less
[Based on IRMA, 2009] than Nitin. Find the time in which they can do it working
77. Two men alone or three women alone can complete together.
a piece of work in 4 days. In how many days can one (a) 15 days (b) 7 days
woman and one man together complete the same piece of (c) 16 days (d) 13 days
work? [Based on MAT (Dec), 2010]
24
(a) 6 days (b) days 84.
A alone would take 8 h more to complete the job than if
5
both A and B worked together. If B worked alone, he took
12
(c) days (d) Cannot be determined 1
1.75 4 h more to complete the job than if A and B worked
2
[Based on Corporation Bank PO, 2011]
together. What time would they take if both A and B
78. 4 girls can do a piece of work in 8 days, same work 3 boys worked together?
can do in 9 days, 7 men in 2 days and 5 women in 4 days.
(a) 8 days (b) 5 days
Who among them have the minimum capacity of work?
(c) 2 days (d) 6 days
(a) Boy (b) Girl
[Based on MAT (Dec), 2010]
(c) Man (d) Woman
[Based on Union Bank of India PO, 2011] 85.
Madhu takes twice as much time as Uma to complete a
work and Ramesh does it in the same time as Madhu and
79. Work done by A in one day is half of the work done by B
Uma together. If all three working together can finish the
in one day. Work done by B is half of the work done by
work in 6 days, then the time taken by Madhu to finish the
C in one day. If C alone can complete the work in 7 days,
work is
in how many days can A, B and C together complete the
work? (a) 12 days (b) 14 days
(a) 28 (b) 14 (c) 36 days (d) 40 days
(c) 4 (d) 21 [Based on MAT (Dec), 2010]
[Based on SBI PO, 2008] 86.
A 12 m long road can be dug by 18 men in 20 days. What
80.
A can do a piece of work in 40 days. He starts working, length of road can be dug by 12 men in 15 days?
but having some other engagements he drops out after 5 (a) 9 m (b) 8 m
days. Thereafter, B completes this work in 21 days. How (c) 6 m (d) 7 m
many days would A and B take to complete this work [Based on MAT (May), 2010]
working together?
87.
A, B and C can do a work in 8, 16 and 24 days respectively.
(a) 16 days (b) 15 days
They all begin together. A continues to work till it is
(c) 17 days (d) 11 days finished, C left after 2 days and B one day before its
[Based on MAT (Feb), 2011] completion. In what time is the work finished?
81.
A and B together can do a piece of work in 30 days. A (a) 7 days (b) 5 days
and B worked for 16 days and B finished the remaining
(c) 6 days (d) 8 days
work alone in 44 days. In how many days will B finish the
[Based on MAT (May), 2010]
whole work alone?

Chapter_10.indd 234 2/1/2016 9:49:12 AM


 Time, Work and Wages  235

88.
Working together, Asha and Sudha can complete an 94.
Two cogged wheels of which one has 16 cogs and the
assigned task in 20 days. However, if Asha worked alone other 27, work into each other. If the latter turns 80 times
and complete half the work and then Sudha takes over the in three quarters of a min, how often does the other turn in
task and completes the second half of the task, the task 8 s?
will be completed in 45 days. How long will Asha take (a) 26 (b) 25
to complete the task if she worked alone? Assume that
(c) 24 (d) 27
Sudha is more efficient than Asha.
[Based on MAT (Dec), 2009, 2008]
(a) 60 days (b) 30 days
(c) 25 days (d) 65 days 1
95.
X runs 1 times as fast as Y. If X gives Y a start of 300 m,
[Based on MAT (May), 2010] 2
how far must X run before he catches up with Y?
89.
A computer can perform 30 identical tasks in 6 h. At that
rate, what is the minimum number of computers that (a) 450 m (b) 400 m
should be assigned to complete 80 of the tasks within 3 h? (c) 1 km (d) 900 m
(a) 12 (b) 7 [Based on MAT (Dec), 2009]
(c) 6 (d) 16 96. B and C together can complete a work in 8 days, A and B
[Based on MAT (Dec), 2009] together can complete the same work in 12 days, and A
and C together can complete the same work in 16 days.
90.
Construction of a road was entrusted to a civil engineer. He In, how many days can A, B and C together complete the
was to finish the work in 124 days for which he employed same work?
120 workmen. Two-thirds of the work was completed
in 64 days. How many workmen can be reduced now 9 5
(a) 3 (b) 7
without affecting the completion of the work on time? 13 13
(a) 80 (b) 64 5 5
(c) 7 (d) 3
(c) 56 (d) 24 12 12
[Based on MAT (Dec), 2009] [Based on Andhra Bank PO, 2009]
91.
Anu can complete a work in 10 days. Manu is 25% more 97.
A team of workers was employed by a contractor who
efficient than Anu and Sonu is 60% more efficient than undertook to finish 360 pieces of an article in a certain
Manu. Working together, how long would they take to number of days. Making four more pieces per day that
finish the job? was planned, they could complete the job a day ahead of
6 6 schedule. How many days did they take to complete the
(a) 2 days (b) 5 days job?
17 7
(a) 8 days (b) 10 days
5 5
(c) 3 days (d) 4 days (c) 9 days (d) 12 days
8 8
[Based on MAT (Sept), 2009]
[Based on MAT (Dec), 2009]
98.
The work done by a woman in 8 h is equal to the work
92. 8 men and 4 women together can complete a piece of done by a man in 6 h and by a boy in 12 h. If working 6 h
work in 6 days. Work done by a man in one day is double per day 9 men can complete a work in 6 days, then in how
the work done by a woman is one day. If 8 men and 4 many days can 12 men, 12 women and 12 boys together
women started working and after 2 days, 4 men left and 4 finish the same working 8 h per day?
new women joined. In how many more days will the work
1 1
be completed? (a) 2 days (b) 1 days
2 2
(a) 5 days (b) 8 days
(c) 6 days (d) 4 days 1
(c) 3 days (d) None of these
[Based on United Bank of India PO, 2009] 2
[Based on MAT (Sept), 2009]
93.
If 15 men or 24 women or 36 boys can do a piece of work
in 12 days, working 8 h a day, how many men must be 99.
A sum of `25 was paid for a work which A can do in 32
associated with 12 women and 6 boys to do another piece days, B in 20 days, B and C in 12 days and D in 24 days.
of work 2.25 times in 30 days working 6 h in a day? How much did C receive if all the four work together?
(a) 9 (b) 7 (a) `15/3 (b) `14/3
(c) 5 (d) 8 (c) `13/3 (d) `16/3
[Based on MAT (Dec), 2009] [Based on MAT (May), 2009]

Chapter_10.indd 235 2/1/2016 9:54:49 AM


236  Chapter 10

100. Ten women can complete a piece of work in 15 days. Six (a) 20 h/day (b) 18 h/day
men can complete the same piece of work in 10 days. In (c) 16 h/day (d) 14 h/day
how many days can five women and six men together
[Based on MAT (Sept), 2008]
complete the piece of work?
(a) 15 days (b) 7.5 days 107.
If 36 men can dig a trench 200 m long, 3 m wide and 2
m deep in 6 days working 10 h a day, in how many days,
(c) 9 days (d) 12.5 days
working 8 h a day will 10 men dig a trench 100 m long, 4
[Based on IRMA, 2006] m wide and 3 m deep?
101.
A, B and C working together completed a job in 10 days. (a) 15 days (b) 27 days
The ratio of their efficiency is 2:3:6. How many days
(c) 20 days (d) 54 days
would be required by the fastest worker to complete the
entire work? [Based on MAT (Sept), 2008]

(a) 15 days (b) 20 days 108.


Bhavika alone would take 8 h more to complete the job
(c) 30 days (d) 40 days than when Bhavika and Rita worked together. If Rita
[Based on MAT (Feb), 2009] 1
worked alone, she would take 4 h more to complete the
2
102.
8 children and 12 men complete a certain piece of work in
job than when Bhavika and Rita worked together. What
9 days. If each child takes twice the time taken by a man
time would they take if both Bhavika and Rita worked
to finish the work, 12 men will finish the same work in
together?
(a) 12 days (b) 15 days
(a) 5 days (b) 4 days
(c) 14 days (d) 8 days
(c) 3 days (d) 6 days
[Based on MAT (Feb), 2009, 2008]
[Based on MAT (Sept), 2008]
103.
If 15 men or 24 women or 36 boys can do a piece of work
in 12 days, working 8 h a day, how many men must be 109.
Ten men can finish a piece of work in 10 days, whereas
associated with 12 women and 6 boys to do another piece it takes 12 women to finish it in 10 days. If 15 men and 6
woman undertake the work, how any days will they take
1 to complete it?
of work 2 times as great in 30 days working 6 h a day?
4
1
(a) 4 (b) 8 (a) 4 days (b) 4 days
2
(c) 6 (d) 10
(c) 5 days (d) 6 days
[Based on MAT (Dec), 2008]
[Based on MAT (Sept), 2008]
104. 18 men can complete a piece of work in 63 days. 9 women
110.
Five painters can paint a wall 100 m long in 10 days of 8
take 189 days to complete the same piece of work. How
h each. How many days of 6 h each will it take for 8 men
many days will 4 men, 9 women and 12 children together
to paint a wall 30 m long?
take to complete the piece of work if 7 children alone can
complete the piece of work in 486 days? 1
(a) 2 (b) 1
(a) 76 (b) 63 2
(c) 54 (d) 81 3
(c) 3 (d) 1
[Based on IRMA, 2005] 4
105.
A certain length of pathway has to be constructed. It is [Based on MAT (May), 2008]
found that three men can construct 1 km less than one- 111.
There are two types of workers—category I and category
fifth of all in 2 days, while 18 men can construct 1 km II. A category I worker can finish a piece of work in 2m
more than two-fifths in one day. The length of the path is per 3 h and a category II worker can finish in m h. If one
(a) 25 km (b) 10 km worker of category I and two workers of category II are
(c) 15 km (d) 20 km employed, the work can be finished in how many hrs?
[Based on MAT (Sept), 2008] (a) 7 m/2 (b) 9 m/2
(c) 2 m/7 (d) 2 m/9
106.
Two coal loading machines each working 12 h per
[Based on MAT (May), 2008]
day for 8 days handle 9000 tonnes of coal with an
efficiency of 90%; while 3 other coal loading machines 112. 8 men can complete a piece of work in 20 days. 8 women
at an efficiency of 80% are set to handle 12000 tonnes of can complete the same work in 32 days. In how many
coal in 6 days. Find how many hrs per day each should days will 5 men and 8 women together complete the same
work? work?

Chapter_10.indd 236 2/1/2016 9:49:12 AM


 Time, Work and Wages  237

(a) 16 days (b) 12 days 1 1


(a) 13 days (b) 6 days
(c) 14 days (d) 10 days 3 3
[Based on CBI (PO), 2010]
1
113.
Two men A and B working together complete a piece of (c) 7 days (d) 7 days
3
work which it would have taken them 30 and 40 days [Based on MAT (May), 2006]
respectively to complete if they worked separately. If they
119.
Children were fallen-in for a drill. If each row contained
received a payment of `2100, B’s share is
4 children less, 10 more rows would have been made. But
(a) `900 (b) `800 if 5 more children were accommodated in each row the
(c) `1200 (d) `1300 number of rows would have reduced by 5. The number of
[Based on MAT (Dec), 2007] children in the school is
(a) 200 (b) 150
114.
Two men undertake to do a piece of work for `600. One
alone could do it in 6 days and the other in 8 days. With (c) 300 (d) 100
the assistance of a boy, they finish it in 3 days. The boy’s [Based on MAT (Feb), 2011]
share should be 120.
A and B together can do a piece of work in 12 days. B and
(a) `75 (b) `225 C together can do the same work in 16 days. After A has
(c) `300 (d) `100 been working at it for 5 days and B for 7 days, C finishes
it in 13 days. In how many days will C alone be able to do
[Based on MAT (May), 2007]
the work?
115.
15 men can complete a work in 210 days. They started the (a) 16 (b) 24
work but at the end of 10 days 15 additional men, with (c) 36 (d) 48
double efficiency, were inducted. How many days in all [Based on MAT, 1998]
did they take to finish the work?
121.
5 persons working eight hrs daily can complete a wall
1 3
(a) 72 days (b) 84 days 1
2 4 in 10 days. When they have worked for 5 days, 5
2
2 more persons are brought to work. The wall can now be
(c) 76 days (d) 70 days
3 completed in
[Based on MAT (May), 2007]
(a) one more day (b) two more days
116.
4 men and 10 women were put on a work. They completed (c) three more days (d) four more days
one-third of the work in 4 days. After this 2 men and 2 [Based on MAT, 1998]
women were increased. They completed two-ninths more
122.
Ramesh is twice as good workman as Sunil and finished
of the work in 2 days. If the remaining work is to be
a piece of work in 3 hrs less than Sunil. In how many hrs,
completed in 3 days, then how many more women must
they together could finish that piece of work?
be increased?
1
(a) 32 (b) 8 (a) 2 (b) 2
3
(c) 50 (d) 55
[Based on MAT (Dec), 2006] 2
(c) 1 (d) None of these
3
117.
C is twice efficient as A. B takes thrice as many days as C. [Based on MAT, 1999]
A takes 12 days to finish the work alone. If they work in
pairs (i.e., AB, BC, CA) starting with AB on the first day, 123.
If a person makes ‘J’ jobs in ‘M’ mins, how many jobs will
BC on the second day and AC on the third day and so on, he make in 2/3 hr?
then how many days are required to finish the work? 2 J 2M
(a) (b)
1 3M 3 J
(a) 6 days (b) 4.5 days
5 J M
(c) 40 (c) 40
1 M J
(c) 5 days (d) 8 days [Based on XAT, 2006]
9
[Based on MAT (Dec), 2006] 124.
A mother and a daughter working together can complete
a work in 4 days. But if the mother works alone, she can
118.
A can build up a wall in 8 days while B can break it in 3
complete the work in 6 days. Both of them worked for one
days. A has worked for 4 days and then B joined to work
day and then the mother had to leave. How long will the
with A for another 2 days only. In how many days will A
daughter take to complete the remaining work?
alone build up the remaining part of the wall?

Chapter_10.indd 237 2/1/2016 9:49:13 AM


238  Chapter 10

(a) 7 days (b) 8 days 132.


A worker is paid `56 for 35-hour week. Upto 40 hrs, he
(c) 9 days (d) 10 days is paid at the normal rate and on overtime, 1.5 times the
[Based on MAT, 1999] normal. How many hrs did he work to get `88?
125.
If 15 women or 10 men can complete a project in 55 days, (a) 48 (b) 52
in how many days will 5 women and 4 men working (c) 58 (d) None the these
together complete the same project? [Based on MAT, 2000]
(a) 75 (b) 95 133.
Aditya, Vedus and Yuvraj alone can do a job in 6 weeks, 9
(c) 55 (d) 85 weeks and 12 weeks respectively. They work together for
[Based on MAT, 1999] 2 weeks. Then, Aditya leaves the the job. Vedus leaves the
126.
24 men working at 8 hrs a day can finish a work in 10 job a week earlier to the completion of the work. The job
days. Working at the rate of 10 hrs a day, the number of would be completed in
men required to finish the same work in 6 days is (a) 4 weeks (b) 5 weeks
(a) 30 (b) 32 (c) 7 weeks (d) None of these
(c) 34 (d) 36 [Based on ITFT, 2009]
[Based on MAT, 1999] 134.
The digging work of the DMRC on the Adchini-
127.
A certain job was assigned to a group of men to do it Andheriamore stretch requires twenty-four men to
in 20 days. But 12 men did not turn up for the job and complete the work in sixteen days. As a part of the task if
the remaining men did the job in 32 days. The original DMRC were to hire thirty-two women, they can complete
number of men in the group was the same work in twenty-four days. Sixteen men and
(a) 32 (b) 34 sixteen wkomen started working the worked for twelve
days. Due to time bound schedule the work had to be
(c) 36 (d) 40
completed in remaining 2 days, for which how many more
[Based on MAT, 1999]
men are to be employed?
128.
12 men complete a work in 18 days. Six days af­ter they (a) 48 (b) 24
had started working, 4 men joined them. How many days
(c) 36 (d) 16
will all of them take to complete the remaining work?
[Based on IIFT, 2007]
(a) 10 days (b) 12 days
(c) 15 days (d) 9 days 135.
A is twice as good a workman as B and together they finish
a piece of work in 14 days. The number of days taken by A
[Based on MAT, 1999]
alone to finish the work is
129.
A, B and C can do a work in 20, 25 and 30 days respectively.
(a) 11 (b) 21
They undertook to finish the work together for `2,220,
(c) 28 (d) 42
then the share of A exceeds that of B by
[Based on FMS (MS), 2006]
(a) `120 (b) `180
(c) `300 (d) `600 1
136.
400 persons working 9 h per day th of the work in 10
[Based on MAT, 2000] 4
days. The number of additional persons, working 8 h per
130.
A and B can together do a piece of work in 15 days. B
day required to complete the remaining work in 20 days,
alone can do it in 20 days. In how many days can A alone
is
do it?
(a) 675 (b) 275
(a) 30 days (b) 40 days
(c) 250 (d) 225
(c) 45 days (d) 60 days
[Based on FMS (MS), 2006]
[Based on MAT, 2000]
131.
Three professors Dr. Gupta, Dr. Sharma and Dr. Singh 137.
A job is completed by 10 men in 20 days and by 20 women
are evaluating answer scripts of a subject. Dr. Gupta is in 15 days. How many days will it take for 5 men and 10
40% more efficient than Dr. Sharma, who is 20% more women to finish that work?
efficient than Dr. Singh. Dr. Gupta takes 10 days less than 1 1
(a) 17 (b) 17
Dr. Sharma to complete the evaluation work. Dr. Gupta 2 7
starts the evaluation work and works for 10 days and then 1
Dr. Sharma takes over. Dr. Sharma evaluates for next 15 (c) 17 (d) 17
120
days and then stops. In how many days, Dr. Singh can
complete the remaining evaluation work? [Based on FMS (MS), 2006]
(a) 7.2 days (b) 9.5 days 138.
A and B can do a piece of work in 12 days; B and C in 15
(c) 11.5 days (d) None of these days; C and A in 20 days. In how many days can A alone
[Based on IIFT, 2010] do it?

Chapter_10.indd 238 2/1/2016 9:49:13 AM


 Time, Work and Wages  239

(a) 30 (b) 24 (a) 1 (b) 3


2 (c) 6 (d) 12
(c) 15 (d) None of these
3 [Based on FMS, 2011]
[Based on FMS, 2005]
145.
Two candles of the same height are lighted at the same
139.
A certain job was assigned to a group of men to do in time. The first is consumed in 4 h and the second in 3 h.
20 days. But 12 men did not turn up for the job and the Assuming that each candle burns at a constant rate, in how
remaining men did the job in 32 days. The original number many hrs after being lighted was the first candle twice the
of men in the group was height of the second?
(a) 32 (b) 34 3 1
(a) h (b) 1 h
(c) 36 (d) 40 4 2
[Based on FMS, 2005]
2
140.
40 men can build a wall 20 m high in 15 days. The (c) 2 h (d) 2 h
5
number of men required to build a similar wall of [Based on FMS, 2011]
25 m high in 6 days will be
146.
A manufacturer builds a machine which will address 500
(a) 100 (b) 125
envelopes in 8 min. He wishes to build another machine
(c) 150 (d) 200 so that when both are operating together they will address
[Based on FMS, 2006] 500 envelopes in 2 min. The equation used to find how
141.
A can complete a job in 9 days, B in 10 days and C in many mintues x it would required the second machine to
15 days. B and C start the work and are forced to leave address 500 envelopes alone, is
after 2 days. The number of days taken by A to finish the 1 1 1
remaining work is (a) 8 – x = 2 (b)  =
8 x 2
(a) 8 (b) 9
500 500 x x
(c) 6 (d) 14 (c)  = 500 (d)  1
8 x 2 8
[Based on FMS, 2006]
[Based on FMS, 2011]
142.
Due to the recent global financial crisis, many companies
in the field of IT Services offer jobs on contractual basis 147.
Ramesh takes twice as much time as Mahesh and thrice
with a clause of fine for the leave taken by the employee. as much time as Suresh to complete a job. If working
An ITS company employs an engineer for 290 days on a together, they can complete the job in 4 days, then the
salary of `500 for eight hrs work per day, and the engineer time taken by each of them separately to complete the
has to pay a fine of `50 for each hour of his absence. work is
The engineer may compensate his one-day absence by (a) 36, 24 and 16 days (b) 20, 16 and 12 days
working 4 h extra for two days. If the engineer receives
(c) 24, 12 and 8 days (d) None of these
`132400 at the end of the contract, how many hrs was he
[Based on IIFT, 2005]
absent from his job?
(a) 110 hrs (b) 112 hrs 148.
A contractor take up an assignment that 20 men can
(c) 114 hrs (d) 118 hrs complete in 10 days. The same assignment could be
finished by 15 women in 20 days. The contractor decides
[Based on FMS, 2009]
to employ 10 men and 10 women for the project. Given
143.
Three machines P, Q and R, working together, can do a job this, mark all the correct options.
in x h. When working alone, P needs an additional 6 hrs to (a) If the wage rate for men and women are `50 and `45
do the job; Q, one additional hour; and R, x additional hrs. respectively, the total wage bill for the project will be
The value of x is `11400.
2 11 (b) If the wage rate for men and women are `45 and `40
(a) (b)
3 12 respectively, the total wage bill for the project will be
3 `10200.
(c) (d) 2 (c) If the wage rate for men and women are equal at `40,
2
[Based on FMS, 2010] the total wage bill for the project will be `9100.
144.
A and B together can do a job in 2 days; B and C can do (d) If the contractor decides to employ 20 men and 30
women for the project and the wage rate for men and
2
it in 4 days; and A and C in 2 days. The number of days women are `40 and `35 respectively, the total wage
5 bill for the project will be `9250.
required for A to do the job alone is [Based on IIFT, 2006]

Chapter_10.indd 239 2/1/2016 9:49:13 AM


240  Chapter 10

EXPLANATORY ANSWERS

1.
(b) 6 machines in 4 mins can produce 1080 bottles. 7.
(d) Suppose A and B are two men who can finish a certain
10 machines in 4 mins can produce work in 6 days and 8 days, respectively. Let the boy C
1080 can finish the same work in x days.
× 10 = 1800 bottles. 1 1 1 1
6 \ + + =
6 8 x 3
2.
(b) 160 bolts/20 s, i.e., 8 bolts/second, i.e., 200 bolts/25 s.
⇒ x = 24
3. (b) Let the number of workers be x.
⇒ `200 must be distributed among A, B and C in the
Then, total work = 9x man days.
1 1 1
As per the given condition, number of workers starting ratio : : , i.e., 4:3:1
6 8 24
with the first day will be
x, x – 2, x – 1, x – 3, x – 2, x – 4, x – 3 ... and so on till 1
\ C’s share = × 200
18 days. 8
So, separating the odd and even terms and adding = `25.
(9x – 36) + (9x – 54) = 9x
⇒ x = 10
1 15
4. (d) A and B together complete the work in 7
8.
(b) 2 = 2
1 2 2
= 24 days
1 1 3 3

40 60
15 3 45
Work completed in 20 days (4 days before scheduled
= × =
2 2 4
completion)
5 1
= 
1 1 = 11 .
  × 20 = th 4
 40 60  6
1 1 1 1
To complete the remaining work, 9.
(a) Given A + B = ,B+C= and C + A =
6 10 12 15
1
1 1 1 654
B will take 6 = 10 days ∴ 2(A + B + C) =   =
1 10 12 15 60
60 15 1
= =
∴ Number of days more to complete the work 60 4
= 30 – 24 = 6 days. 1
∴ A + B + C =
5. (c) Lal Singh can eat 12.5 laddoos in 1 hour and Pal Singh 8
can eat 7 laddoos in 1 hr Hence if A, B and C all work together, they will need
⇒ Together they eat 19.5 laddoos in 1 hr total 8 days to complete the work.
⇒ 507 laddoos will be eaten in 26 hrs.
10. (d) Suppose Rohit types x pages per hr.
6.
(a) Let Men = M, Women = W, Youngster = Y Harsha types y pages per hr
Then, 2M = 3W = 4Y Sanjeev types z pages per hr
2 1 \ z – y = y – x
⇒ W = M and Y = M
3 2 x+z
⇒ y = ...(1)
Given 14M + 12W + 12Y 2
2 1 ⇒ 5z = 7x ...(2)
= 14 + × 12 + × 12
3 2 4 (x + y + z) = 216
= 28 men ⇒ x + y + z = 54 ...(3)
⇒ 28 × 24 = M × 14 ⇒ M = 48 Solving Eqs. (1), (2) and (3), we get
Therefore, additional labour = 48 – 28 = 20 men y = 18, x = 15, z = 21.

Chapter_10.indd 240 2/1/2016 9:49:14 AM


 Time, Work and Wages  241

11.
(c) If A can do a work in 9 days then B can do the same 2
work in 6 days. (B being 50% more efficient than A). Remaining of the work is done by A alone in
3
1 2 20 2
12.
(b) (2M + 7C)’s one day’s work = 10 × = = 6 days.
4 3 3 3
\ 8M + 28C can finish the whole work.
17.
(d) (A + B + C)’s one day’s work
1
Also, (4M + 4C)’s one day’s work =
3 1 1 1 1+ 2 +3 1
= + + = =
\ (12M + 12C) can finish the whole work 18 9 6 18 3
\ 8M + 28C = 12M + 12C (A + B)’s one day’s work
⇒ M = 4C 1 1 1+ 2 1
= + = =
\ 4M + 4C = 5M 18 9 18 6
Since 5 Men can complete the work in 3 days 1
⇒ 1 Man can complete the work in 15 days. (A + B)’s 2 days’ work =
3
13. (b) Total number of seats = 500 Remaining 2/3 of the work is finished by A, B and C
No. of seats in Executive Class = 50 together in 2 days.
No. of seats in Chair Car = 450 \ Total number of days taken to finish the work = 4.
Total seats occupied = 85% of 500 = 425 18. (a) 140 × 40 × x = 100 × 18 × 42
Total seats occupied in Executive Class 100 × 18 × 42
⇒ x = = 18
= 96% of 50 = 48 140 × 30
Total seats occupied in the Chair Car \ Required no. of days = 18.
= 425 – 48 = 377 3 1
\ No. of seats vacant in the Chair Car 19.
(c) B’s 3 days work = =
21 7
= 450 – 377 = 73.
1 6
1 Remaining work = 1 – =
14.
(b) A’s one day’s work = 7 7
25
1 1 5
1 (A + B)’s 2 days’ work = + =
B’s one day’s work = 14 14 42
20
1 1 9 5
(A + B)’s one day’s work = + = work is done by A and B in 1 day
25 20 100 42
45 9 6 42 6 36
(A + B)’s five day’s work = = \ work is done by A and B in × = days
100 20 7 5 7 5
11
Remaining of the work is to be finished by B in 36 1
20 \ Total time taken = 3 + = 10 days.
11 days. 5 5
1 1 3
15.
(a) Ajit’s one day’s work = 20.
(d) 60 men in 25 days can do of work. For th of
12 4 4
1 3
Sujit’s one day’s work = work in 50 days, men required = 60 × = 90 men.
24 2
\ `180 are divided among them in the ratio Additional men = 90 – 60 = 30.
1 1
: , i.e., 2:1. 21.
(b) A man can finish a work in 6 days.
12 24
Work done by the man, woman and child in one day is
2
\ Ajit’s share = × 180 = `120. 1 1 1
3 , , respectively.
6 10 12
1 1 1
16.
(b) (A + B)’s one day’s work = + =
10 15 6 ∴ Work done in 1 day if each works for one-third of
1 1 1 1 1 7
\ (A + B)’s two day’s work = the day =   =
3 3  6 10 12  60

Chapter_10.indd 241 2/1/2016 9:49:14 AM


242  Chapter 10

60 4 30 4 3 2 1
∴ Number of days required = = 8 Hence, by 8.00 p.m. + + + + =1
7 7 40 40 40 40 40
56 Hence, they finish the work by 8.00 p.m.
In 8 days amount of work done =
60 26.
(a) 30 men can produce 1500 units in 144 hrs.
56 1
∴ Work left = 1  = 30 men can produce 1800 units in
60 15
1 1 144 864
In day a man will finish th of the work. × 1800 = hrs
3 18 1500 5
18 men can produce 1800 units in
1 1 1
∴ Work left =  = 864 30
15 18 90 × hrs = 288 hrs
5 18
∴ Woman would be working at that time. = 36 days of 8 hrs each.
22. (a) Let the number of men originally employed = x. x
27.
(b) 1 boy consumes calories in y days.
\ (x – 6) men could finish the work in 15 days and x 4
men could finish the work in 9 days.
z y
\ 9x = 15(x ­– 6) 1 girl consumes calories in days.
6 2
⇒ x = 15.
x
1 ∴ In 1 day, 1 boy consumes calories.
23.
(b) In 1 h, the man does th of the work. After 2 h he 4y
5
2z
2 ∴ In 1 day, 1 girl consumes calories.
has completed th of the work. In 20 min he does 6y
5
1 120 x 30x
th of the work so when he stops working the man In 12 days, 10 boys consume =
15 4y y
2 1  7 120 z 40x
has done    of the work, or in other words In 12 days, 10 girls consume =
 5 15  15 3y y
th of the work. Balance work remaining to be done is 30x 40z 10
∴ Total = + = (3x + 4z) calories.
7 8 y y y
1– =
15 15
28.
(d) To complete the work individually, let A, B and C,
8 respectively, take a, b, and c days.
The woman completes of the work in 100 mins.
15 5 2 3 1 1 1  1 1 3
∴   1 ,   and 2    
   She will complete the whole work on her own (or a b c b c 6  b c a
15 100 ⇒ a = 9.
of the work) in 15 × mins ≅ 185 mins.
15 8 2 3 5 1 1 1
  Man does it in 300 mins; hence the woman is Now,  = 1 – and  =
b c 9 b c 6
faster by 115 mins or 1 hour 55 mins.
2 3 4 1 1 1
3 3 3 ⇒  = and  =
24.
(b) Men can build wall in days. b c 9 b c 6
2 2 2
1 1
⇒ c = 9 and = ⇒ b = 18.
3 b 18
Men can build 6 walls in 6 days.
2 ∴ A, B and C, respectively, can complete the work in
6 Men can build 24 walls in 6 days. 9, 18 and 9 days.
1 1 80
25.
(d) Each engineer does = th of the process in 1 29.
(c) A man can reap hectares in one day. A woman can
58 40 24
hour. 60
30 reap hectares in one day. Together they can reap
Till 4.00 p.m., they have done of the process. 30
40
4 3 2 1 80 60 16
In next each hour,    .   =  hectares in one day.
40 40 40 40 24 30 3

Chapter_10.indd 242 2/1/2016 9:58:38 AM


 Time, Work and Wages  243

3 \ To type 110 pages, it will take


So, they will reap 150 hectares in  150
16 3 33
× 110 =
= 28.125 days. 40 4

30. (c) 4 examiners can examine certain number of Answer 1


= 8 hrs.
Books in 40 hrs. 4
Suppose for x hrs a day, 2 examiners examine the 36.
(b) Since 18 women can complete the work in 40 days,
same number of Answer Books in 10 days. therefore 1 woman can complete the work in 40 × 18
⇒ 4 examiners examine the same number of answer days = 720 days.
books in 5x hrs. 1
∴ 1 woman in 1 day will complete work
\ 5x = 40 ⇒ x = 8. (40  18)
31. (d) 15 men can do a piece of work in 168 hrs. 12
Now 12 women in 1 day complete work
3 women = 2 men (40  18)
⇒ 21 women = 14 men 12  20 1
Let 14 men can do the same piece of work in x days of 12 women in 20 days complete  work
(40  18) 3
6 hrs each i.e., 14 men can do the same piece of work
in 6x hrs. 1 2
∴ 16 men in 20 days will finish = 1   work
\ 6x × 14 = 15 × 168 3 3
⇒ x = 30. 2
∴ 1 man in 1 day can do work
32. (a) Suppose the original number of men was x. (3  16  20)
\ x men could do the job in 20 days and (x – 12) men 12 men and 27 women in 1 day can complete
could do the same job in 32 days.
  27 12  2  
\ 20x = 32 (x – 12)   
  (40  18) (3  16  20)   work
⇒ 12x = 32 × 12
⇒ x = 32. 5
= work in 1 day.
80
33.
(a) Niki can do a job in 4 mins, while Nishu can do the
same job in 5 mins. 80
∴ Time taken to complete the work =  16 days.
Then, in one min they can together do 5
1 1 9 3
 = of the work. 37.
(c) Book read on the first day =
4 5 20 8
5 5 5
Now, 375 = of 150. Hence, they can together do Remaining book to be read =
2 2 8
50
of the work in mins. 4 5 1
9 Book read on the second day = × =
5 8 2
1 1 1 73 5 1 1
34.
(a) Three day’s work = + + = Remaining book to be read = − =
7 8 6 168 8 2 8
73 1
Six day’s work = \ of the book contains 30 pages.
84 8
1 ⇒ The book contains 240 pages.
Seventh day work = , done by Ram
7
38.
(d) 20 men were employed to complete the work in say N
73 1 85 days. Therefore, the estimated work is 20N man days.
Since = + > 1 , therefore Ram was working
84 7 84 N
on the last day. Work completed in days is 5N man days.
3
32 40 16 40 Remaining time according to revised schedule
35.
(c) In one hour, + = +8 = of the pages N
6 5 3 3 3N 9N – 4N 5N
= – = =
are typed. 4 3 12 12

Chapter_10.indd 243 2/1/2016 10:02:51 AM


244  Chapter 10

Remaining work = 15N man days. 5


20 Men in N/3 days do 5N mandays of work. ∴ Total time = 50 + 5
7
Number of men needed in 5N/12 days, to do 15N = 55 hrs 43 mins.
 N   12   15  45.
(d) Mohan’s 20 days work = Bhim’s 30 days work
mandays of work is (20)    = 48
 3   5 N   5 
30
⇒ 28 additional men are needed. ⇒ entire work can be finished by Bhim in 25 
20
= 37.5 days.
39.
(c) (A + B + C)’s one day’s work
Bhim left after 15 days
1 1 1 5 1
= + + = = ⇒ Work of 22.5 days was remaining.
30 30 60 60 12
30
\ Work can be finished in 12 days. Mohan can complete it in  22.5 = 15 days.
20
3 1 1 6 + 3 +1 1 Questions 46 and 47
40.
(a) In 3 days, + + = = of the work
15 10 30 30 3 Let fencing the entire wall be equivalent to 108 units
is done by Sam, Bob and Kim. of work.
\ The whole job will be finished by them in 9 days. ⇒ Rakesh does 9 units per hour, Shiv does 86 units
2 per hour and Ramu does 4 units per hour.
41.
(a) rd of the work was completed in 64 days by In 2 hrs (9 + 6 + 4) × 2 = 38 units were done.
3
120 men. Remaining = 70 units.
1 In the last 5 hrs, Shiv has done = 5 × 6 = 30 units.
rd of the work was completed in 32 days by The other 40 units were done by Shiv and Vijay
3 40
120 men. together in = 4 hrs.
64
1
Also rd of the work is to be completed in 60 days by 46.
(a) Total number of hrs taken = 2 + 5 + 4 = 11 hrs.
3
(120 – x) men, where x is the number of men reduced 47. (a) The pair which does the work in least time, will be
in order to finish the work on schedule. able to fence the wall quicker than any other pair
\ (120 – x) × 60 = 32 × 120 ⇒ x = 56 ⇒ Rakesh and Shiv
M received 5% of `7400 extra for being the working 48.
(b) Percentage of job completed by A in 1 day
member, i.e., `370.
1
42. (b) Let A got `x per day and B got `y per day. = = 5%
20
\ 10x + 9y = 225 and 5x = 3y + 15
Percentage of job completed by B in 1 day
⇒ x = 10.80, y = 13.
1
43.
(d) Time taken by child is twice the time taken by man = = 8.33%
12
4 + 12 = 16 men do the work in 9 days
Percentage of work completed in first two days
16
\ 12 men can do it in = 9 × = 12 days = 5 + 8.33 = 13.33%
12
Work completed in 14 days (7 for each A and B)
44.
(d) Imran can stitch 10/3 shirts/hr.
= 13.33 × 7 = 93.31%
Irfan can stitch 10/4 shirts/hr.
On 15th day (B will work) = 93.33 + 8.33 > 100
Imran can stitch 3 pants/hr.
So, the work will be completed by B on 15th day.
Irfan can stitch 4 pants/hr.
4 1
Now the work will be done fastest when Irfan stitches 49.
(b) Work done by C = 1  =
the pants whereas Imran stitches the shirts. But Irfan 5 5
is finished with 200 pants in 50 hrs, while Imran has 1
stitched only (10/3) (50) = 500/3 shirts by then. So the ∴ C’s amount = × 4500 = `900
5
remaining 100/3 shirts are stitched by both in
50. (c) Given the ratio of working rates on the three types of
  vegetables as 120:72:60 i.e., 10:6:5.
100  1  40 5
= =5 Suppose, he cuts N vegetables in the first K mins
3  10  10  7 7
 3 4 K
⇒ mins was spent cutting say 10x carrots.
= 5 hrs 43 mins 3

Chapter_10.indd 244 2/1/2016 9:49:16 AM


 Time, Work and Wages  245

K 55. (b) Go through option


mins was spent cutting 6x potatoes 140 × 4 = (140 + 120 + 100 +...+ 200)
3
K 560 = 560
and mins was spent cutting 5x beetroots. Alternatively: Let n be the initial number of workers
3
then n + 4 = n + (n – 20) + (n – 40) + ... + (n – 120)
During the next K mins, the time distribution was in
1 1 1 4n = 7n – 420
the ratio : : i.e., 6:10:12 = 3:5:6 ⇒ 3n = 420
10 6 5
⇒ n = 140 workers
3 5 6
Hence K, K and K 56. (b) Amount of work to be done = 10n, where n = Number
14 14 14
of workers originally available.
3  Now 10n = 12(n – 5) ⇒ 2n = 60
 K  90 x
14
⇒ Carrots cut  10 x = 3 carrots Therefore, n = 30
K
  14
  1 1 1 1
3 57.
(b)   = ... (1)
a b c 18
90 x 90 x
Similarly potatoes and beetroots are cut. 1 1 2
14 14  = ... (2)
a b c
Total number of vegetables cut in the first 2K mins
 90 x  1 1 3
= 10x + 6x + 5x +   13 = 282  = ... (3)
 14  a c b
90 x 1 1 2
⇒ x = 7 and = 45 (1) and (2)  ⇒    or c = 54 days
14 18 c c
1 1 3
51. (a) 35 men do the rest of the job in 12 days (37 – 25 = 12) (1) and (3)  ⇒    or b = 72 days
18 b b
12  35
∴ 30 men can do the rest of the job in ∴ a = 43.2 days
30
= 14 days. 58. (b) Let B completes 1 work in 2x days then A will
complete the work in x days given, 2x – x = 30 ⇒ x =
Thus the work would have been finished in (25 + 14)
30 days
= 39 days.
∴ work done by both of them working together
That is, (39 – 38) = 1 day after the scheduled time.
1 1 1
=    work.
52.
(c) 15m = 24w = 36b 30 60 20
x men must be associated. So, they can do it working together in 20 days.
∴ (x)m + 12w + 6b =  x  18  6 boys.
36 59. (c) 1996 is a leap year. Hence Raja takes 7 days to

 15  complete a work. He completes 1 unit work per day,
2 hence work completed in 7 days = 7 units. (Because
53.
(c) Work done = he does not have any holiday). Now T will have two
3
holidays in a week i.e., Tuesday and Thursday and S
1 2
Remaining work = , which is half of will not have any holiday. Hence their arrangement
3 3 of work will follow the following pattern depending

1
 × (20 × 32) = 8 × x upon which day 25 Feb 1996 falls on
2
Sun Mon Tues Wed Thurs Fri Sat
⇒ x = 40 men
2 units 2 units 1 unit 2 units 1 unit 2 units 2 units
Therefore, 20 more men were required.
Hence both of them will take either 4 or 5 days to
1
54.
(c) Work done = complete the same work.
5
60. (c) Raja has worked 38 days. (Feb. = 5 days, March =
4
Remaining work = 31 days, April = 2 days), Therefore, he completes
5 38 units work in 38 days. In a week T takes holidays
∴ 4 (20 × 75) = 40 × x on Tuesday and Thursday, while S takes holiday on
x = 150 Saturday and Sunday. Hence, their work arrangement
Therefore 75 men should be increased. will follow the following pattern:

Chapter_10.indd 245 2/1/2016 9:49:17 AM


246  Chapter 10

Sun Mon Tues Wed Thurs Fri Sat 65.


(a) If X can complete the work in a days then
3a – a = 40 → a = 20
1 unit 2 units 1 unit 2 units 1 unit 2 units 1 unit
\ Work done by (X + Y) for
Therefore, in a week they work 10 units work. Hence,
in 3 weeks, they would complete 30 units work. Now 1 1 4 1
1 day + = =
8 units work can be completed either on 5th or 6th day 20 60 60 15
depending on which day the work begins. Hence, total \ X and Y together will complete the work in 15 days
number of days taken by T and S to complete the job
= (21 + 5) = 26 days or (21 + 6) = 27 days. 66.
(a) Work done by A for 3 days

61. (d) Ram 3 1


= =
12 4
\ Remaining work
1 3
= 1 − =
4 4
10
In 1 hour Ram is at B, in that time Shyam covers 3 1 1
4 \ Work done by (A + B) for 1 day = × =
= 2.5 km 4 3 4
Remaining distance = 2.5 km \ Work done by B for 1 day
2.5 1 1 2 1
Time =  60 = 10 mins = − = =
5  10 4 12 12 6
Therefore, they meet first time at 10.30 a.m. \ B alone will complete the work in 6 days

62. (b) At the time when Shyam overtakes Ram, let Ram 67.
(b) Let the no. of days taken by A to complete the work be
travels for t mins, Shyam till that time travel for t – 45 x days.
mins and both travel same distance. Therefore, days taken by B to complete the same
⇒ 5 × t = 10 (t – 45) = 3x days.
⇒ t = 90 mins So, 3x – x = 60
Hence, Shyam overtake Ram at 10.30 a.m. ⇒ 2x = 60
⇒ x = 30
63. (d) One spindle can produce 9 metres cloth in r mins
and 3x = 3 × 30 = 90
r
One spindle can produce 1 metres cloth in min Therefore, (A + B)’s 1 day’s work
q
r 1 1 3 +1 4 2
On spindle can produce 20000 metres cloth in × = + = = =
q 30 90 90 90 25
20000 min. Hence, A and B together will do the work in
\ p spindles can produce 20000 metres cloth in 45 1
= 22 days.
r r  2000 2 2
× 20000 = hours
pq 60  pq
1
68.
(c) (P + Q)’s 1 day’s work = ...(1)
Work Days Men 12

` 64.
(c) 1 12 7 1
↓ ↓ ↓ (Q + R)’s 1 day’s work = ...(2)
2 8 x 15

Therefore, 1
(R + P)’s 1 day’s work = ...(3)
1 : 2 ::7:x 8:12 20
⇒ 1 × 8 × x = 2 × 12 × 7 Adding equations (1), (2) and (3), we get
8x = 168 2(P + Q + R)’s 1 day’s work
168 1 1 1 5+4+3
x = = 21 = + + =
8 12 15 20 60
Hence, no. of additional men 12 1
= =
= 21 – 7 = 14 60 5

Chapter_10.indd 246 2/1/2016 9:49:18 AM


 Time, Work and Wages  247

\ (P + Q + R)’s 1 day’s work Remaining work


1 1 − 35 36 − 35 1
= ...(4) = = =
10 36 36 36
Therefore, P’s 1 day’s work on subtracting Eq. 4 from Therefore, time taken by
Eq. 2, we get
1 1
1 1 3−2 1 A = ×9 = day
= − = = 36 4
10 15 30 30
1
Hence, P will take 30 days to complete the work. Hence, total time = 10 +
4
69.
(c)
Men Days 40  1 41 1
= = = 10 days
x 30 4 4 4

x+6↑ 20 ↓ 72. (b) Let daily wage earner absents x days then as per
question,
x + 6:x:: 30:20
60 × 150 – 175 × x = 7600
x+6 30 3
⇒ = = ⇒ 9000 – 175x = 7600
x 20 2
⇒ 175x = 1400
⇒ 2(x + 6) = 3x
\ x = 8
⇒ 2x + 12 = 3x
Hence, daily wage earner worked 52 days.
⇒ 3x – 2x = 12
73. (a) Males : Females : Children
⇒ x = 12 men
6 × 12 : 8 × 18 : 18 × 10
70.
(a) Let the no. of days taken by B to complete the work 72 : 144 : 180
be x 2 : 4 : 5
Therefore, no. of days taken by A to complete the So, 2 Males = 4 Females = 5 Children
work
4 Males + 12 Females + 20 Children
3x 1
=  = 4 + 6 + 8 = 18 Males
4 2
Then time taken by A to complete the work Q 6 males finished a piece of work in 12 days.
3x 3x 12 × 6
= 2 × = days \ 18 males finished the work = = 4 days
4 2 18
Thus, (A + B)’s 1 day’s work 2 1
Work in 2 days = =
1 2 3+ 2 5 4 2
= + = =
x 3x 3x 3x Rest of the work will be finished in a day by
5 1 = 18 × 2 = 36 males
⇒ =
3x 18 74. (c) Let Ramesh read x pages/none of engineering Math
⇒ 3x = 90 and y pages/none of engineering drawing.
90 A/Q in a day he can read
⇒ x = = 30
3 80 100 50 250
Total time = + = +
Hence, time taken by B to complete the work = 30 x y x y
days. 30 150
⇒ =
71.
(c) Portion of work done by A and B in first in two days x y
⇒ y = 5x
1 1 4+3 7
= + = = Since, he has to read only engineering maths in a day
9 12 36 36
putting y = 5x in above equation, we get
portion of work done in first
50 250 100
35 + =
10 days = x 5x x
36 He can read 100 pages of maths in total time.

Chapter_10.indd 247 2/1/2016 9:49:19 AM


248  Chapter 10

75. (c) Suppose A can finish the work in x days and B can 1
finish it in y days. 80.
(b) A’s one day work =
40
2 9 3 6
\ + = 1 and + = 1 5 1
x y x y A’s five days work = =
40 8
On solving, we get x = 5 and y = 15
1 7
76. (a) Q 12 men in 36 days can do a work. \ Remaining work = 1 − =
8 8
1
\ 1 man in a day can do work \ B’s completed the remaining work in 21 days
12 × 36
7
=
8 × 20 10 8
\ 8 men in 20 days can do = work
12 × 36 27
7 1
Similarly we find that 20 women in 20 days can do 1 day work of B = =
8 × 21 24
10
work. \ Both (A + B)’s one day work
27
7 1 1 3+5
\ Remaining work is . = + =
27 40 24 120
Now because in 60 days a work is done by 20 women. 8 1
= =
\ in 1 day a work is done by 20 × 60 women. 120 15
20 × 60 × 7 Hence, both complete the work in 15 days.
\ in 4 days 7/27 work is done by
27 × 4 81.
(d) Let A can finish the work alone in x days.
= 70 women 1
But work of (A + B)’s =
77. (b) 2M = 3W 30
3 1 1
\ 1M = W \ One day work of B alone = −
2 30 x
3 5 1
\ 1M + 1W = W + 1W = W So, one day work of A =
2 2 x
3× 4 24 16
Number of days = = days 16 days work of A =
5/2 5 x
78. (b) (8 × 4) Girls = (9 × 3) Boys = (7 × 2) Men = (5 × 4) 16
Women \ Remaining work = 1 –
x
⇒ 32 Girls = 27 Boys = 14 Men = 20 Women
Hence, Girls have minimum capacity of work among  1 1
44 days work of B = 44  − 
them.  30 x 
79. (c) Suppose A does a work in x days 16  1 1
\ 1− = 44  − 
x x x  30 x 
B does similar work in days and C does in days.
2 4 16 44 44
⇒ 1− = −
x 7 x 30 x
Q =
4 1 28 14
⇒ x = 4 × 7 ⇒ =
x 30
x = 28
28 × 30
1 1 1 1+ 2 + 4 ⇒ x =
Q + + = 14
28 14 7 28
⇒ x = 60 days
7 1
= = 1 1 1
28 4 \ One day work of B alone = − =
So, A, B and C together will complete the work in 4 30 60 60
days. Hence, B can do this work in 60 days.

Chapter_10.indd 248 2/1/2016 9:49:20 AM


 Time, Work and Wages  249

6 3 M1D1 M D
82.
(c) Work done by Manoj for 6 days = = 86.
(c) = 2 2
10 5 W1 W2
3 2 18 × 20 12 × 15
Remaining work = 1 − = ⇒ =
5 5 12 W2

2 2  10 × 12  12 × 15 × 12
Manoj and Ajita can finish work in ×   ⇒ W2 = =6m
5 5  10 + 12  18 × 20
days.
87.
(b) Let the work be finished in x days.
2  10 × 12 
So, Ajita actually work for ×   days x ( x − 1) 2
5  10 + 12  Then, 1 = + +
8 16 24
2 120 2
= × =
2 days 11 2x + x − 1
5 22 11 ⇒ =
12 16
83.
(a) Let Ashokan can finish the work in x days. Then, Nitin
16 × 11
can finish the work in 3x days. ⇒ 3x – 1 =
12
3x – x = 40
⇒ x = 20 days 47
⇒ x = ≈ 5 days
and 3x = 60 days 9

 20 × 60  88.
(a) Let Asha can complete the work in x days and Sudha
So, together they can finish the work in   can complete in y days.
 20 + 60 
days =15 days x× y
Then, = 20
84.
(d) (A + B)’s complete the work in x days. x+ y

But A complete the work in (x + 8) days. xy
⇒ x + y = …(1)
 9 20
But B complete the work in  x +  days.
 2 1 1
and x + y = 45
 9 2 2
( x + 8)  x +  ⇒ x + y = 90 ...(2)
Using formula,  2
=x
25 From Eqs. (1) and (2),
2x +
2 xy = 1800
9 25 Now, go through options
⇒ x2 + 8x + x + 36 = 2x2 + x
2 2 xy = 60 × 30
⇒ x2 = 36 ⇒ x = 6 days As Sudha is more efficient than Asha.
So, Asha can complete the work in 60 days.
85.
(c) Let Madhu can complete the work in 2x days. Then,
Uma can complete the work in x days. Together they 89.
(c) Let number of computers required = N
can complete the work in Tasks done by the computers
 2x × x  2 6 h = 30 tasks
  days = x days 1 h = 5 tasks
 2x + x  3
3 h = 15 tasks
2
So, Ramesh can complete the work in x days. So, 15N = 80
3
⇒ N = 5.33 ≈ 6
1 1 3 1
Now, + + =
2x x 2x 6 90.
(c) Total work = 124 × 120 = 14880 men-days
Work completed in 64 days
6 1
⇒ = 2
2x 6 = × 14880 men-days
3
⇒ 2x = 36
= 9920 men-days
So, Madhu can complete the work in 36 days.

Chapter_10.indd 249 2/1/2016 9:49:20 AM


250  Chapter 10


Remaining work for 60 days
Total women and boys in terms of men
1 15 5 20
= × 14880 = + = = 10 men
3 2 2 2
= 4960 men- days Let number of men required = x
M1D1 M D 15 × 12 × 8 × 2.25
Q = 2 2 Then, (x + 10) = = 18
W1 W2 30 × 6

120 × 64 M × 60 ⇒ x + 10 = 18
\ = 2
9920 4960 ⇒ x = 8 men
⇒ M2 = 64 3
\ Workmen to be reduced 94.
(c) 27 cogs turns 80 times in
× 60 min = 45 s
4
= 120 – 64 = 56 27 cogs – 45 s = 80 times
1 27 × 80 × 8
91.
(a) Anu’s 1 day work = \ 16 cogs – 8 s = = 24 times
10 16 × 45
125 1 1 3
Manu’s 1 day work = × = 95.
(d) Speed of X =
Y
100 10 8 2
160 1 1 Distance before catching = D m
Sonu’s 1 day work = × = 3
100 8 5 D = D + 300
2
1 1 1 17
Total work = + + = 1
10 8 5 40 D = 300
2
40 6
Total days = = 2 days ⇒ D = 600 m
17 17
Total distance = 600 + 300 = 900 m
92. (a) 1 male = 2 females 1
8 males and 4 females = 20 females 96. (b) B and C together can complete a work in =
8
After 2 days 4 males have left the work and 4 new
1
females jointed at their place = 4 males + 8 females A and B together can complete a work in =
12
= 8 + 8 females
1
= 16 females A and C together can complete a work in =
16
M1 = 20 females Work completed by 2(A + B + C) in a day
D1 = 6 – 2 = 4 days 1 1 1
M2 = 16 females = + +
8 12 16
D2 = ?
6+4+3 13
M1D1 = M2D2 = =
48 48
20 × 4 = 16 × D2
Work completed by (A + B + C) in day

20 × 4
D2 = 13 13
16 = =
48 × 2 96
D2 = 5 days
96
93.
(d) Men:Women:Boys = 15:24:36 So, A, B and C together can complete the work in
13
= 5:8:12
5
Convert women and boys in terms of men days = 7 days.
13
8 women = 5 men
97.
(b) Let they make x pieces per day.
5 15
12 women = × 12 = men 360 360
8 2 Then, − =1
x x+4
12 boys = 5 men
5 5  4 
6 boys = ×6= men ⇒ 360   =1
12 2  x ( x + 4) 

Chapter_10.indd 250 2/1/2016 9:49:21 AM


 Time, Work and Wages  251

⇒ x(x + 4) = 1440 = 36 × 40 5 5
⇒ x = 36 \ 12 W + 6 B = × 12 + × 6 = 10 M
8 12
360 Now, m1 × d1 × t1 × w2 = m2 × d2 × t2 × w1
\ Required number of days = =10 days
36 Let the number of additional men required be x.
98.
(b) 8 women = 6 men =12 boys
9
6 15 × 12 × 8 × = (10 + x) × 30 × 6
\ 12 women = × 12 = 9 men 4
8
6 15 × 3 × 8 × 9
12 boys = × 12 = 6 men ⇒ (10 + x) =
12 30 × 6
\ M1 D1 T1 = M2 D2 T2 ⇒ x = 18 – 10 = 8
\ 9 × 6 × 6 = (12 + 9 + 6) × D2 × 8 104. (d)
9×6×6 1 Men Women Children
⇒ D2 = = 1 days 18 × 63 = 9 × 189 =7 × 486
27 × 8 2
=
14 M =21 W 42 Children
1 1 1
99.
(d) C’s 1 day’s work = − = \ 1 M = 3 Children
12 20 30

Ratio of their efficiencies 1 W = 2 Children
1 1 1 1 (4M + 9W + 12C) = (12 + 18 + 12) Children
= : : : = 42 Children
32 20 30 24
= 15:24:16:20 7 × 486 = 42 × x
⇒ x = 81 days
16 16
\ C’s share = × 25 =
` 105.
(d) Let the length of the path be x km.
15 + 24 + 16 + 20 3
x 
100. (b) 10 W × 15 = 6 M × 10 Work done by 3 men in 2 days =  − 1 km
5 
15 5
\ 1M = = W
6 2  x − 5
\ Work done by 1 man in 1 day =   km
Now, 5 W + 6 M = (5 + 15) W = 20 W  30 
10 × 15 = 20 × x day
 2x 
\ x = 7.5 days And work done by 18 men in 1 day =  + 1 km
 5 
101.
(b) Ratio of efficiency = 2:3:6
1 1 1  2x + 5 
Ratio of time required = : : = 3 : 2 :1 \ Work done by 1 man in 1 day =   km
2 3 6  90 
Let A, B and C can complete the work in 3x, 2x and x  x − 5  2x − 5 
Given,   =  
days respectively.  30   90 
3x × 2 x × x ⇒ 3(x – 5) = 2x + 5
Then, = 10
3x × 2 x + 3x × x + 2 x × x ⇒ x = 20 km

6 x3 106.
(c) Q M1 × D1 × T1 × W2 = M2 × D2 × T2 × W1
⇒ = 10 \ 2 × 12 × 8 × 12000 × 0.9
11x 2
⇒ x ≈ 20 days = 3 × 6 × x × 9000 × 0.8
2 × 12 × 8 × 12000 × 0.9
102.
(a) 2 children = 1 man ⇒ x =
3 × 6 × 9000 × 0.8
\ (4 + 12) men can complete the work in 9 days.
= 16 h/day
9 × 16
\ 12 men can complete the work in
107.
(b) Q M2 × D1 × T1 × W2 = M2 × D2 × T2 × W1
12
= 12 days. \ 36 × 6 × 10 × 1200 = 10 × d2 × 8 × 1200
36 × 6 × 10 × 1200
103.
(b) 15 M = 24 W = 36 B \ d2 =
10 × 8 × 1200
5 5
⇒ 1 W = M and 1 B = M = 27 days
8 12

Chapter_10.indd 251 2/1/2016 9:49:21 AM


252  Chapter 10

108.
(d) Let Bhavika and Rita working together can finish a 112. (a) 8 × 20 men = 8 × 32 women
job in x days. 5 men = 8 women
1 Now, 5 men + 8 women = 8 + 8 = 16 women
Then, in 1 day they will complete = work
x
D1 × M1 = M2 × D2
Bhavika working alone will do in 1 day
8 × 32 women = 16 × D2
1
= work 32 × 8
( x + 8) D2 =
16
Rita working alone will do in 1 day = 16 days
1 113. (a) A and B can complete the work in 30 and 40 days
= work
 9 respectively.
 x + 
 2 1 1
So, ratio of their wages = : = 40:30
1 2 1 30 40
\ + =
( x + 8) (2 x + 9) x 30
\ Share of B = × 2100 = `900
(2 x + 9) + 2( x + 8) 1 40 + 30
⇒ =
( x + 8)(2 x + 9) x 114.
(a) Ratio of efficiencies of the three persons

4 x + 25 1 24 34 24  24 24 
⇒ = = : : − + 
( x + 8) (2 x + 9) x 6 8 3  6 8 

⇒ x = 6 days = 4:3:1
1
1 \ Boy’s share = × 600 = `75
109.
(c) Work done by 1 man in 1 day = (4 + 3 + 1)
100
1 115.
(c) Total time taken to finish the work
Work done by 1 woman in 1 day =
120 15(210 − 10) 15 × 200
Work done by 15 men and 6 women = 10 + = 10 +
(15 + 15 × 2) 45
15 6
= + 200 2
100 120 10 + = 76 days
3 3
3 1 4 1
= + = = work 116.
(b) Let one man takes x days to complete the work
20 20 20 5
and one woman takes y days to complete the work
\ 15 men and 6 women will take 5 days to complete independently.
the work.
4 × 4 10 × 4 1 6 × 2 12 × 2 2
M1 D1 T1 M D T Then, + = and + =
110.
(a) Q = 2 2 2 x y 3 x y 9
W1 W2
Solving above equations, we get
5 × 10 × 8 8 × D2 × 6
\ = x = 108, y = 216
100 300
Let z women be added to complete the work in 3 days.
5 × 10 × 8 × 30 1
⇒ D2 = = 2 days 6 × 3 3(12 + z ) 1 2 4
100 × 8 × 6 2 Then, + = 1−  + =
108 216 3 9 9
2m 216 × 4
111.
(c) One worker of category I can finish the work in h. ⇒ 36 + 36 + 3z = = 96
3 9
Two workers of category II can finish the work in ⇒ 3z = 96 – 72 = 24
m ⇒ z = 8
h.
2
117. (c) A B C
2m m
× 2 Efficiency 3 : 2 : 6
So, required time = 3 2 = 2m = 2 m h
2m m 7m 7 No. of days 2 : 3 : 1
+
3 2 ⇒ Number of days taken by A = 12,

Chapter_10.indd 252 2/1/2016 10:06:34 AM


 Time, Work and Wages  253

Number of days taken by B = 18 and 1


120.
(b) (A + B)’s one day’s work =
   Number of days taken by C = 6 12
5 1
1 day’s work of (A + B) = (B + C)’s one day’s work =
36 16
8 Suppose C alone can finish the work in k days.
1 day’s work of (B + C) =
36 1
⇒ C’s one day’s work =
k
9
1 day’s work of (C + A) = 1 1 k − 16
36 \ B’s one day’s work = − =
16 k 16k
Day 1 2 3 4 5 6
] 1 k − 16 k + 48
5/36 8/36 9/36 5/36 8/36 1/36 ⇒ A’s one day s work = − =
 12 16k 48k
35/36
5(k + 48)
\    A’s 5 days’ work =
35 48k

In 5 days total work done =
36 7(k − 16)
   B’s 7 days’ work =
 1  16k
Now, the rest of the work I  i.e.,  is done by AC
 36  \ Remaining work to be done by C
Number of days taken by AC for the rest of the work  5(k + 48) 7(k − 16)  11k + 48
= 1−  + =
 48k 16k  24k
1/36 1
= = which is finished by him in 13 days.
9/36 9
1
Therefore, total time taken to complete the work Now in one day, C does of the work.
k
1 1 13
= 5 + =
5 days In 13 days, C does of the work.
9 9 k
13 11k + 48
4 1 \ = ⇒ k = 24
118.
(c) Work done by A in 4 days = = k 24k
8 2
1
 1 1  −5 121.
(c) 5 persons’ one day job =
Net work done by (A + B) in 1 day  −  = 10
 8 3  24
1 1 11 11
−5 −5 5 persons’ 5 days job = × =
Work done by (A + B) in 2 days = ×2= 2 10 2 20
24 12
9 1
1 5 1 Remaining of the job is done by 10 persons. of
\ Work done in 6 days = − = 20 5
2 12 12 9
the job is done by 10 persons in 1 day. of the job is
11 20
\ Remaining of the wall is built by A in
12 1
done by 10 persons in 2 days.
8 × 11 88 22 1 4
= = = 7 days
12 12 3 3 122.
(b) Let Sunil finish the job in x hrs
x
119.
(b) Let the number of rows and columns be x and y. Then, Ramesh will finish the job in hour
2
\ (y – 4)(x + 10) = xy x
Given, x− =3⇒x=6
⇒ 10y – 4x = 40 ...(1) 2
and (y + 5) (x – 5) = xy \ Sunil finishes the job in 6 hrs and Ramesh does it in 3
hrs.
⇒ 5x – 5y = 25
Work done by both of them in 1 hour
⇒ x – y = 5 …(2)
1 1 1
⇒ x = 15, y = 10 = + =
6 3 2
\ Total number of students = 15 × 10 = 150
\ They together could finish the job in 2 hrs.

Chapter_10.indd 253 2/1/2016 9:49:23 AM


254  Chapter 10

123.
(c) In M mins the person makes J jobs. 6 1
128. (d) 12 men’s 6 days’ work = =
M 18 3
In h the person makes J jobs
60 2
J  60 Remaining of the work is done by 16 men.
In 1 h the person makes jobs 3
M
16 men can complete the work in
2 J  60 2
In h the person makes  jobs 12 × 18 1 27
3 M 3 = 13 = days
16 2 2
2 40J
In h the person makes = jobs 2
3 M i.e., 16 men can do of the work in 9 days
3
1
124.
(c) (M + D)’s one day’s work = 1
4 129.
(b) A’s 1 day’s work =
20
3
Remaining work = 1
4 B’s 1 day’s work =
25
1
M’s one day’s work = 1
6 C’s 1 day’s work =
30
1 1 1 A, B, C’s ratio of work
D’s one day’s work = − =
4 6 12
1 1 1
= : :
1 20 25 30
\ th of the work is done by the daughter in one day.
12 = 30:24:20
3 = 15:12:10
th of the work is done by the daughter in
4 Sum of ratio = 15 + 12 + 10 = 37
12 × 3 Hence, difference between A and B’s shares is
= 9 days.
4 15 12
= ` × 2200 – ` × 2220
37 37
125.
(a) 15W = 10M ⇒ 3W = 2M
3
4 M = 6W = × 2220 = `180
37
\ 5W + 4M = 11W
If 15W can do the job in 55 days, 11W will do the 1
130.
(d) (A + B)’s 1 day’s work =
15
55 × 15
same job in = 75 days.
11 1
B’s 1 day’s work =
20
126.
(b)
1 1 1
\ A’s 1 day’s work = − =
15 20 60
Hence, A alone can finish the work in 60 days.
6 :10  131.
(a) As per the given information, the ratio of the
 : 24:x
10 : 8  efficiencies of Dr. Gupta, Dr. Sharma and Dr. Singh
are = 42:30:25. Hence, the ratio of time taken by Dr.
or, 6 × 10 × x = 10 × 8 × 24 Gupta and Dr. Sharma is 5:7. As, Dr. Gupta takes 10
10 × 8 × 24 days less than Dr. Sharma, time taken by them will
\ x = = 32 men
6 × 10 be 25 days and 35 days respectively. Hence, the time
taken by Dr. Singh will be 42 days.
127. (a) Suppose x = original number of men in the group 2
Part of the work completed by Dr. Gupta =
\ (x – 12) men did the job in 32 days. 5
\ 20x = 32(x – 12) 3
Part of the work completed by Dr. Sharma =
i.e., x = 32 7

Chapter_10.indd 254 2/1/2016 9:49:23 AM


 Time, Work and Wages  255

6 1 1 1
The remaining work i.e., will be completed by \  =
35 x 2x 14
Dr. Singh in ⇒ x = 21 days
42  6 3 1
= 7.2 days 136.
(a) 400 × 9 × 10 × = 8 × 20 × ×x
35 4 4
132.
(d)  Let the worker worked for x hrs. ⇒ x = 675
Now, according to the question,
137.
(b) M × 10 × 20 = 20 × W × 15
56 56 3

\ × 40 + (x – 40) × × = 88 2M = 2W
35 35 2
3 15
\ 5M = × 5 =
56  3  2 2

⇒ 40 + x − 60  = 88
35  2  From question,
3 88 × 35  15 

⇒ x = + 20 = 75 x   10 = 20 × 15
2 56 2 


⇒ x = 50 120
⇒ x =
133.
(d) 2 weeks work of Aditya, Vedus and Yuvraj 7
 1 1 1  26 1
= 2      ⇒ x = 17 days
 6 9 12  36 7
10 1 1 1
\ work left = 138.
(a) 2( A  B  C ) =  
36 12 15 20
Let Vedus works for x weeks with Yuvraj after Aditya
1
1 1  7x ⇒ A + B + C =
\ x weeks’s work of both = x    , the 10
 9 12  36
remaining work is done by Yuvraj in 1 week. 1 1 1
\ A can do alone =  =
10 15 30
1 Hence A can alone complete the work in 30 days.
\ One week’s work of Yuvraj =
12 139.
(a) Let original number of men = x
26 7 x 1
\   =1 \ x × 20 = (x – 12) × 32
36 36 12
⇒ 20x = 32x – 384
⇒ 29 + 7x = 36 ⇒ x = l
⇒ 12x = 384
\ Vedus worked for 1 week with Yuvraj.
⇒ x = 32
134.
(b) 16 men and 16 women together in 12 days did the
40  15 6x
1 3 140.
(b) =
work =  12 work 20 25
16 4
⇒ x = 125
3 1
Remaining work = 1  
4 4 x x 2
141.
(c)   =1
16 men and 16 women in two days will do the work 9 10 5
1 1 ⇒ x = 6
=  2  work
16 8 142.
(b) Total working hrs = 290 × 8 = 2320
1 1 1 500
Men needed to do remaining work =   work Wages for one hour work = `
4 8 8 8
24  16 Let he was absent for x hrs.
Required men =  24
28 500
\ His wages = (2320 – x) – 50 × x
135.
(b) Work → A : B 8
2x : x 1160000  900 x
⇒ 132400 =
1 1 8
Time → :
2x x ⇒ x = 112 h

Chapter_10.indd 255 2/1/2016 9:49:24 AM


256  Chapter 10

143.
(a) P takes (x + 6) hrs to do the job alone. 146.
(b) The first machine addresses 500 envelopes in 8 min.
Q takes (x + 1) hrs to do the job alone. 500
In 1 min, it will address = envelopes
R takes (2 x) hrs to do the job alone. 8
The three take x hrs to do the job, when working Let the second machine address 500 envelopes in x
together. min 500
1 1 1 1 500
\   = In 1 mm, it will address envelopes
x  6 x  1 2x x x

2x  7 1 Thus, both the machines together in 1 min will address
⇒ =
2
x  7x  6 2x 500 500
+ envelopes
8 x
⇒ 4x2 + 14x = x2 + 7x + 6
⇒ 3x2 + 7x – 6 = 0 Given, both the machines together will address 500
envelopes in 2 min.
⇒ 3x2 + 9x – 2x – 6 = 0
500
⇒ 3x (x + 3) – 2 (x + 3) = 0 Thus, in 1 min, both together will address
2
⇒ (x + 3) (3x – 2) = 0 envelopes
2
\ x= or x = – 3 500 500 500 1 1 1
3 + = ⇒ + =
2 8 x 2 8 x 2
As x cannot be negative, x =
3
144.
(b) Let the total work = 12 unit 147.
(c) By option we can see only option (c) or (d) will be our
choice.
A+ B B+C A+ C So, first we check option (c)
Out-put 12 12 12 24 × 12 × 8 24 × 12 × 8
 6 units 3
units 5 units =
perday 2 4 12 24 × 12 + 12 × 8 + 24 × 8 12 (24 + 8 + 16)
5
24 × 8
= = 4 days
Now, if we add (A + B), (B + C) and (A + C), we get 48
the work output for A, B and C each working for 2
days (6 + 3 + 5) = 14 units. 148.
(a, b, c, d) 20 × 10 M = 15 × 20 W
Thus, in 1 day A, B and C will do 7 units. Since, B \ 2 M = 3 M
and C can do 3 units in 1 day, A alone in a day can \ 10 M = 15 M
complete = (7 – 3) = 4 units.
⇒ 20 × l5 = (15 W + 10 W) × x = 25 × x
Now in order to do the whole job, i.e., 12 units at the
⇒ x = 12
rate of 4 units/day.
12 Option (a), total wage of men
So, A will take = = 3 days = 12 × 10 × 50
4
= `6000
145.
(d) Let the initial length of the two candles = L
The first one burns in 4 h whereas the second one Total wage for women = 12 × 10 × 45 = `5400
burns in 3 h. Total = `11400
Lt Hence, option (a) is correct.
Length of the first candle after t h = L −
4 Option (b), total wage for men
Lt = 12 × 10 × 45 = `5400
Length of the second candle after t h = L −
3 Total wage for women = 12 × 10 × 40 = `4800
Lt  Lt  Total = `10200
So, L− = 2  L − 
4  3 Hence, option (b) is correct.
2t t Option (c), 12 × 10 × 40 + 12 × 10 × 40 = `9600
− = 1
3 4 Hence, (c) is correct.
12 2 Option (d), 5 × 20 × 40 + 5 × 30 × 35 = `9250
⇒ t = = 2 h
5 5 Hence, option (d) is correct.

Chapter_10.indd 256 2/1/2016 9:49:24 AM


11 pipes and Cisterns

INTRODUCTION
Pipes are connected to a tank or cistern and are used to fi ll  terms of fi lling or emptying a cistern and the time taken is 
or empty the tank. the time taken by a pipe or leak (crack) to fi ll or empty a 
cistern, respectively.
Inlet A pipe connected with a tank or a cistern that fi lls it is Generally,  the  time  taken  to  fi ll  a  cistern  is  taken  as 
known as inlet. positive and the time taken to empty a cistern is taken as
Outlet A pipe connected with a tank or cistern emptying it negative. The amount of work done, i.e. fi lling or emptying 
is known as outlet. a  cistern  is  generally  taken  as  unity,  unless  otherwise 
Pipes and Cistern problems are similar to those on time  specifi ed.
and work. The only difference here is the work done is in 

soMe BAsiC FoRMulAe

Solution: We  have,  1/40  part  of  the  cistern  is  emptied  in 
1. If an inlet can completely fi ll the empty tank in X
= 1 min.
1
hrs, the part of the tank fi lled in 1 hr =  . \ 3/4 part of the cistern is emptied in
X
3
2. If an outlet can empty the full tank in Y hrs, the = 40 ×   = 30 mins.
1 4
part of the tank emptied in 1 hr =  . Illustration 4 A pipe can empty a cistern in 12 hrs. Find the
Y
3. If  both  inlet  and  outlet  are  open,  net  part  of  the  part of the cistern emptied in 4 hrs.
1 1 Solution:  We  have,  part  of  the  cistern  emptied  in  1  hr 
tank fi lled in 1 hr =  − . = 1/12,
X Y
1 1
\ part of the cistern emptied in 4 hrs =  × 4 = .
12 3
Illustration 1 A pipe can fi ll a tank in 5 hrs. Find the part of  Illustration 5 A tap can fi ll a cistern is 8 hrs and another can
tank fi lled in one hour. empty it in 16 hrs. If both the taps are opened simultaneously, 
Solution: The part of the tank fi lled in 1 hr = 1/5. fi nd the time (in hrs) to fi ll the cistern.
Illustration 2 A  pipe  can  fi ll  a  tank  in  28  mins.  Find  the  Solution: Here X = 8 and Y = 16.
time in which 1/7 part of the tank will be fi lled. \  Part of the cistern fi lled in 1 hr
1 1
Solution: We have, 1/28 part of the tank is fi lled in 1 min.     =   −
X Y
\  1/7 part of the tank is fi lled in 28/7 mins
= 4 mins. 1 1 1
    =   −  = 
8 16 16
Illustration 3 A pipe can empty a cistern in 40 mins. Find 
the time in which 3/4 part of the cistern will be emptied. \  Total time taken to fi ll the cistern = 16 hrs.

Chapter_11.indd 257 1/30/2016 1:42:27 PM


258  Chapter 11

soMe useFul shoRt-Cut MethoDs

1
1. Two pipes A and B can fi ll (or empty) a cistern in    Part fi lled by B alone in 1 hr = 
Y
X and Y  hrs,  respectively,  while  working  alone.
If  both  the  pipes  are  opened  together,  then  the  1
  Part fi lled by C alone in 1 hr = 
time taken to fi ll (or empty) the cistern is given by Z
  All the three pipes are opened.
 XY 
  hrs. \  Part fi lled in 1 hr
 X +Y 
1 1 1 XY + YZ + ZX
    =  + +  = 
X Y Z XYZ
Explanation XYZ
\  Cistern will be fi lled in  hrs.
Part of the cistern fi lled (or emptied) by pipe A alone  XY + YZ + ZX
1
in 1 hr =  Note: We  can  generate  more  formulae  like  above  by 
X replacing negative sign wherever a pipe starts emptying a 
Part of the cistern fi lled (or emptied) by pipe B alone  cistern instead of the standard positive sign.
1 Illustration 7 Two pipes A and B can separately fi ll a cistern 
in 1 hr =  .
Y is 8 hrs and 12 hrs, respectively, while a third pipe C can
\ Part fi lled (or emptied) by (A + B) in 1 hr empty it in 6 hrs. In what time will the cistern be full, if all 
the pipes are opened together?
1 1 X +Y
=  +  =  . Solution: Here X = 8, Y = 12 and Z = – 6.
X Y XY
\  The cistern will be full in
Therefore, both the pipes A and B together will fi ll (or 
 XY   8 × 12 × −6 
empty) the cistern in       =
   hrs
 hrs.  8 × 12 − 12 × 6 − 6 × 8 
 X +Y 
Illustration 6 Two pipes A and B  can  fi ll  a  cistern  in  20     576 
  =
   hrs or 24 hrs.
and  30  mins,  respectively.  If  both  the  pipes  are  opened   24 
simultaneously, how long will it take to fi ll the cistern?
Solution: Here X = 20 and Y = 30. 3. Two pipes A and B can fi ll a cistern in X hrs and
\  Part of the cistern fi lled by (A + B) in 1 min Y hrs,  respectively.  There  is  also  an  outlet  C.  If 
all the three pipes are opened together, the tank is 
1 1 1 1 5 1 full in Z hrs. The time taken by C to empty the full 
    =
  +  =  +  =   =  .
X Y 20 30 60 12 tank is given by
\ Both the pipes A and B together will fi
 together will fi ll the cistern   XYZ 
in 12 mins.   hrs.
 XZ + YZ − XY 

2. Three pipes A, B and C can fi ll a cistern in X, Y


Explanation
and Z hrs, respectively, while working alone. If all 
the three pipes are opened together, the time taken Part of the tank emptied by C in 1 hr
to fi ll the cistern is given by  1 1 1
=   + − 
 X ×Y × Z  X Y Z
  hrs.
 XY +YZ +ZX   XYZ 
\ C can empty the full tank in    hrs.
 XZ +YZ − XY 
Explanation Illustration 8 Two taps A and B can fi ll a cistern in 30 mins 
1 and 60 mins, respectively. There is third exhaust tap C at the
  Part of the cistern fi lled by A alone in 1 hr = 
X bottom of the tank. If all taps are opened at the same time, 

Chapter_11.indd 258 1/30/2016 1:42:31 PM


Pipes and Cisterns 259

the cistern will be full in 45 mins. In what time can exhaust 
tap C empty the cistern when full?   (b)  If A can fi ll a cistern in y hrs, then the time in
which the cistern will be full, if both the fi ll 
Solution: Here X = 30, Y = 60 and Z = 45.
\  Exhaust tap C can empty the cistern in  k 
pipes are opened together, is   y hrs.
XYZ  k +1
 
      =    mins
 XZ +YZ − XY 
 30 × 60 × 45  Illustration 11 One fi ll pipe A is 10 times faster than second 
      =   mins fi ll pipe B. If B can fi ll a cistern in 55 mins, then fi nd the 
 30 × 45 × 60 × 45 − 30 × 60 
time  when  the  cistern  will  be  full  if  both  fi ll  pipes  are 
      = 36 mins.
opened together.
4. A tank takes X hrs to be fi lled by a pipe. But due  Solution: Here k = 10 and x = 55.
to a leak, it is fi lled in Y hrs. The amount of time  \  Cistern will be full in
in which the leak can empty the full tank  x 
      =   mins
 XY   k +1
=    hrs.
Y − X   55 
      =
    mins or 5 mins.
Illustration 9 A pipe can fi ll a tank in 12 hrs. Due to leakage   10 + 1 
in the bottom, it is fi lled in 24 hrs. If the tank is full, how  Illustration 12 One fi ll pipe A is 4 times faster than second 
much time will the leak take to empty it? fi ll pipe B. If A can fi ll a cistern in 15 mins, then fi nd the 
Solution: Here X = 12 and Y = 24. time  when  the  cistern  will  be  full  if  both  fi ll  pipes  are 
opened together.
\  The time taken by the leak to empty the full tank
Solution: Here k = 4 and y = 15.
 XY   12 × 24 
  =
   hrs =    hrs or 24 hrs. \     Cistern will be full in
Y − X   24 − 12 
 k   4 
    =
   y mins =    15 mins
5.  A cistern has a leak which can empty it in X hrs.  k + 1   4 +1
A pipe which admits Y  litres  of  water  per  hour           = 12 mins.
into the cistern is turned on and now the cistern is
emptied in Z hrs. The capacity of the cistern is
7.  If one fi ll pipe A is k times faster and takes x mins
 XYZ  less time than the other fi ll pipe B, then
  litres.
Z−X    (a)  the  time  taken  to  fi ll  a  cistern,  if  both  the 
 kx 
Illustration 10 A leak in the bottom of a tank can empty the 
 A leak in the bottom of a tank can empty the  pipes are opened together is   mins.
2
full tank in 6 hrs. An inlet pipe fi lls water at the rate of 4 litres   (k − 1) 
per minute. When the tank is full, the inlet is opened and due to   x 
  (b)  A will fi ll the cistern in    mins.
leak, the tank is empty in 8 hrs. Find the capacity of the tank.  k −1 
Solution: Here X = 6, Y = 4 × 60 = 240 and Z = 8.  kx 
  (c)  B will fi ll the cistern in    mins.
\  The capacity of the tank is  k −1 
 XYZ   6 × 240 × 8 
    =   litres =    litres
Z−X   8−6 
Illustration 13 One fi ll pipe A is 5 times faster than second 
    = 5760 litres. fi ll pipe B and takes 32 mins less than the fi ll pipe B. When
will the cistern be full if both fi ll pipes are opened together?
6.  One fi ll pipe A is k times faster than the other fi ll 
Solution: Here k = 5 and x = 32.
pipe B.
\  Cistern will be full in
  (a)  If B can fi ll a cistern in x hrs, then the time in
which the cistern will be full, if both the fi ll  kx 5 × 32
    =
   mins =  mins
 x  (k − 1) 2 (5 − 1) 2
pipes are opened together, is   hrs.
 k +1          = 10 mins.

Chapter_11.indd 259 1/30/2016 1:42:34 PM


260  Chapter 11

Multiple Choice Questions

1. Two pipes A and B can fill a tank in 6 hrs and 9 hrs (a) 7580 (b) 7960
respectively when working alone. A third pipe C can (c) 8290 (d) 8640
empty the same tank in 8 hrs. The pipes are operated such [Based on MAT, 2005]
that A and C are open for the first couple of hrs, then again
B and C for the next 2 hrs, again A and C for the next 2 8.
Pipes A and B running together can fill a cistern in 6 mins.
hrs, and so on. Operating in such a manner, if 68 litres of If B takes 5 mins more than A to fill the cistern, then the
water is filled in 30 hrs, then find the volume of the tank. time in which A and B will fill the cistern separately will
be respectively
(a) 144 litres (b) 108 litres
(c) 72 litres (d) 216 litres (a) 15 mins, 20 mins
(b) 15 mins, 10 mins
2. Three-fourths of a tank is full of water. If 5 litres are added
to it, then four-fifths of the tank becomes full. What is the (c) 10 mins, 15 mins
capacity of the tank? (d) 25 mins, 20 mins [Based on MAT, 2001]
(a) 75 litres (b) 80 litres 9. Two taps can fill a tank in 20 mins and 30 mins
(c) 100 litres (d) 120 litres respectively. There is an outlet tap at exactly half level
[Based on MAT, 2004] of that rectangular tank which can pump out 50 litres of
3.
A tank is filled with water through five pipes. The first water per minute. If the outlet tap is open, then it takes
pipe can fill it in 40 mins. The second, the third and the 24 mins to fill an empty tank. What is the volume of the
fourth together can fill it in 10 mins; the second, the third tank?
and the fifth fill it in 20 mins; the fourth and the fifth (a) 1200 litres (b) 1500 litres
together in 30 mins. In what time will the tank be filled if (c) 1800 litres (d) 2400 litres
all the five pipes work simultaneously? [Based on IIT Joint Man. Ent. Test, 2004]
2 3 10. A tap can fill a tank in 48 mins, whereas another tap can
(a) 8 mins (b) 7 mins
5 4 empty it in 2 hrs. If both the taps are opened at 11:40 a.m.,
4 1 then the tank will be filled at
(c) 8 mins (d) 8 mins
7 7 (a) 12:40 p.m. (b) 1:30 p.m.
4. A vessel is fully filled with a special liquid. Four litres (c) 1:00 p.m. (d) 1:20 p.m.
of liquid is drawn out of this vessel and is replaced [Based on IIT Joint Man. Ent. Test, 2004]
with water. If the ratio of the special liquid to the water 11. A cistern can be filled by pipes A and B in 4 hrs and 6 hrs
becomes 1:2, then what is the capacity of the vessel? respectively. When full, the tank can be emptied by pipe
(a) 8 litres (b) 10 litres C in 8 hrs. If all the taps were turned on at the same time,
(c) 12 litres (d) 14 litres then the cistern will be full in
[Based on MAT, 2002] (a) 3 hrs 18 mins. (b) 3 hrs 26 mins.
5. A pipe can fill a cistern in 6 hrs. Due to a leak in its bottom, (c) 3 hrs 42 mins. (d) 3 hrs 48 mins.
it is filled in 7 hrs. When the cistern is full, in how much [Based on FMS (Delhi), 2003]
time will it be emptied by the leak?
12.
Two pipes A and B can fill a cistern in 12 mins and
(a) 42 hrs (b) 40 hrs 15 mins respectively but a third pipe C can empty the full
(c) 43 hrs (d) 45 hrs tank in 6 mins. A and B are kept open for 5 mins in the
[Based on MAT, 2002] beginning and then C is also opened. In what time will the
6. 12 buckets of water fill a tank when the capacity of each cistern be emptied?
bucket is 13.5 litres. How many buckets will be needed to (a) 30 mins. (b) 33 mins.
fill the same tank, if the capacity of each bucket is 9 litres? (c) 37.5 mins. (d) 45 mins.
(a) 8 (b) 16 [Based on FMS (Delhi), 2002]
(c) 15 (d) 18 13.
A steady stream flows into a cistern partly full which has
7. A leak in the bottom of a tank can empty the full tank in 8 a number of equal holes at the bottom. If 12 holes are
hrs. An inlet pipe fills water at the rate of 6 litres a minute. opened, the cistern is emptied in 4 hrs and if 10 holes are
When the tank is full, the inlet is opened and due to the opened the cistern is emptied in 8 hrs. How many holes
leak, the tank is empty in 12 hrs. How many litres does the should be opened so as to empty the cistern in 2 hrs?
cistern hold?

Chapter_11.indd 260 1/30/2016 1:42:35 PM


 Pipes and Cisterns  261

(a) 14 (b) 16 20.


There is a leak in the bottom of a cistern. Before the leak,
(c) 15 (d) 12 1 1
it could be filled in 4 hrs. It now takes hhrs longer. If
2 2
14.
A tin of oil was four-fifths full. When six bottles of oil the cistern is full, in how much time would the leakage
were taken out and four bottles of oil were poured into it, empty the full cistern?
it was three-fourths full. How many bottles of oil can it
(a) 23 hrs (c) 35 hrs
contain?
(c) 52 hrs (d) 45 hrs
(a) 20 (b) 10
[Based on MAT (Dec), 2010, 2009]
(c) 30 (d) 40
21.
Two filling pipes A and B can fill a tank in 30 hrs and
[Based on MAT (Feb), 2008]
20 hrs respectively. Pipe B alone is kept open for half the
15.
A certain tank can be filled by pipes A and B separately time and both pipes are kept open for the remaining time.
in 4 and 5 min respectively, whereas pipe C can empty in In how many hrs, will the tank be completely full?
3 mins. How long will it take to fill or empty the four- (a) 25 hrs (c) 40 hrs
fifths of the full tank, if all the three pipes start together? (c) 15 hrs (d) 28 hrs
5 2 [Based on MAT (Dec), 2010]
(a) 2 mins to fill (c) 6 mins to fill
7 7 22.
Two pipes A and B can fill a tank in 20 and 30 hrs,
respectively. Both the pipes are opened to fill the tank but
6 5 when the tank is one-third full, a leak develops in the tank
(c) 6 mins to fill (d) 1 mins to fill
7 7 through which one-fourth water supplied by both pipes
[Based on MAT (Feb), 2011] goes out. What is the total time taken to fill the tank?
16.
When the waste pipe is closed, two taps can separately fill 2
(a) 14 hrs
h (c) 15 hrs
a cistern in 10 and 12 mins respectively. When the waste 3
pipe is opened they together fill it in 15 mins. How long 1 1
does it take the waste pipe to empty the cistern, when the (c) 12 hrs
h (d) 9 hrs
h
2 2
taps are closed?
[Based on MAT (Dec), 2010]
(a) 7 mins 10 s (c) 8 mins 34 s
23.
A tank is filled by three pipes with uniform flow. The first
(c) 12 mins (d) 10 mins
two pipes operating simultaneously fill the tank in the
[Based on MAT (Feb), 2011] same time during which the tank is filled by the third pipe
17. A tank is connected with four pipes A, B, C and D of alone. The second pipe fills the tank 5 hrs faster than the
which two are filling the tank and other two are emptying first pipe and 4 hrs slower than the third pipe. The time
it. The time taken by A, B, C and D to finish their jobs are required by the first pipe is
10 hrs, 15 hrs, 20 hrs and 30 hrs, respectively. All four (a) 6 hrs (c) 10 hrs
pipes are opened. When the tank was empty, it took 12 hrs (c) 15 hrs (d) 30 hrs
to fill it completely. Which two are the outlet pipes? [Based on MAT (Sept), 2010]
(a) A and B (b) C and D
24.
A booster pump can be used for filling as well as
(c) A and C (d) B and D for emptying a tank. The capacity of the tank is
18.
Two pipes can fill a tank in 8 hrs and 12 hrs, respectively 2400 m3. The emptying capacity of the tank is 10 m3 per
whereas an escape pipe can empty it in 6 hrs. If the three minute higher than its filling capacity and the pump needs
pipes are opened at 1 pm, 2 pm and 3 pm respectively, at 8 min lesser to empty the tank than it needs to fill it. What
what time will the tank be filled? is the filling capacity of the pump?
(a) 8 am (c) 7 am (a) 50 m3/mins (c) 60 m3/mins
3
(c) 72 m /mins (d) 36 m3/mins
(c) 5 am (d) 7.30 am
[Based on MAT (Sept, Dec), 2010]
[Based on MAT (Feb), 2011]
25.
Two pipes can fill a tank in 10 hrs and 15 hrs, respectively.
19.
Two pipes P and Q can fill a cistern in 3 and 6 mins, However, leakage at the bottom of the tank delays the
respectively, while an empty pipe R can empty the cistern filling of the tank by 3 hrs when both the pipes are open
in 4 min. All the three pipes are opened together and after simultaneously. How much time would the leak take to
2 min pipe R is closed. Find when the tank will be full. empty the full cistern?
(a) 3 mins (c) 6 mins (a) 22 hrs (b) 18 hrs
(c) 5 mins (d) 8 mins (c) 12 hrs (d) 21 hrs
[Based on MAT (Dec), 2010] [Based on MAT (May), 2010]

Chapter_11.indd 261 1/30/2016 1:42:36 PM


262  Chapter 11

26.
Two pipes can fill a cistern in 15 mins and 18 mins 33.
A cistern can be filled by two pipes filling separately in 12
respectively. Both the pipes are operating together but and 16 mins, respectively. Both pipes are opened together
3 mins before the cistern is full, one of the pipes is closed. for a certain time but being clogged, only seven-eighths of
The cistern will be filled now in full quantity water flows through the former and only five-
1 3 sixths through the latter pipe. The obstructions, however,
(a) 9 mins (c) 3 mins being suddenly removed, the cistern is filled in 3 mins from
7 11
that moment. How long was it before the full flow began?
3
(c) 7 mins (d) None of these (a) 4.5 mins (c) 2.5 mins
11 (c) 3.5 mins (d) 5.5 mins
[Based on MAT (May), 2010] [Based on MAT (May), 2009]
27.
Two taps A and B can fill a cistern in 12 mins and 34.
A cistern has a leak which would empty it in 8 hrs. A tap
15 mins respectively. They are opened together but after a is turned on which admits 6 litres a minute into the cistern
few min, A is turned off and the rest of the cistern is filled and it is now emptied in 12 hrs. The cistern can hold
by B in 5 mins. After how many mins was A turned off?
(a) 7860 litres (c) 6840 litres
(a) 4 mins (c) 7 mins
(c) 8640 litres (d) None of these
(c) 6 mins (d) None of these
[Based on MAT (Feb), 2009]
[Based on MAT (May), 2010]
28.
Three pipes A, B and C can fill a tank in 20 mins, 10 mins 35.
A bath tube can be filled by a cold water pipe in 20 mins
and 30 mins, respectively. When the tank is empty, all the and by a hot water pipe in 30 mins. A person leaves the
three pipes are opened. A, B and C discharge chemical bathroom after turning on both pipes simultaneously and
solutions x, y and z respectively. The proportion of returns at the moment when the bath tub should be full.
solution y in the liquid in the tank after 3 mins is Finding however, that the waste pipe has been open, he
now closes it. In 3 mins more the bath tub is full. In what
(a) 6/11 (c) 7/11
time would the waste pipe empty it?
(c) 8/11 (d) 5/11
(a) 38 mins (c) 45 mins
[Based on MAT (Feb), 2010]
(c) 43 mins (d) 48 mins
29.
Three taps A, B and C can fill a tank in 12, 15 and 20 hrs,
[Based on MAT (Dec, May), 2008]
respectively. If A is open all the time and B and C are open
for one hour each alternatively, the tank will be filled in 36.
Six pipes are fitted to a water tank. Some of these are inlet
(a) 6 hrs (c) 7 hrs pipes and the others outlet pipes. Each inlet pipe can fill
the tank in 9 hrs and each outlet pipe can empty the tank
(c) 5 hrs (d) None of these
in 6 hrs. On opening all the pipes, an empty tank is filled
[Based on MAT (Feb), 2010]
in 9 hrs. The number of inlet pipes is
30.
In what time would a cistern be filled by three pipes whose
(a) 2 (b) 4
diameters are 2 cm, 3 cm and 4 cm running together, when
(c) 3 (d) 5
the largest alone can fill it is 58 mins? The amount of
[Based on MAT (Sept), 2008]
water flowing in each pipe is proportional to the square of
its diameter. 37. A leak in the bottom of a cistern can empty the tank in
(a) 36 mins (c) 32 mins 12 hrs. An inlet pipe fills water at the rate of 5 litres a
(c) 23 mins (d) 28 mins minute. When the tank is full, the inlet is opened and due
[Based on MAT (Dec), 2009] to the leak, the tank is emptied in 15 hrs. How many litres
does the cistern hold?
31.
A cistern has 3 pipes A, B and C. A and B running alone
can fill it up in 2 hrs and 3 hrs respectively, while C is a (a) 8260 (b) 12000
waste pipe. If all the 3 pipes be opened at once, 7.24 of (c) 15000 (d) 18000
the cistern will be filled up in 30 mins. In what time can C 38.
Two taps can fill a tank in 20 mins and 30 mins,
empty the full cistern? respectively. There is an outlet tap at exactly half level
(a) 5 hrs (c) 4 hrs of that rectangular tank which can pump out 100 litres of
(c) 3 hrs (d) 6 hrs water per minute. If the outlet tap is open, then it takes 24
[Based on MAT (Dec), 2009] mins to fill an empty tank. What is the volume of the tank?
32.
One fill pipe A is 3 times faster than second fill pipe B (a) 1800 litres (c) 1500 litres
and takes 32 mins less than the fill pipe B. When will the (c) 1200 litres (d) 2400 litres
cistern be full if both pipes are opened together? [Based on MAT (May), 2007]
(a) 12 mins (c) 24 mins 39.
A cistern can be filled by two pipes filling separately in 12
(c) 30 mins (d) Data inadequate and 16 mins, respectively. Both pipes are opened together
[Based on MAT (Sept), 2009] for a certain time but being clogged, only seven-eighths

Chapter_11.indd 262 1/30/2016 1:42:37 PM


 Pipes and Cisterns  263

of the full quantity of water flows through the former (a) 15 hrs (b) 30 hrs
and only 5/6 through the latter pipe. The obstructions, (c) 38.7 hrs (d) 42 hrs
however, being suddenly removed, the cistern is filled in
45.
A cistern is normally filled in 8 hrs, but takes two hrs
3 mins from that moment. How long was it before the full
longer to fill because of a leak in its bottom. If the cistern
flow began?
is full, the leak will empty it in
(a) 2.5 mins (c) 4.5 mins
(a) 16 hrs (b) 25 hrs
(c) 3.5 mins (d) 5.5 mins
(c) 40 hrs (d) 20 hrs
[Based on MAT (May), 2006]
[Based on MAT, 2000]
40.
There are two indentical vessels X and Y. Y is filled
46.
A tank can be filled by a tap in 20 mins and by another
with water to the brim and X is empty. There are two pails
tap in 60 min. Both the taps are kept open for 10 min and
A and B, such that B can hold half as much water as A. One
then the first tap is shut of. After this, the tank will be
operation is said to be executed when water is transferred
completely filled in
from Y to X using A once and water is transferred to Y
from X using B once. If A can hold half a litre of water and (a) 10 min (b) 12 min
it takes 40 operations to equate the water level in X and Y, (c) 15 min (d) 20 min
what is the total volume of water in the system? [Based on FMS (MS), 2006]
(a) 10 litres (b) 20 litres 47. Two taps can fill a tank in 12 mins and 18 mins,
3 respectively. Both the taps are kept open for 2 mins and
(c) 40 litres (d) 20 litres then the tap that fills the tank in 12 min is turned off. In
4
[Based on MAT (Sept), 2009] how many more mins will the tank be filled?
(a) 9 (b) 10
41. Pipe A can fill a tank in 3 hrs and 45 mins. 2 hrs after
(c) 12 (d) 13
the pipe started filling the empty tank the motor stopped
working. What per cent of the tank was left empty? 48. Pipe A and Pipe B can completely fill a cistern in 8 and
12 hrs, respectively. The two pipes are simultaneously
2
(a) 58% (b) 46 % opened but due to a leak at the bottom of the cistem it
3 takes 6 hrs extra to fill the cistern. Find the time in which
1 1 the leak can empty the full cistern.
(c) 33 % (d) 53 %
3 3 120
(a) 10 hrs (b) hrs
42.
Pipes A and B can fill a tank in 5 and 6 hrs, respectively. 13
Pipe C can empty it in 12 hrs. The tank is half full. All 123
(c) hrs (d) none of these
the three pipes are in operation simultaneously. After how 15
much time the tank will be full? 49.
Two pipes can fill a cistern in 14 hrs and 16 hrs
9 respectively, the pipes are opened simultaneously and it is
(a) 3 hrs (b) 11 hrs
17 found that due to leakage in the bottom, 32 min extra are
taken for the cistern to be filled up. If the cistern is full, in
8 13
(c) 2 hrs (d) 1 hrs what time would the leak empty it?
11 17
(a) 96 hrs (b) 102 hrs
[Based on MAT, 1999]
(c) 106 hrs (d) 112 hrs
43. A cistern has two taps (which fill it in 12 mins and [Based on IIFT, 2005]
15 mins, respectively) and an exhaust tap. When all the
50.
Pavan builds an overhead tank in his house, which has
three taps are opened together, it takes 20 mins. to fill
three taps attached to it. While the first tap can fill the
an empty cistern. How long will the exhaust tap take to
tank in 12 hrs, the second one takes one and a half times
empty it?
more than the first one to fill it completely. A third tap is
(a) 20 mins (b) 16 mins attached to the tank which empties it in 36 hrs. Now, one
(c) 12 mins (d) 10 mins day, in order to fill the tank. Pavan opens the first tap and
[Based on MAT, 1999] after two hrs opens the second tap as well. However, at
44. Two pipes A and B fill a swimming pool at constant rates the end of the sixth hour, he realizes that the third tap has
of 10 gallons per minute and 15 gallons per minute, been kept open right from the beginning and promptly
respectively. The pool can be filled in 60 hrs, 40 hrs or closes it. What will be the total time required to fill the
24 hrs depending on, whether pipe A alone, pipe B alone, tank?
or both pipes are used. If the pool is filled by using pipe (a) 8 hrs 48 mins (b) 9 hrs 12 mins
B alone for half the time and using both pipes for half the (c) 9 hrs 36 mins (d) 8 hrs 30 mins
time, how many hrs does it take to fill the pool? [Based on IIFT, 2006]

Chapter_11.indd 263 1/30/2016 1:42:38 PM


264  Chapter 11

51.
A cylindrical overhead tank is filled by two pumps–P1 been opened he now closes it. In 4 mins more, bath is full.
and P2. P1 can fill the tank in 8 hrs while P2 can fill the In what time would be the waste pipe empty it?
tank in 12 h. There is a pipe P3 which can empty the tank (a) 6 mins (b) 9 mins
in 8 hrs. Both the pumps are opened simultaneously. The
(c) 3 mins (d) 15 mins
supervisor of the tank, before going out on a work, sets a
[Based on ATMA, 2008]
timer to open P3 when the tank is half filled so that tank is
exactly filled up by the time he is back. Due to technical 56.
One pipe fills a water tank three times faster than another
fault P3 opens when the tank is one third filled. If the pipe. If the two pipes together can fill the empty tank in
supervisor comes back as per the plan what per cent of the 36 mins, then how much time will the slower pipe alone
tank is still empty? take to fill the tank?
(a) 25% tank (b) 12% tank (a) 1 hr 21 mins (b) 1 hr 48 mins
(c) 10% tank (d) None of these (c) 2 hrs (d) 2 hour 24 mins
[Based on ITFT, 2009] [Based on SSC (GL), 2010]
52.
Three pipes A, B and C are connected to a tank. These 57. One filling pipe A is 6 times faster than second filling pipe
pipes can fill the tank separately in 5 hrs, 10 hrs and B. If B can fill a cistern in 28 mins, then find the time
15 hrs, respectively. When all the three pipes were opened when the cistern will be full if both the pipes are opened
simultaneously, it was observed that pipes A and B were together.
supplying water at three-fourths of their normal rates for the
(a) 6 mins (b) 8 mins
1st hrs after which they supplied water at the normal rate.
Pipe C supplied water at two-thirds of its normal rate for (c) 4 mins (d) 7 mins
first 2 hrs, after which it supplied at its normal rate. In how 58. A, B, C are pipes attached to a cistern. A and B can fill it
much time, tank would be filled? in 20 and 30 mins respectively, while C can empty it in
(a) 1.05 hrs (b) 2.05 hrs 15 mins. If A, B and C are kept in operation successively
(c) 3.05 hrs (d) None of these for one minute each, how soon will the cistern be filled?
[Based on IIFT, 2010] (a) 167 mins (b) 160 mins
53. A water tank has three taps A, B and C. Tap A, when (c) 166 mins (d) 164 mins
opened, can fill the water tank alone in 4 hrs. Tap B, when
59. Pipe A can fill an empty tank in 30 hrs while B can fill it in
opened, can fill the water tank alone in 6 hrs and tap C,
45 hrs. Pipes A and B are opened and closed alternatively
when opened, can empty the water tank alone in 3 hrs. If
i.e., first pipe A is opened, then B, again A and then B
tap A, B and C are opened simultaneously, how long will
and so on for 1 hr each time without any time lapse. In
it take to fill the tank completely?
how many hrs the tank will be filled when it was empty,
(a) 10 hrs (b) 8 hrs initially?
(c) 18 hrs (d) 12 hrs (a) 36 (b) 54
[Based on Indian Bank PO, 2011]
(c) 48 (d) 60
54. A vessel has three pipes connected with it, two to supply
liquid and one of draw liquid. The first alone can fill 60. An inlet pipe can fill a tank in 5 hrs and an outlet pipe
can empty the same tank in 36 hrs, working individually.
1
the vessel in 4 hrs, the second in 3 hrs and the third How many additional number of outlet pipes of the same
2 capacity are required to be opened, so that the tank never
1 over flows?
can empty it in 1 h.hrs. If all the pipes are opened
2 (a) 3 (b) 6
simultaneously when the vessel is half full, how soon will (c) 8 (d) 7
it be emptied?
1
61. Three pipes A, B and C are attached to a cistern.
1
(a) 5 hhrs (b) 3 hrs
h A can fill it in 10 mins B in 15 mins, C is a waste pipe
2 8 for emptying it. After opening both the pipes A and B, a
1 man leaves the cistern and returns when the cistern should
(c) 4 hrs
h (d) None of these
3 have been just full. Finding, however, that the waste pipe
[Based on NMAT, 2006]
had been left open, he closes it and the cistern now gets
55.
A bath can be filled by the cold water pipe in 10 mins filled in 2 mins. In how much time the pipe C, if opened
and by hot water pipe in 15 mins (independently each). alone, empty the full cistern.
A person leaves the bathroom after turning on both pipes (a) 12 mins (b) 16 mins
simultaneously and returns at the moment when the bath
(c) 18 mins (d) 15 mins
should be full. Finding, however, that the waste pipe has

Chapter_11.indd 264 1/30/2016 1:42:39 PM


 Pipes and Cisterns  265

EXPLANATORY ANSWERS

1. (a) In the first 2 hrs, After 2nd operation


 1 1 V Quantity of special liquid
A and C can fill 2 ×    V =
 6 8 12 6 24
= ( x − 4) − ( x − 4) i.e. x + − 10
 1 1 V x x
In 2 hrs, B and C can fill 2 ×    V =
 9 8 36 6 24
Water = 4 − × 4 + 6 = 10 −
In 30 hrs, A and C will operate for 16 hrs and B and C x x
24
for 14 hrs. x+ − 10
x 1
16 V 14  V  \ =
   24 2
⇒  = 68 10 −
2 12 2  36  x
⇒ V = 144 litres
48 24
⇒ 2x + − 20 = 10 −
2.
(c) Let the capacity of the tank be x litres. x x
3 4 72
\ x + 5 = ⇒ 2x + − 30 = 0
4 5 x
⇒ x = 100. ⇒ 2x2 – 30x + 72 = 0
3. (c) Suppose that in one minute each pipe separately can ⇒ x2 – 15x + 36 = 0
fill x, y, z, u, v parts of the tank. Then
⇒ x = 12.
40x = 1
10y + 10z + 10u = 1 1
5.
(a) In one hour, of the cistern can be filled.
and 20y + 20z + 20v = 1 6
30u + 30v = 1 1
1 1 In one hour, only of the cistern can be filled due to
⇒ y + z + u = ,y+z+v= 7
10 20 leak in its bottom.
1 1 1 1
and u + v = \ In one hour, − = of the cistern is empty.
30 6 7 42
11 \ The whose cistern will be emptied in 42 hrs.
⇒ 2(u + v + z + y) =
60
6. (d) Capacity of the tank = 12 × 13.5 = 162 litres
1
and 2x =  Number of buckets required, if the capacity of the
20
162
1 11 14 7 bucket is 9 litres =  = 18
∴ 2(x + y + z + u + v) =    9
20 60 60 30
7 1 1 1
∴ x + y + z + u + v =  7.
(d) Work done by inlet in 1 hr = − =
60 8 12 24
60 4 1 1 1
∴ All together they take or 8 mins to fill the Work done by inlet in 1 min = × =
tank. 7 7 24 60 1440

4. (c) Suppose the volume of the vessel be x litres. 1


Q Volume of part = 6 litres
Therefore, the quantity of special liquid in the vessel 1440
= x litres. \ Volume of whole = 1440 × 6 = 8640 litres.
After 1st operation
8.
(c) Suppose pipe A fills the cistern in x mins.
Quantity of special liquid = x – 4
Therefore pipe B will fill the cistern in (x + 5) mins.
Water = 4

Chapter_11.indd 265 1/30/2016 4:04:45 PM


266  Chapter 11

\ In one minute, pipes A and B together can fill Let one (equal) hole can empty the cistern in y hrs.
1 1  1
So, in one hour, one hole can empty th of the
 x + x + 5  of the cistern. y
  cistern.
1 1 1 12
⇒ + = Therefore, in one hour 12 holes can empty th of
x x+5 6 y
⇒ x = 10 the cistern
\ Pipe A and B can fill the cistern in 10 mins and 15 1 12
Hence, portion emptied in one hour = 
mins respectively. x y
 1 12 
1 1 1 Portion emptied in 4 hrs = 4    ...(1)
9.
(a) In one minute, + = of the tank can be x y
20 30 12
filled. Portion emptied by 10 holes in 8 hrs
The whole tank can be filled in 12 mins.  1 10 
= 8    ...(2)
Total time to fill the tank = 24 mins x y
\ Outlet tap is taking 12 mins to empty half tank at x 1
Equating equations (1) and (2) we get, 
the speed of 50 litre/minute y 8
\ Volume of half tank = 50 × 12 = 600 litres
 1 n
\ Volume of full tank = 1200 litres. Portion emitted by n holes in 2 hrs = 2    .
 x y
1 1 1
10.
(c) In one minute, − = of tank can be filled. Now equating this equation with either equation (1) or
48 120 80
(2), we get n = 16.
\ The whole tank can be filled in 80 mins.
14.
(d) Let the tin oil contain x number of bottles.
1 1 1 6+4−3 7
11.
(b) In one hour, + − = = of the 4 3
4 6 8 24 24 x − 6 + 4 = x
5 4
cistern can be filled.
24 1
\ Whole cistern could be filled in hrs, i.e., 3 hrs ⇒ x = 2
7 20
and 26 mins appox. \ x = 40 bottles
1 1 1 1 1 1
12.
(d) In one minute, pipes A, B and C can fill + − 15.
(c) Net part filled in 1 min = + −
12 15 6 4 5 3
1
= – of the cistern, i.e. if all the three pipes are 15 + 12 − 20 7
60 = =
60 60
opened simultaneously, then the cistern can be
emptied in 60 mins. 7
Q part is filled = 1 min
1 1 3 60
In one minute, pipes A and B can fill + =
12 15 20 60
of the cistern. ⇒ 1 part is filled = min
7

15 3 4 60 4 48
In five mins, = of the cistern will be filled. ⇒ th part is filled = × =
20 4 5 7 5 7

3 6
of the cistern can be emptied in
\ = 6 min to fill
4 7
3 16.
(b) Work done by the waste pipe in 1 min
× 60 = 45 mins.
4 1 1 1
= − + 
13. (b) Let the steady stream can fill the cistern in x hrs. 15  10 12 
1 8 − (12 + 10) 14
So, in one hour, stream can fill th of the cistern = =–
x 120 120

Chapter_11.indd 266 1/30/2016 4:05:06 PM


 Pipes and Cisterns  267

\ Waste pipe will empty the full cistern 21.


(c) Let the tank be completely full in x hrs.
120 60 x
= = Pipe A is open for hrs and pipe B is open for x hrs.
14 7 2
= 8 min 34 s
x 1 1
17. (b) Efficiency of A = 10% \ × + x× =1
2 30 20
Efficiency of B = 6.66%
Efficiency of C = 5% x + 3x
⇒ =1
Efficiency of D = 3.33% 60
Efficiency of A + B + C + D = 8.33 ⇒ x = 15 hrs
(time = 12 hous)
 20 × 30 
Now, go through options and consider A and B as inlet 22.
(a) Together both pipes can fill the tank in   hrs
pipes and C and D as outlet pipes, then  20 + 30 
= 12 hrs
(10 + 6.66) – (5 + 3.33) = 8.33
One-third tank can be filled in 4 hrs.
Which is required, hence, it is certain that
C and D are outlet pipes. Now, there is a leak which can empty the tank in
(12 × 4) hrs = 48 hrs
18.
(c) Upto 3 pm both pipe fill the tank So, two-thirds tank can be filled in
2 1 1 1 4 1
= + = + = = part 2  12 × 48  2
8 12 4 12 12 3 ×  hhrs = 10 hhrs
3  48 − 12  3
1 2
\ Remaining part = 1 − = So, total time to fill the tank
3 3
2
1 1 1 2 = 4 + 10
\  + − x = 3
 8 12 6  3
(6 + 4 − 8) x 2 2
⇒ = = 14 hhrs
48 3 3

2x 3 23.
(c) Let first pipe can fill the tank in x h.
⇒ =
48 2 Second pipe can fill the tank in (x – 5) hrs.
⇒ x = 16 hrs Third pipe can fill the tank in (x – 9) hrs.
Hence, the cistern can be filled at 1 p.m + 16 hrs = 5 a.m. According to the question,
2 2 2 1
x × ( x − 5)
19.
(a) Part of the tank filled in 2 min =  + −  =
3 6 4 2 =x–9
x+ x−5

1
P and Q can fill part of tank in ⇒ x2 – 5x = 2x2 – 23x + 45
2
⇒ x2 – 18x + 45 = 0
1  3× 6 
×  = 1 min ⇒ (x – 15)(x – 3) = 0
2 3+ 6
⇒ x = 15 hrs as x = 3 hrs is not possible.
So, tank will be full in 3 mins.
(a) Let the filled capacity of the tank be x m3/min.
24.
20.
(d) Let leakage can empty the full cistern in x hrs.
Then, emptied capacity of the tank
9
×x = (x + 10) m3/min
Then, 2 = 5
9 2400 2400
x− \ − =8
2 x x + 10

9 45  10 
⇒ x = 5x −
2 2 ⇒ 2400   =8
 x( x + 10) 
1 45
⇒ x = ⇒ x(x + 10) = 3000
2 2
⇒ x = 45 hrs \ x = 50 m3/min

Chapter_11.indd 267 1/30/2016 1:42:51 PM


268  Chapter 11

29.
(b) Filling done by pipe A and B in 1 hrs
 10 × 15 
25.
(b) Two pipes can fill the tank in   hhrs = 6 hrs
 10 + 15  1 1 3
= + =
12 15 20
Due to leakage, the tank is filled in 9 hrs.
Let full tank can be empty in x hrs. Filling done by pipe A and C in 1 hrs

6× x 1 1 2
So, 9 = = + =
x−6 12 20 15

⇒ 9x – 54 = 6x 3 2 17
Filling done in first 2 hrs = + =
⇒ x = 18 hrs 20 15 60
17 51
26.
(d) Let the cistern will be filled in x min. Filling done in 6 hrs = ×3=
60 60
Case I First pipe was closed
x−3 x 51 3
+ =1 Remaining filling = 1 − =
15 18 60 20
Now in 7th hour, filling done by pipes A and B, so
6 x − 18 + 5 x
⇒ =1 time
90 3
⇒ 11x – 18 = 90 = 20 = 1 hhrs
3
9
⇒ x = 9 mins 20
11
So, total time = 7 hrs
Case II Second pipe was closed.
30.
(b) Pipe whose diameter is 4 cm fills the cistern in 58 min.
x x- 3 Let the capacity of cistern be x l.
+ =1
15 18 x
Then, flow rate of 4 cm diameter pipe = l/min
6 x + 5 x - 15 58
=1 x
90 \ ∝ (4)2
58
11x – 150 = 90
6 x x
x = 9 mins ⇒ = K × 16 ⇒ K =
11 58 928
Note: It is not given that which pipe was closed. x
Flow rate of 3 cm diameter pipe = × (3)2 l/min
So, you have to calculate both the cases. 928
\ Time required by 3 cm diameter pipe to fill the
27.
(d) Let A was turned off after x min.
x 1
x x+5 cistern = × 928 = 103 min
Then, + =1 9x 9
12 15
x
5 x + 4 x + 120 Flow rate of 2 cm diameter pipe = × (2)2 l/min
⇒ =1 928
60 \ Time required by 2 cm diameter pipe to fill the
⇒ 9x + 20 = 60
x
4 cistern = × 928 = 232 min
⇒ x = 4 min 4x
9
\ Time required by all the three pipes working
28.
(a) Filling done by all 3 pipes in 3 mins
1
3 3 3 11 58 × 103 × 232
= + + = together = 9
20 10 30 20 1 1
58 × 103 + 58 × 232 + 103 × 232
3 9 9
Filling done by 2nd pipe in 3 mins =
10 58 × 928 × 232
=
3 / 10 6 58 × 928 + 58 × 232 × 9 + 928 × 232
So, required ratio = =
11 / 20 11 = 32 min

Chapter_11.indd 268 1/30/2016 1:42:55 PM


 Pipes and Cisterns  269

1 35.
(d) The usual time required to fill the tank when both
31.
(b) A’s work in 1 hrs =
2  1 1  1
the pipes are opened =  +  =i.e., 12 min.
1  20 30  12
B’s work in 1 hrs = It means work done by all the three pipes for 12 min
3
+ work done by both the pipes for 3 min = 1. Let the
1 waste pipe takes x min to empty the tank. Then,
C’s work in 1 hrs =
x
1 1 1 1 1
7 12  + −  + 3 +  = 1
Work done by all 3 pipes in 30 min = full  20 30 x   20 30 
24
 1 1 1
7 60 7 ⇒ 12  −  + 3   = 1
Work done by all 3 pipes in 1 hrs = × = full 12 x  12 
24 30 12
1 1 1 7  x − 12  3
so, + + = ⇒ 12   =
2 3 x 12  12 x  4
5 1 7 ⇒ 16x – 192 = 12x
⇒ + =
6 x 12 ⇒ 4(x – 12) = 3x
⇒ x = 48 min
1 1
⇒ =–
x 4 36.
(b) Let the number of inlet pipes be x, then number of
⇒ x = – 4 hrs outlet pipes will be (6 – x).
Hence, C can empty in 4 hrs. x (6 − x) 1
Then, − =
32.
(a) Let pipe A takes x min to fill the cistern. 9 6 9
Then, pipe B takes 3x min to fill the cistern. 2 x − 18 + 3 x 1
⇒ =
\ 3x – x = 32 18 9
⇒ x = 16 min and 3x = 48 min ⇒ 5x = 20
 48 × 16 
So, both pipes can fill the cistern in   min \ x = 4
 48 + 16  Hence, number of inlet pipes is 4.
= 12 min
33.
(a) Part of the cistern filled in 3 min 1 1 1
37. (d) Part of tank filled in one hour - =
12 15 60
3 3 21 7
= + = =
12 16 48 16 So, the inlet can fill the tank in 60 hrs.
∴ Capacity of the tank = 60 × 60 × 5 = 18000.
9
Let remaining part was filled in x min.
16 38.
(a) The two filler taps can fill the tank in
x 7 x 5 9  20 × 30 
Then, × + × =
12 8 16 6 16   = 12 min
(30 + 20) 

7 + 5 9 The above information implies that half of the tank
⇒ x  =
 96  16 will be filled in 6 min.
9 96 Hence, it took (24 – 6 = 18) min to fill the remaining
⇒ x = × =
4.5 min
16 12 half of the tank when the outlet pump was open. Thus,
the total time required to empty half of the cistern
34.
(c) Let the tap can fill the cistern in x hrs.
x×8
\ = 12 18 × 6 18 × 6
x −8 0 = = = 9 min
18 – 6 12
⇒ 8x = 12x – 96
⇒ x = 24 hrs Thus, the capacity of the tank
\ Capacity of cistern = 100 × 9 × 2
= 24 × 60 × 6 = 8640 l = 1800 l

Chapter_11.indd 269 1/30/2016 1:42:58 PM


270  Chapter 11

1 1 7 Let Tap C can empty the cistern in x mins.


39.
(b) Both the pipes A and B can fill + = of the
12 16 48 1 1 1 1
\ + − =
cistern in one minute, when there is no obstruction 12 15 x 20
1 7 ⇒ x = 10
with obstruction, both the pipes can fill ×  + 
12 8 (1 / 2) t (1 / 2) t
1 5 7 5 1 44. (b) Let the time taken be t ⇒    = 1
 ×  = + = of the cistern in one minute 40 24
16 6 96 96 8 t t
⇒  = 1
Let the obstruction were remained after x min. 80 48
x 80  48

\ With obstruction, of the cistern could be filled in ⇒ t = = 30 hrs
8 128
x 8− x 1
x min and so the remaining 1 – = of the 45.
(c) Cistern’s 1 hr filling capacity =
8 8 8
cistern was filled without obstruction is 3 min, i.e., in
1
8− x Cistern’s 1 hr filling capacity due to leak =
1 min of the cistern was filled with obstruction 10
24
\ Cistern’s 1 hr emptying capacity due to leak
8− x 7
⇒ = 1 1 1
24 48 = − =
8 10 40
⇒ 16 – 2x = 7
Hence, when the cistern is full, it will get empty in
⇒ x = 4.5 mins 40 hrs.
1
40.
(b) In one operation, litre water is transferred from Y 46.
(d) Part filled in 10 mins.
4
to X. 10 10 2
 =
In 40 operations, total quantity of water transferred 20 60 3

1 Remain = 1/3 filled by second
= × 40
4 time = 1/ 3 × 60 = 20 mins
= 10 litre
\ Total volume of water in the system 1 1 5
47. (d) In 2 mins, the taps fills 2    or of the tank.
 12 18  18
= 10 × 2 = 20 litres
13
41. (b) The tank is filled in 3 hrs and 45 mins = 225 mins. Therefore, of the tank is to be filled by the second
18
120 8 1
∴ In 120 mins, the pipe fills = . tap at the rate of of the tank per minute. This will
225 15 18
7 2 take another 13 mins.
Required percentage = × 100 = 46 %
15 3
48. (d) Time taken to fill the cistern when pipe and pipe B are
42.
(d) In one hour, (8  12)
opened simultaneously =  = 4.8 hrs
1 1 1 12 + 10 − 5 (8  12)
+ − =
5 6 12 60 Time taken due to the leek to fill the cistem
17 = 4.8 + 6 = 10.8 hrs
= of the tank is filled.
60 Time taken by the leak alone to empty the cistern
(10.8  4.8)
1 = = 8.64 hrs
Therefore, of the tank is filled in (10.8  4.8)
2
60 1 30 13 49.
(d) The time taken by two pipes of fill the tank
hrs = 1 hrs
× = hours hours
17 2 17 17 1 112 112
= = =
1 1 8 + 7 15
43.
(d) Let Tap A can fill the cistern in 12 mins. +
14 16
Let Tap B can fill the cistern in 15 mins.
= 7 hrs 28 mins

Chapter_11.indd 270 1/30/2016 4:07:40 PM


 Pipes and Cisterns  271

Now, total time taken = 7 hrs 28 min + 32 min = 8 hrs. 139


Remaining part of the tank to be filled =
Let leakage can empty the tank in x hour. 360
1 1 1 1 11
\ + − = In 1 h, all the three pipes together will fill =
14 16 x 8 30
⇒ x = 112 hrs Hence, the time taken to fill the remaining tank
50.
(b) After 6 hrs remaining part was filled by 2 pipes.  139   30 
=  = 1.0530 hrs
6 4 6 20 5  360   11 
\   = =
12 18 36 36 9 Thus, the total time taken to fill the tank = 3.05 hrs.
4 53. (d) Required time to fill the tank
\ Remaining part =
9 1 1 1
= = = = 12 hrs
36 36  1 1  1 5 1 1
A + B can fill the tank in = hhrs  + − −
32 5 4 6 3 12 3 12

4 9
\ part filled in 54. (d) 1st pipe can fill in hrs.
9 2
36 4 16 1 2nd pipe can fill in 3 hrs.
 = 3 h
5 9 5 5 3
And 3rd pipe can empty in hrs.
h.
2
1
\ Total time = 6 + 3 hhrs The vessel in emptied is
5
= 9 hrs 12 mins 2 1 2 1
+ − = −
9 3 3 9
24
51.
(c) P1 and P2 can fill the tanks hhrs
5 \ Then vessel full emptied in 9 hrs
9 1
1 1  \ Then vessel half emptied in hhrs is 4 h.
hrs.
[Q in 1 hrs these fill    part of tank] 2 2
 8 12 

12 55.
(d) Waste pipe alone empties the bath in
\ It takes hrs in filling half the tank
5 xy  xy 
1 +  mins ... (1)
Far remaining half of the tanks P3 will open and this x+ y  ( x + y) t 
will take 6 hrs
Here x = 10 mins, y = 15 mins and t = 4 mins
 12 
\ Supervisor has gone out for   6 hhrs Putting these values in Eq. (1), we get
5 
8 10 × 15  10 × 15 
Now, l/3rd tank will fill in hhrs 1 + 
5 10 + 15  (10 + 15) × 4 
42 33
\ In remaining hhrs only th part of tank will 10 × 15  10 × 15 
5 60 1 +  = 15 mins.
fill 25  25 × 4 

 1 33  6 56.
(d) Let the time taken by the pipe at faster rate to fill the
\ empties part of tank = 1     
 3 60  60 tank be x mins
which is 10% of tank.
1 1 1
Therefore, + =
52.
(c) The part of the tank filled by A and B in first two hrs x 3x 36
31 1  1 1  3 +1 1
=       ⇒ =
4  5 10   5 10  3x 36
The part of tank filled by C in first two hrs 4 1
⇒ =
 2  1  3x 36
= 2    
 3   15  ⇒ 3x = 4 × 36

Chapter_11.indd 271 1/30/2016 4:08:57 PM


272  Chapter 11

⇒ 3x = 144 1
59. (a) In one hour pipe A can fill = part of the tank.
148 30
⇒ x = = 48 mins
3 Therefore, in 36 hrs the tank will be completely filled.
Hence, time taken by slower pipe Alternatively: Efficiency of pipe A = 3.33%
= 3x = 3 × 48 = 144 mins Efficiency of pipe B = 2.22%
= 2 hrs 24 mins. and Combined efficiency = 5.55%
1 Therefore in 2 hrs pipe A and B fill 5.55%.
57. (c)  In one minute, B will fill cistern.
28 Thus to fill 100% tank, these pipe will take 36 hrs.
6 60. (d) Since, an inlet pipe is 7.2 times more efficient than an
Hence, A will fill cistern. Work done by both the
28 outlet pipe, therefore, in order to ensure that the tank
never overflows, we will need total 8 outlet pipes.
 1 6 1
pipes in 1 min =    cistern. Hence, they
 28 28  4 Thus, we need only 7 more (8 – 1 = 7) outlet pipes.

will fill 1 cistern in 4 mins. 61. (c) Let the pipe C alone empties the cistern in x mins. A
 1 1 1 1 10  15
58. (a) Work done in 3 mins =    . and B together can fill the cistern in  6 mins.
 20 30 15  60 10  15
55 Since, waste pipe was left open for 6 mins, then in 6
∴ Work done in 3 × 55 = 165 mins = . Remaining 6
60 mins, part of the cistern will be emptied by waste
 55  5 1 x
tank = 1   = = .
 60  60 12 6
pipe C. Now part of the cistern would be filled by
1 x
Now, its A’s turn. part of the tank is filled by A in
20 A and B together in 2 mins. Therefore cistern will be
1 1 1 x
1 min, Since there is still     tank to be filled in mins.
 12 20  30 3
x
filled, which will be filled by B in 1 min. Therefore, ∴   = 6 ⇒ x = 18 mins.
required time = (165 + 2) = 167 mins. 3

Chapter_11.indd 272 1/30/2016 6:11:41 PM


12 Time and Distance

INTRODUCTION Notes:
1. If the time taken is constant, the distance travelled is
The terms ‘Time’ and ‘Distance’ are related to the speed of
proportional to the speed, that is, more the speed; more
a moving object.
the distance travelled in the same time.
Speed: We defi ne the speed of an object as the distance 2. If the speed is constant, the distance travelled is
covered by it in a unit time interval. It is obtained by proportional to the time taken, that is, more the distance
dividing the distance covered by the object, by the time it travelled; more the time taken at the same speed.
takes to cover that distance. 3. If the distance travelled is constant, the speed is
inversely proportional to the time taken, that is, more
Distance travelled the speed; less the time taken for the same distance
Thus, Speed = .
Time taken travelled.

SOME BASIC FORMULAE

Illustration 1 Calculate the speed of a train which covers a


Distance distance of 150 km in 3 hrs.
1. Speed = 2. Distance = Speed × Time
Time Distance covered 150
Solution: Speed = = = 50 km/hr.
Distance Time taken 3
3. Time =
Speed Illustration 2 How long does a train 100 m long running at
Units of Measurement the rate of 40 km/hr take to cross a telegraphic pole?
Solution: In crossing the pole, the train must travel its own
Generally, if the distance is measured in kilometre,
length.
we measure time in hrs and speed in kilometre per
\ Distance travelled is 100 m.
hour and is written as km/hr and if the distance is
measured in metre then time is taken in second and 40 × 1000 100
Speed = 40 km/hr = = m/s
speed in metre per second and is written as m/s. 60 × 60 9
100
Conversion of Units \ Time taken to cross the pole = =9s
100/9
1000 m 5
One kilometre/hr = = m/s. Illustration 3 A train running at a speed of 90 km/hr passes
60 × 60 s 18
a pole on the platform in 20 s. Find the length of the train
18 in metres.
\ One metre/s = km/hr.
5 Solution: Speed of the train = 90 km/hr
 5 5
Thus, x km/hr =  x ×  m/s. = 90 × = 25 m/s.
 18  18
\ Length of the train = Speed of the train
 18  × time taken in crossing the pole
and, x m/s =  x ×  km/hr.
 5 = 25 × 20 = 500 m.

Chapter_12.indd 273 1/30/2016 4:17:52 PM


274 Chapter 12

SOME USEFUL SHORT-CUT METHODS

1. (a) If A covers a distance d1 km at s1 km/hr and 2. A person goes certain distance (A to B) at a speed
then d2 km at s2 km/hr, then the average of s1 km/hr and returns back (B to A) at a speed of
speed during the whole journey is given by s2 km/hr. If he takes T hrs in all, the distance
s s (d + d ) between A and B is
Average speed = 1 2 1 2 km/hr
s1d 2 + s2 d1  ss 
T 1 2 .
(b) If A goes from X to Y at s1 km/hr and comes  s1 + s2 
back from Y to X at s2 km/hr, then the average
speed during the whole journey is given by Explanation
2s1s2 Let the distance between A and B be d km.
Average speed =
s1 + s2
d
Time taken during onward journey = t1 = hrs.
s1
Explanation d
Time taken during return journey = t2 = hrs.
(a) Time taken to travel d1 km at s1 km/hr is s2
d1 \ Total time taken during the entire journey is
t1 = hr
s1 d d d ( s1 + s2 )
T = t 1 + t2 = + =
Time taken to travel d2 km at s2 km/hr is s1 s2 s1s2
d2  ss 
t2 = hr \ d = T 1 2 .
s2
 s1 + s2 
d d  Thus, the distance between A and B is
Total time taken = t1 + t2 =  1 + 2  hr
 s1 s2   ss 
= T 1 2 
s d +s d 
=  1 2 2 1  hr  s1 + s2 
 s1s2  Product of two speeds
= Total time taken × .
Total distance covered = (d1 + d2)km. Therefore, Sum of two speeds
Total distance covered Illustration 5 A boy goes to school with the speed of 3 km
Average speed =
Total time taken an hour and returns with a speed of 2 km/hr. If he takes 5
s1s2 (d1 + d 2 ) hrs in all, fi nd the distance in km between the village and
= km/hr ...(1) the school.
( s1d 2 + s2 d1 )
Solution: Here s1 = 3, s2 = 2 and T = 5.
(b) Let the distance from X to Y be d km
\ The distance between the village and the school
Take d1 = d2 = d in (1), we get
2ds1s2 2s1s2  ss   3× 2 
Average speed = = . = T 1 2  = 5  = 6 km.
d ( s1 + s2 ) s1 + s2 s + s
 1 2  3+ 2 

Illustration 4 A ship sails to a certain city at the speed of 3. If two persons A and B start at the same time from
15 knots/hr and sails back to the same point at the rate of two points P and Q towards each other and after
30 knots/hr. What is the average speed for the whole journey? crossing they take T1 and T2 hrs in reaching Q and
Solution: Here s1 = 15 and s2 = 30. P, respectively, then
2s1s2 2 × 15 × 30 A's speed T2
\ Average speed = = = .
s1 + s2 15 + 30 B's speed T1
= 20 knots/hr

Chapter_12.indd 274 1/30/2016 4:17:53 PM


Time and Distance 275

Explanation Illustration 6 Nikita starts her journey from Delhi to


Let the total distance between P and Q be d km. Bhopal and simultaneously Nishita starts from Bhopal to
Delhi. After crossing each other they finish their remaining
Let the speed of A be s1 km/hr and that of B be s2 km/hr.
4
journey in 5 hrs and 9 hrs, respectively. What is Nishita’s
9
speed if Nikita’s speed is 36 km/hr?
Solution:
Since they are moving in opposite directions, their Nikita's speed T2 9 9
= = =
relative speed is (s1 + s2) km/hr. Nishita's speed T1 4 49
5
9 9
 d 
They will meet after   hr 81 9
 s1 + s2  = = .
49 7
 d 
Distance travelled by A in   hr 7
∴ Nishita’s speed = Nikita’s speed
 s1 + s2  9
 ds1  7
= × 36 = 28 km/hr.
= PO =   km 9
 s1 + s2 

 d  4. If a body travels d1, d2, d3, ..., dn metres with different


Distance travelled by B in   hr speeds s1, s2, s3, ... , sn m/s in time T1, T2, T3, ... ,
 s1 + s2  Tn s, respectively, then the average speed of the
body throughout the journey is given by
 ds2 
= QO =   km Total distance travelled
 s1 + s2  Va =
Total time taken
Time taken by A to travel QO d1 + d 2 + d3 + ... + d n
=
 ds2  T1 + T2 + T3 + ... + Tn
 
 s1 + s2  (If d1, d2, ..., dn and T1, T2, ..., Tn are known)
=
s1
s1T1 + s2T2 + s3T3 + ... + snTn
= T1 (given) ...(1) and Va =
T1 + T2 + T3 + ... Tn
Time taken by B to travel PO (If d1, d2, ..., dn and s1, s2, ..., sn are known)
 ds1 
 
s +s
=  1 2 Illustration 7 A car during its journey travels 40 min at a
s2 speed of 30 km/hr, another 50 min at a speed of 60 km/hr
= T2 (given) ...(2) and 1 hr at a speed of 30 km/hr. Find the average speed of
the car.
Dividing Eq. (2) by Eq. (1), we get 40 50
Solution: Here T1 = ,T = , T = 1, s1 = 30,
s1/s2 T 60 2 60 3
= 2
s2 /s1 T1 s2 = 60, s3 = 30.
∴ Average speed of the car
2
 s1  T2 s T2 40 50
or,   = or, 1 = . 30 × + 60 × + 30 × 1
 s2  T1 s2 T1 s1T1 + s2T2 + s3T3 60 60
= =
T1 + T2 + T3 40 50
+ +1
A's speed T2 60 60
∴ = .
B's speed T1 = 40 km/hr.

Chapter_12.indd 275 1/30/2016 4:20:43 PM


276 Chapter 12

a 7. A train travels a certain distance at a speed of s1


5. If the new speed is of the original speed,
b km/hr without stoppages and with stoppages, it
then the change in time taken to cover the same covers the same distance at a speed of s2 km/hr.
distance is given by The stoppage time per hour is given by
b   s1 − s2   Difference of speed 
Change in time =  − 1 × original time.
a    hr or,  
 s1   Speed without stoppages 

Illustration 8 By walking at four-fi


By walking at four-fi fths of his usual speed,
Mohan is 6 min late to his offi ce. Find his usual time to Explanation
cover the distance. Let the distance travelled be d km.
a 4
Solution: Here change in time = 6 and = . \ Time taken by the train without stopping any where
b 5
d
b  = hrs
We have change in time =  − 1 × original time s1
a 
Also, time taken by the train with stoppages
Change in time d
⇒ Original time =
b  = hrs
 − 1 s2
a 
6 d d s −s 
= = 24 min Total stoppage time = − =  1 2  d hrs
5  s2 s1  s1s2 
 − 1
4   s1 − s2 
 d
 s1s2 
6. A body covers a distance d in time T1 with speed \ Stoppage time per hour =
d
s1, but when it travels with speed s2 covers the
same distance in time T2. s2

The following relations hold s −s 


=  1 2  hrs.
Product of speed s
= 1 = 2 =
s Difference of speed  s1 
d T2 T1 Difference of time Illustration 10 Without stoppages, a train travels certain
Equating any two of the above, we can fi nd the distance with an average speed of 80 km/hr and with
unknowns as per the given question. stoppages, it covers the same distance with an average
speed of 60 km/hr. How many minutes per hour the train
stops?
Illustration 9 Two bicyclists do the same journey by
Solution: Here s1 = 80 and s2 = 60
travelling respectively, at the rates of 9 and 10 km an hour.
Find the length of the journey when one takes 32 min longer \ Stoppage time/hr
than the other. s −s 80 − 60 1
= 1 2 = = hr
Solution: Here change in speed = 10 – 9 = 1; product of s1 80 4
32 = 15 mins
speed = 9 × 10 = 90 and difference of time = .
60
Product of speed Difference of speed 8. (a) If a train overtakes a pole or a man or a
We have = milestone, then the distance covered in
d Difference of time
overtaking is equal to the length of the train.
 Difference of time 
⇒ d = Product of speed ×   (b) If a train overtakes a bridge or a tunnel or
 Difference of speed  a platform or another train, then the distance
32 covered is equal to the sum of the two lengths.
= 90 × = 48 km
60

Chapter_12.indd 276 1/30/2016 4:17:55 PM


Time and Distance 277

Illustration 11 A 600 m long train crosses a pole in 9 s. L1 + L2 135


What is the speed of the train in km/hr? \ The time taken = =
s1 + s2 5
Solution: Speed of the train 54 ×
135 × 18 18
Length of the train = = 9 s.
= 54 × 5
time taken in crossing the pole
Illustration 14 Two trains of length 110 metres and 90 m
600 600 18 are running on parallel lines in the same direction with a
= m/s = × = 240 km/hr. speed of 35 km/hr and 40 km/hr, respectively. In what time
9 9 5
will they pass each other.
Illustration 12 A train 130 m long passes a bridge in 21 s Solution: Here L1 = 110 m, L2 = 90 m, s1 = 35 km/hr and
moving with a speed of 90 km/hr. Find the length of the s2 = 40 km/hr
bridge. 5
\ s2 – s1 = 40 – 35 = 5 km/hr = 5 × m/s
Solution: We have speed of the train 18
L +L
length of the train + length of the bridge \ Time taken = 1 2
= s2 − s1
time taken in crossing the bridge
110 + 90 200 × 18
= =
5 130 + length of the bridge 5 5× 5
⇒ × 90 = 5×
18 21 18
= 144 s.
\ Length of the bridge = 525 – 130 = 395 m.
10. Two trains of lengths L1 m and L2 m run on parallel
9. Relative Speed tracks. When running in the same direction, the
(a) If two trains of lengths L1 km and L2 km, faster train passes the slower one in T1 s, but when
respectively, are travelling in the same they are running in opposite directions with the
direction at s1 km/hr and s2 km/hr, respectively, same speeds as earlier, they pass each other in T2 s.
such that s1 > s2, then s1 – s2 is called their Then, the speed of the faster train
relative speed and the time taken by the faster
L1 + L2  1 1 
train to cross the slower train is given by =  +  m/s
2  T1 T2 
 L1 + L2 
  hr and the speed of the slower train
 s1 − s2 
L1 + L2  1 1 
(b) If two trains of length L1 km and L2 km, =  −  m/s.
2  T1 T2 
respectively, are travelling in the opposite
directions at s1 km/hr and s2 km/hr,
respectively, then s1 + s2 is called their Explanation
relative speed and the time taken by the trains
Let the speed of the faster train be s1 m/s and that of the
to cross each other is given by
slower train be s2 m/s.
 L1 + L2 
  hr Total distance covered when the two trains cross each
 s1 + s2  other = L1 + L2.
When the two trains are running is the same direction,
their relative speed = (s1 – s2) m/s.
Illustrations 13 A train 135 m long is running with a speed
of 49 km/hr. In what time will it pass a man who is walking L1 + L2
\ (s1 – s2) = ...(1)
at 5 km/hr in the direction opposite to that of the train? T1
Solution: Here L1 = 135, L2 = 0, s1 = 49 km/hr, s2 = 5 km/ When the two trains are running in the opposite
hr. directions, their relative speed = (s1 + s2) m/s.
5 L +L
\ s1 + s2 = 49 + 5 = 54 km/hr = 54 × m/s \ s1 + s2 = 1 2 ...(2)
18 T2

Chapter_12.indd 277 1/30/2016 4:17:55 PM


278 Chapter 12

Adding Eqs. (1) and (2), we get Therefore, speed of the faster train
L1 + L2 L1 + L2  75 15 
2s1 = + +
T1 T2  L1 + L2   T1 + T2   200 + 175   2 2 
=   =    75 15 
 2   T1T2   2  × 
1 1   2 2 
= (L1 + L2)  + 
 T1 T2  375 45 × 4
= × = 30 m/s.
2 75 × 15
 L + L  T + T  Speed of slower train
or, s1 =  1 2   1 2  .
 2   T1T2   75 15 

On subtracting Eq. (1) from Eq. (2), we get  L1 + L2   T1 − T2   200 + 175   2 2 
=    =    75 15 
 2   T1T2   2  × 
 1 1  2 2 
2s2 = (L1 + L2)  − 
 T2 T1  375 30 × 4
= × = 20 m/s.
2 75 × 15
 L + L  T − T 
or, s2 =  1 2   1 2  m/s.
 2   T1T2  11. (a) A train starts from a place at s1 km/hr and
Therefore, another fast train starts from the same place
after T hrs at s2 km/hr in the same direction.
speed of the faster train Then, the distance from the starting place at
 L + L  T + T  which both the trains will meet is given by
=  1 2   1 2  m/s.
 2   T1T2   s1 × s2 × T 
  km.
and speed of the slower train  s2 − s1 
Also, the time after which the two trains will
 L + L  T − T 
=  1 2   1 2  m/s. meet is given by
 2   T1T2 
 s1T 
Note: If the two trains are of equal length that is,   hr.
L1 = L2 = L (say), then  s2 − s1 
(b) The distance between two stations A and B is
T +T 
s1 = L  1 2  m/s and
 T1T2  d km. A train starts from A to B at s1 km/hr.
T hrs later another train starts from B to A at
 T −T  s2 km/hr. Then, the distance from A, at which
s2 = L  1 2  m/s.
 T1T2  both the trains will meet is given by

Illustration 15 Two trains of lengths 200 m and 175 m run  d + s2T 


s1   km.
on parallel tracks. When running in the same direction the  s1 + s2 
1 Also, the time after which the two trains will
faster train crosses the slower one in 37 s. When running
2 meet is given by
in opposite directions at speeds same as their earlier speeds,
1  d + s2T 
they pass each other completely in 7 s. Find the speed of   hr.
2  s1 + s2 
each train.

75 Illustration 16 A train starts from Mumbai at 10 a.m. with


Solution: We have L1 = 200, L2 = 175, T1 = and
2 a speed of 25 km/hr and another train starts from there on
the same day at 3 p.m. in the same direction with a speed
15 of 35 km/hr. Find at what distance from Mumbai both the
T2 = .
2 trains will meet and fi nd also the time of their meeting.

Chapter_12.indd 278 1/30/2016 4:17:56 PM


Time and Distance 279

Solution: Time from 10 a.m. to 3 p.m. = 5 hr.


12. Two trains start simultaneously from the stations
Distance of meeting point from Mumbai
A and B towards each other with speeds s1
 s × s ×T  km/hr and s2 km/hr, respectively. When they meet
=  1 2  km.
 s2 − s1  it is found that the second train had travelled d km
more than the first. Then the distance between the
 25 × 35 × 5  1 two stations is given by
=   km = 437 km.
 35 − 25  2 s +s 
d  1 2  km.
Also, time of their meeting  s2 − s1 
 sT   25 × 5 
=  1  hr. =   hr.
 s2 − s1   35 − 25 
Explanation
125 1 Let the distance between the two stations be x km. If
= = 12 hr. after 3 P.M.
10 2 the first train travels y km then the second travels y +
That is, 3.30 a.m. next day d km.
∴ x = y + y + d = 2y + d.
Illustration 17 Chennai is at a distance of 560 km from
Mumbai. A train starts from Mumbai to Chennai at 6 a.m. Since the time taken by both the trains is same
with a speed of 40 km/hr. Another train starts from Chennai y+d y
to Mumbai at 7 a.m. with a speed of 60 km/hr. At what ∴ =
s2 s1
distance from Mumbai and at what time will the two trains
be at the point of crossing? ⇒ s1 y + s1d = s2 y
s1d
Solution: Time from 6 a.m. to 7 a.m. = 1 hr. ⇒ (s2 – s1)y = s1d or, y = .
s2 − s1
Therefore, distance of meeting point from Mumbai
 sd  d ( s1 + s2 )
 d + s2T  ∴ x = 2 1 +d = km.
= s1   km.  s2 − s1  ( s2 − s1 )
 s1 + s2 
Illustration 18 Two trains start at the same time from Delhi
 560 + 60 × 1  and Rohtak and proceed towards each other at the rate of
= 40   = 248 km.
 40 + 60  75 km and 65 km/hr, respectively. When they meet, it is
found that one train has travelled 10 km more than the other.
Also, time of their meeting Find the distance between Delhi and Rohtak.
 d + s2T  Solution: Distance between Delhi and Rohtak
=   hr
 s1 + s2  s +s 
= d  1 2  km.
 560 + 60 × 1 
= 
31  s1 − s2 
 = hr
 40 + 60  5  75 + 65 
= 10   km
= 6 hr 12 min. after 6 a.m.  75 − 65 
That is, at 12.12 noon. = 140 km.

MULTIPLE CHOICE QUESTIONS

1. The metro trains which travel at a uniform speed him, then what is the time-gap between one train that
between stations A and B run at a regular interval of overtakes him from behind and the immediately next
12 min. If Ajay, walking along the railway track at train overtaking him?
uniform speed, observes that every 10 min there is a (a) 15 mins (b) 16 mins
train coming from the opposite direction and passes
(c) 13.5 mins (d) None of these

Chapter_12.indd 279 1/30/2016 5:35:36 PM


280  Chapter 12

2. In covering a distance of 30 km, Amit takes 2 hrs more make same number of revolutions per second. If both of
than Suresh. If Amit doubles his speed, then he would them meet after 10 s, the speed of the smaller wheel is
take 1 hour less than Suresh. Amit’s speed is (a) 22 cm/s (b) 44 cm/s
(a) 5 km/hr (b) 7.5 km/hr (c) 66 cm/s (d) 132 cm/s
(c) 6 km/hr (d) 6.25 km/hr [Based on MAT, 2003]
[Based on MAT, 2003] 9. Two champion swimmers start a two-length swimming
3.
Two persons A and B are at two places P and Q, race at the same time but from opposite ends of the pool.
respectively. A walks at v km/hr and B is 2 km/hr faster They swim in line and at constant but different speeds.
than A, starting simultaneously from where they stand. If They first pass at a point 18.5 m from the deep end and
they walk towards each other, they meet in 72 min. If they having completed one length, each swimmer is allowed to
walk in the same direction, the faster overtakes the slower rest on the edge of the pool for 45 s. After setting off on
in 6 hrs. Find their respective speeds (in km/hr). the return length, the swimmers pass for the second time
just 10.5 m from the shallow end. Thus, the length of the
(a) 3 and 5 (b) 4 and 6 pool is
1 1 1 1 (a) 90 m (b) 45 m
(c) 2 and 4 (d) 3 and 5
2 2 2 2 (c) 26 m (d) 52 m
4. Two cars are running along the same road. The first one, 10. An express train travelled at an average speed of
which is running at the rate of 30 km/hr, starts 6 hrs 100 km/hr, stopping for 3 mins after every 75 km. A
ahead of the second one, which is running at the rate of local train travelled at a speed of 50 km/hr, stopping for
50 km/hr. How long will it take the second car to catch up 1 minute after every 25 km. If the trains began travelling
with the first one? at the same time, then how many kilometres did the local
(a) 6 hrs (b) 9 hrs train travel in the time it took the express train to travel
600 km?
(c) 12 hrs (d) 15 hrs
(a) 307.5 km (b) 900 km
[Based on MAT, 2003]
(c) 1200 km (d) 1000 km
5. A train can travel 20% faster than a car. Both start from [Based on MAT, 2003]
a point A at the same time and reach point B 75 km away
from A at the same time. On the way, however, the train 11. Two trains of equal length are running on parallel lines in
lost about 12.5 mins while stopping at the stations. Find the same direction at 46 km and 36 km/hr. The faster train
the speed of the car in km/hr. passes the slower train in 36 s. The length of each train is
(a) 50 m (b) 80 m
(a) 50 km/hr (b) 55 km/hr
(c) 72 m (d) 82 m
(c) 60 km/hr (d) 65 km/hr
[Based on MAT, 2003]
[Based on MAT, 2003]
12. There are 20 poles with a constant distance between each
6. A train can travel 20% faster than a car. Both start from pole. A car takes 24 s to reach the 12th pole. How much
the point A at the same time and reach point B 75 km away time will it take to reach the last pole?
from A at the same time. On the way, however, the train
(a) 25.25 s (b) 17.45 s
lost about 12.5 mins while stopping at the stations. The
speed of the car is (c) 35.75 s (d) 41.45 s
[Based on MAT, 2003]
(a) 50 km/hr (b) 55 km/hr
(c) 60 km/hr (d) 65 km/hr 13. The jogging track in a sports complex is 726 m in
circumference. Suresh and his wife start from the same
[Based on MAT, 2003]
point and walk in opposite directions at 4.5 km/hr and
7. A starts 3 min after B for a place 4.5 km distant B, on 3.75 km/hr, respectively. They will meet for the first time
reaching his destination, immediately returns and after in
walking a km meets A. If A can walk 1 km in 18 mins, (a) 5.5 mins (b) 6.0 mins
then what is B’s speed ?
(c) 5.28 mins (d) 4.9 mins
(a) 5 km/hr (b) 4 km/hr [Based on MAT, 2003]
(c) 6 km/hr (d) 3.5 km/hr
14.
A student got down at a tram stop A and walked the
[Based on MAT, 2003] remaining distance to school. If he had stayed in the tram
8. Wheels of diameters 7 cm and 14 cm start rolling until the next stop B and then walked to school he would
simultaneously from X and Y, which are 1980 cm apart, have taken a minute longer. If he had walked the entire
towards each other in opposite directions. Both of them distance from A to school at twice his usual speed, he

Chapter_12.indd 280 1/30/2016 5:36:54 PM


 Time and Distance  281

would have taken as much time as the tram would take for 20.
A car travelled 75% of the distance from town A to B by
travelling from A to B. If the school is 300 m from A, and travelling at T hrs at an average speed of V km/hr. The car
100 m from B, the walking speed of the student in km/hr travels at an average speed of S km/hr for the remaining
is part of the trip. Which of the following expressions
1 represents the average speed for the entire trip?
(a) 2 (b) 3
2 12VS 4VS
(a) (b)
1 V S 3S  V
(c) 3 (d) 4
2 VT
(c) (d) None of these
15.
In a shopping Mall, Raja decided to walk down the 3S
escalator. He did some quick calculation in his mind. He
found that if he walks down thirty steps, he requires 18 s Directions (Questions 21 and 22):  Refer to the following
to reach the bottom. However, if he is able to step down information and answer the questions that follow.
thirty four stairs, he would only require 6 s to get to the
   Two trains A and B, 100 m long are moving on parallel tracks
bottom. If the time is measured from the moment the top
at speeds of 20 m/s and 30 m/s respectively. They are travelling in
step begins to descend to the time he steps off the last step
opposite direction.
at the bottom, the height of the stair way in steps is
21. A boy in front of train A sees another boy in front of train
(a) 46 (b) 38
B when he is closest to high. He throws a ball at a speed
(c) 36 (d) 32 of 2 m/s which hits the tail of train B. What is the distance
16. A circular running path is 726 m in circumference. Two between the two trains?
men start from the same point and walk in opposite (a) 0 m (b) 10 m
directions @ 3.75 km/hr and 4.5 km/hr respectively. (c) 4 m (d) 8 m
When will they meet for the first time?
22. If now the trains are travelling in the same direction
(a) 5.5 mins (b) 6.0 mins and the boy repeats his action according to the previous
(c) 5.28 mins (d) 4.9 mins question where will the ball hit?
[Based on MAT, 2002] (a) 10 m from the front of train B
17. The distance of P from Q is 7 km. An aeroplane flew from (b) 20 m from the front of train B
P to Q against the wind and then comes back in 22 mins. If (c) at the front of train A
its speed was decreased by 12½% flying against the wind (d) at the tail of train B
and increased by 5% when flying with the wind, how long
would the flight have taken, had there been no wind? 23.
It takes eight hrs for a 600 km journey, if 120 km is done
(a) 15 mins (b) 21 mins by train and the rest by car. It takes 20 mins more, if
200 km is done by train and the rest by car. The ratio of
(c) 36 mins (d) 60 mins the speed of the train to the speed of the car is
18.
Two trains 100 m and 80 m long, run at the rate of 30 km/ (a) 4:3 (b) 3:4
hr and 50 km/hr respectively on parallel rails. How long (c) 3:2 (d) 2:3
will it take a man sitting in the second train to pass the first [Based on MAT, 1999]
train if they run in opposite direction? 24.
Points A and B are 70 km apart on a highway. One car
(a) 2.4 s (b) 4.5 s starts from A and another one from B at the same time. If
(c) 8.3 s (d) 11.2 s they travel in the same direction, they meet in 7 hrs. But if
they travel towards each other, they meet in one hour. The
19.
Two trains, 100 km apart, travel towards each other on the speeds of the two cars are
same track. One train travels at 40 km/hr; the other travels (a) 45 and 25 km/hr (b) 70 and 10 km/hr
at 60 km/hr. A bird starts flying at a speed of 90 km/hr,
(c) 40 and 30 km/hr (d) 60 and 40 km/hr
at the same location of the faster train. When it reaches
[Based on MAT, 1999]
the slower train, it turns around and flies in the opposite
direction at the same speed. When it reaches the faster 25.
If train runs at 40 km/hr, it reaches its destination late by
train, again it turns around and so on. When the two trains 11 mins but if it runs at 50 km/hr it is late by 5 mins only.
collide, how far has the bird flown? The correct time for the train to complete its journey is
(a) 90 km (b) 45 km (a) 13 mins (b) 15 mins
(c) 180 km (d) 135 km (c) 19 mins (d) 21 mins
[Based on MAT, 2005]

Chapter_12.indd 281 1/30/2016 4:17:57 PM


282  Chapter 12

26.
A train of length 150 m takes 40.5 s to cross a tunnel of 33. A lives on 9th floor and B lives on 44th floor. A goes up at
length 300 m. The speed of the train (in km/hr) is a rate of 34 floors per minute and B comes down at a rate
(a) 13.33 (b) 26.67 of 36 floors per minute. At which floor they will meet?
(c) 40 (d) 400 (a) 27 (b) 26
[Based on MAT, 2005] (c) 18 (d) 32

27.
A train 100 m long passes a bridge at the rate of 72 km/hr 34.
Two trains of equal length are running on parallel lines
in 25 s. The length of the bridge is in the same direction @ 46 km and 36 km/hr. The faster
train passes the slower train in 36 s. The length of each
(a) 150 m (b) 400 m
train is
(c) 300 m (d) 200 m
(a) 50 mts. (b) 80 mts.
[Based on MAT, 1999]
(c) 72 mts. (d) 82 mts.
28.
A train 110 m in length travels at 60 km/hr. How much [Based on MAT, 2001]
time does the train take in passing a man walking at 6 km/
35. A car travelled 80 km at 16 km/hr. What should be its
hr against the train?
average speed for next 4 hrs so that the total average speed
(a) 6 s (b) 12 s becomes 14 km/hr?
(c) 16 s (d) 18 s (a) 15 km/hr (b) 11.5 km/hr
[Based on MAT, 1999] (c) 15.4 km/hr (d) 12 km/hr
29. A train of 24 m length runs with a speed of 250 m/s. A 36. Dinesh travels 760 km to his home, partly by train and
man in the train at the tail end of the train runs with a partly by car. He takes 8 hrs if he travels 160 km by train
speed of 10 m/s. When he reaches the front end he turns and the rest by car. He takes 12 mins more if he travels
back with a speed of 6 m/s and this process continues. 240 km by train and the rest by car. The speeds of the train
How many rounds (up and down) he will complete if the and the car respectively are
train runs 8 kms, providing that during running he will not
loose contact with the train? (a) 80 km/hr, 100 km/hr (b) 100 km/hr, 80 km/hr
(c) 120 km/hr, 100 km/hr (d) 100 km/hr, 120 km/hr
(a) 3 (b) 4
[Based on MAT, 2001]
(c) 5 (d) 6
37.
In a flight of 3000 km, an aircraft was slowed down by
30.
Two rackets approach each other, one at 42,000 mph and bad weather. Its averae speed for the trip was reduced by
the other at 18,000 mph. They start 3256 miles apart. How 100 km/hr and the time increased by one hour. Find the
far are they apart (in miles) 1 min before impact? original duration of the flight.
(a) 1628 (b) 1000 (a) 5 hrs (b) 6 hrs
(c) 826 (d) 1200 (c) 4 hrs (d) 10 hrs
[Based on MAT, 2000] [Based on MAT, 2001]

31.
Train X starts at 6.00 a.m. from a certain station with P 38.
The diameter of a cycle wheel is 70 cm. A cyclist takes
km/hr and train Y starts at 8.30 a.m. from the same station 30 hrs to reach a destination at the speed of 22 km/hr.
at Q km/hr. If Q > P, then how many hrs will train Y take How many revolutions will the wheel make during this
to overtake train X? journey?
5P 2P (a) 3 million (b) 3 lakh
(a) hrs (b) hrs
2(Q  P ) 5(Q  P ) (c) 4 lakh (d) None of these
[Based on MAT, 2001]
2(Q  P ) 5(Q  P )
(c) hrs (d) hrs 39. Rashmi leaves office at 6.00 p.m. and catches a 6.30 p.m.
5P 2P
local train that arrives in her town at 7.00 p.m. Her father
32.
Wheels of diameters 7 cm and 14 cm start rolling leaves home to pick her up at 7.00 p.m. from the station
simultaneously from X and Y, 1,980 cm apart, towards as she gets off the train. Yesterday, Rashmi left her office
each other in opposite directions. Both of them make early and took a 6.00 pm train and arrived at 6.30 p.m. As
same number of revolutions per second. If both of them her father was not there to pick her up, she started walking
meet after 10 s, find the speed of the smaller wheel. towards home. Her father left home at the usual time, saw
her daughter walking, turned around, picked her up and
(a) 22 cm/s (b) 44 cm/s
drove home, arriving there 10 mins earlier than the usual.
(c) 66 cm/s (d) 132 cm/s For how long did Rashmi walk before her father picked
[Based on MAT, 2001] her up?

Chapter_12.indd 282 1/30/2016 4:17:58 PM


 Time and Distance  283

(a) 10 mins (b) 15 mins 46. Vaibhav can walk up a moving ‘up’ escalator in 30 s. He
(c) 20 mins (d) 25 mins can also walk down this moving ‘up’ escalator in 90 s. If
[Based on FMS (Delhi), 2004] his walking speed is same upwards and downwards, then
how much time will he take to walk up the escalator, when
40. There are two friends A and B. A starts with his car at the escalator is stationary?
the speed 40 km/hr. B starts one hour later from the same
(a) 30 s (b) 45 s
place, in the same direction on his bike at the speed 50
km/hr. After how many hrs they will meet? (c) 60 s (d) 90 s
(a) 12 hrs (b) 6 hrs 47.
Points A and B are 60 km apart. A bus starts from A and
(c) 4 hrs (d) 11 hrs another from B at the same time. If they go in the same
direction, then they meet in 6 hrs and if they go in opposite
41. Ram travels from P to Q at 10 km/hr and returns at 15 km/ directions, they meet in 2 hrs. The speed of the bus with
hr. Sham travels from P to Q and returns at 12.5 km/hr. If greater speed is
he takes 12 mins less than Ram, then what is the distance
(a) 10 km/hr (b) 20 km/hr
between P and Q?
(c) 30 km/hr (d) 40 km/hr
(a) 60 km (b) 45 km
[Based on FMS (Delhi), 2003]
(c) 36 km (d) 30 km
[Based on IIT Joint Man. Ent. Test, 2004] 48. Walking at three-fourths of his usual pace, a man reaches
his office 20 mins late. Find out his usual time.
42. Two trains are 35 m apart and running in same direction
with speeds 30 km/hr and 45 km/hr, respectively. If they (a) 1 hr (b) 2 hrs
cross each other in 5 mins then what is the total length of (c) 3 hrs (d) 4 hrs
both the trains? [Based on IIFT, 2003]
(a) 354 m (b) 225 m 49. A train travelled 75% of the way from town X to
(c) 1215 m (d) 1322 m town Y by travelling for A hrs at an average speed of
B km/hr. The train travels at an average speed of S km/
43. Mir went on a ten-mile test drive of his new imported hr for the remaining part of the journey. Which of the
bike. He started with a certain speed and after covering following expressions represents the average speed of the
each mile, his speed decreased by 20% for the next mile. entire journey?
If he took 5 mins to cover the first five miles of the drive,
what is the approximate time taken by him to cover the (a) 0.75B + 0.25 (b) (4BS)/(3S + B)
next five miles? (c) AB/3S (d) 0.75A + 0.25S
(a) 14 mins and 14 s [Based on SCMHRD Ent. Exam., 2003]
(b) 15 mins and 15 s 50. Two trains traveling in the same direction at 40 km/hr
(c) 16 mins and 16 s and 22 km/hr completely pass each other in 1 min. If the
length of the first train is 125 m, what is the length of the
(d) 17 mins and 17 s
second train?
44. A motorcyclist goes from Delhi to Bharatpur, a distance (a) 125 m (b) 150 m
of 192 km, at an average speed of 32 km/hr. Another man (c) 175 m (d) 200 m
starts from Delhi by car 2.5 hrs after the motorcyclist
started and reaches Bharatpur half an hour late. What is 51. Plane A takes off at 4 pm and flies at a constant speed of
the ratio of speed of the person on the motorcycle to the X mph. Plane B takes off at 5.30 p.m. and flies the same
person going by the car? route as that of A at a constant speed of Y mph. Assuming
(a) 1:2 (b) 2:3 that Y is greater than X, how many hrs after 5:30 p.m. will
the plane B overtake plane A?
(c) 10:27 (d) 5:4
(a) 3/2 X hrs (b) 3/(2Y) hrs
45. To get to a business meeting, John drove m miles in hrs
(c) 3/2 (Y – X) hrs (d) 3X/2 (Y – X) hrs
hrs, and arrived 1/2 hour early. At what rate should he
[Based on SCMHRD Ent. Exam., 2003]
have driven to arrive exactly on time?
m 2m 52.
A 100m long train passes a man, running in the same
(a) (b) direction at 6 km/hr, in 5 s and a car travelling in the same
2h 2h + 1
direction in 6 s. At what speed is the car travelling (length
2m 2m − h of both the man and car is negligible)?
(c) (d)
2h − 1 2h (a) 18 km/hr (b) 20 km/hr
[Based on REC Tiruchirapalli, 2003] (c) 24 km/hr (d) 30 km/hr

Chapter_12.indd 283 1/30/2016 4:17:58 PM


284  Chapter 12

53. Manu and Tanu run back and forth between the town hall 60. A circular playground has an area of 616 sq. m. What time
and the county station at respective speeds of 12 km/hr will it take for a runner to run around the circular ground
and 18 km/hr. They start simultaneously — Manu from at the speed of 22 km/hr?
the town hall and Tanu from the county station. If they (a) 4 hrs (b) 3 hrs
cross each other for the first time 14 mins from the start, at
(c) 2 hrs (d) None of these
what distance from the county station will they cross each
[Based on I.P. Univ., 2002]
other for the fifth time?
(a) 4.2 km (b) 2.8 km 61. A bus covered a distance of 250 km, partly at an average
speed of 40 km/hr and partly at 60 km/hr. If the total time
(c) 3.6 km (d) None of these
taken is 5 hrs, then the distance covered at 40 km/hr is
54. Excluding stoppages, the speed of a train is 45 km/hr and (a) 130 km (b) 120 km
including stoppages, it is 36 km/hr. For how many mins, (c) 100 km (d) None of these
does the train stop per hour?
(a) 10 (b) 12 62. Only a single rail track exists between stations A and B on a
railway line. One hour after the north bound superfast train
(c) 15 (d) 18 N leaves station A for station B, a south bound passenger
55.
If the speed of a railway train is increased by 5 km/hr from train S reaches station A from station B. The speed of the
its normal speed, then it would have taken 2 hrs less for a superfast train is twice that of a normal express train E,
journey of 300 km, What is its normal speed? while the speed of a passenger train S is half that of E.
On a particular day N leaves for station B from station A,
(a) 10 km/hr (b) 25 km/hr
20 mins behind the normal schedule. In order to maintain
(c) 20 km/hr (d) 30 km/hr the schedule, both N and S increased their speeds. If the
[Based on I.P. Univ., 2002] superfast train doubles its speed, what should be the ratio
56.
An aircraft was to take off from a certain airport at 8 a.m. (approximately) of the speed of passenger train to that of
but it was delayed by 30 mins. To make up for the lost the superfast train so that passenger train S reaches exactly
time, it was to increase its speed by 250 km/hr from the at the scheduled time at station A on that day.
normal speed to reach its destination 1500 km away, on (a) 1:3 (b) 1:4
time. What was the normal speed of the aircraft? (c) 1:5 (d) 1:6
(a) 650 km/hr (b) 750 km/hr [Based on CAT, 2002]
(c) 850 km/hr (d) 1000 km/hr 63. On a 20 km tunnel connecting two cities A and B, there
[Based on I.P. Univ., 2002] are three gutters. The distance between gutters 1 and 2 is
half the distance between gutters 2 and 3. The distance
57.
Raja was on a long distance trip. He travelled by air 2/5 of
from city A to its nearest gutter, gutter 1 is equal to the
the distance which was 1200 km. Then he hired a car and
distance of city B from gutter 3. On a particular day, the
travelled 1/3 of the whole trip. Thereafter, he completed
hospital in city A receives information that an accident has
the rest of the journey by train. Calculate the distance that
happened at the third gutter. The victim can be saved only
Raja travelled by train.
if an operation is stated within 40 mins. An ambulance
(a) 480 km (b) 800 km started from city A at 30 km/hr and crossed the first gutter
(c) 1600 km (d) 1800 km after 5 mins. If the driver had doubled the speed after
[Based on I.P. Univ., 2002] that, what is the maximum amount of time the doctor
would get to attend the patient at the hospital. Assume 1
58. A train T1 starts from Ahmedabad to Mumbai at 7 a.m. minute is elapsed for taking the patient into and out of the
and reaches at 12 noon. A second train T2 starts at 7 a.m. ambulance.
from Mumbai reaches Ahmedabad at 1 p.m. When did the
(a) 4 mins (b) 2.5 mins
two trains cross each other?
(c) 1.5 mins
(a) 10.13 a.m. (b) 10.00 a.m.
(d) Patient died before reaching the hospital.
(c) 9.43 a.m. (d) 9.35 a.m.
[Based on CAT, 2002]
59. If a child walks at the rate of 5 m/min from his home, he is 64. At a bookstore, “MODERN BOOK STORE” is flashed
6 mins late for school; if he walks at the rate of 7 m/min, using neon lights. The words are individually flashed at
he reaches half an hour earlier. How far is his school from
his home? 1 1 1
intervals of 2 , 4 , 5 s respectively, and each word is
(a) 450 mins (b) 540 mins 2 4 8
(c) 630 mins (d) 360 mins put off after a second. The least time after which the full
name of the bookstore can be read again, is:

Chapter_12.indd 284 1/30/2016 4:17:58 PM


 Time and Distance  285

(a) 49.5 s (b) 73.5 s double his usual rowing rate for this 24 mile round trip,
(c) 1744.5 s (d) 855 s the downstream 12 miles would then take only one hour
[Based on CAT, 2002] less than the upstream 12 miles. What is the speed of the
current in miles per hour?
65. A train approaches a tunnel AB. Inside the tunnel a cat
(a) 7/3 (b) 4/3
located at a point that is three-eighths of the distance AB
measured from the entrance A. When the train whistles, (c) 5/3 (d) 8/3
the cat runs. If the cat moves to the entrance of the tunnel, [Based on CAT, 2001]
A, the train catches the cat exactly at the entrance. If the
71. Shyam and Vyom walk up an escalator (moving stairway).
cat moves to the exit B, the train catches the cat at exactly
The escalator moves at a constant speed. Shyama takes
the exit. The speed of the train is greater than the speed of
three steps for every two of Vyom’s steps. Shyam gets to
the cat by what order?
the top of the escalator after having taken 25 steps, while
(a) 3:1 (b) 4:1 Vyom (because his slower pace lets the escalator do a little
(c) 5:1 (d) None of these more of the work) takes only 20 steps to reach the top. If
[Based on CAT, 2002] the escalator were turned off, how many steps would they
66. Six technicians working at the same rate completely work have to take to walk up?
of one server in 10 hrs. If they start at 11: 00 a.m. and one (a) 40 (b) 50
additional technician per hour being added beginning at (c) 60 (d) 80
5:00 p.m., at what time the server will be complete? [Based on CAT, 2001]
(a) 6:40 p.m. (b) 7.00 p.m.
72. There’s a lot of work in preparing a birthday dinner. Even
(c) 7:20 p.m. (d) 8:00 p.m
after the turkey is in the oven, there’s still the potatoes and
[Based on CAT, 2002] gravy, yams, salad, and cranberries, not to mention setting
67. A tiger is 50 of its own leaps behind a deer. The tiger takes the table.
5 leaps per min to the deer’s 4. If the tiger and the deer Three friends, Asit, Arnold, and Afzal, work together
cover 8 m and 5 m per leap respectively, what distance to get all of these chores done. The time it takes them to
will the tiger have to run before it catches the deer? do the work together is six hrs less than Asit would have
(a) 600 m (b) 700 m taken working alone, one hour less than Arnold would
(c) 800 m (d) 1000 m have taken alone, and half the time Afzal would have
taken working alone.
68. A can complete a piece of work in 4 days. B takes double
How long did it take them to do these chores working
the time taken by A, C takes double that of B, and D takes
together?
double that of C to complete the same task. They are
paired in groups of two each. One pair takes two-thirds (a) 20 mins (b) 30 mins
the time needed by the second pair to complete the work. (c) 40 mins (d) 80 mins
Which is the first pair? [Based on CAT, 2001]
(a) A, B (b) A, C
73. Two trains, 130 m 110 m long, are going in the same
(c) B, C (d) A, D direction. The faster train takes one minute to pass
[Based on CAT, 2001] the other completely. If they are moving in opposite
69. Two men X and Y started working for a certain company directions, they pass each other completely in 3 s. Find
at similar jobs on January 1, 1950. X asked for an initial the speed of each train.
salary of `300 with an annual increment of `30. Y asked (a) 38 m/s, 36 m/s (b) 42 m/s, 38 s
for an initial salary of `200 with a rise of `15 every six
(c) 36 m/s, 42 m/s (d) None of these
months. Assume that the arrangements remained unaltered
till December 31, 1959. Salary is paid on the last day of [Based on MAT, 2008]
the month. What is the total amount paid to them as salary 74. Two identical trains A and B running is opposite directions
during the period? at same speed take 2 mins to cross each other completely.
(a) `93,300 (b) `93,200 The number of bogies of A are increased from 12 to 16.
(c) `93,100 (d) None of these How much more time would they now require to cross
each other?
[Based on CAT, 2001]
(a) 40 s (b) 50 s
70.
At his usual rowing rate, Rahul can travel 12 miles
downstream in a certain river in six hrs less than it takes (c) 60 s (d) 20 s
him to travel the same distance upstream. But if he could [Based on SNAP, 2007]

Chapter_12.indd 285 1/30/2016 4:17:58 PM


286  Chapter 12

Directions for questions (75 to 76): The petrol consumption (a) 140 m (b) 80 m
rate of a new model car ‘Palto’ depends on its speed and may be (c) 340 m (d) 360 m
described by the graph below: [Based on CAT, 2000]
78. Arun, Barun and Kiranmala start from the same place and
travel in the same diretion at speed of two hour after Arun.
If Barun and Kiranmala overtake Arun at the same instant,
how many hrs after Arun did Kiranmala starts?
(a) 3 (b) 3.5
(c) 4 (d) 4.5
[Based on CAT, 2006]
79. Golu and Mayank start running simultaneously. Golu runs
from point A to point B and Mayank from point B to point
75. Manasa makes the 200 km trip from Mumbai to Pune at
6
a steady speed of 60 km/hr. What is the amount of petrol A. Golu’s speed is
of Mayank’s speed. After crossing
consumed for the journey? 5
(a) 12.5 litres (b) 13.33 litres 1
Mayank, if Golu takes 2 h to reach B, how much time
(c) 16 litres (d) 19.75 1itres 2
[Based on CAT, 2001] does Mayank take to reach A after crossing Golu?
(a) 3 hrs 6 min (b) 3 hrs 16 min
76. Manasa would like to minimize the fuel consumption for
(c) 3 hrs 26 min (d) 3 hrs 36 min
the trip by driving at the appropriate speed. How should
she change the speed? 80. Shruti and Archana start travelling together in the same
(a) Increase the speed direction at 8 km/hr and 13 km/hr respectively. After 4 h
(b) Decrease the speed Shruti doubles her speed and Archana reduces her speed
by 1 km/hr and reached the destination together. How
(c) Maintain the speed at 60 km/hrour
long the entire journey last?
(d) None of these
(a) 5 hrs (b) 9 hrs
[Based on CAT, 2001]
1 1
Directions for questions (77 to 78): Answer the questions based (c) 7 hhrs (d) 9 hhrs
2 2
on the following in formation.
Directions (Questions 81 and 82): Answer the questions based on
    There are five machines — A, B, C, D, and E — situated
the following information.
on a straight line at distances of 10m, 20m, 30m, 40m and 50m
respectively from the origin of the line. A robot is stationed at the     A thief, after committing the burglary, started fleeing at 12
origin of the line. The robot serves the machines with raw material noon, at a speed of 60 km/hr. He was then chased by a policeman
whenever a machine becomes idle. All the raw materials are X. X started the chase, 15 min after the thief has started, at a speed
located at the origin. The robot is in an idle state at the origin at of 65 km/hr.
the beginning of a day. As soon as one or more machines become 81. At what time did X catch the thief?
idle, they send messages to the robot-station and the robot starts
(a) 3.30 p.m. (b) 3 p.m.
and serves all the machines from which it received messages. If a
message is received at the station while the robot is away from it, (c) 3.15 p.m. (d) None of these
the robot takes notice of the message only when it returns to the 82. If another policeman had started the same chase along
station. While moving, it serves the machines in the sequence in with X, but at a speed of 60 km/hr, then how far behind
which they are encountered, and then returns to the origin. If any was he when X caught the thief?
messages are pending at the station when it returns, it repeats the (a) 18.75 km (b) 15 km
process again. Otherwise, it remains idle at the origin till the next (c) 21 km (d) 37.5 km
message(s) is (are) received.
Directions (Questions 83 to 85): Answer the questions based on
77. Suppose on a certain day, machines A and D have sent the following information.
the first two messages to the origin at the beginning of the
first second, C has sent a message at the beginning of the    Boston is 4 hrs ahead of Frankfurt and 2 hrs behind India. X
5th second, B at the beginning of the 6th second and E at leaves Frankfurt at 6 p.m. on Friday and reaches Boston the next
the beginning of the 10th second. How much distance has day. After waiting there for 2 h, he leaves exactly at noon and
the robot travelled since the beginning of the day, when reaches India at 1 a.m. On his return journey, he takes the same
it notices the message of E? Assume that the speed of route as before, but halts at Boston for 1 hrs less than his previous
movement of the robot is 10 m/s. halt there. He then proceeds to Frankfurt.

Chapter_12.indd 286 1/30/2016 5:38:07 PM


 Time and Distance  287

83. If his journey, including stoppage, is covered at an average 89.


A 180 m long train crosses another 270 m long train
speed of 180 mph, what is the distance between Frankfurt running in the opposite direction in 10.8 s. If the speed of
and India? the first train is 60 km/hr, what is the speed of the second
train in km/hr?
(a) 3,600 miles (b) 4,500 miles
(a) 80 (b) 90
(c) 5,580 miles (d) Data insufficient
(c) 150 (d) Cannot be determined
84. If X had started the return journey from India at 2.55 a.m. [Based on Gramin Bank U.P. (SO) Exam, 2012]
on the same day that he reached there, after how much 90.
Paschim Express left Delhi for Mumbai at 14.30 hrs
time would he reach Frankfurt? travelling at a speed of 60 km/hr. August Kranti Express
(a) 24 hrs (b) 25 hrs left Delhi for Mumbai on the same day at 16.30 hrs
travelling at a speed of 80 km/hr. How far away from
(c) 26 hrs (d) Data insufficient Delhi will the two trains meet (stop-pages excluded)?
85. What is X’s average speed for the entire journey (to and (a) 500 kms (b) 480 kms
fro)? (c) 360 kms (d) 240 kms
(a) 176 m/h (b) 180 m/h [Based on U.P. P.C.S., 2012]

(c) 165 m/h (d) Data insufficient 91.


A man starts from his home and walks 10 m towards
South. Then he turns right and walks 6 km, again he turns
86. Two straight roads R1 and R2 diverge from a point A at right and goes 10 km. Finally he turns right and walks 5
an angle of 120º. Ram starts walking from point A along km. At what distance is he from his starting point?
R1 at a uniform speed of 3 km/hr. Shyam starts walking (a) 31 km (b) 2 101 km
at the same time from A along R2 at a uniform speed
of 2km/hr. They continue walking for 4 hrs along their (c) 1 km (d) 125 + 136 km
respective roads and reach points B and C on R1 and R2, [Based on U.P. P.C.S., 2012]
respectively. There is a straight line path connecting B and
92.
City Bus Corporation runs two buses from terminus A to
C. Then Ram returns to point A after walking along the
terminus B, each bus making 5 round trip in a day. There
line segments BC and CA. Shyam also returns to A after
are no stops in between. These buses play back and forth
walking along line segments CB and BA. Their speeds
on the same route at different but uniform speeds. Each
remain unchanged. The time interval (in hrs) between
morning the buses start at 7 a.m. from the respective
Ram’s and Shyam’s return to the point A is
terminuses. They meet for the first time at a distance of 7
10 19  26 2 19  10 km from terminus A. Their next meeting is at a distance
(a) (b) of 4 km from terminus B, while travelling in opposite
3 3
directions. Assuming that the time taken by the buses at
terminuses is negligibly small, and the cost of running a
19  26 19  10
(c) (d) bus is `20 per km, find the daily cost of running the buses
3 3 (In `).
87.
A train, 300 m long, passed a man, walking along the line (a) 3200 (b) 4000
in the same direction at the rate of 3 km/hr in 33 s. The (c) 6400 (d) 6800
speed of the train is [Based on XAT, 2012]
(a) 30 km/hr (b) 32 km/hr 93. Car A runs at the speed of 65 km/hr and reaches its
destination in 8 hrs. Car B runs at the speed of 70 km/hr
8 8 and reaches its destination in 4 hrs. What is the respective
(c) 32 km/hr (d) 35 km/hr
11 11 ratio of distances covered by Car A and Car B?
[Based on SSC (GL), 2010] (a) 11:7 (b) 7:13
(c) 13:7 (d) 7:11
88.
Buses start from a bus terminal with a speed of 20 km/hr [Based on Syndicate Bank PO, 2010]
at intervals of 10 mins. What is the speed of a man coming
from the opposite direction towards the bus terminal if he 94. Deepa drives a bike at an average speed of 30 km/hr and
meets the buses at intervals of 8 mins? reach her destination in 6 hrs. Hema covers that distance
in 4 hrs. If Deepa increases her average speed by 10 km/
(a) 3 km/hr (b) 4 km/hr hr and Hema increases her average speed by 5 km/hr,
(c) 5 km/hr (d) 7 km/hr then what will be the difference in time taken by them to
[Based on SSC (GL), 2010]
reach their destination?

Chapter_12.indd 287 1/30/2016 4:18:00 PM


288  Chapter 12

(a) 54 mins (b) 1 hr far will the hare have gone when the hound will catch the
(c) 40 mins (d) 45 mins hare?
[Based on Syndicate Bank PO, 2010] (a) 60 m (b) 120 m
95. The ratio between the speed of a train and a car is 16:15, (c) 90 m (d) 100 m
respectively. Also, a bus covered a distance of 480 km in [Based on ATMA, 2008]
8 hrs. The speed of the bus is three-fourth the speed of the
train. How much distance will the car cover in 6 hrs? A Mariti 800 crosses a lamp post in 4 × 4 2 s . If the
101.
speed of the car is denoted by 2x m/s and length of the car
(a) 450 km (b) 480 km
is 23.25 m, then actual speed of the car is
(c) 360 km (d) Cannot be determined
(a) 3.25 m/s (b) 4 m/s
[Based on Bank of Baroda PO, 2010]
1
96. Two cities A and B, at a distance of 50 km, are connected (c) 8 m/s (d) 1 m/s
by two separate roads. The speed of any vehicle travelling 4
between the two cities on road 1 is 50 km/hr, while the [Based on ATMA, 2008]

 80  102.
A car driver driving in fog, passes a pedestrian who was
speed on road 2 is   km/hr, where n is the number of
 n  walking at the rate of 2 km/hr in the same direction. The
vehicles (including the concerned vehicle.) If you travel pedestrian could see the car for 6 min and it was visible to
in a vehicle from A to B on road 1 and come back from him upto a distance of 0.6 km. The speed of the car would
B to A on road 2 (where there are already three vehicles be
playing), your approximate average speed is (a) 8 km/hr (b) 800 m/h
(a) 26 km/hr (b) 29 km/hr (c) 200 m/h (d) 15 km/hr
(c) 32 km/hr (d) 35 km/hr [Based on ATMA, 2006]
[Based on JMET, 2009] 103. A train 270 m long is moving at a speed of 25 km/hr. It
97. A man jogging inside a railway tunnel at a constant speed will cross a man coming from the opposite direction at a
hears a train approaching the tunnel from behind at a speed of 2 km/hr in
speed of 30 km/hr, when he is one third of the way inside (a) 13 s (b) 36 s
the tunnel. Whether he keeps running forward or turns (c) 41 s (d) 20 s
back, he will reach the end of the tunnel at the same time
[Based on ATMA, 2005]
the train reaches that end. The speed at which the man is
running is 104. Two trains start simultaneously (with uniform speeds)
(a) 6 km/hr (b) 8 km/hr from two stations 270 km apart, each to the opposite
1
(c) 12 km/hr (d) 10 km/hr station; they reach their destinations in 6 hrs and 4 hrs
[Based on JMET, 2009] 4
after they meet. The rate at which the slower train travels
98. A father runs after his son, who is 1000 m ahead. The is (km/hr)?
father runs at a speed of 1 km every 8 mins and the son (a) 24 (b) 30
runs at a speed of 1 km every 12 mins. How much distance
has the son covered at the point when the father overtakes (c) 25 (d) 16
him? [Based on ATMA, 2005]
(a) 2500 m (b) 2000 m 105. A dog takes 4 leaps for every five leaps of a hare, but three
(c) 1500 m (d) 1000 m leaps of dog are equal to four leaps of hare. Compare their
[Based on JMET, 2009] speeds.
(a) 16:15 (b) 17:18
99. A 320 m long train moving with an average speed of 120
km/hr crosses a platform in 24 s. A man crosses the same (c) 19:20 (d) None of these
platform in 4 min. What is the speed of man in m/s? [Based on NMAT, 2006]
(a) 2.4 (b) 1.5 106. A and B walk around a circular course 35 km in
(c) 1.6 (d) 2.0 circumference starting together from the same point. If
[Based on Bank of Baroda PO Exam, 2011] they walk at the speed of 4 km/hr and 5 km/hr, respectively
in the same direction, when will they meet?
100.
A hare pursued by a round is 30 m before the hound at
starting. Whilst the hare takes 4 leaps the hound takes 3. (a) 2 mins (b) 1 min
1 1 (c) 3 mins (d) None of these
In one leap the hare goes 1 and the hound 2 m. How [Based on NMAT (2006]
2 2

Chapter_12.indd 288 1/30/2016 4:18:00 PM


 Time and Distance  289

107.
Two stations A and B are 110 km apart on a straight line. 114. A man walked at a speed of 4 km/hr from point A to B
One train train starts from ‘A’ at 7 am and travel towards and come back from point B to A at the speed of 6 km/hr.
‘B’ at 20 km/hr speed. Another train starts for ‘B’ at 8 am What would be the ratio between the time taken by man in
and travel towards ‘A’ at 25 km/hr speed. At what time walking from point A to B to point B to A respectively?
will they meet? (a) 5:3 (b) 2:3
(a) 9 a.m. (b) 10 a.m. (c) 2:1 (d) 4:3
(c) 11 a.m. (d) None of these [Based on Corporation Bank PO, 2009]
[Based on NMAT, 2006]
115.
As bus started its journey from Ramgarh and reached
108. The speed of a train pulling out of a station is given by Devgarh in 44 mins with its average speed of 50 km/hr. If
the equation s = t2 + t, where s is the speed in kilometers the average speed of the bus is increased by 5 km/hr, how
per hour and t is the time is seconds from when the train much time will it take to cover the same distance?
starts moving. The equation holds for all situations where (a) 40 mins (b) 38 mins
0 ≤ t ≤ 4. In kilometers per hour, what is the difference in (c) 36 mins (d) 31 mins
the speed of the train four seconds after it starts moving,
[Based on Corporation Bank PO, 2009]
compared to the speed two seconds after it starts moving?
(a) 14 (b) 6 116. The bus fare for one person is `420 from Agra to Aligarh
and train fare between the same places for one person is
(c) 0 (d) 20
equal to three-fourth the bus fare for two persons between
[Based on NMAT, 2005]
the same places. What is the total fare paid by 2 persons
109. A train running at the speed of 20 m/s crosses a pole in traveling by bus and 4 persons traveling by train between
24 s less than the time it requires to cross a platform thrice the two places?
its length at the same speed. What is the length of the train? (a) `3360 (b) `3460
(a) 270 m (b) 340 m (c) `3440 (d) `3406
(c) 180 m (d) None of these [Based on CBI (PO), 2010]
[Based on IRMA, 2009]
117. Train-A crosses a stationary Train-B in 50 s and a pole in
4 20 s with the same speed. The length of the Train-A is 240
110. The average speed of a car is 1 times the average speed metre. What is the length of the stationary Train-B?
5
of a bus. A tractor covers 575 km in 23 hrs. How much (a) 360 m (b) 260 m
distance will the car cover in 4 hrs if the speed of the bus (c) 300 m (d) Cannot be determined
is twice speed of the tractor? [Based on CBI (PO), 2010]
(a) 340 km (b) 480 km 118. A bike covers a certain distance at the speed of 64 km/
(c) 360 km (d) 450 km hr in 8 hrs. If the bike was to cover the same distance in
[Based on Corporation Bank PO, 2011] approximately 6 hrs, at what approximate speed should
the bike travel?
111. Train-A crosses a pole in 25 s and another Train-B crosses
a pole in 1 min and 15 s. Length of Train-A is half length (a) 80 km/hr (b) 85 km/hr
of Train-B. What is the respective ratio between the speeds (c) 90 km/hr (d) 75 km/hr
of Train-A and Train-B? [Based on Punjab National Bank PO, 2010]
(a) 3:2 (b) 3:4 119.
Two cars, an Alto and a Swift, start at the same time in
(c) 4:3 (d) Cannot be determined opposite directions from two distinct points P and Q.
[Based on Union Bank of India PO, 2011] Starting from P, the Alto reaches Q in 6 hrs 20 mins and
the Swift starting from Q, reaches Pin 4 hrs 45 mins. What
112. A 240 m long train takes 40 s longer to cross a platform
is the speed of the Swift, if the speed of the Alto is 60 km/
twice its length than the time it takes to cross a pole at the
hr?
same speed. What is the speed of the train?
(a) 110 km/hr (b) 100 km/hr
(a) 6 m/s (b) 24 m/s
(c) 90 km/hr (d) 80 km/hr
(c) 48 m/s (d) 12 m/s
[Based on JMET, 2011]
[Based on Dena Bank PO, 2008]
120.
Amarendra and Dharmendra are brothers. One day they
113. A 200 m long train crosses a platform of double its length
start at the same time from their home for Tatanagar
in 36 s. What is the speed of the train in km/hr?
railway station in their respective cars. Amarendra took
(a) 60 (b) 48 25 mins to reach the station. After reaching the station
(c) 64 (d) 66 Amarendra found that Dharmendra is 2500 m away from
[Based on SBI PO, 2008] the station. The distance of Tatanagar Station from their

Chapter_12.indd 289 1/30/2016 4:18:00 PM


290  Chapter 12

home is 15 km. Next day Dharmendra decided to start find the approximate optimal speed (in km/hr) of Fortuner
7 mins early. If they drive at the speed same as the previous that will minimize the total cost of the round trip of
day then Amarendra will reach the station 800 km.
(a) 120 s earlier than Dharmendra (a) 49 (b) 55
(b) 120 s later than Dharmendra (c) 50 (d) 53
(c) 300 s earlier than Dharmendra [Based on IIFT, 2009]
(d) 300 s later than Dharmendra 125.
The Ghaziabad-Hapur-Meerut EMU and the Meerut-
 [Based on XAT, 2010)] Hapur-Ghaziabad EMU start at the same time from
Ghaziabad and Meerut and proceed towards each other at
121.
Rajesh walks to and fro to a shopping mall. He spends
16 km/hr and 21 km/hr, respectively. When they meet, it
30 mins shopping. If he walks at speed of 10 km an hour,
is found that one train has travelled 60 km more than the
he returns to home at 19.00 hrs. If he walks at 15 km an
other. The distance between two stations is
hour, he returns to home at 18.30 hrs. How fast must he
walk in order to return home at 18.15 hrs? (a) 445 km (b) 444 km
(a) 17 km/hr (b) 17.5 km/hr (c) 440 km (d) 450 km
[Based on IIFT, 2007]
(c) 18 km/hr (d) None of these
[Based on XAT, 2009] 126.
Laxman and Bharat decide to go from Agra to Delhi for
watching a cricket match and board two different trains
122.
Mukesh, Suresh and Dinesh travel from Delhi to Mathura for that purpose. While Laxman takes the first train that
to attend Janmashtmi Utsav. They have a bike which can leaves for Delhi, Bharat decides to wait for some time
carry only two riders at a time as per traffic rules. Bike and take a faster train. On the way Laxman sitting by
can be driven only by Mukesh. Mathura is 300 km from the window-seat noticed that the train boarded by Bharat
Delhi. All of them can walk at 15 km/hr. All of them start crossed him in 12 s. Now, the faster train can travel
their journey from Delhi simultaneously and are required 180 km in 3 hrs, while the slower train takes twice as much
to reach Mathura at the same time. If the speed of bike is time to do it. Given this, mark all the correct options.
60 km/hr, then what is the shortest possible time in which
(a) If the faster train has taken 30 s to cross the entire
all three can reach Mathura at the same time?
length of the slower train, the difference between the
2 2 lengths of the two trains is 50 m.
(a) 8 hhrs (b) 9 hhrs
7 7 (b) If the faster train had been running twice as much
(c) 10 hrs (d) None of these faster, it would have taken 10 s to overtake the slower
[Based on IIFT, 2010] train.
123.
Two motorists Anil and Sunil are practicing with two (c) Had the faster train taken 24 s to cross the entire
different sports cars; Ferrari and Maclarun, on the circular length of the slower train, the length of the slower
racing track, for the car racing tournament to be held next train would have been 100 m.
month. Both Anil and Sunil start from the same point on (d) If the slower train had been running at one and a half
the circular track. Anil completes one round of the track in times of its current speed, the faster train would have
1 min and Sunil takes 2 mins to complete a round. While taken 24 s to overtake Laxman.
Anil maintains same speed for all the rounds, Sunil halves [Based on IIFT, 2006]
his speed after the completion of each round. How many 127.
Two boys A and B start at the same time to ride from Delhi
times Anil and Sunil will meet between the 6th round and to Meerut, 60 km away. A travels 4 km an hour slower
and 9th round of Sunil (6th and 9th round is excluded)? than B. B reaches Meerut and at once turns back meeting
Assume that the speed of Sunil remains steady throughout A 12 km from Meerut. The rate of A was
each round and changes only after the completion of that
round. (a) 4 km/hr (b) 8 km/hr
(a) 260 (b) 347 (c) 12 km/hr (d) 16 km/hr
[Based on FMS, 2011]
(c) 382 (d) None of these
[Based on IIFT, 2009] 128.
A train, an hour after starting, meets with an accident
which detains it for a half hour, after which it proceeds
124.
Fortuner, the latest SUV by Toyota Motors, consumes
3 1
1  1000   at of its former rate and arrives 3 hrs late. Had the
diesel at the rate of    x  L/km, when 4 2
400  x   accident happened 90 km farther along the line, it would
driven at the speed of x km/hrour. If the diesel is `35/L have arrived only 3 hrs late. The length of the trip in
and the driver is paid at the rate of `125 per hour, then kilometres was

Chapter_12.indd 290 1/30/2016 4:18:00 PM


 Time and Distance  291

(a) 400 (b) 465 134.


The radius of the wheel of a vehicle is 70 cm. The wheel
(c) 600 (d) 640 makes 10 revolutions in 5 s. The speed of the vehicle is
[Based on FMS, 2011] (a) 29.46 km/hr (b) 31.68 km/hr
129.
Two cyclist, k kilometres apart, and starting at the same (c) 36.25 km/hr (d) 32.72 km/hr
time, would be together in r hrs if they travelled in the [Based on FMS, 2006]
same direction, but would pass each other in t hrs if they 135.
Two cyclists start from the same place in opposite
travelled in opposite direction. The ratio of the speed of directions. One goes towards North at 18 km/hr and the
the faster cyclist to that of the slower is other goes towards South at 20 km/hr. What time will they
r t r take to be 47.5 km apart?
(a) (b)
r t r t 1 1
(a) 2 hhrs (b) 1 hhrs
rt r 4 4
(c) (d)
r t 1
(c) 2 hrs 23 mins (d) 23 hhrs
[Based on FMS, 2011] 4
130.
A man drives 150 km to the seashore in 3 hrs and 20 mins. [Based on FMS, 2006]
He returns from the shore to the starting point in 4 hrs and 136.
A boy is running at a speed of p km/hr to cover a distance
10 mins. Let r be the average rate for the entire trip. Then of 1 km. But, due to slippery ground, his speed is reduced
the average rate for the trip going exceeds r, in kilometres by q km/hr (p > q). If he takes r hour to cover the distance,
per hour, by then
1 1 1 1 1
(a) 5 (b) 4 (a)   (b)  p  q
2 r p q r
(c) 4 (d) 2 (c) r = p + q (d) r = p – q
[Based on FMS, 2010] [Based on FMS, 2006]
131.
Two men at points R and S, 76 km apart, set out at the 137.
The ratio between the rates of walking of A and B is 2:3
same time to walk towards each other. The man at R and therefore A takes 10 min more than the time taken by
1 B to reach the destination. If A had walked at double the
walks uniformly at the rate of 4 km/hr; the man at S
2 speed, he would have covered the distance in
1 (a) 15 mins (b) 20 mins
walks at the constant rate of 3 km/hr for the first hour
4 (c) 25 mins (d) 30 mins
3 [Based on FMS, 2005]
at 3 km/hr for the second hour, and so on, in arithmetic
4 138.
Two trains running in the same direction at 40 km/hr and
progression. If the men meet x km nearer R than S in an 22 km/hr completely pass one another in 1 min. If the
integral number of hrs, then x is length of the first train is 125 m, the length of the second
(a) 10 (b) 8 train is
(c) 6 (d) 4 (a) 125 m (b) 150 m
[Based on FMS, 2010] (c) 175 m (d) 200 m
132.
A 500 g stone was dropped from the roof of a building. [Based on FMS, 2005]
What is the height of the building if the stone reached the 139.
A man covers a certain distance on scooter. Had he moved
ground in 4 s? 3 km/hr faster, he would have taken 40 mins less. If he
(a) 108.4 m (b) 98.4 m had moved 2 km/hr slower, he would have taken 40 mins
(c) 88.4 m (d) 78.4 m more. The distance (in km) is
[Based on FMS, 2009] (a) 20 (b) 36
133.
A flight of Jet Airways from Delhi to Mumbai has an (c) 37.5 (d) 40
average speed of 700 km/hr without any stoppage, [Based on FMS, 2005]
whereas a flight of Kingfisher from Delhi to Mumbai has 140.
If a student walks from his house to school at 5 km/hr, he
an average speed of 560 km/hr with stoppage at Baroda. is late by 30 mins. However, if he walks at 6 km/hr, he is
What is the average stoppage time per hour of kingfisher late by 5 mins only. The distance of his school from his
flight if both the planes fly at the same speed? house is
(a) 8 mins (b) 12 mins (a) 2.5 km (b) 3.6 km
(c) 16 mins (d) 24 mins (c) 5.5 km (d) 12.5 km
[Based on FMS, 2009] [Based on FMS (MS), 2006]

Chapter_12.indd 291 1/30/2016 5:39:17 PM


292  Chapter 12

141.
A cyclist drove one kilometre, with the wind in his back, 148.
An express train travels 299 km between two cities.
in three mins and drove the same way back, against the During the first 111 km of the trip, the train traveled
wind in four mins. If we assume that the cyclist always through mountainous terrain. The train traveled 10 km/
puts constant force on the pedals, how much time would it hr slower through mountainous terrain than through level
take him to drive one kilometer without wind? terrain. If the total time to travel between two cities was
1 3 7 hrs, what is the speed of the train on level terrain?
(a) 2 min (b) 3 min (a) 56 km/hr (b) 55 km/hr
3 7
(c) 47 km/hr (d) 88 km/hr
3 7
(c) 2 min (d) 3 min [Based on MAT (Feb), 2011]
7 12
[Based on SNAP, 2008] 149.
Two goods trains each 500 m long are run­ning in opposite
directions, on parallel tracks. Their speeds are 45 km/
142.
Two identical trains A and B running in opposite directions hr and 30 km/hr respectively. Find the time taken by the
at same speed take 2 min to cross each other completely. slower train to pass the driver of the faster one.
The number of bogies of A are increased from 12 to 16.
(a) 60 s (b) 48 s
How much more time would they now require to cross
each other? (c) 24 s (d) 12 s
[Based on MAT, 2000]
(a) 40 s (b) 50 s
(c) 60 s (d) 20 s 150.
X express which goes from Hyderabad to Chennai, leaves
[Based on SNAP, 2007] Hyderabad at 5:30 am and travels at a constant speed of
50 km/hr towards Nalgonda which is 100 km away. At
143.
Two rockets approach each other, one at 42,000 mph and 6:00 am, Y express leaves from Nalgonda for Hyderabad
the other at 18,000 mph. They start 3256 miles apart. How at a constant speed of 40 km/hr. At 6:30 am Mr Shah, the
far are they apart (in miles) 1 minute before impact? Control Officer realizes that both the trains are on the
(a) 1628 (b) 1000 same track. How much time does Mr Shah have to avert
(c) 826 (d) 1200 the accident?
[Based on MAT, 2000] (a) 20 min (b) 30 min
144.
A certain distance is covered at a certain speed. If half of (c) 25 min (d) 15 min
this distance is covered in double the time, the ratio of the [Based on MAT (Feb), 2011]
two speeds is 151.
A man covers a certain distance on a toy train. If
(a) 4:1 (b) 1:4 the train moved 4 km/hr faster, it would take 30 min
(c) 2:1 (d) 1:2 less. If it moved 2 km/hr slower, it would have taken
[Based on MAT, 2000] 20 min more. Find the distance.
145.
Ram travels a certain distance at 3 km/hr and reaches 15 (a) 60 km (b) 45 km
min late. If he travels at 4 km/hr, he reaches 15 min earlier. (c) 30 km (d) 20 km
The distance he has to travel is [Based on MAT (Dec), 2010, 2008]
(a) 4.5 km (b) 6 km 152.
The average speed of a train is 20% less on the return
(c) 7.2 km (d) 12 km journey than on the onward journey. The train halts for
[Based on MAT, 2000] half an hour at the destination station before starting on
146.
A train covers a distance in 50 mins, if it runs at a speed of the return journey. If the total time taken for the to and
48 km/hr on an average. The speed at which the train must fro journey is 23 hrs, covering a distance of 1000 km, the
run to reduce the time of journey to 40 mins, will be speed of the train on the return journey is
(a) 50 km/hr (b) 55 km/hr (a) 60 km/hr (b) 40 km/hr
(c) 60 km/hr (d) 70 km/hr (c) 50 km/hr (d) 55 km/hr
[Based on MAT, 2000] [Based on MAT (Dec), 2010]

147.
Suresh travelled 1,200 km by air which formed (two- 153.
Two trains move from station A and station B towards
fifths) of his trip. One-third of the whole trip, he travelled each other at the speed of 50 km/hr and 60 km/hr. At
by car and the rest of the journey he performed by train. the meeting point, the driver of the second train felt that
The distance travelled by train was the train has covered 120 km more. What is the distance
between A and B?
(a) 1, 600 km (b) 800 km
(a) 1320 km (b) 1100 km
(c) 1,800 km (d) 480 km
[Based on MAT, 2000] (c) 1200 km (d) 960 km
[Based on MAT (Dec), 2010]

Chapter_12.indd 292 1/30/2016 4:18:01 PM


 Time and Distance  293

154.
A pilot flies an aircraft at a certain speed for a distance of (a) 35 mins (b) 25 mins
800 km. He could have saved 40 min by increasing the (c) 12.5 mins (d) 50 mins
average speed of the plane by 40 km/hr. Find the average [Based on MAT (May), 2010]
speed of the aircraft.
161.
Two cars A and B are travelling on the same road towards
(a) 200 km/hr (b) 300 km/hr
each other. If car A is travelling at a speed of 120 km/hr
(c) 240 km/hr (d) None of these and car B is travelling 15% slower than A, how much time
[Based on MAT (Dec), 2010] will it take the cars to meet, if the initial distance between
155.
An Auto travels 10 km/hr faster than a Scooty for a the two is 668.4 km and car A started to drive one and a
journey of 1000 km. The Scooty takes 5 hrs more than the half hour before car B started?
Auto. Find the speed of the Scooty. (a) 2 hrs and 12 min (b) 2 hrs
(a) 40 km/hr (b) 70 km/hr (c) 1 hour and 30 min (d) 3 hrs and 15 min
(c) 50 km/hr (d) None of these [Based on MAT (May), 2010]
[Based on MAT (Dec), 2010] 162.
A train travelling at 100 km/hr overtakes a motorbike
travelling at 64 km/hr in 40 s. What is the length of the
156.
A train running at 7/11 of its own speed reached a place in
train in metres?
22 hrs. How much time could be saved if the train runs at
its own speed? (a) 400 (b) 1822
(a) 7 hrs (b) 8 hrs (c) 1777 (d) 1111
[Based on MAT (May), 2010]
(c) 14 hrs (d) 16 hrs
[Based on MAT (Sept), 2010] 163.
A train travels a distance of 600 km at a constant
speed. If the speed of the train is increased by
157.
A train overtakes two persons walking along a railway 5 km/hr, the journey would take 4 hrs less. Find the speed
track. The first one walks at 4.5 km/hr. The other one walks of the train.
at 5.4 km/hr. The train needs 8.4 and 8.5 s respectively to
(a) 100 km/hr (b) 25 km/hr
overtake them. What is the speed of the train if both the
persons are walking in the same direction as the train? (c) 50 km/hr (d) None of these
[Based on MAT (May), 2010]
(a) 66 km/hr (b) 72 km/hr
(c) 78 km/hr (d) 81 km/hr 164.
A plane left 30 min later than its scheduled time to reach
its destination 1500 km away. In order to reach in time
[Based on MAT (Sept, May), 2010]
it increases its speed by 250 km/hr. What is its original
158.
A train travelling at 48 km/hr completely crosses another speed?
train having half its length and travelling in opposite (a) 1000 km/hr (b) 750 km/hr
direction at 42 km/hr in 12 s. It also passes a railway
(c) 600 km/hr (d) 800 km/hr
platform in 45 s. The length of the platform is
[Based on MAT (Feb), 2010, 2006]
(a) 400 m (b) 450 m
165.
I have to reach a certain place at a certain time and find
(c) 560 m (d) 600 m
that I shall be 15 min too late, if I walk at 4 km an hour and
[Based on MAT (Sept), 2010 (May), 2008]
10 min too soon, if I walk at 6 km an hour. How far have
159.
A car takes 15 mins less to cover a distance of 75 km. If it I to walk?
increases its speed by 10 km/hr from its usual speed, how (a) 25 km (b) 5 km
much time would it take to cover a distance of 300 km (c) 10 km (d) None of these
using this speed?
[Based on MAT (Feb), 2009]
1
(a) 5 hrs (b) 5 hrs
h 166.
A train travels a distance of 300 km at a constant speed.
2 If the speed of the train is increased by 5 km an hour, the
1 journey would have taken 2 hrs less. The original speed of
(c) 6 hrs (d) 6 hhrs the train was
2
[Based on MAT (Sept), 2010] (a) 25 km/hr (c) 20 km/hr
(c) 28 km/hr (d) 30 km/hr
160.
Mohan and Puran are running towards each other, each
[Based on MAT (Feb), 2009]
one from his own house. Mohan can reach Puran’s house
in 25 min of running, which is half the time it takes Puran 167.
A man takes 6 hrs 30 mins in going by a cycle and coming
to reach Mohan’s house. If the two started to run at the back by scooter. He would have lost 2 hrs 10 mins by
same time, how much more time will it take Puran to going on cycle both ways. How long would it take him to
reach the middle than Mohan? go by scooter both ways?

Chapter_12.indd 293 1/30/2016 4:18:01 PM


294  Chapter 12

1 (a) 20 s (c) 100 s


(a) 5 hrs (c) 5 hrs (c) 300 s (d) 200 s
2
[Based on MAT (Sept), 2009]
1 1
(c) 4 hrs (d) 3 hrs 174.
A car travels a distance of 45 km at the speed of 15 km/hr.
3 2
[Based on MAT (Dec), 2009] It covers the next 50 km of its journey at the speed of 25
1
168.
The distance between two stations A and B is 450 km. km/hr and the last 25 km of its journey at the speed of 8
A train starts from A and moves towards B at an average 3
speed of 15 km/hr. Another train starts from B, 20 min km/hr. What is the average speed of the car?
earlier than the train at A and moves towards A at an (a) 40 km/hr (c) 24 km/hr
average speed of 20 km/hr. How far from A will the two (c) 15 km/hr (d) 18 km/hr
trains meet? [Based on MAT (May), 2009]
(a) 190 km (c) 320 km 175.
A car travels a distance of 170 km in 2 hrs partly at a
(c) 180 km (d) 260 km speed of 100 km/hr and partly at 50 km/hr. The distance
[Based on MAT (Dec), 2009] travelled at speed of 50 km/hr is
169.
The ratio between the speeds of Ramesh and Suresh is (a) 50 km (c) 40 km
3:4. If Ramesh takes 30 min more than Suresh to cover (c) 30 km (d) 60 km
a distance, then the actual times taken by Ramesh and [Based on MAT (May), 2009]
Suresh respectively are
176.
A train is scheduled to cover the distance between two
1 1 1 stations 46 km apart in one hour. If it travels 25 km at
(a) 2 hrs and 1 hhrs (c) 2 hhrs and 1 hrs
h
2 2 2 a speed of 40 km/hr, find the speed for the remaining
journey to complete it in the scheduled time.
1
(c) 2 hrs and 1 hr (d) 2 hhrs and 1 hrs (a) 66 km/hr (c) 56 km/hr
2
(c) 46 km/hr (d) 36 km/hr
[Based on MAT (Dec), 2009]
[Based on MAT (May), 2009]
170.
Ramu sees a train passing over a 1 km long bridge. The
177.
A train started from station A and proceeded towards
length of the train is half that of the bridge. If the train
station B at a speed of 48 km/hr. 45 min later, another train
clears the bridge in 2 min, the speed of the train is
started from station B and proceeded towards station A at
(a) 50 km/hr (c) 43 km/hr 50 km/hr. If the distance between the two stations is 232
(c) 45 km/hr (d) None of these km, at what distance from station A will the trains meet?
[Based on MAT (Dec), 2009] (a) 108 km (c) 144 km
171.
A motor cyclist goes from Mumbai to Pune, a distance of (c) 132 km (d) None of these
192 km, at an average speed of 32 km/hr. Another man [Based on MAT (May), 2009]
1 178.
A railway passenger counts the telegraph poles on the
starts from Mumbai by car, 2 hhrs after the first and
2 rail road as he passes them. The telegraph poles are at a
reaches Pune half an hour earlier. What is the ratio of the distance of 50 m. What will be his count in 4 hrs if the
speeds of the motor cycle and the car? speed of the train is 45 km/hr?
(a) 10:27 (c) 1:3 (a) 2500 (c) 600
(c) 1:2 (d) 5:4 (c) 3600 (d) 5000
[Based on MAT (Dec), 2009] [Based on MAT (Feb), 2009]
172.
A man sitting in a train travelling at the rate of 50 km/hr 179.
A car driver, driving in a fog, passes a pedestrain who was
observes that it takes 9 s for a goods train travelling in the walking at the rate of 2 km/hr in the same direction. The
opposite direction to pass him. If the goods train is 187.5 pedestrain could see the car for 6 min and it was visible to
m long, find its speed. him up to a distance of 0.6 km. What was the speed of the
(a) 25 km/hr (c) 40 km/hr car?
(c) 35 km/hr (d) 36 km/hr (a) 15 km/hr (c) 30 km/hr
[Based on MAT (Sept), 2009] (c) 20 km/hr (d) 8 km/hr
173.
Two cyclists start on a circular track from a given point [Based on MAT (Feb), 2009]
but in opposite directions with speeds of 7 m/s and 8 m/s 180.
A cyclist moving on a circular track of radius 100
respectively. If the circumference of the circle is 300 m, m completes one revolution in 2 min. What is the
after what time will they meet at the starting point? approximate speed of the cyclist?

Chapter_12.indd 294 1/30/2016 4:18:01 PM


 Time and Distance  295

(a) 200 m/min (c) 314 m/min 188.


Two buses, one moving towards North and the other
(c) 300 m/min (d) 900 m/min towards East, leave the same place at the same time. The
[Based on MAT (Feb), 2009] speed of one of them is greater than that of the other by
5 km/hr. At the end of 2 hrs, they are at a distance of 50
181.
A train 110 m in length travels at 60 km/hr. How much km from each other. The speed of the bus going slower
time does the train take in passing a man walking at 6 km/ is
hr against the train?
(a) 15 km/hr (b) 12 km/hr
(a) 6 s (b) 12 s
(c) 10 km/hr (d) 20 km/hr
(c) 16 s (d) 18 s
[Based on MAT (Sept), 2008]
[Based on MAT, 1999]
189.
There is a ring road connecting points A, B, C and D.
182.
Alok walks to a viewpoint and returns to the starting point The road is in a complete circular form but having
by his car and thus takes a total time of 6 hrs 45 min. He several approach roads leading to the centre. Exactly in
would have gained 2 hrs by driving both ways. How long the center of the ring road there is a tree which is 20
would it have taken for him to walk both ways? km from point A on the circular road. You have taken a
(a) 7 hrs 45 mins (c) 8 hrs 45 mins round of circular road starting from point A and finish
(c) 5 hrs 30 mins (d) None of these at the same point after touching points B, C and D. You,
[Based on MAT (Feb), 2009 (May), 2008] then drive 20 km interior towards the tree from point A
and from there, reach somewhere in between B and C on
183.
A train 100 m long passes a bridge at the rate of the ring road. How much distance do you have to travel
72 km/hr in 25 s. The length of the bridge is from the tree to reach the point between B and C on the
(a) 150 m (b) 400 m ring road?
(c) 300 m (d) 200 m (a) 80 km (b) 15 m
[Based on MAT, 1999]
(c) 20 km (d) 40 m
184.
A train passes a station platform in 36 s and a man standing [Based on MAT (Sept), 2008]
on the platform in 20 s. If the speed of the train is 54 km/
hr, find the length of the platform. 190.
A passenger train takes two hrs less for a jour­ney of 300
km if its speed is increased by 5 km/hr from its normal
(a) 225 m (c) 235 m speed. The normal speed is
(c) 230 m (d) 240 m (a) 35 km/hr (b) 50 km/hr
[Based on MAT (Dec), 2008]
(c) 25 km/hr (d) 30 km/hr
185.
Two trains, 130 m and 110 m long, are going in the [Based on MAT, 1999]
same direction. The faster train takes one minute to pass
the other completely. If they are moving in opposite 191.
A train 75 m long overtook a person who was walking at
directions, they pass each other completely in 3 s. Find 1
the rate of 6 km/hr and passes him in 7 s. Subsequently,
the speed of the faster train. 2
(a) 38 m/s (c) 46 m/s 3
it overtook a second person and passes 3 him in 6 s. At
(c) 42 m/s (d) None of these 4
[Based on MAT (Dec, Sept, Feb), 2008] what rate was the second person travelling?
186.
A man walks half of the journey at 4 km/hr by cycle, does (a) 4 km/hr (c) 1 km/hr
one-third of journey at 12 km/hr and rides the remainder (c) 2 km/hr (d) 5 km/hr
journey in a horse cart at 9 km/hr, thus completing the whole [Based on MAT (May), 2008]
journey in 6 hrs and 12 min. The length of the journey is
192.
Train A leaves Ludhiana for Delhi at 11 am, running at
(a) 40 km (b) 153 km the speed of 60 km/hr. Train B leaves Ludhiana for Delhi
(c) 36 km (d) 28 km by the same route at 2 pm on the same day, running at the
[Based on MAT (Sept), 2008] speed of 72 km/hr. At what time will the two trains meet
each other?
187.
A train consists of 12 bogies, each bogie is 15 m long.
The train crosses a telegraph post in 18 s. Due to some (a) 5 a.m. on the next day
problem, two bogies were detached. The train now crosses (c) 2 a.m. on the next day
a telegraph post in (c) 5 p.m. on the next day
(a) 15 s (b) 12 s
(d) None of the above
(c) 18 s (d) 20 s [Based on MAT (May), 2008]
[Based on MAT (Sept), 2008]

Chapter_12.indd 295 1/30/2016 4:18:01 PM


296  Chapter 12

193.
A person travels 285 km in 6 hrs in two stages. In the first that they meet in the middle of the tower. The velocity of
part of the journey, he travels by bus at the speed of 40 projection of the second body is
km/ hr. In the second part of the journey, he travels by (a) 20 m/s (c) 25 m/s
train at the speed of 55 km/hr. How much distance did he (c) 24.5 m/s (d) None of these
travel by train?
[Based on MAT (Dec), 2006]
(a) 205 km (c) 165 km
200.
Two trains are 2 km apart and their lengths are 200 m and
(c) 145 km (d) 185 km
300 m. They are approaching towards each other with a
[Based on MAT (Dec, May), 2007] speed of 20 m/s and 30 m/s, respectively. After how much
194.
Two men A and B walk from P to Q at a distance of 21 km time will they cross each other?
at the rates of 3 and 4 km an hour, respectively. B reaches (a) 50 s (c) 100 s
Q and returns immediately and meets A at R. The distance (c) 25/3 s (d) 150 s
from P to R is
[Based on MAT (Dec), 2006]
(a) 14 km (c) 16 km
201.
A jet plane is rising vertically with a velocity of 10 m/s. It
(c) 20 km (d) 18 km
has reached a certain height when the pilot drops a coin,
[Based on MAT (Dec), 2007] which takes 4 s to hit the ground. Assuming that there
195.
Two trains are moving in opposite directions at speeds is no resistance to the motion of the coin, the height of
of 60 km/hr and 90 km/hr their lengths are 1.10 km and the plane and the velocity of the coin on impact with the
0.9 km respectively, the time taken by the slower train to ground are
cross the faster train in seconds is (a) 38.4 m, 29.2 m/s (c) 38.4 m, 28.7 m/s
(a) 36 (c) 49 (c) 26.5 m, 13.5 m/s (d) None of the above
(c) 45 (d) 48 [Based on MAT (Dec), 2006]
[Based on MAT (Sept), 2007]
202.
A train starts from Delhi at 6:00 am and reaches Ambala
196.
Excluding stoppages, the speed of a bus is 54 km/hr and Cantt at 10 a.m. The other train starts from Ambala Cantt
including stoppages, it is 45 km/hr. For how many mins at 8 am and reaches Delhi at 11:30 a.m. If the distance
does the bus stop per hour? between Delhi and Ambala Cantt is 200 km, then at what
(a) 12 (c) 10 time did the two trains meet each other?
(c) 9 (d) 20 (a) 8:56 a.m. (c) 8:46 a.m.
[Based on MAT (May), 2007] (c) 7:56 a.m. (d) 8:30 a.m.
[Based on MAT (Dec), 2006]
197.
Rampur is 100 km from Sitapur. At 3 pm, Bharat Express
leaves Rampur for Sitapur and travels at a constant speed 203.
A train with 90 km/hr crosses a bridge in 36 s. Another
of 30 km/hr. One hour later, Laxman Mail leaves Sitapur train 100 m shorter crosses the same bridge at 45 km/hr.
for Rampur and travels at a onstant speed of 40 km/hr. What is the time taken by the second train to cross the
Each train makes one stop only at a station 10 km from its bridge?
starting point and remains there for 15 min. Which train is (a) 61 s (c) 63 s
nearer to Rampur when they meet?
(c) 62 s (d) 64 s
(a) Both are equidistant (c) Laxman Mail [Based on MAT (May), 2006]
(c) Bharat Express (d) None of these
204.
Ramesh travels 760 km to his home, partly by train and
[Based on MAT (May), 2007] partly by car. He takes 8 hrs, if he travels 160 km by train
198.
A car starts running with the initial speed of 40 km/hr, and the rest by car. He takes 12 min more, if he travels 240
with its speed increasing every hour by 5 km/hr. How km by train and the rest by car. What are the speeds of the
many hrs will it take to cover a distance of 385 km? train and the car?
(a) Speed of car = 90 km/hr, Speed of train = 60 km/hr
1
(a) 9 hrs (c) 9 hrs (b) Speed of car = 100 km/hr, Speed of train = 80 km/hr
2
(c) Speed of car = 80 km/hr, Speed of train = 90 km/hr
1
(c) 8 hrs (d) 7 hrs (d) Speed of car = 100 km/hr, spped of train = 90 km/hr
2
[Based on MAT (May), 2007] [Based on MAT (May), 2006]

199.
A tower is 61.25 m high. A rigid body is dropped from 205.
A car travels 25 km/hr faster than a bus for a journey of
its top and at the same instant another body is thrown 500 km. If the bus takes 10 hrs more than the car, then the
upwards from the bottom of the tower with such a velocity speeds of the bus and the car are

Chapter_12.indd 296 1/30/2016 4:18:01 PM


 Time and Distance  297

(a) 25 km/hr and 40 km/hr respectively 212.


A plane left 30 mins later than the scheduled time and in
(c) 25 km/hr and 60 km/hr respectively order to reach its destination 1500 km away in time, it has
(c) 25 km/hr and 50 km/hr respectively to increase its speed by 250 km/hr from its usual speed.
Find its usual speed.
(d) None of the above
[Based on MAT (May), 2006]
(a) 1000 km/hr (b) 750 km/hr
(c) 850 km/hr (d) 650 km/hr
206.
A train covered a certain distance at a uniform speed. If
the train had been 6 km/hr faster, it would have taken 4 hrs [Based on MAT, 1999]
less than the scheduled time. And, if the train were slower 213.
Two trains, each 120 m in length, run in opposite directions
by 6 km/hr, the train would have taken 6 hrs more than the with velocities of 40 m/s and 20 m/s respectively.
scheduled time. The length of the journey is How long will it take for the tail end of the two trains to
(a) 700 km (c) 740 km meet each other from the time their engines crossed each
(c) 720 km (d) 760 km other?
[Based on MAT (Feb), 2006] (a) 2 s (b) 3 s
207.
A small aeroplane can travel at 320 km/hr in still air. The (c) 4 s (d) 5 s
wind is blowing at a constant speed of 40 km/hrs. The total [Based on MAT, 1997]
time for a journey against the wind is 135 mins. What will
be the time, in mins, for the return journey with the wind? 214.
A journey of 260 km between two cities takes 3 hrs less by
(Ignore take off and landing times for the aeroplane). train than by car. If the average speed of travel by car is 6
km/hr less than the speed of the train, the average speed of
(a) 94.5 (b) 105
the train is
(c) 108.125 (d) 120
[Based on MAT, 1997]
(a) 26 km/hr (b) 52 km/hr
(c) 20 km/hr (d) 13 km/hr
208. A journey of 192 km between the two cities takes 2 hrs
less by a fast train than by a slow train. If the average [Based on MAT, 1998]
speed of the slow train is 16 km/hr less than that of the fast 215.
A passenger train takes 2 hrs less for a jour­ney of 300
train, then the average speed of the fast train is km if its speed is increased by 5 km/hrs from its normal
(a) 36 km/hr (b) 64 km/hr speed. The normal speed is
(c) 32 km/hr (d) 48 km/hr (a) 10 km/hr (b) 20 km/hr
[Based on MAT, 1999]
(c) 25 km/hr (d) 30 km/hr
209.
A certain distance is covered by a train with a certain [Based on MAT, 1998]
speed. If half the distance is covered in double time, then
the ratio of this speed to that of the original is 216.
Suresh travels 600 km to his home partly by train and
partly by air. It takes him 8 hrs if he travels 120 km by
(a) 1:4 (b) 4:1
train and the rest by car. It takes him 20 mins more if he
(c) 2:1 (d) 1:2 travels 200 km by train and the rest by car. The speed of
[Based on MAT, 1997] the car is
210.
Points A and B are 70 km apart on a highway. One car (a) 20 km/hr (b) 30 km/hr
starts from A and another one from B at the same time. If
(c) 60 km/hr (d) 80 km/hr
they travel in the same direction, they meet in 7 hrs. But if
they travel towards each other they meet in one hour. The [Based on MAT, 1998]
speeds of the two cars are 217.
The time taken to run around a circular field of area 616 sq
(a) 45 and 25 km/hr (b) 70 and 10 km/hr km at a speed of 22 km/hr is
(c) 40 and 30 km/hr (d) 60 and 40 km/hr (a) 1 hr (b) 2 hrs
[Based on MAT, 1999] (c) 3 hrs (d) 4 hrs
211.
It takes eight hrs for a 600 km journey, if 120 km is done [Based on MAT, 1998]
by train and the rest by car. It takes 20 mins more, if 200
km is done by train and the rest by car. The ratio of the 218.
A man takes 50 mins to cover a certain dis­tance at a speed
speed of the train to the speed of the car is of 6 km/hr. If he walks with a speed of 10 km/hr, he covers
the same distance in
(a) 4:3 (b) 3:4
(c) 3:2 (d) 2:3 (a) 30 mins (b) 40 mins
[Based on MAT, 1999] (c) 20 mins (d) 10 mins
[Based on MAT, 1998]

Chapter_12.indd 297 1/30/2016 4:18:01 PM


298  Chapter 12

219.
A train takes 50 mins for a journey if it runs at 48 km/hr. 223.
Two cyclists start on a circular track from a given point
The rate at which the train must run to reduce the time to but in opposite directions with speeds of 7 m/s and 8 m/s
40 mins will be respectively. If the circumference of the circle is 300 m,
(a) 50 km/hr (b) 55 km/hr after what time will they meet at the starting point?
(c) 60 km/hr (d) 65 km/hr (a) 20 s (b) 100 s
[Based on MAT, 1998] (c) 300 s (d) 200 s
[Based on MAT, 1998]
220.
A goods train in five successive mins from its start runs
68 m, 127 m, 208 m, 312 m and 535 m and for the next 5 224.
Two cars start together in the same direction from the
mins maintains an average speed of 33 km/hr. The average same place. The first goes with a uniform speed of 10 km/
speed of the train for these 10 mins is hr. The second goes at a speed of 8 km/hr in the first hour
(a) 23 km/hr (b) 24 km/hr and increases the speed by half km, each succeeding hour.
(c) 25 km/hr (d) 26 km/hr After how many hrs will the second car overtake the first
if both go non-stop?
[Based on MAT, 1998]
(a) 9 hrs (b) 5 hrs
221.
A certain distance is covered by a cyclist at a certain
(c) 7 hrs (d) 8 hrs
speed. If a jogger covers half the distance in double the
[Based on MAT, 1999]
time, the ratio of the speeds of the jogger to that of the
cyclist is 225. A train is moving at a speed of 132 km/hr. If the length of
(a) 1:4 (b) 4:1 the train is 110 m, how long will it take to cross a railway
(c) 1:2 (d) 2:1 platform 165 m long?
[Based on MAT, 1998] (a) 5 s (b) 7.5 s
(c) 10 s (d) 15 s
222.
In climbing a 21 m long round pole, a monkey climbs 6 m
[Based on MAT, 1998]
in the first minute and slips 3 m in the next minute. What
time (in mins) the monkey would take to reach the top of 226.
A car can finish a certain journey in 10 hrs at the speed of
the pole? 48 km/hr. In order to cover the distance in 8 hrs, the speed
(a) 11 (b) 14 of the car must be increased by
(a) 6 km/hr (b) 7.5 km/hr
2
(c) 14 (d) 9 (c) 12 km/hr (d) 15 km/hr
3
[Based on MAT, 1999] [Based on MAT, 1998]

EXPLANATORY ANSWERS

1.
(a) Let the speed of train be V and speed of Ajay be v. 30 30
Then, by relative speed = – 1 ...(2)
2x y
10(V + V) = 12V ...(1)
From Eqs. (1) and (2)
Similarly,
30 30
T(V – v) = 12V ...(2) ⇒ – 2 = +1
x 2x
From Eqs. (1) and (2), ⇒ x = 5, y = 7.5.
V  v V  v 12 12 3.
(b) Let d kilometres be the distance between A and B.
 = 
V V 10 T d
When A and B walk towards each other =
v  (v  2)
10
⇒ T = 12 × = 15 min
8 72 d
and if they walk in the same direction
2.
(a) Let Amit’s speed = x km/hr 60 (v  2)  v
Let Suresh’s speed = y km/hr d
= 6 or d = 12 km
= 6, i.e.
2
30 30 d 72

\ = + 2 ...(1) So, = and v = 4 km/hr
x y 2v  2 60
If Amit’s speed becomes 2x km/hr, then Ratio of speeds = 4:6

Chapter_12.indd 298 1/30/2016 4:18:02 PM


 Time and Distance  299

4. (b) Suppose car 1 starts at 12 noon. of pool. As both have constant speeds, hence at the
\ Car II starts at 6 p.m. second meeting each swimmer had covered 3 times
as much distance as at the first meeting. Since the
Time Distance Covered Distance Covered swimmer starting at the deep end had covered 18.5
by Car I by Car II m when they first met, he covered 18.5 × 3 = 55.5 m
7 p.m. 210 km 50 km when they next met. It is clear than this distance is
12.00 night 360 km 300 km 10.5 m more than the length of the pool which is
3 a.m 450 km 450 mn hence, 55.5 – 10.5 = 45 m.

\ Car II will crosss Car I after 9 hrs. 10.


(a) For express train, the time needed to travel 75 km
60
5. (c) Let the speed of the car be x km/hr. = × 75
100
\ Speed of the train = x + 20% of x
= 45 (plus 3 min. stoppage) = 48 mins.
6x
= km/hr The time needed to travel 300 km = 192 mins
5
The time needed to travel 600 km
5 25 75 = 384 min – 3 min = 381 min
\ × 75 + = ⇒ x = 60.
6x 2 × 60 x For local train, time needed to travel 25 km
6. (c) Let the speed of the car be x km/hr = 30 (plus 1 min. stoppage) = 31 mins
\ Speed of the train = x + 20% of x The time needed to travel 300 km = 372 mins
6x 50
= km/hr In 9 min, the local train will cover × 9 = 7.5 km
5 60
\ In 381 min, the local train will cover 307.5 km.
5 25 75
\ × 75 + = ⇒ x = 60. 11. (a) Let the length of each train be x metres.
6x 2 × 60 x
Relative motion of the faster train w.r.t the slower
7.
(a) A covers 3.5 km before meeting B in (18 × 3.5) + 3 = train is the difference of their speeds i.e. 10 km/hr.
66 mins
\ The two trains will be clear of each other in 36 s
66 which is the time taken in moving the distance of
B covers a distance of 5.5 km in 66 mins, i.e. hrs, 2x m @ 10 km/hr.
60
11 i.e., 100 m/36 s
i.e. hrs.
10 ⇒ 2x = 100
11 10 ⇒ x = 50
\ B’s speed = × = 5 km/hr.
2 11 \ Length of each train = 50 m.
8. (c) Wheel of diameter 7 cm starts from X. 12. (d) Suppose distance between each pole is 1 m.
Its circumference = 2 × p × 3.5 = 22 m \ Total distance = 19 m
Wheel of diameter 14 cm starts from Y. It takes 24 s to cover 11 m.
Its circumference = 44 m 24
\ To cover 19 m, it will take × 19 = 41.45.
Distance from X to Y = 1980 cm 11
Let both the wheels make x revolutions per second. 13.
(c) Suppose they meet after x mins.
\ Distance covered by both the wheels in one second 4500 meters are covered by Suresh in 60 mins.
= 22x + 44x = 66x cm 4500
\ In x min, Suresh covers x metres .
Distance covered by both the wheels in 10 s 60
= 660x cm 3750 meters are covered by Suresh’s wife in 60 mins.
\ 660x = 1980 ⇒ x = 3 3750
\ In x min, Suresh’s wife covers x metres
\ Speed of the smaller wheel = 22x/s 60
= 66 cm/s. 4500 3750
⇒ x+ x = 726
9.
(b) When they pass for the first time, the combined 60 60
distance = Length of the pool. When they meet for
⇒ x = 5.28.
second time, the combined distance = 3 × length

Chapter_12.indd 299 1/30/2016 4:18:02 PM


300  Chapter 12

14.
(b) 19.
(a) Since the trains are 100 km apart, and the trains are
travelling towards each other at 40 and 60 km/hr, the
trains will collide in one hour. The bird must be flying
at 90 km/hr during this time, so the bird travels 90 km.

Suppose the speed of the boy is u m/minute and the Total distance
20.
(b) Average speed =
speed of the tram is v m per minute. Total time
300 200 100 4 TV
= + +1 Total distance = TV = TV +
u v u 3 3
200 200 = 75% of total distance
or = + 1 ...(1)
u v  + 25% of total distance.
300 200 TV
= ,  i.e., 3v = 4u ...(2) Total time = T +
2u v 3S
Substitute in Eq. (1), Hence, average speed
200 200  3 4
= +1 TV
u 4u 4TVS 4VS
= 3 = =
200 600 TV 3 ST  TV 3S V
⇒ = +1 T
u 4u 3S
200 50 21.
(c) Since the trains are travelling in opposite direction
⇒ = 1 ⇒ = 1, velocity for the boy in the faster train = 50 m/s.
4u u
i.e., u = 50 m/min = 3000 m/h = 3 km/hr Distance travelled = length of the train = 100 m
Time taken by the ball from one train to the other
15. (c) Let the speed of the escalator be x steps per second.
100
\ In 18 s the escalator will come down by 18x steps = =2s
and in 6 s it will come down by 6x steps. Since 50
height of the stair way is same, Ball is thrown at 2 m/s, \ distance between the two
trains = 2 × 2 = 4 m.
30 + 18x = 34 + 6x \ x = 1/3
22. (b) If the trains are travelling in the same direction, speed
\ height of the stair way = 30 + 18(1/3) = 36 steps
of faster train for the boy = 30 – 20 = 10 m/s.
16. (c) The two men walk in opposite directions @ 62.5 m/ \ In 2 s the front of the faster train travels 2 × 10
min and 75 m/min respectively. = 20 m
\ They will meet for the first time after \ The balls hits at 20 m from the front of the train B.
726 726 23. (b) Let speed of the train be x km/hr
= = 5.28 min.
75 + 62.5 137.5 Let speed of the car be y km/hr
17.
(b) Let the speed of plane be X km/hr. Then 120 480
\ + = 8
7 7 22 x y
 =
X  .125 X X  0.05 X 60 200 400 1 25
+ = 8 =
7 7 22 x y 3 3
⇒  =
.875 X 1.05 X 60 600 2400
⇒ + = 40
On solving it, we get X = 39.98 km/hr » 40 km/hr x y
7 7 14 600 1200
\  hour or 21 mins. + = 25
40 40 40 x y
18.
(b) For opposite direction 1200
⇒ = 15 ⇒ y = 80, x = 60.
Relative speed = 80 km/hr y
Distance = 100 m \ Required ratio = 60:80 = 3:4.
100  3600 9 24.
(c) Let x km/hr and y km/hr be the speeds of the two cars
= s
\ Required time = respectively.
80  1000 2
= 4.5 s 7x – 70 = 7y (If go in same direction)

Chapter_12.indd 300 1/30/2016 4:18:03 PM


 Time and Distance  301

y = 70 ­– x (If go in opposite direction) 32. (c) Let the speed on the smaller wheel be x cm/s.
⇒ x + y = 70 and x – y = 10 \ Distance covered by the smaller wheel in 10 s =
⇒ x = 40, y = 30. 10x cm.
25.
(c) Let the correct time to complete the journey be x mins. Distance covered by the smaller wheel in one
revolution
Distance covered in (x + 11) mins at 40 km/hr.
22
= distance covered in (x + 5) mins at 50 km/hr. = 2 × × 3.5 = 22 cm
7
x + 11 x+5
\ × 40 = × 50 \ No. of revolutions made by the smaller wheel in
60 60
10 x 5x
⇒ x = 19 mins. 10 s = = ...(1)
22 11
26.
(c) Total distance covered = 150 + 300 = 450 m
Distance covered by the bigger wheel in 10 s = (1980
81 – 10x) cm.
Time = s.
2 \ No. of revolutions made by the bigger wheel in 10
2 18 1980 − 10 x
Speed = 450 × × = 40 km/hr. s= ...(2)
81 5 44
27.
(b) Let the length of the bridge be x meters. 5x 1980 − 10 x
\ = ⇒ x = 66.
100 + x 25 11 44

\ = ⇒ x = 400.
72000 3600 33. (b) There are 35 floors between them.
28.
(a) The relative speed of the train w.r.t. the speed of the Rate of descent of B = 36 floors per minute.
man Rate of ascent of A = 34 floors per minute.
= speed of the train + speed of the man Thus, relative velocity = 34 + 36 = 70 floors per
= 60 + 6 = 66 km/hr minute.
\ Time taken by the train in crossing the man = Time Hence, B would have travelled
taken in moving a distance equal to the length of
1
the train (110 m) @ 66 km/hr. 36  = 18 floors.
2
3600
= × 100 = 6 s. 34.
(a) Let the length of each train be x meters.
66000
Then the time in which the two trains are clear of each
8000 other is the time taken by the faster train in moving a
29.
(c) Train requires = 32 s to reach the target distance 2x m (sum of the lengths of the two trains)
250
w.r.t. the relative speed of 46 – 36 = 10 km/hr.
24
Up time = = 2.4 s 36 60 2 x
10 \ = × ⇒ x = 50.
60 10 1000
24 35.
(b) Distance Time
Down time = =4s
6
80 km 5 hrs
\ For 1 round i.e. back and forth he will require 6.4 d km 4 hrs
s.
(80  d )
32 Total average speed = = 14
\ In 32 s he will complete = 5 rounds. (5  4)
6.4
⇒ d = 46 km
30.
(b) Because they cover 60000 miles in 60 mins, i.e. 1000
miles in one minute, therefore they should be 1000 ⇒ Average speed for next 4 hrs = 11.5 km/hr
miles apart, 1 minute before impact. 36.
(a) Let speed of the train be x km/hr and speed of the car
31.
(a) Let after t hrs train Y overtakes train X, then by the be y km/hr.
condition given in the problem 160 600
\ + = 8
 5 5 x y
Qt = P  t   ⇒ t (Q – P) = P
 2 2 240 520 1 41
+ = 8 =
5P x y 5 5
⇒ t = hrs
2(Q – P ) ⇒ y = 100, x = 80.

Chapter_12.indd 301 1/30/2016 4:18:03 PM


302  Chapter 12

37.
(a) Let original duration of the flight be x hrs. ⇒ The time taken by Mir to cover the last five miles is
3000  4
5
\ Original speed =  km/hr times the time taken by him to cover the first
x  
5
3000 3000 5
\ = – 100 ⇒ x = 5.  5 3125
x +1 x five miles =    5 =  5 ≈ 3.05 × 5 ≈ 15.25
 4 1024
38.
(b) Circumference of the wheel mins ≈ 15 mins and 15 s
22 44.
(b) The motocyclist takes 6 hrs to complete the journey
= 2 × × 35 = 220 cm
7 form Delhi to Bharatpur and the person driving by car
\ Number of revolutions made by the wheel during will take 4 hrs. Ratio of the speed = 2:3
the journey
1 2h − 1
2200000 × 30 45.
(c) m miles in h − = hrs
= = 300000 = 3 lakh. 2 2
220
2m
39.
(d) As her father reached home 10 mins earlier than the ⇒ miles per hour.
usual time, therefore he saw his daughter five mintues 2h − 1
before the scheduled time and he turned back (saving 46.
(b) Relative speed of Vaibhav when he is moving up =
5 mins each side). So he met her at 6.55 p.m., i.e. she Vaibhav’s speed + speed of escalator = V + E
walked for 25 mins from the station towards her home Relative speed of Vaibhav when he is moving down =
before her father picked her up. Vaibhav’s speed – speed of escalator = V – E
40.
(c) Let the number of hrs be x. According to the question.
Total distance travelled by A = 40 + 40 × x  1 1 x x
(V + E) =  6  2   ,(V  E ) =
Total distance travelled by B = 50 × x  2 2  30 90
40 where x is the total stationary length of the escalator.
40 + 40 × x = 50 × x  ⇒  x =
10
⇒ x = 4 hrs  x x
Hence, Vaibhav’s speed, V = 0.5   
 30 90 
41. (d) Let the distance between P and Q be x km.
x x x x 12  4x  x
\ + = + + ⇒ x = 30. ⇒        0.5   
10 15 12.5 12.5 60  90  45
Hence he will take 45 s if the escalator is stationary.
42.
(c) Respective speed of the trains = 15 km/hr
Let the total length of both the trains be x. 47.
(b) If buses I and II move in the same direction, then they
meet after covering 120 km and 60 km respectively @
(35  x)  60
5 = 20 km/hr and 10 km/hr respectively. If the buses move
15000 in opposite directions, then they meet after covering
⇒ x = 1,215 m 40 km and 20 km respectively.
43. (b) Let the speed of Mir for the first mile be v miles/min 48. (a) Suppose the distance covered = 4x km
⇒ The speed of Mir for the 2nd, 3rd, 4th, . . . miles Suppose the usual speed = 4x km/hr
4 16 64 \ Time taken = 1 hr
is v, v, v . . . respectively.
5 25 125 Reduced speed = 3x km/hr
Since the speed of Mir for the successive miles is in With this speed, the distance of 4x km will be covered
4 in 1 hour and 20 mins.
geometric progression (with r = ), the speeds of Mir
5 49.
(b) Suppose distance between X and Y be K km X ® Y :
5
 4 3
for the 6th, 7th, 8th, 9th and 10th miles will be   K
 5 3
times the speeds for the 1st, 2nd, 3rd, 4th and 5th It takes 4 hrs to cover K km.
B 4
miles respectively.
Hence the average speed of Mir of the last five miles 1
K
1
5 It takes 4 hrs to cover K km.
 4 S 4
is   times the average speed of the first five miles.
 5

Chapter_12.indd 302 1/30/2016 4:18:04 PM


 Time and Distance  303

3 1 \ Total distance covered by them when they cross


K+ K each other for the fifth time is = AB + 4(2AB) =
4 4 K ( BS )
\ Average speed = = 9AB = 7(9) = 63 km.
3 1 3 1
K K KS + KB
4 +4 4 4 63 1
Time taken = =2 hrs
B S 30 10
4 BS \ Distance from the county station
= km/hr.
3S + B  21
= 18   = 37.8 km
50. (c) Relative speed = 18 km/hr = 5 m/s  10 
Distance travelled in 60 s = 300 m = 4 (7) + 9.8
Sum of lengths of first and second train = 300 m = 7 + 2.8 = 2.8 km from the town hall.
Length of the second train = 300 – 125 = 175 m i.e.,  4.2 km from the county station.
3
51.
(d) Upto 5.30 p.m., the plane A has covered X miles. 54.
(b) Let the distance be x. So, the train stops for
2
x x x
Suppose that the plane B overtakes plane A, T hrs after   hrs.
5.30 p.m. Then 36 45 180

3 3X x x
TY =
X + TX ⇒ T = . i.e., in hrs stoppage is .
2 2(Y − X ) 36 180
x  36 1
52.
(a) Since the train passes the man in 5 s, the relative speed So, in 1 hour, stoppage will be  hour
180  x 5
100
of the train with respect to the man =  i.e., 20 m/s = 12 mins.
5
or 72 km/hr. So the speed of the train = 78 km/hr. The 55.
(b) Let the normal speed be a km/hr
train passes the car in 6 s, the relative speed of the
train with respect to the car 300 300
\ = +2
a a+5
 100  18
=   = 60 km/hr. ⇒ 300 (a + 5) = 300a + 2a (a + 5)
 9  5

⇒ 2a2 + 10a – 1500 = 0
\ Speed of the car = 78 – 60 = 18 km/hr
⇒ a2 + 5a – 750 = 0
53.
(a) Relative speed of Manu with respect to Tanu ⇒ (a + 30) (a – 25) = 0
= 12 + 18 km/hr = 30 km/hr
⇒ a = 25.
Distance = Speed × Time
14 56.
(b) Let the normal speed of the aircraft be x km/hr.
= 30 × = 7 km
60 1500 1 1500
\ − =
\ The distance covered by both of them, when they x 2 250 + x
cross each other for the first time is 7 km ⇒ x2 + 250x – 750000 = 0
⇒ x = 750.

2
57.
(b) of the distance = 1200 km
5
\ Whole distance = 3000 km
Let the distance covered by Manu and Tanu when they
cross each other for the first time be AB. Distance travelled by car = 1000 km
   For such a situation, when the ratio of speeds = 3:2, \ Distance travelled by train = 800 km.
it can be observed the total distance covered by both
58. (c) Let the distance between Mumbai and Ahmedabad be
the runners put together, between two consecutive
x km.
meetings is always twice the distance AB.
x
   When they cross each other for the 2nd time, they T1 takes 5 hrs, \ its speed = km/hr
must have covered a distance of AB + 2AB 5
   When they cross each other for the 3rd time, they x
T2 takes 6 hrs, \ its speed = km/hr
must have crossed AB + 4AB. 6

Chapter_12.indd 303 1/30/2016 4:18:04 PM


304  Chapter 12

They will meet after 64.


(b) Because each word is lit for a second
x 30 5 17 41   7 21 49 
= hrs. i.e., 2 hrs 43 min approximately LCM  + 1, + 1, + 1  = LCM  , , 
x x 11  2 4 8  2 4 8 

5 6
LCM (7, 21, 49)
i.e.,  9.43 a.m. =
HCF(2, 4, 8)
59. (c) Let the distance be ‘d’, then we have
d = 5(t + 6) = 7(t – 30); 49 × 3
= = 73.5 s.
where t is usual time 2
⇒ 5t + 30 = 7t – 210 65.
(b) Let tunnel = 8 km and speed of cat = 8 km/hr
⇒ 2t = 240 Time taken to reach entrance of tunnel by cat = 3 hr
⇒ t = 120 min. Time taken to reach exit of tunnel by cat = 5 hr
Then, d = 7(120 – 30) = 7 × 90 = 630 min Train will cover sum (length of tunnel) = 2 hr
60.
(d) pr2 = 616 Therefore, ratio of speeds of train and cat = 4:1.
616 × 7 66.
(d) Total amount of work = 60 man hrs
⇒ r2 = = 196
22 From 11 a.m. to 5 p.m., 6 technicians = 36 man hrs
⇒ r = 14 From 5 p.m. to 6 p.m., 7 technicians = 7 man hrs
22 From 6 p.m. to 7 p.m., 8 technicians = 8 man hrs
\ 2pr = Circumference = 2 × × 14 = 88 meters
7 From 7 p.m. to 8 p.m., 9 technicians = 9 man hrs
\ It will take 14.4 s for a runner to run around a
Total = 60 man hrs.
circular ground @ 22 km/hr.
67. (c) Speed of tiger = 40 m/min
61. (c) Let distance travelled at 40 km/hr = x km
Speed of deer = 20 m/min
Then, distance travelled at 60 km/hr = (250 – x) km
x 250  x Relative speed = 40 – 20 = 20 m/min
Given:  = 5 Difference in distances = 50 × 8 = 400 m
40 60
⇒ 3x + 500 – 2x = 600 \ Time taken in overtaking (or catching)
⇒ x = 100 km 400
= = 20 min
2
62. (d) If speed of N = 4 units
\ Distance travelled in 20 min = 20 × 40 = 800 m
Speed of S = 1 unit
2 × 4 ×1 1 1 1
⇒ Average speed = = 1.6 units 68.
(d) Work done in one day by A, B, C and D are , ,
4 +1 4 8 16

2 3 1
Because time available is , speed = . and , respectively. Using answer choices, we note
3 2 32
Now average speed = 2.4 3
that the pair of B and C does of work in one day;
Speed of N now = 8 16
Speed of S now = y 1 1 9
the pair of A and D does + = of the work in
2×8× y 4 32 32
= 2.4 one day.
8+ y
32
⇒ y = 1.3 Hence, A and D take days.
9
Required ratio = 1.3:8 ≈ 1:6.
16 32
63.
(c) AG1 = 5 min at 30 km/hr = 2.5 km B and C take = days. Hence, the first pair
3 6
G1G3 = 15 km must comprise of A and D.
Time for AG1 = 5 min
69.
(a) Amount of money given to X
Time for G1G3 + G3A = 32.5 min
= 12 × 300 + 12 × 330 + ... + 12 × 570
= total of 37.5 mins
= 12 [300 + 330 + ... + 540 + 570]
1 min is taken for transferring the patient into and out
of the ambulance. 10
= 12 × [600 + 9 × 30] = 52,200
Hence (40 – 37.5 – 1) = 1.5 mins are remaining. 2

Chapter_12.indd 304 1/30/2016 4:18:05 PM


 Time and Distance  305

Amount of money given to Y is 72. (c) Let the time taken by Asit, Arnold and Afzal to do the
6 × 200 + 6 × 215 + 6 × 230 + 6 × 245 + ... to 20 terms work alone be x, y and z hrs respectively.
20 \ Time taken to do the work together is
= 6 ×
[400 + 19 × 15] = 6 [400 + 285] z
2 x – 6 = y – 1 =
= 60 × 685 = 41,100 2
1 1 1
\ Total amount paid = 52,200 + 41,100 = 93,300. In one hour, they can do + + of the work.
x y z
70.
(d) Let x be rowing rate of Rahul and y, be the rate of flow \ Total work can be completed by them in
of current in mph. xyz xyz
hrs, i.e. =x–6
12 12 xy + yz + zx xy + yz + zx
− = 6
x−y x+y Now, put y = x – 5 and z = 2 (x – 6) in the above
20
y 1 equation and solve for x. We get x = hrs.
⇒ 2 2
= 3
x −y 4
Hence, time taken by them to complete the work when
x 2 − y2 20 2
⇒ y = ...(1) working together = −6 = hour = 40 mins.
4 3 3
When Rahul dobles his rowing rate, then we have 73.
(a) Suppose the speeds of the trains are x m/s and y m/s
12 12 respectively.
− = 1
2x − y 2x + y 110 + 130
Then = 60 …(1)
2y 1 x−y
⇒ =
4 x 2 − y2 12 110 + 130
= 3 …(2)
2 2 x−y
4x − y
⇒ y = ...(2) By Eqs. (1) and (2) we get
4
x = 42 m/s
Hence, from Eqs. (1) and (2), we have 2x2 = 5y2
and y = 38 m/s
5 2
Putting x2 = y in (1), we get 74.
(d) Total bogies initially is 12 + 12 = 24
2
Additional bogies = 16 – 12 = 4
3 2 24 bogies takes = 2 mins
y
8
y = 2 ⇒y= . 2
4 3 1 bogy takes =
24
71.
(b) If Shyam takes 1 min for every 3 steps, then he takes 2 1
1 4 bogies takes = ×4 = min
min for every step. 21 3
3
1
= × 60 s = 20 s.
25 3
For 25 steps, he takes mins, i.e., 8.33 mins.
3
75. (b) 60 km/hr is travelled in 4 litres petrol (from the graph)
1 \ 1 litre is required for 15 km.
So, Vyom takes min for every step.
2 i.e., for 15 km, 1 litre petrol is required.
20
For 20 steps, he takes mins, i.e., 10 mins.
2
Difference between their times = 1.66 mins.
Escalator takes 5 steps in 1.66 mins.
Speed of escalator is 1 step for 0.33 min. i.e. 3
steps per minute.
If escalator is moving, then Shyam takes 25 steps
and escalator also takes 25 steps.
Hence, total number of steps = 50. For 200 km, = 13.33 litres is required.

Chapter_12.indd 305 1/30/2016 4:18:05 PM


306  Chapter 12

40 60 5 5 2 5D
76.
(b) = 16.66 km; = 15 km ⇒ VM =  VG   D 
2.4 4 6 6 11 33
80 Mayank covers 5D in 33 hrs
= 10 km approximately
7.9 6 33 6
\ Answer is 16.66 km, decrease the speed. D in
So he will cover  = 3.6 hrs
11 5 11
77. (a) The robot begins to give material to machine A and   = 3 hrs 36 min
then to D, it thus covers 40 m in that time span and Alternative method 2:
takes 4s. Also then it returns to the origin, and takes
4s, while covering 40 m again. When it arrives at the
origin, the messages of B and C are already there, thus VG tM
=
it moves to give the material to them, which takes it VM tG
in total 6s, and covers 30 + 30 = 60 m in total. Hence
2
the distance travelled by the robot will be 40m + 40m  6 tM
+ 60 m = 140m.
⇒   =
5 tG
78. (a)
36 5
79. (d) \ tm =  = 3 hrs 36 min
25 2
80.
(c) After 4 hrs distance between Shruti and Archana = (13
× 4 – 8 × 4) = 20 km.
Let x be the point where they meet on the way.    Let both of them reach the destination C after x
AX 6 km, then time taken by A in travelling (20 + x) km is
⇒      [As their speeds are in this ratio] equal to time taken by B in travelling x km as given
XB 5 
t m d m / S m d m SG
Now     
tG dG / SG dG Sm

d m AX 6 tm 6 6 36
⇒ Since         20  x x  60
dG XB 5 tG 5 5 25 
   4  ⇒ x = 60 km
16 12  12 
2.5  36
      
tm = = 3.6 hr = 3 hr 36 min \ Total time of the journey = = 9 hrs
25
Alternative method 1: 81.
(c) Distance travelled by the thief in 15 min
Conventional method of solving 15
= 60  = 15 km.
60
Hence, distance between police and thief when police
started to chase = 15 km. Relative speed = (65 – 60) =

5 km/hr.
Let Golu and Mayank met at point C which is x
Hence, time taken by police to catch the thief
kilometres from A, and A and B are D kilometres
apart. 15
= = 3 hrs.
V 6 5
The ratio of speeds = G  . Hence, required time = (12 hr + 15 min + 3 hrs) = 3.15
VM 5
p.m.
They take same time to reach point C.
x Dx 82.
(b) Since the speed of the another policeman is same as
⇒ =  that of thief. Hence distance between thief and him
6 5
will be 15 km. And this is the required distance.
6
\ x =  D 83.
(b) In all, X has travelled for 25 hrs (including stoppages)
11 at an average speed of 180 miles per hour. Hence, the
5 5 distance between Frankfurt and India = (25 × 180) =
Now Golu covers D in h.
11 2 4,500 miles
5D 2D 84.
(a) For the return journey, X halts at Boston for one hour
So VG =   km/h
5 11 less than his previous halt. Hence time taken by X for
11 
2 his return journey is 24 hrs.

Chapter_12.indd 306 1/30/2016 4:18:06 PM


 Time and Distance  307

85.
(a) The distance between Frankfurt and India = 4,500 88.
(c) Let the speed of the man be x km/hr
miles. Therefore, total distance travelled by him = Relative speed = 20 + x
(4,500 + 4,500) = 9,000 miles. Time taken by him
⇒ Distance covered at (20 + x) km/hr in 8 mts.
 11  = Distance covered at 20 km/hr in 10 mts.
including halting time 1 h
 12 
Solving we get x = 5 km/hr
 11 
=  25  24  1  = 50.9 hrs 8
 12  = 35 km/hr
11
Hence, average speed
89.
(b) Let the speed of the second train be x km/hr.
Total distance 9,000 \ Relative speed = (x + 60) km/hr
= = = 176.81 m/hr
Total time 50.9
and the total distance
86.
(b) 180 + 270 450
= km = km
1000 1000
450 10.8
Q = ( x + 60) ×
1000 60 × 60
450 × 60 × 60
⇒ (x + 60) = = 150
10.8 × 1000
Applying cosine rule to find the third side
\ x = 150 – 60
BC2 = AB2 + AC2 – 2AB . AC cos 120º
= 90 km/hr.
1
= 144  64  2  12  8  90.
(b) Let both the trains will meet at x km far away from
2 Delhi.
= 144 + 64 + 96 Then as per question,
⇒ BC = 304 = 4 19 x x
= +2
\ Time taken by Ram to travel to A 60 80
⇒ 80x – 60x = 2 × 80 × 60
12  4 19  8
= ⇒ 20x = 2 × 80 × 60
3
\ x = 480 km.
Time taken by Shyam to go to A
91.
(c)
8  4 19  8
=
2
\ Required difference
24  12 19  36  24  8 19  16
=
6

4 19  20 2 19  10
= = \ AE = 1 km

6 3
87.
(d) Let the speed of the train be x km/hr 92.
(d)
Relative speed = x – 3 km/hr
Distance covered in 33 s = 300 m Let the distance between A and B be P km and speeds
of A and B be x km/hr and y km/hr, respectively
300 × 3600
= x – 3 When they meet first:
33 × 1000
7 P−7
360 = ... (1)
⇒ = x – 3 x y
11
When they met second
8 P+4 2P − 4
⇒ x = 32 + 3 = … (2)
11 x y

Chapter_12.indd 307 1/30/2016 4:18:06 PM


308  Chapter 12

From (1) and (2) Now, train is at A and man is at C and both will take
7 P−7 same time for reaching at B.
=
P+4 2P − 4 y x
\ =
⇒ 2
14P – 28 = P – 7P + 4P – 28 30 3( M )
⇒ P2 = 17P 10x
⇒ M = …(1)
⇒ P = 17 km. y
Total distance travelled Also train and man will take same time for reaching
= 34 × 10 = 340 km at ‘D’
Total cost = 340 × 20 = `6800 y+x 2x
\ =
30 3M
93. (c) Distance travelled by Car A = 65 × 8 = 520 km
20x
Distance travelled by Car B = 70 × 4 = 280 km \ M = (2)
y+x
520
Ratio = = 13:7 Solving equation (1) and (2), we get the value of
280
M = 10 km/hr
94. (a) Distance = 30 × 6 = 180 km 98. (b) Speed of father = 7.5 km/hr
180 Speed of son = 5 km/hr
Speed of Hema = = 45 km/hr
4 \ relative speed = 2.5 km/hr
Speed of Deepa after increasing average speed
1
180 1 \ time taken to catch the son = h = 24 min
= = 4 h = 4 hrs 30 min 2.5
30 + 10 2 \ in 24 min son would cover 2 km = 2000 m
Speed of Hema after increasing average speed
180 3 99. (d) Speed of the train = 120 km/hr
= = 3 h = 3 hrs 36 min
45 + 5 5 5
= 120 × m/s
Difference = 4 hrs 30 min – 3 hrs 36 min = 54 min 18
480 100
95. (a) Speed of bus = = 60 km/hr = m/s
8 3
60 × 4 Suppose the length of the platform = x m
Speed of train = = 80 km/hr
3 Then,
Speed of train : Speed of car = 16:15 x + 320
= 24
80 100
\ Speed of car = × 15 = 75 km/hr 3
16
Distance covered by car in 6 hrs = 75 × 6 = 450 km ⇒ 3(x + 320) = 100 × 24
⇒ x + 320 = 800
50
96. (b) Time taken from road 1 = = 1 hrs ⇒ x = 800 – 320 = 480 m
50
Hence, speed of a man
Time Taken from road 2 is = 50/(80/4) = 2.5 hrs
\ Total time taken = 3.5 hrs 480
= m/s = 2 m/s
4 × 60
\ Total distance travelled = 100 km
100 100.
(b) Let the hare take 4 leaps/m/min and hound 3 leaps/
\ Average speed = km/hr ≈ 29 km/hr min
3.5
1
97. (d) Let train is at a distance y km from the tunnel and the \ Speed of hare = 4  1 = 6 m/min
2
length of the tunnel is x km. Man is at point ‘C’ which
1 1
x and speed of hound = 3 × 2 = 7 m/min
is km away from B. 2 2
3
Initial distance = 30 m
3
Difference in speed = m/min
2
Let M km/hr be the speed of man.

Chapter_12.indd 308 1/30/2016 4:18:07 PM


 Time and Distance  309

Now, Q Speed × Time = Distance 108. (a) s = t2 + t


3 If t = 2,
⇒ × t = 30 s = 22 + 2 = 6
2
⇒ t = 20 mins If t = 4,
Time taken by hound to catch hare is 20 mins, hare s = 42 + 4 = 20
travels 20 × 6 = 120 m Hence, required difference = 20 – 6 = 14

101.
(b) t = 4×42 109. (d) Let the length of the train = x m
\ So, the length of the platform = 3x m
1 5
4x
= 2 × 2 4 = 2 4 = 21.25 \ Time taken in crossing the platform = s
20
and distance = 23.25 x
Time taken in crossing the pole = s
Distance 23.25 20
\ Speed = = 1.25 = 22 = 4 m/s
Time 2 x 4x
\ + 24 =
20 20
2 2
102.
(a) Traveller distance in 6 mins = ×6 = km ⇒ x = 160
60 10
2 6 8 110. (c) Average speed of a tractor
Total distance in 6 mins = + = km 575
10 10 10 = = 25 km/hr
23
8
Speed = × 10 = 8 km/hr The speed of a bus in an hour = 25 × 2 = 50 km
10
9
103. (b) According to question The speed of a car in an hour = 50 × = 90 km
5
270 18 So, the distance covered by car in 4 hrs is
× = 36 s
25 + 2 5 90 × 4 = 360 km
111. (a) Let the length of train-B = x m.
x2 25/4 5
104. (a) Ratio of speeds =
= = x
x1 4 4 then the length of train-A = m
2
By option method, we can conclude the speed of x
slower train is 24 km/hr. x
Speed of train-A = 2 =
105. (a) 25 50
x
Speed of train-B =
75
x
A 50 75
Ratio of speed = = = = 3:2
Ratio = 16:15 B x 50
75
106. (d) Relative speed = (5 – 4) km = 1 km
112. (d) Suppose the speed of the train = x m/s
35
Time taken = = 35 hrs 240 240 + 480
1 + 40 =
107.
(b) x x
240 + 40x = 720
40x = 720 – 240
40x = 480
Distance travelled in 1 hrs = 20 km 480
x = = 12 m/s
Remaining distance = 110 – 20 = 90 km 40
Time taken = Distance/Relative speed or, Distance = 2 × 240 m = 480 m
90 Time = 40 s
= = 2 hrs
(20 + 25) 480
Speed = = 12 m/s
So, time = 8 am + 2 = 10 a.m. 40

Chapter_12.indd 309 1/30/2016 4:18:08 PM


310  Chapter 12

200 + 400 600 121.


(d) Rajesh has to walk 5 km in mall.
113. (a) Speed of the train = =
36 36 Let, the distance from mall to his home = x
50 50 18 5 x
= m/s = × = 60 km/hr \ Time taken by him =  h
3 3 5 10 10
Time taken by him with the speed of 15 km/hr
114. (b) From point A to B, speed = 4 km/hr
5 x
From point B to A, speed = 6 km/hr =  h
15 15
Ratio of required time = 4:6 = 2:3
5 x 5 x 1
115. (a) Distance between Ramgarh and Devgarh Now,    = [19:00 – 18:30]
10 10 15 15 2
50 × 44 110 ⇒ x = 10
= =
60 3 Now, the total distance Rajesh has to travel = 15 km
Average speed of the bus is increased by 5 km/hr then
the speed of the bus = 55 km/hr 122.
(b) Mukesh starts from Delhi (say A). He has to take one
of the other two (say Dinesh) on his bike, take him
Distance
Required time = upto a certain point (say C) drop him there and return
Speed for Suresh. Mean while Suresh starts walking. Suresh
110 60 and Mukesh meet at (say B) Mukesh picks up Suresh
= × = 40 min at B and turns towards Mathura. All of them arrive
3 55
together at Mathura (say D)
116. (a) Bus fare = `420
3
Train fare = 420 × 2 × = `630
4
Total fare = 2 × 420 + 4 × 630
= 840 + 2520 = `3360

117. (a) If the length of train B is x m then, speed of train
  As M drives at 60 km/hr and S (as well as D) walk
240 + x 240 at 15 km/hr
⇒ =
50 20
AC + CB = 4(AB)
240 + x BC + CB = 3(AB)
= 12
50 CB = 1.5(AB)
240 + x = 600
Let AB = 2, BC = 3 (Also CD = 2)
x = 600 – 240
600 900 600
x = 360 m Actually, AB = , BC = , CD =
7 7 7
118. (b) Total distance = 64 × 8 = 512 km
600 00  600
512 Time taken = 
Now speed = ≈ 85 km/hr 7  15 7  60
6
40 25 2
119.
(d) =  = 9
7 7 7
123.
(c) Time taken by Sunil for 1st round = 2 min

2nd round = 4 min, 3rd round = 8 min,
1
Distance PQ = 60 × 6 = 380 km 4th round = 16 min, 5th round = 32 min,
3
6th round = 64 min, 7th round = 128 min,
380  4
Speed of Swift = = 80 km/hr 8th round = 256 min
19
Q Anil takes one min for every round.
15000 m \ He meets 127 times in 7th and 255 times in 8th
120.
(b) Speed of Amarendra = = 10 m/s.
1500 s round with Sunil.

12500 m 124.
(a) Expenditure of diesel for 1 km
Speed of Dharmandra = = 25/3 m/s
1500 s 1  1000  
=    x
Consider the 7 min early departure of Dharmendra. 400  x  

Chapter_12.indd 310 1/30/2016 4:18:08 PM


 Time and Distance  311

128.
(c) Let the journey be from A to B and the total distance
 1000  
Expenditure of diesel for 800 km = 2    x between A and B be D km.
 x  
\ Expenditure on diesel Let the original speed of the train be V km/hr.
 1000  
= 35 × 2 
   x  rupees
 x 
 800 
Time of journey =  hhrs
 x 
125  800   Let the point where the accident occurred be C
\ Driver’s pay = rupees
x and consider that the time the train would have taken
to cover the distance CB at V km/hr be t hrs.
Total cost = 70 
 x
 
 1000  125  800
 x 
 x    Now, travelling at
3
4
V, the time taken by the train
Now if we go throw the options 4
to cover the distance CB will be t , i.e., the train will
Then total cost at speed 49 km/hr = `6899.38 3
At speed 55 km/hr = `6940.90
 t t
At speed 50 km/hr = `6900.00 take  t   or extra time than what it would have
 3 3
At speed 53 km/hr = `6917.55 taken had no accident taken place.
125.
(b) 1
   The train was late by 3 hrs as hr
h was idle as it
2
was detained.
t
= 3 hrs ⇒ t = 9 hrs
Suppose they meet after time t. 3
Distance travelled by one train = 21t Without any accident, the train would have taken
Distance travelled by second train = 16t 10 hrs for the entire journey, as accident occurred after
having travelled for an hour.
21t – 16t = 60
⇒ 5t = 60
⇒ t = 12 hrs
Distance between them It is given that, had the accident occurred 90 km
further, the train would have been late by 3 hrs. So,
= 21t + 16t
3
= 37t = 37 × 12 travelling at V over a distance of 90 km, train takes
4
= 444 km
1
126.
(a, c, d)  Laxman takes the first train which is slower one. hr extra.
2
Bharat takes the faster train. Let the trains be A and B V T = 90 ...(1)
respectively. To overtake, train A; train B, has to cover
3
its length. As we cannot determine the length of the and V T1 = 90 ...(2)
slower train, we cannot find the time taken in over 4
take. Hence, option (b) is not correct. T hrs and T1 hrs = The time taken to cover 90 km
3
127.
(b) When the two boys meet for the first time, A covers respectively at V km/hr and V km/hr
(60 – 2) = 48 km and B covers (60 + 12) = 72 km 4
3
VT = VT1
4

4 1
T1 = T ⇒ T1 = T + T
  As the speed of A is 4 km/hr slower than B, B 3 3
covers 4 km more than A every one hour. Now, by the
1 1 3
time they meet, B has covered (72 – 48) = 24 km more T = ⇒T= h
than A. At the rate of 4 km/hr, B covers 24 km more 3 2 2
in 6 hrs. Thus, they meet after 6 h. A covers 48 km in 3
Substituting T = hrs in Eq. (1),
6 hrs. So, speed of A = 8 km/hr. 2

Chapter_12.indd 311 1/30/2016 5:41:04 PM


312  Chapter 12

90 1 2
V = = 60 km/hr 132.
(d) s = ut + at
3 2
2 1
So, total distance between A and B = 60 × 10 = 600 km \ Distance (s) = 0  4 
 9.8  4  4
2
129.
(a) Let the speeds of the faster cyclist and the slower (\ a = g = 9.8; u = initial speed = 0)
cyclist be F and S respectively. = 78.4 m
The distance between the two cyclists = D
133.
(b) Let the distance between Delhi and Mumbai be x km.
If they travel in the same direction, the faster one will
x
catch up the slower one in t hrs. Time taken by Jet airways flight = hrs
700
(F – S) r = D ...(1)
If they travel in opposite directions, together they x
Time taken by Kingfisher flight = hrs
must cover the total distance of D km in t hrs. 560
(F + S) t = D ...(2) x x 10 x  8 x
Stoppage time =  =
From Eqs. (1) and (2), 560 700 5600
(F – S) r = (F + S) t x
= hhrs
FS r 2800
=
FS t x / 2800
Average stoppage time per hour =
By componendo and dividendo, x / 560
F r t 560
= = hhrs = 12 mins
S r t 2800

150 22
130.
(a) Speed while going == 45 km/hr 134.
(b) 2πr = 2   70 = 440 cm
20 7
3
60
Distance in 10 revolution = 440 × 10 cm
Total distance
Average speed = 440  10 4400 18
Total time \ Speed =  
5 5  100 5
300 Speed = 31.68 km/hr
r =
7.5
135.
(b) Distance in one hour = 18 + 20 = 38 km
r = 40 km/hr
Relative speed (20 + 18) = 38 km/hr
\ Average rate for the trip going exceeds r by 45 –
40 = 5 km/hr 47.5 1
Time =  1 hhrs
38 4
131.
(d) The two men meet when the sum of the distances
covered by them is 76. 136.
(b) D = S × T
1
Distance covered by
Sum of dis-
\ =p–q
r
No. of tances covered
hrs Man at R Man at S by the men 137.
(a) Ratio of speed = 2:3
(in km) Ratio of time = 3:2
1 4.5 3.25 7.75 \ 3x – 2x = 10
2 4.5 + 4.5 = 9 3.25 + 3.75 = 7 16 ⇒ x = 10
3 9 + 4.5 = 13.5 7 + 4.25 = 11.25 24.75 A takes 3x = 30 min
4 13.5 + 4.5 = 18 11.25 + 4.75 = 16 34
5 18 + 4.5 = 22.5 16 + 5.25 = 21.25 43.75
B takes 2x = 20 min
6 22.5 + 4.5 = 27 21.25 + 5.75 = 27 54 30
When speed of A is double, then time = = 15 min
7 27 + 4.5 = 31.5 27 + 6.25 = 33.25 64.75 2
8 31.5 + 4.5 = 36 33.25 + 6.75 = 40 76 138.
(c) Relative speed = (40 – 22) km/hr
The men meet 36 km from R and 40 km from S. They 5
= 18 × = 5 m/s
meet 4 km nearer to R than S. 18
1 min = 300 m

Chapter_12.indd 312 1/30/2016 4:18:09 PM


 Time and Distance  313

⇒ L1 + L2 = 300 In second case when x/2 km is covered in 2t hrs, speed


⇒ 125 + L2 = 300 x /2 x
to (s2) = =
\ L2 = 175 m 25 4t
x /t
D D 40 Hence, ratio of speeds = = 4 = 4:1
139.
(d)  = …(1) x /4t
x x  3 60
145. (b) Suppose Ram has to travel 1 km
D D 40 Time taken to cover 1 km in first case
and  = …(2)
x  2 x 60 1
= hr = 20 min
\ x = 12 km/hr and D = 40 km 3
Time taken to cover 1 km in second case
x x 25
140.
(d)  = [Let x distance] 1
5 6 60 = hr = 15 min
4
6x  5x 25
= Difference in time to cover 1 km = 20 mins – 15
30 60 mins = 5 mins
x = 12.5 km But Actual difference =15 mins +15 mins = 30 mins
141.
(b) Let the speed of the cyclist be x km/hr and wind be y If the difference is 5 mins, distance = 1 km
km/hr. 1
1 3 If the difference is 30 mins, distance × 30 = 6 km.
\ = 5
x  y 60
146
(c) Distance covered by train in 50 mins
1 1
⇒ = 48
x  y 20 = × 50 km = 40 km
60
⇒ 20 = x + y …(1)
Now this distance has to be covered in 40 mins.
1 4 40
and = Hence, new speed of train = × 60 = 60 km/hr
x  y 60 60
⇒ x – y = 15 …(2)
147.
(b) Suppose total journey = x km
On solving Eqs. (1) and (2), we get According to the question,
35 5 2
x = ,y= x = 1200
2 2 5
2 \ x = 3000 km
Now, time taken by cyclist without wind =  60
35 1
3 Distance covered by car = (3000) km = 1000 km
= 3 min 3
7 Hence, the distance travelled by train
142.
(d) Total initially bogies is 12 + 12 = 24 = 3000 – (1200 + 1000)
Additional bogies =16 – 12 = 4 = 3000 – 2200 = 800 km.
24 bogies take 2 mins 148.
(c) Let the speed of train on level terrain = x km/hr
2  60
4 bogies will take =  4 = 20 s
24
143.
(b) Relative speed of rockets
= (42000 + 18000) = 60000 miles per hour
Then, the speed of train through mountainous
   It means both of them together cover 60000 miles
in 60 mins, i.e., 1000 miles in one minute. Therefore, = (x – 10) km/hr
they should be 1000 miles apart, 1 minute before According to the question,
impact. 188 111
+ =7
144.
(a) Suppose x km is covered in t hrs x x − 10

x 188 x – 1880 + 111x
\ Original speed (s1) = ⇒ =7
t x( x − 10)

Chapter_12.indd 313 1/30/2016 4:18:10 PM


314  Chapter 12

⇒ 7x2 – 369x + 1880 = 0 ⇒ 60x = 50x + 6000


⇒ x = 47 km/hr ⇒ x = 600
149.
(b) Relative speed of the trains \ Total distance = x + (x + 120) = 1320 km
= 45 + 30 = 75 km/hr 154.
(a) Let the average speed be x km/hr.
Distance to be covered = 500 + 500 = 1000 m 800
Time taken by aircraft (t) = …(1)
Now 75 km is covered in 1 hour = 3600 s x
3600 × 1000 40 80
\ 1000 m is covered in = = 48 s t− = …(2)
75 × 1000 60 x + 40
From Eqs. (1) and (2),
150.
(a) Distance travelled by X in 1 hrs = 50 km
800 800 2
1 = +
Distance travelled by Y in hr = 20 km x x + 40 3
2
32000 2
Hyderabad Nalgonda Chennai ⇒ =
50 km/h x( x + 40) 3
X ← 100 km → Y 40 km/h
⇒ x (x + 40) = 48000
At 6:30, distance between 2 trains = 30 km
\ x = 200 km/hr
Time taken to travel this 30 km
155.
(b) Let the speed of Scooty be x km/hr.
30 1 1000
= = hhr Time taken by Scooty (t) = …(1)
50 + 40 3
x
1 Time taken by Auto
= × 60 =
20 min
3 1000
(t – 5) = …(2)
x + 10
151.
(a) Let the distance be d km and normal speed be s km/hr.
d From Eqs. (1) and (2),
\ Normal time (t) = hhr …(1) 1000
s 1000
= +5
1 d x x + 10
\ t− = …(2)
2 s+4 10000
⇒ =5
x( x + 10)
1 d
and t+ = ...(3) ⇒ x (x + 10) = 2000
3 s−4
\ x = 40 km/hr
Solving Eqs. (1), (2) and (3), we get
156.
(c) Actual time to reach that place
d = 60 km, s = 20 km/hr and t = 3 hrs
7
152.
(b) Let the speed of train on onward journey be x km/hr. = 22 × = 14 hrs
11
Then, the speed of train on return journey = 0.8 x km/
hr So, total time saved = 22 – 14 = 8 hrs

500 1 500 157.


(d) Let the speed of train be x km/hr.
Total time = + +   As both the persons are walking in the same
x 2 0.8 x
direction of train.
1125 1
⇒ 23 = + So, (x – 4.5) × 8.4 = (x – 5.4) × 8.5
x 2
⇒ 0.1 x = 8.1
2
⇒ x = 1125 × = 50 km/hr ⇒ x = 81 km/hr
45
\ Speed of train on return journey = 40 km/hr 158.
(a) Let the length of platform be x m, length of first train
y
153.
(a) Let the distance covered by first train be x km. be y m and length of second train be m. As both
2
Then, the distance covered by second train trains are travelling in opposite direction.
= (x + 120) km y 5
As both trains have travelled for same time. So, y+ = (48 + 42) × × 12
2 18
x x + 120 3
\ = ⇒ y = 300
50 60 2

Chapter_12.indd 314 1/30/2016 4:18:10 PM


 Time and Distance  315

⇒ y = 200 m 165.
(b) Distance (D) = Speed (S) × Time (T)
5
Now, y + x = 48 × × 45 = 600  15 
18 \ D = 4 ×  T + 
 60 
⇒ x = 600 – 200 = 400 m
⇒ D = 4T + 1 ...(1)
159.
(a) Let the usual speed of car be x km/hr.
 10 
75 75 15 and D = 6  T − 
Then, = +  60 
x x + 10 60
D = 6T – 1 ...(2)
 10  1
⇒ 75   = Solving Eqs (1) and (2),
 x( x + 10)  4
T = 1 hrs
⇒ x(x +10) = 3000 = 50 × 60
D = 4 × 2 + l = 5km
\ x = 50 km/hr
300 166.
(a) Let the normal speed of train = x km/hr
\ Required time = = 5 hrs
50 + 10 Let the normal time of train = T hrs
160.
(c) Mohan can reach the middle in 12.5 mins. Then,
300
=T
Puran can reach the middle in 25 mins. x
So, required time = 25 – 12.5 = 12.5 mins. 300
and =T–2
161.
(a) Speed of car A = 120 km/hr x+5

Speed of car B = 120 × 0.85 = 102 km/hr Solving Eqs (1) and (2),
1 x = 25 – 30
Distance traveled by car A in 1 hrs h
2 Discarding the negative value,
1 Speed of train = 25 km/hr
= 120 × 1 = 180 km
2
As the cars are travelling towards each other. 167.
(c) Time taken by cycle = x mins
668.4 − 180 Time taken by scooter = y min
So, required time = = 2.2 hhrs
120 + 102 So, x + y = 390 ...(1)
= 2 hrs 12 mins and 2x = 520 ...(2)
5 Solving Eqs. (1) and (2),
162.
(a) Length of train = 40 × (100 – 64) × = 400 m
18 y = 130 mins
163.
(b) Let the speed of train be x km/hr. Moving both ways by scooter = 2 × 130 = 260 mins
600 600 = 4 hrs 20 mins
Then, = +4
x x+5 168.
(a) Let the time of meet = t hrs
 5 
⇒ 600   =4
 x( x + 5) 
⇒ x(x + 5) = 750 = 25 × 30
⇒ x = 25 km/hr
 20 
164.
(b) Let the original time = T hrs 15  t −  + 20t = 450
 60 
Let the original speed = x km/hr
⇒ t = 13 hrs
1500
\ = T ...(1)  1
x Distance from A = 15 13 −  = 190 km
1500  3
30
and =T– ...(2)
x + 250 60 169.
(a) Speed of Ramesh = 3x

Solving Eqs. (1) and (2), Speed of Suresh = 4x
Speed of plane = x = 250 – 1000 Let the distance = D
Discarding negative value, D D 1
− =
x = 750 km/hr 3x 4 x 2

Chapter_12.indd 315 1/30/2016 4:18:11 PM


316  Chapter 12

D 1  1 25 5
⇒   = 176.
(b) Time taken to cover 25 km = hrs = hrs
x  12  2 40 8
⇒ D = 6x Remaining distance
\ Remaining speed =
D 6x Remaining time
Time taken by Ramesh = = = 2 hrs
3x 3x 46 − 25 21
= = = 56 km/hr
D 6x 5 3
Time taken by Suresh = = = 1.5 hrs 1−
4x 4x 8 8

170.
(c) Length of bridge = 1000 m 177.
(c) Let the trains will meet x hrs after the train from
station B started.
Length of train = 500 m
 45 
Total length = 1000 + 500 = 1500 m 48 ×  x +  + 50 × x = 232
 60 
1500 60
Speed of train = × = 45 km/hr ⇒ 48x + 36 + 50x = 232
1000 2
⇒ x = 2 hrs
192 Distance travelled by the train starting from station A
171.
(c) Time taken by motorcyclist = = 6 hrs
32 at the meeting point
 45 
= 48 ×  2 + =132 km
 60 
Time taken by car = 6 – (2.5 + 0.5) = 3 hrs
178.
(c) Distance travelled by train in 4 hrs = 45 × 4 =180 km
Ratio of the speeds of motor cycle:car
180 × 1000
= 3:6 = 1:2 Number of telegraph poles = = 3600
50
172.
(a) Let the speed of goods train be x km/hr.
179.
(d) Let the speed of car be x km/hr.
As the trains are running in opposite direction.
As the pedestrain is walking in the same direction
5 0.6
187.5 = (50 + x) × ×9 \ (x – 2) = × 60
18 6
⇒ 50 + x = 75
⇒ x = 6 + 2 = 8 km/hr
⇒ x = 25 km/hr
22
173.
(c) Time taken by 1st cyclist to complete a track 180.
(b) Distance travelled = 2 × × 100
7
300
= s 22
7 2× × 100
\ Speed = 7 = 314.29 ≈ 314 m/min
Time taken by 2nd cyclist to complete a track
2
300
= s 181.
(a) The relative speed of the train with respect to the
8
speed of the man
300 300 = speed of the train + speed of the man
\ Required time = LCM of and = 300 s
7 8 = 60 + 6 = 66 km/hr
45 + 50 + 25 \ Time taken by the train in crossing the man
174.
(c) Average speed of car = = Time taken in moving a distance equal to the
45 50 25 × 3
+ + length of the train (110 m) at the rate of 66 km/
15 25 25
hr.
120
= = 15 km/hr 3600
3+ 2+3 = × 100 = 6 s
66000
175.
(c) Let the distance travelled at the speed of 50 km/hr be 182.
(b) Time taken by car for one way
x km and (170 – x) km at the speed of 100 km/hr.
6 h 45 min − 2 h 1
x 170 − x = = 2 h 22 min
Then, + =2 2 2
50 100 Time taken in walking to go one way
⇒ 2x + 170 – x = 200
1
⇒ x = 30 km = 6 hrs 45 min – 2h 22 min
2

Chapter_12.indd 316 1/30/2016 5:42:11 PM


 Time and Distance  317

1 New distance = 15 × 10 = 150 m


= 4 hrs 22 min
2 150
\ Required time = = 15 s
\ Time taken in walking to go both ways 10
 1  188.
(a)
= 2  4 h 22 min 
 2 
= 8 hrs 45 min
183.
(b) Let length of the bridge be x m
100 + x 25
\ = ⇒ x = 400
72000 3600

184.
(d) Train with a speed of 54 km/hr passes the man in 20 s. Distance travelled in 2 hrs will be 2x and 2(x + 5)
5 respectively.
\ Length of the train = 54 × × 20 = 300 m
18 \ (50)2 = (2x)2 + [2(x + 5)]2
Let the length of platform be x m. 2500 = 4x2 + 4x2 + 100 + 40x
5 2
Then, (300 + x) = 54 × × 36 ⇒ 8x + 40x – 2400 = 0
18 x2 + 5x – 300 = 0
\ x = 540 – 300 = 240 m (x – 15) (x + 20) = 0 ⇒ x = 15 km/hr
185. (d) Let the speed of the faster train be x km/hr and that of 189.
(c)
slower train be y km/hr.
Then, in first case, relative speed = (x – y) km/hr
5
\ (130 + 110) = (x – y) × × 60
18
⇒ (x – y) = 14.4 …(1)
In second case, relative speed = (x + y) km/hr

\ (130 + 110) = (x + y) × × 30
18
⇒ (x + y) = 288 ...(2) Clearly, the radius of the circle is 20 km. Hence, one
From Eqs. (1) and (2), has to cover a distance of 20 km to reach a point
between B and C.
2x = 302.4
302.4 190.
(c) Let the normal speed be x km/hr.
\ x = = 151.20 km/hr
2 300 300
\ = +2
x x+5
186.
(c) Let the distance of the journey be x km.
⇒ x = 25
x x
Then, time taken to cover km = h 191.
(c) Let the speed of train be x km/hr.
2 8
5 75
x x Then, ( x − 6) × = ×2
Time taken to cover km = h 18 15
3 36
18
x x ⇒ x – 6 = 10 ×
and time taken to cover km = h 5
6 54
⇒ x = 42 km/hr
x x x 31
Given, + + = Let the speed of second person be y km/hr.
8 36 54 5
5 75
⇒ x ≈ 36 km Then, (42 − y ) × = ×4
18 24
187.
(a) Length of train = 12 × 15 = 180 m
100 18
Time = 18 s ⇒ 42 – y = ×
9 5
180
Speed = = 10 m/s 18 ⇒ y = 42 – 40 = 2 km/hr
18

Chapter_12.indd 317 1/30/2016 5:43:59 PM


318  Chapter 12

192.
(c) Distance travelled by train A in 3 hrs ⇒ 5n2 + 75n – 770 = 0
= 3 × 60 = 180 km \ n = 7
Relative speed = 72 – 60 = 12 km/hr
199.
(c) Let the body moving downwards take ‘t’ s to reach
180 half the height.
Time taken to meet = = 15 hrs
12
245 1
So, two trains meet at (2 pm + 15 h) ⇒ = × 9.8 × t2 (g = 9.8)
8 2
= 5 pm on next day
245 × 2 25 5
193.
(b) Let he travel x km by train. ⇒ t2 = = ⇒t= s
8 × 9.8 4 2
x
\ Total time he travelled by train = h Again, assume that the second body is projected with
55
velocity v upwards
285 − x
Total time he travelled by car = h 245 5 245
40 ⇒ = v× −
x 285 − x 8 2 8
\ + =6
55 40 5 245 245 245
⇒ v× = + =
⇒ 8x + 3135 – 11x = 2640 2 8 8 4
⇒ 3x = 495 245 2 49
= × = = 24.5 m/s
⇒ x = 165 km 4 5 2
194.
(d) Let the distance from P to R is x km. 200.
(a) The trains will cross each other after
As both A and B travel for same time. 2000 + 200 + 300 2500
x 21  (21  x) = = 50 s
\ = 20 + 30 50
3 4
201.
(a) The coin will move up with a initial velocity of 10 m/s
till it comes to rest. Time taken is given by
⇒ 4x = 126 – 3x 10
0 = 10 – 9.8 t ⇒ t = s
⇒ x = 18 km 9.8
195.
(c) As the trains are running in opposite directions. Time taken to reach the ground from the highest point
 1.10 + 0.90  10 39.2 − 10 29.2
\ Required time =   hhrs = 4− = = s
 60 + 90  9.8 9.8 9.8
2 Velocity of coin on impact
= × 60 × 60 s = 45 s
150 29.2
=0+ × 9.8 = 29.2 m/s
196.
(b) Suppose the distance is 270 km. 9.8
[Q L.C.M. of 54 and 45 = 270] If ‘h’ is the height from which the coin was dropped,
then (29.2)2 – (10)2 = 2 × 9.8 × hrs ⇒ hrs = 38.4 m
Without stoppage, time taken in the whole journey
270 202.
(a) Average speed of the train leaving Delhi
= = 5 hhrs
54 200
= = 50 km/hr
With stoppage time taken in the whole journey 4
270 Average speed of the train leaving
= = 6 hrs
h
45
200 × 2 400
(6 − 5) 1 Ambala Cantt = = km/hr
\ Stoppage per hour = = hrs = 10 mins 7 7
6 6
By the time the other train starts from Ambala Cantt
197.
(a) Wherever the two trains may meet, they will be the first train had travelled 100 km.
equidistant from a given place.
Therefore, the trains meet after
n 100 100 × 7 2×7 14
198.
(d) We have [2 × 40 + (n – 1)5] = 385 = = hrs = × 60 = 56 mins
2 400 750 15 15
50 +
n 7
⇒ (80 + 5n – 5) = 385
2 Hence, they meet at 8:56 a.m.

Chapter_12.indd 318 1/30/2016 4:18:13 PM


 Time and Distance  319

203.
(d) Let the length of first train be x m and that of bridge is x(V + 6 – V )
ym ⇒ 4 =
V (V + 6)
90 × 5
Speed = 90 km/hr = = 25 m/s
18 4 x
⇒ =
Q D = S × T 6 V (V + 6)
\ (x + y) = 25 × 36 = 900 ...(1) 2
\ x = V(V + 6)
Length of another train = (x – 100) 3
45 × 5 2
Speed = m/s From Eq. (1), 12 × V(V + 6) = 10(V2 – 36)
18 3
Q D = S × T
⇒ V = 30
45 × 15
\ (x – 100 + y) = ×t \ x = 720 km
18
207.
(b) Speed of the aeroplane in still air = 320 km/hr Speed
25
⇒ (900 – 100) = ×t [from Eq. (1)] of the wind = 40 km/hr
2
\ Aeroplane will travel with the wind at
800 × 2
⇒ t = = 64 s (320 + 40) = 360 km/hr
25
Aeroplane will travel against the wind at
204.
(b) Let the speed of the train be x km/hr and the speed of
(320 – 40) = 280 km/hr.
car be y km/hr.
Suppose distance to be travelled = K km.
160 600
Then, + = 8 …(1) K 1
x y \ = 2 (135 mins)
280 4
240 520 1 41
and + = 8 = …(2) 280 × 9
x y 5 5 i.e., K = = 630
4
From Eqs. (1) and (2), x = 80 and y = 100
630
\ Speed of the train = 80 km/hr Therefore, it takes hrs to cover a distance of
360
Speed of car = 100 km/hr K = 630 at 360 km/hr. i.e., 105 mins
205.
(c) Let the speed of the bus be x km/hr. 208.
(d) Let the speed of the fast train be x km/hr.
\ Speed of the car = (x + 25) km/hr
\ Speed of the slow train = (x – 16) km/hr
500 500
\ = + 10 192 192
x x + 25 \ = −2
x x − 16
⇒ x2 + 25x – 1250 = 0
⇒ 192 (x – 16) = 192x – 2x(x – 16)
⇒ x = 25
⇒ x = 48
\ Speed of the bus = 25 km/hr
209.
(b) Let the distance be x km and time be t hrs.
Speed of the car = 50 km/hr
Now, according to the question,
206.
(c) Let the length of the journey be x km and speed of
x
train be V km/hr.
x 2
x v1 = and v2 =
Then, = t – 4 …(1) t 2t
V +6
x x
or v1:v2 = : = 4:1
x t 4t
and = t + 6 …(2)
V –6
210.
(c) Let x km/hrour and y km/hr be the speeds of the two
x x cars respectively.
\ − = 10
V −6 V +6 7x – 70 = 7y (If go in same direction)

⇒ 12x = 10(V2 – 36) ...(3) y = 70 – x
From Eq. (1), (If go in opposite directions)
x x ⇒ x + y = 70
= −4
V +6 V and x – y = 10

Chapter_12.indd 319 1/30/2016 4:18:13 PM


320  Chapter 12

x = 40, y = 30 or 300x +1500 – 300x = 2x(x + 5)


211.
(b) Let speed of the train be x km/hr. Let speed of the car or 1500 = 2x2 + 10x
be y km/hr. or 2
x + 5x – 750 = 0
120 480 or 2
x + 30x – 25x – 750 = 0
\ + =8
x y or x(x + 30) – 25(x + 30) = 0

200 400 1 25 or (x – 25)(x + 30) = 0
+ = 8 =
x y 3 3 or x = 25 or – 30

600 2400 \ x = 25 km/hr
⇒ + = 40
x y (neglecting the –ve sign).

600 1200 216.
(d) Let speed of train = x km/hr and speed of car = y km/hr
+ = 25
x y According to the question,

1200 120 (600 − 120)
⇒ = 15 + =8
y x y

⇒ y = 80, x = 60 or 480x + 120y = 8xy ...(1)
\ Required ratio = 60:80 = 3:4 200 400
and + = 8.33
212.
(b) Suppose the usual speed = K km/hr x y

1500 1 1500 or 384x + 192y = 8xy ...(2)
− =
K 2 K + 250 From Eqs. (1) and (2),
⇒ K = 750 km/hr 120y + 480x = 192y + 384x
213.
(c) Length of the trains = 120 m 480x – 384x = 192y – 120y
Relative velocity of the two trains running in opposite or 95x = 72y
directions = (40 + 20 =) 60 m/s \ x = 0.75y
Time taken to cross each other Substituting the value of x in Eq. (1),
120  120 120 480
=4s + =8
60 0.75y y

214.
(a) Suppose the average speed of train = x km/hr 160 480
or + =8
\ Average speed of car = (x – 6) km/hr y y

260
\ Time taken by car to cover 260 km = hr 640
x−6 or y = = 80 km/hr.
8
260
Time taken by train to cover 260 km = hr (d) Area of circular field = pr2 = 616
217.
x
616 616
According to the question, or r2 == = 196
π 22 / 7
260 260 260( x − x + 6)
− = 3 or =3 \ r = 14 km
x−6 x x( x − 6)
Circumference of the circular field
or 520 = x2 – 6x
22
or 2
x – 6x – 520 = 0 = 27pr = 2 × × 14 = 88 km
7
or x2 – 26x + 20x – 520 = 0
Time taken to cover 22 km = 1 hr
or (x – 26)(x + 20) = 0
1
\ x = 26 or x = –20 \ Time taken to cover 88 km = × 88 = 4 hrs
22
Since x cannot be –ve,
218.
(a) 1st speed = 6 km/hr
\ x = 26 km/hr.
Distance covered by man in 50 @ 6 km/hr
215.
(c) Suppose normal speed of train = x km/hr
\ Increased speed of train = (x + 5) km/hr 50
= 6 × = 5 km
According to the question, 60
300 300 300( x  5)  300 x 2nd speed = 10 km/hr
 = 2 or =2
x ( x  5) x( x  5)

Chapter_12.indd 320 1/30/2016 4:18:14 PM


 Time and Distance  321

Time taken to cover 10 km = 1 hour = 60 mins    Hence monkey will travel remaining 6 m in next
60 minute.
Time taken to cover 5 km = × 5 = 30 mins
10 \ Total time taken = (10 + 1) = 11 min.

219.
(c) Speed of the train = 48 km/hr 223.
(c) When two or more persons start from the same place
Time taken = 50 mins at the same time and travel round a circle in the same
direction or in opposite directions, then they will
\ Distance covered by train in 50 mins
be first together at the starting point again after an
48 interval of time which is the L.C.M. of the times in
= × 50 km = 40 km
60 which each of them makes one complete round.
Now to cover the distance in 50 mins speed
300 300
= 48 km/hr Here each one takes s and s in making one
7 8
Now to cover the distance in 40 mins speed complete round.
48 × 50 300 300
= km/hr = 60 km/hr \ Required time = L.C.M. of and
40 7 8
220.
(b) Average speed for 1st 5 mins L.C.M. of 300 and 300 300
= = s
= 300 sec.
68 + 127 + 208 + 312 + 555 H.C.F. of 7 and 8 1
= m/mins
5 224.
(a) Let the second car overtakes the first car after n hrs
1250 \ Distance covered by first car = Distance covered
= m/mins = 250 m/mins
5 by second car
250 × 60  1  n − 1
= km/hr = 15 km/hr or, 10n = 8 +  8 +  + ... +  8 + 
1000  2   2 
Average speed for next 5 mins = 33 km/hr 1
Hence, average speed for or, 10n = 8n + [1 + 2 + ... + (n − 1)]
2
15 + 33 48
10 mins = = = 24 km/hr. 1  n(n − 1) 
2 2 or, 10n = 8n +
2  2 
221.
(a) Let the speed of the cyclist be k km/hr.
1 n2 − n
\ Speed of the jogger = k km/2 hrs or, 2n =
2 4

1 \ n = 9 as n ≠ 0
= k km/hhrs
4 225.
(b) Speed in km/hr = 132 km/hr
\ Ratio of the speeds of the jogger and the cyclist 5 110
Speed in m/s = 132 × = m/s
1 18 3
k
4 = 1 Total distance to be covered = 110 + 165 = 275 m
k 4
3 × 275
\ Time taken = s
= 7.5 sec
222. (a) Net distance travelled by monkey in 2 mins = 3 mins 110
\ In 10 mins distance travelled = 15 mins
226.
(c) 480 km are to be covered in 8 hrs at the rate of
As it is given in the question that in first minute it 60 km/ hr. Hence increased speed
climbs 6 m and in other minute it slips down 3 m. And
= 60 – 48 = 12km/hr
this happens al­ternatively.

Chapter_12.indd 321 1/30/2016 4:18:14 PM


13 Boats and streams

Some Important Terms 4. Downstream If a boat (or a swimmer) moves with the
stream, i.e. along the direction of the stream, it is called
1. Still Water If the speed of the water in the river is zero, downstream.
it is still water.
Note: When the speed of a boat or a swimmer is given, it
2. Stream If the water of the river is moving, it is called a usually means speed in the still water.
stream.
3. Upstream If a boat (or a swimmer) moves against
the stream, i.e. in the direction opposite to that of the
stream, it is called upstream.

soME BasIC ForMUlaE

1. If the speed of a boat (or a swimmer) be x km/ (b) Speed of the stream
hr and the speed of the stream or the current be y 1
= (Downstream Speed – Upstream Speed)
km/hr, then 2
(a) speed of the boat (or swimmer) downstream
= (x + y) km/hr. Illustration 2 A boat is rowed down a river 40 km in 5 hr
(b) speed of the boat (or swimmer) upstream and up a river 21 km in 7 hr. Find the speed of the boat and
= (x – y) km/hr. the river.
40
Solution: Speed of the boat downstream = = 8 km/hr.
5
Illustration 1 The speed of a boat in still water is 20 km/hr. 21
If the speed of the stream be 4 km/hr, fi nd its downstream Speed of the boat upstream = = 3 km/hr.
7
and upstream speeds. \ Speed of the boat
Solution: Speed of the boat (x) = 20 km/hr 1
= (Downstream Speed + Upstream Speed)
Speed of the stream ( y) = 4 km/hr 2
1 11
\ Downstream speed = x + y = (20 + 4) = 24 km/hr = (8 + 3) = = or 5 ×5 km/hr.
2 2
and upstream speed = x – y = (20 – 4) = 16 km/hr. and speed of the river
1
2. (a) Speed of the boat (or swimmer) in still water = (Downstream Speed – Upstream Speed)
2
1
= (Downstream Speed + Upstream Speed) 1 5
2 = (8 – 3) = or 2.5 km/hr.
2 2

Chapter_13.indd 322 1/30/2016 4:46:43 PM


Boats and Streams 323

soME UsEFUl sHort-CUt MEtHoDs

Explanation
1. If a man capable of rowing at the speed of x km/hr
in still water, rows the same distance up and down Let the speed of the man in still water be x km/hr.
a stream which fl ows at a rate of y km/hr, then his Then, downstream speed = (x + y) km/hr
average speed throughout the journey is and upstream speed = (x – y) km/hr.
Uptream × Downstream Since the distance covered downstream and upstream
=
Man's rate in still water are equal, we have
( x − y )( x + y ) (x + y)t1 = (x – y)t2
= km/hr.
x
or, xt1 + yt1 = xt2 – yt2
Illustration 3 A man rows at a speed of 8 km/hr in still or, x(t2 – t1) = y(t2 + t1)
water to a certain distance upstream and back to the starting
point in a river which fl ows at 4 km/hr. Find his average t +t 
speed for total journey. \ x = y  2 1  km/hr.
Solution: Average speed  t2 − t1 

Upstream × Downstream Illustration 5 A motorboat covers a certain distance


=
Man's rate in still water downstream in 6 hrs but takes 8 hrs to return upstream to
the starting point. If the speed of the stream be 6 km/hr, fi nd
(8 − 4)(8 + 4)
= = 6 km/hr. the speed of the motor boat in still water.
8 Solution: Speed of the motorboat in still water
2. A man can row a boat in still water at x km/hr. In t +t 
= y  2 1  km/hr
a stream fl owing at y km/hr, if it takes t hrs more  t2 − t1 
in upstream than to go downstream for the same
distance, then the distance is given by 8+6
= 6  = 42 km/hr.
2 2
( x − y )t 8−6
km
2y
4. A man can row a boat in still water at x km/hr. In
a stream fl owing at y km/hr if it takes him t hrs to
Illustration 4 A man can row 7 km/hr in still water. If the
row to a place and come back, then the distance
river is running at 3 km/hr, it takes 6 hrs more in upstream
between the two places is
than to go downstream for the same distance. How far is
the place? t ( x2 − y 2 )
km.
Solution: The required distance 2x
( x 2 − y 2 )t
= Explanation
2y
(49 − 9)6 Downstream speed = (x + y) km/hr
= = 40 km.
2×3 Upstream speed = (x – y) km/hr.

3. A man rows a certain distance downstream in t1 Let the distance between the two places be d km. We
hrs and returns the same distance upstream in t2 have,
hrs. If the speed of the stream be y km/hr, then the Total time = Sum of time taken downstream and
speed of the man in still water is given by upstream
t +t 
y  2 1  km/hr. d d
 t2 − t1  ⇒ t= +
x+ y x− y

Chapter_13.indd 323 1/30/2016 4:46:44 PM


324 Chapter 13

 ( x − y) + ( x + y)  5. A boat (or a swimmer) takes n times as long to


=d 
 ( x − y )( x + y )  row upstream as to row downstream the river. If
the speed of boat (or swimmer) be x km/hr and the
 2x  speed of stream be y km/hr, then
=d 2 2
x − y   n +1
x = y .
t ( x2 − y 2 )  n −1 
\ d= km.
2x
Illustration 7 A man can row at the rate of 4 km/hr. in still
Illustration 6 A man can row 6 km/hr in the still water. If water. If the time taken to row a certain distance upstream
the river is running at 2 km/hr, it takes him 3 hrs to row to a is 3 times as much as to row the same distance downstream,
place and back. How far is the place? fi nd the speed of the current.
Solution: The required distance Solution: We have,
 n +1
t ( x2 − y 2 ) Speed of the man =   speed of the current
= km  n −1 
2x
 3 +1
3(36 − 4) ⇒ 4=   speed of the current.
= = 8 km.  3 −1 
2×6 \ Speed of the current = 2 km/hr.

MUltIplE CHoICE QUEstIons

1. A man can row three-quarters of kilometer against the 5. A ship carrying 1,000 people moves 12 km down stream
1 1 and then 4 km upstream. The river current is 1 km/hr.
stream in 11 mins and returns in 7 mins. The speed
4 2 Within what limits must the speed of the ship (which is
of the man in still water is denoted as V) lie for the entire trip to take not less than 3
(a) 2 km/hr (b) 3 km/hr hrs and not more than 4 hrs?
(c) 4 km/hr (d) 5 km/hr 1
(a)  V  2  3 (b) 2  3  V  2  3
[Based on MAT, 2005] 3
1 1
2. A man swimming in a stream which fl ows 1 km/hr fi nds (c) V  5 (d) 2 ≤ V ≤ 5
2 3
that in a given time he can swim twice as far with the
6. A and B in boat B1 challenge C and D in boat B2 in a race
stream as he can against it. At what rate does he swim?
of 50 km. A and B take turns to row the stretches of 6 km
1 1
(a) 5 km/hr (b) 4 km/hr and A begins. C and D take turns to row an hour and C
2 2 begins. A and C can each row 5km/hr, B and D each can
1 1
(c) 7 km/hr (d) None of these row only 5 km/hr. Which boat wins and by what time?
2 2
[Based on MAT, 2008]
(a) B1 wins by 2 mins
3. A boat goes 24 km upstream and 28 km downstream in 6
hrs. If it goes 30 km upstream and 21 km downstream in 6 7
(b) B2 wins by 1 mins
hrs and 30 mins, fi nd the speed of the stream. 11
(a) 10 km/hr (b) 5 km/hr (c) Both reach the fi nishing spot together
(c) 4 km/hr (d) 6 km/hr 4
(d) B1 wins by 5 mins
[Based on MAT, 2001] 11
4. A person can row with the stream at 8 km per hour and 7. At his normal speed, Ramesh can travel 18 km downstream
against the stream at 6 km an hour. The speed of the in a fast fl owing stream in 9 hrs less than what he takes
current is to travel the same distance upstream. The downstream
(a) 1 km/hr (b) 2 km/hr trip would take one hour less than what the upstream trip
(c) 4 km/hr (d) 5 km/hr would take, provided he doubles his rate of rowing. What
[Based on FMS (Delhi), 2002] is the speed of the stream in km/hr?

Chapter_13.indd 324 1/30/2016 4:46:45 PM


 Boats and Streams  325

2 1 took 1 hr 15 mins to cover the same distance. What was


(a) 6 (b) 8 the average speed during the whole journey?
3 3
(a) 40 mph (b) 48 mph
8 10 950  960
(c) (d) (c) 50 mph (d) 55 mph
3 1000
16. A boat takes 90 mins less to travel 36 miles downstream
8. In a stream that is running at 2 km/hr, a man goes 10 km than to travel the same distance upstream. If the speed of
upstream and comes back to the starting point in 55 mins. the boat in still water is 10 mph, the speed of the stream is
Find the speed of the man in still water.
(a) 4 (b) 3
(a) 20 km/hr (b) 22 km/hr
(c) 2.5 (d) 2
(c) 24 km/hr (d) 28 km/hr
17. Speed of a speed boat when moving in the direction
9.
A boat sails 15 km of a river towards upstream in perpendicular to the direction of the current is 16 km/hr,
5 hrs. How long will it take to cover the same distance speed of the current is 3 km/hr. So the speed of the boat
downstream, if the speed of current is one-fourth the against the current will be (in km//h)
speed of the boat in still water
(a) 22 (b) 9.5
(a) 1.8 hrs (b) 3 hrs
(c) 10 (d) None of these
(c) 4 hrs (d) 5 hrs
18. Two boats, travelling at 5 and 10 km/hr, head directly
10.
A motorboat went the river for 14 km and then up the river towards each other. They begin at a distance of 20 km
for 9 km. It took a total of 5 hrs the entire journey. Find the from each other. How far apart are they (in kms) one
speed of the river flow if the speed of the boat in still water minute before they collide?
is 5 km/hr. (a) 1/12 (b) 1/6
(a) 1 km/hr (b) 1.5 km/hr (c) 1/4 (d) 1/3
(c) 2 km/hr (d) 3 km/hr
19. At his usual rowing rate, Rahul can travel 12 miles
11.
The different between downstream speed and upstream downstream in a certain river in 6 hrs less than it takes
speed is 3 km/hr and the total time taken during upstream him to travel the same distance upstream. But if he could
and downstream is 3 hrs. What is the downstream double his usual rowing rate for this 24 mile round trip,
speed, if the downstream and upstream distance are 3 km the downstream 12 miles would then take only one hour
each? less than the upstream 12 miles. What is the speed of the
(a) 2.5 km/hr (b) 4.3 km/hr current in miles per hour?
(c) 4 km/hr (d) 3.3 km/hr (a) 7/3 (b) 4/3
12.
The current of the stream is 1 km/hr. A boat goes 35 km (c) 5/3 (d) 8/3
upstream and back to the starting point in 12 hrs. The 20. A man can row 30 km upstream and 44 km downstream
speed of the motorboat in still water is in 10 hrs. Also, he can row 40 km upstream and 55 km
(a) 6 km/hr (b) 7 km/hr downstream in 13 hrs. Find the rate of the current and the
(c) 8.5 km/hr (d) 8 km/hr speed of the man in still water.
(a) 3 km/hr, 8 km/hr (b) 3×5 km/hr, 7×5 km/hr
13.
A boat takes 5 hrs more while going back in upstream than
(c) 4 km/hr, 7 km/hr (d) 4×5 km/hr, 6×5 km/hr
in downstream. If the distance between two places is 24
km and the speed of boat in still water so that if can row 21. P, Q, R are three towns on a river which flows uniformly.
donwstream, 24 km, in 4 hrs? Q is equidistant from P and R. A man rows from P to Q
(a) 1.5 km/hr (b) 3.5 km/hr and back in 10 hrs. He can row from P to R in 4 hrs. The
(c) 4.5 km/hr (d) 3 km/hr ratio of speed of man in still water to the speed of the
current is
14.
A man can row 30 km upstream and 44 km downstream in (a) 5:3 (b) 3:5
10 hrs. It is also known that he can row 40 km upstream
(c) 2:5 (d) 1:2
and 55 km downstream in 13 hrs. Find the speed of the
man in still water. 22. A boatman goes 2 km against the current of the stream in
(a) 4 km/hr (b) 6 km/hr 1 hr and goes 1 km along the current in 10 mins. How long
(c) 8 km/hr (d) 12 km/hr will he take to go 5 km in stationary water?
(a) 1 hr (b) 1 hr 15 mins
15. A boat, while going downstream in a river covered a
1
distance of 50 miles at an average speed of 60 miles per (c) 1 hrs (d) 40 mins
hour. While returning because of the water resistance, it 2

Chapter_13.indd 325 1/30/2016 4:46:45 PM


326  Chapter 13

23. A man can swim 3 km/hr in still water. If the velocity of 31. Speed of a man is 10 km/hr in still water. If the rate of
the stream be 2 km/hr, the time taken by him to swim to a current is 3 km/hr, then the effective speed of the man
place 10 km upstream and back, is upstream is
1 1 (a) 7 km/hr (b) 8.5 km/hr
(a) 8 hrs (b) 9 hrs (c) 9 km/hr (d) None of these
3 5
(c) 10 hrs (d) 12 hrs 32.
A man can row 6 km/hr in still water. If the speed of the
current is 2 km/hr, it takes 3 hrs more in upstream than in
24. Twice the speed downstream is equal to the thrice the
the downstream for the same distance. The distance is
speed upstream, the ratio of speed in still water to the
speed of the current is (a) 30 km (b) 24 km
(c) 20 km (d) 32 km
(a) 1:5 (b) 5:1
[Based on SSC (GL), 2011]
(c) 1:3 (d) 2:3
33.
On a river, B is between A and C and is equidistant from A
25. A man rows upstream 12 km and downstream 28 km and C. A boat goes from A to B and back in 7 hrs 15 mins
taking 5 hrs each time. The velocity of water current is and from A to C in 7 hrs. How long will it take to go from
1 1 C to A, if water flows from A to C.
(a) 2 km/hr (b) 2 km/hr 1
5 2 (a) 3 hrs (b) 3 hours
hrs
2
3
(c) 3 km/hr (d) 1 km/hr 1
5 (c) 4 hrs (d) 4 hrs 3 sec q
hours
2
1 [Based on GBO Delhi University, 2011]
26. A man swimming in a stream which flows 1 km/hr finds
2
that in a given time he can swim twice as far with the 34.
A man can row 6 km/hr in still water. If it takes him twice
stream as he can against it. At what rate does he swim? as long to row up, as to row down the river, then the rate
of current in the stream would be
1 1
(a) 4 km/hr (b) 5 km/hr (a) 4 km/hr (b) 2 km/hr
2 2 (c) 3 km/hr (d) 8 km/hr
1 [Based on ATMA, 2006]
(c) 7 km/hr (d) None of these
2 35.
A boat goes 30 km upstream and 44 km downstream in 10
27. A boat travels upstream from B to A and downstream hrs. In 13 hrs, it can go 40 km upstream and 55 km down
from A to B in 3 hrs. If the speed of the boat in still water stream. The speed of the boat in still water is
is 9  km/hr and the speed of the current is 3  km/hr, the (a) 3 km/hr (b) 4 km/hr
distance between A and B is (c) 8 km/hr (d) None of these
(a) 8 km (b) 16 km [Based on ITFT, 2008]

(c) 12 km (d) None of these 36.


A motor boat can travel at 10 km/hr in still water. It
traveled 91 km downstream in river and then returned,
28. In a river flowing at 2 km/hr, a boat travels 32 km upstream taking altogether 20 hrs. Find the rate of flow of river.
and then returns downstream to the starting point. If its (a) 3 km/hr (b) 5 km/hr
speed in still water be 6 km/hr, find the total journey time. (c) 6 km/hr (d) 7 km/hr
(a) 16 hrs (b) 12 hrs [Based on IIFT, 2005]
(c) 14 hrs (d) None of these 37.
A man can row at 5 km/hr in still water. If the river is
running at 1 km/hr, it takes him 75 min to row to a place
29. If a man’s rate with the current is 12 km/hr and the rate of and back. How far is the place?
1 (a) 2.5 km (b) 3 km
the current is 1 km/hr, then his rate against the current is
2 (c) 4 km (d) 5 km
(a) 13 km/hr (b) 7 km/hr [Based on FMS, 2005]
(c) 9 km/hr (d) None of these 38.
A boat goes 24 km upstream and 28 km downstream in
30. A swimmer covers a distance of 28 km against the current 6 hrs. It goes 30 km upstream and 21 km downstream in
and 40 km in the direction of the current. If in each case 6 hrs and 30 mins. The speed of the boat in still water is
he takes 4 hrs, then the speed of the current is (a) 10 km/hr (b) 4 km/hr
(a) 3.5 km/hr (b) 1.5 km/hr (c) 14 km/hr (d) 6 km/hr
[Based on MAT, 1999]
(c) 2.5 km/hr (d) None of these

Chapter_13.indd 326 1/30/2016 4:46:45 PM


 Boats and Streams  327

39.
A motor boat whose speed is 15 km/hr in still water goes (a) 30 m/mins (b) 29 m/mins
30 km downstream and comes back in 4 hrs and 30 mins. (c) 31 m/mins (d) 32 m/mins
Determine the speed of the stream. [Based on MAT (Dec), 2008]
(a) 10 km/hr (b) 4 km/hr 47.
A boatman goes 2 km against the current of the
(c) 5 km/hr (d) 6 km/hr stream in 1 hr and goes 1 km along the current in
[Based on MAT, 1999] 10 mins. How long will he take to go 5 km in stationary
40.
A motor boat whose speed is 30 km/hr in still water goes water?
60 km downstream, and comes back in 4 and a half hrs. (a) 1 hr 30 mins (b) 1 hr 15 mins
The speed of the stream is (c) 1 hr (d) 40 mins
(a) 5 km/hr (b) 10 km/hr [Based on MAT (Sept), 2010 (Dec), 2009, 2007]

(c) 15 km/hr (d) 20 km/hr 48.


A man rowed against a stream flowing 1.5 km/hr to a
[Based on MAT, 1998] certain point and then turned back, stopping 2 km short
of the place from where he originally started. If the whole
41.
A boat takes 90 mins less to travel 36 miles downstream time occupied in rowing be 2 hrs 10 min and his uniform
than to travel the same distance upstream. If the speed of speed in still water be 4.5 km/hr, the man went up the
the boat in still water is 10 mph, the speed of the stream is stream a distance of
(a) 4 mph (b) 3 mph (a) 4 km (b) 8 km
(c) 2.5 mph (d) 2 mph (c) 7 km (d) 5 km
[Based on MAT, 1997] [Based on MAT (Sept), 2010 (Dec), 2009]
42.
Speed of a speed-boat when moving in the direction 49.
A man can row 30 km upstream and 44 km downstream
perpendicular to the direction of the current is in 10 hrs. Also, he can row 40 km upstream and 55 km
16 km/hr, speed of the current is 3 km/hr. So, the speed of downstream in 13 hrs. The rate of the current is
the boat against the current will be (a) 3 km/hr (b) 3.5 km/hr
(a) 22 km/hr (b) 9.5 km/hr (c) 4 km/hr (d) 4.5 km/hr
(c) 10 km/hr (d) None of these [Based on MAT (Sept), 2009]
[Based on MAT (Feb), 2006] 50.
A man rows 8 km/hr in still water. If the river is running at
43.
Twice the speed of a boat downstream is equal to thrice 2 km/hr, it takes 32 min to row to a place and back. How
the speed upstream. The ratio of its speed in still water to far is the place?
its speed of current is (a) 1.5 km (b) 2.5 km
(a) 1:5 (b) 1:3 (c) 2 km (d) 3 km
(c) 5:1 (d) 2:3 [Based on MAT (Sept), 2009]
[Based on MAT (Dec), 2007] 1
51. A man swimming in a stream which flows 1 km/hr finds
2
44.
A man can row three-fourths of a km against the stream in that in a given time he can swim twice as far with the
1 1 stream as he can against it. At what rate does he swim?
11 mins and return in 7 mins. Find the speed of the 1 1
4 2 (a) 4 km/hr (b) 5 km/hr
man in still water. 2 2
(a) 4 km/hr (b) 3 km/hr 1
(c) 7 km/hr (d) None of these
(c) 5 km/hr (d) 6 km/hr 2 [Based on MAT (Sept), 2009 (Feb), 2008]
[Based on MAT (May), 2008]
52.
A boat travels upstream from B to A and downstream from
45.
A ship 55 km from the shore springs a leak which admits A to B in 3 hrs. If the speed of the boat in still water is 9
2 tonnes of water in 6 mins, 80 tonnes would suffice to km/hr and the speed of the current is 3 km/hr, the distance
sink her, but the pumps can throw out 12 tonnes an hour. between A and B is
The average rate of sailing that she may just reach the (a) 4 km (b) 8 km
shore as she begins to sink is (c) 6 km (d) 12 km
(a) 9.17 km/hr (b) 0.97 km/hr [Based on MAT (Dec), 2008]
(c) 55 km/hr (d) 5.5 km/hr 53.
A motor boat can travel at 10 km/hr in still water. It
[Based on MAT (Sept), 2008 (Dec), 2002, 2006 (Feb), 2004] travelled 91 km downstream in a river and then returned,
46.
A man who can swim 48 m/mins in still water, swims taking altogether 20 hrs. Find the rate of flow of the river.
200 m against the current and 200 m with the current. If (a) 6 km/hr (b) 5 km/hr
the difference between those two times is 10 mins, find (c) 8 km/hr (d) 3 km/hr
the speed of the current. [Based on MAT (Dec), 2008]

Chapter_13.indd 327 1/30/2016 4:46:45 PM


328  Chapter 13

Explanatory Answers

3 4 6.
(b) For boat B1
1.
(d) Speed upstream = × × 60 = 4 km/hr.
4 45
3 2
Speed downstream = × = 6 km/hr.
4 15
1 6 12
Speed in still water = (6 + 4) = 5 km/hr. To row 6 km, A takes = hrs.
2 1 11
5
2
3  3
2.
(b) x+ = 2  x −  6
2  2 To row 6 km, B takes hrs.
5
1
\ x = 4 km/hr \ For each 12 km, they take

2
12 6 126
3.
(c) Let the speed of the stream be x km per hour.  = hrs
11 5 55
Let speed of the boat in still water be y km/hr. At the end, there remains 2 km, which is covered by A
\ Boat will travel downstream @ (y + x) km/hr and 2 4
upstream @ (y – x) km/hr. in = hrs
1 11
5
24 28 2
\ + = 6
y−x y+x \ Total time required by boat B1

30 21 1 13 126  4 4 4  126  524 29


and + = 6 = =  =   1  9 hrs
y−x y+x 2 2 55 11 11  5  55 55
⇒ y + x = 14, y – x = 6 For boat B2
⇒ x = 4, y = 10.
4.
(a) Let the speed of the current be x km/hr and speed of
the person in still water be y km/hr.
In every 2 hrs, C and D cover 10.5 km in 8 hrs, B2
\ y + x = 8 covers 42 km ⇒ in the next 1 hour, C covers 5.5 km
y – x = 6 1
1 2 1
⇒ y = 7, x = 1 To cover the last 2 km, D takes 2 = hr
2 5 2
\ Speed of the current = 1 km/hr.
1 1
Total time required by boat B2 = 8 + 1 + = 9 hrs
5.
(a) Let V be the speed of the ship. 2 2
12 4
Then time t =  29 1 3 3 7
V 1 V 1 B2 wins by  = hour =  60 = 1
55 2 110 110 11
12 4 mins.
\   3 ≤  ≤4
V 1 V 1 7.
(a) Let Ramesh’s normal speed be u km/hr and the speed
2
⇒  3(V – 1) ≤ 12(V – 1) + 4 (V + 1 ≤ 4(V – 1) 2 of the current be v km/hr.
⇒  (3V2 – 3) ≤ (16V – 8) ≤ (4V2 – 4) 18 18
– = 9
u–v uv
⇒  3V2 – 16V + 5 ≤ 0 and 4V2 – 16V + 4 ≥ 0
1 18 18
1st inequality is satisfied when  V  5 and the 2nd and – = 1
3 2u – v 2u  v
inequality is satisfied for (2 – 3 ) ≤ V ≤ (2 + 3 ). Let u = kv
1 1 1 v
\ we get V  2  3 – = ...(1)
3 k 1 k 1 2

Chapter_13.indd 328 1/30/2016 4:46:47 PM


 Boats and Streams  329

1 1 v ⇒ 12x2 – 70x – 12 = 0
and – = ...(2) or x = 6 km/hr
2k – 1 2k  1 18
24 24
Dividing Eqs. (1) by (2) 13.
(b)  =5
(5.5  R ) (5.5  R )
2 4k 2 – 1 ⇒ R = 2.5 km/hr;  R ® Speed of river/current
= 9
k2 – 1 2 24
2 2
Again (B2 + R) = =6
⇒ 4k – 1 = 9k – 9 4
⇒ (B2 + 2.5) = 6 ⇒ B2 = 3.5 km/hr
⇒ k = 85
14.
(c) Let the speed of the man in still water be x km/hr and
4(5) 42 speed of the stream be y km/hr then,
From Eq. (1) ⇒ v = 2 = = 6
k –1 3 3 30 44
 = 10 ...(1)
8.
(b) Let the speed of the man in still water be x km/hr. ( x  y) ( x  y)
40 55
Then  = 13 ...(2)
( x  y) ( x  y)
10 10 55 Solving (1) and (2) x = 8 km/hr and y = 3 km/hr
 = ⇒ x = 22 km/hr
( x  2) ( x  2) 60 15. (b) Time taken by boat in down stream
9.
(b) Upstream speed = B – S 50 5
= = h
Downstream speed = B + S 60 6
15 5
B – S = = 3 km/hr Time taken by boat in upstream = h
5 4
Again B = 4S 2  50 100  24
Average speed = = = 48 mph
5 5 50
\ B – S = 3 = 3S 
6 4
⇒ S = 1 and B = 4 km/hr
16.
(d) Speed of boat in still water = 10 mph
\ B + S = 5 km/hr
Let the speed of the steam = x mph
15
\ Time during downstream = =3h Then, speed of boat downward stream = (10 + x) mph
2
Speed of boat upward steam = (10 – x) mph
10.
(c) Let the speed of the stream be x km/hrours
36 90 36
Then Upward speed = (5 – x) km/hr and Downward \  =
(10  x) 60 (10  x)
speed = (5 + x) km/hr
14 9 1  1 1 
 = 5 ⇒ x = 2 km/hr ⇒ = 4  
(5  x) (5  x) 6  10  x 10  x 

11.
(b) Let x be the upstream speed, then the downstream 1  2x 
⇒ = 4  
speed will be (x + 3) 6  100  x 2 
3 3 ⇒ 100 – x2 = 48x
\  = 3
x x3 2
⇒ x + 48x – 100 = 0
⇒ x2 + x – 3 = 0 ⇒ x = 2 because x ≠ –50
1 13 17. (c) Speed of speed-boat = 16 – 3 = 13 km/hr.
⇒ x =
2 \ Speed of boat against the current
1  3.6 = 13 – 3 = 10 km/hr
= = 1.3 km/hr
2 18.
(c) In the final 1 min before collision, the two boats travel
\ (x + 3) = 4.3 km/hr 1 1 1 1
5 km and 10  km i.e., km and km.
12.
(a) Let the speed in the still water be x km/hr 60 60 12 6
As they move in opposite directions, distance between
35 35 the boats 1 min before collision is
 = 12
( x  1) ( x  1) 1 1 1
 =  km
or 35 × 2x = 12(x2 – 1) 12 6 4

Chapter_13.indd 329 1/30/2016 4:46:48 PM


330  Chapter 13

19.
(d) Let the speed of man in still water is x mph and speed 26
of the current be y mph. \ Speed in still water = = 4 km/hr
2
12 12 \ Time required to go 5 km in still water
Then,  = 6 ...(1)
x y x y 5
= hrs = 1 hr 15 min
12 12 4
and  = 1 ...(2)
2x  y 2x  y 23.
(d) Speed upstream = (3 – 2) km/hr = 1 km/hr
Solving Eqs. (1) and (2), y = 8/3 Speed downstream = (3 + 2) km/hr = 5 km/hr
20.
(a) Let upstream speed = x km/hr and downstream speed  10 10 
Total time taken =    h = 12 hrs
= y km/hr 1 5
30 44 40 55
Then,  = 10, and  = 13 24. (b) Let speed in still water = x km/hr
x y x y
Speed of current = y km/hr
or, 30u + 44v = 10, and 40u + 55v = 13 Speed downstream = (x + y) km/hr
1 1 Speed upstream = (x – y) km/hr
where u = and v =
x y \ 2(x + y) = 3(x – y)
1 1 \ x = 5y
Solving, we get u = and v =
5 11 x 5
\ x = 5 and y = 11 or, = or 5:1
y 1
5  11
\ Rate in still water = = 8 km/hr 25. (d) Let man’s rowing speed in still water = x km/hr and
2 speed of current = y km/hr
11  5 Speed upstream = (x – y) km/hr and speed downstream
Rate of current = = 3 km/hr
2 = (x + y) km/hr
21. (a) Let the speed of man in still water = x km/hr \ 5(x – y) = 12 and 5(x + y) = 28
Speed of the current = y km/hr subtracting 10y = 16
Speed downstream = (x + y) km/hr 8 3
\ y = = 1 km/hr
Speed upstream = (x – y) km/hr 5 5
Let river be flowing from P to R and PQ = QR = a  n  1
26.
(a) Speed of man =  speed of stream
Then, PR = 2a  n  1
 2  1 3 9 1
=   = or 4 km/
a a  2  1 2 2 2
\  = 10. ...(1) hr
x y x y
2a 27.
(c) The distance between A and B is
and, = 4
x y
t ( x2  y 2 ) 3(81  9)
= km = = 12 km
a 2x 29
\ = 2 ...(2)
x y
28.
(b) Let the total journey time be t
a
\ (1) ⇒ = 8 ...(3) t ( x2  y 2 )
x y Then, we have d =
2x
Dividing (2) and (3), we get
x y 1 t (36  4)
= ⇒ 32 =
x y 4 26
\ t = 12 h
\ 4x – 4y = x + y
29. (c) Speed of the man downstream = 12 km/hr
or, 3x = 5y
3
x 5 Speed of the stream = km/hr
or, = or 5:3 2
y 3 Let the speed of the man upstream = x km/hr
22.
(b) Upstream speed = 2 km/hr We have, Speed of the stream
Downstream speed = 6 km/hr 1
= (Downstream Speed – Upstream Speed)
2

Chapter_13.indd 330 1/30/2016 4:46:49 PM


 Boats and Streams  331

3 1 34.
(b) D = distance
⇒ = (12  x)
2 2 x = speed of stream,
\ x = 12 – 3 = 9 km/hr 2D 2D
Then =
30.
(b) Speed of the swimmer upstream 6+ x 6−x
28 \ x = 2 km/hr
= = 7 km/hr
4 36.
(c) Suppose 30 km upstream is covered in 3x hrs and 40
Speed of the swimmer downstream km upstream is covered in 4x hrs.
40   44 km downstream is covered in 4y hrs and
= = 10 km/hr 55 km downstream is covered in 5y hrs.
4
\ Speed of the stream Then,
1 3x + 4y = 10 ...(1)
= (Downstream Speed – Upstream Speed) 4x + 5y = 13 ...(2)
2
3 On solving, we get x = 2 and y = 1
= (10 – 7) = = 1.5 km/hr
2 30 km
\ Upstream speed =
31.
(a) Speed of man in still water = 10 km/hr 3x
Speed of current = 3 km/hr 30
= = 5 km/hr
\ Speed of man upstream = 10 – 3 = 7 km/hr 3 2

32.
(b) Let the reqd. distance be x km. 44 km
and downstream speed =
4y
x x
Q − = 3
6−2 6+2 44
= = 11 km/hr
x x 4 1
⇒ − = 3
4 8 11  5
\ Speed of boat = = 8 km/ hr
x 2
⇒ = 3
8 91 91
\ x = 24 km 36.
(a) Since, + = 20
x+ y x− y
33.
(b)
91 91
\  = + = 20
10 + y 10 − y
Let the distance AC = S km
By option, if y = 3
Speed of the stream = x km/hr.
91 91
Speed of the boat in still water = y km + = 20
13 7
S S 21
+ = x x 75
2 ( x + y) 2 ( x − y) 4 37.
(b) Q  =
4 6 60
S
= 7 10 x 5
( x + y) \ =
24 4
⇒ S = 7 (x + y) ⇒ x = 3 km
7 S 21
⇒ + = 38.
(a) Let a km /hour be the speed of the boat in still water.
2 2 ( x − y) 4
Let b km/hrour be the speed of the cur­rent.
S 21 7 \ Boat will travel downstream @ (a + b) km/hr and
= −
2 ( x − y) 4 2 upstream @ (a – b) km/hr.
21 − 14 7 28 24
= = \ + =6
4 4 a+b a−b

S 7 7 21 30 1 13
= × 2 = hrs. + = 6 =
( x − y) 4 2 a+b a−b 2 2

Chapter_13.indd 331 1/30/2016 4:46:50 PM


332  Chapter 13

84 72 42.
(c) Speed of speed boat = 16 – 3 = 13 km/hr
⇒ + = 18 …(1)
a+b a−b \ Speed of boat against the current
= 13 – 3 = 10 km/hr
84 120
+ = 26 …(2)
a+b a−b 43.
(c) Let the speed in still water be x km/hr and speed of

Equation (1) and (2) give current be y km/hr.
48 Then, 2(x + y) = 3(x – y)
− =–8⇒a–b=6
a−b ⇒ x = 5y

28 24 x 5
\ Eq. (1) ⇒ + =6 ⇒ =
a+b 6 y 1

⇒ a + b = 14
44.
(c) Let the speed of man in still water be x km/hr and
\ a = 10, b = 4 speed of stream be y km/hr.
39.
(c) Let speed of the stream be K km/hr.
45 3
Speed of the stream in still water = 15 km/hr \ ( x − y ) × =
4 × 60 4
\ Speed of the boat downstream = (15 + K) km/hr
\ Speed of the boat upstream = (15 – K) km/hr 3 × 60
⇒ x – y = = 4 ...(1)
30 30 9 45
\ + =
15 + K 15 − K 2
15 3
⇒ K = 5 and (x + y) × =
2 × 60 4
40. (b) Suppose speed of stream = x km/hr
3 × 2 × 60
\ Speed of boat upstream = (30 – x) km/hr ⇒ x + y = = 6 ...(2)
4 × 15
and speed of boat downstream = (30 + x) km/hr.
\ Time taken to cover 60 km upstream Solving Eqs. (1) and (2), x = 5 km/hr
60 45.
(d) In 1 h water entered into ship = (20 – 12) = 8 tonnes
= hr
30 − x
   Now, it will take 10 hrs to allow to enter 50 tonnes
and time taken to cover 60 km upstream of water into ship and in this time ship has to cover
60 55 km of distance.
= hr
30 − x Hence, required speed = 5.5 km/hr
According to the question,
46.
(d) Let the speed of current be x m/min. Then, speed with
60 60 1 9
+ = 4 = current = (48 + x) m/min and speed against current =
30 − x 30 + x 2 2 (48 – x) m/min

60(30 + x + 30 − x) 9 200 200
or = − = 10
(30 − x)(30 + x) 2 (48 − x) (48 + x)

or 60 × 60 × 2 = 9(900 – x2) ⇒ 40x = (48)2 – x2
or 900 – x2 = 800 ⇒ x2 + 40x – 2304 = 0
or x2 =100 ⇒ (x + 72)(x – 32) = 0
\ x = 10 km/hr. ⇒ x = 32 m/min
41.
(d) Suppose speed of the stream = K mph
47.
(b) Upstream speed = 2 km/hr
Speed of the boat in still water =10 mph
1
\ Boat will travel with the stream (downstream) Downstream speed = × 60 = 6 km/hr
@ (10 + K) mph and boat will travel against the 10
stream (upstream) @ (10 – K) mph. \ Speed in stationary water
Now, from the question, 2+6
= = 4 km/hr
36 90 36 2
+ =
10 + K 60 10 − K 5
\ Required time = = 1 h 15 min
K=2
⇒     4

Chapter_13.indd 332 1/30/2016 4:46:51 PM


 Boats and Streams  333

48.
(d) Let the man went up the stream for x km. Then, he 51.
(a) Let the man was swimming at the rate of x km/hr and
turned back for (x – 2) km. he can swim D km upstream.
x x−2 D 2D
\ + = 2 hrs 10 mins \ =
(4.5 − 1.5) (4.5 + 1.5) 1 1
x −1 x +1
2x + x − 2 1 2 2
⇒ = 2
6 6 3
⇒ x+ = 2x – 3
⇒ 3x – 2 = 13 2
⇒ x = 5 km 1
⇒ x = 4 km/hr
49. (a) Let the speed of man and current be x km/hr and y km/ 2
hr respectively.
52.
(d) Let the distance between A and B be x km.
30 44
Then, + = 10 …(1) Given, speed of boat in still water = 9 km/hr and speed
x− y x+ y of current = 3 km/hr
40 55 \ Upward speed = (9 – 3) = 6 km/hr and downward
and
+ = 13 …(2)
x− y x+ y speed = (9 + 3) = 12 km/hr

Solving Eqs. (1) and (2), x x
\ + =3
⇒ x + y = 11 ...(3) 6 12
and x – y = 5 ...(4) ⇒ x = 12 km
Solving Eqs. (3) and (4), 53. (d) Let the rate of flow of river be x km/hr.
y = 3 km/hr
Then, downward speed = (10 + x) km/hr
50.
(c) Let the place be x km. and upward speed = (10 – x) km/hr
x x 32
Then, + = 91 91
8−2 8+2 60 Given,   + = 20
(10 + x) (10 – x)
5 x + 3x 32
⇒ = For x = 3, the above expression holds true.
30 60
Hence, speed of flow of river is 3 km/hr.
32 30
⇒ x = × 2 km
=
60 8

Chapter_13.indd 333 1/30/2016 4:46:51 PM


14 Races and Games of skill

INTRODUCTION Suppose A and B are two contestants in a race. We give below


certain statements and their corresponding mathematical
Race A contest of speed in running, driving, riding sailing meanings, which are frequently used:
or rowing over a specifi ed distance is called race.
Statements Mathematical Interpretation
Race Course The ground or path on which contests are
arranged is called a race course. 1. A beats B by t s A fi nishes the race t s before B
fi nishes.
Starting Point The point from where a race begins is called 2. A gives B a start of t A starts t s after B starts
the starting point. s from the same point.
Winning Post (or Goal) The point where the race fi nishes 3. A gives B a start of x While A starts at the starting
m point, B starts x m ahead from
is called the winning post or finishing point or goal.
the starting point at the same
Dead-heat Race If all the persons contesting a race reach time.
the fi nishing point exactly at the same time, then the race is 4. Game of 100 A game in which the participant
called a dead-heat race. scoring 100 points fi rst is the
winner.
Winner The person who fi rst reaches the fi nishing point is 5. In a game of 100, “A While A scores 100 points, B
the winner. can give B 20 points” scores only (100 – 20) or 80
points.

soMe useFul shoRt-Cut MethoDs

1. If A is n times as fast as B and A gives B a start of x  5/3   5 


= 60   = 60   = 150 m.
m, then the length of the race course, so that both  5/3 − 1  5−3
A and B reach the winning post at the same time,
must be 2. If A can run xm race in t1 s and B in t2 s, where t1
 n  < t2, then A beats B by a distance
x  m.
 n −1  x
× (t2 − t1 )m .
t2
2
Illustration 1 A is 1 times as fast as B. If A gives B a start
3
of 60 m, how long should the race course be so that both of Illustration 2 A can run 100 m in 27 s and B in 30 s. By
them reach at the same time? what distance A beats B?
5 Solution: A beats B by a distance
Solution: Here n = and x = 60.
3 x 100 300
 n  = × (t2 − t1 ) = (30 − 27) = = 10 m.
\ Length of race course = x   t2 30 30
 n −1 

Chapter_14.indd 334 1/30/2016 4:27:44 PM


 Races and Games of Skill  335

Multiple Choice Questions

1. Alok and Atul are competing in 800 m race. Alok gives 8. A and B run a 1,760 m race ending in a dead heat. At first
Atul a lead of 100  m. Initially, Alok runs at x times of A runs 20% faster than B. B then quickens his pace, and
Atul’s speed (x is an integer > 1), but after crossing 600 m, for the remaining distance runs 20% faster than A. When
1 B quickens his pace. A has already run
he slows down to a speed which is time of Atul’s speed.
x (a) 800 m (b) 1,000 m
Atul continues to run at his original speed. If both of them (c) 790 m (d) 960 m
meet for the second time at a distance of 50 m from the
finish line, then who wins the race and by what distance? 9. In a running race, A gives a head start of 350 m to B. If
(a) Atul, 33.33 m (b) Alok, 37.5 m the ratio of speeds of A and B is 20:13, how far must the
winning post be so that A and B finish the race at the same
(c) Atul, 37.5 m (d) Alok, 25 m
time?
2. A racecourse is 400 m long. A and B run a race and A wins (a) 1 km (b) 2 km
by 5 m. B and C run over the same course and B wins by
(c) 3 km (d) None of these
4 m. C and D run over it and D wins by 16 m. If A and D
run over it, then who would win and by how much? 10. A beats B by 100 m in a race of 1200 m and B beats C by
(a) A by 8.4 m (b) D by 8.4 m 200 m in a race of 1600 m. Approximately by how many
(c) D by 7.2 m (d) A by 7.2 m metres can A beat C in a race of 9,600 m?
[Based on MAT, 2002] (a) 1,600 m (b) 1,800 m
3. A and B start simultaneously from the same end of a (c) 1,900 m (d) 2,400 m
50 m long swimming pool. The ratio of their speeds was 11. In an 800 m race around a stadium having the
2:1, respectively. If the race was one of 1,000  m, how circumference of 200 m, the top runner meets the last
many times after the start would they have met at the end runner on the 5th minute of the race. If the top runner runs
of the race ? at twice the speed of the last runner, what is the time taken
(a) 19 (b) 18 by the top runner to finish the race?
(c) 16 (d) 15 (a) 20 mins (b) 15 mins
4. In a kilometre race, A beats B by 100 m and B beats C by (c) 10 mins (d) 5 mins
150 m. In the same race, by how many metres does A beat
C? 12. In a race, Guninder was not the first. Joginder came in
(a) 250 m (b) 240 m after Harinder. Inderjeet was not ahead of Maninder.
Guninder was not in front of Joginder. Inderjeet was not
(c) 225 m (d) 235 m
fourth or fifth. Maninder was not the first. Who finished
[Based on IIT Joint Man. Ent. Test, 2004]
first and second in the race?
5. In a 1,000 m long race, Karun gives Varun a head start of
(a) Harinder followed by Maninder
40 m, and still beats Varun by 10 m. Find the distance by
which Karun will beat Varun, if Varun gives a start of 40 (b) Harinder followed by Joginder
m to Karun. (c) Harinder followed by Guninder
(a) 88 m (b) 52 m (d) Cannot be determined
(c) 40 m (d) 50 m
Directions (Q. 13 to 15):  Answer the questions based on the
6. Abha beats Mansi in a kilometre race by 50 s and Mansi following information.
beats Lata by 40 s in the same race. If Abha beats Lata by
   A certain race is made up of three stretches: A, B and C, each
450 m in a race of a 1 kilometre. How much time Lata
2 km long, and to be covered by a certain mode of transport. The
takes to run a kilometre?
table given further gives these modes of transport for the stretches,
(a) 200 s (b) 150 s and the minimum and the maximum possible speeds (in km/hr)
(c) 110 s (d) 120 s over these stretches. The speed over a particular stretch is assumed
7. In a 100 m race, Sujit beats Rishi by 5 m and Rishi to be constant. The previous record for the race is 10 min.
beats Praveen by 5 m. By what distance does Sujit beats
A Car 40 60
Praveen?
B Motorcycle 30 50
(a) 10 m (b) 11 m
C Bicycle 10 20
(c) 9 m (d) 9.75 m

Chapter_14.indd 335 1/30/2016 4:27:44 PM


336  Chapter 14

13. Anshuman travels at minimum speed by car over A and (a) 395 m (b) 355 m
completes stretch B at the fastest speed. At what speed (c) 400 m (d) 410 m
should he cover stretch C in order to break the previous [Based on MAT (Sept), 2008]
record ?
21. There are two concentric circular tracks of radii 100 m
(a) Maximum speed for C and 102 m, respectively. A runs on the inner track and
(b) Minimum speed for C goes once round the track in 1 min 30 s, while B runs on
(c) Cannot be determined the other track in 1 min 32 s. Who runs faster?
(d) None of these (a) Both A and B are equal
14. Mr Hare completes the first stretch at the minimum speed (b) A (c) B
and takes the same time for stretch B. He takes 50% more (d) None of the above
time than the previous record to complete the race. What [Based on MAT (Sept), 2008]
is Mr Hare’s speed for the stretch C? 22. A can run 22.5 m while B runs 25 m. In kilometre race B
(a) 10.9 km/hr (b) 13.3 km/hr beats A by
1
(c) 17.1 km/hr (d) None of these (a) 100 m (b) 111 m
9
15. Mr Tortoise completes the race at an average speed of (c) 25 m (d) 50 m
20 km/hr. His average speed for the first two stretches is 4 [Based on FMS (MS), 2006]
times that for the last stretch. Find the speed over stretch C.
23. In a 10 km race, First beats Second by 2 km and First
(a) 15 km/hr (b) 12 km/hr beats Third by 4 km. If the runners maintain constant
(c) 10 km/hr (d) Cannot be determined speeds throughout the race, by how many kilometers does
16. In a race of 600 m, A can beat B by 60 m and in a race of Second beat Third?
500 m, B can beat C by 25 m. By how many metres will A 1 1
(a) 2 (b) 2
beat C in a 400 m race? 4 2
(a) 56 m (c) 60 m 3
(c) 2 (d) 3
(c) 58 m (d) 54 m 4 [Based on FMS, 2010]
[Based on MAT (Dec), 2009, 2008, 2007] 24. Hari and Ravi started a race from opposite ends of the
17. A can give B a start of 20 m and C a start of 39 m in a pool. After a minute and a half, they passed each other in
walking race of 400 m. How much start can B given C? the centre of the pool. If they lost no time in turning and
(a) 22 m (c) 20 m maintained their respective speeds, how many minutes
(c) 15 m (d) 26 m after starting did they pass each other the second time?
[Based on MAT (Dec, Sept, May), 2009 (Dec), 2008 ] 1
(a) 3 (b) 4
2
18. A and B run a 5 km race on a round course of 400 m. If
their speeds be in the ratio 5:4, how often does the winner 1
(c) 6 (d) 7
pass the other? 2
[Based on FMS, 2011]
1 3
(a) 4 times (c) 2 times 25. The respective ratio between the speeds of a car, a train
2 4
and a bus is 5:9:4. The average speed of the car, the bus
1 1 and the train is 72 km/hr together. What is the average
(c) 3 times (d) 2 times
2 2 speed of the car and the train together?
[Based on MAT (Dec), 2008]
(a) 82 km/hr (b) 78 km/hr
19. A long distance runner runs 9 laps of a 400 m track every (c) 84 km/hr (d) Cannot be determined
day. His timings (in min) for four consecutive days are 88, [Based on Punjab and Sindh Bank PO, 2010]
96, 89 and 87 respectively. On an average, how many m/
min does the runner cover? 26. In a thousand metre race A beats B by 100 m and B beats
C by 100 m. How many metres start can A give to C?
(a) 17.78 (b) 90
(a) 195 m (b) 200 m
(c) 40 (d) None of these
(c) 205 m (d) 190 m
[Based on MAT (Sept) 2008]
[Based on ATMA, 2005]
20.
Muan and Sanjay ran a race which lasted a minute and a
27.
Four runners started running simultaneously from a point
half. Muan gave Sanjay a start of 9 m and beat him by 1
on a circular track. They took 200 s, 300 s, 360 s and 450 s
m. Muan ran 40 m while Sanjay ran 39 m. The length of
to complete one round. After how much time do they meet
the course is
at the starting point for the first time?

Chapter_14.indd 336 1/30/2016 4:27:45 PM


 Races and Games of Skill  337

(a) 1800 s (b) 3600 s 32. Karan and Arjun run a 100 metre race, where Karan beats
(c) 2400 s (d) 4800 s Arjun by 10 m. To do a favour to Arjun, Karan starts 10 m
[Based on SSC (GL), 2011] behind the starting line in a second 100 metre race. They
both run at their earlier speeds. Which of the following is
28. In a 100 m race, Kamal defeats Bimal by 5 s. If the speed true in connection with the second race?
of Kamal is 18 km/hr, then the speed of Bimal is
(a) Karan and Arjun reach the finishing line simultaneously
(a) 15.4 km/hr (b) 14.5 km/hr
(b) Arjun beats Karan by 1 m
(c) 14.4 km/hr (d) 14 km/hr
[Based on SSC (GL), 2010] (c) Arjun beats Karan by 11 m
(d) Karan beats Arjun by 1 m
29. In a race of 200 m run, A beats S by 20 m and N by 40 m.
If S and N are running a race of 100 m with exactly same 33. A sprinter starts running on a circular path of radius r m.
speed as before, then by how many metres will S beat N ? Her average speed (in metres/minute) is pr during the first
(a) 11.11 m (b) 10 m 30 s, pr/2 during next 1 min, pr/4 during next 2 mins, pr/8
(c) 12 m (d) 25 m during next 4 mins, and so on. What is the ratio of the time
taken for the nth round to that for the previous round?
30. In a mile race, Akshay can be given a start of 128 m by
(a) 4 (b) 8
Bhairav. If Bhairav can give Chinmay a start of 4 m in a
100 m dash, then who out of Akshay and Chinmay will (c) 16 (d) 32
win a race of one and half miles, and what will be the final
34. A runs 1 times as fast as B. If A gives B a start of 120 m
lead given by the winner to the loser?
and they reach the goal at the same time, the goal is at a
(One mile is 1,600 m) distance of
1 1 (a) 360 m (b) 440 m
(a) Akshay, mile (b) Chinmay, mile
2 32 (c) 460 m (d) 380 m
1 1
(c) Akshay, mile (d) Chinmay, mile 35. A can run 500 m in 30 sec and B in 35 s. How many metres
24 16
start can A give to B in a km race so that the race may end
31. Three runners A, B, and C run a race, with runner A finish in a dead-heat?
12 m ahead of runner B and 18 m ahead of runners C,
5 5
while runner B finishes 8 m ahead of runner C. Each (a) 139 (b) 138
runner travels the entire distance at a constant speed. 7 7
What was the length of the race? 6 5
(c) 142 (d) 140
(a) 36 m (b) 48 m 7 7
(c) 60 m (d) 72 m

EXPLANATORY ANSWERS

1. (a) It is clear that Atul wins the race. 2. (c) When A covers 400 m, B covers 395 m
   So (a) and (c) are left. Since they meet at a distance When B covers 400 m, C covers 396 m
of 50 m from the finish line, total distance covered by When B covers 395 m, C covers
Atul = 650 m and Alok = 750 m 396
× 395 = 391.05 m
600 400
⇒ 100 + + 150x = 750
x \ When A covers 400 m, B covers 391.05 m
(in the same time, distance is proportional to speed) When D covers 400 m, C covers 384 m
⇒ (3x – 4) (x – 3) = 0 When C covers 391.05 m, D covers 407.3 m
4 \ When A covers 400 m, D covers 407.3 m
⇒ x = 3 or
3 When D covers 400 m, A covers 392.8 m
\ D wins by 7.2 m, when D and A run the race.
Since x is an integer, it has to be 3.
   So, when Atul covers the remaining 50 m, Alok 3.
(d) After the start at the end of every 200  m of A they
would have met thrice and both would be at the
50
covers only , i.e., 16.67 m starting point again. Hence at the end of the 1,000 m
3  1000 
Therefore, Atul wins by 33.33 m race they would have met 3  = 15 times.
 200 

Chapter_14.indd 337 1/30/2016 4:27:45 PM


338  Chapter 14

4.
(d) By the time A covers 1000 m, B has covered 900 m. Now lag of B in time t1
\ By the time B covers 900 m, = (1.2x – x)t1 = 0.2 × t1 ...(1)
By the time B covers 1000 m, C has covered 850 m, C Also, gain of B in time t2
will cover = (1.44x – 1.2x)t2 = 0.24 × t2 ...(2)
850
× 900 = 765 m Since both reach at the same time,
1000
\ lag = lead
\ A beats C by 235 m
t1 0.24 6
⇒ = =
5.
(a) As the race is of 1,000 m. t2 0.20 5
   In first case, Karun runs 1,000 m while Varun runs
1760  t1
(960 – 10) = 950 m \ A covers of the distance
t1  t2
   In second case, Karun runs 960 m then Varun will
950  960 1760  5
run = 912 m. = = 800 m
1000 11
\ Karun will beat Varun by 1000 – 912 = 88 m 9.
(a) Let their speeds be 20x and 13x. then relative speed
of both of them moving in the same direction is
6.
(a) In a race of 1,000 m if Abha takes t sec, then Mansi
(20x – 13x) = 7x
takes (t + 50) sec and Lata takes (t + 90) s
   Now B is 350 m ahead of A and this distance has
   Now, If Abha travels 1,000 m, Lata travels 550 m
to be covered with a relative speed of 7x. Therefore,
   It is thus clear that Lata travels 1,000 m in (t + 90) 350
sec and 550 m in t s. time to cover this distance =
7x
\ 1000:550 = (t + 90):t
⇒ 100t = 55t + 4950 ⇒ t = 110 s 350
Distance covered by A in this time =
7x
Therefore, Lata takes (110 + 90) = 200 s in travelling
1,000 m. 350
Speed × time = 20x × = 1,000 m = 1 km
7.
(d) When Sujit runs 100 m, Rishi runs 95 m. 7x
When Rishi runs 100 m, Praveen runs 95 m. 10.
(c) Ratio of speeds of A:B = 12:11 and ratio of speeds of
\ When Rishi runs 95 m, Praveen runs 90.25 m. B:C = 8:7
When Sujit runs 100 m, Praveen runs 90.25 m and is Therefore ratio of speeds of A:B:C = 96:88:77
beaten by 9.75 m. So in 9,600 m race A will beat C by 1,900 m
Alternative method: 11.
(c)
The ratio of speeds of Sujit and Rishi 12.
(c)
= 100:95 13.
(c) Total time taken to cover stretch A at a minimum
= 20:19.  2 1
speed =    hrs = 3 mins.
Similarly, the ratio of speeds of Rishi and Parveen  40  20
= 20:19. Likewise total time taken to vover stretch B at a
\ The ratio of speeds of Sujit and Parveen  2
maximum speed =   = 2.4 mins. Total time taken
 50 
= 202 : 102
in covering these two stretches = (3 + 2.4) = 5.4 mins.
⇒ When Sujit goes 100 m, Parveen goes
   To break the previous record the third stretch will
361
 100 = 90.25 m have to be covered in (10 – 5.4) = 4.6 mins.
400
2
\ The lead that can be given is 100 – 90.25 Required speed =  = 0.434 km/min = 26.08 km/
4.6
= 9.75 m hr. Since the maximum speed is 20 km/hr hence it is
8.
(a) Let t1 be the time at which B switches the speed and not possible for C to break the previous record.
t1 + t2 be the total time between start and finish. Let x 14.
(b) Time taken to cover the stretch A at minimum
be the speed of B initially. So A’s speed = 1.2x and B’s 2
final speed = 1.44x speed = = 3 mins.
40

Chapter_14.indd 338 1/30/2016 4:27:46 PM


 Races and Games of Skill  339

Time taken to cover stretch B = 3 min. Time taken by 88 + 96 + 89 + 87


him in covering the entire race = (1.5 × 10) = 15 mins. 19.
(c) = 90 mm
4
   Hence remaining time to cover stretch = (15 – 6) = It means on an average he runs (400 × 9 = 3600) m
9 mins.
3600
 2 distance in 90 min or in other words he runs =
Therefore required speed =   = 0.22 km/mins. 90
 9 40 m/min
⇒ 13.3 km/hr
40 4
15.
(c) Let the average speed for the last stretch be x km/ 20.
(c) Speed of Muan = = m/s
90 9
hr, hence his average speed for the first two stretches 39 13
= 4x. So, total time taken to cover the three stretches Speed of Sanjay = = m/s
90 30
4 2
= 
4x x
4 2 6
\    ⇒  x = 10 km/hr
4 x x 20
Relative speed of Muan
16. (c) In 600 m race, Ratio of distances
 4 13  40 − 39 1
A : B =  −  = = m/s
 9 30  90 90
600 : 540

Now, in order to cover a distance of 10 m, Muan has
10 : 9
1
to run at a speed of m/s
In 500 m race, Ratio of distances 90
B : C t = 900 s
500 : 475 It cover the distance of course, he covers it with a
20 : 19 4
speed of m/s in 900 s.
9
A:B:C = (10 × 20):(9 × 20):(19 × 9)
4
= 200:180:171 \ D = × 900 = 400 m
9
So, when A runs 200 m → C runs 171 m
21.
(a)
171
When A runs 1 m → C runs m
200
When A runs 400 m → C runs = 342 m
A can beat C by = 400 – 342 = 58 m
17.
(b) In a 400 m race — Ratio of distances
A : B : C

400 : 380 : 361
Circumference of inner track = 2 × p × 100 = 200 pm
So, when B runs 380 m → C runs 361 m
Circumference of outer track = 2 × p × 102 = 204 pm
361 × 400
When B runs 400 m → C runs = 380 m 200
380 Speed of A = = 133.33 m/min
1.5
The start B can give C = 400 – 380 = 20 m
204
Speed of B = = 133.33 m/min
18.
(d) It is clear from the question that when A covers 500 m, 1.53
B covers 400 m i.e., A takes a lead of 100 m in every
500 m of distance. Therefore, a lead of 400 m will be 22.
(a) In 25 m race B beats A by 2.5 m
taken in travelling a distance of 2000 m or in other 2.5
In 1000 m race B beats A by × 1000 = 100 m
words A passes B every after 2000 m. 25
Hence, total number of such pass 23.
(b) If the First runs 10 km then the Second runs 8 km and
5000 5 1 the Third runs 6 km.
= = = 2 times
2000 2 2
\ When the Second runs 8 km the Third runs 6 km.

Chapter_14.indd 339 1/30/2016 4:27:46 PM


340  Chapter 14

When the second runs 10 km then the third runs Therefore, when S runs 100 m, N will run
6  10
= 7.5 km  160 
8 = 100   = 88.89 m.
 180 
\ The Second beats the Third by 2.5 km in a 10 km
race. Hence in a 100m race, S will beat N by (100 – 88.89)
= 11.11 m.
24. (b) To meet for the first time, both together cover the
length of the pool whereas to meet for the second 30.
(d) When Bhairav covers 1,600 m, Akshay covers (1600
time, both together need to cover 3 times the length of – 128) m. So when Bhairav covers (1600/6) = 100 m,
the pool. Akshay covers (128/16) m = 8 m less.
1    When Bhairav covers 100 m, Chinmay covers
   If to cover the length of the pool, they take 1
2 (100 – 4) = 96 m.
min, then to cover 3 times the length of the pool, they
   Thus, the ratio in which Akshay and Chinmay
1 cover distance is 92:96. In 96 m, Chinmay gains
will take 4 min.
2 (96 – 92) = 4 m over Akshay. So, in 1.5 miles,
1 1
   So, after 4 min, they will meet each other for Chinmay gains 100 m =   miles over Akshay.
2  16 
the second time. 31.
(b) Let the distance of race be x m which is covered by A
25. (c) Total speed of car, bus and train in t s. Then, in the same time B covers (x – 12) m and
C covers (x – 18) m.
= 72 × 3 = 216 km
x
5+9 \ Speed of A = m/s,
Speed of car and train = × 216 =
168 km t
5+9+4
( x  12)
168 Speed of B = m/s .
Average = = 84 km t
2
( x  18)
and Speed of C = m/s
26. (b) ____|______|________ t
C 100 m B 100 m A Time taken by B to finish the race
 A  1000m  x x
 = = t s.
B 900m   A 
 C 200 ( x  12) ( x  12)
C  800m  t
Now, distance travelled by C in this time
27.
(a) Required = L.C.M. of 200, 300, 350 and 450 s
= 1800 s x ( x  18)
= t =x–8
( x  12) t
28.
(c) Time taken by Kamal to run 100 m
100 x( x  18)
= = 20 s ⇒ = x – 8
5 ( x  12)
18 ×
18
⇒ x = 48 metre.
Therefore, time taken by Bimal to run 100 m
= 20 + 5 = 25 s 32.
(d) In the first race when Karan runs 100 m, Arjun runs
only 90 m. Hence, the ratio of speeds of Arjun and
Hence, Bimal’s speed
Karan is 90:100 = 9:10. In the second race, Karan has
100 to run 110 m. When he finishes the race, Arjun would
= = 4 m/sec
25 9
have run  110 = 99 m
10
4 × 18
= km/hr = 14.4 km/hr (i.e., 1m less than 100 m)
5
Hence Karan beats Arjun by 1 metre
29.
(a) In the time when A runs 200 m, S runs 180 m and N
runs 160 m. In other words, in the time when S runs 33.
(c) The radius of the track is r m
180 m, N runs 160 m. \ the circumference is 2pr m

Chapter_14.indd 340 1/30/2016 4:27:47 PM


 Races and Games of Skill  341

πr πr πr x x  120
The average speed for successive πr , , , , \ =
2 4 8 11s 8s
time intervals of 1/2, 1, 2, 4 etc. minutes is \ 3x = 11 × 120
πr πr πr πr \ x = 440
, , , etc., m/min. Therefore in each
2 2 2 2 35.
(c) Time taken by A to run 1 km
interval (of increasing duration) the distances travelled
are etc., (i.e., exactly the same). For such intervals = 30 × 2 = 60 sec
are needed to cover one round. The next four intervals Time taken by B to run 1 km = 35 × 2 = 70 s
are needed for the next round. As each interval in the \ A can give B a start of (70 – 60) = 10 s
second group is 16 times the corresponding interval in
In 35 s B runs 500 m
the previous group, the total time for each round is 16
500 1000
times taken for the previous round. \ In 10 s B runs = × 10 =
35 7
34.
(b) The speed of A and B are in the ratio 11:8.
Let speeds be 11s and 8s (in m/sec) 6
= 142 m
7
Let race be of x m
Then time taken by A to run xm is same as that of B to 6
  So, A can give B a start of 142 m in a km race.
run (x – 120) m. 7

Chapter_14.indd 341 1/30/2016 4:27:47 PM


15 Alligation or Mixture

INTRODUCTION
Quantity of cheaper quality d −m
Alligation literally means “linking”.Itisaruletofind: =
Quantity of superior quality m−c
(a)
) the ratio in which two or more ingredients at their
respectivepricesshouldbemixedtogiveamixtureat
a given price. Explanation
(b) The mean or average price of a mixture when the Suppose x kg of cheaper quality is mixed with y kg of
pricesoftwoormoreingredientswhichmaybemixed superior quality.
togetherandtheproportioninwhichtheyaremixedare  Priceofcheaperingredient=`cx
given.  Priceofsuperioringredient=`dy
 Herecostpriceofaunitquantityofmixtureiscalled
\Priceofmixture=`(cx + dy)
price.
the mean price
 andquantityofmixture=(x + y ) kg.
Alligation Rule  cx + dy 
\Priceofmixture/kg=`  
Suppose `dperunitbethepriceoffirstingredient  x+ y 
(superior quality) mixed with another ingredient cx + dy
(cheaper quality) of price `c per unit to form a \ = m ⇒ cx + dy = mx + my
x+ y
mixturewhosemeanpriceis`m per unit, then the ⇒ dy – my = mx – cx
twoingredientsmustbemixedintheratio:
Quantity of cheaper ⇒ y (d – m ) = x(m – c )
Quantity of superior x d −m
⇒ = .
y m−c
C. P. superior − Mean price
= Illustration 1 In what ratio two varieties of tea one costing
Mean price − C. P. of cheaper
`27 per kg and the other costing `32 per kg should be
i.e.thetwoingredientsaretobemixedintheinverse blendedtoproduceablendedvarietyofteaworth`30 per
ratioofthedifferencesoftheirpricesandthemeanprice. kg.Howmuchshouldbethequantityofsecondvarietyof
Theaboverulemayberepresentedschematicallyas tea,ifthefirstvarietyis60kg.
under : Solution:

Chapter_15.indd 342 1/30/2016 4:34:18 PM


 AlligationorMixture 343

  Therequiredratioofthetwovarietiesofteais2:3,
i.e.
Quantity of cheaper tea 2
=
Quantity of superior tea 3

60 × 3
\ Quantity of superior tea = = 90 kg
2
 Thus,thesecondvarietyofteais90kg.

Illustration 2 Sugar at `15perkgismixedwithsugarat Quantity of cheaper sugar 20 − x


`20 per kg in the ratio 2:3. Find the price per kg of the =
Quantity of dearer sugar x − 15
mixture.
20 − x 2
Solution: Letthemeanpriceofthemixturebe`x \ = ⇒60–3x = 2x – 30
x − 15 3
⇒ 5x = 90 or x = 18.
Thus,thepriceperkgofthemixtureis`18.

SOME USEFUL METHODS

n
1. A vessel, full of wine, contains ‘a’ litres of which Wine left after nth operation  b
\ = 1 −  .
‘b’litresarewithdrawn.Thevesselisthenfilled Original quantity of wine  a
with water. Next, ‘b’ litres of the mixture are
withdrawn and again the vessel is filled with Illustration 3 A vessel contains 125 litres of wine. 25 litres
water.Thisprocessisrepeatedn times. Then, ofwinewastakenoutofthevesselandreplacedbywater.
Wine left in the vessel after nth operation Then,25litresofmixturewaswithdrawnandagainreplaced
Original quantity of wine in the vessel bywater.Theoperationwasrepeatedforthirdtime.How
much wine is now left in the vessel?
n
 a −b 
=   . Solution: Amount of wine left in the vessel
 a 
3
 25  100 × 100 × 100 × 125
= 1 −  × 125 = =64litres.
Explanation  125  125 × 125 × 125
Amount of wine after 1st operation
 b
= a – b = 1 −  × a 2. There are nvesselsofequalsizefilledwithmixtures
 a of liquids A and B in the ratio a1:b1:a2, b2...,
Ratioofwineandwaterafter1stoperationis(a – b):b. an:bn, respectively. If the contents of all the
\ In ‘b’ ’ litres of mixture withdrawn in 2nd operation, vesselsarepouredintoasinglelargevessel,then
amountofwinewithdrawn
a −b b Quantity of liquid A
= × b = (a – b)
( a − b) + b a Quantity of liquid B
\ Amountofwineleftafter2ndoperation
2  a1
b  b a2 an 
= (a – b) – (a – b) × = a 1 −   + + ... + 
a  a a + b a2 + b2 an + bn 
=  1 1
In general, quantity of wine left after nth operation  b1 b2 bn 
n  + + ... + 
 b
= 1 −  a  a1 + b1 a2 + b2 an + bn 
 a

Chapter_15.indd 343 1/30/2016 4:34:21 PM


344 Chapter 15

Explanation Solution:
Letthecapacityofeachvesselbec litres. 3 5 9
+ +
 AmountofliquidAindifferentvessels Quantity of milk 3 + 1 5 + 3 9 + 7
=
Quantity of water 1 3 7
a1c a2c a3c an c + +
= , , , ..., 3 +1 5 + 3 9 + 7
a1 + b1 a2 + b2 a3 + b3 an + bn
31/16
 AmountofliquidBindifferentvessels = =31:17.
17/17
b1c b2c b3c bn c
= , , , ...
a1 + b1 a2 + b2 a3 + b3 an + bn 3. There are n vessels of sizes c1, c2, ... , cnfilledwith
mixturesofliquidsAandB in the ratio a1:b1, a2:
 So,intheresultingmixture,amountofliquidA
b2, ..., an:bn, respectively. If the contents of all the
 a a2 an  vesselsarepouredintoasinglelargevessel,then
=  1 + + ... +  ×c
 a1 + b1 a2 + b2 an + bn  a1c1 ac a c
+ 2 2 + ... + n n
 AmountofliquidB Quantity of Liquid A a1 + b1 a2 + b2 an + bn
=
Quantity of liquid B b1c1 bc bc
 b b2 bn  + 2 2 + ... + n n
=  1 + + ... +  ×c a1 + b1 a2 + b2 an + bn
 a1 + b1 a2 + b2 an + bn 

Quantity of liquid A Illustration 5 Three glasses of sizes 3 litres, 4 litres and
Quantity of liquid B 5litrescontainmixtureofmilkandwaterintheratio2:3,
3:7 and 4:11, respectively. The contents of all the three
 a1 a2 an  glassesarepouredintoasinglevessel.Findtheratioofmilk
 + + ... + 
a + b a + b an + bn 
towaterintheresultingmixture.
=  1 1 2 2
.
 b1 b2 bn  Solution:
 + + ... +   2 × 3 3× 4 4 × 5 
+ +
 a1 + b1 a2 + b2 an + bn  Quantity of milk 
 2 + 3 3 + 7 4 + 11 

=
Illustration 4Threeequalglassesarefilledwithmixtureof Quantity of water  3 × 3 7 × 4 11× 5 
 + + 
milkandwater.Theproportionofmilkandwaterineach  2 + 3 3 + 7 4 + 11 
glassisasfollows:Inthefirstglassas3:1,inthesecond 6 12 20
glass as 5:3 and in the third as 9:7. The contents of the + +
threeglassesareemptiedintoasinglevessel.Whatisthe = 5 10 15 =56:124or,14:31.
9 28 55
proportionofmilkandwaterinit? + +
5 10 15

MULTIPLE CHOICE QUESTIONS

1. Tea worth `126perkgand`135perkgaremixedwith 3. Inamixtureof45litres,theratioofmilkandwateris3:2.


athirdvarietyintheratio1:1:2.Ifthemixtureisworth Howmuchwatermustbeaddedtomaketheratio9:11?
`153perkg,thenthepriceofthethirdvarietyperkgwill (a) 10 litres (b) 15 litres
be
(c) 17 litres (d) 20 litres
(a) `169.50 (d) `170 [Based on MAT, 2002]
(c) `175.50 (d) `180
4. Twojarscontainmilkandwaterintheratio5:4and2:1,
[Based on MAT, 2001]
respectively.Whatvolumeshouldbetakenoutfromthe
2. Ifthecostpriceofwaterisone-tenththatofmilkandthe firstjarifvolumeshavetobetakenoutfrombothjarsso
milkmanclaimstosellthemilkataprofitof20%,then astofillupathird30litresjarwithmilktowaterinthe
whatishisactualnetprofitpercentage? ratio1:1?
(a) 72% (b)82% (a) 7.5 litres (b) 15 litres
(c) 79% (d)68% (c) 22.5 litres (d)Itisimpossible

Chapter_15.indd 344 1/30/2016 4:34:24 PM


 Alligation or Mixture  345

5.
An empty container is filled with pure alcohol. The alcohol (a) 13:7 (b) 7:17
is slowly allowed to run out and when the container (c) 15:17 (d) 17:15
is one-fourth empty, it is replaced with water. Next,
[Based on IIT Joint Man. Ent. Test, 2004]
when the container is half empty it is again filled with
water. Finally, when it is three-fourths empty, it is again Directions (Questions 6 and 7):  Refer to the data below and
filled with water. What percentage of container is alcohol answer the questions that follow. Selling price of milk depends
now? on the fat content of milk. Normal fat content is 60% and normal
1 3 selling price is `20/litre. If the fat content falls below 55% then
(a) 8 % (b) 11 %
2 4 the selling price decreases by 20%. A milkman procures milk at
3 3 `16/litre.
(c) 9 % (d) 14 %
8 8 12. How much water as per cent of pure milk can be added so
6. A dairy man pays `6.4 per litre of milk. He adds water and that selling price is not affected?
sells the mixture at `8 per litre, thereby making 37.5% (a) 9% (b) 12.375%
profit. Find the proportion of the water to that of the milk (c) 10% (d) 15%
received by the customers.
(a) 1:15 (b) 1:10 13. What per cent of the water should be added to pure milk
so that even if selling price decreases the profit percentage
(c) 1:20 (d) 1:12
is not affected?
[Based on MAT, 2003]
(a) 10% (b) 30%
7. Mr X mixed 10 kg of variety A rice with 15 kg of variety
(c) 20% (d) 25%
B rice and sold the mixture at a price 40% more than that
of A. He did not get any profit. What is the ratio of the cost 14. Milk sold by a milkman contains 5% water. What quantity
price of variety A to that of B per kg? of pure milk should be added to 20 litres so that water
(a) 2:5 (b) 3:5 content comes down to 2%?
(c) 4:5 (d) 5:8 (a) 16 litres (b) 20 litres
8.
Several litres of acid were drawn off a 54-litre vessel full (c) 25 litres (d) None of these
of acid and an equal amount of water added. Again the [Based on IRMA, 2002]
same volume of the mixture was drawn off and replaced
by water. As a result, the vessel contained 24 litres of pure 15. There are 3 containers A, B and C which contain water,
acid. How much of the acid was drawn off initially? milk and acid respectively in equal quantities. 10% of the
content of A is taken out and poured into B. Then, the same
(a) 12 litres (b) 16 litres
amount from B is transferred to C, from which again the
(c) 18 litres (d) 24 litres same amount is transferred to A. What is the proportion of
[Based on MAT, 2001] milk in container A at the end of the process?
9.
If 50% of the 2:3 solution of milk and water is replaced (a) 9/10 (b) 1/11
with water, then the concentration of the solution is (c) 1/121 (d) 2/13
reduced by
[Based on FMS (Delhi), 2002]
(a) 25% (b) 33.33%
(c) 50% (d) 75% 16. Milk and water are in the ratio of 3:2 in a mixture of 80
litres. How much water should be added so that the ratio
[Based on MAT, 2001]
of the milk and water becomes 2:3?
10.
Two solutions of milk and water are kept in two vessels A (a) 25 litres (b) 40 litres
and B. The ratio of quantity of milk in vessels A and B is
4:5 while the ratio of quantity of water in vessels A and B (c) 35 litres (d) 20 litres
is 8:9. It is known that the concentration of milk in one of [Based on FMS (Delhi), 2002]
these vessels in between 60% and 80%. What could be the
17. A beaker contains a 105 ml mixture of alcohol and water.
concentration of milk in the other vessel?
three-sevenths of the mixture is alcohol; the remainder
(a) 55% (b) 65% is water. An additional quantity of 105 ml of alcohol is
(c) 75% (d) 85% poured into the mixture. What is the ratio of the volume
of alcohol to water in the new mixture?
11. Two vessels contain spirit of 0.5 and 0.75 concentrations.
If two litres from the first vessel and three litres from the (a) 3:7 (b) 3:4
second vessel are mixed, then what will be the ratio of the (c) 5:2 (d) 1:1
spirit and the water in the resultant solution?

Chapter_15.indd 345 1/30/2016 4:34:25 PM


346  Chapter 15

18. How much water must be mixed in 300 ml of sugar alcohol as in the other. The mixtures in the two containers
solution which contains 40% sugar by weight, such that it are then mixed and it is found that the ratio of water to
becomes a 30% sugar solution? alcohol is 3:2. Find the ratio of water to alcohol in each of
(a) 50 ml (b) 75 ml the original containers.
(c) 90 ml (d) 100 ml (a) 2:1, 3:4 (b) 1:3, 1:2
19. Two vessels A and B contain mixtures of spirit and water. (c) 2:3, 4:1 (d) None of the above
A mixture of 3 parts from A and 2 parts from B is found to
contain 29% of spirit and a mixture of 1 part from A and 9 26. An alloy contains only zinc and copper. One such alloy
parts from B is found to contain 34 per cent of spirit. Find weighing 15 gm contains zinc and copper in the ratio of
the percentage of spirit in B and A. 2:3 by weight. If 10 gm of zinc is added then find what
amount of copper has to be removed from the alloy such
(a) 35, 25 (b) 40, 20
that the final alloy has zinc and copper in the ratio of 1:4
(c) 25, 25 (d) 50, 50 by weight?
20. Three vessels whose capacities are as 5:3:2 are completely (a) 5 gm (b) 5.5 gm
filled with milk mixed with water. The ratio of milk to
(c) 6 gm (d) 4.8 gm
water in the mixture of vessels are as 3:2, 2:1 and 3:1
respectively. Find the percentage of water in the new 27. Three beakers namely, A, B and C each contain 100 ml
mixture obtained when one-third of first, half of second 1
and two-thirds of the third vessel is taken out and mixed of milk water solution. The ratio of milk to water in
m
together.
the beakers A, B and C is 1:3, 1:4 and 2:3 respectively.
(a) 66.66% (b) 50% 40 ml of solution is transferred from beaker A to beaker
(c) 16.66% (d) 33.33% C and than 28 ml of solution is transferred from beaker C
to beaker B. Find the final ratio of milk in the beakers A,
21. Three vessels contain equal mixtures of milk and water in
B and C.
the ratio 6:1, 5:2 and 3:1 respectively. If all the solutions
are mixed together, the ratio of milk to water in the final (a) 3:6:8 (b) 6:15:20
mixture will be
(c) 15:28:42 (d) None of these
(a) 64:65 (b) 65:64
(c) 19:65 (d) 65:19 28. Aqua regia is a mixture containing 50 per cent
concentrated HCL and 70% concentrated HNO3 in the
22. In what ratio must a person mix three kinds of wheat ratio 1:3, respectively. Aqua regia is to be formed with
costing him `1.20, `1.44 and `1.74 per kg, so that the 15 litres of HCL of 80% concentration and 56 litres of
mixture may be worth `1.41 per kg? HNO3 of 90% concentration by adding water as a diluting
(a) 15:77:11 (b) 7:11:15 agent. Another mixture ‘X’ having 40% concentrated HCL
(c) 11:77:15 (d) 11:77:7 and 30% concentrated H2SO4 in the ratio 5:7, respectively
is added to the Aqua regia to form a solution of 111 litres.
23. Two bottles A and B contain diluted sulphuric acid. In Find the ratio of HCL to water in the solution. Only water
bottle A, the amount of water is double the amount of is used as a diluting agent.
acid while in bottle B, the amount of acid is 3 times that
of water. How much mixture should be taken from each (a) 3:8 (b) 4:7
bottle in order to prepare 5 litres of diluted sulphuric acid (c) 1:3 (d) 2:5
containing equal amount of acid and water?
(a) 1, 4 (b) 3, 2 29. I added 1 litre of water to 5 litres of a 20 per cent solution
of alcohol and water. The strength of alcohol is
(c) 2, 3 (d) 4, 1
(a) unaltered (b) 5%
24. The ratio of kerosene to petrol in 100 kg of mixed petrol
normally used by three wheelers is 7:25. The amount of (c) 12.5% (d) 16.66%
kerosene to be added to 100 kg of mixed petrol to make
the ratio 9:25 is 30. Gold is 19 times as heavy as water and copper is 9 times
as heavy as water. In what ratio these two metals be mixed
(a) 5 kg (b) 6.625 kg so that the alloy is 15 times as heavy as water is
(c) 6.25 kg (d) 6.35 kg
(a) 2:1 (b) 3:2
25. Two equal containers are filled with a mixture of water
(c) 1:2 (d) 2:3
and alcohol. One of them contains three times as much

Chapter_15.indd 346 1/30/2016 4:34:25 PM


 Alligation or Mixture  347

31. 25% of a solution containing 20% petrol, 50% diesel and 38.
A solution of sugar syrup has 15% sugar. Another solution
30% kerosene was replaced with kerosene. Now, two- has 5% sugar. How many litres of the second solution
thirds of the solution obtained in the previous step was must be added to 20 litres of the first solution to make a
replaced with petrol. What is the percentage of diesel in solution of 10% sugar?
this new solution? (a) 10 (b) 5
(a) 15% (b) 15.83% (c) 15 (d) 20
(c) 12.5% (d) None of these 39.
How many litres of a 3% hydrogen peroxide solution
32. 2  l of Bournvita drink contains 236 calories and 2  l of should be mixed with 6 litres of a 30% hydrogen peroxide
complan drink contains 166 calories. If l mixture of both solution so as to get a 12% solution?
the drinks contains 88 calories, then the percentage of (a) 3 litres (b) 6 litres
Bournvita drink in the mixture is (c) 9 litres (d) 12 litres
(a) 25% (b) 16.66% 40.
How much water must be added to 100 cc of 80 per cent
(c) 14.28% (d) 83.33% solution of boric acid to reduce it to a 50 per cent solution?
(a) 30 (b) 40
33.
A vessel of capacity 2 litre has 25% alcohol and another
vessel of capacity 6 litre has 40% alcohol. The total liquid (c) 50 (d) 60
of 8 litre is poured out in a vessel of capacity 10 litre and 41. Milk and water are in the ratio of 3:2 in a mixture of 80
the rest part of the vessel is filled with water. What is the litres. How much water should be added so that the ratio
new concentration of mixture? of the milk and water becomes 2:3?
(a) 31% (b) 71% (a) 25 litres (b) 40 litres
(c) 49% (d) 29% (c) 35 litres (d) 20 litres
34.
One type of liquid contains 25% of milk, the other contains 42. There are 3 containers A, B and C which contain water,
30% of milk. A can is filled with 6 parts of the first liquid milk and acid respectively in equal quantities. 10% of the
and 4 parts of the second liquid. Find the percentage of content of A is taken out and poured into B. Then, the same
milk in the new mixture. amount from B is transferred to C, from which again the
(a) 28% (b) 25% same amount is transferred to A. What is the proportion of
milk in container A at the end of the process?
(c) 30% (d) 27%
(a) 9/10 (b) 1/11
35.
A jar contains a mixture of two liquids A and B in the ratio (c) 1/121 (d) 2/13
4:1. When 10 litres of the mixture is taken out and 10
litres of liquid B is poured into the jar, the ratio becomes 43. Milk sold by a milkman contains 5 per cent water. What
2:3. How many litres of liquid A was contained in the jar? quantity of pure milk should be added to 20 litres so that
water content comes down to 2%?
(a) 14 litres (b) 18 litres
(a) 16 litres (b) 20 litres
(c) 20 litres (d) 16 litres
(c) 25 litres (d) None of these
36.
A housewife has 11 litre of solution that contains milk and 44.
If 50 per cent of the 2:3 solution of milk and water is
water in the ratio 3:1. She adds 250 ml of 3:2 solution of replaced with water, then the concentration of the solution
milk and water to it and then uses 250 ml of the combined is reduced by
mixture to make curd How much of pure milk is she left
(a) 25% (b) 33.33%
with?
(c) 50% (d) 75%
(a) 1,000 ml (b) 912.5 ml
(c) 750 ml (d) 720 ml 45.
Several litres of acid were drawn off a 54-litre vessel full
of acid and an equal amount of water was added. Again the
37.
There are two alloys made up of copper and aluminium. same volume of the mixture was drawn off and replaced
In the first alloy copper is half as much as aluminium and by water. As a result, the vessel contained 24 litres of pure
in the second alloy copper is thrice as much as aluminium. acid. How much of the acid was drawn off initially?
How many times the second alloy must be mixed with (a) 12 litres (b) 16 litres
first alloy to get the new alloy in which copper is twice as (c) 18 litres (d) 24 litres
much as aluminium?
(a) 2 (b) 3 46. A dairyman pays `8.0 per litre of milk. He adds water
and sells the mixture at `9 per litre, thereby making 40%
(c) 4 (d) 5 profit. The proportion of water to milk received by the
customers is

Chapter_15.indd 347 1/30/2016 4:34:25 PM


348  Chapter 15

(a) 45:11 (b) 11:45 amount of the mixture as before is released and replaced
(c) 2:45 (d) 45:2 by nitrogen for the second time. As result, the oxygen
content becomes 9% of total volume. How many litres of
47. A dairy man pays `6.4 per litre of milk. He adds water and
mixture is released each time?
sells the mixture at `8 per litre, thereby making 37.5%
profit. Find the proportion of the water to that of the milk (a) 7 litres (b) 40 litres
received by the customers. (c) 2 litres (d) None of these
(a) 1:15 (b) 1:10 [Based on MAT (Sept), 2009, 2008 (Dec), 2005]

(c) 1:20 (d) 1:12 55.


Two gallons of a mixture of spirit and water contain 12%
of water. They are added to 3 gallons of another mixture,
48. In a mixture of 60 litres, the ratio of milk to water is 2:1. If containing 7% of water and half a gallon of water is then
this ratio is to be 1:2, then the quantity of water (in litres) added to the whole. Find the percentage of water in the
to be further added is resulting mixture.
(a) 20 (b) 30 3 12
(c) 40 (d) 60 (a) 17 % (b) 16 %
11 11
49. Mira’s expenditure and savings are in the ratio 3 : 2. Her 1
income increases by 10 per cent. Her expenditure also (c) 14 % (d) None of these
11
increases by 12 per cent. By how much per cent does her [Based on MAT (Sept), 2009]
saving increase?
56.
A jar full of milk contains 40% water. A part of this milk
(a) 7% (b) 10%
is replaced by another containing 19% water and now the
(c) 9% (d) 13% percentage of water is found to be 26%. The quantity of
50. A trader has 50 kg of rice, a part of which he sells at 10 per milk replaced is
cent profit and the rest at 5 per cent loss. He gains 7 per (a) 2/3 (b) 1/3
cent on the whole. What is the quantity sold at 10 per cent (c) 3/7 (d) 4/7
gain and 5 per cent loss? [Based on MAT (Feb), 2009]
(a) 30 kg, 10 kg (b) 40 kg, 15 kg
57.
One litre of water is evaporated from 6 litres of a solution
(c) 35 kg, 40 kg (d) 40 kg, 10 kg containing 5% salt. Find the percentage of salt in the
51.
The wheat sold by a grocer contained 10% low quality remaining solution.
wheat. What quantity of good quantity wheat should be (a) 8% (b) 10%
added to 150 kg of wheat so that the percentage of low (c) 6% (d) 4%
quality wheat becomes 5%? [Based on MAT (Feb), 2009]
(a) 85 kg (b) 50 kg
58.
When processing flower nectar into honey, a considerable
(c) 135 kg (d) 150 kg amount of water is added. How much flower nectar must
[Based on MAT (Feb), 2011] be processed to yield 1 kg of honey, if nectar contains 70%
52.
One type of liquid contains 25% of milk, the other contains water and the honey obtained from this nectar contains
30% of milk. A container is filled with 6 parts of the first 17% water?
liquid and 4 parts of the second liquid. The percentage of (a) 4.11 kg (b) 1.76 kg
milk in the mixture is (c) 0.36 kg (d) 2.77 kg
(a) 27% (b) 31% [Based on MAT (Sept), 2008]
(c) 29% (d) 33% 59.
The amount of water (in mL) that should be added to
[Based on MAT (Dec), 2009 (Sept), 2009] reduce 9 mL lotion, containing 50% alcohol, to a lotion
53.
A solution of sugar syrup has 15% sugar. Another solution containing 30% alcohol, is
has 5% sugar. How many litres of the second solution (a) 5 mL (b) 4 mL
must be added to 20 litres of the first solution to make a (c) 3 mL (d) 6 mL
solution of 10% sugar? [Based on MAT (May), 2007]
(a) 10 litres (b) 5 litres 60.
The ratio of milk to water in three containers of equal
(c) 15 litres (d) 20 litres capacity is 3:2, 7:3 and 11:4 respectively. The three
[Based on MAT (Sept), 2009] containers are mixed together. What is the ratio of milk to
54.
An 8-litre cylinder contains a mixture of oxygen and water after mixing?
nitrogen, the volume of oxygen being 16% of total (a) 38:8 (b) 21:9
volume. A few litres of the mixture is released and an (c) 61:29 (d) 41:18
equal amount of nitrogen is added. Then, the same [Based on MAT (May), 2010]

Chapter_15.indd 348 1/30/2016 4:34:26 PM


 Alligation or Mixture  349

61.
There are 2 bottles containing a mixture of wine, water and 68.
A vessel contains liquid P and Q in the ratio 5:3. If 16
alcohol. The first bottle contains wine, water and alcohol litres of the mixture are removed and the same quantity
in the ratio 3:5:2. The second bottle contains water and of liquid Q is added, the ratio become 3:5. What quantity
wine in the ratio 5:4. 1 litre of the first and 2 litres of the does the vessel hold?
second are mixed together. What fraction of the mixture is (a) 35 litres (b) 45 litres
alcohol? (c) 40 litres (d) 50 litres
(a) 1/15 litres (b) 6/13 litres [Based on MAT (Dec), 2007]
(c) 2/15 litres (d) 6/19 litres 69.
A bottle is full of dettol. One-third of it is taken out and
[Based on MAT (May), 2010] then an equal amount of water is poured into the bottle
62.
Two vessels contain spirit of 0.5 and 0.75 concentrations. to fill it. This operation is done four times. Find the final
If 2 litres from the first vessel and 3 litres from the second ratio of dettol and water in the bottle.
vessel are mixed, then what will be the ratio of the spirit (a) 13:55 (b) 20:74
and the water in the resultant solution? (c) 16:65 (d) 10:48
(a) 15:17 (b) 7:17 [Based on MAT (Dec), 2007]
(c) 13:7 (d) 17:15 70.
A person has a chemical of `25 per litre. In what ratio
[Based on MAT (Dec), 2009 (Feb), 2008] should water be mixed in that chemical, so that after
63.
In a mixture of 60 litres, the ratio of milk and water is 2:1. selling the mixture at `20 per litre he may get a profit of
If the ratio of milk and water is to be 1:2, then the amount 25%?
of water to be further added must be (a) 13:16 (b) 12:15
(a) 40 litres (b) 30 litres (c) 16:9 (d) 19:22
[Based on MAT (Dec), 2007]
(c) 20 litres (d) 60 litres
[Based on MAT (Dec), 2009] 71.
How many kg of sugar costing `5.75 per kg should be
mixed with 75 kg of cheaper sugar costing `4.50 per kg
64.
One test tube contains some acid and another test tube
so that the mixture is worth `5.50 per kg?
contains an equal quantity of water. To prepare a solution,
20 g of the acid is poured into the second test tube. Then, (a) 350 kg (b) 300 kg
two-thirds of the so-formed solution is poured from the (c) 250 kg (d) 325 kg
second tube into the first. If the fluid in the first test tube is [Based on MAT (May), 2007]
four times that in the second, what quantity of water was 72.
5 litres of water is added to a certain quantity of pure milk
taken initially? costing `3/litres. If by selling the mixture at the same
(a) 80 g (b) 60 g price as before, a profit of 20% is made, then what is the
(c) 40 g (d) None of these amount of pure milk in the mixture?
[Based on MAT (May), 2009] (a) 20 litres (b) 30 litres
65. A trader has 50 kg of pulses, part of which he sells at 8 per (c) 25 litres (d) 35 litres
[Based on MAT (May), 2006]
cent profit and the rest at 18 per cent profit. He gains 14
per cent on the whole. What is the quantity sold at 18 per 73.
A sink contains exactly 12 litres of water. If water is
cent profit? drained from the sink until it holds exactly 6 litres of
(a) 30 kg (b) 25 kg water less than the quantity drained away, how many litres
(c) 20 kg (d) 40 kg of water were drained away?
(a) 2 litres (b) 6 litres
66. A bottle contains three-fourths of milk and the rest
water. How much of the mixture must be taken away and (c) 3 litres (d) 9 litres
replaced by an equal quantity of water so that the mixture [Based on MAT (Feb), 2006]
has half milk and half water? 74.
A chemist has 10 litres of a solution that is 10% nitric
1 acid by volume. He wants to dilute the solution to 4%
(a) 25% (b) 33 % strength by adding water. How many litres of water must
3
be added?
(c) 45% (d) 50%
[Based on MAT (Dec), 2008] (a) 15 (b) 20
(c) 18 (d) 25
67.
A mixture of 45 litres of spirit and water contains 20% of [Based on MAT (Feb), 2006]
water in it. How much water must be added to it make the
water 25% in the new mixture? 75.
Suppose, that a maximum of 25 g of salt dissolves in 100 g
of water. Any more salt, if added, remains undissolved and
(a) 5 litres (b) 3 litres
a sediment falls at the bottom. Now water is evaporated
(c) 4 litres (d) 6 litres
from 1 kg of a 4% solution at the rate of 28% g/h. After
[Based on MAT (Dec), 2008] how long will it start sedimenting?

Chapter_15.indd 349 1/30/2016 4:34:26 PM


350  Chapter 15

(a) 35 hrs (approx) (b) 29 hrs (approx) km/hr. In the second part of the journey, he travels by train
(c) 31 hrs (approx) (d) 23 hrs (approx) at the speed of 55 km/hr. How much distance did he travel
[Based on MAT (Sept), 2008] by train?
76.
An alloy contains copper and zinc in the ratio 5:3 and (a) 205 km (b) 145 km
another alloy contains copper and tin in the ratio 8:5. If (c) 165 km (d) 185 km
equal weights of both the alloys are melted together, then 84. A person has a chemical of `25 per litre. In what ratio
the weight of tin in the resulting alloy per kg will be should water be mixed in that chemical so that after
(a) 26/5 (b) 5/26 selling the mixture at `20 per litre he may get a profit of
(c) 7/31 (d) 31/7 25 per cent?
[Based on MAT, 1997] (a) 13:16 (b) 16:9
77.
Alok bought 25 kg of rice at the rate of `6 per kg and 35 (c) 12:15 (d) 19:22
kg of rice at the rate of `7 per kg. He mixed the two and 85. 300 g of salt solution has 40% salt in it. How much salt
sold the mixture at the rate of `6.75 per kg. What was his should be added to make it 50% in the solution?
gain or loss in the transaction? (a) 40 g (b) 60 g
(a) `16 gain (b) `16 loss (c) 70 g (d) 80 g
(c) `20 gain (d) `10 gain [Based on SNAP, 2010]
[Based on MAT, 2000] 86.
A mixture contains milk and water in the ratio 5:1 or
78.
How much water must be added to 100 cc of 80% solution adding 5 litres of water, the ratio of milk to water becomes
of boric acid to reduce it to a 50% solu­tion? 5:2. The quantity of milk in the original mixture is
(a) 30 (b) 40 (a) 16 litres (b) 25 litres
(c) 50 (d) 60 (c) 22.75 litres (d) 32.5 litres
[Based on MAT, 2000] [Based on FMS (MS), 2006]

79.
A bought 4 bottles of beer and B bought one bottle of lager. 87.
A mixture contains milk and water in the ratio 5:1. On
Lager per bottle costing twice that of the beer. C bought adding 5 litres of water, the ratio of milk and water
nothing but paid `50 for his share of the drink which they becomes 5:2. The quantity of milk in the original mixture
mixed together and shared equally. If C’s `50 covered his is
share, then what is the cost of the lager? (a) 16 litres (b) 22.75 litres
(a) 50 (b) 75 (c) 25 litres (d) 32.5 litres
(c) 30 (d) 46 [Based on FMS, 2005]
[Based on MAT, 2000] 88.
An alloy contains zinc and copper in the ratio 5:8 and
another alloy contains zinc and copper in the ratio 5:3. If
80.
An alloy of gold and silver weighs 50 g. It contains 80%
equal amounts of both the alloys are melted together, then
gold. How much gold should be added to the alloy so that
the ratio of zinc and copper in the resulting alloy is
percentage of gold is increased to 90?
(a) 25:24 (b) 3:8
(a) 50 g (b) 60 g
(c) 103:105 (d) 105:103
(c) 30 g (d) 40 g
[Based on FMS, 2006]
[Based on SNAP, 2009]
89.
Two vessels A and B contain milk and water mixed in the
81.
How many kg of tea worth `25 per kg must be blended
ratio 5:3 and 2:3. When these mixtures are mixed to form
with 30 kg of tea worth `30 per kg so that by selling the
a new mixture containing half milk and half water, they
blended variety at `30 per kg there should be a gain of
must be taken in the ratio
10%?
(a) 2:5 (b) 3:5
(a) 36 kg (b) 40 kg
(c) 4:5 (d) 7:3
(c) 32 kg (d) 42 kg
[Based on FMS, 2006]
[Based on SNAP, 2009]
90. The average weekly salary per head of all employees
82.
300 gm of salt solution has 40% salt in it. How much salt (supervisors and labourers) is `100. The average weekly
should be added to make it 50% in the solution? salary per head of all the supervisors is `600 while the
(a) 40 gm (b) 60 gm average weekly salary per head of all the labourers is `75.
(c) 70 gm (d) 80 gm Find the number of supervisors in the factory if there are
[Based on SNAP, 2009] 840 labourers in it.
83.
A person travels 285 km in 6 h in two stages. In the first (a) 46 (b) 42
part of the journey, he travels by bus at the speed of 40 (c) 44 (d) 48

Chapter_15.indd 350 1/30/2016 4:34:26 PM


 Alligation or Mixture  351

91.
Two alloys contain zinc and copper in the ratio of 2:1 and (a) 75% (b) 88%
4:1. In what ratio the two alloys should be added together (c) 90% (d) None of these
to get as new alloy having zinc and copper in the ratio of [Based on Indian Overseas Bank PO, 2009]
3:1?
98.
The ratio of the quantities of an acid and water in a mixture
(a) 7:5 (b) 5:7 is 1:3. If 5 litres of acid is further added to the mixture, the
(c) 3:5 (d) None of these new ratio becomes 1:2. The quantity of new mixture in
[Based on IIFT, 2005] litres is
92.
Three equal glasses are filled with mixtures of spirit and (a) 32 (b) 40
water. The ratio of the spirit to water is as follows: in the (c) 42 (d) 45
first glass as 3:4, in the second glass as 4:5 and in the third [Based on SSC (GL), 2011]
glass as 5:6. The contents of the three glasses are emptied 99.
An alloy contains copper, zinc and nickel in the ratio of
into a single vessel. What is the ratio of the spirit to water 5:3:2. The quantity of nickel in kg that must be added to
in the mixture now? 100 kg of this alloy to have the new ratio 5:3:3 is
(a) 920:1159 (b) 820:1149 (a) 8 (b) 10
(c) 1120:1134 (d) 1010:1122 (c) 12 (d) 15
[Based on IIFT, 2005] [Based on SSC (GL), 2011]
93.
The ratio of milk and water in mixtures of four containers 100. There are two vessels of equal capacity, one full of milk,
are 5:3, 2:1, 3:2 and 7:4, respectively. In which container and the second one-third full of water. The second vessel
is the quantity of milk, relative to water, minimum? is, then filled up out of the first, the contents of the second
(a) First (b) Second are then poured back into the first till it is full and then
(c) Third (d) Fourth again the contents of the first are poured back into the
[Based on SSC (GL), 2010]
second till it is full. What is the proportion of milk in the
second vessel, if capacity of the vessel is 20 litres?
94.
Ashok a master adulterator cum grosser sells haldi powder
37 20
(turmeric powder), which contains five per cent saw dust. (a) (b)
What quantity of pure haldi should be added to two kilos 20 27
of haldi (containing five per cent saw dust) so that the 20 27
proportion of saw dust becomes four per cent? (c) (d)
37 20
(a) 1 kg (b) 2 kg [Based on NMAT, 2005]
(c) 0.5 kg (d) None of these 101.
In three vessels, the ratio of water and milk is 6:7, 5:9
[Based on IIFT, 2007] and 8:7, respectively. If the mixture of the three vessels is
95.
A can contains a mixture of two liquids A and B in the mixed, then what will be the ratio of water and milk?
ratio 7:5. When 9 litres of mixture are drawn of and the (a) 2431:3781 (b) 3691:4499
can is filled with B, the ratio of A and B becomes 7:9. (c) 4381:5469 (d) None of these
Litres of liquid A contained by the can initially was [Based on U.P. P.C.S., 2012]
(a) 10 (b) 20 102.
In a glass of milk, the proportion of pure milk and water
(c) 21 (d) 25 is 3:1, how much of the mixture must be withdrawn and
[Based on SSC (GL), 2011] substituted by water so that the resulting mixture may
96.
A manufacturer has 200 litres of acid solution which become half pure milk and half water?
has 15% acid content. How many litres of solution with 1 1
(a) unit (b) unit
30% acid content may be added so that acid content in 4 3
the resulting mixture will be more than 20% but less than
3 1
25% ? (c) unit (d) unit
4 2
(a) More than 100 litres but less than 300 litres
[Based on ATMA, 2006]
(b) More than 120 litres but less than 400 litres
103. Two barrels contain a mixture of ethanol and gasoline.
(c) More than 100 litres but less than 400 litres The content of ethanol is 60% in the first barrel and 30%
(d) More than 120 litres but less than 300 litres in the second barrel. In what ratio must the mixtures from
[Based on XAT, 2010] the first and the second barrels be taken to form a mixture
97. In a mixture of milk and water the proportion of water by containing 50% ethanol?
weight was 75%. If in the 60 gm mixture, 15 gm water was (a) 1:2 (b) 2:1
added, what would be the percentage of water? (Weight (c) 2:3 (d) 3:2
in gm) [Based on JMET, 2006]

Chapter_15.indd 351 1/30/2016 4:34:28 PM


352  Chapter 15

EXPLANATORY ANSWERS

1. (c) Suppose the quantities of Tea worth `126 per kg, S.P. × 100
`135 per kg and `x per kg purchased are y, y and \ C.P. =
100 + Gain%
2y kg respectively.
126 y + 135 y + 2 xy (8 x + 8 y ) × 100
\ = 153 ⇒ 6.4x =
4y 100 + 37.5

⇒ 261 + 2x = 612 ⇒ 880x = 800x + 800y
351 ⇒ 80x = 800y ⇒ x = 10y
⇒ x = = 175.50
2 x y
⇒ =
\ Tea of the third variety is purchased @ `175.50 10 1
per kg. \ Required ratio = 1:10.
1 7. (b) Let `X, `Y be the cost price per kg of variety A and
2. (a) Since x + y = , he mixes milk and water in the ratio
2 veriety B of rice
2:1.
By data 10X + 15Y = 25 (1.4) X.
Let us say, in 100 l milk, he mixes 50 l water.
Since sale price of the mixture is 40% more than the
C.P. of milk = 10x per 100 l
cost price of A.
x
C.P. of water = x per 100 l, i.e., per 50 l Therefore 10X + 15Y = 35X
2
⇒ 15Y = 25X
S.P. of milk = (1.2) × 15x = 18x
Hence, X:Y = 15:25 = 3:5
(18 x  10.5 x)
So, profit percentage = × 100
10.5 x 8. (c) Suppose x litres of acid was drawn off initially.
= 72% (approximately) \ Remaining acid in the 54-litre vessel full of acid.
   = (54 – x) litres and water added = x litres.
3. (b) Let M = 3K, W = 2K
Now out of the 54 litres of mixture of acid and water,
\ 3K + 2K = 45 ⇒ K = 9 x litres of mixture is drawn off.
⇒ Milk = 27 litres and Water = 18 litres  54 − x 
⇒ Quantity of acid drawn off =  × x  litres
Now suppose x litres of water is added to the mixture  54 
such that
x2
27 9 and quantity of water drawn off = litres.
= ⇒ 162 + 9x = 297 54
18 + x 11
⇒ 9x = 135 ⇒ x = 15.   54 − x  
Now the vessel contains 54 − x −   x  litres
  54  
4.
(d) In both jars concentration of milk is more than 50%. of acid.
Therefore, in jar three concentration of milk cannot be  54 − x 
50%. Hence, we cannot decide the volumes. \ 54 − x −   x = 24
 54 
5.
(c) Ratio of quantity of alcohol left to total quantity ⇒ x2 – 108x + 1620 = 0
3 1 1 ⇒ x = 90, 18.
1 4 2 4 3
=     Since 90 > 54, therefore x = 90 is ruled out.
1 1 1 1 32 Hence x =18.
3 3 9.
(c) Milk : Water
Alcohol percentage =  100 
9 %
32 8 20 30
6. (b) Suppose the quantity of milk purchased = x litres –10 –15
Suppose quantity of water mixed = y litres +25
10 40
\ Required ratio of the water and the milk in the
mixture = y:x   Therefore, the concentration of the solution is
reduced from 20 to 10, i.e. reduced by 50%. The
C.P. of x litres of milk = `6.4 x
dilution of the solution increased from 30 to 40, i.e.
S.P. of x litres of milk = `8 (x + y) 1
Profit % = 37.5 increased by 33 %.
3

Chapter_15.indd 352 1/30/2016 6:00:39 PM


 Alligation or Mixture  353

10.
(b,c) Milk and water in A = 4x and 8y `16 if profit % is 25%
Milk and water in B = 5x and 9y 16
\ cost price = `12.80
   If concentration of milk in A is between 60% 1.25
and 80%, then ratio of milk and water would be 60% fat is available in 1 liter at `16.
between 60:40 and 80 : 20 Cost price of the mixture should be `12.80.
60 4 x 80 By alligation,
⇒         
40 8 y 20 \ Water as % of pure milk = 25%
60 8 5 5 x 80 8 5 14.
(d) Total milk = 20 litres
⇒         
40 4 9 9 y 20 4 9 Water Milk Total
5 5 x 40 1 litre 19 litres 20
⇒          
3 9y 9 1 19 + x 20 + x
⇒ Concentration of milk in B is between 62.5% and Now 1 = 2% of (20 + x), where x is the quantity of
81% If concentration of milk in B is between 60% pure milk added to the mixture.
and 80%, then ratio of milk and water would be ⇒ x = 30
between 60:40 and 80:20. where x = quantity of pure milk added to the mixture.
60 5 x 80
⇒          15.
(c) Suppose there is 100 cc of liquid in each container
40 9 y 20
initially.
60 9 4 4 x 80 9 4 I Step: Container B has 10 cc water in 110 cc of
⇒         
40 5 8 8 y 20 5 8 mixture.
27 4 x 18 II Step: 10 cc of this mixture contains.
⇒          
20 8 y 5 100 100 10
10 × =  cc of milk and  cc of water,
⇒ Concentration of milk in A is between 57.44% and 100 11 11
78.26% which is transferred to container C.
11. (a) From vessel A (0.5 concentration), 1 litre of spirit 100
Container C has 100 cc of acid,  cc of milk and
and 1 litre of water is taken and from vessel B (0.75 11
concentration), 2.25 litres of spirit and 0.75 litre of 10
 cc of water.
water is taken. 11
\ Ratio of spirit to water in the resulting mixture Total quantity of mixture in container C = 110 cc.
1 + 2.25 3.25 13 III Step: 10 cc of this mixture is transferred to
= = = .
1 + 0.75 1.75 7 A which contains.
12.
(a) If selling price is not to be affected then fat content of 100 100
× 10 cc of milk =  cc of milk
the mixture should be 55%. Fat content of milk is 60 1210 121
and of water is 0.
100
By alligation Thus, in 100 cc of mixture, there is  cc of milk of
121
1
the th proportion of the mixture.
121
16.
(b) Quantity of milk in the mixture = 48 litres.
Quantity of water in the mixture = 32 litres
Let x litres of water be added in the mixture so that
48 2
1 = ⇒ 2x + 64 = 144 ⇒ x = 40.
i.e., 11:1. Water as % of milk =  100 
9% 32 + x 3
11
3
13. (d) Current selling price = `20 17.
(c) The amount of alcohol in the old mixture is × 105
7
Procurement price = `16 = 45 ml. Further 105 ml alcohol is added.
Profit % = 25    Therefore, total alcohol in new mixture is 150 ml.
Decrease in selling price = 20% Water content is (105 – 45) = 60 ml. Alcohol to water
\ Selling price if fat content is less than 55% ratio is 150:60 or 5:2

Chapter_15.indd 353 1/30/2016 6:01:29 PM


354  Chapter 15

18.
(d) Total quantity = 300 ml
Sugar by weight = 120 ml
Water = 180 ml
Let x be the amount of water added to the solution.
120 30
Then,  =
300  x 100
⇒ 1200 = 900 + 3x ⇒ 300 = 3x ⇒ x = 100 ml
By alligation rule:
19.
(a) Let X% be the percentage of spirit in A and Y% in B.
(Quantity of 1st kind of wheat) 33 11
3X 2Y = =
 = 29% of (3 + 2) (Quantity of 3rd kind of wheat) 21 7
100 100 i.e., they must be mixed in the ratio 11:7
3X 2Y Step II: Mix wheat of 1st kind and 2nd kind to obtain
 = 1.45 ⇒ 3x + 2Y = 145 ...(1)
100 100 a mixture worth of `1.41 per kg.
X 9Y 34
 =  10 = X + 9Y = 340 ...(2)
100 100 100
Solving (1) and (2), we get X = 25 and Y = 35
A contains 25% spirit and B contains 35% of spirit.
5X 3X 4 X
20.
(d) The quantity of new mixture =  
3 2 3

27 X 9X
= = \ By alligation rule:
6 2
(Quantity of 1st kind of wheat) 3 1
5X 3 3X 2 3X 2 = =
Percentage of milk =
     (Quantity of 2nd kind of wheat) 21 7
3 5 2 3 2 3
i.e., they must be mixed in the ratio 1:7
= 3X
5 X 2 3 X 1 1X 2
Percentage of water =      (Quantity of 2nd kind of wheat)
3 5 2 3 2 3 Thus,
(Quantity of 3rd kind of wheat)
3X
= (Quantity of 2nd kind of wheat)
2    =
(Quantity of 3rd kind of wheat)
3X / 2
Percentage of water =  100 (Quantity of 1st kind of wheat)
9X / 2 
(Quantity of 3rd kind of wheat)
100 1
= = 33 %  7 11 11
3 3    =    = .
1 7  1
21.
(d) Given that all vessels contain equal amount of mixture \ Quantities of wheat of (1st kind:2nd kind:3rd kind)
say V.
 7
So in the first vessel → water:milk = 1/7:6/7    = 1: 7 :  = (11:77:7)
 11
In the second vessel → 2/7:5/7 23.
(b).
In the third vessel → 1/4:3/4. A B
(6 / 7)  (5 / 7)  (3 / 4) Acid Water Acid Water
Hence, the final ratio is =
(1 / 7)  (2 / 7)  (1 / 4) 1 : 2 3 : 1
65 Acid = 1/3 Acid = 3/4
  =
19
Mixture: Acid:Water = 1:1
22.
(d) Step 1: Mixture of wheats of first and third kind to get Required acid = 1/2
a mixture worth `1.41 per kg? Now

Chapter_15.indd 354 1/30/2016 4:34:36 PM


 Alligation or Mixture  355


Required ratio = 1/4:1/6 = 3:2

So, the required quantity is 3 and 2 litres respectively.
i.e., 5:3
24.
(c) Kerosene in 100 kg = 7/32 × 100 = 21.875 kg
3
Petrol in 100 kg = 25/32 × 100 = 78.125 kg \ × 15 = 9 l
5
Now 78.125 is 25 parts out of (25 + 9) = 34 parts
\ 9 litres of water is added to HCL solution Similarly
Hence 9 parts = 9/25 × 78.125 = 28.125 16 litres of water is added to 90% concentrated
Amount to be added = 28.125 – 21.875 = 6.25 kg solution of HNO3 to dilute it to 70% concentration.
25. (c) In the new mixture, water:alcohol = 3:2 = 12:8 \ The total quantity of HCL and HNO3 solutions are
⇒  The capacity of each container = (12 + 8)/2 24 litres and 72 litres respectively and that of the
= 10 units aqua regia formed is 96 litres and that of water in
it is 33.6 litres.
⇒  The ratios of water to alcohol of the containers are
4:6 and 8:2 = 2:3 and 4:1 \ The quantity of mixture ‘X’ is 111 – 96 = 15
26.
(a) Weight of zinc in the alloy = 6 gms and weight of In the mixture X, the quantity of HCL is
copper in the alloy = 9 gms. 5
× 0.4 × 15 = 2.5 litres
Suppose ‘x’ gms of copper is removed. 12
Weight of copper in the alloy = 9 – x and the quantity of water is
Weight of zinc in the alloy = 6 + 10 = 16. 5 7 
Now, 16 = 4(9 – x) 12  0.6  12  0.7  × 15 @ (0.25 + 0.41) × 15 = 9.91
 
Therefore, x = 5 gm
12  2.5 14.5 1
27.
(a) Initial quantity of milk and water in the beakers. \ The required ratio is = =
33.6  9.9 43.5 3
Beaker A: Milk = 25 ml and Water = 75 ml
Beaker B: Milk = 20 ml and Water = 80 ml 29.
(d) 5 litres of 20% of solution = 1 litre i.e., the amount of
Beaker C: Milk = 40 ml and Water = 60 ml 1
alcohol. Now new strength of alcohol will be i.e.,
   After 40 ml is transferred from beaker A to beaker 6
C, the quantity of milk and water in the beakers is as 16.66%
follows: 30.
(b)
Beakers A: Milk = 25 – 10 = 15 ml and
Water = 75 – 30 = 40 ml
Beaker B: Milk = 20 ml and Water = 80 ml
Beaker C: Milk = 40 + 10 = 50 ml and
Water = 60 + 30 = 90 ml
Now, Milk: Water in Beaker C = 5.9
After 20 ml is transferred form beaker C to beaker B:
Beaker A: Milk = 15 and Water = 40 ml The alloy formed should contain gold and copper in
Beaker B: Milk = 20 + 10 = 30 ml and the ratio 6:4 i.e., 3:2
Water = 80 + 18 = 98 ml 31.
(c)
Beaker C: Milk = 50 – 10 = 40 ml and
Petrol Diesel Kerosene
Water = 90 – 18 = 72 ml
Initially 20 50 30
Required ratio = 15:30:40 = 3:6:8
Step 1 15 37.5 47.5
28.
(c) 80% concentrated HCL is to be diluted to form a 50%
concentrated HCL solution. The quantity of water to Step 2 71.66 12.5 15.83
be added to the 15 litres solution is

Chapter_15.indd 355 1/30/2016 4:34:38 PM


356  Chapter 15

32.
(c) After using 250 ml to make curd milk used
250
= × 900 = 180 ml
1250
Pure milk left = 900 – 180 = 720 ml
37.
(c) First Alloy Second Alloy
  C Al C Al

\ Required alloy
(Q  IL mixture)
C Al
Bournvita 5 1 2 1
= =
Complan 30 6 1
\ Copper in first alloy =
1 3
Bournvita = of the mixture
7 3
copper in second alloy =
33.
(d). Amount of alcohol in first vessel = 0.25 × 2 = 0.5 litre 4
amount of alcohol in second vessel = 0.4 × 6 = 2.4 litre 2
copper in required alloy (mixture) =
   Total amount of alcohol out of 10 liters of mixture 3
Now, by alligation
is 0.5 + 2.4 = 2.9 litre
Hence, the concentration of the mixture is 29%
 2.9 
   100
10 

30  x 6 3
34.
(d)   ⇒ 5x = 135 or x = 27 so, required
x  25 4 2
percentage of milk = 27%
1 1
:
35.
(d) % of liquid B in the original mixture. 12 3
1 ⇒ 1 : 4
= × 100 = 20%
5 38.
(d)
In the final mixture % of liquid B
3
= × 100 = 60%
5
Now using the rule of alligation


Hence both the types should be added in the ratio of
1:1 to obtain the required strength. Hence 20 litres of
first type should be added to the 20 litres of the second
type to get the desired solution.
Hence reduced quantity of the first mixture and the 39.
(d)
quantity of mixture B which is to be added are the
same.
\ Total mxiture = 10 + 10 = 20 liters and quantity of
20
liquid A = × 4 = 16 litres
5
36.
(d) In a mixture of 1,000 ml, milk:water = 3:1.
Hence, milk = 750 ml, water 250 ml
   A 250 ml of 3:2 solution contains 150 ml milk and
100 ml water.
i.e.,  2:1
Total milk = 900 ml, total water = 350 ml
Hence x litres must be 12 litres, i.e., (6 × 2).

Chapter_15.indd 356 1/30/2016 4:34:40 PM


 Alligation or Mixture  357

40. (d) In 100 cc solution, quantity of boric acid = 80 cc and 45.
(c) Suppose x litres of acid was drawn off initially.
quantity of water = 20 cc. Suppose x litres of water be \ Remaining acid in the 54-litre vessel full of acid.
added to make the solution 50%.
= (54 – x) litres and water added = x litres.
\ 50% (100 + x) = 80 ⇒ x = 60
Now out of the 54 litres of mixture of acid and
41. (b) Quantity of milk in the mixture = 48 litres
water, x litres of mixture is drawn off.
Quantity of water in the mixture = 32 litres
Let x litres of water be added in the mixture so that  54  x 
⇒ Quantity of acid drawn off =   x litres
 54 
48 2
= ⇒ 2x + 64 = 144 ⇒ x = 40
32  x 3 x2
and quantity of water drawn off = litres.
54
42.
(c) Suppose there is 100  cc of liquid in each container
initially.   54  x  
Now the vessel contains 54  x   x
I Step: Container B has 10 cc water in 110 cc of   54  
mixture. litres of acid.
II Step: 10 cc of this mixture contains  54  x 
\ 54  x   x = 24
100 100 10  54 
   10  =  cc of milk and  cc of water,
100 11 11
⇒ x2 – 108x + 1620 = 0
which is transferred to container C.
⇒ x = 90, 18
100
  Container C has 100 cc of acid,  cc of milk Since 90 > 54, therefore x = 90 is ruled out. Hence
11 x = 18
10
and
 cc of water.
11 46.
(b) Suppose the quantity of milk purchased = x litres
Total quantity of mixture in container C = 110 cc Suppose quantity of water mixed = y litres
III Step: 10 cc of this mixture is transferred to A \ Required ratio of the water and the milk in the
which contains. mixture = y:x
100 100
 10 cc of milk =  cc of milk C.P. of x litres of milk = `8.0x
1210 121
S.P of x litres of milk = 9 (x + y)
100
   Thus, in 100 cc of mixture, there is  cc of S.P.  100
121 \ C.P. =
1 100  Gain%
milk of the th proportion of the mixture.
121 (9 x  9 y )  100
⇒ 6.4 =
43.
(d) Total milk = 20 litres 100  40
Water Milk Total ⇒ 1120x = 900x + 900y
1 litre 19 litres 20 ⇒ 220x = 900y
1 19 + x 20 + x y 220 11
⇒ = =
   Now 1 = 2% of (20 + x), where x is the quantity of x 900 45
pure milk added to the mixture.
47.
(b) Suppose the quantity of milk purchased = x litres
⇒ x = 30
where x = quantity of pure milk added to Suppose quantity of water mixed = y litres
the mixture. \ Required ratio of the water and the milk in the
44. (b) mixture = y:x
Milk : Water C.P. of x litres of milk = `6.4 x
20 30 S.P. of x litres of milk = `8 (x + y)
10  15
Profit % = 37.5
 25
S.P.  100
10 40 \ C.P. =
100  Gain%
  Therefore, the concentration of the solution is (8 x  8 y )  100
reduced from 20 to 10, i.e., reduced by 50%. The ⇒ 6.4 =
100  37.5
dilution of the solution increased from 30 to 40, i.e.,
increased by 33% ⇒ 880x = 800x + 800y

Chapter_15.indd 357 1/30/2016 4:34:43 PM


358  Chapter 15

⇒ 80x = 800y ⇒ x = 10y 53.


(d) Let x litres of second solution must be added.
x y Then,
⇒ =
10 1 15 × 20 + 5 × x
= 10
\ Required ratio = 1:10 20 + x

48.
(d) Milk = 40 litres ⇒ 300 + 5x = 200 + 10x
Water = 20 litres 100
⇒ x = = 20 litres
5
To make the ratio M:W = 1:2, 60 litres of water
should be added. 54.
(c) Go through options, options (a) and (b) are ruled out
49.
(a) as released amount cannot be either 7 or 8.
For option (c),
Oxygen = 1.28 and Nitrogen = 6.72
After first released oxygen
= 1.28 – 0.32 = 0.96 litres
After second released oxygen
= 0.96 – 0.24 = 0.72 litres
which is 9% of 8 litres.
55.
(a) Total quantity of mixture = 2 + 3 + 0.5 = 5.5 gallon
Total quantity of water

12 7
We get two values of x, 7 and 13. But to get a = 2 × +3× + 0.5
viable answer, we must keep in mind that the central 100 100
value (10) must lie between x and 12. Thus, the value = 0.95 gallon
of x should be 7 and not 13. \ Required percentage
Required % increase = 7% 0.95
= × 100
50.
(d) \ Ratio of quantities sold at 10 % profit and 5% loss 5.5
= 12:3 = 4:1 190 3
= = 17 %
11 11
56.
(a) Let the quantity of milk replaced be x.
40 19 26
Then, (1 − x) + ×x = ×1
100 100 100
⇒ 40 – 40x + 19x = 26
⇒ 21x = 14
2
⇒ x =
3
\ The quantity sold at 10% profit 57.
(c) Quantity of salt in 6 litres of solution
50 5
= × 4 = 40 kg = × 6 = 0.3 litres
4+1 100
and, the quantity sold at 5% loss Percentage of salt in 5 litres of solution
= 50 – 40 = 10 kg 0.3
= × 100 = 6%
51.
(d) Here 10% of 150 kg = 15 kg 5
\ Good quality of wheat is 135. 58.
(d) Given that nectar contains 70% water.
To becomes the 5% low quality of wheat, we add And honey obtained contains 17% water.
150 kg of more wheat. It means that out of 1 kg honey 17% is water i.e., pure
25 30 270 honey = 0.83 kg
52.
(a) Milk in mixture = 6 × + 4× =
100 100 100 Which is 20% of the flower nectar
270 × 10 [Q 70% is water].
For 10 parts = = 27% Now, let quantity of flower nectar processed be x kg.
100

Chapter_15.indd 358 1/30/2016 4:34:46 PM


 Alligation or Mixture  359

Then, 2
0.3 × x = 0.83 ⇒ x – 20 = (x + 20)
3
0.83 ⇒ 3x – 60 = 2x + 40
\ x = = 2.77 kg
0.3 ⇒ x = 100 g
59.
(d) Required quantity of water
65.
(a) \ Ratio of quantities sold at 8% profit and 18% profit
9 × (50 – 30) 9 × 20 = 4:6 = 2:3
= = = 6 mL
30 30
60.
(c) LCM of (3 + 2), (7 + 3) and (11 + 4) is 30.
Let the capacity of each container be 30 litres.
\ Quantity of milk after mixing
 3 7 11 
=  + +  × 30
 5 10 15 
= 61 litres
Quantity of water after mixing
2 3 4
=  + +  × 30
 5 10 15 
= 29 litres Therefore, the quantity sold at 18% profit
\ Required ratio = 61: 29 50
= × 3 = 30 kg
2+3
61.
(a) Quantity of alcohol in 1 litres mixture of first bottle
2 1 3
= ×1 = L 66.
(b) Milk = × 100 = 75
10 5 4
As second bottle does not contains alcohol. Ratio of milk and water = 3:1
1 1 1
So, required fraction = × = L 1
3 5 15 Water = × 100 = 25
4
62.
(c) Spirit Water Let x litres of mixture is taken away, then quantity of
Vessel 1 0.5 : 0.5 3x 

1 : 1 milk left =  3 − 
 4 
Vessel 2 0.75 : 0.25
3 : 1  x
and water left = 1 −  + x
 4
1 3
×2+ ×3
Spirit 13 3x x
= 2 4 = Given, 3− = 1− + x
Water 1 × 2 + 1
×3 7 4 4
2 4
3x x
⇒ 3 – 1 = − +x
2 4 4
63.
(d) Quantity of milk = × 60 = 40 litres
3 6x
⇒ 2 =
1 4
Quantity of water = × 60 = 20 litres
3
4
Quantity of water added = x litres (suppose) ⇒ x =
3
40 1
Now, = 4
20 + x 2
1
⇒ x = 60 litres \ Required percentage = 3 × 100 =
33 %
4 3
64.
(d) Initially, let xg of water was taken.
According to the problem: 67. (b) Quantity of water in the mixture
2 1 20
(x – 20) + (x + 20) = 4 × (x + 20) = × 45 =
9L
3 3 100

Chapter_15.indd 359 1/30/2016 4:34:49 PM


360  Chapter 15

Let x litres of water must be added to make the water 71.


(b) Using Alligation Method,
25% in the solution.
9+ x
\ × 100 = 25
45 + x

⇒ 36 + 4x = 45 + x
⇒ 3x = 9
i.e., 4:1
⇒ x = 3 litres
Hence, the required quantity of Sugar I
68.
(c) Let the quantity of liquid P and Q be 5x and 3x litres 75
= × 4 = 300 kg
respectively. 1
5
Quantity of P removed = × 16 = 10 litres 72.
(c) Let the quantity of pure milk be x litres.
5+3
If 5 litres of water is added to it, then cost of (5 + x)
3 litres = `(3x + 5)
Quantity of Q removed = × 16 = 6 litres \ Profit = `15
5+3
Given, 20% of 3x =15
5 x − 10 3 3x
Now, = ⇒ = 15
3 x − 6 + 16 5 5

⇒ 25x – 50 = 9x + 30 ⇒ x = 25 litres
⇒ 16x = 80 \ The amount of pure milk in the mixture was
⇒ x = 5 25 litres.

\ Quantity that vessel hold = 8 × 5 = 40 litres 73.


(d) Water (W) + Drained (D) = 12
⇒ W + D = 12
69.
(c) Amount of dettol after
Now, W = D – 6
2
First operation = D – 6 + D = 12
3
⇒ D = 9 litres
2 2 4
Second operation = × = 1
3 3 9 74.
(a) Quantity of nitric acid = 10 ×= 1 litre
10
4 2 8 Water =10 – 1 = 9 litres
Third operation = × =
9 3 27 Let x litres of water be added.
8 2 16 4
Fourth operation = × = Then,  (10 + x) × = 1 ⇒ x = 15 litres
27 3 81 100
Amount of water after fourth operation 75.
(b) Given, that sedimenting starts when the ratio of salt to
16 65 water is 1:4.
= 1 − =
81 81 Now, 1000 g solution has 40 g salt and 960 g water.
Process of sedimenting will start when the ratio of salt
16 65
\ Required ratio = : = 16 : 65 to water is 1:4 i.e., 40:160.
81 81 Hence, 800 g of water has to be evaporated.
70.
(c) Selling price of mixture = `20 Now, time required to evaporate 800 g of water
100 800
Cost price of mixture = × 20 = `16 = = 28.57 hrs ≈ 29 hrs
125 28
By the rule of alligation 5
76.
(b) of the tin is in 1 kg of alloy. Since quantity of tin is
13
constant (because the other alloy does not contain tin),
5
therefore, there is of the tin in 2 kg of alloy.
13
5
Hence, there is of tin in 1 kg of alloy.
So, required ratio = 16:9 26

Chapter_15.indd 360 1/30/2016 4:34:51 PM


 Alligation or Mixture  361

77.
(d) CP of 25 kg of rice @ `6 = `25 × 6 = `150 83.
(c) In this question, the alligation method is applicable
CP of 35 kg of rice @ `7 = `35 × 7 = `245 for the speed.

\ CP of 25 + 35, i.e., 60 kg of rice


= `(150 + 245) = `395
SP of 1 kg of rice = `6.75
\ SP of 60 kg of rice = `6.75 × 60 = `405
Hence, total gain = `405 – `395 = `10.

78.
(d) Concentration of basic acid = 80% = 80 cc
Quantity of water = 20 cc
Let x cc of water be added to get the concentration of
50%. \ Time spent in bus:time spent in train

80 50 45 45
= = : = 1:1.
100 + x 100 6 6

80 1 \ Distance travelled by train = 55 × 3 = 165 km
⇒ =
100 + x 2 84. (b) In this question the alligation method is applicable on

⇒ x = 60 cc prices, so we should get the average price of mixture.
S.P. of mixture = `20/litre, profit = 25%
79.
(a) Let Cost of beer = `x/ bottle 100
\ Average price = 20 × = `16/litre
\ Cost of lager = `2x/ bottle 125
\ A spent `4x and B spent `2x C paid `50
\ Cost of lager = `50 per bottle
Cost of beer = `12.50 per bottle

80
80.
(a) Content of gold in alloy = 50 × = 40 g
100
Let x g of gold should be added to alloy and solve the
equation for ‘x’.
40  x
 100 = 90
50  x \ Chemical:Water = 16:9

85.
(b) 300 g solution has 40% salt in it.
81.
(a) Let x kg tea is blended.
So, the amount of salt in it
\ Total CP = `(25x + 900)
40  300
\ Selling price on 10% profit =  120 g
100
10 For the salt percentage to be 50%,
(25x + 900) + (25x + 900) × = `(27.5x + 990)
100
120  x 1
Total weight of tea = (30 + x) kg =
300  x 2
\ Selling price at the rate of 30 kg = `(900 + 30x)
\ 240 + 2x = 300 + x
On comparing both the selling prices, we get ‘x’. x = 60 g
82.
(b) Write now the amount of salt in the solutions Hence, option (b).
90 5x 5
= 300  = 12 g 86.
(b) = ⇒ 10x = 5x + 25 ⇒ x = 5
100 x5 2
Let x g salt is added to the solution Milk = 25 litres
120  x M 5x 5
\ × 100 = 50 87.
(c) = =
300  x W x5 2
\ x = 5
⇒ x = 60
\ Quantity of milk = 5x = 25 litres

Chapter_15.indd 361 1/30/2016 6:04:22 PM


362  Chapter 15

88.
(d) Milk in the second vessel
Zinc Copper 2
= = 0.66
5 8 3
13 13 Milk in the third vessel
5 3 3
8 8 = = 0.6
5
 5 5   8 3 Milk in the fourth vessel
\    :    = 105:103
 13 8   13 8  7
= = 0.636
11
5 2 3 3
89.
(c) x y = x y 94.
(c) Amount of saw dust in 2 kg of haldi
8 5 8 5
5
x 4 = 2000  = 100 g
⇒ = 100
y 5
Required proportion = 4%
⇒ Ratio = 4:5 100 4
90.
(b) ⇒ =
2000  x 100

⇒ x = 500 ⇒ x = 0.5 kg
95.
(c) Let the quantity initially of liquid A be 7x litre
\ Let the quantity initially of liquid B be 5x litre
Quantity of A after drawn 9 litres mixture
21 28 x − 21
= 7 x − = litres
4 4
Quantity of B after drawn 9 litres mixture and adding
21 20 x + 21
The required ratio is 500:25 or 20:1 9 litres of B = 5 x + =
4 4
Number of labourers 20
= (28 x − 21) 20 x + 21 7
Number of supervisors 1 \ : =
4 4 9
840 20 ⇒ (252 – 140)x = 189 + 147
⇒ =
Number of supervisors 1 336
⇒ x = =3
840 112
\ Number of supervisors = = 42 Reqd. quantity of A = 7x = 21 litres
20
96.
(c) Let x litres of solution is mixed
91.
(c) 1st 2nd
 3x 
\ Content of acid in new mixture =  30   L
 10 
\ Range of the new solution can be found by the
300  3 x
equations × 100 = 20% and
10(200  x)
1 1
\ Ratio = : = 3:5 300  3 x
20 12  100 = 25%
10(200  x)
92.
(a) Spirit:Water
97. (d) Weight of water in the mixture of 60 g water
3 4 5  4 5 6 
=  + +  :  + +  75
 7 9 11   7 9 11  = 60 × = 45 g
100
= 920:1159 weight of water in the mixture of 45 g water
93.
(c) Milk in the first vessel = 45 + 15 = 60 g
5 60 × 100
= = 0.625 \ Percentage of water = = 80%
8 75

Chapter_15.indd 362 1/30/2016 4:34:58 PM


 Alligation or Mixture  363

98.
(d) Let the quantity of acid in original mixture be x litre 101.
(b) Quantity of milk in the mixture
and quantity of water in the original mixture be 3x 6 5 8
litre. = + +
13 14 15
x+5 1
Therefore, = 1260 + 975 + 1456
3x 2 =
2730
2(x + 5) = 3x
⇒ 2x + 10 = 3x 3691
=
⇒ 3x – 2x = 10 2730
⇒ x = 10 Quantity of milk in the mixture
Therefore, quantity of new mixture 7 9 7
= + ×
= 4x + 5 = 4(10) + 5 13 14 15
= 45 litres 1470 + 1755 + 1274
=
99.
(b) Let the quantity of nickel mixel be x kg 2730
Therefore, 4499
=
20 + x 3 2730
=
100 + x 11 \ Required ratio = 3691:4499
⇒ 11(20 + x) = 3(100 + x) 102.
(d) Milk and water ratio = 3:1
⇒ 220 + 11x = 300 + 3x \ Total = 4
⇒ 11x – 3x = 300 – 220 \ We take half of pure milk and half of water, then
⇒ 8x = 80
2 1
80 total remainder = = unit
⇒ x = = 10 kg 4 2
8
103. (b) Applying allegation
100. (b) Amount of liquid left after n operations, when the
container originally contain x units of liquid from
n
 y
which y units is taken out each time is x 1 − 
 x
units.
2 40
Here x = 20 litres, y = 20 × = ,n=3
3 3
3 3
Hence, the required ratio is 2:1.
 40/3   2
\ 20 1 −  = 20 1 − 
 20   3
1 1 1 20
= 20 × × × =
3 3 3 27

Chapter_15.indd 363 1/30/2016 4:35:01 PM


16 Problems on Ages

INTRODUCTION
Problems based on ages are generally asked in most of (iii) Age some years hence
the competitive examinations. To solve these problems, Two of these situations are given and it is required to
the knowledge of linear equations is essential. In such find the third. The relation between the age of two persons
problems, there may be three situations: may also be given. Simple linear equations are framed and
(i) Age some years ago their solutions are obtained. Sometimes, short-cut methods
(ii) Present age given below are also helpful in solving such problems.

SOME USEFUL SHORT-CUT METHODS

Illustration 1 The age of father is 4 times the age of his son.


1. If the age of A, t years ago, was n times the age If 5 years ago father’s age was 7 times the age of his son at
of B and at present A’s age is n2 times that of that time, what is father’s present age?
B, then
Solution: The father’s present age
 n −1 
A’s present age =  1  n2 t years
 n1 − n2   n −1 
=  1  n2t (Here n1 = 7, n2 = 4 and t = 5)
 n1 − n2 
 n −1 
and B’s present age =  1  t years
 n1 − n2   7 −1  6× 4×5
=   4×5= = 40 years.
 7 − 4  3

Explanation 2. The present age of A is n1 times the present age of


Let the present age of B be x years. B. If t years hence, the age of A would be n2 times
Then, the present age of A = n2 x years that of B, then
 n −1 
Given, t years ago, A’s present age =  2  n2 t years
 n1 − n2 
n1 (x - t ) = n2x - t or, (n1 – n2) x = ( n1 - 1 ) t
 n −1 
 n −1  and B’s present age =  2  t years
or, x =  1  t.  n1 − n2 
 n1 − n2 
 n −1  Explanation
Therefore, B’s present age =  1  t years Let the present age of B be x years.
 n1 − n2 
Then, the present age of A = n1x
 n −1 
and A’s present age =  1  n2 t years . Given, t years hence,
 n1 − n2  (n1x + t) = n2 (x + t)

Chapter_16.indd 364 1/30/2016 4:57:34 PM


Problems on Ages 365

or, (n1 - n2)x = (n2 - 1)t Solution: Present age of Anu


 n −1  t2 (n2 − 1) + t1 (n1 − 1)
x =  2 =
or, t n1 − n2
 n1 − n2 
(Here n1 = 4, n2 = 2, t1 = 10 and t2 = 10)
 n −1  10(2 − 1) + 10(4 − 1) 10 + 30
Therefore, B’s present age =  2  n1t years = = = 20 years.
 n1 − n2  4−2 2
 n −1 
and A’s present age =  2  n1t years. 4. The sum of present ages of A and B is S years. If,
 n1 − n2 
t years ago, the age of A was n times the age of
Illustration 2 The age of Mr Gupta is four times the age B, then
Sn − t (n − 1)
of his son. After 10 years, the age of Mr Gupta will be only Present age of A = years
n +1
twice the age of his son. Find the present age of Mr Gupta’s
S + t (n − 1)
son. and Present age of B = years.
n +1
Solution: The present age of Mr Gupta’s son
 n −1 
=  2 t Explanation
 n1 − n2  Let the present ages of A and B be x and y years, respectively.
 2 −1  Given x+y =S ...(1)
=   10
 4−2 and x - t = n (y - t)
(Here n1 = 4 , n2 = 2 and t = 10) or x - ny = (1 - n) t ...(2)
= 5 years. Solving Eqs. (1) and (2), we get
Sn − t (n − 1)
3. The age of A, t1 years ago, was n1 times the age of x=
n +1
B. If t2 years hence A’s age would be n2 times that
of B, then, S + t (n − 1)
and y= .
n (t + t )(n − 1) n +1
A’s present age = 1 1 2 2 + t1 years Illustration 4 The sum of the ages of A and B is 42 years. 3
n1 − n2
years back, the age of A was 5 times the age of B . Find the
t2 (n2 − 1) + t1 (n1 − 1) difference between the present ages of A and B.
and B’s present age = years.
n1 − n2 Solution: Here S = 42, n = 5 and t = 3
\ Present age of A
Explanation Sn − t (n − 1) 42 × 5 − 3(5 − 1)
= =
Let A’s present age = x years and B’s present age = y years. n +1 5 +1
Given x - t1 = n1 (y - t1) and x + t2 = n2 (y + t2) 198
= = 33 years
i.e., x - n1 y = (1 - n1) t1 ...(1) 6
and x - n2 y = (-1 + n2 ) t2 ...(2) and present age of B

Solving Eqs. (1) and (2), we get 5 + t (n + 1) 42 + 3(5 − 1)


= =
n (t + t )(n − 1) n +1 5 +1
x= 1 1 2 2 + t1 54
n1 − n2 = = 9 years.
6
t2 (n2 − 1) + t1 (n1 − 1)
and, y= . \ Difference between the present ages of A and B =
n1 − n2
33 - 9 = 24 years.
Illustration 3 10 years ago Anu’s mother was 4 times older Note: If, instead of sum (S), difference (D) of their ages is
than her daughter. After 10 years, the mother will be twice given, replace S by D and in the denominator (n + 1) by
older than the daughter. Find the present age of Anu. ( n - 1) in the above formula.

Chapter_16.indd 365 1/30/2016 4:57:37 PM


366 Chapter 16

5. The sum of present ages of A and B is S years. If, 6. If the ratio of the present ages of A and B is a:b and
t years hence, the age of A would be n times the t years hence, it will be c:d, then
age of B, then at (c − d )
Sn + t (n − 1) A’s present age =
present age of A = years ad − bc
n +1 bt (c − d )
S − t (n − 1) and, B’s present age = .
and present age of B = years. ad − bc
n +1
Illustration 6 The ratio of the age of father and son
Explanation at present is 6:1. After 5 years, the ratio will become
Let the present ages of A and B be x and y years, respectively 7:2. Find the present age of the son.
Given x+y =S ...(1) bt (c − d )
Solution: The present age of the son =
and x + t = n ( y + t) ad − bc
or x - ny = t (n - 1) ...(2) (Here a = 6, b = 1, c = 7, d = 2 and t = 5)
Solving Eqs. (1) and (2), we get 1× 5(7 − 2)
= = 5 years.
Sn + t (n − 1) 6 × 2 − 1× 7
x =
n +1 Note: If, with the ratio of present ages, the ratio of ages t
S − t (n − 1) years ago is given, then replace t by (-t) in the above
and y = . formula.
n +1
Illustration 7 Six years ago Mahesh was twice as old as
Illustration 5 The sum of the ages of a son and father is 56 Suresh. If the ratio of their present ages is 9:5 respectively,
years. After four years, the age of the father will be three what is the difference between their present ages?
times that of the son. Find their respective ages. Solution: Present age of Mahesh
Solution: The age of father
−at (c − d )
Sn + t (n − 1) 56 × 3 + 4(3 − 1) =
= = ad − bc
n +1 3 +1
−9 × 6(2 − 1)
(Here S = 56, t = 4 and n = 3) =
1× 9 − 5 × 2
176
= = 44 years. (Here a = 9, b = 5, c = 2, d = 1 and t = 6)
4
= 54 years
S − t (n − 1)
The age of son = Present age of Suresh
n +1 −bt (c − d )
56 − 4(3 − 1) =
= ad − bc
3 +1 −5 × 6(2 − 1)
= = 30 years.
48 1× 9 − 5 × 2
= = 12 years.
4 \ Difference of their ages = 54 - 30 = 24 years.

MULTIPLE CHOICE QUESTIONS

1. A’s age is one-sixths of B’s age. B’s age will be twice of C’s 2. Sachin was twice as old as Ajay 10 years back. How
age after 10 years. If C’s eighth birthday was celebrated old is Ajay today if Sachin will be 40 years old 10 years
two years ago, then the present age of A must be hence?
(a) 5 years (b) 10 years (a) 20 years (b) 10 years
(c) 15 years (d) 20 years (c) 30 years (d) None of these
[Based on MAT, 2002] [Based on MAT, 2005]

Chapter_16.indd 366 1/30/2016 4:57:40 PM


 Problems on Ages  367

3.
A demographic survey of 100 families in which two 10. A’s age is thrice that of B’s and four times that of C’s. Find
parents were present revealed that the average age A, of A:B:C.
the oldest child, is 20 years less than half the sum of the (a) 1:3:4 (b) 3:4:12
ages of the two parents. If F represents the age of one
(c) 2:3:8 (d) None of these
parent and M, the age of the other parent, then which of
the following equivalent to A? 11. Namrata’s father is now four times her age. In five years,
F + M − 20 F+M he will be three times her age. In how many years, will he
(a) (b) + 20 be twice her age?
2 2
(a) 5 (b) 20
F+M
(c) − 20 (d) F + M – 10 (c) 25 (d) 15
2
[Based on MAT, 2001] [Based on SCMHRD Ent. Exam., 2003]
4. Rohan is two years younger than Mohan who is three 12. A father is twice as old as his son. 20 years back, he was
years younger than Sohan who is four years older than twelve times as old as the son. What are their present
Rohit who is two years older than Mohit who is three ages?
years younger than Sohit. Thus (a) 24, 12 (b) 44, 22
(a) Sohan is 7 years older than Mohit. (c) 48, 24 (d) None of these
(b) Rohit is 2 years younger than Sohit. [Based on IMT Ghaziabad, 2002]
(c) Mohan is 3 years older than Mohit. 13. There were 15 students in a class. When the ages of a
(d) Rohit is 2 years older than Sohit. teacher and a new boy are added, the average age of the
class increases by 10 per cent while it remains the same
5.
Two groups of student, whose average ages are 20 years
when only the age of a boy is added. If the teacher’s age
and 30 years, combine to form a third group whose
is eight more than twice the age of the new boy, then find
average age is 23 years. What is the ratio of the number of
the initial average age of the class.
students in the first group to the number of students in the
second group? (a) 15.4 years (b) 16.5 years
(a) 5:2 (b) 2:5 (c) 11.4 years (d) None of these
(c) 7:3 (d) None of these 14. The age of a person k years ago was half of what his age
[Based on IIT Joint Man. Ent. Test, 2004] would be k years from now. The age of the same person
p years from now would be thrice of what his age was
6. A years ago, a father was four times his son’s age. In six
p years ago. What is the value of the ratio k:p?
years, his age will be 9 more than twice his son’s age.
What is the present age of the son? (a) 3:2 (b) 2:3
(a) 10 years (b) 9 years (c) 1:4 (d) 4:1
(c) 20 years (d) None of these 15. The ratio of ages of Rahul and Deepesh is 3:5. 10 years
[Based on IIT Joint Man. Ent. Test, 2004] later this ratio becomes 5:7. What is the present age of
7. 5 years ago his mother’s age was thrice that of Amit. Deepesh?
Amit’s present age is 20. What will be the ratio of their (a) 20 years (b) 50 years
ages 10 years from now? (c) 25 years (d) 40 years
(a) 30:70 (b) 1:3
16. Ten years ago, the ages of the members of a joint family of
(c) 5:2 (d) 1:2 eight people added up to 231 years. Three years later, one
8.
Sister’s age is 3 times that of her brother’s. After 5 years member died at the age of 60 years and a child was born
the sister shall be twice as old as her brother. How many during the same years. After another three years, one more
years before, the sister’s age was 6 times of her brother’s member died, again at 60, and a child was born during the
age? same years. The current average age of this eight-member
joint family is nearest to
(a) 1 year (b) 3 years
(a) 21 years (b) 25 years
(c) 5 years (d) 10 years
(c) 24 years (d) 23 years
9. The average age of a class is 15.8 years. The average age [Based on CAT, 2007]
of the boys in the class is 16.4 years and that of the girls is
17. 10 years ago the age of Karisma was two-thirds of
15.4 years. What is the ratio of boys to girls in the class?
the age of Babita. 14 years hence the ratio of ages of
(a) 1:2 (b) 3:4 Karishma and Babita will be 5:9. Find the ratio of their
(c) 2:3 (d) None of these present ages.

Chapter_16.indd 367 1/30/2016 4:57:40 PM


368  Chapter 16

(a) 13:29 (b) 11:27 27. Three times the present age of a father is equal to eight
(c) 29:17 (d) 13:25 times the present age of his son. Eight years hence the
father will be twice as old as his son at that time. What are
18. The ratio of ages of A and B is 8:9 and the age of B is their present ages?
two-thirds of C’s age and age of C is nine-thirteenths
(a) 35, 15 (b) 32, 12
times the age of D. If the age of B is 18 years, then the
age of C is (c) 40, 15 (d) 27, 8
(a) 36 years (b) 39 years 28. Five years ago Mr Sohanlal was thrice as old as his son
(c) 27 years (d) 54 years and ten years hence he will be twice as old as his son.
Mr Sohanlal’s present age (in years) is
19. If Dennis is one-third the age of his father Keith now, and (a) 35 (b) 45
was one-fourth the age of his father 5 years ago, then how (c) 50 (d) 55
old will his father Keith be 5 years from now?
(a) 20 years (b) 45 years 29.
If 6 years are subtracted from the present age of Randheer
and the remainder is divided by 18, then the present age
(c) 40 years (d) 50 years
of his grandson Anup is obtained. If Anup is 2 years
20.
Fifteen years hence, a man will be four times as old as he younger to Mahesh whose age is 5 years, what is the age
was fifteen years ago. His present age is of Randheer?
(a) 25 years (b) 20 years (a) 84 years (b) 48 years
(c) 30 years (d) 45 years (c) 60 years (d) 96 years
[Based on MAT (Sept), 2008]
21.
Father is 5 years older than mother and mother’s age now
is thrice the age of the daughter. The daughter is now 10 30.
If 1 is added to the age of the elder sister, then the ratio of
years old. What was father’s age when the daughter was the ages of two sisters becomes 0.5:1, but if 2 is subtracted
born? from the age of the younger one, the ratio becomes 1:3.
(a) 20 years (b) 15 years Find the age of the two sisters.
(c) 25 years (d) 30 years (a) 8 and 5 years (b) 11 and 6 years
(c) 9 and 5 years (d) 8 and 6 years
22. The average age of the mother and her 6 children is 12 [Based on MAT (Dec), 2010]
years which is reduced by 5 years if the age of the mother
is excluded. How old is the mother? 31. Jayesh is as much younger to Anil as he is older to
Prashant. If the sum of the ages of Anil and Prashant is 48
(a) 42 years (b) 40 years
years, what is the age of Jayesh?
(c) 48 years (d) 50 years (a) 20 years (b) 24 years
23.
A father’s age is three times the sum of the ages of his two (c) 30 years (d) Cannot be determined
children, but 20 years hence his age will be equal to the
32.
The ratio of the ages of the father and the son at present is
sum of their ages. Then the father’s age is
7:1. After 4 years, the ratio will become 4:1. What is the
(a) 30 years (b) 40 years sum of the present ages of the father and the son?
(c) 35 years (d) 45 years (a) 29 years (b) 35 years
24.
The ages of A, B and C together total 185 years. B is (c) 32 years (d) None of these
twice as old as A and C is 17 years older than A. Then, the [Based on MAT (Sept), 2008]
respective ages of A, B and C are 33.
A boy was asked of his age by his friend. The boy said,
(a) 40, 86 and 59 years (b) 42, 84 and 59 years “The number you get when you subtract 25 times my age
(c) 40, 80 and 65 years (d) None of these from twice the square of my age will be thrice your age.”
If the friend’s age is 14, then the age of the boy is
25. One years ago a father was four times as old as his son.
(a) 28 years (b) 21 years
In 6 years time his age exceeds twice his son’s age by 9 (c) 14 years (d) 25 years
years. Ratio of their ages is
[Based on MAT (Feb), 2011]
(a) 13:4 (b) 12:5
34.
The product of the present ages of Sarita and Gauri is 320.
(c) 11:3 (d) 9:2
Eight years from now, Sarita’s age will be three times the
26. The sum of ages of a father and son is 45 years. Five years age of Gauri. What was the age of Sarita when Gauri was
ago, the product of their ages was four times the father’s born?
age at that time. The present age of the father is (a) 40 years (b) 32 years
(a) 39 years (b) 36 years (c) 48 years (d) 36 years
(c) 25 years (d) None of these [Based on MAT (Feb), 2011]

Chapter_16.indd 368 1/30/2016 4:57:40 PM


 Problems on Ages  369

35.
Anil is at present one-fourth the age of his father. After (a) 13:12 (b) 3:1
16 years, he will be one-half age of his father. Find the (c) 11:3 (d) 25:7
present age of Anil’s father. [Based on MAT (Dec), 2007]
(a) 40 years (b) 36 years
43.
The average age of a husband, his wife and son 3 years
(c) 32 years (d) 28 years ago was 27 years and that of his wife and son 5 years ago
[Based on MAT (Sept), 2009] was 20 years. What is the husband’s present age?
36.
Honey was twice as old as Vani 10 years ago. How old is (a) 35 years (b) 32 years
Vani today, if Honey will be 40 years old 10 years hence? (c) 37 years (d) 40 years
(a) 25 years (b) 20 years [Based on MAT (Feb), 2008]
(c) 15 years (d) 35 years
44.
In a class, there are 20 boys whose average age is
[Based on MAT (Feb), 2009] decreased by 2 months, when one boy age 18 years is
37.
The age of the father 5 years ago was 5 times the age of replaced by a new boy. The age of the new boy is
his son. At present the father’s age is 3 times that of his (a) 14 years 8 months (b) 16 years 4 months
son. What is the present age of the father? (c) 15 years (d) 17 years 10 months
(a) 33 years (b) 30 years [Based on MAT (Dec), 2007]
(c) 45 years (d) None of these
45.
The age of the father of two children is twice that of the
[Based on MAT (Feb), 2009]
elder one added to four times that of the younger one.
38.
Sonu is 4 years younger than Manu while dolly is 4 years If the geometric mean of the ages of the two children
younger than Sumit but one-fifth times as old as Sonu. If is 4 3 and their harmonic mean is 6, then what is the
Sumit is eight years old, how many times as old is Manu father’s age?
as Dolly? (a) 48 years (b) 32 years
(a) 6 (b) 1/2
(c) 40 years (d) 56 years
(c) 3 (d) None of these [Based on MAT (Sept), 2008]
[Based on MAT (Sept), 2008]
46.
The age of a man is 3 times that of his son. 15 years ago,
39.
If the ages of P and R are added to twice the age of Q, the man was 9 times as old as his son. What will be the age
the total becomes 59. If the ages of Q and R are added to of the man after 15 years?
thrice the age of P, the total becomes 68 and if the age of
(a) 45 years (b) 60 years
P is added to thrice the age of Q and thrice the age of R,
the total becomes 108. What is the age of P? (c) 75 years (d) 65 years
(a) 17 years (b) 19 years [Based on MAT, 1999]

(c) 15 years (d) 12 years 47.


Ashu’s mother was three times as old as Ashu 5 years ago.
[Based on MAT (Sept), 2008] After 5 years, she will be twice as old as Ashu. How old is
Ashu today?
40.
A father’s age is three times the sum of the ages of his two
(a) 35 years (b) 10 years
children, but 20 years hence his age will be equal to the
sum of their ages. Then, the father’s age is (b) 20 years (d) 15 years
(a) 30 years (b) 40 years [Based on MAT, 1999]

(c) 35 years (d) 45 years 48.


Father is 5 years older than the mother and the mother’s
[Based on MAT (May), 2008 (Sept), 2007] age now is thrice the age of the daugh­ter. The daughter
is now 10 years old. What was the father’s age when the
41.
If 6 years are subtracted from the present age of Randheer daughter was born?
and the remainder is divided by 18, then the present age of
his grandson Anup is obtained. If Anup is 2 years younger (a) 20 years (b) 15 years
to Mahesh, whose age is 5 years, then what is the age of (c) 25 years (d) 30 years
Randheer? [Based on MAT, 1999]
(a) 84 years (b) 96 years 49.
A father told his son, “I was as old as you are at present at
(c) 48 years (d) 60 years the time of your birth”. If the father is 38 years old now,
[Based on MAT (Feb), 2008] then what was the son’s age five years back?
42.
1 year ago, a mother was 4 times older to her son. After (a) 14 years (b) 19 years
6 years, her age becomes more than double her son’s age (c) 38 years (d) 33 years
by 5 years. The present ratio of their age will be [Based on MAT, 1999]

Chapter_16.indd 369 1/30/2016 4:57:41 PM


370  Chapter 16

50.
Fifteen years hence, a man will be four times as old as he 58.
The ratio of Laxmi’s age to the age of her mother is 3:11.
was fifteen years ago. His present age is The difference of their ages is 24 years. The ratio of their
(a) 25 years (b) 20 years ages after 3 years will be
(c) 30 years (d) 45 years (a) 1:3 (b) 2:3
[Based on MAT, 1999] (c) 3:5 (d) 2:5
51.
The average age of an adult class is 40 years. 12 new [Based on FMS, 2006]
students with an average age of 32 years join the class, 59.
A man born in the first half of the nineteenth century was
thereby decreasing the average by 4 years. The original x years old in the years x2. He was born in
strength of the class was (a) 1806 (b) 1836
(a) 10 (b) 11 (c) 1812 (d) 1825
(c) 12 (d) 15 [Based on FMS, 2011]
[Based on MAT, 2000]
60. The ratio between the ages of a father and a son at present
52.
The father is five times older than his son. After 4 years, is 5:2, respectively. Four years, hence the ratio between
the sum of their ages would be 44 years. Then the son’s the ages of the son and his mother will be 1:2 respectively.
age at present is What is the ratio between the present ages of the father
(a) 5 years (b) 6 years and the mother respectively?
(c) 7 years (d) 8 years (a) 3:4 (b) 5:4
[Based on MAT, 2000] (c) 4:3 (d) Cannot be determined
53.
Father’s age is 4 times that of his son. 5 years back, it was [Based on Allahabad Bank PO, 2010]
7 times. His age now is 61. Radha’s present age is three years less than twice her
(a) 30 (b) 35 age 12 years ago. Also the respective ratio between Raj’s
(c) 40 (d) 45 persent age and Radha’s present age in 4:9. What will be
[Based on MAT, 2000] Raj’s age after 5 years?
(a) 12 years (b) 7 years
54.
Sushil was thrice as old as Snehal 6 years back. Sushil
will be five-thirds times as old as Snehal 6 years hence. (c) 21 years (d) None of these
How old is Snehal today? [Based on Punjab and Sindh Bank PO, 2010]
(a) 18 years (b) 24 years 62. The respective ratio of the present ages of Swati and
(c) 12 years (d) 15 years Trupti is 4:5. Six years hence the respective ratio of their
[Based on FMS (MS), 2006] ages will be 6:7. What is the difference between their
ages?
55.
In a cricket 11, the average age of 11 players is 28
years. Out of these, the average ages of three groups of (a) 2 years (b) 3 years
three players each are 25 years, 28 years and 30 years, (c) 4 years (d) Cannot be determined
respectively. If in these groups, the captain and the [Based on Punjab National Bank PO, 2010]
youngest player are not included and the captain is eleven 63.
Four years ago Shayam’s age was three-fourths times that
years older than the youngest player, what is the age of the of Ram. Four years hence, Shayam’s age will be five-sixths
captain? times that of Ram. What is the present age of Shayam?
(a) 33 years (b) 34 years (a) 15 years (b) 20 years
(c) 35 years (d) 36 years (c) 16 years (d) 24 years
[Based on FMS (MS), 2006] [Based on Corporation Bank PO, 2009]
56.
The average age of an adult class is 40 years. Twelve 64. The ratio of the present ages of Anju and Sandhya is
new students with an average age of 32 years join the 13:17, respectively. Four years ago the respective ratio of
class, thereby decreasing the average age of the class by their ages was 11:15. What will be the respective ratio of
4 years. The original strength of the class was their ages six years hence ?
(a) 10 (b) 11 (a) 3:4 (b) 7:8
(c) 12 (d) 15 (c) 5:4 (d) None of these
[Based on FMS (MS), 2006]
[Based on Corporation Bank PO, 2010]
57. Ratio of Ashok’s age to Pradeep’s age is 4:3. Ashok will 65. Five years ago, Bina’s age was three times that of Arti.
be 26 years old after 6 years. How old is Pradeep now? Ten years ago, Bina’s age was half that of Chitra. If c
(a) 18 years (b) 21 years represents Chitra’s current age, which of the following
(c) 15 years (d) 24 years represents Arti’s current age?

Chapter_16.indd 370 1/30/2016 4:57:41 PM


 Problems on Ages  371

(a) (c – 10)/3 (b) c/6 + 5 73. The ratio of A’s and B’s ages is 4:5. If the difference
(c) 3c – 5 (d) 5c/3 – 10 between the present age of A and the age of B 5 years
[Based on MHT-CET MBA, 2010] hence is 3 years, then what is the total of present ages of A
and B?
66. The ages of Melwyn and Louis are in the ratio of 7:10 (a) 68 years (b) 72 years
respectively. After 6 years the ratio of their age will be
17:23. What is the difference in their ages? (c) 76 years (d) 64 years
(a) 8 years (b) 4 years 74. The ages of Nishi and Vinnee are in the ratio of 6:5
(c) 12 years (d) 10 years respectively. After 9 years the ratio of their ages will be
[Based on New Indian Insurance PO, 2009]
9:8. What is the difference in their ages?
(a) 9 years (b) 7 years
67. The ages of Bhakti and Neil are in the ratio of 8:7
(c) 5 years (d) 3 years
respectively. After 6 years, the ratio of their ages will be
19:17. What is the difference in their ages? [Based on SBI PO, 2008]

(a) 4 years (b) 8 years 75. The total of the ages of a class of 75 girls is 1050, the
(c) 10 years (d) 12 years average age of 25 of them is 12 years and that of another
25 is 16 years. Find the average age of the remaining girls.
[Based on Haryana Grameen Bank PO, 2009]
(a) 12 years (b) 13 years
68.  The ages of Sulekha and Arunima are in the ratio of 9:8
(c) 14 years (d) 15 years
respectively. After 5 years the ratio of their ages will be
[Based on SBI PO, 2008]
10:9. What is the difference in years between their ages?
(a) 4 years (b) 5 years 1
76. Michelle got married 9 years ago. Today her age is 1
(c) 6 years (d) 7 years 3
[Based on Andhra Bank PO, 2008] times her age at the time of marriage. At present her daughter’s
age is one-sixth of her age. What was her daughter’s age
69. The present ages of Amit and his father are in the ratio two years ago?
of 2:5, respectively. Four years hence the ratio of the
their ages will become 5:11, respectively. What was the (a) 6 years (b) 7 years
father’s age five years ago? (c) 3 years (d) None of the above
(a) 40 years (b) 45 years [Based on Dena Bank PO, 2008]

(c) 30 years (d) 35 years 77. The respective ratio between the present ages of Ram and
[Based on Andhra Bank PO, 2009] Rakesh is 6:11. Four years ago, the ratio of their ages was
1:2, respectively. What will be Rakesh’s age after five
70. The ages of Ranjana and Rakhi are in the ratio of 15:17 years?
respectively. After 6 years the ratio of their ages will be
(a) 45 years (b) 29 years
9:10. What will be the age of Ranjana after 6 years?
(c) 49 years (d) Cannot be determined
(a) 40 years (b) 30 years
[Based on Corporation Bank PO, 2011]
(c) 34 years (d) 36 years
78. If twice the son’s age in years be added to the father’s age,
[Based on Uttrakhand GBO PO, 2007]
the sum is 70 and if twice the father’s age is added to the
71. The respective ratio between the present age of Manisha son’s age, the sum is 95. Father’s age is
and Deepali is 5:x. Manisha is 9 years younger than (a) 40 years (b) 35 years
Parineeta. Parineeta’s age after 9 years will be 33 years.
(c) 42 years (d) 45 years
The difference between Deepali’s and Manisha’s age is
same as the present age of Parineeta. What will come in 79.
Sneh’s age is one-sixth of her father’s age. Sneh’s father’s
place of x? age will be twice of Vimal’s age after 10 years. If Vimal’s
(a) 23 (b) 39 eighth birthday was celebrated 2 years before, then what
is Sneh’s present age?
(c) 15 (d) None of these
(a) 30 years (b) 24 years
[Based on IBPS Bank PO, 2011]
(c) 6 years (d) None of these
72. The ages of Shirish and Kunder are in the ratio of 5:6 [Based on NMAT, 2005]
respectively. After 8 years the ratio of their ages will be
7:8. What is the difference in their ages? 80. A man’s age is 125% of what it was 10 years ago, but
(a) 4 years (b) 8 years 1
83 % of what it will be after ten 10 years. What is his
(c) 10 years (d) 12 years 3
[Based on OBC PO, 2009] present age?

Chapter_16.indd 371 1/30/2016 4:57:41 PM


372  Chapter 16

(a) 45 years (b) 50 years of the present ages of all of them is 46 years. What is
(c) 55 years (d) 60 years the difference between the Ram’s son’s present age and
Ram’s father’s present age?
81. The age of a person is equal to 4 times the sum of the (a) 68 years (b) 88 years
ages of her three daughters. 8 years hence her age will be (c) 58 years (d) None of these
double the sum of their ages. What is her age now? [Based on Bank of Baroda PO, 2010]
(a) 20 years (b) 40 years 89. At present, Meena is eight times her daughter’s age.
(c) 60 years (d) 80 years Eight years from now, the ratio of the ages of Meena and
[Based on ATMA, 2005] her daughter will be 10:3 respectively. What is Meena’s
82.
If 1 is added to the age of the elder sister, then the ratio of present age?
the ages of two sisters become 0.5:1, but if 2 is subtracted (a) 32 years (b) 40 years
from the age of the younger one, the ratio becomes 1:3, (c) 36 years (d) Cannot be determined
the age of the younger sister will be? [Based on IDBI PO, 2009]
(a) 9 years (b) 5 years 90. The respective ratio of the ages of Anubha and her mother
(c) 18 years (d) 15 years is 1:2. After 6 years the ratio of their ages will be 11:20. 9
[Based on ATMA, 2008] years before, what was the respective ratio of their ages?
83.
The sum of the reciprocals of the ages of two colleagues (a) 3:5 (b) 2:7
is five times the difference of the reciprocals of their ages. (c) 1:4 (d) 2:5
If the ratio of the product of their ages to the sum of their [Based on Syndicate Bank PO, 2010]
ages is 14.4:1, the age (in years) of one of the colleagues 91. The ratio of the age of Tina and Rakesh is 9:10,
must be between (both inclusive) respectively. Ten years ago the ratio of their ages was 4:5,
(a) 20 and 23 (b) 23 and 26 respectively. What is the present age of Rakesh?
(c) 26 and 30 (d) 30 and 35 (a) 25 years (b) 20 years
[Based on ATMA, 2008] (c) 30 years (d) 24 years
84. The respective ratio between the present ages of Ram [Based on Bank of Baroda PO, 2010]
Rohan and Raj is 3:4:5. If the average of their present ages 92. The ages of Ram and Shyam differ by 16 years. Six years
is 28 years then what would be the sum of the ages of Ram ago, Mohan’s age was thrice as that of Ram’s, find their
and Rohan together after 5 years? present ages.
(a) 45 years (b) 55 years (a) 14 years, 30 years (b) 12 years, 28 years
(c) 52 years (d) 59 years (c) 16 years, 34 years (d) 18 years, 38 years
[Based on Bank of Baroda PO Exam, 2011]
93.
The average age of women and child workers in factory
1 was 15 years. The average age of all the 16 children was
85. The age of Mr. Chetan in 2002 was of his birth year. 8 years and the average age of women workers was 22
90
What is his age in 2006? years. If ten women workers were married the num­ber of
(a) 30 (b) 28 unmarried women workers is
(c) 26 (d) 22 (a) 16 (b) 12
[Based on JMET, 2006] (c) 8 (d) 6
[Based on U.P. P.C.S., 2012]
86. 15 years hence, Rohit will be just four times as old as he
was 15 years ago. How old is Rohit at present? 94.
The age of a father is three times of that of his son. After 5
(a) 20 (b) 25 years, the double of father’s age will be five times the age
of son. The present age of father and son is
(c) 30 (d) 35
(a) 30 years, 10 years (b) 36 years, 12 years
87. The present ages of Vishal and Shekhar are in the ratio (c) 42 years, 14 years (d) 45 years, 15 years
of 14:17 respectively. Six-years from now, their ages will [Based on U.P. P.C.S., 2012]
be in the ratio of 17:20 respectively. What is Shekhar’s
present age ? 95.
In a family, mother’s age is twice that of daughter’s age.
Father is 10 years older than mother. Brother 20 years
(a) 17 years (b) 51 years younger than his mother and 5 years older than his sister.
(c) 34 years (d) 28 years What is the age of the father?
[Based on Bank of India PO, 2010] (a) 62 years (b) 60 years
88. Ram’s present age is three times his son’s present age (c) 58 years (d) 55 years
and two-fifth of his father’s present age. The average [Based on SSC (GL), 2011]

Chapter_16.indd 372 1/30/2016 4:57:42 PM


 Problems on Ages  373

96.
The ratio of the ages of Ram and Rahim 10 years ago was Devan and he is 7 years junior to him. So what is the age
1:3. The ratio of their ages five years hence will be 2:3. difference between Devan and Shan?
Then the ratio of their present ages is (a) 18 years (b) 15 years
(a) 1:2 (b) 3:5 (c) 13 years (d) 7 years
(c) 3:4 (d) 2:5 [Based on SSC (GL), 2011]
[Based on SSC (GL), 2011]
99. Ten years ago, Mohan was thrice as old as Ram was but
97.
The average age of 11 players of a cricket team is increased 10 years hence, he will be only twice as old. Find Mohan’s
by 2 months when two of them aged 18 years and 20 years present age.
are replaced by two new players. The average age of the (a) 60 years (b) 80 years
new players is (c) 70 years (d) 76 years
(a) 19 years 1 month (b) 19 years 6 months
100. A man is 3 years older than his wife and four times as old
(c) 19 years 11 months (d) 19 years 5 months
as his son. If the son becomes 15 years old after 3 years,
[Based on SSC (GL), 2011]
what is the present age of the wife?
98.
Shan is 55 years old, Sathian is 5 years junior to Shan and (a) 60 years (b) 51 years
6 years senior to Balan. The youngest brother of Balan is
(c) 48 years (d) 45 years
[Based on SSC (GL), 2010]

EXPLANATORY ANSWERS

1 By the condition given in the problem,


1.
(a) A = B , B + 10 = 2 (C + 10), C = 10
6 x 
x + 5 = 2   5   ⇒  x = 15 years
\ B = 30, A = 5 years 3 
2. (a) Sachin’s age today = 30 years Again, let before t years sister’s age was 6 times that
Sachin’s age 10 years back = 20 years of her brother’s.
Ajay’s age 10 years back = 10 years \ 15 – t = 6 (5 – t)  ⇒  t = 3
Ajay’s age today = 20 years. 9. (c) Let number of boys = x
F+M Let number of girls = y
3.
(c) A = – 20. \ Total number of students = x + y
2
4. (c) Rohan = Mohan – 2 = (Sohan – 3) – 2 = (Rohit + 4) ⇒ (x + y) × 15.8 = 16.4x + 15.4y ⇒ 0.6x = 0.4y
– 3 – 2 = (Mohit + 2) + 4 – 3 – 2 = (Sohit – 3) + 2 + x 0.4 2
⇒ = =
4 – 3 – 2 = (Sohit – 2) y 0.6 3
5. (c) Let the number of students in the two groups be x and 10.
(d) Ratio of ages of A and B = 3:1
y respectively.
Ratio of ages of A and C = 4:1
x 7
\ 20x + 30y = 23 (x + y) ⇒ 3x = 7y ⇒ = . A B C
y 3
3 1
6. (b) (F – 1) = 4 (S – 1) ...(1) 4 1
where F and S are the Father’s and the Son’s ages ------------------------
respectively at present. 12 4 3
\ (F + 6) = 2 (S + 6) + 9 ...(2) 11. (b) Let Namrata’s age = x years
From Eqs. (1) and (2), S = 9. Let Namrata’s father’s age = y years
7.
(d) Amit’s present age is 20. 5 years ago he was 15; \ y = 4x and y + 5 = 3 (x + 5)
therefore his mother was 45. 10 years from now his \ x = 10, y = 10
mother will be 60; Amit will be 30. Hence the ratio of Let y + K = 2 (x + K)
Amit’s age to mother’s is 1:2.
(i.e., After K years, the father will be double of her age)
8.
(b) Let the age of the sister = x years. So, the age of the ⇒ 40 + K = 2 (10 + K)  ⇒  K = 20
x \ After 20 years, Namrata’s father will be double of
brother = .
3 her age.

Chapter_16.indd 373 1/30/2016 4:57:43 PM


374  Chapter 16

12. (b) F = 2S, F – 20 = 12 (S – 20) 19.


(d) Let the present age of Dennis and his father be x and y
⇒ 2S – 20 = 12S – 240 ⇒ S = 22 respectively. Then
\ F = 44. 1
x = y ...(1)
3
13. (c) Let the initial average age of the class be x years.
Then, from the conditions given, the age of the new 1
and (x – 5) = ( y  5) ...(2)
boy and the teacher is x and 8 + 2x years respectively. 4
15 x  8  3 x From Eqs. (1) and (2), y = 45 years
\ = 1.1x  ⇒  x = 11.4 years
17 Hence, required age = (x + 5) = 50 years
14.
(b) Let the present age of the persons be y years. The 20. (a) Let the present age of the man = x years
given data can be written as: \ (x + 15) = 4 (x – 15)
1 ⇒ 3x = 75  ⇒  x = 25
(y + k) = (y + k) and ...(1)
2
21. (c) Given F = M + 5, M = 3D, D = 10.
(y + p) = 3 (y – p) ...(2)
\ M = 3 × 10 = 30 and F = 30 + 5 = 35.
yk 2 \ The father’s age when daughter was born = 35 – 10
From Eq. (1), =
y–k 1 = 25 years
y 3 22. (a) Total age of the mother and six children = 12 × 7 = 84
⇒ = ...(3)
k 1 years.
(by componendo and dividendo) Total age of six children = 7 × 6 = 42 years.
y p 3 \ Mother is 42 years old.
From Eq. (2), = ;
y–k 1 23.
(a) Let the present age of father be x years and the present
age of son be y years.
y 4
⇒ = = 2 ...(4) \ x = 3y ...(1)
p 2
Also, (x + 20) = (y + 20 + 20) ...(2)
Dividing Eq. (4) by Eq. (3), x
Solving Eqs. (1) and (2), we get
R 3x x = 30 years
15.
(c) =
D 5x 24.
(b) Let A’s age be x years
R 3 x  10 5 B’s age be 2x years
⇒ 
D 5 x  10 7 C’s age = (x + 17) years
⇒ x = 5 According to the question,
R 15 x + 2x + (x + 17) = 185
Hence, =
D 25 \ 4x = 185 - 17 = 168  \  x = 42
\ A’s age = 42 years
16. (c) 10 years age, total age of 8 members = 231. After
there years, sum of the ages = 231 + 8 × 3 – 60 = 195 B’s age = 84 years
There more years later sum of ages C’s age = 42 + 17 = 59 years
= 198 + 8 × 3 – 60 = 169 25. (c) Let the present ages of father and son be x and y years,
191 respectively
Current average age = ≈ 24 years Then, (x - 1) = 4(y - 1)
8
17. (a) or 4y - x = 3 ...(1)
18. (c) A:B = 8:9 and (x + 6) - 2(y + 6) = 9
B:C = 2:3 or -2y + x = 15 ...(2)
C:D = 9:13 Solving Eqs. (1) and (2), we get, x = 33, y = 9
A:B:C:D = 144x:162x:243x:351x \ Ratio of their ages = 33:9 = 11:3
But we need not solve this, since we already know 26. (b) Let father’s present age = x years
that B:C = 2x:3x Then, son’s present age = (45 - x) years
\ 2x:3x : : 18:K ⇒ K = 27 years Given: (x - 5)(45 - x - 5) = 4(x - 5)
y k 2 or, x2 - 41x + 180 = 0  or,  (x - 36)(x - 5) = 0
× = ⇒ k:p = 2:3
p y 3 \ x = 36 years

Chapter_16.indd 374 1/30/2016 4:57:45 PM


 Problems on Ages  375

27.
(b) 34.
(b) Let the present ages of Sarita and Gauri are x and y.
28.
(c) Let Mr Sohanlal’s age (in years) = x Then xy = 320
and his son’s age = y and (x + 8) = 3(y + 8)
Then, x - 5 = 3 (y – 5) ⇒ x – 3y = 16
i.e., x - 3y + 10 = 0  320 
⇒ x – 3  = 16
and, x + 10 = 2 ( y + 10)  x 
i.e., x - 2y - 10 = 0 ⇒ x2 – 16x – 960 = 0
Solving the two equations, we get ⇒ (x – 40) (x + 24) = 0
x = 50, y = 20 ⇒ x = 40 and y = 8
R−6 At the time of Gauri born, the age of Sarita is 32 years.
29.
(c) =A
18 35.
(c) Let the present age of Anil’s father be x years.
Given Mahesh = 5 years x
\ Anup = 3 years Then Anil’s present age = years
4
\ R = 18 × 3 + 6 = 60 years
x 1
30.
(c) Let the ages of two sisters be x and y. \ + 16 = (x + 16)
4 2
x 0.5
= x x
y +1 1 ⇒ + 16 = + 8
4 2
⇒ 2x = y + 1 ...(1)
x
x−2 1 ⇒ =8
and = 4
y 3
⇒ x = 32 years
⇒ 3x – 6 = y ...(2)
36.
(b) Present age of Honey = 30 years
From Eqs. (1) and (2),
Honey’s age 10 years ago = 20 years
x = 5 and y = 9
\ Vani’s age 10 years ago = 10 years
So, their ages are 9 years and 5 years.
\ Present age of Vani = 20 years
31.
(b)
37.
(b) Let the present age of father be x years.
32.
(c) Let the present age of father and son be 7x and x years,
respectively. x
\ Present age of son = years
After 4 years, 3
age of father = (7x + 4) years x 
x – 5 = 5 ×  − 5 
age of son = (x + 4) years 3 
7x + 4 4 5x
Given, = ⇒ x – 5 = − 25
x+4 1 3

⇒ 7x + 4 = 4x + 16
2x
⇒ 3x = 12 ⇒ = 20
3
\ x = 4 ⇒ x = 30 years
\ 7x + x = 28 + 4 = 32 years
38.
(a) Sonu = Manu – 4
33.
(c) Let the age of boy be x years.
Dolly = Sumit – 4
\ 2x2 – 25x = 3 × 14
1
⇒ 2x2 – 25x – 42 = 0 Dolly = Sonu
5
25 ± 625 + 336 Sumit = 8 years, Dolly = 4 years, Sonu = 20 years and
\ x =
2×2 Manu = 24 years
Manu = 6 × Dolly
25 ± 961 25 ± 31
= =
4 4 39.
(d) 2Q + P + R = 59 …(1)
25 + 31 56 Q + R + 3P = 68 ...(2)
x = = = 14 yr P + 3Q + 3R = 108 ...(3)
4 4

Chapter_16.indd 375 1/30/2016 4:57:48 PM


376  Chapter 16

From Eqs. (2) and (3), 2EY


3Q + 3R + 9P = 204 And = 6 ⇒ E + Y = 16 …(3)
E +Y
P + 3Q + 3R = 108 Now, (E – Y)2 = (E + Y)2 – 4EY
⇒ 8P = 96 = (16)2 – 4 × 48
\ P = 12 years = 256 – 192 = 64
40.
(a) Let the father’s age be x years and age of his children ⇒ E – Y = 8 ...(4)
be a and b years. From Eqs. (3) and (4), E =12
x and Y = 4
\ (a + b) =
3 From Eq. (1),
and (a + b) + 20 + 20 = x + 20 F = 2 × 12 + 4 × 4 = 40 years
x 46.
(c) Let the age of man’s son be x years.
⇒ + 20 = x
3 \ age of the man = 3x years.
⇒ x = 30 years 15 years ago, age of the son = x – 15 years and age of
41.
(d) Present age of Mahesh = 5 years the man = (3x – 15) years
Present age of Anup = 3 years Now, according to the question,
Present age of Randheer = 3 × 18 + 6 = 60 years 3 x − 15
=9
42.
(d) Let present age of mother and son be x and y years x − 15

respectively. or, 3x – 15 = 9x – 135
Then, x – 1 = 4(y – 1) or, 6x = 120
⇒ x = 4y – 3 …(1) \ x = 20 years
and x + 6 = 2(y + 6) + 5 \ age of the man is; 20 × 3 = 60 years
⇒ x = 2y + 11 ...(2) \ age of the man after 15 years
From Eqs. (1) and (2), = 60 + 15 = 75 years
4y – 3 = 2y + 11 47.
(d) Let the age of Ashu at present be x years and her
14 mother be y years.
⇒ y = = 7 years Now, according to the question,
2
and x = 25 years 3(x – 5) = (y – 5)
\ Required ratio = 25:7 or, 3x – 15 = y – 5
or, 3x – y = 10 ....(1)
43.
(d) Let present age of husband, his wife and son be x, y
and z respectively. Again, according to the question,
According to the given condition, 2(x + 5) = (y + 5)
( x − 3) + ( y − 3) + ( z − 3) or, 2x + 10 = y + 5
= 27 or, 2x – y = – 5 ...(2)
3
Subtracting Eq. (2) from (1), we get
⇒ x + y + z = 90 …(1)
x = 15
( y − 5) + ( z − 5)
and = 20 Hence, Ashu’s today’s age is 15 years.
2
⇒ y + z = 50 …(2) 48.
(c) Age of mother = 3 × 10 = 30 years
From Eqs. (1) and (2), we get Age of father = 30 + 5 = 35 years.
Age of father when the daughter was born
x = 90 – 50 = 40 years
= 35 – 10 = 25 years.
44.
(a) Total age decreases = 20 × 2 = 40 months
49.
(a) Let the present age of the son = x years
= 3 years 4 months
Now, according to the question,
\ The age of new boy = 18 years – 3 years 4 months
x = 38 – x or, x = 19 years.
= 14 years 8 months
Five years back son’s age = 19 – 5 = 14 years.
45.
(c) F = 2E + 4Y ...(1)
50.
(a) Let age of the man = x years
and EY = 4 3 \ x + 15 = 4(x – 15)
⇒ EY = 48 ...(2) ⇒ x = 25

Chapter_16.indd 376 1/30/2016 4:57:49 PM


 Problems on Ages  377

51.
(c) Suppose original strength = x 56.
(c) According to question
\ Total age of adult class = 40x years x  40  12  32
= 36
Average age of 12 new students = 32 years x  12
\ Total age of 12 new students = 32 × 12 ⇒ x = 12
= 384 years 57. (c) Let the present ages of Ashok and Pradeep be 4x
According to the question, and 3x
40 x + 384 So that 4x + 6 = 26 ⇒ x = 5
= 40 – 4 = 36
x + 12 \ Present age of  Pradeep is 3x = 3 × 5, i.e., 15 years

or 40x + 384 = 36x + 432 58.
(a) 11x – 3x = 24
or 4x = 48 or x = 12. ⇒ 8x = 24
52.
(b) Suppose the present ages of father and son are 5x, ⇒ x = 3
x years respectively. Present age = 9, 33
According to the question, After 3 years = 12, 36
(5x + 4) + (x + 4) = 44 \ Ratio = 1:3
or 6x + 8 = 44 or 6x = 36 59.
(a) The man was born between 1800 and 1850 and he was
\ x = 6 x years old in the years x2.
Hence, present age of son = x = 6 years. Now, we can conclude that the years, when he was
x years old must be a perfect square. Now, the only
53.
(c) Let the age of son be x years
perfect square in between 1800 and 1900 is 1849, i.e.,
\ Father’s age = 4x years 432. So, he was 43 years old in the years 1849, which
5 years back age of son = x – 5 and age of father means he was born in (1849 – 43) = 1806.
= 4x – 5 60. (d) Suppose the ages of father and son are 5x and
Now, according to the question, 2x years.
4x − 5 After four years the age of son = 2x + 4
=7
x−5 After four years the age of mother = 4x + 8

⇒ x = 10 years So the present age of mother = 4x + 4
\ Father’s age now is 10 × 4 = 40 years. Ratio of the age of father and mother = 5x:(4x + 4)
Data are insufficient, so cannot be determined.
54.
(c) Sushil Snehal
61. (d) Present age of Radha = x years
3 x x
According to question,
5
(3 x  12)  ( x  12) x + 3 = 2(x – 12)
3
x + 3 = 2x – 24
4x = 24 ⇒ x = 6 x = 27
Present age = 6 + 6 = 12 years Present age of Raj:Present age of Radha = 4:9
55.
(c) Total age, 28 × 11 = 308 27
\ Present age of Raj = ×4 = 12
3 × 25 = 75 9
3 × 28 = 84 After 5 years age of Raj = 12 + 5 = 17 years
3 × 30 = 90 S +6 6
—–– 62. (b) =
249 T +6 7
Total age of a group = 75 + 84 + 90 = 249 7S + 42 = 6T + 36
Difference of captain and youngest players = 308 –  S 4
7S – 6T = 36 – 42  = 
249 = 59  T 5
x – y = 11 4T
7× – 6T = –6
x + y = 59 S
—–———–
2x = 70 28T − 30T
x = 35 = –6
5
\ Captain age = 35 Years –2T = –30

Chapter_16.indd 377 1/30/2016 4:57:51 PM


378  Chapter 16

4T 4 × 15 x = 4
\ T = 15 then S = = = 12 Age difference between Melwyn and Louis
5 5
Difference = 15 – 12 = 3 years = 10x – 7x
63. (c) Age before 4 years = 3x = 3 × 4
Shyam:Ram = 3:4 = 12 years [Putting, x = 4]
After 4 years, age is 67. (a) Suppose the ages of Bhaki and Neil are 8x and 7x
3x + 8 5 respectively.
= After 6 years
4x + 8 6
8x + 6 19
20x + 40 = 18x + 48 =
7x + 6 17
20x – 18x = 48 – 40
2x = 8 ⇒
136x + 102 = 133x + 114
x = 4 ⇒
136x – 133x = 114 – 102
⇒ 3x = 12
13 x − 4 11
64. (d) = 12
17 x − 4 15 ⇒ x = =4
3
195x – 60 = 187x – 44
Age of Bhakti = 8x = 8 × 4 = 32
195x – 187x = –44 + 60
Present age of Neil = 7 × x = 7 × 4 = 28
8x = 16
Required difference = 32 – 28 = 4 years
x = 2
13  2  6 68.
(b) Suppose the age of Sulekha and Arunima is 9x and 8x.
Ratio of their ages after 6 yr = 9x + 5 10
17  2  6 \ =
8x + 5 9
32 4
=  = 4:5 81x + 45 = 8x + 50
40 5
81x – 80x = 50 – 45
65. (b) Let the Bina’s present age = x and Arti’s present x = 5
age = y \ Difference = 9 × 5 – 8 × 5
Then (x – 5) = 3(y – 5) = 45 – 40 = 5 years
x – 5 = 3y – 15 …(1)
69. (d) Suppose the present age of Amit and his father is 2x
1 and 5x years respectively.
Again, x – 10 = (c – 10)
2 After 4 years the ratio of their ages,
1 2x + 4 5
x = (c – 10) + 10 …(2) =
2 5x + 4 11
On putting value of x in Eq. (1) 25x + 20 = 22x + 44
1 25x – 22x = 44 – 20
(c – 10) + 10 – 5 = 3y – 15
2 3x = 24
c − 10 + 10 x = 8
⇒ + 15 = 3y
2 Age of his father before 5 years
c = 5x – 5
⇒ + 15 = 3y
2 = 5 × 8 – 5 [Q x = 8]
c = 40 – 5
⇒ y = +5
6 = 35 years
66. (c) Suppose the present age of Melwyn and Louis are 7x 70.
(d) Suppose the age of Ranjana and Rakhi is 15x years
and 10x. After 6 years the age is and 17x years.
7x + 6 17 After 6 years the age of Ranjana and Rakhi
=
10 x + 6 23 15 x + 6 9
=
⇒ 170x + 102 = 161x + 138 17 x + 6 10
⇒ 170x + 161x + 138 – 102 ⇒ 153x + 54 = 150x + 60
⇒ 9x = 36 153x – 150x = 60 – 54

Chapter_16.indd 378 1/30/2016 4:57:54 PM


 Problems on Ages  379

3x = 6x = 2 75. (c) Average age of the remaining grils


So, the age of Ranjana after 6 years 1050 − (25 × 12 + 25 × 16)
=
= 15 × 2 + 6 = 30 + 6 = 36 years 25
71.
(d) Given Parineeta’s age after 9 years = 33 years 1050 − (300 + 400)
=
\ Parineeta’s present age = 33 – 9 25
= 24 years 1050 − 700
=
\ Manisha’s present age = 24 – 9 25
= 15 years = 14 years
\ Deepali’s present age = 15 + 24 76. (d) Suppose at the time of marriage, the age of Michelle
= 39 years was x years.
Hence, ratio between Manisha and Deepali 4
x + 9 = x
= 15:39 = 5:13 3
\ x = 13 x
\ = 9
72.
(a) Suppose the age of Shirish = 5x years 3
⇒ x = 3 × 9 = 27 years
age of Kunder = 6x years
\ Present age of Michelle
5x + 8 7
= = 27 + 9 = 36 years
6x + 8 8
⇒ 42x + 56 = 40x + 64 Here daughter’s age two years ago
42x – 40x = 64 – 56 36
= − 2 = 6 – 2 = 4 years
2x = 8 6
x = 4 x 6
77. (c) Let the age of Ram = x and Rakesh = y then =
Required difference = 6x – 5x y 11
= 6 × 4 – 5 × 4 6y
\ x =
= 24 – 20 11
= 4 years According to question,
A 4 5 x−4 1
73.
(b) Given =   or,  B = A =
B 5 4 y−4 2
and B - (A + 5) = 3  or,  B = A + 8 2x – 8 = y – 4
5 6y
\ A = A + 8 2× − 8 = y – 4
4 11
5  12 y
or A   1 = 8 − y = –4 + 8
4  11
\ A = 32 years y
= 4
5 11
and B = × 32 = 40 years \ y = 44 years
4
\ A + B = 40 + 32 = 72 years \ Age of Rakesh after 5 years
= 44 + 5 = 49 years
74. (d) Suppose the ages of Nishi and Vinnee and 6x and
78.
(a) Let son’s age (in years) = x and father’s age (in years)
5x years.
=y
6x + 9 9
Q = Given: 2x + y = 70  and, x + 2y = 95
5x + 9 8
Solving for y, we get y = 40
⇒ 48x + 72 = 45x + 81
⇒ 48x – 45x = 81 – 72 79. (d) Vimal’s present age = 8 + 2 = 10
⇒ 3x = 9 Father’s age after 10 years = (10 + 10) × 2 = 40 years
Q x = 3 Father’s present age = 40 – 10 = 30 years
Required difference 1
\ Sneh’s present age = × 30 = 5 years
6x – 5x = x = 3 years 6

Chapter_16.indd 379 1/30/2016 4:57:56 PM


380  Chapter 16

80.
(b) Let the present age be x years. Again by question
Then, 125% of (x - 10) = x xy 14.4 144
= =
1 x+ y 1 10
and, 83 % of (x + 10) = x
3
1 xy 72
\ 125 % of (x - 10) = 83 % of (x + 10) ⇒ =
3 x+ y 5
⇒ 5xy = 72 (x + y) ...(1)
5 5
or,  (x - 10) = (x + 10) 3
4 6 By Eq. (1), put y = x , we have
2
5 5 50 50
or, x - x = + 3 3
4 6 6 4 5x ⋅ x = 72 ( x + x)
2 2
5x 250
or, = 15 2 5
12 12 ⇒ x = 72 × x
or, x = 50 years 2 2
81. (d) Let age of 3 girls = x 72 × 5
⇒ x = = 24 years
\ Age of person = 4x + 8 15
Also 2(x + 24) = 4x + 8 i.e., age of one of colleagues lies between 23 and 26
\ x = 20 years.
\ Age of person = 4x = 4 × 20 = 80 years 84. (d) Let the ages of Ram, Rohan and Raj be 3x, 4x and 5x
respectively.
82.
(b) Suppose that age of age of elder sister be x years and
younger sister be y years. Then, Then,
y 3x + 4 x + 5 x

0.5 1 = 28
= = 3
x +1 1 2
⇒ 4x = 28
⇒ 2y = x + 1
28
⇒ x – 2y = – 1 ... (1) ⇒ x = = 7 years
4
y−2 1
Again given, = So, the present ages of Ram and Rohan together
x 3
= 3x + 4x
⇒ 3y – 6 = x
= 7x = 7 × 7
⇒ x – 3y = – 6 ... (2)
= 49 years
After subtracting Eq. (2) from Eq. (1), we get
Hence, the sum of ages of Ram and Rohan together
y = 5 after 5 years
Put this value in Eq. (1), we have = 49 + 5 × 2
x – 10 = – 1 ⇒ x = 10 – 1 = 49 + 10
x = 9 = 59 years
So, the age of younger sister is 5 years. 85. (c) Let age of Mr. Chetan in 2002 be x.
83.
(b) Suppose that age of two colleagues be x years and y Then, his birth years = 2002 – x
years. 2002 − x
According to question, x = ⇒ x = 22
By question, 90
1 1 1 1 So, his age in 2006 = 22 + 4 = 26
+ = 5  − 
x y x y 86.
(b) Let the present age of Rohit be x years
y+x  y− x Then, given: x + 15 = 4 (x - 15) ⇒ x = 25
⇒ = 5  
xy  xy  V 14
87. (c) =
⇒ y + x = 5y – 5x S 17
⇒ 6x – 4y = 0 14 S
\ V =
⇒ 3x – 2y = 0 17
3 V +6 17
\ y = x … (1) Again, =
2 S +6 20

Chapter_16.indd 380 1/30/2016 4:58:01 PM


 Problems on Ages  381

20V + 120 = 17S + 102 Before 9 years ratio of Anubha and her mother
14 S 27 − 9 18
20 × + 120 = 17S + 102 = = = 2:5
17 27 × 2 − 9 45
280 S 91. (b) Suppose age of Tina and Rakesh is 9x and 10x.
+ 120 = 17S + 102
17 9 x − 10 4
=
17 S − 280 S 10 x − 10 5
120 – 120 = 45x – 50 = 40x – 40
17
5x = 10
289 S − 280 S
18 = x = 2
17
\ Present age of Rakesh
18 × 17 = 10x = 10 × 2 = 20 years
= S
9
34 = S 92. (a) Let Ram’s age = x years
So, Mohan’s age = ( x + 16 ) years
88. (d) Suppose age of Ram = R
Also, 3 ( x - 6 ) = x + 16 - 6  or,  x = 14
His son’s age = S
\ Ram’s age = 14 years
and his father’s age = F
R 2 and, Mohan’s age = 14 + 16 = 30 years
According to question, S = and R = F ×
3 5 93.
(d) Let unmarried women workers are x, then as per
5R question,
\ F = 16 × 8 + 22 × (10 + x)
2 = 15
16 + 10 + x
RSF
and = 46 ⇒ 128 + 220 + 22x = 390 + 15x
3 ⇒ 7x = 42
R + S + F = 46 × 3
\ x = 6
R 5R
R+  = 138 94.
(d) Let present age of son is x years and then present age
3 2
R = 36 of father is 3x years then,
36 5 (x + 5) = 2 (3x + 5)
S = = 12 ⇒ 5x + 25 = 6x + 10
3
\ x = 15 years
5  36
F = = 90 Present age of father
2
= 45 years.
Difference = 90 – 12 = 78 years
95.
(b) Let the age of the daughter be x.
89. (a) Suppose the age of daughter = x years Then, age of brother
Age of Meena = 8x years = x + 5 years
After 8 years Therefore, age of mother
8x + 8 10 = 2x years
=
x+8 3 \ 2x – 20 = x + 5
24x + 24 = 10x + 80 ⇒ 2x – x = 5 + 20
24x – 10x = 80 – 24 ⇒ x = 25 years.
14x = 56 Age of mother = 2x
x = 4 = 2 × 25 = 50 years
So, the age of Meena = 8x = 8 × 4 = 32 years Age of father = 50 + 10 = 60 years
90. (d) Ratio of present age of Anubha and her mother = 1:2 96.
(b) let the age of Ram and Rahim 10 years ago be x and
x+6 11 3x years respectively. Ages of Ram and Rahim after
According to question, =
2x + 6 20 5 years from now,
20x + 120 = 22x + 66 x + 15 2
=
2x = 54 3 x + 15 3
x = 27 ⇒ 2(3x + 15) = 3(x + 15)

Chapter_16.indd 381 1/30/2016 6:14:25 PM


382  Chapter 16

⇒ 6x + 30 = 3x + 45 Then, according to the first condition,


⇒ 6x – 3x = 45 – 30 x - 10 = 3 ( y - 10 )
⇒ 3x = 15 or, x - 3y = –20 ...(1)
15 Now, Mohan’s age after 10 years
⇒ x = = 5 years
3 = (x + 10) years
97.
(c) Total increase = 11 × 2 = 22 months Ram’s age after 10 years = (y + 10)
Therefore, sum of the ages of both cricketers = (18 + \ (x + 10) = 2 (y + 10)
20) years 22 months = 38 years 22 months or, x - 2y = 10 ...(2)
Hence, Average age = 19 years 11 months Solving Eqs. (1) and (2), we get
98.
(a) Shan’s age = 55 years x = 70 and y = 30
Sathian’s age = 50 – 5 \ Mohan’s age = 70 years and Ram’s age = 30 years.
= 50 years 100. (c) Let the present age of the son be x years.
Balan’s age = 50 – 6 = 44 years Therefore, the present age of the father = 4x years
Devan’s age = 44 – 7 = 37 years According to the question x + 3 = 15 years
Difference between Shan’s age and Devan’s age = 55 Therefore, as x = 15 – 3 = 12 years
– 37 = 18 years Hence, the present age of the father = 4x
99.
(c) Let Mohan’s present age be x years and Ram’s present = 4 × 12 = 48 years.
age be y years.

Chapter_16.indd 382 1/30/2016 4:58:04 PM


17 simple interest

INTRODUCTION
When A borrows some money from B, then A has to pay Interest can be of two types:
certain amount to B for the use of this money. This amount 1. Simple Interest
paid by A is called interest. The total amount of money
2. Compound Interest
borrowed by A from B is called the principal. The money
paid back to B, which comprises the principal and the
interest is called the amount. SIMPLE INTEREST
In other words, When the interest is payable on the principal only, it is
Amount = Principal + Interest called simple interest. For example, simple interest on `100
The interest is usually charged according to a specified at 5% per annum will be `5 each year, that is, at the end of
term, which is expressed as some per cent of the principal one year, total amount will be `105. At the end of second
and is called the rate of interest for the fixed period of year, it will be `110 and so on.
time. This fixed period may be a year, six months, three Thus, simple interest is the interest computed on the
months or a month and correspondingly the rate of interest principal for the entire period it is borrowed.
is charged annually, semi-annually, quarterly or monthly. In this chapter, we will limit ourselves to simple
For example, the rate of interest is 5% per annum means the interest. Compound interest will be discussed in the next
interest payable on `100 for one year is `5. chapter.

soMe BAsiC FoRMulAe

If P stands for principal, R is the rate per cent per 100 × Simple Interest
annum, T is the number of years, I is the simple 2. Principal =
Rate × Time
interest and A is the amount, then
100 × I
Principal × Rate × Time or, P =
1. Simple Interest = R ×T
100
P × R ×T Illustration 2 A man earns `450 as an interest in 2 years
or I =
100 on a certain sum invested with a company at the rate of 12
per cent per annum. Find the sum invested by the man in
Illustration 1 Find the simple interest on `5200 for 2 years the company.
at 6% per annum. Solution: We have I = `450, T = 2 years,
Solution: Here P = `5200, T = 2 years and R = 6% R = 12% per annum
P × R ×T 5200 × 6 × 2 I × 100 450 × 100
\ Simple interest = = \ P= = = `1875.
100 100 R ×T 12 × 2
= `624. Thus, the money invested by the man was `1875.

Chapter_17.indd 383 1/30/2016 5:56:05 PM


384 Chapter 17

Solution: Here P = `1200, I = `240, R = 5%


100 × Simple Interest
3. Rate = 100 × I 100 × 240
Principal × Time \ T = = = 4 years.
P×R 1200 × 5
100 × I
or, R =
P ×T
5. Amount = Principal + Simple Interest
Principal × Rate × Time
Illustration 3 At what rate per annum will a sum of `5000
5000 = Principal +
amount to `6000 in 4 years? 100
Solution: Here P = `5000, A = `6000,  Rate × Time 
= Principal 1+ 
T = 4 years  100 
So, I = A – P = `(6000 – 5000) = `1000  R ×T 
or, A = P 1+ 
100 × I  100 
\ R =
P ×T
100 × 1000 Illustration 5 Mahesh borrowed `3000 from his friend
= = 5%.
5000 × 4 Suresh at 15% per annum for 3 years. Find the interest and
money returned by Mahesh to Suresh.
100 × Simple Interest Solution: Here P = `3000, R = 15% per annum, T = 3 years
4. Time =
Rate × Principal
P × R ×T 3000 × 15 × 3
100 × I \ I = = = `1350.
or T = 100 100
R×P
\ A = P + I = `3000 + `1350 = `4350.
Illustration 4 In what time will `1200 earn an interest of Thus, Mahesh paid `1350 as interest to Suresh and the
`240 at 5% per annum? amount returned by Mahesh to Suresh = `4350.

soMe useFul shoRt-Cut MethoDs

Solution: We have, A = `570, R = 4% per annum,


1. If a certain sum in T years at R% per annum
amounts to `A, then the sum will be 7
T= years.
2
100 × A
P = 100 × A 100 × 570
100 + R × T \ P = =
100 + R × T 100 + 4 × 7/2
Explanation
100 × 570
Let the principal be ` x = = `500
114
\ Simple interest = ` (A – x)
x × R ×T Thus, `500 will amount to `570 at 4% per annum in
\ A–x= 1
100 3 years.
⇒ 100 A – 100 x = xRT 2
⇒ (100 + RT) x = 100 A
100 × A 2. The annual payment that will discharge a debt of
\ x= . `A due in T years at R% per annum is
100 + R × T
 100 A 
Illustration 6 What principal will amount to `570 at 4% per Annual payment = ` 
RT (T − 1) 
1  100 T + 
annum in 3 years?  2 
2

Chapter_17.indd 384 1/30/2016 5:56:09 PM


Simple Interest 385

Explanation
3. If a certain sum is invested in n types of
Let the annual payment be `x. investments in such a manner that equal amount
Since the first instalment is paid at the end of first year, is obtained on each investment where interest
rates are R1, R2, R3, ..., Rn, respectively and time
\ Amount of first instalment at the end of t years
periods are T1, T2, T3, ..., Tn, respectively, then the
(T − 1) × R × x ratio in which the amounts are invested is
=x+
100 1 1
: :
Similarly, amount of second instalment at the end of t 100 + R1T1 100 + R2T2
years
(T − 2) × R × x 1 1
=x+ , and so on. : ... .
100 100 + R3T3 100 + RnTn

Thus, total amount of T instalments


Explanation
 (T − 1) × R × x 
A = x +  Let P1, P2, ..., Pn be invested in n types of investments
 100 whose interest rates are R1, R2, ..., Rn and time periods are
 (T − 2) × R × x  T1, T2, ..., Tn.
+ x +  + ... + x
 100 100 × A
Then, P1 =
100 + R1T1
Rx
= Tx + [(T − 1) + (T − 2) + ... + 1] 100 × A
100 P2 =
100 + R2T2
Rx  (T − 1) × T 
= Tx +    
100 2
100 × A
 T (T − 1)  Pn = .
100 Tx + Rx  100 + RnTn
or
 2  = 100 A
\ P1 : P2 : ... : Pn
 RT (T − 1) 
or x 100 T +  = 100 A 100 × A 100 × A 100 × A
 2 = : : ...
100 + R1T1 100 + R2T2 100 + RnTn
100 A
\x = . 1 1 1
RT (T − 1) = : : ...
100 T + 100 + R1T1 100 + R2T2 100 + RnTn
2
[ the amount A is same for all]
Illustration 7 Find the annual instalment that will discharge
a debt of `12900 due in 4 years at 5% per annum simple Illustration 8 A sum of `1586 is divided among three such
interest. parts that amount obtained on these three parts of money
Solution: Here A = `12900, T = 4 years, R = 5% per annum after 2, 3 and 4 years, respectively at the rate of 5% per
annum remains equal. Find such three parts of the sum.
100 × A Solution: Since the amount accured from each of the three
\ Annual instalment =
RT (T − 1) parts of `1586 at the rate of 5% p.a. in 2, 3 and 4 years,
100 × T +
2 respectively, remains equal, such three parts of `1586 will
100 × 12900 be in the ratio of
=
5(4 − 1) × 4 1 1 1
(100 × 4) + : :
2 100 + R1T1 100 + R2T2 100 + R3T3
100 × 12900 100 × 12900 Hence, the ratio
= =
400 + 30 430 1 1 1
= : :
= `3000. 100 + 5 × 2 100 + 5 × 3 100 + 5 × 4

Chapter_17.indd 385 1/30/2016 5:56:13 PM


386 Chapter 17

1 1 1 (2 − 1) × 100
= : : = years
110 115 120 8
1 × 30360 1 × 30360 1 × 30360 1
= : : = 12 years.
110 115 120 2
( L.C.M. of 110, 115 and 120 is 30360)
\ ratio = 276 : 264 : 253 6. If a certain sum of money becomes n times itself
Sum of proportionals = 276 + 264 + 253 = 793 in T years at a simple interest, then the time T in
276 which it will become m times itself is given by
\ 1st part = × 1586 = `552,
793  m −1
T’ =   × T years.
264  n −1 
2nd part = × 1586 = `528
793
253 Explanation
and, 3rd part = × 1586 = `506.
793
Let the principal be `P.
4. If a certain sum of money becomes n times Let it become m times in T ′ years.
itself in T years at simple interest, then the rate of Then, the amount in T years = `nP
interest per annum is
and the amount in T ′ years = `mP.
100(n − 1)
R= % P × R ×T
T \ nP – P =
100
Explanation P × R ×T
or (n – 1) P = ...(1)
Let `P become `nP in t years 100
\ Simple interest I is given by P × R × T'
I = nP – P = (n – 1) P and (m – 1) P = ...(2)
100
\ Rate of interest R is given by
(m − 1) P P × R × T' 100
100 × I 100 × (n − 1) P \ = ×
R = = (n − 1) P 100 P × R ×T
P ×T P ×T
m −1 T'
100 (n − 1) or =
= . n −1 T
T
Illustration 9 A certain sum of money trebles itself in  m −1
\ T′ =   T years.
5 years simple interest. Find the rate per cent per annum.  n −1 
Solution: Here n = 3, T = 5 years
Illustration 11 A sum of money put out on simple interest
100 (n − 1) 100 (3 − 1)
\ R= %= % = 40%. 1
T 5 doubles itself in 12 years. In how many years would it
2
5. If a certain sum of money becomes n times itself treble itself?
at R% per annum simple interest in T years, then 25
Solution: Here n = 2, m = 3, T = years.
 n −1  2
T=   × 100 years.
 R   m −1
\ Required time (T ′) =   × T years
 n −1 
Illustration 10 In what time a sum of money will double
itself at a rate of simple interest of 8% per annum?  3 − 1  25
=  × years
(n − 1) × 100  2 −1 2
Solution: Required time (T) = years
R = 25 years.

Chapter_17.indd 386 1/30/2016 5:56:17 PM


Simple Interest 387

7. Effect of change of P, R and T on simple interest P


We have 80 = × (4 × 4 –5 × 3)
is given by the following formula : 100
Change in Simple Interest ⇒ P = `8000.
Product of fixed parameter
= 8. If a debt of `Z is paid in ‘n’ number of instalments
100
and if the value of each instalment is `a, then the
× [difference of product of variable parameters] borrowed (debt) amount is given by
For example, if rate (R) changes from R1 to R2 and
Ra n (n − 1)
P, T are fixed, then Z = na + ×
100 × b 2
PT
Change in SI = × ( R1 − R2 )
100 where R = rate of interest per annum
Similarly, if principal (P) changes from P1 b = no. of instalments/year
to P2 and R, T are fixed, then change in SI = b = 1, when each instalment is paid yearly
RT b = 2, when each instalment is paid half-
× ( P1 − P2 ) yearly
100
b = 4, when each instalment is paid quarterly
Also, if rate (R) changes from R1 to R2 and time b = 12, when each instalment is paid monthly.
(T) changes from T1 to T2 but principal (P) is
P Illustration 15 A sum of `2 is lent to be paid back in
fixed, then change in SI = × ( R1T1 − R2T2 ) .
100 3 equal monthly instalments of Re. 1 each. Find the rate
per cent.
Illustration 12 If simple interest on `600 increases by Solution: Here Z = `2, a = Re. 1, n = 3, b = 12, R = ?
`30, when the rate % increases by 4% per annum, find the Using the formula
time. Ra n (n − 1)
Z = na + × ,
Solution: Here P = 600, change in SI = 30, R1 – R2 = 4, T 100 × b 2
=?
PT R ×1 3× 2
Using, change in SI = × ( R1 − R2 ) we have 2 = 3 × 1 + × ⇒ R = 400%
100 100 × 12 2
\ The rate % p.a. is 400%.
600 T 5 1
we have 30 = × 4 ⇒ T = , i.e., 1 years.
100 4 4 9. If a certain sum of money P lent out at SI amounts
Illustration 13 If the simple interest on `1400
1400 be more to A1 in T1 years and to A2 in T2 years, then
than the interest on `1000 by `60
60 in 5 years, find the rate A T − A2T1
P = 1 2
per cent per annum. T2 − T1
Solution: Here change in SI = 60, P1 – P2 = 400, T = 5, A1 − A2
R=? and R = × 100%
A1T2 − A2T1
RT
Using change in SI = × ( P1 − P2 )
100
Illustration 16 If a certain sum of money at simple interest
5R amounts to `5184 in 2 years and to `5832 in 3 years, what
We have 60 = × 400 ⇒ R = 3%.
100 is the sum and the rate of interest?
Illustration 14 If the simple interest on a certain sum at A1T2 − A2T1
Solution: Principal =
4% per annum for 4 years is `80 more than the interest on T2 − T1
the same sum for 3 years at 5% per annum, find the sum.
=
 Here A1 5184,
= A2 5832 
Solution: Here change in SI = 80, R1 = 4, R2 = 5, T1 = 4, T1 2,= 
= T2 3
T2 = 3, P = ?
P 5184 × 3 − 5832 × 2
Using change in SI = × ( R1T1 − R2T2 ) = = `3888
100 3−2

Chapter_17.indd 387 1/30/2016 5:56:21 PM


388 Chapter 17

(A2 − A1 ) × 100 (5832 − 5184) × 100 5000 × 2 + 2000 × 4 4


and Rate = = We get R = = 2 %.
T1 A2 − T2 A1 2 × 5832 − 3 × 5184 5000 + 2000 7
64800 2
= = 16 %. 12. If a certain sum of money is lent out in n parts
3888 3
in such a manner that equal sum of money is
10.If a certain sum of money P lent out for a certain obtained as simple interest on each part where
time T amounts to A1 at R1% per annum and to A2 interest rates are R1, R2, ..., Rn, respectively and
at R2% per annum, then time periods are T1, T2, ..., Tn, respectively, then
the ratio in which the sum will be divided in n
A2 R1 − A1R2 parts is given by
P=
R1 − R2
1 1 1
A1 − A2 : :... .
and T= × 100 years. R1T1 R2T2 RnTn
A2 R1 − A1R2
Explanation
Illustration 17 A certain sum is invested for certain time. Let the n equal parts be P1, P2, ..., Pn and let I be the equal
It amounts to `450 at 7% per annum. But when invested at interest earned on each part.
5% per annum, it amounts to `350. Find the sum and time.
I × 100
Solution: Here A1 = 450, R1 = 7, A2 = 350, R2 = 5. THEN, P1 =
R1T1
Using the formula,
A2 R1 − A1R2 I × 100
P = P2 =
R1 − R2 R2T2

350 × 7 − 450 × 5  
We get P = = `100
7−5 I × 100
PN = .
RnTn
Also, using the formula,
I × 100 I × 100 I × 100
 A1 − A2  ∴ P1 : P2 : ... : PN = : : ...
T=   × 100 R1T1 R2T2 RnTn
 A2 R1 − A1R2 
1 1 1
 450 − 350  = : :... .
we get T =   × 100 = 5 years. R1T1 R2T2 RnTn
 350 × 7 − 450 × 5 
Illustration 19 If a sum of `1600 is divided into two such
11. If an amount P1 lent at simple interest rate of parts that the simple interest on the first part for two and half
R1% per annum and another amount P2 at simple years at the rate of 4% p.a. equals the simple interest on the
interest rate of R2% per annum, then the rate of second part for 5 years at the rate of 3% p.a., then find two
interest for the whole sum is such divisions of the sum.
Solution: Ratio of one part to other part of `1600
 P R + P2 R2 
R=  1 1 . 1 1
 P1 + P2  = :
R1T1 R2T2
Illustration 18 Mohan deposits `5000 in NSC at 2% per 1 1
\ 1st part : 2nd part = :
annum and `2000 in mutual funds at 4% per annum. Find 4 × 5/2 3 × 5
the rate of interest for the whole sum.
5
Solution: Here P1 = 5000, R1 = 2, P2 = 2000, R2 = 4. [Here R1 = 4% p.a., T1 = years, R2 = 3% p.a.,
2
Using the formula T2 = 5 years]
 P R + P2 R2  1 1
R =  1 1  or, 1st part : 2nd part = : =3:2
 P1 + P2  10 15

Chapter_17.indd 388 1/30/2016 5:56:25 PM


 Simple Interest  389

Sum of proportionals = 3 + 2 = 5 1
14. Out of a certain sum P, part is invested at R1%,
3 a
\ 1st part = × 1600 = `96
5 1  1 1
part at R2% and the remainder 1 − −  say
2 b  a b
and   2nd part = × 1600 = `640.
5 1
part at R3%. If the annual income from all these
  13. When there is a change in principal (P), Rate (R) c
and Time (T), then the value of simple interest I also investments is `A, then the original sum is given by
changes and is given by
 A × 100 
I1 P × R ×T P=  .
= 1 1 1 R1 R2 R3
I2 P2 × R2 × T2  + + 
 a b c 
A1 − P1 P × R ×T
⇒ = 1 1 1 Illustration 21 Out of a certain sum, one-third is invested at
A2 − P2 P2 × R2 × T2
3%, one-sixth at 6% and the rest at 8%. If the annual income
as I1 = A1 – P1 and I2 = A2 – P2. is `300, then the original sum is
Illustration 20 If `85 amounts to `95 in 3 years, what 1 1 1 1
Solution: Here = , = ,
`102 will amount to in 5 years at the same rate per cent? a 3 b 6
Solution: Here P1 = `85, A1 = `95, T1 = 3 years,
P2 = `102, T2 = 5 years, R1 = R2 = R (say).
1 1 1 1
= 1 –  +  = ,
c 3 6 2
Then, using the formula
R1 = 3%, R2 = 6%, R3 = 8%, A = `300.
A1 − P1 P × R ×T
= 1 1 1 A × 100
A2 − P2 P2 × R2 × T2 \ The original sum =
R1 R2 R3
+ +
95 − 85 85 × R × 3 a b c
We have =
A2 − 102 102 × R × 5 300 × 100 30000
= =
⇒ A2 – 102 = 20 3 6 8 6
+ +
⇒ A2 = 122 3 6 2
\ The amount is `122. = `5000.

Multiple Choice Questions

1. A sum of money invested at simple interest triples itself 1


in 8 years. How many times will it become in 20 years 3. Two equal sums of money were invested, one at 4 %
2
time? and the other at 4%. At the end of 7 years, the simple
(a) 8 times (b) 7 times interest received from the latter exceeded that received
(c) 6 times (d) None of these from the former by `31.50. Each sum was
(a) `1,000 (b) `500
2. Ravi gave `1200 on loan. Some amount he gave at 4%
(c) `750 (d) `900
per annum simple interest and remaining at 5% per annum [Based on MAT, 2001]
simple interest. After two years, he got `110 as interest.
Then the amounts given at 4% and 5% per annum simple 4.
I derive an annual income of `688.25 from `10,000
interest are respectively, invested partly at 8% p.a. and partly at 5% p.a. simple
interest. How much of my money is invested at 5%?
(a) `500, `700 (b) `400, `800
(a) `3,725 (b) `4225
(c) `800, `300 (d) `1100, `1100 (c) `4,800 (d) `5,000
[Based on MAT, 2003] [Based on MAT, 2000]

Chapter_17.indd 389 1/30/2016 5:56:28 PM


390  Chapter 17

5. A certain sum amounts to `2,300 in 3 years and `2,500 13. A sum of `600 amounts to `720 in 4 years at simple
in 5 years at simple interest. Find the sum and the rate of interest. What will it amount to if the rate of interest is
interest. increased by 2%?
(a) `1200, 6% (b) `1800, 5% (a) `648 (b) `768
(c) `2000, 5% (d) `1500, 6% (c) `726 (d) `792
[Based on IIT Joint Man. Ent. Test, 2004]
14. A person takes a loan of `200 at 5% simple interest. He
6.
Mr Anand deposited a total amount of `65000 in three returns `100 at the end of one year. In order to clear his
different schemes A, B and C with rates of interest dues at the end of 2 years, he would pay
12% per annum, 16% per annum and 18% per annum, (a) `125.50 (b) `110
respectively and earned a total interest of `10,180 in one
(c) `115.50 (d) None of these
year. If the amount invested in Scheme A was 72% of the
amount invested in Scheme C, then what was the amount 15. `2,189 are divided into three parts such that interest on
invested in Scheme B? them after 1, 2 and 3 years, respectively may be equal, the
(a) `25000 (b) `22000 rate of simple interest being 4% per annum in all cases.
The smallest part is
(c) `18000 (d) Cannot be determined,
[Based on Based on IRMA, 2002]
(a) `702 (b) `398
(c) `756 (d) `1,093
7. The simple interest on a certain sum at 5% for 9 months
is `10 greater than the simple interest on the same sum at 16. The simple interest on a sum of money is one-ninth of the
the rate of 3% for 14 months. What is the sum of interest sum. The number of years is numerically equal to the rate
in both the cases (i.e., total sum of interest)? per cent per annum. The rate per cent is
(a) `130 (b) `290 1
(a) 3 (b) 5
(c) `120 (d) `330 3
8. In 4 years, the simple interest on a certain sum of money 2
(c) 6 (d) 10
is 7/25 of the principal. The annual rate of interest is 3
(a) 4% (b) 4.5%
17. The rates of simple interest in two banks A and B are in the
(c) 7% (d) 9% ratio 5 : 4. A person wants to deposit his total savings in
[Based on SNAP, 2007] two banks in such a way that he recieves equal half yearly
9. Divide `6,000 into two parts so that simple interst on the interest from both. He should deposit the savings in banks
first part for 2 years at 6% per annum may be equal to A and B in the ratio
simple interest on the second part for 3 years at 8% per (a) 2 : 5 (b) 4 : 5
annum. (c) 5 : 2 (d) 5 : 4
(a) `4,000, `2,000 (b) `5,000, `1,000
18. Some amount was lent at 6% per annum simple interest.
(c) `3,000, `3,000 (d) None of these
After 1 year, `6,800 is repaid and the rest of the amount
10.
`25000 amount of `2600 in 5 years at simple interest. If is repaid at 5% per annum. If the second year’s interest
the interest rate were increased by 3%, it would amount to 11
how much? is   of the first year’s interest, find what amount of
20
(a) `2900 (b) `3200 money was lent out.
(c) `3600 (d) None of these (a) `17,000 (b) `16,800
[Based on I.P. Univ., 2002] (c) `16,500 (d) `17,500
11.
A sum of `4000 is lent out in two parts, one at 8% simple 19. A bicycle can be purchased on cash payment of `1,500.
interest and other at 10% simple interest. If the annual The same bicycle can also be purchased at the down
interest is `352, the sum lent at 8% is payment (initial payment, at the time of purchasing) of
(a) `1600 (b) `2400 `350 and rest can be paid in 3 equal installments of `400
(c) `1800 (d) `2800 for next 3 months. The rate of SI per annum charged by
[Based on MAT, 2005] the dealer is
12. If the rate of simple interest is 12% per annum, the amount 9 9
(a) 23 % (b) 17 %
that would fetch interest of `6,000 per annum is: 17 23
(a) `7,200 (b) `48,000 9
(c) 13 % (d) None of these
(c) `50,000 (d) `72,000 17

Chapter_17.indd 390 1/30/2016 5:56:29 PM


 Simple Interest  391

20. An article costing `9,000 is sold at a discount which is 27. Two equal sums of money are lent at the same time at
equal to the simple interest on `3,000 for N months. Find 8% and 7% per annum simple interest. The former is
N if the rate of discount is same as the rate of interest. recovered 6 months earlier than the later and the amount
(a) 4 months (b) 6 months in each case is `2,560. The sum and the time for which the
sums of money are lent out are
(c) 5 months (d) 36 months
(a) `1,500, 3.5 years and 4 years
21. What should be the least number of years in which the (b) `2,000, 3.5 years and 4 years
2 (c) `2,000, 4 years and 5.5 years
simple interest on `2,600 at 6 % will be an exact
3 (d) `3,000, 4 years and 4.5 years
number of rupees?
(a) 2 (b) 3 28.
Subbarao was approached by two neighbours for loan.
He had `2,540, a part of which he lent to one person at
(c) 4 (d) 5
12% interest per annum, and the other part was lent to the
22. Pratibha invests an amount of `15,860 in the names of her second person at 12.5%. At the end of a year, Subbarao
three daughters A, B and C in such a way that they get the received `311.60 as interest on the total loan. Calculate
same interest after 2, 3 and 4 years, respectively. If the rate the amount of money lent by him at 12% interest.
of simple interest is 5% p.a., then the ratio of the amounts (a) `1,360 (b) `1,340
invested among A, B and C will be (c) `1,240 (d) `1,180
1 1 1 1 1 1 29.
Vikram borrowed `6,450 at 5 per cent simple interest
(a) : : (b) : :
15 10 20 10 15 20 repayable in 4 equal instalments. What will be the annual
1 1 instalment payable by him?
(c) : :10 (d) None of these (a) `1,710 (b) `1,810
15 20
(c) `1,910 (d) `1,860
23. If x is the simple interest on y and y is the simple interest
on z, the rate per cent and the time being the same in both 30. A person closes his account in an investment scheme by
cases, what is the relation between x, y and z? withdrawing `10,000. One year ago, he had withdrawn
(a) x2 = yz (b) y2 = xz `6,000. Two years ago he had withdrawn `5,000. Three
years ago he had not withdrawn any money. How much
(c) z2 = xy (d) xyz = 1 money had he deposited approximately at the time of
24. A sum of `18,750 is left by will by a father to be divided opening the account 4 years ago, if the annual simple
between two sons, 12 and 14 years of age, so that when interest is 10%?
they attain maturity at 18, the amount (principal + interest) (a) `15,600 (b) `16,500
received by each at 5% simple interest will be the same. (c) `17,280 (d) None of these
Find the sum allotted at present to each son. 31. Two equal sums of money were invested, one at 4%
(a) `9,500, `9,250 (b) `8,000, `1,750 1
and the other at 4 %. At the end of 7 years, the simple
(c) `9,000, `9,750 (d) None of these 2
interest received from the latter exceeded that received
25.
A sum of `1,440 is lent out in three parts in such a way
from the former by `31.50. Each sum was
that the interest on first part at 2% for 3 years, second part
at 3% for 4 years and third part at 4% for 5 years are equal. (a) `1,000 (b) `500
Then the difference between the largest and the smallest (c) `750 (d) `900
sum is 32.
The rate of interest on a sum of money is 4% per annum
(a) `400 (b) `560 for the first 2 years, 6% per annum for the next 4 years and
8% per annum for the period beyond 6 years. If the simple
(c) `460 (d) `200 interest accrued by the sum for a total period of 9 years is
26. Arun borrowed a sum of money from Jayant at the rate of `1,120, then the sum is
8% per annum simple interest for the first four years, 10% (a) `2,400 (b) `2,200
per annum for the next 6 years and 12% per annum for the (c) `2,000 (d) None of these.
period beyond 10 years. If he pays a total of `12,160 as
33.
Brinda borrowed `1,000 to build a hut. She pays 5%
interest only at the end of 15 years, how much money did
simple interest. She lets the hut to Ramu and receives the
he borrow?
1
(a) `12,000 (b) `10,000 rent of ` 12 per month from Ramu. In how many years
2
(c) `8,000 (d) `9,000 Brinda ought to clear off the debt?

Chapter_17.indd 391 1/30/2016 5:56:30 PM


392  Chapter 17

1 40.
Prem invested a certain sum of money in a simple interest
(a) 10 years (b) 10 years bond whose value grew to `300 at the end of 3 years and
4
to `400 at the end of another 5 years. What was the rate of
1 interest in which he invested his sum?
(c) 10 years (d) None of these
2
(a) 12% (b) 12.5%
34.
Sumit lent some money to Mohit at 5% per annum simple (c) 6.67% (d) 8.33%
interest. Mohit lent the entire amount to Birju on the same [Based on MAT (Feb), 2010]
1
day at 8 % per annum. In this transaction after a year 41.
A money lender lent out `25000 in two parts, one at 8%
2
and the other at 8.5%. If the total annual income on the
Mohit earned a profit of `350. Find the sum of money lent amount is `2031.25, the money lent at 8% is
by Sumit to Mohit.
(a) `12500 (b) `6250
(a) `9,000 (b) `10,000
(c) `10000 (d) `18750
(c) `10,200 (d) None of these
[Based on MAT (Feb), 2010]
35.
`1,500 is invested at a rate of 10% simple interest and 42.
A man wants to sell his scooter. There are two offers,
interest is added to the principal after every 5 years. In one at `12000 cash and the other at a credit of `12880 to
how many years will it amount to `2,500? be paid after 8 months, money being at 18% per annum.
1 1 Which is the better offer?
(a) 6 years (b) 6 years
9 4 (a) `12000 in cash (b) `12880 at credit
(c) 7 years (d) None of these (c) Both are equal (d) None of these
36.
If `1000 be invested at interest rate of 5% and the interest [Based on MAT (Feb), 2009]
be added to the principal after 10 year, then the number of 43.
A trader owes a merchant `10028 due 1 year, hence
years in which it will amount to `2000 is the trader wants to settle the account after 3 months.
2 1 If the rate of interest is 12% per annum, how much cash
(a) 16 year (b) 16 year
3 4 should he pay?
(c) 16 year (d) 11 year (a) `9025 (b) `9200
[Based on MAT (Sept), 2010] (c) `9600 (d) `9200
37.
A sum of `7700 is to be divided among three brothers [Based on MAT (Feb), 2009]
Sunil, Sumant and Surat in such a way that simple interest 44.
Asmita invests an amount of `9535 at the rate of 4% per
on each part at 5% per annum after 1, 2 and 3 year annum, for how many years did she invest the amount to
respectively remains equal. The share of Sunil is more obtain the double her sum?
than that of Surat by
(a) 10 years (b) 25 years
(a) `2800 (b) `2500
(c) 5 years (d) 4 years
(c) `3000 (d) `2700
[Based on MAT (May), 2009]
[Based on MAT (Sept), 2010]
45.
Anu owes Biresh `1120 payable 2 years hence, Biresh
38.
A person invested some amount at the rate of 12% simple
owes Anu `1081.50 payable 6 months. If they decide to
interest and a certain amount at the rate of 10% simple
settle their accounts forthwith by payment of ready money
interest. He received yearly interest of `130. But if he
and the rate of interest be 6% per annum, then who should
had interchanged the amounts invested, he would have
pay and how much?
received `4 more as interest. How much did he invest at
12% simple interest? (a) Anu, `70 (b) Biresh, `50
(a) `700 (b) `500 (c) Anu, `50 (d) Biresh, `70
(c) `800 (d) `400 [Based on MAT (Sept), 2008]
[Based on MAT (Feb), 2010] 46.
The present worth of bill due 7 months, hence is `1200.
39.
A sum was put at simple interest at a certain rate for 3 If the bill were due at the end of two and half years its
years. Had it been put at 1% higher rate, it would have present worth would be `1016. The rate per cent of the
fetched `5100 more. The sum is bill is
(a) `170000 (b) `150000 (a) 16% (b) 8%
(c) `125000 (d) `120000 (c) 10% (d) 18%
[Based on MAT (Feb), 2010] [Based on MAT (Sept), 2008]

Chapter_17.indd 392 1/30/2016 5:56:31 PM


 Simple Interest  393

47.
A owes B `1573, payable one and half years hence. Also 53.
What price should a shopkeeper mark on an article costing
B owes A `1444.50, payable 6 months hence. If they want him `153 to gain 20% after allowing a discount of 15%?
to settle the account forthwith, keeping 14% as the rate of (a) `162 (b) `184
interest, then who should pay whom and how much? (c) `216 (d) `224
(a) A to B, `28.50 (b) B to A, `37.50 [Based on MAT, 1999]
(c) A to B, `50 (d) B to A, `50 54.
A lent `600 to B for 2 years and `150 to C for 4 years and
[Based on MAT (Dec), 2006] received altogether from both `90 as simple interest. The
rate of interest is
48.
Consider the following statements
(a) 4% (b) 5%
If a sum of money is lent at simple interest, then the
(c) 10% (d) 12%
I.
money gets doubled in 5 years, if the rate of interest
[Based on MAT, 2000]
2
is 16 %. 55.
The rate of interest on a sum of money is 4% per annum
3
for the first 2 years, 6% per annum for the next 4 years and
II.
money gets doubled in 5 years, if the rate of interest is 8% per annum for the period beyond 6 years. If the simple
20%. interest accrued by the sum for a total period of 9 years is
III.
money becomes four times in 10 years, if it gets `1,120, what is the sum?
doubled in 5 years. (a) `1,500 (b) `2,000
Of these statements (c) `2,500 (d) `4,000
[Based on MAT, 2000]
(a) I and III are correct (b) II alone is correct
56.
A sum was put at simple interest at a certain rate for 2
(c) III alone is correct (d) II and III are correct
years. Had it been put at 1% higher rate, it would have
[Based on MAT (Dec), 2006] fetched `24 more. The sum is
49.
A man divided his share to his sons A and B in such a (a) `600 (b) `800
way that the interest received by A at 15% per annum for (c) `1,200 (d) `480
3 years is double the interest received by B at 12% per [Based on MAT, 2000]
annum for 5 years. At what ratio was his share divided?
57.
A sum of `2540 is lent out into two parts, one at 12%
(a) 2/3 (b) 8/3
and another one at 12.5%. If the total annual income is
(c) 3/8 (d) 3/2 `311.60, the lent money at 12%
[Based on MAT (May), 2010] (a) `1,180 (b) `1,360
50.
Divide `1586 in three parts in such a way that their (c) `1,240 (d) `1,340
amounts at the end of 2, 3 and 4 years, respectively, at 5% [Based on MAT, 2000]
per annum simple interest be equal 58.
I derive an annual income of `688.25 from `10,000
(a) `552, `528, `506 (b) `560, `520, `506 invested partly at 8% pa and partly at 5% pa simple
(c) `556, `524, `506 (d) `548, `528, `510 interest. How much of my money is invested at 5%?
[Based on MAT (Dec), 2006] (a) `3,725 (b) `4,225
(c) `4,800 (d) `5,000
51.
If a certain sum of money becomes double at simple
[Based on MAT, 2000]
interest in 12 years, what would be the rate of interest per
annum? 59.
The simple interest on a sum of money will be `600 after
1 10 years. If the principal is trebled after 5 years, what will
(a) 8 (b) 10 be the total interest at the end of the tenth year?
2
(a) `1,200 (b) `1,190
(c) 12 (d) 14
(c) `1,210 (d) None of these
[Based on MAT, 1998]
60.
A man purchased 40 fruits; apples and oranges for `17.
52.
Two equal sums were borrowed at 8% simple interest per
Had he purchased as many oranges as apples and as many
annum for 2 years and 3 years respectively. The difference
apples as oranges, he would have paid `15. Find the cost
in the interests was `56. The sums borrowed were
of one pair of an apple and an orange,
(a) `690 (b) `700 (a) 70 paise (b) 60 paise
(c) `740 (d) `780 (c) 80 paise (d) 1 rupee
[Based on MAT, 1998] [Based on SNAP, 2007]

Chapter_17.indd 393 1/30/2016 5:56:32 PM


394  Chapter 17

61.
A man earns 6% SI on his deposits in Bank A while he 68.
A certain sum is invested for T years. It amounts to `400
earns 8% simple interest on his deposits in the Bank. at 10% per annum. But when invested at 4% per annum, it
If the total interest he earns is `1800 in three years on an amounts to `200. Find the time (T).
investment M of `9000, what is the amount invested at (a) 41 years (b) 39 years
6%?
(c) 50 years (d) None of these
(a) `3000 (b) `6000
69. Arun borrowed a sum of money from Jayant at the rate
(c) `4000 (d) `4500
of 8% per annum simple interest of the first 4 years, 10%
[Based on SNAP, 2007] per annum for the next 6 years and 12% per annum for the
62.
In 4 years, `6000 amounts to `8000. In what time at the period beyond 10 years. If he pays a total of `12160 as
same rate will `525 amount to `700? interest only at the end of 15 years, how much money did
he borrow?
(a) 2 years (b) 3 years
(a) `12000 (b) `10000
(c) 4 years (d) 5 years
(c) `8000 (d) `9000
[Based on SNAP, 2009]
[Based on NMAT, 2005]
63.
A man invested one-third of his capital at 7%, one-fourth
at 8% and the remainder at 10% at simple interest. If his 70. A sum of `1440 is lent out in three parts in such a way that
annual income is `561, the capital is the interests on first part at 2% for 3 years, second part at
3% for 4 years and third part at 4% for 5 years are equal.
(a) `5400 (b) `6000 Than the difference between the largest and the smallest
(c) `6600 (d) `7200 sum is
[Based on FMS (MS), 2006] (a) `400 (b) `560
64. A part of `9600 is invested at a 5% annual return, while (c) `460 (d) `200
the remainder is invested at a 3% annual return. If the [Based on NMAT, 2005]
annual income from both portions is the same, what is the
71. A sum of `18750 is left by will by a father to be divided
total income from the two investments?
between two sons, 12 and 14 years of age, so that when
(a) `380 (b) `320 they attain maturity at 18, the amount (principal + interest)
(c) `440 (d) None of these received by each at 5% simple interest will be the same.
[Based on MHT-CET MBA, 2010] Find the sum allotted at present to each son.
(a) `9500, `9250 (b) `8000, `1750
65. The simple interest accrued on a sum of certain principal
is `1200 in 4 years at the rate of 8% per annum.. What (c) `9000, `9750 (d) None of these
would be the simple interest accrued on thrice of that [Based on NMAT, 2005]
principal at the rate of 6% per annum in 3 years? 72. Ms. Rao paid equated monthly installments (EMIs) of
(a) `2025 (b) `3025 `25000 each in January and February towards her home
(c) `2250 (d) `2150 loan, whose outstanding principal amount was `1000000
in December. Each EMI consists of interest of outstanding
[Based on OBC PO, 2010]
loan amount for the month and part payment of the loan
66.
Mr Mani invested an amount of `12,000 at the simple amount. If the interest on the loan is 12% per annum
interest rate of 10% per annum and another amount at (interest is paid monthly) on the diminishing outstanding
the simple interest rate of 20% per annum. The total in January and February was
interest earned at the end of one year on the total amount (a) `30150 (b) `20000
invested became 14% per annum. Find the total amount
(c) `19850 (d) `19700
invested.
[Based on JMET, 2006]
(a) `20,000 (b) `20,800
73. A certain sum of money is invested at an interest rate of
(c) `21,000 (d) None of these
5% per annum and a second sum, twice as large as the
67.
A sum of `7,700 is to be divided among three brothers first, is invested at 5.5% per annum. The total amount
Vikas, Vijay and Viraj in such a way that simple interest of interest earned from the two investments together is
on each part at 5% per annum after 1, 2 and 3 years, `1000 per year and the interest is withdrawn every year.
respectively remains equal. The share of Vikas is more The second sum invested is
than that of Viraj by (a) `6250 (b) `10500
(a) `2,800 (b) `2,500 (c) `12500 (d) `15000
(c) `3,000 (d) None of these [Based on JMET, 2006]

Chapter_17.indd 394 1/30/2016 5:56:32 PM


 Simple Interest  395

74.
A man invests `3,965 in the names of his three daughters (a) 4 (b) 8
Neeta, Sita and Gita in such a way that they get the same (c) 5 (d) Cannot be determined
amount after 2, 3 and 4 years, respectively. If the rate of
[Based on Gramin Bank U.P. (SO) Exam, 2012]
interest is 5% per annum, then the amount invested for
Neeta, Sita and Gita is 77.
In what time will a sum of money double itself @ 20% per
(a) `1,380, (b) `1,330, annum simple interest
`1,320, `1,360, (a) 10 years (b) 5 years
`1,265 `1,380 (c) 2 years (d) 14 years
(c) `1,265, (d) None of these [Based on SSC (GL), 2011]
`1,320,
78.
800 becomes `956 in 3 years at a certain rate of simple
`1,340
interest. If the rate of interest is increased by 4%, what
75.
If the amount obtained by Mahesh by investing `1,500 for amount will `800 become in 3 years?
two and half years at the rate of 8% per annum is equal to (a) `1020.80 (b) `1025
the amount obtained by Suresh by investing a certain sum
(c) `1052 (d) `1050
for 2 years at 5% per annum simple interest, then the sum
[Based on SSC (GL), 2011]
invested by Suresh is
(a) `1636 (b) `1,636 16
79.
Simple interest on a certain sum is of the sum. The
(c) `1636 (d) None of these 25
rate per cent if the rate per cent and time (in years) are
76.
Arun invested a sum of money at a certain rate of simple equal, is
interest for a period of four years. Had he invested the
same sum for a period of six years, the total interest (a) 6% (b) 8%
earned by him would have been fifty per cent more than (c) 10% (d) 12%
the earlier interest amount? What was the rate of interest [Based on SSC (GL), 2011]
per cent per annum?

EXPLANATORY ANSWERS

P×8× R 4.
(a) Let money invested at 5% be `k
1.
(c) 2P = ⇒ R = 25%
100 k × 1 × 5 (10000 − k ) × 1 × 8
\ + = 688.25
100 100
P × 25 × 20
S.I. = = 5P ⇒ 5k – 8k + 80000 = 68825
100
⇒ 3k = 11175
\ Amount = (5P + P) = 6P ⇒ k = 3725.
Therefore, it will become 6 times in 20 years time.
5. (c) Let P be the sum and R be the rate %
2.
(a) Let the amount given at 4% per annum be `x P × 3× R
\ P + = 2300 ...(1)
\ Amount given at 5% per annum = ` (1200 – x) 100
x × 4 × 2 (1200 − x) × 5 × 2 P ×5× R
⇒   + = 110 and P + = 2500 ...(2)
100 100 100
⇒ x = 500. 2 PR
(1) – (2) gives = 2000 ⇒ PR = 10000, which is
3. (d) Let each sum be `x 100
satisfied only by alternative (c).
1
x×4 ×7
2 x×4×7 6. (b) Suppose Amount invested in scheme A = ` x
\ − = 31.50
100 100 Amount invested in scheme B = `y
7x 1 63 Amount invested in scheme C = `z
⇒ × = \ x + y + z = 65000 ...(1)
100 2 2
18
⇒ x = 900. x = 72% of z = z ...(2)
25

Chapter_17.indd 395 1/30/2016 5:56:33 PM


396  Chapter 17

12x + 16y + 18z = 1018000 Amount = P = I = 2000 + 900


i.e., 6x + 8y + 9z = 509000 ...(3) = `2900.
Using (2) in (1) and (3), we get 11.
(b) Total interest on `4000 in 1 year = `352
43 352 × 100
z + y = 65000 ...(4) Average rate = = 8.8%
25 4000 × 1
108 1.2 : 0.8 = 3.2
z + 8 y + 9 z = 509000
25 First part = `2400.

333 P × 12 × 1
⇒ z + 8 y = 509000 ...(5) 12.
(c) 6000 = 
25 100
From (4), we have ⇒ P = `50,000
344 600 × 4 × R
+ 8 y = 520000 ...(4) 13.
(b) 120 =
5z 100
\ (vi) – (v)
120
11z ⇒ R =
%
⇒ = 11000 24
25
= 5% new rate = (5 + 2) = 7%
11z = 275000
600 × 4 × 7
z = 25000 \ S.I. at the new rate = = `168
100
\ x = 18000,
Therefore, amount = `(600 + 168) = `768
y = 22000.
14.
(c) Amount to be paid in first year
P × 5 × 9 P × 14 × 3
7.
(b) − = 10 200 × 5 × 1
100 × 12 100 × 12 = + 200 = 210
100
⇒ P = `4,000
Amount left as a principal for the second year
4000 = 210 – 100 = 110
Now, [5 × 9 + 14 × 3] = `290
100 × 12 \ Amount paid in second year
P × R ×T 110 × 5 × 1
8.
(c) Interest = = 110 + = 115.5
100 100

7 x × r4 × r 15.
(b) Let the amount invested for 1 year, 2 years and 3 years
x = be x, y and z, respectively.
25 100
x ×1× 4 y×2×4 z × 3× 4
7 x × 100 then, = =
r = = 7% 100 100 100
25 × 400
25 25
9.
(a) Let one part be `x and other = `(6,000 – x) \ x = 25K,  y = K ,  z = K
2 3
x×2×6 (6000 − x) × 3 × 8
= 25 25
100 100 x : y : z = 25 : : =6:3:2
2 3
12x = 144000 – 24x
x = `4,000 and other part 2
Smallest part = × 2189 = `398
= (6000 – 4000) = `2,000 11
16. (a)
P × R ×T
10.
(a) I = 17. (b) Rate of interest = 5x and 4x
100
Let he invests `P in bank A and Q in bank B
2000 × 5
600 = P × 5x × 1 Q × 4x × 1
100 then, =
2 × 100 2 × 100
R = 6%
5P = 4Q or P : Q = 4 : 5
New R = 6 + 3 = 9%
200 × 9 × 5 18.
(a) Let the amount of money lent out be `P.
I = P×6 3P
100 then first year interest = =`
= 900 100 50

Chapter_17.indd 396 1/30/2016 5:56:37 PM


 Simple Interest  397

3P 53P Equations (1) and (2)


Amount to be returned = P +
=` s 100 x 100 y
50 50 = ⇒ y2 = xz
Amount returned = `6,800, y z
 53P 
Balance amount =  − 6800  x × 5 × (18 − 2)
 50  24.
(c) x + = (18750 – x)
100
 53P  5× 4
 − 6800  × 5 + (18750 – x) ×
Second year interest =  50  100
100 ⇒ x = `9,000
53P − 340000 \ Other son gets = `9,750
= `
1000 x×2×3 y × 3× 4 z×4×5
25.
(b) = =
11 3P 100 100 100
It is given that second year interest = ×
20 50 ⇒ 3x = 6y = 10z = k
33P 53P − 340000 k k k
\ = ⇒ x = , y = , z =  x + y + z = 1440
1000 100 3 6 10
k k k
340000 + + = 1440  ⇒  k = 2400
⇒ P = = `17,000 3 6 10
20
k k 7k 7
P × r × 20 \ − = = × 2400 = 560
19.
(d) P = 3 10 30 30
100
(Interest = Amount – Principal) 8× 4 10 × 6 12 × 5
26.
(c) x × + x× + x× = 12160
r = 5% p.a. 100 100 100
⇒ x = `8,000
20.
(d) Let R be the rate of interest.
900 × R 27. (b) Let the sum be `x
Discount =
100 \ `x are lent at 8% for t years and `x are lent at 7%
3000 × R × T  1
Interest = for  t +  years
100  2
900 × R x×t ×8
By the given information; \ + x = 2560 ...(1)
100 100
x + (2t + 1) × 7
3000 × R × T and + x = 2560 ...(2)
= 2 × 100
100
T = 3 years = 36 months Solving Eqs. (1) and (2), we get
x = `2,000, t = 3.5 years
2600 × 20 × T 250 × T
21.
(b) S.I. = = 28. (d) Let the amount of money lent at 12% interest be `x.
3 × 100 3
\ Amount of money lent at 12.5% interest
To make simple interest in exact number of rupees t
should be 3. = `(2540 – x)
x × 12 × 1 (2540 − x) 12.5 × 1
P1 × 2 × 5 P × 3× 5 P ×4×5 \ + = 311.60
22.
(b) = 2 = 3 100 100
100 100 100
−0.5 x 31750
⇒ 10P1 = 15P2 = 20P3 ⇒ + = 311.60
100 100
1 1 1 ⇒ 0.5x = 31750 – 31160 = 590
⇒ P1 : P2 : P3 = 30 : 20 : 15 = : :
10 15 20 ⇒ x = `1,180
y × R ×T 100x 29.
(b) Let the annual instalment be `x
23.
(b) x=   \ RT = ...(1)
100 y Amount of `100 after 4 years
z × R ×T 100 y  100 × 5 × 4 
and y =   \ RT = ...(2) = ` 100 +  = `120
100 z  100 

Chapter_17.indd 397 1/30/2016 5:56:43 PM


398  Chapter 17

\ Present Value (P.V.) of `120 due after 4 years 31.


(d) Let each sum be `x
= `100 1
Present Value (P.V.) of `x due after 4 years x×4 ×7
2 x×4×7 7x 1 63
\ − = 31.50 ⇒ × =
100 5 100 100 100 2 2
= x = x
120 6 ⇒ x = 900
20
Similarly, P.V. of `x due after 3 years = x 32.
(c) Let the sum be `x.
23
10 x× 4× 2 x×6× 4 x×8×3
P.V. of `x due after 2 years = x Given: + + = 1120
11 100 100 100
20 112000
P.V. of `x due after 1 year = x ⇒ 56x = 112000  or,  x = = `2,000
21 56
Hence
5 20 10 20 33.
(a) Simple interest paid by Brinda on `1,000 for 1 year =
x+ x+ x+ x = 6450 1000 × 5 × 1
6 23 11 21 = `50.
⇒ x ≈ `1,810 100
Rent received by Brinda from Ramu in 1 year
30. (d) Suppose the person had deposited `x at the time of
opening the account. 1
= 12 × 12 = `150.
\ After one year, he had 2
 x × 10 × 1  11x \ Net savings = `100.
` x + = ` 
 100  10 Thus, Brinda will clear the debt of `1,000 in 10 years.
After two years, he had 34.
(b) Let the sum of money lent by Sumit to Mohit be `x.
11x 11x 10 × 1  121x Then, simple interest paid by Mohit after 1 year
` + ×  = 100 ...(1)
 10 10 100  x × 5 ×1 5x
= =` .
121x 100 100
After withdrawing `5,000 from ` , the balance =
100 Also, the simple interest received by Mohit from Birju
121x − 500000 after 1 year
`
100 17
x× ×1
After 3 years, he had 2 17 x
= =` .
121x − 500000 121x − 500000 10 × 1 100 200
+ ×
100 100 100 5x 17 x
Given: + 350 =
11(121x − 500000) 100 200
= ...(2)
1000 5 x + 35000 17 x
⇒ =
After withdrawing `6,000 from amount (2), the 100 200
balance ⇒ 1700x – 1000x = 7000000
1331x  or, 700x = 7000000
= `  − 11500 
 1000  7000000
\ After 4 years, he had or, x = = `10,000.
700
1331x − 5500000  1331x − 5500000  Thus, the sum of money lent by Sumit to Mohit is
`  + 10% of `  
 1000   1000  `10,000.
11 1331x  35.
(a) The simple interest on `1,500 invested at a rate of
= ` − 11500  ...(3)
10  1000  10% p.a. for 5 years is
[After withdrawing `10,000 from amount (3), the 1500 × 10 × 5
= = ` 750.
balance = 0] 100
11 1331x  Now, principal after 5 years = `1,500 + 750
\ − 11500  – 10,000 = 0
10  1000  = `2,250.
⇒ x = `15,470 Also, final amount = `2,500.

Chapter_17.indd 398 1/30/2016 6:24:35 PM


 Simple Interest  399

\ Simple interest = `2,500 – 2,250 = `250. P×r ×8


and + P = 400 …(2)
250 × 100 10 100
\ Time (T) = = years
2250 × 10 9 Subtracting Eq. (1) from Eq. (2), we get
10 55 1 P×r×5
Hence, total time = 5 + =   or  6 years = 100
9 9 9 100
\ P × r = 2000
1000 × 5 × 10
36.
(a) SI for 10 year = = ` 500 From Eq. (1),
100
Now, P = `1500, 2000 × 3
+ P = 300 ⇒ P = `240
A = `2000 100
\ 240 × r = 2000
\ SI = `500
⇒ r = 8.33%
1500 × 5 × T
\ 500 = 41.
(d) Let the amount in lented at 8% be `x.
100
Then, amount lented at 8.5% be (25000 – x).
500 × 100 2
⇒ T = = 6 year x × 8 × 1 (25000 − x) × 8.5 × 1
1500 × 5 3 \ + = 2031.25
100 100
2
\ Total time = 16 year ⇒ 8x + 212500 – 8.5x = 203125
3
⇒ – 0.5x = – 9375
37.
(a) Let Sunil, Sumant and Surat get x, y and z amount, ⇒ x = `18750
respectively.
42.
(a) In Ist case, the amount is `12000.
x × 5 ×1 y ×5× 2 z ×5×3
= = In IInd case, let the present value of money be x.
100 100 100
x × 18 × 8
⇒ x = 2y = 32 \ + x = 12880
12 × 100
⇒ x : y : z = 6 : 3 : 2
⇒ 0.12x + x = 12880
6−2
\ Required amount = × 7700 = `2800 12880
6+3+ 2 ⇒ x = = `11500
1.12
38.
(b) Amount invested at 12% = `x \ `12000 in cash is the better offer.
Amount invested at 10% = `x 43.
(b) Let the present value of money be x.
x × 12 × 1 y × 10 × 1 x × 12
130 = + Then, + x = 10028
100 100 100
⇒ 13000 = 12x + 10y ...(1) ⇒ 0.12x + x = 10028
10028
x × 10 × 1 y × 12 × 1 ⇒ x =
and 134 = + 1.12
100 100
This amount after 3 months
⇒ 13400 = 10x + 12y ...(2)
10028
Solving Eqs. (1) and (2), we get × 12 × 3
10028
x = `500 = 1.12 +
12 × 100 1.12
So, amount invested at 12% is `500. 10028 × 3 10028

= +
5100 1.12 × 100 1.12
39.
(a) Simple interest for 1 year = = `1700
3 10028 × 103
= = 9222.17 ≈ ` 9200
1% of sum = 17000 112
1700 × 100 44.
(b) Let she invest for x year.
\ Sum = = `170000
1 A sum will be double when interest is equal to
principal.
40.
(d) Let principle be P and rate of interest is r.
9535 × 4 × x
Then, \ 9535 =
100
P×r×3 ⇒ 4x = 100
+ P = 300 …(1)
100 ⇒ x = 25 years

Chapter_17.indd 399 1/30/2016 6:27:55 PM


400  Chapter 17

45.
(b) Present worth of money for Anu Therefore, I is not correct.
P×6×2 P × 20 × 5
1120 – P = ⇒ P = `1000 II. 
SI after 5 years = =P
100 100
Present worth of money for Biresh Amount after 5 years = P + P = 2P
P × 6 ×1 Therefore, II is correct.
1081.50 – P =
2 × 100 III. Amount after 10 years at the rate of 20%
⇒ 108150 – 100P = 3P P × 20 × 10
= P+ = P + 2P = 3P
⇒ P = `1050 100
\ Biresh should pay `50. Therefore, III is not correct.
46.
(c) Let the rate of interest be r % . Hence, (b) is the correct option.
Then 49.
(b) Let A and B received x and y amount, respectively.
1200 × r × 7
A = 1200 + x × 15 × 3 y × 12 × 5
12 × 100 Then, = 2×
100 100
⇒ A = 1200 + 7r
x 2 × 12 × 5 8
1016 × r × 2.5 ⇒ = =
Again, 1200 + 7r = 1016 + y 15 × 3 3
100
⇒ 1200 + 7r = 1016 + 25.4r 50.
(a) Let the three parts be `x, `y and `z.
⇒ 18.4r = 184 According to question,
184 x×2×5 y × 3× 5
\ r = = 10% x+ =y+
18.4 100 100

47.
(d) Let the present value of what A owes B be `x. z×4×5
= z +
x × 14 × 3 100
Then, x + =
1573
2 × 100 ⇒ 1.1x = 1.15 = 1.2z
21 x 1.15 23
⇒ x+ x = 1573 ⇒ = =
100 y 1.1 22

121x y 1.2 24
⇒ = 1573 and = =
100 z 1.15 23
1573 × 100 ⇒ x : y : z = 276 : 264 : 253
⇒ x = = `1300
121 276
⇒ x = × 1586 = `552,
Let y be the present value of what B owes A. 793
1 14 276
Then, y + y × × = `1444.50 y = × 1586 = `528
2 100 793
7 253
⇒ y+ y = 1444.50 and z = × 1586 = `506
100 793
1444.50 × 100 Hence, the required three parts are `552, `528 and
⇒ y = = `1350
107 `506.
Hence, B pay `50 to A. 51.
(a) Suppose sum = P, SI = P
48.
(b) Let the sum of money lent be `P. t = 12 years
Then, P × 100 1
Rate % = =8
P × 50 × 5 5 P × 12 2
SI received after 5 years = = P
3 × 100 6 52.
(b) Let S be the sum.
5 11P S × 8(3 − 2)
Amount after 5 years = P + P = \ = 56 ⇒ S = `700
6 6 100

Chapter_17.indd 400 1/30/2016 5:56:55 PM


 Simple Interest  401

53.
(c) CP = `153 Time (t) = 1 year
SP = 153 + 20% of 153 In 1st case:
= 153 + 30.60 = `183.60 Rate (r) = 12.5%
Let marked price be `x. x × 12.5 × 1
\ Simple Interest = `
\ x – 15% of x = 183.60 100
85 x In 2nd case:
⇒ = 183.60 ⇒ x = 216 Rate (r) = 12%
100
(2540 − x) × 12 × 1
54.
(b) Suppose rate = r % pa \ Simple Interest = `
100
In 1st case:
According to the question,
Sum (P) = `600; Time (T) = 2 years
12.5 x (2540 − x)12
600 × 2 × r + = 311.60
\ Simple Interest = ` = `12r 100 100
100
or, 0.5x + 30480 = 31160
In 2nd case:
or, 0.5x = 680
Sum (P) = `150; Time (T) = 4 years
\ x = 1360
150 × 4 × r
\ Simple Interest = ` = `6r Hence, the money lent at 12%
100
= `(2540 – 1360)
According to the question,
= `1180
12r + 6r = 90
or 18r = 90 58.
(a) Let money invested at 5% be `k.
\ r = 5%. k × 1 × 5 (10000 − k ) × 1 × 8
+ = 688.25
55.
(b) Suppose sum = `P 100 100
Total interest earned in 9 years ⇒ 5k – 8k + 80000 = 68825
⇒ 3k = 11175
P×4×2 P×6×4 P×8×3
= `  + `  + ` ⇒ k = 3725
100 100 100
According to the question, 59. (a) Interest for 5 years on the sum = `300.
When the principal is tripled, the interest is also
8 P 24 P 24 P
+ + = 1120 tripled.
100 100 100
\ Interest for another 5 years on this increased sum =
or 14P = 1120 × 25
`(300 × 3) = `900.
= 28000
\ Total interest = `300 + `900 = `1,200
or P = `2000
60.
(c) Man buys x apples at m price and y oranges at n price,
56.
(c) Suppose sum = `P then
Simple interest at certain rate r% for 2 years x + y = 40
P×r×2 Pr mx + ny = 17 ...(1)
= ` =`
100 50 Also, nx + my = 15 ...(2)
Also, simple interest at (r + 1)% for 2 years From Eqs. (1) and (2),
P × (r + 1) × 2 P (r + 1) (m + n) (x + y) = 17 + 15
= ` =`
100 50 32
⇒ (m + n) = = `0.80 = 80 paise
According to the question, 40
P (r + 1) Pr 61.
(b) Suppose, the man invests x in 6% and y in 8%.
– = 24
50 50 x + y = 9000 …(1)
P  x×6 y ×8
or = 24 and 3 ×  +  = 1800
50  100 100 
\ P = `1200 ⇒ 3x + 4y = 30000 …(2)
57.
(a) Suppose first part = `x From Eqs. (1) and (2), we get
\ Second part = `(2540 – x) \ x = `6000

Chapter_17.indd 401 1/30/2016 5:56:58 PM


402  Chapter 17

62.
(c) Let the rate of interest is ‘r’ The shares of Vikas, Vijay and Viraj will be in the
ratio
6000 × r × 4
\ = 2000 1 1 1 1 1 1
100 : : = : :
R1T1 R2T2 R3T3 1× 5 2 × 5 3 × 5
25
⇒ r = %
3 1 1 1
= : : = 6 : 3 : 2.
Now, calculate for `525 1 2 3
Sum of proportionals = 6 + 3 + 2 = 11.
63.
(c) Let capital be 12x.
6
\ 4x × 7% + 3x × 8% + 5x × 10% = 561 \ Share of Vikas = × 7700 = `4,200
11
x = 550
3
\ 550 × 12 = `6600t Share of Vijay = × 7700 = `2,100
11
64. (d) Let the annual amount investment at 5% and 3% be 2
`x and `(9600 – x), respectively. Share of Viraj = × 7700 = `1,400
11
x × 5 ×1 (9600 − x) × 3 × 1 Therefore, Vikas’s share is 4200 – 1400 = `2,800
Then, =
100 100 more than that of Viraj.
⇒ 5x = 28800 – 3x
68.
(c) We have A1 = `400, A2 = `200, R1 = 10%, R2 = 4%.
⇒ 8x = `28800
A1 − A2
28800 \ Time (T) = × 100
⇒ x = ` A2 R1 − A1R2
8
= `3600 400 − 200 20000
= × 100 =
So the total income 200 × 10 − 400 × 4 400
3600 × 5 × 1 (9600 − 3600) × 3 × 1 = 50 years
= +
100 100 69. (c) Simple interest
= `180 + `180 = `360 = (8 × 4)% + (10 × 6)% + (12 × 5)% = 12160
⇒ 152% = 12160
P× 4×8
65. (a) 1200 = ⇒ 100% = `8000
100
70. (b) SI1 = SI2 = SI3
1200 × 1000
P = = 3750 \ x × 6% = y × 12% = z × 20% = k
4×8
k k k
3750 × 3 × 6 × 3 \ x : y : z = : : × 60 = 10 : 5 : 3
Now, S.I. = = 2025 6 12 20
100
7
66.
(a) Here P1 = `12,000, R1 = 10%, P2 = ?, R2 = 20%, \ Required difference = × 1440 = `560
18
R = 14%.
71. (c) Let principal of two sons are x and y, respectively.
Therefore, using the formula
x × 130 y × 120 x 12
P1R1 + P2 R2 \ = ⇒ =
R = 100 100 y 13
P1 + P2
12
12000 × 10 + P2 × 20 \ x = × 18750 = `9000
We get 14 = 25
12000 + P2
13
or P2 = `8,000. y = × 18750 = `9750
25
\ Total amount invested
72. (c) Outstanding amount in December = `1000000
= ` (12000 + 8000) Interest to be paid in January = 1% of `1000000
= `20,000 = `10000 …(1)
67.
(a) Here T1 = 1, T2 = 2, T3 = 3, (Since, 12% per annum ⇒ 1% per month)
R1 = R2 = R3 = 5%. Past payment = `25000 – `10000
= `15000

Chapter_17.indd 402 1/30/2016 5:57:01 PM


 Simple Interest  403

Outstanding amount in January 1 1


That is, :  or,  1 : 1
= `1000000 – `15000 = 985000 5 100 + 5 × 2 120 110
100 + 8 ×
Interest to be paid in February = 1% of 985000 2
= 9850 …(2) 1 1
Total interest paid = `10000 + `9850 Given 1500 : x : : :
120 110
= `19850
1500 × 1 1
73. (c) Let the sum of money be x and 2x. ⇒ = ×x
110 120
Interest on both the sums
1500 × 1 × 120 4
x × 5 × 1 2 x × 5.5 × 1 or, x = = 1636 .
= + 110 11
100 100
4
5 x 11x 16 x \ The sum invested by Suresh is `1,636
= + = 11
100 100 100
76.
(d) Let the rate of interest be x% per annum.
According to questions,
P× x×4 3 P × x×6
16 x \ × =
= 1000 100 2 100
100 6x = 6x
1000 × 100 \ The value of x cannot be determined.
⇒ x = = `6250
16 77.
(b) S.I. = 2P – P = P
Hence, second sum invested = 2x = `12500 P × 20 × t
\ P =
74. (a) We have T1 = 2 years, T2 = 3 years, T3 = 4 years. 100
⇒ t = 5 years.
R1 = R2 = R3 = 5% p.a.
\ The ratio in which the amount is invested 79.
(c) S.I. = 956 – 800 = `156
1 1 1 Therefore, rate of interest
= : :
100 + R1T1 100 + R2T2 100 + R3T3 S.I.× 100
=
1 1 1 Principal × Time
i.e.,      : :
100 + 2 × 5 100 + 3 × 5 100 + 4 × 5 156 × 100
= = 6.5%
1 1 1 800 × 3
i.e., 
: :   or,  276 : 264 : 253. per annum.
110 115 120 Thus, new rate = 10.5%
Their sum = 276 + 264 + 253 = 793 so,
\ The amount invested for Principal × Time × Rate
S.I. =
3965 100
Neeta = × 276 = `1,380
793 800 × 3× 105
= = `252
3965 100
Sita
= × 264 = `1,320
793 Hence, Amount = 800 + 252
3965 = `1052
Gita = × 253 = `1,265
793 Interest 16
79.
(b) =
Principal 25
75.
(a) Let the sum of money invested by Suresh be `x.
  Since the amount obtained in both the cases is equal, Therefore, rate of interest
the ratio in which the sums are invested is S.I.× 100
=
1 1 Principal × Time
:
100 + R1T1 100 + R2T2
16 100
⇒ x = ×
5 25 x
where R1 = 8%, T1 = years, R2 = 5%,
2 ⇒ x2 = 16 × 4 = 64
T2 = 2 years. ⇒ x = 64 = 8 % per annum

Chapter_17.indd 403 1/30/2016 5:57:05 PM


18 Compound interest

INTRODUCTION
In Chapter 17, we discussed simple interest. A second to it. This method is used in investments such as savings
method of paying interest is the compound interest method, account and bonds. An understanding of compound interest
where the interest for each period is added to the principal is important not only for people planning careers with
before interest is calculated for the next period. With financial institutions but also for anyone planning to invest
this method the principal grows as the interest is added money.

soMe BAsiC FoRMulAe

1. (a) The amount A due after t years, when a 15000 × 21 × 21


= = `16537.50.
principal P is given on compound interest at 20 × 20
the rate R% per annum is given by
t Illustration 2 Find compound interest on `5000 for 2 years
 R  at 4% per annum.
A = P 1 +  .
 100  Solution: Here P = 5000, R = 4 and t = 2.
(b) Compound interest (CI ) = A – P t
 R  
 t
 \ CI = P 1 +  − 1
R   100  
= P  1 +  − 1
 100  
 2 
4 
 A  1/t  = 5000 1 +  − 1
(c) Rate of interest (R) =   − 1 % p.a.  100  
 P  
 26 2 
Note: Simple interest and compound interest for 1 year = 5000   − 1 = 5000((1.04)2 – 1)
at a given rate of interest per annum are always equal.  25  
= 5000(1.0816 – 1) = `408.
Illustration 1 Mohan invested an amount of `15000 \ The compound interest is `408.
at compound interest rate 5% per annum for a period of
2 years. What amount will he receive at the end of 2 years? Illustration 3 Rashi invested `16000 for two years at
Solution: Here P = 15000, R = 5 and t = 2. compound interest and received an amount of `17640 on
t maturity. What is the rate of interest?
 R 
\ Amount = P 1 +  Solution: Here P = 16000, t = 2 and A = 17640.
 100 
2 2  A 1/t 
 5   1  ∴ R = 100   − 1 % p.a.
= 15000 1 +  = 15000 1 + 
 100   20   P  

Chapter_18.indd 404 1/30/2016 5:48:53 PM


Compound Interest 405

 17640 1/2  1
= 100   − 1 % p.a.  ×4 
 A t
 16000   (c) Rate (R) = 4 × 100   − 1 % p.a.

 P  
 441 1/2 
= 100  In general, if the interest is compounded n
 − 1 % p.a.
 400  
times a year, then
 1  n×t
2× 
  21  2 R 
= 100   − 1 % p.a. (a) Amount (A) = P 1 + 
 20    100 × n 
1 (b) Compound interest (CI)
= 100 × = 5% p.a.
20  n×t 
R 
= P  1 +  − 1
2. If the interest is compounded half-yearly, then  100 × n  
 
2t
 R  (c) Rate of interest (R)
(a) Amount (A) = P 1 +  .
 100 × 2   1 
×n
 A t
(b) Compound interest (CI) = n × 100   − 1 % p.a.

 P  
 2t 
R 
= P  1 +  − 1
 100 × 2   Illustration 5 Find the compound interest on `1000 at
 
40% per annum compounded quarterly for 1 year.
 1 
×2 Solution: Here P = 1000, R = 40 and t = 1.
 A t
(c) Rate (R) = 2 × 100   − 1 % p.a.

\ Compound interest (CI)
 P  
  4×t
 R  

= P 1 +  − 1
Illustration 4 Find the amount of `8000 in one and half  100 × 4  
 
years at 5% per annum compound interest payable half-
 4×1 
yearly. 40   11 4 
= 1000 1 +  − 1 =1000   − 1
3  100 × 4    10  
Solution: Here P = 8000, R = 5 and t = .  
2
2t 14641 − 10000 
 R  = 1000   = `464.10.
\ Amount = P 1 +   10000
 100 × 2 
3 Illustration 6 Find the compound interest on `4000 at 24%
2× 3
 5  2  41  per annum for 3 months, compounded monthly.
= 8000 1 +  = 8000  
 100 ×2  40  3
Solution: Here P = 4000, R = 24 and t = .
12
8000 × 41 × 41 × 41
= = `8615.13.
40 × 40 × 40  12×t 
R 
\ 
CI = P 1 +  − 1
3. If the interest is compounded quarterly, then  100 × 12  
 
4t 3
 P   12× 
(a) Amount (A) = P 1 +   24  12 
 100 ×4 = 4000 1 +  − 1
 100 × 12  
(b) Compound interest (CI)
 4t   51 3  4000 × 7651
R  = 4000   − 1 =
= P  1 +  − 1  50  50 × 50 × 50
 100 × 4    
 
= `244.83.

Chapter_18.indd 405 1/30/2016 5:48:54 PM


406 Chapter 18

soMe useFul shoRt-Cut MethoDs

Illustration 8 What will be the compound interest on


1. When the rates of interest are different for `15625 for two and half years at 4% per annum?
different years, say R1, R2, R3 per cent for first,
second and third year, respectively, then   1 
 2 4×  
R  R  R  4   2 −1
 Solution: CI = 15625 1 +  1 +  
Amount = P 1 + 1 1 + 2 1 + 3  .  100   100  
 100  100  100 
 26 26 51 
= 15625  × × −1
Explanation  25 25 50 
Let the given sum of money be `P. Amount after first year
15625 × 3226
 R  = = `1613.
= P 1 + 1  . 31250
 100 
3. (a) The difference between the compound interest
This amount will be the principal for the second year. and the simple interest on a certain sum of
\ Amount after second year money for 2 years at R% per annum is given by
 R  R  2
= P 1 + 1 1 + 2  .  R 
CI – SI = P   [in terms of P and R]
 100  100   100 
This amount will be the principal for the third year. R × SI
and CI – SI = [in terms of SI and R]
\ Amount after third year 2 × 100
 R  R  R 
= P 1 + 1 1 + 2 1 + 3  .
 100  100  100  Explanation
Let `P be given sum of money. Simple interest on `P for 2
Illustration 7 Anu invests `5000 in a bond which gives years at R% per annum
interest at 4% per annum during the first year, 5% during P×R×2
the second year and 10% during the third year. How much =
100
does she get at the end of the third year.
and compound interest on `P for 2 years at R% per annum
Solution: Here P = 5000, R1 = 4, R2 = 5 and R3 = 10.
 2 
\ Amount at the end of third year R 
= P  1 +  − 1 .
 R  R  R   100  
= P 1 + 1 1 + 2 1 + 3 
 100  100  100   R 
2  P×R×2
\ CI – SI = P 1 +  − 1 −

= 5000 1 +
4  5  10   100   100
1 + 1 + 
 100  100  100 
 R2 2R 2R 
26 21 11 = P 1 + + −1− 
= 5000 × × × = `6006.  10000 100 100 
25 20 10
2
 R 
2. When the time is given in the form of fraction, say = P  .
 100 
3
years, then 2
4  R  R  P× R×2
Also, CI – SI = P   = × 
 3   100  100 × 2  100 
3 R
 R   4 .
Amount = P 1 +  × 1 +  R × SI
 100   100  = .
2 × 100

Chapter_18.indd 406 1/30/2016 5:48:55 PM


Compound Interest 407

Illustration 10 If the difference between the compound


(b) The difference between the compound interest and simple interest on a certain sum of money for 3
interest and the simple interest on a certain years at 5% per annum is `61, find the sum.
sum of money for 2 years at R% per annum is
given by Solution: Here CI – SI = 61 and R = 5.
  R 3  R  
2   R 3  R  
2
CI – SI = P   + 3    ∴ CI – SI = P   + 3   
 100   100    100   100  
[in terms of P and R]
  5 3  5  
2

 R  2 ⇒ 61 = P   + 3   
SI  R   100   100  
and, CI – SI =   + 3 
3  100   100  
  1 3 2
 1  
[in terms of SI and R] = P   + 3   
 20   20  
Explanation
 1 + 3 × 20   61 
Let `P be the given sum of money. Simple interest on `P for =P  = P 
3 years at R% per annum  20 × 20 × 20   20 × 20 × 20 
P× R×3 ⇒ P = `8000.
=
100
4. If a certain sum becomes n times in t years at
and compound interest on `P for 3 years at R% per annum compound interest, then the same sum becomes
 R 
3  nm times in mt years.
= P  1 +  − 1
 100  
Explanation
 3  P × R ×3 Let `P be the given sum of money. We have,
R 
∴ CI – SI = P 1 +  − 1 − t t
 100   100  R   R 
nP = P 1 +  ⇒n= 1 +  ...(1)
3 2  100   100 
 R 3R 3R 3R 
= P 1 + + + −1−  Let the sum become nm times in T years.
 100000 10000 100 100 
T
 R 
 R3 3R 2  Then, nm =  1 + 
=P +   100 
1000000 1000  T /m
 R 
or, n = 1 +  ...(2)
  R 3  R  
2
 100 
= P   + 3  .
 100   100   On comparing Eqs. (1) and (2), we get
2 T/m = t or T = mt years.
P × R × 3 1  R   R 
= ×   + 3  Therefore, the sum becomes nm times in mt years.
100 3  100   100  
Illustration 11 A sum of money placed at compound
SI  R  2  R  interest doubles itself in 3 years. In how many years will it
=   + 3  .
3 amount to four times itself?
 100   100  
Solution: Here n = 2, t = 3 and m = 2
Illustration 9 What will be the difference between simple
∴ The given sum of money will become four times
and compound interest on a sum of `4500 put for 2 years at
itself in mt, i.e., 2 × 3 = 6 years.
5% per annum?
Here P = 4500 and R = 5. 5. If a certain sum becomes n times in t years, then
2 2 the rate of compound interest is given by
 R   5  4500
∴ CI – SI = P   = 4500   = 25.
= `11×25. R = 100[(n)1/t – 1].
 100   100  20 × 20

Chapter_18.indd 407 1/30/2016 5:52:58 PM


408 Chapter 18

Illustration 12 At what rate per cent compound interest


does a sum of money become four-fold in 2 years? 7. If a loan of `P at R% compound interest per
annum is to be repaid in n equal yearly instalments,
Solution: The required rate per cent is then the value of each instalment is given by
R = 100[(n)1/t – 1] = 100[(4)1/2 – 1]
P
= 100(2 – 1) = 100%. ` 2 n
.
 100   100   100 
[Here n = 4 and t = 2]  +  + ...  
 100 + R   100 + R   100 + R 
6. If a certain sum of money at compound interest
amounts to `x in A years and to `y in B years, then Explanation
the rate of interest per annum is Let each instalment be of `X.
\ Principal for the amount of `X due at end of first
 y 1/B − A 
R =   − 1 × 100%. 100 X
year at R% = .
 x   100 + R
Principal for the amount of `X due at the end of second
Explanation 2
 100 
Let the principal be `P and the rate of interest be R% per year at R% =   X
annum.  100 + R 
A B   
 R   R 
Given x = P 1 +  and y = P 1 +  . Principal for the amount of `X due at the end of nth
 100   100  n
B  100 
 R  year at R% =   X.
1 +  R 
B− A  100 + R 
y 100  
\ =  A
=  1 +  . 2 n
x  R   100  100 X  100   100 
1 +  \ +  X + ...   X = P.
 100  100 + R  100 + R   100 + R 
1/B − A
R P
 y or X =` .
\   =1+ 2 n
x 100  100   100   100 
 +  + ...  
1/B − A  100 + R   100 + R   100 + R 
R  y
or =   –1
100 x Illustration 14 If a sum of `13040 is to be paid back in two
 y 1/B − A  3
equal annual instalments at 3 % per annum, what is the
or R =   − 1 × 100. 4
 x   amount of each instalment?
Solution: Each instalment
Illustration 13 A sum of money at compound interest
P 13040
amounts to `4050 in one year and to `4723.92 in 3 years. = 2
= 2
Find the rate of interest per annum.  100   100   100   100 
 +   +
 100 + R   100 + R  15   15 
Solution: Here x = 4050, y = 4723.92, A = 1 and B = 3.  100 +   100 + 
 4   4 
 y 1/B − A 
\ R =   − 1 × 100%  15 
 x    Here P = 13040 and R = 4 

 4723 ⋅ 92 1/2  13040 13040


= 2
=
=   − 1 × 100% 400  400  400  400 
 4050   +  1 + 
415  415  415  415 
 27  415 815
=  − 1 × 100% = 8%. = 13040 × × = `6889.
 25  400 400

Chapter_18.indd 408 1/30/2016 5:52:08 PM


Compound Interest  409

Multiple Choice QuestionS

1. A scooter costs `25,000 when it is brand new. At the end 8. What is the present value (in lakh of `) of an income of
of each year, its value is only 80% of what it was at the `2 lakh to be received after 2 years if the rate of interest
beginning of the year. What is the value of the scooter at is 5%?
the end of 3 years? (a) 1.81 (b) 1.67
(a) `10,000 (b) `12,500 (c) 2.2 (d) None of these
(c) `12,800 (d) `12,000 [Based on IIT Joint Man. Ent. Test, 2004]
[Based on MAT, 2004]
9. A sum is invested for 3 years compounded at 5%, 10%
2. A and B each borrowed equal sums for 3 years at the rate and 20% respectively. In three years, if the sum amounts
of 5% simple and compound interest respectively. At the to `16,632, then find the sum.
time of repayment B has to pay `76.25 more than A. The
sum borrowed and the interest paid by A (in `) is (a) `11000 (b) `12000
(a) `10,000, `1,500 (b) `11,000, `1,100 (c) `13000 (d) `14000
(c) `10,000, `1,400 (d) `9,000, `200 [Based on IIT Joint Man. Ent. Test, 2004]
10. The value of a fixed asset depreciates at the rate of 10% on
3. Sanjay borrowed a certain sum from Anil at a certain rate
the value at the beginning of each year. If the value of the
of simple interest for 2 years. He lent this sum to Ram
asset, two years ago, was `12,000 more than the value of
at the same rate of interest compounded annually for the
the asset one year ago, then find the present value of the
same period. At the end of two years, he received `4200 as
asset, given that the asset was bought two years ago.
compound interest but paid `4000 only as simple interest.
Find the rate of interest. (a) `14520 (b) `96,000
(a) 15% (b) 20% (c) `97,200 (d) `17,520
(c) 35% (d) 10% [Based on IIT Joint Man. Ent. Test, 2004]
[Based on MAT, 2002] 11. What will be the difference in the compound interest on
4. If the ratio of compound interest and simple interest for `50,000 at 12% for one year, when the interest is paid
the same principal and rate of interest for two years is yearly and half-yearly?
1.2% per annum, find the rate of interest. (a) `500 (b) `600
(a) 3% (b) 15% (c) `180 (d) `360
(c) 140% (d) 300% [Based on IIT Joint Man. Ent. Test, 2004]
5. If the difference between the simple and the compound 12. Akash borrows `65,000 at 10% per annum simple interest
interests on some principal amount at 20% for 3 years is for 3 years and lends it at 10% per annum, compound
`48, then the principal amount must be interest for 3 years. Find his gain after three years.
(a) `650 (b) `600 (a) `2015 (b) `1330
(c) `375 (d) `400 (c) `1300 (d) None of these
[Based on MAT, 2001]
[Based on IIT Joint Man. Ent. Test, 2004]
6. A bank offers 10% interest rate compounded annually. A
13.
A father divides his property between his two sons A
person deposits `10,000 every year in his account. If he
and B. A invests the amount at compound interest of 8%
does not withdraw any amount, then how much balance
per annum B invests the amount at 10% per annum simple
will his account show after four years?
interest. At the end of 2 years, the interest received by B is
(a) `51051 (b) `45095 `1336 more than the interest received by A. Find the share
(c) `36410 (d) `51000 of A in the father’s property of `25,000.
[Based on FMS (Delhi), 2004]
(a) `12,000 (b) `13,000
7. Two customers borrowed the same amount of money, one (c) `12,500 (d) `10,000
at compound interest and the other at simple interest. If
[Based on IIT Joint Man. Ent. Test, 2004]
after 2 years, the interest payable by one was `220 and by
the other `200, then, what was the principal money lent to 14. What is the compound interest on `5,000 for 4 years if the
each one of them? rate of interest is 10% p.a. for the first 2 years and 20% per
(a) `450 (b) `500 annum for the next 2 years?
(c) `550 (d) None of these (a) `2,320.50 (b) `3,712
[Based on IIT Joint Man. Ent. Test, 2004] (c) `3,745 (d) None of these

Chapter_18.indd 409 1/30/2016 5:48:56 PM


410  Chapter 18

15.
Rohit earns an interest of `1656 for the third year and required to pay at the end of second year to discharge his
`1440 for the second year on the same sum. Find the rate loan which was calculated at compound interest?
of interest if it is lent at compound interest. (a) 420 (b) 440
(a) 18% (b) 12% (c) 450 (d) 528
(c) 15% (d) None of these
23.
The population of vultures in a particular locality decreases
[Based on IIT Joint Man. Ent. Test, 2004]
by a certain rate of interest (compounded annually). If the
16. The population of a town increases annually by 25%. current population of vultures be 29,160 and the ratio of
If the present population is one crore, then what is the decrease in population for second year and 3rd year be
difference between the population 3 years ago and 2 years 10 : 9. What was the population of vultures 3 years ago?
ago? (a) 30,000 (b) 35,000
(a) 25,00,000 (b) 12,80,000 (c) 40,000 (d) 50,000
(c) 15,60,000 (d) None of these
24.
Equal amounts of each `43,892 is lend to two persons for
[Based on IIT Joint Man. Ent. Test, 2004]
3 years. One at the rate of 30% S.I. and second at the rate
17. Anuj has deposited certain amount in the bank to earn of 30% C.I. annually. By how much per cent the C.I. is
compound interest at 10% per annum. The difference in greater than the simple interest received in this 3 years
the interest on the amount between the 3rd and 2nd years duration?
is `1,100. What amount has Anuj deposited ? (a) 23% (b) 33%
(a) `10000 (b) `11000 (c) 33.33% (d) None of these
(c) Data inadequate (d) None of these
25.
What is the net present value of stock valued at `54,880
[Based on Narsee Manjee Inst. of Man. Studies, 2003]
after 3 years? (Rate of interest = 40% per annum and
18.
The difference between compound interest and simple interest in compounded annually)
interest on a sum for 2 years at 10% per annum, when (a) `20,000 (b) `30,000
the interest in compounded annually is `16. If the interest
(c) `40,000 (d) `50,000
were compounded half yearly, the difference in two
interests will be 26.
A sum of `1,100 was taken as a loan. This is to be repaid
(a) `24.81 (b) `31.61 in two equal instalments. If the rate of interest be 20%
(c) `32.40 (d) `26.90 compounded annually, then the value of each instalment
is
19. Divide `1301 between A and B, so that the amount of A (a) `842 (b) `792
after 7 years is equal to the amount of B after 9 years, the
(c) `720 (d) `700
interest being compounded at 4% per annum.
(a) `676 and `625 (b) `650 and `651 27.
Ravi Shankar takes money from the employees
(c) `670 and `631 (d) `660 and `630 cooperative society at lower rate of interest and saves in
a scheme, which gives him a compound interest of 20%,
[Based on IIFT, 2003]
compounded annually. Find the least number of complete
20.
The compound interest on a certain sum for 2 years years after which his sum will be more than double.
is `756 and S.I. (simple interest) is `720. If the sum is (a) 4 years (b) 2 years
invested such that the S.I. is `900 and the number of years
(c) 6 years (d) 8 years
is equal to the rate per cent per annum, find the rate per
[Based on MAT (Feb), 2011]
cent.
(a) 4 (b) 5/2 28.
Equal sums of money are deposited in two different
banks by M/s Enterprises, one at compound interest,
(c) 6 (d) 1.0
compounded annually and the other at simple interest,
21.
A person invested a certain amount at simple interest at both at 5% per annum. If after two years, the difference
the rate of 6% per annum earning `900 as an interest at in the amounts comes to `200, what are the amounts
the end of three years. Had the interest been compounded deposited with each bank?
every year, how much more interest would he have earned (a) `72000 (b) `64000
on the same amount with the same interest rate after three
(c) `80000 (d) `8400
years?
[Based on MAT (Feb), 2011]
(a) `38.13 (b) `25.33
29.
Rohit took a loan of `20000 to purchase one LCD TV
(c) `55.08 (d) `35.30
set from a finance company. He promised to make
22.
Amit borrowed `800 at 10% rate of interest. He repaid the payment after three years. The company charges
`400 at the end of first year. What is the amount he is compound interest at the rate of 10% per annum for the

Chapter_18.indd 410 1/30/2016 5:48:56 PM


Compound Interest  411

same. But suddenly the company announces the rate of 36.


The population of a colony was 3600 three years back.
interest as 15% per annum for the last one year of the loan It is 4800 right now. What will be the population three
period. What extra amount does Rohit have to pay due to years down the line, if the rate of growth of population has
this announcement of the new rate of interest? been constant over the years and has been compounding
(a) `7830 (b) `4410 annually?
(c) `6620 (d) None of these (a) 6000 (b) 6400
[Based on MAT (Dec), 2011] (c) 7200 (d) 9600
[Based on MAT (Feb), 2010]
30.
A tree was planted three years ago. The rate of its growth
is 30% per annum. If at present, the height of the tree is 37.
`5887 is divided between Shyam and Ram, such that
670 cm, what was it when the tree was planted? Shyam’s share at the end of 9 years is equal to Ram’s
(a) 305 cm (b) 500 cm share at the end of 11 years, compounded annually at the
rate of 5%. The share of Shyam is
(c) 405 cm (d) 625 cm
(a) `2088 (b) `2000
[Based on MAT (Dec), 2010]
(c) `3087 (d) None of these
31.
Sanju puts equal amounts of money, one at 10% per annum [Based on MAT (Feb), 2010]
compound interest payable half yearly and the second at
a certain per cent per annum compound interest payable 38.
The population of a village is 10000. If the population
yearly. If he gets equal amounts after 3 years, what is the increases by 10% in the first year, by 20% in the second
value of the second per cent? year and due to mass exodus, it decreases by 5% in the
third year, what will be its population after 3 years?
1
(a) 10 % (b) 10% (a) 13860 (b) 11540
4
(c) 12860 (d) 12540
1 1 [Based on MAT (Feb), 2009]
(c) 9 % (d) 8 %
2 4
39.
A sum of money is accumulating at compound interest
[Based on MAT (Dec), 2010]
at a certain rate of interest. If simple interest instead of
1 compound were reckoned, the interest for the first two
32.
A man borrows `4000 from a bank at 7 % compound years would be diminished by `20 and that for the first
2
interest. At the end of every year, he pays `1500 as part three years by `61. Find the sum.
repayment of loan and interest. How much does he still (a) `7000 (b) `8000
owe to the bank after three such instalments? (c) `7500 (d) `6500
(a) `123.25 (b) `125 [Based on MAT (Dec), 2008]
(c) `400 (d) `469.18 40.
The difference between the compound interest and simple
[Based on MAT (Sept), 2010] interest earned at the end of second year on a sum of
money at 10% per annum is `20. The sum is
33.
Amit borrowed a certain sum of money for 2 years at 8%
per annum on simple interest and immediately lent it to (a) `4,000 (b) `2,000
Ravi but at compound interest and gained by `16. What (c) `1,500 (d) Data inadequate
amount did Amit borrow? 41.
Ram takes a loan of `10000 and pays back `13310 after
(a) `1600 (b) `2500 3 years. The compound interest rate per annum will be
(c) `24000 (d) `1800 approximately
[Based on MAT (May), 2010] (a) 8% (b) 9%
34.
Raju’s factory kept increasing its output by the same (c) 10% (d) 11%
percentage every year. Find the percentage if it is known [Based on MAT (Sept), 2007]
that his output is doubled after two years. 42.
The ratio of the amount for two years under CI annually
(a) 100 2% (b) 100( 2 + 1)% and for one year under SI is 6 : 5. When the ratio of
interest is same, then the value of the rate of interest is
(c) 100( 2 − 1)% (d) 50( 3 − 1)% (a) 12.5% (b) 18%
[Based on MAT (May), 2010] (c) 20% (d) 16.66%
35.
The effective annual rate of interest corresponding to a [Based on MAT (Dec), 2006]
nominal rate of 8% per annum payable half yearly is 43.
A computer is available for `39000 cash or `17000 as
(a) 8% (b) 8.01% cash down payment followed by five monthly instalments
(c) 8.13% (d) 8.16% of `4800 each. What is the rate of interest per annum
[Based on MAT (Feb), 2010] under the instalment plan?

Chapter_18.indd 411 1/30/2016 5:48:57 PM


412  Chapter 18

(a) 35.71% (b) 37.71% (a) `16534.5 (b) `16756.5


(c) 36.71% (d) 38.71% (c) `17875.2 (d) `17556.8
[Based on MAT (May), 2006] [Based on FMS, 2009]
44.
Under the Rural Housing Scheme, the Delhi Development 51.
If the daily compounding rate of interest is 10% on an
Authority (DDA) allotted a house to Kamal Raj for investment, what is the present value of `50000 that is to
`126100. This payment is to be made in three equal be received after two years?
annual instalments. If the money is reckoned at 5% per (a) `40936.54 (b) `41037.33
annum compound interest, then how much is to be paid (c) `41322.31 (d) `40000
by Kamal Raj in each instalment? [Based on FMS, 2009]
(a) `45205 (b) `47405
52.
Mr Jeevan wanted to give some amount of money to his
(c) `46305 (d) `48505 two children, so that although today they may not be using
[Based on MAT (May), 2006] it, in the future the money would be of use to them. He
45.
A sum of `30600 is divided between Anjali and Arun, who divides a sum of `18750 between his two sons of age
are respectively 18 and 19 years old, in such a way that 10 years and 13 years respectively in such a way that
if their shares are invested at 4% per annum compounded each would receive the same amount at 3% per annum
annually, they shall receive the same amount on reaching compound interest when he attains the age of 30 years.
21 years of age. What is the share of Anjali? What would be the original share of the younger son?
(a) `16000 (b) `15000 (a) 8959.80 (b) 8559.80
(c) `15600 (d) `14600 (c) 8969.80 (d) 8995.80
[Based on MAT (May), 2010] [Based on IIFT, 2007]

46.
Two friends A and B jointly lent out `81600 at 4% per 53.
To start a new enterprise, Mr Yogesh has borrowed a total
annum compound interest. After 2 years, A gets the same of 60000 from two money lenders with the interest being
amount as B gets after 3 years. The investment made by B compounded annually, to be repaid at the end of 2 years.
was Mr Yogesh repaid `38800 more to the first money lender
compared to the second money lender at the end of 2 years.
(a) `40000 (b) `30000 The first money lender charged an interest rate, which was
(c) `45000 (d) `38000 10% more than what was charged by the second money
[Based on MAT (Feb), 2008] lender. If Mr Yogesh had instead borrowed `30000 from
47.
A sum of money becomes eight times in 3 years if the each at their respective initial rates for 2 years, he would
rate is compounded annually. In how much time the same have paid `7500 more to the first money lender compared
amount at the same compound interest rate will become to the second. Then, money borrowed by Mr Yogesh from
sixteen times? first money lender is
(a) 6 years (b) 4 years (a) 20000 (b) 35000
(c) 8 years (d) 5 years (c) 40000 (d) 42000
[Based on IIFT, 2010]
[Based on MAT, 1999]
54.
Mungeri Lai has two investment plans: A and B, to choose
48.
A sum of money is borrowed and paid back in two equal
from plan A offers interest of 10% compounded annually
annual installments of `882 allowing 5% compound
while plan B offers simple interest of 12% per annum. Till
interest. The sum borrowed was
how many years is plan B a better investment?
(a) `1,620 (b) `1,600
(a) 3 (b) 4
(c) `1,680 (d) `1,700
(c) 5 (d) 6
[Based on MAT, 2000]
[Based on XAT, 2009]
49.
The difference between compound interest and simple 55. A man gets a simple interest of `1000 on a certain
interest at the same rate `5000 for 2 years is `72. The rate principal at the rate of 5% per annum in 4 years. What
of interest per annum is compound interest will the man get on twice the principal
(a) 6% (b) 8% in two years at the same rate?
(c) 10% (d) 12% (a) `1050 (b) `1005
[Based on FMS, 2006] (c) `11025 (d) None of these
50.
The present value of an optical instrument is `20000. If its [Based on Punjab and Sind Bank PO, 2010]
value will depreciate 5% in the first year, 4% in the second 56.
A loan was repaid in two annual instalments of `121 each.
year and 2% in the third year, what will be its value after If the rate of interest be 10% per annum, compounded
three years? annually, the sum borrowed was

Chapter_18.indd 412 1/30/2016 5:48:57 PM


Compound Interest  413

(a) `200 (b) `210 (a) 25,00,000 (b) 12,80,000


(c) `217.80 (d) `216 (c) 15,60,000 (d) None of these
57. Sonika invested an amount of `5800 for 2 years. At what 65.
Rohit earns an interest of `1,656 for the third year and
rate of compound interest will she get an amount of `1,440 for the second year on the same sum. Find the rate
`594.5 at the end of two years? of interest if it is lent at compound interest.
(a) 5% per annum (b) 4% per annum (a) 18% (b) 12%
(c) 6% per annum (d) 8% per annum (c) 15% (d) None of these
[Based on Corporation Bank PO, 2010]
66.
A father divides his property between his two sons A and
58. The total compound interest earned on an amount @ 15% B. A invests the amount at compound interest of 8% per
per annum at the end of three years is ` 9844.5375. What annum and B invests the amount at 10% per annum simple
is the amount? interest. At the end of 2 years, the interest received by B
(a) `17500 (b) `18900 is `1,336 more than the interest received by A. Find the
(c) `19700 (d) `19400 share of A in the father’s property of `25,000.
[Based on IRMA, 2005] (a) `12,000 (b) `13,000
59.
Anu borrowed a sum of money and returned it in three (c) `12,500 (d) `10,000
equal quarterly instalments of `17,576 each. Find the 67. Mr Duggal invested `20000 with rate of interest @ 20%
sum borrowed, if the rate of interest charged was 16% per per annum. The interest was compounded half yearly for
annum compounded quarterly. Find also the total interest first one year and in the next year it was compounded
charged. yearly. What will be the total interest earned at the end of
(a) 46,900 and 4,700 (b) 48,775 and 3,953 two years?
(c) 68,320 and 1,200 (d) None of these (a) `8800 (b) `9040
60. The simple interest accrued on an amount of `27500 (c) `8040 (d) `9800
at the end of three years is `10230. What would be the [Based on United Bank of India PO, 2009]
approximate compound interest accrued on the same 68. The value of a fixed asset depreciates at the rate of 10% on
amount at the same rate in the same period? the value at the beginning of each year. If the value of the
(a) `11550 (b) `12620 asset, two years ago, was `12,000 more than the value of
(c) `10950 (d) `11900 the asset one year ago, then find the present value of the
[Based on New Indian Insurance PO, 2009] asset, given that the asset was bought two years ago.
61. The population of a city increases at a rate of 4% per (a) `14520 (b) `96,000
annum. There is an additional annual increase of 1% in (c) `97,200 (d) `17,520
the population due to the influx of job seekers. The per
69. Sanjay borrowed a certain sum from Anil at a certain rate
cent increase in the population after 2 years is
of simple interest for 2 years. He lent this sum to Ram
(a) 10.25 (b) 10 at the same rate of interest compounded annually for the
(c) 10.50 (d) 10.75 same period. At the end of two years, he received `4,200
62.
A scooter costs `25,000 when it is brand new. At the end as compound interest but paid `4,000 only as simple
of each year, its value is only 80% of what it was at the interest. Find the rate of interest.
beginning of the year. What is the value of the scooter at (a) 15% (b) 20%
the end of 3 years? (c) 35% (d) 10%
(a) `10,000 (b) `12,500 70.
In how many years will a sum of `800 at 10% per annum
(c) `12,800 (d) `12,000 compound interest, compounded semi-annually becomes
63. Anuj has deposited certain amount in the bank to earn `926.10?
compound interest at 10% per annum. The difference in 1 2
(a) 1 (b) 1
the interest on the amount between the 3rd and 2nd years is 2 2
`1,100. What amount has Anuj deposited? 1 1
(a) `10,000 (b) `11,000 (c) 2 (d) 2
3 2
(c) Data inadequate (d) None of these [Based on SSC (GL), 2010]
64. The population of a town increases annually by 25%. 71. Kruti took a loan at simple interest rate of 6% in the first
If the present population is one crore, then what is the year with an increase of 0.5% in each subsequent year.
difference between the population 3 years ago and 2 years She paid interest of `3375 after four years. How much
ago? loan did she take?

Chapter_18.indd 413 1/30/2016 5:48:57 PM


414  Chapter 18

(a) `12500 (b) `33250 76. A man borrows a certain sum and pays it back in 2 years
(c) `15800 (d) Cannot be determined in two equal instalments. If the compound interest is
[Based on Dena Bank PO, 2008] reckoned at 4% and if he pays back annually `676 the
sum borrowed is
72.
A sum of money placed at compound interest doubles
(a) `1275 (b) `1078
itself in 4 years. In how many years will it amount to four
times itself? (c) `1870 (d) None of these
[Based on NMAT, 2006]
(a) 12 years (b) 13 years
(c) 8 years (d) 16 years 77.
A sum of `12,000 deposited at compound interest becomes
[Based on SSC (GL), 2011] double after 5 years. After 20 years, it will become
(a) `48,000 (b) `96,000
73. Shudhir invested `16000 in a scheme which earned him
simple interest @ 15% per annum. After two years he (c) `1,90,000 (d) `1,92,000
withdrew the principal amount plus interest and invested [Based on SSC (GL), 2011]
the entire amount in amount scheme for two years, which 78.
If the difference between S.I. and C.I. for 2 years on a sum
earned him compound interest @ 12% per annum. What of money lent at 5% is `6, then the sum is
would be the total interest earned by Sudhir at the end of (a) `2200 (b) `2400
4 years?
(c) `2600 (d) `2000
(a) `9792 (b) `10152.11
[Based on SSC (GL), 2011]
(c) `9012.14 (d) None of these
79.
A sum of money at compound interest doubles itself in 15
[Based on IRMA, 2008]
years. It will become eight times of itself in
74. A man borrows `4000 at 20% compound rate of interest. (a) 45 years (b) 48 years
At the end of each year he pays back `1500. How much
(c) 54 years (d) 60 years
amount should he pay at the end of the third year to clear
all his dues? [Based on SSC (GL), 2010]

(a) `2952 (b) `2852 80. A person takes `10,000 loan at the rate of 10% interest
(c) `2592 (d) `2953 com­pounding yearly for the period of 4 years. How
much interest he has to pay?
[Based on NMAT, 2005]
(a) `4,371 (b) `4,581
75. One can purchase a flat from a house building society for
(c) `14,641 (d) `4,641
`55000 cash or on the terms that he should pay `4275
as cash down payment and the rest in three equal yearly [Based on U.P. P.C.S., 2012]
instalments. The society charges interest at the rate of 81. Rohit invested some amount at the rate of 6 pcpa and
16% per annum compounded half yearly. If the flat is at the end of 3 years he got `8730 simple interest. How
purchased under instalments plan, find the value of each much compound interest he will get on same amount and
instalments. same rate of interest after 2 years.
(a) `39683 (b) `19683 (a) `5820 (b) `5949.60
(c) `29683 (d) `22803 (c) `5900 (d) `5994.60
[Based on NMAT, 2005] [Based on Syndicate Bank PO, 2010]

EXPLANATORY ANSWERS

3
 20  4 4 4  21 21 21  15 P
(c) 25000 1 −
1.  = 25000 × 5 × 5 × 5  = P  × ×  – P – = 76.25
 100   20 20 20  100
= 12800. 9261P − 8000 P − 1200 P
 = = 76.25
2.
(a) Given Compound Interest – Simple Interest = `76.25 8000
n 61P = 8000 × 76.25 ⇒ P = 10,000

 R  PRN
⇒ P 1 +  − P − 100 = 76.25 PRN
 100  Simple Interest paid by A =
100
3
 5  P ×5×3 10,000 × 5 × 3
⇒ P 1 +  – P – 100 = 76.25  = = `1500
 100  100

Chapter_18.indd 414 1/30/2016 5:48:57 PM


Compound Interest  415

3. (d) Suppose the sum borrowed = `x `10000 after 3 years become


Rate of interest = R% 11 11 11
= 10000 × × × = 13310
Time = 2 years 10 10 10
x ×R ×2
\ 4000 = `10000 after 2 years become
100
11 11
⇒ Rx = 200000 ...(1) = 10000 × × = 12100
Now 10 10
2 `10000 after 1 year become `11000
 R 
x 1 +  = x + 4200
 100  \ Total amount after 4 years = `51051.

xR 2 2 RX 7. (b) S.I. for two years = `200
⇒ + = 4200 S.I. for one years = `100
10000 100
⇒ 20R + 4000 = 4200 C.I. for two years = `220
⇒ R = 10. ⇒ `20 is the interest on `100 for one year.
4.
(c) Let P be the principal, r be the rate of interest. If interest is `20, then amount = `100.
Pr If interest is `100, then amount
×r
Pr 100 Pr  r  100
C.I. = + = 1 +  = × 100 = `500.
100 100 100  100  20
2 Pr 8.
(a) P = Present value of `2 lakh
S.I. =
100 2
 5 
⇒ P 1 +  = 2 Lakh
Pr  r   100 
C.I. 1 + 
100  100  2 × 20 × 20
\ = 1.2 ⇒
S.I. Pr ⇒ P = ≈ 1.81.
2 21 × 21
100
9.
(b) Let P be the sum.
r r
⇒ 1 + = 2.4  or = 1.4 21P
100 100 \ Amount after one year = P + 5% of P =
r = 140% 20

5.
(c) Let the principal be `100. 21P 21P
Amount after two years = + 10% of
100 × 20 × 3 20 20
\ S.I. = = `60
100 21P 21P 231P
= + =
3 20 200 200
 20 
C.I. = 100 1 +  − 100 Amount after three years
 100 
231P 231P
6 6 6 = + 20% of
= 100 × × × − 10C 200 200
5 5 5
231P 231P 1386 P
= + =
 216 − 125  4 364 200 1000 1000
= 100   = 5 × 91 = 5
 125 
1386 P
364 64 \ = 16632 ⇒ P = 12000.
\ C.I. – S.I. = − 60 = 1000
5 5
10. (c) Suppose value of the asset two years ago = `x
64 \ Value of the asset one year ago
If difference is ` , principal = `100
5
9x
If difference is `48, principal = x – 10% of x =
10
100 × 5
= × 48 = `375. 9x
64 \ x− = 12000 ⇒ x = 120000
10
6.
(a) `10000 after 4 years become
\ Value of the asset at present
4
 10  11 11 11 11 9x 9x 81x
= 10000 1 +  = 10000 × 10 × 10 × 10 × 10 = − 10% of = = 81 × 1200
 100  10 10 100
= 14641 = `97200.

Chapter_18.indd 415 1/30/2016 5:48:59 PM


416  Chapter 18

11.
(c) C.I. on `50000 @ 12% for one year, when the interest 15.
(c) Interest on `1440 = `216 for the third year
is paid half yearly 216 × 100
2 ⇒ Rate % = = 15.
 6  1440 × 1
= 50000 1 +  − 50000
 100  16.
(b) Let the population be x, three years ago
53 53 5x
= 50000 × × − 50000 \ Population two years ago = x + 25% of x =
50 50 4
= `6180 5x 5x
Population one year ago = + 5% of
C.I. when the interest is paid yearly 4 4
50000 × 12 × 1 5x 5x 25 x
= = `6000 = + =
100 4 16 16
\ Required difference = C.I. – S.I. 25 x 25x
\ Present population = + 25% of
= 6180 – 6000 = `180. 16 16
12.
(a) S.I. on `65000 @ 10% for 3 years 125 x
= = 10000000
65000 × 10 × 3 64
= = `19500 (Given)
100 ⇒ x = 5120000
C.I. on `65000 @ 10% for 3 years 5x x
\ Required difference = −x =
10 
3 4 4

= 65000 1 +  − 65000 = 1280000.
 100 
17.
(b) S.I. for one year = `1100
11 × 11 × 11 − 10 × 10 × 10  1100 × 100
= 65000  
 1000  \ Amount at the end of 2nd year =
10 × 1
= `21515    = `11000.
\ Required gain = 21515 – 19500 = `2015. 18.
(a) `16 is the S.I. on S.I. for 1 year
13. (d) Suppose A get `x x × 10 × 1
\ 16 = or x = `160
\ B gets `(25000 – x) 100
Therefore, `160 is the simple interest for the first year.
Interest received by A @ 8% p.a. C.I.
 100 × 160 
2 Now principal = `   = `1,600
 8   10 × 1 
= x 1 +  −x
 100  Amount for 2 years compounded half yearly
2 5
 27  104 x   5  
=   x − x = = 1600 × 1 ×
 25  625   = `1944.81
  100  
Interest received by B @ 10% of p.a. S.I. C.I. = (1944.81 – 1600) = `344.81
(25000 − x) × 10 × 2 1600 × 10 × 2
= S.I. = = `320
100 100
\ C.I. – S.I. = (344.81 – 320) = `24.81
25000 − x
=
5 19. (a) Let Amount of A = P
Given: \ Amount of B = 1301 – P
25000 − x 104 x 7 9
= + 1336 ⇒ x = 10000.  4   4 
⇒ P 1 +  = (1301 – P)  1 + 
5 625  100   100 

 2 2   26 26 
10   20  ⇒ P = (1301 – P)  × 
14.
(b) C.I. = 5000 1 +  × 1 +  − 1  25 25 
 100   100  
⇒ 625P = 1301 × 676 – 676P
1856 ⇒ P = 676
= 5000 × = `3,712
2500 \ A gets `676 and B gets `625.

Chapter_18.indd 416 1/30/2016 5:49:00 PM


Compound Interest  417

20.
(b) C.I. for 2 years = `756 25.
(a) 54880 = P[1.4]3
S.I. for 2 years = `720 P = `20,000
It means the interest on the interest for the first year Alternative method:
= `36 (=756 – 720) Even if we assume S.I., there must be a growth of
This implies that the rate of interest is 5% as 40% × 3 = 120% over initial amount.
36 Now work with answer choices.
× 100 = 5%
720 If we increase (b), (c), (d ) by even 100% they would
It means the principal for first year was 14400 exceed 54880. Thus, option (a) is the answer.
P × 5 ×1 26.
(c) Let each instalment be `x
Q = 720
100 Then, (present worth of `x due 1 year hence) +
⇒ P = 14400 (Present worth of `x due 2 years hence) = `1,100
P×k ×k x x
Now, = S.I.,  where r = t = k \ + = 1100
100  20   2
1 + 20 
  1+
14400 × k 2  100   100 
⇒ = 900
100
5 x 25 x
5 or, + = 1100
⇒ k = 6 36
2
or 55x = 36 × 1100
900 × 100 36 × 1100
21.
(c) Sum = = `5,000 \ x = = `720
6×3 55
\ Interest on `5,000 by C.I. 27.
(b) Let the sum of money be P.
3
 6  T
= 5000 1 +  – 5000  20 
 100  \ P 1 +  ³ 2P
 100 
= `955.08
T
\ More interest = `(955.08 – 900) 6
⇒   ³2
= `55.08 5
22.
(d) Amount to be paid at the end of 2 years ⇒ (1.2)T ³ 2
800 × 10 × 2 Hence, least value of T is 2 year.
= + 800 = 880
100 28.
(c) Let P be the amount deposit in two different banks.
Amount left as principal for the second year In 1st bank,
2
= 480 = (880 – 400)  5 
CI = P 1 +  −P
480 × 10  100 
Amount to be paid after 2nd year = 480 +
100 2
 = `528  21 
= P   − P
Decrease in second year 100 10  20 
23.
(c) = =
Decrease in third year 100 – r 9 441P
= −P
⇒ r = 10% 400
Let the population of vultures 3 years ago be P, then 41P
=
 10 
3 400

P 1 −  = 29160
 100  In IInd bank,
⇒ P = 40,000 P × R ×T P ×5× 2 P
SI = = =
100 100 10
24.
(b) 100(1.3)3 = 219.7 ⇒ C.I. = 119.7
100 × 3 × 30 41P P
and S.I. = = 90 \ − = 200
100 400 10
\ C.I. is greater than S.I. by `29.7  (119.7 – 90) P
⇒ = 200
29.7 400
\ % increase = × 100 = 33.0%
90 ⇒ P = `80000

Chapter_18.indd 417 1/30/2016 5:49:00 PM


418  Chapter 18

29.
(d) Actual amount to be paid 35.
(d) Rate of 8% per annum payable half yearly.
3
 10  So, effective rate = 4%
= 20000 × 1 + 
 100  4×4
Effective annual rate = 4 + 4 + = 8.16%
= `26620 100
\ Extra amount that Rohit have to pay 36.
(b) Let rate of interest = R
2 3
 10   15   R 
= 20000 1 +  1 + = ` 27830 Then, 4800 = 3600 1 + 
 100   100   100 
\ Required amount = 27830 – 26620 = `1210 3
4  R 
⇒ = 1+ …(1)
30.
(a) Let the height of tree when it was planted be x cm. 3  100 
3
 30  Now, the population after 3 years
\ x × 1 +  = 670
 100  3
 R 
10 × 10 × 10 = 4800 1 + 
⇒ x = 670 × ≈ 305 cm  100 
13 × 13 × 13 From Eq. (1),
4
31.
(a) Let the value of second rate of interest be x% and 4800 × = 6400
equal amounts be P each. 3
6 3
 5   x  37.
(c) Let Shyam’s share = x
\ P × 1 +  = P × 1 + 
 100   100  9 11
 5   5 
2 Then, x 1 +  = (5887 – x) 1 + 100 
 5   x   100   
⇒  1 +  = 1 + 
 100   100  2
x  5 
105 × 105 100 + x ⇒ = 1 + 
⇒ = 5887 − x  100 
100 × 100 100
x
⇒ 110.25 = 100 + x ⇒ = 1.1025
⇒ x = 10.25% 5887 − x
⇒ x = `3087
32.
(a) Amount remaining after
7.5  38.
(d) Population after 3 years

1 year = 4000 1 +  − 1500 = `2800 110 120 95
 100  = 10000 × × × = 12540
100 100 100
 7.5 
2 year = 2800 1 +  − 1500 = `1510
 100  39.
(b) Let the principal be P and rate of interest be r %.
 7.5  Then, principal (when difference between CI and SI is
3 year = 1510 1 +  − 1500 = `123.25 for 2 years) is given by
 100 
20 × (100) 2
33.
(b) Let Amit borrow `x. P = …(1)
2
r2
 8   x×8× 2
x  1 +  − 1 − = 16 and principal (when difference between CI and SI is
 100   100 for 3 years) is given by

⇒ 0.1664x – 0.16x = 16 61 × (100)3
16 P = …(2)
⇒ x = = `2500 r 2 (300 + r )
0.0064
From Eqs. (1) and (2),
34.
(c) Let the output be x and percentage be a.
Then, 20 × 104 61 × 106
2 2
= 2
 a  r r (300 + r )
\ x × 1 +  = 2x
 100  ⇒ r = 305 – 300 = 5%
a From Eq. (1),
⇒ 1+ = 2
100 20 × 104
P =
a 25
⇒ = 2 – 1 ⇒ a = ( 2 – 1) × 100%
100 = `8000

Chapter_18.indd 418 1/30/2016 5:52:09 PM


Compound Interest  419

2 44.
(c) Let the amount of each instalment be `x.
 100 
40.
(b) Sum = Difference   Amount of `100 for 3 years
 R  3
2  5 
 100  = 100 1 + 
= 20 ×   = 20 × 100  100 
 10 
21 21 21 9261
= `2,000 = 100 × × × =
`
20 20 20 80
41.
(c) Let the rate of compound interest be r.
9261
3 Present value of `due after 3 years = `100
 r  80
Then, 10000 × 1 +  = 13310
 100  Present value of `x due after 3 years
r 
3 100 × 80 8000 x
 1331 = ×x=`
⇒ 1 +  = 9261 9261
 100  1000
Amount of `100 for 2 years
r 11
⇒ 1+ =  5 
2
21 21 441
100 10 = 100 1 +  = 100 × × =
r 1  100  20 20 4
⇒ =
100 10 400
Present value of `x due after 2 years = ` x
\ r = 10% 441
Similarly, present value of `x due after 1 year
2
 r  20
P 1 +  =` x
42.
(c)  100  = 6 21
 Pr  5
P +  \
8000
x+
400
x+
20
x = 126100
 100 
9261 441 21
 r  6 ⇒ 8000x + 8400x + 8820x = 126100 × 9261
⇒ 1 +  =
 100  5 ⇒ 25220x = 126100 × 9261
⇒ r = 20% 126100 × 9261
⇒ x =
43.
(d) Cash price, CP = `39000 25220
Cash down payment, DP = `17000 = `46305

45.
(b) Let the share of Anjali be x.
Balance due, after Ist instalment, BD = `22000
\ Then, the share of Arun is (30600 – x).
P = value of instalment = `4800
3 2
n = no. of instalments = 5  4   4 
x × 1 +  = (30600 − x) 1 + 
R = rate of interest  100   100 
 nR   ( n − 1) R  104
\ 1 +  BD = 1 +  nP ⇒ x× = 30600 – x
 1200   2400  100

 5R   4R  204
⇒ 1 + ⇒ x = 30600
 22000 = 1 +  5 × 4800 100
 1200   2400  ⇒ x = `15000
 5R   4R 
⇒ 1 + 11 = 1 + 12 46.
(a) Let the investment made by B = x
 1200   2400 
Then, investment made by A = (81600 – x)
55 R 24 R 2 3
⇒ 11 + = 12 +  4   4 
1200 1200 \ (81600 − x) 1 +  = x 1 + 
 100   100 
55 R 24 R ⇒ 81600 – x = 1.04x
⇒ − =1
1200 1200
81600
x = = `40000
31R 2.04
⇒ =1
1200
47.
(b) Quicker Method: If a sum becomes ‘x’ times in ‘y’
1200 years at compound interest it will be (x)n times in ‘ny’
⇒ R = = 38.71%
31 years.

Chapter_18.indd 419 1/30/2016 5:49:03 PM


420  Chapter 18

Here x = 8 and (x)n = 16. Here we have to find the We know that
value of n. t
 R 
A = P 1 
4  100 
(8)n = 16 ⇒ 23n = 24 ⇒ n =
3 Since, both receive the same amount.
y = 3 years.
20 17
4  3   3 
Hence the money will become sixteen times in ×3 \ x 1  = (18750  x) 1 
3  100   100 

= 4 years .
3
 3  18750  x
48.
(b) Suppose sum borrowed = `x ⇒ 1   =
100  x
Amount paid, A = `(882 × 2) = `1764
3
Rate (r) = 5%, Time (t) = 2 years  103  18750
⇒   = 1
Thus, 100  x
2 2
 5   21  441 18750
1764 = x 1 +  = x   = x ⇒ 2.092727 =
 100   20  400 x
1764 × 400 18750
x = = `1600. ⇒ x = = 8959.60
441 2.092727
       ≈ 8959.80
Pr 2 53.
(c) If the rate of interest changed by the second part is r%,
49.
(d) Difference =
1002 then on the first it is (r + 10)%.
2 2
Pr 2  r  10   r 
⇒ 72 = 30000 1    30000 1  100  = 7500
1002  100
      r = 20%
72  100  100
⇒ r2 =  144 Let the first part be x, then the second part will be
5000 (60000 – x).
\ r = 12% 2 2
 30   20 
x 1    (60000  x) 1  100  = 38800
50.
(c) Value after one year  100 

5    x = 40000
= 20000 – 20000 × = `19000
100 54.
(b) Till forth year plan B will fetch `48 and plan A will
4 fetch `46. Fifth year interest from plan B will be `60
Value after 2 years = 19000 – 19000 ×
100 and from plan A will be `61.
= 19000 – 760
1000 × 100
= `18240 55.
(d) Principle = = ` 5000
5× 4
2
Value after 3 years = 18240 – 18240 × 2
100  5  
= `17875.2 Compound interest = 10000 1 +  − 1
 100  
51.
(c) Let the investment be `100.
41
 10 
2 = 10000 × = ` 1025
\ After two years the sum = 100 1  = `121 100
 100 
56.
(b) Principal = (Present worth of `121 due 1 year hence)
Q `121 is received when the investment was 100. + (Present worth of `121 due 2 years hence)
100 121 121
\ `1 is received when the investment was . = +
121  10   2
10 
\ `50000 is received when the investment was 1 +  1 + 
 100   100 
100  50000
= = `41322.31 121 121
121 = + = `210
1.01 1.01 × 1.01
52.
(a) Let the amount with the young son be x, time be 20
years rate 3%  2
r  
   The amount with the elder son be (18750 – x) time 57.
(a) 594.5 = 5800 1 +   −1
 100  
be 17 years, rate 3%

Chapter_18.indd 420 1/30/2016 5:49:04 PM


Compound Interest  421

2
594.5  r  112.4 × 112.4 × 112.4 
= 1 +  −1  − 100 × 100 × 100 
5800  100  = 27500  
 100 × 100 × 100 
2
 r   
0.1025 + 1 = 1 + 
 100  1420034.624 + 1000000 
2
= 27500  
(100 + r )  1000000 
1.1025 =
10000  420034.624 
= 27500 
1.1025 × 10000 = (100 + r)2  1000000 

11025 = (100 + r)2
= 27500 × 0.42
(105)2 = (100 + r)2
= `11550
105 = 100 + r
5×5
r = 5% 61.
(a) Required % = 5 + 5 + = 10.25%
100
 3 
r  62.
(c) After first year, the value of the scooter = `20,000
58.
(b) CI = P 1 +  − 1 After second year, the value of scooter = `16,000
 100  
After third year, the value of scooter = `12,800
3
  15   63. (b) S.I. for one year = `1,100
P  1 +  − 1 = 9844.5375
  100   \ Amount at the end of 2nd year
1100 × 100
⇒ P[1.520875 – 1] = 9844.5375 = = `11,000
10 × 1
9844.5375
\ P = = `18900
0.520875 64.
(b) Let the population be x, three years ago
5x
59.
(b) Rate of interest = 16% annum \ Population two years ago = x + 25% of x =
4
Actual rate of interest = 4% per quarter
Principal of all three instalments 5x 5x
Population one year ago = + 5% of
4 4
   25   25 2  25 3  
= 17576    +   +     5x 5x 25 x
  26    = + =
   26   26  4 16 16
17576 × 25 × 1951 25 x 25x
= = 48775 \ Present population = + 25% of
26 × 676 16 16
Total amount paid = `17576 × 3 = `52,728 125 x
=
Interest charged = 52728 – 48775 = `3,953 64

S.I. × 100 = 10000000 (Given)


60.
(a) R = ⇒ x = 5120000
P ×T
5x x
10230 × 100 \ Required difference = −x=
= = 12.4% p.q. 4 4
27500 × 3
= 12,80,000
  t 65. (c) Interest on `1,440 = `216 for the third year
r 
CI = P 1 +  − 1 216 × 100
 100   ⇒ Rate % = = 15
1440 × 1
 12.4 3  66. (d) Suppose A gets `x
= 27500 1 +  − 1
 100   \ B gets `(25000 – x)
Interest received by A at the rate of 8% p.a. C.I.
 112.4 3  2
= 27500   − 1  8 
 100   = x 1 +  −x
 100 

Chapter_18.indd 421 1/30/2016 5:49:05 PM


422  Chapter 18

2 70.
(a) Rate of interest = 10% per annum. So, rate of interest
 27  104 x
=   x − x = for half yearly = 5%
 25  625
(1 + R )T
Interest received by B at the rate of 10% of p.a. S.I. Therefore, A = P
100
(25000 − x) × 10 × 2
= (1 + 5)T
100 926.10 = 800
100
25000 − x
= (100 + 5)T
5 926.10 = 800
Given: 100
25000 − x 104 x (21)T
= + 1336 ⇒ x = 10,000 926.10 = 800
5 625 20
T
67.
(b) When interest was compounded half-yearly then 926.1 × 10  21 
=  
20 8000 × 10  20 
R = = 10%
2 T
9261  21 
T = 2 units for 1 year =  
8000  20 
Accumalated interest in 2 years
3 T
 2  21   21 
 10   20     =  
= 20000 1 +  1 +   − 20000  20   20 
  100   100  
Hence, time = 3 half years
 11 11 6  1
= 20000 × × ×  − 20000 = 1 years
 10 10 5  2
= 29040 – 20000
= `9040 71.
(a) Suppose the principle is `x.

68. (c) Suppose value of the asset two years ago = `x  6 6.5 7.0 7.5 
x  + + + = 3375
\ Value of the asset one year ago 100 100 100 100 
9x x
= x – 10% of x = × 27 = 3375
10 100
3375 × 100
9x \ x =
\ x− = 12000 ⇒ x = 120000 27
10
= `12500
\ Value of the asset at present

4 × log 4
9x 9x 72.
(c) Reqd. time = = 8 years
= − 10% of log 2
10 10
81x 16000 × 30
= = 81 × 1200 73.
(d) SI = = 4800
100 100
= `97,200 \ Amount = 16000 + 4800 = 20800
69. (d) Suppose the sum borrowed = `x 2
Rate of interest = R%  12 
Now, CI = 20800 × 1 + 
Time = 2 years  100 
x×R×2 28 28
\ 4000 = ⇒ Rx = 200000 ...(1) = 20800 × ×
100 25 25
2 = 26091.52 (Amount)
 R 
Now, x 1 +  = x + 4200 \ A – P = 26091.52 – 20800
 100 
= 5291.52
xR 2 2 RX After four years, interest = 5291.52 + 4800 = 10091.52
⇒ + = 4200
10000 100
⇒ 20R + 4000 = 4200 120
74.
(a) At the end of 1st year = 4000 × = 4800
⇒ R = 10 100

Chapter_18.indd 422 1/30/2016 5:49:06 PM


Compound Interest  423

But he pays back = 1500 S


 R 
At the end of 2nd year ⇒ 2 = 1 + 
 100 
120
= 3300 × = 3960 – 1500 = 2460 20
100  R 
⇒ 24 = 1 + 
120  100 
At the end of 3rd year = 2460 × = `2952
100 20
 R 
75.
(b) Cash price of the flat = `55000 ⇒ 16 = 1 + 
 100 
In the instalment plan, cash down payment = `4275
Hence, the principal will become 16 times in 20 years.
= 55000 – 4275 = 50725
Let each instalment be `x. = `(16 × 12000)
Rate = 16% per annum = 8% half-yearly = `1,92,000

r 
t Pr 2
 78.
(b) Difference =
\ A = P 1 −  100
 100 
1 P55
 8  25 ⇒ 6  ⇒ 25 P = 6000
x = P 1 +  =` x 10000
 100  75
2 6000
 25  ⇒ P = = `2400
Similarly, principal for the 2nd instalment =   x 25
 27 
79.
(a) It becomes 2 times in 15 years.
3
 25  It becomes 4 times in 30 years.
Principal for 3rd year instalment = `   x
 27 
It becomes 8 times in 45 years.
Total principal for the three instalments
2 3 80.
(d) Required interest
25  25   25 
= x +   x +   x = 50725 4
27  27   27   10 
= 10000 1 +  − 10000
On solving, we get x = 19683  100 
\ Each instalment = `19683
(11) 4
= ` 10000 × 10000
676 676 10000
76.
(a) + 2
 4 
1 +  1 + 4  = `(14641 – 10000)
 100   100  = `4641
676 × 25 676 × 625 8730 × 100
= + 81.
(d) Principle = = 48500
26 676 6×3
= 650 + 625 = `1275 Compound interest
T
 R   2 
77.
(d) A = P 1 +  = 48500 1 + 6  − 1
 100   100  
T
A  R  = 48500 × 0.1236
⇒ = 1 + 
P  100  = `5994.60

Chapter_18.indd 423 1/30/2016 5:49:07 PM


19 Logarithms

INTRODUCTION logab = m
Logarithm, in mathematics, is the ‘exponent’ or ‘power’ to ‘log’ being the abbreviation of the word ‘logarithm’. Thus, 
which a stated number called the base is raised to yield a am = b ⇔ logab = m
2
specifi c number. For example, in the expression 10  = 100,  where, am = b is called the exponential form and log b = m
a
the  logarithm  of  100  to  the  base  10  is  2.  This  is  written  is called the logarithmic form.
as log10  100  =  2.  Logarithms  were  originally  invented  to 
help simplify the arithmetical processes of multiplication, Illustration 1 Refer to the following table
division,  expansion  to  a  power  and  extraction  of  a  ‘root’,  Exponential form Logarithmic form
but they are nowadays used for a variety of purposes in pure  5
and applied mathematics.  3 = 243 log3 243 = 5
24  = 16  log2 16  = 4
Logarithm 0
3   = 1  log3 1  = 0
If for a positive real number (a ≠ 1), am = b, then the index 1
m is called the logarithm of b to the base a. We write this as 81/3 = 2 log8 2 =
3

LAWS OF LOGARITHMS

1. Product formula 4. Base changing formula


    The logarithm of the product of two numbers is  log a m log m
equal to the sum of their logarithms. lognm = . So, logn m = .
log a n log n
    i.e.,   loga (mn) = logam + logan.
where m, n, a are positived and n ≠ 1, a ≠ 1.
Generalisation: In general, we have 
loga (mnpq...) = loga m + loga n + loga p + loga q +... 5. Reciprocal relation
2. Quotient formula logba × logab = 1,
    The logarithm of the quotient of two numbers is  where a and b are positive and not equal to 1.
equal to the difference of their logarithms. 1
6. logba =
m log a b
    i.e., loga   = logam – logan,
n 7. aloga x = x, where a and x are positive, a ≠ 1.
where a, m, n are positive and a ≠ 1.
3. Power formula 8. If a > 1 and x > 1, then loga x > 0.
    The logarithm of a number raised to a power is equal  9.  If 0 < a < 1 and 0 < x < 1, then loga x > 0.
to the power multiplied by logarithm of the number.  10.  If 0 < a < 1 and x > 1, then loga x > 0.
    i.e., loga (mn) = n loga m,
11. If a > 1 and 0 < x < 1, then loga x < 0.
where a, m are positive and a ≠ 1.

Chapter_19.indd 424 1/30/2016 5:20:00 PM


  Logarithms  425

SOME USEFUL FORMULAE

natural logarithms. The logarithms to base 10 are called the 
1.  Logarithm of 1 to any base is equal to zero. common logarithms. 
    i.e., loga 1 = 0, where a > 0, a ≠ 1.
2.  Logarithm of any number to the same base is 1. log1010 = 1, since 101 = 10.
    i.e., loga a = 1, where a > 0, a ≠ 1. log10100 = 2, since 102 = 100.
log1010000 = 4, since 104 = 10000.
Common Logarithms log100.01 = –2, since 10–2 = 0.01.
There  are  two  bases  of  logarithms  that  are  extensively  log100.001 = –3, since 10–3 = 0.001
used these days. One is base e (e = 2. 71828 approx.) and 
 = 2. 71828 approx.) and  and log101 = 0, since 10° = 1.
the  other  is  base  10.  The  logarithms  to  base  e are called

MULTIPLE CHOICE QUESTIONS

1. If log10 x − log10 x = 2 logx10, then a possible value of  n


1
x is given by:
5. ∑ log (a)
=
n =1 2n
    (a)  10  (b) 1/100
n (n + 1) n (n − 1)
    (c)  1/1000  (d) None of these     (a)  log a 2   (b)  log 2 a
2 2
[Based on CAT, 2004]
2. What is the sum of ‘n’ terms in the series:  (n + 1) 2 n 2
    (c)  log a 2   (d) None of these
4
 m2   m3   m4 
log m + log  + log  2  + log  3  + ...
 n  n  n  a b
      6. If log + log  = log (a + b), then
b a
n/2 n/2
 n( n−1)   nm      (a)  a + b = 1  (b) a – b
    (a)  log  ( n+1)    (b)  log  n 
    (c)  a2 – b = 1  (d) a = b
 m   n 
[Based on FMS (Delhi), 2007]
n/2 n/2
m (1− n)  m ( n +1)  7. The number of real solutions of the equation
    (c)  log  (1+ n)    (d)  log  ( n−1) 
  log (– x) = 2 log (x + 1) is 
 n   n 
    (a)  One  (b) Two
[Based on CAT, 2004]
    (c)  Three  (d) Four
3.  The  length  of  the  circumference  of  a  circle  equals  the 
[Based on MAT, 2005]
perimeter of a triangle of equal sides, and also the
2
perimeter  of  a  square.  The  areas  covered  by  the  circle,  8. If log5 (x + x) – log5 (x + 1) = 2, then the value of x is
triangle, and square are c, t and s respectively. Then,     (a)  5  (b) 32
    (a)  s > t > c  (b) c > t > s     (c)  25  (d) 10
    (c)  c > s > t  (d) s > c > t [Based on FMS (Delhi), 2007]
[Based on CAT, 2004] 9. If log12 27 = a, then log6 16 is
4. If logn 48 = a and logn 108  =  b,  what  is  the  value  of      (a)  (3 – a)/4 (3 + a)  (b) (3 + a)/4 (3 – a)
logn 1296 in terms of a and b?     (c)  4(3 + a)/(3 – a)  (d) 4(3 – a)/(3 + a)
2(2a + b) (a + 3b) [Based on MAT (Dec), 2006]
    (a)    (b) 
5 5 1 1 k
10. If logab = , logbc = and logca = ,  then the value of 
4(2a + b) 2(a + 3b) 2 3 5
    (c)    (d)  k is
5 5

Chapter_19.indd 425 1/30/2016 5:20:01 PM


426  Chapter 19

(a) 25 (b) 35 If log10 (x2 – 6x + 45) = 2, then the value of x are


19.
(c) 30 (d) 20 (a) 6, 9 (b) 9, – 5
[Based on MAT (May), 2006] (c) 10, 5 (d) 11, – 5
11.
Which of the following is true? [Based on FMS, 2005]
(a) log17 275 = log19 375 20.
If log10 125 + log10 8 = x, then x is equal to
(b) log17 275 < log19 375 (a) –3 (b) 3
(c) log17 275 > log19 375 1
(d) None of the above (c) (d) 0.064
3
[Based on MAT (May), 2005] [Based on FMS, 2005]
2 2 2
1 21.
log (a / bc) + log (b / ac) + log (c / ab) is
12. log10 25 – 2 1og10 3 + log10 18 equals to
2 (a) 1 (b) 0
(a) 18 (b) 1 (c) 39 (d) abc
(c) log10 3 (d) None of these [Based on FMS, 2006]
[Based on MAT (Sept), 2003]
22.
If log7 log5 ( x  5  x ) = 0, what is the value of x?
13.
The difference between the logarithms of sum of the
(a) 2 (b) 3
squares of two positive numbers A and B and the sum of
logarithms of the individual numbers is a constant C. If (c) 4 (d) 5
A = B, then C is [Based on FMS (2009]
(a) 2 (b) 1.3031 23. What is the value of x, if
(c) log 2 (d) exp (2) 1 1 1 1
[Based on MAT (Sept), 2003]
+ + + ... +
log 442 x log 443 x log 444 x log 899 x
x/2
14.
If logx a, a , and logb x are in GP, then x is 441 442 443 898
(a) loga(logb a) 1
(b) loga(logea) + loga(loge b) + = 2?
log 900 x
(c) – loga (loga b) 899
(d) loga (loge b) – loga (loge a) 2
(a) (b) 1
[Based on MAT (Dec), 2002] 21
15.
If loga (ab) = x, then logb (ab) is 7 10
(c) (d)
1 x 10 7
(a) (b)
x x +1 What is the value of [log10 (5 log10 100)]2?
24.
x x (a) 25 (b) 10
(c) (d)
x −1 1− x (c) 2 (d) 1
[Based on MAT (May), 2002] [Based on FMS, 2010]
1 1 25.
If log10 2 = a and log10 3 = b, then log5 12 equals
16.
If log8 x + log 8 = , then the value of x is
6 3 ab 2a  b
(a) 18 (b) 24 (a) (b)
1 a 1 a
(c) 16 (d) 12
[Based on MAT (May), 1998] a  2b 2a  b
(c) (d)
1 1 1 a 1 a
17.
If log 8 x + log 8 = , then the value of x is
6 3 [Based on FMS, 2010]
(a) 18 (b) 24 26.
If a = log8 225 and b = log2 15, then a in terms of b, is
(c) 16 (d) 12 (a) b/2 (b) 2b/3
[Based on MAT, 1998]
(c) b (d) 3b/2
1
18.
If log x (0.1) = – , then the value of x is [Based on FMS, 2010]
3
(a) 10 (b) 100 27.
If log x – 5 log 3 = –2, then x equals
1 (a) 1.25 (b) 0.81
(c) 1000 (d)
1000 (c) 2.43 (d) 0.8
[Based on FMS (MS), 2006] [Based on FMS, 2011]

Chapter_19.indd 426 1/30/2016 5:20:02 PM


427
Logarithms 

(125) (625) If (log3 x) (logx 2x) (log2x y) = logx x2, then y equals
37.
28.
The value of log5 is equal to
25 (a) 4.5 (b) 9
(a) 725 (b) 6 (c) 18 (d) 27
(c) 3125 (d) 5 [Based on GBO Delhi University, 2011]
[Based on FMS, 2011]
38.
If loga 3 = 2 and logb 8 = 3, then loga b is
The values of a in the equation:log10 (a2 – 15a) = 2 are
29.
(a) log3 2 (b) log2 3
15  233
(a) (b) 20, – 5 (c) log3 4 (d) log4 3
2
39.
log4 2 – log8 2 + log16 2 – ... to ∞ is
15  305
(c) (d) ± 20 (a) e2 (b) ln 2 + 1
2
[Based on FMS, 2011] (c) ln 2 – 1 (d) 1 – ln 2
30.
What is the value of the following expression?
40.
If log7 2 = m, then log49 28 is equal to
log (9/14) – log (15/16) + log (35/24)
1 + 2m
(a) 0 (b) 1 (a) 2(1 + 2m) (b)
2
(c) 2 (d) 3
[Based on IIFT, 2005] 2
(c) (d) 1 + m
1 + 2m
log x log y log z
31.
If   , mark all the correct options. 41.
If log2x + log2 y ≥ 6, then the least value of xy is
bc ca ab
(a) xyz = 1 (b) xa yb zc = 1 (a) 4 (b) 8
b+c c+a a+b
(c) x y z = 1 (d) xb+c yc+a za+b = 0 (c) 64 (d) 32
[Based on ITFT, 2006]
If log10 x = y, then log1000 x2 is equal to
42.
32. If 2 log x = 5 log y + 3, then the relation between x and y
(a) y2 (b) 2y
is
(a) x2 = 100y5 (b) x1/5 = 1000 y1/2 3y 2y
(c) (d)
(c) x2 = 1000y5 (d) x2 = y5 + 1000 2 3

a 43.
Solve for x, log10 x + log x + log 3 100 x =
27
33.
What is the value of , if log 4 log 4 4a–b = 2 log4 10
b (a) 1 (b) 106
( a  b )  1? 4
(c) 10 (d) 10
5
(a)  (b) 2 If y = 21/logx 4, then x is equal to
44.
3
5 (a) y (b) y
(c) (d) 1
3 [Based on IIFT, 2010] (c) y2 (d) y4

 1010
10
 log a log b log c
34. The value of log10log10 log10log10 1010  45.
If = = , , then which of the following
  b−c c−a a−b
  options holds true?
1
(a) 1 (b) (a) ab . bc . ca = 1 (b) aabbcc = 1
10
(c) 10 (d) Cannot be determined (c) a2ab2bc2c = 1 (d) aabbbccac = 1
35. The characteristics of the logarithm of 21735 is 46.
If a, b, c are distinct positive numbers different from
(a) 2 (b) 1 1 such that (logba . logca – logaa) + (logab . logcb – logbb)
(c) 3 (d) 4 + (logac . logbc – logcc) = 0 then abc =
[Based on ATMA, 2005] (a) 0 (b) e
36. log10(log2 3) + log10(log3 4) + … + log10(log1023 1024) (c) 1 (d) None of these
equals 2log93
(a) 10 (b) e 47.
The value of 3 is
(c) 1 (d) 0 (a) 4 (b) 3
[Based on JMET, 2006] (c) 47 (d) 8

Chapter_19.indd 427 1/30/2016 5:36:34 PM


428  Chapter 19

If log10 x − log10 x = 2 logx10, then a possible value of


48. 3 3
(c) M3 = (d) N9 =
x is given by N N
(a) 10 (b) 1/100
(c) 1/1000 (d) None of these 58. If log10x – log10 x = 2 logx 10, then a possible value of
x is given by
49. The length of the circumference of a circle equals the
1
perimeter of a triangle of equal sides, and also the (a) 10 (b)
perimeter of a square. The areas covered by the circle, 100
triangle, and square are c, t and s respectively. Then, 1
(c) (d) None of these
(a) s > t > c (b) c > t > s 1000
(c) c > s > t (d) s > c > t
59. Let u = (log2 x)2 – 6 log2 x + 12, where x is a real number.
50. The difference between the logarithms of sum of the Then, the equation xn = 256, has
squares of two positive numbers A and B and the sum (a) no solution for x
of logarithms of the individual numbers is a constant C.
(b) exactly one solution for x
If A = B, then C is
(c) exactly two distinct solutions for x
(a) 2 (b) 1.3031
(d) exactly three distinct solutions for x
(c) log 2 (d) exp (2)
1 1
1 60. If log8x + log8  = , then the value of x is
51. log10 25 − 2log10 3 + log10 18 equals 6 3
2
(a) 18 (b) 24
(a) 18 (b) 1
(c) 16 (d) 12
(c) log103 (d) None of these
61. Which of the following is true?
52. If loga(ab) = x, then logb(ab) is
(a) log17275 = log19375 (b) log17275 < log19375
1 x
(a) (b) (c) log17275 > log19375 (d) None of these
x x +1
62. Find log3/2 3.375
x x
(c) (d) (a) 2 (b) 3
x −1 1− x
(c) 5/2 (d) 17/2
53.
The number of real solutions of the equation
63. If x = log2a a , y = log3a 2a and z = log 4a 3a,
log (–­x) = 2 log (x + 1) is
find yz (2 – x).
(a) One (b) Two
(a) 1 (b) –1
(c) Three (d) Four
(c) 2 (d) ­–2
54. A certain type of bacteria reproduces itself at the rate of
10% every 5 mins. In how many minutes will the number log x log y log z
64. = = , find x2y2z2.
double itself? l + m − 2m m + n − 2l n + l − 2m
(a) 20 mins (b) 30 mins (a) 2 (b) –1
(c) 40 mins (d) None of these (c) 4 (d) 1

( )
55. If log7 log5 ( x + 5 + x ) = 0, find the value of x. x+ y
65. If log
5
1
= (log x + log y), then
2
x
y
+
y
x
=
(a) 1 (b) 0
(a) 20 (b) 23
(c) 2 (d) None of these
(c) 22 (d) 21
56. log2[log7 (x2 – x + 37)] = 1, then what could be the value
of x?  3x − 3 y 
66. If log (x + y) = log    , then log x ­­­­– log y =
(a) 3 (b) 5  2 
(c) 4 (d) None of these (a) log 2 (b) log 3
(c) log 5 (d) log 6
1
57. If log3 M + 3 log3N = 1 + log50.008, then
3 67. If log2 x + log4 x + log16 x = 21/4, then x =
9 9 (a) 8 (b) 4
(a) M9 = (b) N9 = 
N M (c) 2 (d) 16

Chapter_19.indd 428 1/30/2016 5:20:03 PM


Logarithms  429

16 25 81 80. If log10 87.5 = 1.9421, then the number of digits in (875)10


68. 7 log + 5 log + 3 log = is
15 24 80
(a) log 2 (b) log 3 (a) 30 (b) 29
(c) log 5 (d) None of these (c) 20 (d) 19

69. If 0 < a x, the minimum value of loga x + logx a is 81. If log10 2 = 0.3010, log10 3 = 0.4771, then the number of
(a) 1 (b) 2 zeros between the decimal point and the first significant
(c) 3 (d) 5 figure in (0.0432)10 is
(a) 10 (b) 13
log x log y log z
70. If = = , then xyz = xa  × yb  × zc (c) 14 (d) 15
b−c c−a a−b
= xb+c × yc+a × za+b = 82. If (4.2)x = (0.42)y = 100, then
1

1
=
(a) 1 (b) 0 x y
(c) 2 (d) None of these (a) 1 (b) 2
71. xlog y–log z × ylog z–log x × zlog x–log y = (c) 1/2 (d) –1
(a) 0 (b) 2 log9 11 log3 11
(c) 1 (d) None of these 83. – =
log5 13 log 5 13

72. If log10  [98 + x 2 − 12 x + 36] = 2, then x = (a) 1 (b) –1


(a) 4 (b) 8 (c) 0 (d) None of these
(c) 12 (d) 4, 8
log x log y log z
84. If = = , then yz in terms of x is
73. If x = loga bc, y = logb ca, z = logc ab, then 2 3 5
(a) xyz = x + y + z + 2 (b) xyz = x + y + z + 1 (a) x (b) x2
(c) x + y + z = 1 (d) xyz = 1 (c) x3 (d) x4
74. If ax = by = cz = dw, then loga (bcd) = 1 1
x+ x−
1 1 1 1  1 1 1 85. If 4x + 22x – 1 = 3 2 +3 2 , then x =
(a)  + +  (b) x  + + 
x y z w  y z w (a) 1/2 (b) 3/2

y+z+w (c) 5/2 (d) 1


(c) (d) None of these
x
log 49 7 + log 25 5 − log 4 2
75. If log10 2 = 0.3010, then log10 (1/2) = 86. =
log17.5
(a) –0.3010 (b) 0.6990
(a) 5 (b) 2
(c) 1 .6990 (d) 1 .3010
(c) 5/2 (d) 3/2
76. If log2 (32x–2 + 7) = 2 + log2 (3x–1 + 1), then x =
87. log10 tan40° × log10 41° ... log10 tan 50° =
(a) 0 (b) 1
(c) 2 (d) 1 or 2 (a) 1 (b) 0
(c) –1 (d) None of these
77. If loga b = logb c = logc a, then
(a) a > b c (b) a < b < c 88. If log8 p = 2.5, log2 q = 5, then p in terms of q is
(c) a = b = c (b) a < b ≤ c 3
(a) q (b) 2q
1 2 (c) q (d) q/2
78. If = – 2, then x =
log x 10 log a 10
1 1
(a) a/2 (b) a/100 89. If y = ,z= and x = ak, then k =
a1− log a x a1− log a y
(c) a2/10 (d) a2/100
1 1
1 1 (a) (b)
79. If a2 + b2 = c2, then + = a 1− log a z 1 − log a z
log c + a b log c − a b
(a) 1 (b) 2 1 1
(c) (d)
(c) –1 (d) ­–2 1 + log z a 1 − log z a

Chapter_19.indd 429 1/30/2016 5:20:04 PM


430  Chapter 19

90. If loge 2 × logb 625 = log1016 × loge 10, then b = log 2 a log3 b log 4 c


102. If = = and a1/2 × b1/3 × c1/4 = 24,
(a) 4 (b) 5 3 3 4
(c) 1 (d) e then
log 7 log 7 5
(a) a = 24 (b) b = 81
91. 5 5 − 7
(c) c = 64 (d) c = 256
(a) log 2 (b) 1
(c) 0 (d) None of these log 2 x log 2 y log 2 z z
103. If = = and 3 4 = 1, then k =
3 4 5k x y
92. 2log37 – 7log32 (a) 3 (b) 4
(a) log2 7 (b) log 7 (c) 5 (d) –5
(c) log 2 (d) 0
3 + log10 343
93. If log30 3 = a, log30 5 = b, then log30 8 = 104. =
1  49  1  1 
(a) 3(1 – a – b) (b) a – b + 1 2 + log   + log  
2  4  3  125 
(c) 1 – a – b (d) 1(a – b + 1) (a) 3 (b) 3/2
94. If 0 < a < 1, 0 < x < 1 and x < a, then loga x (c) 2 (d) 1
(a) < 1 (b) > 1 log x log y log z
(c) < 0 (d) ≤ 1 105. If 2 2
= 2 2
= , then
a + ab + b b + bc + c c + ca + a 2
2

95. log5 2 is xa–b × yb–c × zc–a =


(a) an integer (b) a rational number (a) 0 (b) –1
(c) an irrational number (d) a prime number (c) 1 (d) 2
 1  1  1 106. If 3x–2 = 5 and log10 2 = 0.20103, log10 3 = 0.4771,
96. log5 1 +  + log5 1 +  + log5 1 +  + ...
 5  6  7 then x =
1  22187 22187
 (a) 1 (b) 2
+ log5 1 +  47710 47710
 624 
(a) 5 (b) 4 22187
(c) 3 (d) None of these
(c) 3 (d) 2 47710
107. If log2 = 0.30103 and log3 = 0.4771, then number of digits
97. If log10 2986 = 3.4751, then log10 0.02986 =
in (648)5 is
(a) 1 .2986 (b) 2 .4751 (a) 12 (b) 13
(c) 0.34751 (d) None of these (c) 14 (d) 15
98. If log (2a – 3b) = log a – log b, then a = log y log z
108. If log x = = , then x4 ×  y3 × z–2 =
3b 2 3b 2 5
(a) (b)
2b − 1 2b − 1 (a) 2 (b) 10
(c) 1 (d) 0
b2 3b 2
(c) (d)
2b + 1 2b + 1 log 27 + log 1000 + log8
109.
1 1 x y log120
99. If log(x – y) – log 5 – log x – log y = 0, then + =
2 2 y x (a) 1/2 (b) 1
(a) 25 (b) 26 (c) 3/2 (d) 2
(c) 27 (d) 28 x
10 log10
110. For x > 0, if y = 2
and x = ya, then a =
100. If log x:3 = log y:4 = log z:5, then zx = x
(a) 2y (b) y2 (a) 1 (b) –1
(c) 8y (d) 4y (c) 0 (d) 2
101. If 3 + log5 x = 2 log25 y, then x = 111. If x = 1004/3(1/2), y = log1/2(1/3), then
(a) y/125 (b) y/25 (a) x > y (b) x < y
(c) y2/625 (d) 3 – y2/25 (c) x = y (d) x ≥ y

Chapter_19.indd 430 1/30/2016 5:20:05 PM


Logarithms  431

EXPLANATORY ANSWERS

logn 1296 = 4 (logn 2 + logn 3)


1.
(b) log10 x − log10 x = 2 logx10
 2b − a 3a − b 
= 4  +
1 2  5 10 
⇒ log10 x − log10 x =
2 log10 x
 a + 3b  2(a + 3b)
= 4   =
log10 x 2  10  5
⇒ =
2 log10 x
n n
(log10x)2 = 4 1
5. (a). ∑ log (a)
= ∑ log a (2n )
⇒ log10x = ±2 n =1 2n n =1

1 n
⇒ x = 10–2, 102 =
100
, 100. = ∑ n log a 2
n =1

 m2   m3  n
n (n + 1)
2.
(d) log m + log 
 n 



+ log  2 
n 
 
= log a 2 ∑n 2
 = loga2
n =1

 m4 
+ log  3  + ... + nth term a b
n  6.
(a) log + log = log (a + b)
  b a

 m ⋅ m 2 ⋅ m 2 ... m n   m(1+ 2 + 3+...+ n )  a b


= log  = log ⇒ log  ×  = log (a + b)
2 3 n −1   (1+ 2 + 3+...+ n −1)  b a
 n ⋅ n ⋅ n ... n   n 
\ a + b = 1
 n ( n +1)  n /2 7.
(a) – x = (x + 1)2
m 2   m( n +1) 
= log  ( n −1) n  = log  ( n −1)  . ⇒ 2
x + 3x + 1 = 0
   n 
n
2
 −3 + 5 −3 − 5
\ x = ,
2 2
3.
(c) If the length of the circumference of a circle equals the
perimeter of a regular polygon then −3 + 5
But only satisfies the other condition of x +
Area of circle > Area of regular polygon 2
1 > 0.
    Also, if two regular polygons have the same
perimeter, then the regular polygon having larger no. log( x 2 + x) log( x + 1)
of sides will have area greater than that of the regular 8.
(c) – = 2
log 5 log 5
polygon having less number of sides.
\ c > s > t. log(x2 + x) – log (x + 1) = 2 log 5
 x2 + x 
4.
(d) Given logn48 = 4 logn2 + logn3 = a and ⇒ log   = log 52
logn 108 = 2.logn 2 + 3 logn 3 = b  x +1 
Let logn 2 = x and logn 3 = y x2 + x
⇒ = 25
⇒ 4x + y = a ...(1) x +1

2x + 3y = b ...(2) ⇒ x2 – 24x – 25 = 0
2 × (2) – (1) gives ⇒ (x – 25) (x + 1) = 0
6y – y = 2b – a ⇒ x = 25
2b − a 9.
(d) log12 27 = a
⇒ y =
5 ⇒ log12 33 = a
3a − b a
Similarly, x = ⇒ log12 3 =
10 3

Chapter_19.indd 431 1/30/2016 5:20:06 PM


432  Chapter 19

3 = log10 5 − log10 9 + log10 18


⇒ log3 12 =
a 5
3 = log10 + log10 18
⇒ log3 4 + log3 3 = 9
a 5 × 18
3−a = log10 = log10 10 = 1
⇒ 2 log3 2 = 9
a
(c) Given log(A2 + B2) – (log A + log B) = C
13.
3−a
⇒ l og3 2 = …(1) If A = B, then
2a 2
⇒ log(2A ) – 2 log A = C
Now, log6 16 = 4 log6 2 = A (let)
⇒ log(2A2) – log A2 = C
A
⇒ log6 2 =  2 A2 
4 ⇒ log  2  = C
 A 
 
4
⇒ log2 6 = ⇒ log 2 = C
A
(a) logx a, ax/2 and logb x are in GP, then
14.
4− A
⇒ log2 3 = (ax/2)2 = (logx a) × (logb x)
A
⇒ ax = logb a
A
⇒ log3 2 = …(2) ⇒ x log a = loga (logb a)
4− A
⇒ x = loga (logb a)
From Eqs. (1) and (2),
15.
(c) loga (ab) = x
3−a A 2a 4− A
= ⇒ = log ab
2a 4 − A 3−a A ⇒ =x
log a
2a 4− A
⇒ +1 = +1 log b
3−a A ⇒ =x–1
log a

2a + 3 − a 4− A+ A
⇒ = log a 1
3−a A ⇒ =
log b x −1

4(3 − a )
\ A = log ab log a + log b
(3 + a ) Now, logb (ab) = =
log b log b
1 1 k log a 1
10.
(c) loga b = , logb c = , logc a = = +1= +1
2 3 5 log b x −1
log b 1 log c 1 log a k 1+ x −1 x
⇒ = , = , = = =
log a 2 log b 3 log c 5 x −1 x −1

1 1 k 1 1
⇒ × × = 1 ⇒ k = 30 16.
(d) log8 x + log8 =
2 3 5 6 3
11.
(b) Let log17 275 = log19 375  1 1
⇒ log8  x ×  =
log 275 log 375  6  3
⇒ =
log 17 log 19
 x 1
⇒ log8   =
275 375 6 3
⇒ =
17 19
x
\ 16.18 < 19.74 ⇒ (8)1/3 =
6
Hence, log17 275 < log19 375
 [Q if loga y = x, than (a)x = y]
1 x
12.
(b) log10 25 − 2 log10 3 + log10 18 ⇒ (23 )1/3 =
2 6
= log10 (25)1/2 − log10 (3) 2 + log10 18 ⇒ x = 12

Chapter_19.indd 432 1/30/2016 5:20:07 PM


433
Logarithms 

(d) [log10 (5 log10 100)]2 = [log10 (5 × 2)]2


24.
1 1 = [log10 10]2
17.
(d) log8 x + log8 =
6 3 = 12 = 1
1 (d)  log5 12 = log5 (3 × 4)
25.
log
log x 6 = 1 = log5 3 + log5 4
⇒ +
log 8 log 8 3 = log5 3 + 2 1og5 2

log x + log 1 − log 6 1 log10 3 2log10 2
⇒ = = +
log 2 3 3 log10 5 log10 5

⇒ log x + log 1 – log 6 = log 2 log10 3 2 log10 2
= 
log x + log 1 = log 6 + log 2 log10 10  log10 2 log10 10  log10 2
log x = log 12 b 2a 2a  b
\ x = 12 =  =
1 a 1 a 1 a
1
18.
(c) log x (0.1) =  26.
(b) a = log8 225, b = log2 15
3
2 2

1 \ a = log 23 152 = s log215 =  b
x 3 3
⇒ = 0.1 ⇒ x = 1000
3

19.
(d) 2
x – 6x + 45 = 100 27.
(c) log x – 5 log 3 = – 2
x = –5, 11 35
⇒ log10 = – 2
x
20.
(b) log10 125 + log10 8 = x
⇒ log10 (125 × 8) = x x
= 10– 2
⇒ 10x = 1000 243
\ x = 3 243
⇒ x = ⇒ x = 2.43
a 2 2 2 100
b c
21.
(b) log  2  2  2  = log (1) = 0
b c a  (125) (625) (53 ) (54 )
28.
(d) log5 = log5
25 52
22.
(c) log7 log5 ( x  5  x ) = 0
= log557 – 2 = log5 55
⇒ 70 = log5 ( x  5  x ) = 5 log 55 = 5
⇒ log5 ( x  5  x ) = 1 (b) log10 (a2 – 15a) = 2
29.

a2 – 15a = 102
⇒ 51 = x  5  x 2
⇒ a – 15a – 100 = 0
⇒ x = 4
(a – 20) (a + 5) = 0
23.
(d) The given equation ⇒ a = 20, – 5
442
log 9  15   35 
1 441 log 442 30.
(a) log   − log   + log  
= = = x  14   16   24 
log 442 x log x 441
441 9 16 35
= log × × = log 1 = 0
442 443 444 14 15 24
log x + log x + log x + ...
441 442 443 31.
(a, b, c) Option (d) is wrong as the expression evaluate to
1 as in (c) and not zero. In all, options (a), (b) and (c)
899 900
+ log x + log x =2 are correct.
898 899
32. (c) 2 log x = 5 log y + 3
 442 443 444 899 900  log x2 = log y5 + 3 log1010
⇒ log x  × × × ... × ×  =2
 441 442 443 898 899  log x2 = log y5 + 3 log10103
900 900 30 10 = log (y5 × 103)
⇒      log x = 2 ⇒ x2 = ⇒x= =
441 441 21 7 ⇒ x2 = 1000y5

Chapter_19.indd 433 1/30/2016 5:36:34 PM


434  Chapter 19

1 1
(c) Given, log4 log4 4a–b = 2 log4 ( a  b )  1
33. \ loge2 = 1 – + – ... to ∞
2 3
log4 (a – b) log4 4 = log 4 ( a  b ) 2  log 4 4 1 1 1
⇒ − + .. to ∞ = 1 – loge2 = 1 – ln 2
2 2 3 4
log 4 (a  b) = log 4 4( a  b )

1 1
(a – b) = 4( a  b ) 2 40.
(b) log4928 = log 7 (7 × 4) = (1 + log 7 4)
2 2
⇒      a  b  4 a  4 b 1 1 1
= + .2log 7 2 = + log 7 2
3 a = 5 b 2 2 2

1 1 + 2m
a 5 = +m =
⇒ = 2 2
b 3
41.
(c) Given: log2x + log2y ≥ 6
34.
(c) logaa = 1
10
⇒ log2(xy) ≥ 6
1010
= log10 1010 ⇒ xy ≥ 64

1010 1010 2 2
= 1010 log1010 = 1010 42.
(d) log1000(x2) = log10 x = y
3 3
i.e., with each log10 one 1010 is removed there are 43.
(b) Changing the base to 10, we get
5 – 10s (including the ones in powers) and 4 – log10
therefore, last will be log101010 = 10log1010 = 10 log10 x
log 10
x = = 2 log10 x
log10 10
35.
(d) Characteristics = number of digits – 1 = 5 – 1 = 4
36.
(c) log10 (log2 3) + log10 (log3 4) + … + log10 log10 x 3log10 x
log 3 100 x = =
log1023 1024) log10 3 100 2
= log10 [(log2 3)(log3 4)(log4 5) … (log1023 1024)] 3
\ log10 x + 2log10 x + log10 x =
27
2
 log 3 log 4 log 5 log1024 
= log10  × × × ... ×
 log 2 log 3 log 4 log1023  9
⇒     log10 x = 27
2
 log a 
∵ logb a = log b  ⇒ log10x = 6
  \ x = 106
= log10 (log2 1024) = log10 log2 210 = log10 10 = 1 1 1
log2 x
2 log x 4
37.
(b) (log3 x) (logx 2x) (log2x y) = (logx x ) 44.
(c) y = 2 = 2log 4 x = 2 2

Converting to common base 10 x
= 2log 2 = x
log x log 2 x log y log x 2
+ + = \ x = y2
log 3 log x log 2 x log x

log y log a log b log c


⇒ = 2 45.
(b) = = =r
log 3 b−c c−a a−b
⇒ log y = 2 log 3 = x log 32 \ a log a + b log b + c log c
⇒ y = 9 = r[a(b – c) + b(c – a) + c(a – b)]
38.
(c) logb8 = 3 ⇒ 3logb2 = 3 ⇒ logb2 = 1 ⇒ log aa + log bb + log cc = 0
logab = log2b × loga2 = log2b × log32 × loga3 ⇒ log(aa bb cc) = log 1
= 1 × log32 × 2 = 2log32 = log34 ⇒ aa bb cc = 1

1 1 1 46.
(c) (logba logca – 1) + (logab . logcb – 1)
39.
(d) Required sum = − + – ... to ∞
2 3 4 + (logac logbc – 1) = 0
x x2 3 log a log a log b log b log c log c
Now, loge(1 + x) = x – + – ... ⇒ . + . + . =3
2 3 log b log c log a log c log a log b

Chapter_19.indd 434 1/30/2016 5:20:10 PM


435
Logarithms 

⇒ (loga)3 + (logb)3 + (logc)3 = 3 log a log b log c log a 1


⇒ =
⇒ (loga + logb + logc) = 0 log b x −1
[Q If a3 + b3 + c3 – 3abc = 0 then a + b + c = 0 if a ≠ b ≠ c]
log ab log a
⇒ logabc = log 1 ⇒ abc = 1 \ logbab = = 1+
log b log b
1
2. log3 3 1 x
47. 32log9 3 3=
(b) = 2 3 = 1 + =
x −1 x –1
48.
(b) log10 x − log10 x = 2 logx10
53.
(a) – x = (x + 1)2
1 2 ⇒ x2 + 3x + 1 = 0
⇒ log10 x − log10 x =
2 log10 x
−3 + 5 −3 − 5
\ x = ,
log10 x 2 2 2
⇒ =
2 log10 x
−3 + 5
But only = satisfies the other condition of
⇒ (log10x)2 = 4 ⇒ log10x = ±2 2
1 x+1>0
⇒ x = 10–2, 102 = , 100
100
54.
(d) Let the number of bacteria in the beginning be P,
49. (c) If the length of the circumference of a circle equals the which doubles after time T.
perimeter of a regular polygon, then, T
 10 
Area of circle > Area of regular polygon \ P 1 +  = 2P
 100 
  Also, if two regular polygons have the same [Here 1 unit of time = 5 mins]
perimeter, then the regular polygon having larger
number of sides will have area greater than that of the ⇒ (1.1)T = 2
regular polygon having less number of sides. ⇒ T log (1.1) = log 2
\ c > s > t log 2
⇒ T =
50. 2 2
(c) log (A + B ) – [log (A) + log (B)] = C log1.1
\ A = B ⇒ log (2A2) –­2 log A = C 0.3010 3010
⇒ = T =
⇒ log 2A2 – log A2 = C 0.0414 414
2 A2 = 7.27 units
⇒ log = C ⇒ C = log 2
A2 = 36.35 mins

1 55.
(b) log7 log5 ( x + 5 + x ) = 0
51.
(b)
log 25 – 2 log 3 + log 18
2
1 ⇒ log5 ( x + 5 + x ) = 70 = 1
= log 52 – 2 log 3 + log (2 × 3 × 3)
2
= log 5 – 2 log 3 + log 2 + log 32 x + 5 + x = 51 = 5
= log 5 + log 2 ⇒ 2 x = 0 \ x = 0
= log (5 × 2) = log 10 = 1
56.
(c) log2 [log7(x2 – x + 37)] = 1
52.
(c) logaab = x
⇒ 2 = log7 (x2 – x + 37)
log ab
⇒ = x ⇒ 49 = x2 – x + 37
log a
or x2 – x – 12 = 0
log a + log b
⇒ = x ⇒ (x – 4)(x + 3) = 0. \ x = 4
log a
log b 1
⇒ + 1 = x 57.
(b) log3M + 3 log3N = 1 + log0.008 5
log a 3

log b log e 5
⇒ = x – 1 ⇒ log3M1/3 + log3N3 = 1 +
log a log e 0.008

Chapter_19.indd 435 1/30/2016 5:36:35 PM


436  Chapter 19

log e 5 1 1
⇒ log3 (M. N9)1/3 = 1 + 60.
(d) log8x + log8 =
8 6 3
log e
1000  1 1  x 1
⇒ log8  x ×  = or log8   =
log e 10 − log e 2  6  3 6
  3
= 1 +
log e 8 − log e 1000 x x
⇒ (8)1/3 = ⇒  (23)1/3 =
log e 10 − log e 2 6 6
= 1 +
3 log e 2 − 3 log e 10 \ x = 12
log e 10 − log e 2 61.
(b) log17275 = log19375
= 1 +
− 3 (log e 10 − log e 2) log 275 log 375
⇒ =
1 2 log17 log19
\ log3 (MN9)1/3 = 1 − =
3 3 275 375
⇒ =
1 2 1 17 19
⇒ ( MN 9 ) 3 = 33 = (9) 3 Q 16.18 < 19.74
⇒ MN9 = 9 \ log17275 < log19375
9 9
\ N = 62.
(b) log3/2 3.375 = x
M
x
3
58.
(b) log10x – log10 x = 2 logx 10 ⇒   = 3.375
2
1
⇒ log10x – log10x = 2 logx10 ⇒ (1.5)x = (1.5)3 ⇒ x = 3
2
63.
(a) yz(2 – x) = 2yz – xyz = 2 log4a 2a – log4a a
1
⇒ log10 x = 2. logx10  4a 2 
2 = log4a  =1
 a 
⇒ log10 x = 4. logx10 64.
(d) Each is equal to k  
⇒ log10x = logx104 x = k (l + m – 2n),
⇒ log 
or log10x = logx10000 log y = k (m + n – 2l),
Now putting the value of x = 10 log z = k (n + l – 2m).
1 = 4, which is not possible ⇒ log xyz = k (0)
1 ⇒ xyz = e0 = 1 ⇒ x2y2z2 = 1
Putting the value of x =  , we get – 2 = – 2.
100
Thus answer is (b). x+ y 1
65.
(b) log   = [log x + log  y]
59.
(b) 2
u = (log2 x) – 6(log2 x) + 12  5  2
Let log2 x = p ...(1) ⇒ x + y = 5 xy ⇒ x2 + y2 = 23xy
⇒ u = p2 – 6p + 12 x y
⇒ + = 23
xu = 256 = (28) y x
   Applying log to base 2 on both sides, we get u log2
x = log2 28. 3x − 3 y
66.
(c) x + y =
⇒ u log2x = 8 ...(2) 2

Dividing (2) by (1), we get x


⇒ x = 5 y ⇒ =5
u = 8/p y

⇒ 8/p = p2 – 6p + 12 ⇒ log  x – log y = log 5


⇒ 8 – p3 – 6p2 + 12p 67.
(a) log2 x +
1 1
log 2 x + log 2 xs =
21
or 3 2
p – 6p + 12p – 8 = 0 2 4 4
⇒ (p – 2)3 = 0 or p = 2  1 1 21
⇒ log 2x 1 + +  =
\ log2 x = 2 ⇒ 2 ⇒ x = 22 = 4  2 4 4
Thus, we have exactly one solution. ⇒ log2x = 3 ⇒ x = 8

Chapter_19.indd 436 1/30/2016 5:20:12 PM


437
Logarithms 

 24   52   34  1
68.
(a) 7 log   + 5log  3  + 3log  4 (c) log10   = –log10 2 = – 0.3010
75.
 5×3  2 ×3  2 × 5  2
     
= 28 log 2 – 7 log 5 – 7 log 3 + 10 log 5 – 15 log 2 – = 1 – 0.3010 – 1 = 1 .6990
5 log 3 + 12 log 3 – 12 log 2 – 3 log 5 = log 2
(d) log2 (32x–2 + 7) = log 24 + log2 (3x–1 + 1)
76.
69.
(b) 0 < a ≤ x; Min. value of loga x + logx a is 2 when we
[Q 2 = 2 log22 = log222]
put x = a
⇒ 32x–2 + 7 = 4 (3x–1 + 1)
log x log y log z
70.
(a) = = = k (say) ⇒ t2 + 7 = 4(t + 1), where, 3x–1 = t
b−c c−a a−b 2
⇒ t – 4t + 3 = 0 ⇒ t = 1, 3
⇒ log x = k (b – c), log y = k (c – a),
When t = 1 ⇒ 3x–1 = 1 ⇒ x = 1
log z = k (a – b)
When t = 3 ⇒ 3x–1 = 31 ⇒ x = 2
⇒ log x + log y + log z = 0 ⇒ xy z = 1.
Also, a log x + b log y + c log z = 0 77.
(c) loga b = logb c = logc a = k (say)
⇒ a  b  c
x . y . z = 1. ⇒ b = ak, c = bk, a = ck
Again (b + c) log x + (c + a) log y + (a + b) log z = 0. ⇒ c = (ak)k = ak2 = (ck2)k = ck3
⇒ xb+c × yc+a × za+b = 1. ⇒ k3 = 1 ⇒ k = 1.  \  a = b = c
\ xyz = xa × yb × zc = xb+c × yc+a × za+b = 1 78.
(d) log10 x = 2 log10 a – 2

(c) xlog y–log z × ylog z – log x × zlog x–log y = k (say)


71. ⇒ log10 x = 2 (log10 a – 1)
⇒ (log y – log z) log x + (log z – log x) log y a a2
⇒ log10 x = 2 log10   ⇒ x =
+ (log x – log y) log z = log k = 0  10  100
⇒ k = 1 79.
(b) logb(c + a) + logb(c – a)
= logb(c2 – a2) = logbb2 = 2
72.
(d) 98 + x 2 − 12 x + 36 = 100
80.
(a) x = (875)10 = (87.5 × 10)10
⇒ x 2 − 12 x + 36 = 2 \ log10 x = 10(log1087.5 + 1)
⇒ x2 – 12x + 32 = 0 = 10(1.9421 + 1)
⇒ x = 8, 4 = 10(2.9421) = 29.421.
73.
(a) x = loga bc \ x = Antilog (29.421).
⇒ ax = bc
\ Number of digits in x = 30
⇒ ax+1 = abc
10
⇒ a = (abc)1/x+1.  432 
81.
(b) x = (0.0432)10 =  
Similarly, b = (abc)1/y+1 and c = (abc)1/z+1  10000 
1 1 1 10
+ +  33.24 
\ abc = (abc) x +1 y +1 z +1 =  4 
 10 
 
1 1 1
⇒ 1 = + + \ log10x = 10 (3log103 + 4log102 – 4)
x +1 y +1 z +1
= 10 (1.4313 + 1.2040 – 4)
⇒ (x + 1) (y + 1) (z + 1) = (y + 1) (z + 1)
= 10 (–1.3647) = –13.647
+ (x + 1) (z + 1) + (x + 1) (y + 1)
⇒ xyz = x + y + z + 2 = 14 .353
x x x \ x = Antilog (.053)
y x y
74.
(b) b = a ⇒ b = a , c = az ,d= aw \ Number of zeros between the decimal and the first
significant figure = 13.
 x x x
  x x x 82.
(c) (4.2)x = 100
loga (bcd) = log a  a y .a z .a w  = + +
y z w ⇒ (42)x = 102+x
1 1 1 2
= x  + +  +1
 y z w ⇒ 42 = 10 x ...(1)

Chapter_19.indd 437 1/30/2016 5:36:36 PM


438  Chapter 19

 42 
y 1
⇒ x = = ak (given)
  = 100 1− log az
 100  a
⇒ (42)y = 102+2y 1
2
\ k =
+2 1 − log a z
y
⇒ 42 = 10 ...(2)
(b) loge 2 × 4 logb 5 = 4 × log10 2 × loge 10 = 4 loge 2
90.
2 2 1 1 1 ⇒ logb 5 = 1 ⇒ b = 5
From (1) and (2), − = 1 ⇒ − =
x y x y 2
1
log9 11 log3 11 log3 11 log3 11 91.
(c) 5
log5 7
– (7 log 7 5
)
log 7 5
83.
(c) – = – =0
log5 13 log 5 13 2.log5 13 2.log5 13
log5 7 1
= 5 −
log x log y log z 5
log 7 5
84.
(d) = = = k (say) log 57 log57
2 3 5 ⇒ 5 −5 =0
⇒ log x = 2k, log y = 3k, log z = 5k
(d) 2log37 – 7log32 = 2log27 × log32 – 7log32
92.
⇒ log yz = 3k + 5k = 8k; log x4 = 8k
= 7log32 – 7log32 = 0
\ log yz = log x4 ⇒ yz = x4
 30 
4x 3x 93.
(a) a + b = log3015 = log30   = 1 – log30 2
85.
(b) 4 + x
= + 3x. 3  2 
2 3
⇒ log30 2 = 1 – a – b.
x
3 4 4 8 \ log30 8 = 3(1 – a – b)
⇒ 4x ×  = 3x ×  ⇒   =
2 3 3 3 3 94.
(b) 0 < a < 1, 0 < x < 1 and x < a
x 3/2 ⇒ loga x > loga a ⇒ loga x > 1
4 4 3
⇒   =   ⇒x=
3
  3 2 p
95.
(c) log52 = ⇒ 2 = 5p/q = 2q = 5p
q
log 75/2 + log 55/2 − log 25/2 ⇒ even number = odd number,
86.
(c)
log 17.5 which is a contradiction.
5(log 7 + log 5 − log 2) 5 \ log52 is an irrational number.
= =
 35  2 6 7 8 625
2log  
 2  96.
(b) log5 + log5 + log5 + ... + log5
5 6 7 624
87.
(b) log10 tan40° × log10 tan41° ... log10 tan50°  6 7 8 625   625 
= log5  ...  = log5   =4
= 0, since log10 tan45° = 0  5 6 7 624   5 
15
5  2986 
88.
(a) log8 p = ⇒ p = (8)5/2 = 2 2 = (25)3/2 97.
(b) log10(0.02986) = log10 
2 
 100000 
log2 q = 5 ⇒ q = 25. \ p = q3/2 = 3.4751 – 5 = –1.5249
1 1 =  2.4751
89.
(b) loga  y = , loga z =
1 − log a x 1 − log a y
a
98.
(a) 2a – 3b =
1 1 − log a x b
\ loga z = =
 1  − log a x ⇒ 2ab – 3b2 = a
1−  
1 − log x
⇒ 3b2 = a(2b – 1)
 a 

1 3b 2
⇒ – loga z = –1 + ⇒ a =
log a x 2b − 1
1 99.
(c) (x – y)2 = 25xy
⇒ = 1 – loga z
log a x ⇒ x2 + y2 = 27xy
1 x y
\ loga x = ⇒ + = 27
1 − log a z y x

Chapter_19.indd 438 1/30/2016 5:20:15 PM


Logarithms  439

log x log y log z 105.


(c) Each ratio = k ⇒ logx = k(a2 + ab + b2)
100.
(b) = = =k
3 4 5 ⇒ (a – b)log x = k(a3 – b3)
⇒ log x = 3k; log y = 4k; log z = 5k. ⇒ log xa–b = k(a3 – b3) ⇒ xa–b = ek(a3–b3)
⇒ log (zx) = log z + log x = 8k = 2 log y Similarly, yb–c = ek(b3–c3), zc–a = ek(c3–a3).
a–b  b–c 
\ zx = y2 \ x . y . zc–a = e0 = 1

101.
(a) 3 + log5 x = log5 y 106.
(c) 3x–2 = 5
 90 
⇒ log5(125x) = log5 y ⇒ 3x = 45 =  
 2 
⇒ x log103 = log1090 – log102
y
⇒ x = = 2 log103 + 1 – log102
125
⇒ x(0.4771) = 1.65317
log 2 a log3 b log 4 c
102.
(d) = = =k 165317 22187
3 3 4
⇒ x= = 3
47710 47710
⇒ a = 22k, b = 33k, c = 44k and
a  × b1/3 × c1/4 = 2k × 3k × 4k = 24
1/2 (d) log (648)5 = 5 log (81 × 8) = 20 log3 + 15 log2
107.
⇒ 24k = 241 = 20(0.4771) + 15(0.30103)
= 14.05745.
⇒ k = 1.
\ Number of digits in (648)5 is 15
\ a = 4, b = 27, c = 256

z log x log y log z


103.
(c) = 1 108.
(c) = = =k
x y3 4 1 2 5
⇒ log x = k, log y = 2k, log z = 5k.
⇒ log2 z – 3 log2 x – 4 log2 y = 0
\ log(x4 . y3 . z–2) = 4 log x + 3 log y – 2 log z = 0
3.3 4
⇒ log2 z –
⋅ log 2 z − 4 ⋅ ⋅ log 2 z = 0 ⇒ x4 . y3 . z–2 = 1
5k 5k
9 16 log 27 + log 1000 + log8

⇒ 1– − = 0 109.
(c)
5k 5k log120
⇒ 5k – 25 = 0 3
(log 3 + log10 + log 4)
⇒ k = 5 2 3
= =
log 3 + log10 + log 4 2
3(1 + log10 7) 3(1 + log10 7)
104.
(a) =
7 1 7 10log10 x 1 1
2 + log + log 2 + log   110.
(b) y= = = a = y–a ⇒ a = –1
2 5  10  x2 x y

3(1 + log10 7) 111.


(b) x = log4/3 (1/2) = –log4/3 2 < 0
= =3
1 + log10 7 and y = log1/2 (1/3) = log2 3 > 0
⇒ y > x

Chapter_19.indd 439 1/30/2016 5:20:15 PM

You might also like